You are on page 1of 738

Legal Edge 01 Mock

Answers & Details

ENGLISH LANGUAGE

1. Ans: (c) This question asks for an abstract view of what the passage as a whole is primarily doing. The
passage introduces a particular methodology that scholars of women’s history have been encouraged to
employ, explaining why the use of the methodology is supported. The passage then goes on to raise
some concerns about the use of the methodology and cites one example in which caution is needed.

(a) The passage is primarily concerned with only one methodology.

(b) The passage mentions why the methodology had been encouraged but does not give the history of
its origins; while it cautions historians to employ the methodology carefully, it is not concerned with
drawbacks of its proper use.

(c) Correct. The passage discusses why the use of a methodology is being encouraged and then offers
some concerns about its use.

(d) The passage does not discuss any criticism of recent scholarship in women’s history.

Hence, the answer is (c).

2. Ans: (b) This question asks for an identification of specific information provided by the passage.
In the second paragraph, the passage describes certain concerns about using oral narratives. One
of these concerns is that the stories people tell to explain themselves are shaped by ... storytelling
conventions and other influences tied to the teller‘s cultural and historical context.
(a) The passage uses standard histories to refer to the usual work of scholars and not to something
that influences oral narratives.
(b) Correct. The passage raises as a concern that oral narratives may be influenced by storytelling
conventions present in the culture of the speaker.
(c)The passage does not mention the family of origin of women storytellers.
(d)The passage does not mention the expectations of the listeners of oral narratives.
Hence, the answer is(b).

3. Ans: (d) Answering this question involves recognizing what the author believes about oral
narratives and then applying this belief to a hypothetical situation in which the author makes
recommendations to scholars of women‘s history. While acknowledging the appeal of oral
narratives to these scholars, in the second paragraph the author urges caution when using these
narratives as sources of disinterested commentary. The passage then states that people‘s oral
narratives are shaped by cultural and historical factors, which presumably relate to the cultural and
historical context within which the narratives are spoken.
(a)The passage does not mention the personal life experiences of scholars.
(b)The passage does not mention the intended audiences of oral narratives.
(c)The passage mentions standard histories only as a reference to scholarly works that often have
shortcomings.
(d) Correct. The passage cautions that oral narratives may be biased due to cultural and historical
factors, and it is therefore reasonable to suppose that the author would recommend that scholars
consider this when using such information.
Hence, the answer is(d).

4. Ans: (b) This question requires recognizing how a particular part of the passage is related to the
overall reasoning in the passage. The first paragraph introduces a methodology and describes the
methodology‘s appeal. The second paragraph then raises concerns about the use of the
methodology, drawing attention to the cultural and historical bias that may be present in oral
narratives. In the passage refers specifically to the influence political rhetoric may have on a
woman‘s understanding of her experience. In the final sentence, the passage provides a specific
hypothetical example of a woman at the time of the Second World War to illustrate this concern.
(a) The last sentence employs a hypothetical example and does not describe a particular event as
being important to historians.
(b) Correct. After contending that political rhetoric may influence oral narratives, the passage uses
the example of the Second World War in the final sentence to support this claim.
(c)The last sentence does not provide a particular example of an oral narrative.
(d) The passage does not claim that some women are more aware than others of the social forces
that may bear on them.
Hence, the answer is (b).

5. Ans: (d) Answering this question requires recognizing which option is directly inferable from
information in the passage. After describing in the first paragraph why oral narratives are appealing
to historians, the passage begins the second paragraph by imploring scholars of women‘s history
to be as cautious about accepting oral narratives ... as ... written memories. The passage then goes
on to describe potential bias in oral narratives, suggesting that scholars should be as critical of
them as they are of written sources.
(a) The passage does not claim that traditional historical sources should be avoided by scholars.
(b) The passage mentions the influence of political rhetoric merely as one example of potential bias.
(c) The passage suggests that scholars should attempt to be aware of cultural and historical
factors.
(d)Correct. The passage implies that written histories and oral narratives should receive the same
level of critical scrutiny by scholars. Hence, the answer is(d).

6. Ans: (c)This question asks for information that is stated in the passage, and it requires a process
of elimination. In the passage, oral narratives are presented as being unlike most standard
histories, and the passage then goes on in the passage to list characteristics of oral histories that
most standard histories do not have. The answer to this question will therefore contain a
characteristic of women‘s oral histories that is not described in the passage.
(a)The passage states that, unlike most standard histories, women‘s oral histories affirm the
importance of women’s contributions.
(b)The passage states that, unlike most standard histories, women‘s oral histories represent
experience from the perspective of women.
(c) Correct. The passage does not mention the influence of well-known women on women‘s oral
histories.
(d) The passage states that, unlike most standard histories, women‘s oral histories furnish present-
day women with historical continuity.
Hence, the answer is (c).

7. Ans: (a) The gods were immortals so come what may they could never die. Hence (A) has to be
the correct answer. Remember that in this question the correct answer will be the one that states
the opposite of what has been stated in the passage. The options which are out of scope or which
have not been discussed in the passage at all will not be correct because we have no way of
knowing whether they are true or false.
(B) This could be a possibility because the Gods definitely had this power.
(c) This is stated in the passage.
(D) This could also be a possibility.
The correct answer is A.

8. Ans: (a) The opening sentence of the last paragraph states that most gods dwelled on Mount
Olympus. From this, we cannot conclude that all gods dwelled there. Hence, (A) is the correct
answer.
(B) Stated in paragraph 3.
(C) Stated in the last sentence of the passage.
(D) Stated in paragraph 3.
The correct answer is A.

9. Ans: (b)The passage talks of the intermingling of Gods with humans and also discusses the
hardships they faced despite their Godliness. There is an attempt to understand the lives they led
among the humans and also the price that they had to pay for it. Hence, option (B) is the best
answer.
(A) The scope is too narrow. The passage is not just explaining the lives as mentioned.
(C) Same as A. The passage does more than just this.
(D) Nowhere in the passage is the author using any adulatory words to this effect.
The correct answer is B.

10. Ans: (b) The author has set the tone of the passage with the use of these words. The reader
knows that whatever similarities might now be mentioned, the Gods were always going to be
different from the mortals. Hence,
(B) is the correct answer.
(A) This is the function of the entire first paragraph and not just of the quoted phrase.
(C) No such relationship is highlighted in this paragraph.
(D) Cannot be inferred due to lack of any reference to criticism.
The correct answer is B.

11. Ans: (d) Although each of the potential influences listed are arguably correct, it is clear that the
author sees a specific set of circumstances as being responsible. Although religion is discussed
throughout the passage, it is clearly stated that the theological aspects are considered part of a
larger cultural context, so Statement I can be rejected. Similarly, while political matters are also
discussed in the passage, the author uses them primarily as support for a larger examination of
cultural matters, so Statement II can be rejected. While the passage strongly implies that economic
factors are a major contributor (pointing out that greater availability of material is based on the
perception of profits), the political reference disqualifies Statement III from consideration. Only
Statement IV combines the cultural concerns with the economic underpinning the author discusses;
thus, it is the correct answer.

12. Ans: (c) In the passage, the author argues for the inclusion of global turmoil as a contributing
factor to a larger cultural shift, but does not include an argument for this factor on its own, so option
(a) can be discounted. The first factor cited in the passage is the growth in LDS membership, but
due to the phrasing of the passage and its position in the passage, it is clear that the author views
this as the least important factor; thus the option (b) is clearly not valid. While the larger cultural
shift of American audience is addressed in the bulk of the text, the last paragraph makes it clear
that this factor is superseded by an even larger one, so option (d) can also be rejected. The Only
answer option (c) which is specifically cited in the passage, meets the questions criterion.

13. Ans: (b)Statement I can be discounted, because the fact that \/eggje Tales retells Biblical
stories is why it fits within the cultural shift described in the passage, returning to a b traditional
cultural theme.
However, it also highlights the primary difference: Veggie , Tales is a TV show, for a different
audience and produced
I it under different rules and standards, which reduces its relevance to the main thrust of the
argument, which regards cinema. Thus, option (b), which highlights this, is correct.
14. Ans: (b) Statement II is mentioned of the passage. The use of certain basic touchstone values
(last sentence in the passage) in recent films discounts statement I, and makes it irrelevant

15. Ans: (d) The point of the passage (the Japanese sword is not just a fine weapon but something
even more special) is clearly established in the first two sentences; the purpose of the passage is to
explain and support that point. The passage as a whole describes the immense value of the
Japanese sword to both the samurai (the sword's owner) and the Smith (its maker).
A.The sentence The sword is the soul of the samurai' is referenced in the passage to indicate its
importance. The passage also talks about the details of the tremendous effort that is put into each
sword, reflecting the importance of each one. So, correct answer is Statement IV.

16. Ans: (d) For an 'except' question , you should use the process of elimination to identify and
cross out those details mentioned in the passage.
A.The passage does not state that most major historians consider master smiths themselves to be
artists. Also, the Passage states that "Master Smiths may not have been considered artists in the
classical sense', so the correct answer is option (d).

17. Ans: (c) When looking for statements with which the author could agree, be sure to avoid
extreme words and positions that go beyond the author's statements in the passage. The passage
says that every Master Smith had a structural signature due to his own secret forging process.
Therefore,
A.if a physical signature is present on a blade, that blade's structural signature could then be
associated with a Master Smith, whose master status implies the creation of numerous swords. So,
the correct answer is Statement III.

18. Ans: (b) In the passage, the author states that every Master Smith had a 'structural signature'
due to his own secret forging process. The word signature implies the uniqueness of the Smith's
process. Be careful not to infer any additional information, particularly when the question refers to a
specific sentence or phrase. So, the correct answer is Statement II.

19. Ans: (c) The second paragraph extends the idea introduced in the first paragraph that the
Japanese sword is special and a unique work of art. In several places, the second paragraph
underscores the uniqueness of individual Japanese swords. The first sentence mentions that 'no
sword was created in exactly the same way'. Later in the second paragraph, it is mentioned that
every Master Smith had a 'structural signature1; finally the last sentence indicates that 'this effort
made each blade as unique as the samurai who wielded it'. So, the correct answer is option (c).

20. Ans: (c) Refer to the first two lines of the second paragraph'Master Smiths may not have been
considered artists in the classical sense, but everyone of them took great care in how he created a
sword and no sword was created in exactly the same way. The fact that no sword was created in
exactly the same way implies that each sword was unique or the only one of its kind. Moreover, the
uniqueness of the blade of each sword is also referred to in the last lines of the passage.
21. Ans: (d) The passage‘s primary mode of composition is definition as it sets out to define poetry,
the poet, and imagination. The rhetorical questions in the first paragraph help make this clear.

22. Ans: (c) The rhetorical strategy that is used twice in the first paragraph is the rhetorical
question. These questions help achieve the purpose of exploring the definitions of poetry, poets,
and imagination by introducing the questions to be answered with the extended definitions to follow
in the next paragraphs

23. Ans: (a) ―Subordination‖ is ―the treatment of something as less valuable or important.‖ In this
context, the poet brings the soul of man into activity, and each faculty is ranked below another
according to their worth.

24. Ans: (d) The poet is described as first bringing the soul of man into activity (bringing to life),
then subordinating (separating and ordering) the different faculties (abilities), and then joining them
(blending or fusing). Paradoxically, it is the poet‘s job to both separate and join

25. Ans: (c) The writer says that he would ―exclusively appropriate the name of Imagination‖(i.e., he
would call Imagination) ―that synthetic and magical power.‖

26. Ans: (d) ―Novelty‖ and ―freshness‖ are synonymous in the pairings and are set up in opposition
with ―old and familiar objects.‖ Both ―novelty‖ and ―freshness‖ are about being new, which is in
opposition with what is old.

27. Ans: (b)From the list, only nature is seen as superior to its counterpart, art. The manner is seen
as less than the matter. This can be seen from the construction that begins this list of three pairs. It
states, ―still subordinates art to nature‖ (i.e., art is placed under nature in importance or value).

28. Ans: (d) There is no apostrophe in the poem. There is personification of the soul, imagery of fire
and wings, simile comparing the soul‘s power of conversion to fire and food, and an ABAB rhyme
scheme throughout the three stanzas.

29. Ans: (b) The writer of the passage includes the poem, which is about the soul, to characterize
the Imagination. Before providing the poem, the writer says, ―and his words may with slight
alteration be applied, and even more appropriately, to the poetic Imagination.‖ In other words, the
poem can be used to describe the Imagination

GENERAL KNOWLEDGE/CURRENT AFFAIRS

30. Ans – (c) Prahlad Singh Patel


Prahlad Singh Patel is an Indian politician who is the current Minister of Culture and Minister of
Tourism of India, and Member of Parliament from DamohLoksabha Constituency in Madhya
Pradesh.
31. Ans – (b) 21.67%
According to India State of Forest Report 2019, prepared by the Forest Survey of India (FSI), India
now has a total of 21.67 per cent of its area under forest and tree cover which is still far from its
target of 33 per cent.

32. Ans – (c) 168


India ranked 168th out of 180 countries in 2020 EPI, according to researchers at Yale and
Columbia universities, who say country faces serious environmental health risks, including poor air
quality.

33. Ans – (b) Grass


Bamboo, despite being a member of grass family, was classified as a ‗tree‘ under the Indian Forest
Act, 1927. After amending Section 2(7) of Indian Forest Act, 1927, bamboo is no longer a tree and
felled bamboo too is not timber. So any bamboo grown in private or homestead land by millions of
farmers does not require a felling permission or transit permission from any state forest department.

34. Ans – (c) Madhya Pradesh


In terms of area, Madhya Pradesh has the largest forest cover in the country followed by Arunachal
Pradesh, Chhattisgarh, Odisha and Maharashtra. In terms of maximum percentage of its area
covered by forests, the top state would be Mizoram (85.41%).

35. Ans – (d) Jim Bridenstine


James Frederick (Jim) Bridenstine is an American politician and the Administrator of the National
Aeronautics and Space Administration. Bridenstine was the United States Representative for
Oklahoma's 1st congressional district, based in Tulsa from January 3, 2013 to April 23, 2018. He is
a member of the Republican Party.

36. Ans – (c) 16 Psyche


NASA is about to begin building its latest spacecraft. Called ―Psyche‖ it will explore a 140 miles/226
kilometers-wide asteroid called ―16 Psyche.‖ Located in the Solar System‘s main asteroid belt
between Mars and Jupiter, metal-rich 16 Psyche is thought to be the exposed metallic iron, nickel
and gold core of a protoplanet.

37. Ans – (b) Space X


Hurely and Behnkenare the crew of the NASA‘s Demo-2 mission. They became a part of space
history as the first American astronauts to be sent into space by a private company- Elon Musk‘s
SpaceX. It will also be the first time in 11 years that American astronauts would have been
launched from US soil since NASA ended its space shuttle programme in 2011.
38. Ans – (b) Washington DC
Mary W. Jackson NASA Headquarters, better known as NASA Headquarters and formerly named
Two Independence Square, is a low-rise building in the two-building Independence Square complex
at 300 E Street SW in Washington D.C. The building houses NASA leadership who provide overall
guidance and direction to the US government executive branch agency NASA, under the leadership
of the NASA administrator.
39. Ans – (d) Europa Clipper
Europa Clipper is an interplanetary mission in development by NASA comprising an orbiter. Set for
a launch in 2024, the spacecraft is being developed to study the Galilean moon Europa through a
series of flybys while in orbit around Jupiter.
40. Ans- (a) Scheme for Agro-Marine Processing and Development of Agro-Processing Clusters
Finance Minister, Nirmala Sitharaman on 5th July 2019, announced to launch
'MatsyaSampadaYojana‘ aimed to increase the fish and aquatic products through appropriate
policy, marketing and infrastructure support. The CCE has approved renaming of SAMPADA which
was Scheme for Agro-Marine Processing and Development of Agro-Processing Clusters earlier, as
Pradhan MantriKisanSampadaYojana (PMKSY). It was approved by the CCEA in May 2017 for the
period of 2016-20 coterminous with the 14th Finance Commission cycle.
41. Ans- (d) Blue
Government has made it clear that 'Blue Revolution' or `NeeliKranti‘ has the potential to attain the
first place in the world in fish production. It includes MoFPI‘s schemes such as Food Parks, Food
Safety and Infrastructure

42. Ans- (d) Harsimrat Kaur Badal


The Ministry of Food Processing Industries (MOFPI) is a ministry of the Government of India
responsible for formulation and administration of the rules and regulations and laws relating to food
processing in India. The ministry was set up in the year 1947, with a view to develop a strong and
vibrant food processing industry, to create increased employment in rural sector and enable
farmers to reap the benefits of modern technology and to create a surplus for exports and
stimulating demand for processed food. The ministry is currently headed by Harsimrat Kaur Badal,
a Cabinet Minister.

43. Ans- (b) Cabinet Committee on Economic Affairs


CCEA has a mandate to review economic trends on a continuous basis, as also the problems and
prospects, with a view to evolving a consistent and integrated economic policy framework for the
country.

44. Ans- (d) None of the above


All the above statements are aims and objects of PMMSY as provided by the Ministry.

45. Ans-(a) KirenRijiju


KirenRijiju (born 19 November 1971) is an Indian lawyer and politician from Arunachal Pradesh. A
member of the Bharatiya Janata Party, Rijiju is the current Minister of State (Independent Charge)
of the Ministry of Youth Affairs and Sports and Minister of State in the Ministry of Minority Affairs of
India.
46. Ans- (b) Nada India
On 30th June 2020, the National Anti-Doping Agency (NADA) has launched its mobile application
‗NADA INDIA‘ for creating a bridge with the athletes under which information on prohibited
substances would be provided easily.
47. Ans- (a) New Delhi
NADA is headquartered in New Delhi, founded on 24th November 2005 for monitoring and
promoting doping control programs in all forms of sports in India.
48. Ans- (a) International Olympic Committee
The World Anti-Doping Agency is a foundation initiated by the International Olympic Committee
based in Canada to promote, coordinate and monitor the fight against drugs in sports. The agency's
key activities include scientific research, education, development of anti-doping capacities, and
monitoring of the World Anti-Doping Code, whose provisions are enforced by the UNESCO
International Convention against Doping in Sport. The aims of the Council of Europe Anti-Doping
Convention and the United States Anti-Doping Agency are also closely aligned with those of
WADA.
49. Ans- (b) Societies Registration Act
National Anti Doping Agency (NADA) was set up as registered society under the Societies
Registration Act of 1860 on November 24, 2005 with a mandate for Dope free sports in India. The
primary objectives are to implement anti-doping rules as per WADA code, regulate dope control
programme, to promote education and research and creating awareness about doping and its ill
effects.
50. Ans- (b) Bhutan
K.K. Venugopal has been conferred with the second highest( Padma Vibhushan) and the third-
highest (Padma Bhushan) civilian honour by the Indian Government. In 2002, he was conferred
with Padma Bhushan while in 2015 he was awarded the Padma Vibhushan. When Bhutan‘s
Constitution was drafted, he served as the Constitutional Adviser for the Royal Government of
Bhutan.
51. Ans- (d) Tushar Mehta
Currently, the Solicitor General of India is Tushar Mehta. Like the Attorney General for India, the
Solicitor General and the Additional Solicitors General advise the Government and appear on
behalf of the Union of India in terms of the Law Officers (Terms and Conditions) Rules, 1972.
52. Ans- (c) M.C Setalvad
M.C Setalvad served as the first Attorney General for India from 1950 to
1963.ChanderKishanDaphtary( 1 April 1893 – 18 February 1983) was an Indian lawyer and was the
first Solicitor General of India from 1950 to 1963.
53. Ans- (d) None of the above
The Solicitor General of India is subordinate to the Attorney General for India. He/She is the second
law officer of the country, assists the Attorney General, and is himself/herself assisted by Additional
Solicitors General for India. Like the Attorney General for India, the Solicitor General and the
Additional Solicitors General advise the Government and appear on behalf of the Union of India in
terms of the Law Officers (Terms and Conditions) Rules, 1972. However, unlike the post of Attorney
General for India, which is a Constitutional post under Article 76 of the Constitution of India, the
posts of the Solicitor General and the Additional Solicitors General are merely statutory.
Appointments Committee of the Cabinet(ACC) recommends the appointment and president
officially appoints the Solicitor General. The proposal for appointment of Solicitor General,
Additional Solicitor General is generally moved at the, level of Joint secretary/Law Secretary in the
Department of Legal Affairs and after obtaining the approval of the Minister of Law & Justice, the
proposal goes to the ACC and then to the president.
54. Ans- (b) Padma Shri
Padma Shri, also spelt Padma Shree, is the fourth-highest civilian award in the Republic of India,
after the Bharat Ratna, the Padma Vibhushan and the Padma Bhushan. It is awarded by the
Government of India every year on India's Republic Day.
K.K. Venugopal has been conferred with the second highest( Padma Vibhushan) and the third-
highest (Padma Bhushan) civilian honour by the Indian Government.
55. Ans- (a) Maharashtra
The World‘s largest Convalescent Plasma Therapy Trial project was launch on 29th June 2020 by
the Maharashtra State Government. The first plasma bank however will be set up in Delhi.
56. Ans-(d) Platina
The name of the project is Platina. The project was virtually launched by the Chief Minister of
Maharashtra Uddhav Thackeray. For this trial project, the Indian Council of Medical Research has
given the approval to conduct plasma therapy in COVID-19 positive patients admitted in the 21
COVID-19 hospitals of the Maharashtra State Government.
57. Ans- (a) Covaxin
COVAXIN, India's indigenous COVID-19 vaccine by Bharat Biotech is developed in collaboration
with the Indian Council of Medical Research (ICMR) - National Institute of Virology (NIV). The
indigenous, inactivated vaccine is developed and manufactured in Bharat Biotech's BSL-3 (Bio-
Safety Level 3) high containment facility.
The vaccine received DCGI approval for Phase I & II Human Clinical Trials and the trials will
commence across India from July, 2020.
58. Ans- (d) Favipiravir
Glenmark Pharmaceuticals Limited has announced the introduction of Favipiravir under the brand
name FabiFlu in tablet form for the treatment of mild & moderate coronavirus (COVID-19) cases in
India. Nearly 80% of all COVID-19 positive cases in India are under this category. FabiFlu is India‘s
first oral antiviral drug for treatment of mild & moderate COVID-19 patients, the company said.
59. Ans- (c) It is responsible for blood clotting.
Plasma is the main component of blood and consists mostly of water, with proteins, ions, nutrients,
and wastes mixed in. Red blood cells are responsible for carrying oxygen and carbon dioxide.
Platelets are responsible for blood clotting. White blood cells are part of the immune system and
function in immune response.
60. Ans- (b) Lazarus Chakwera
Lazarus Chakwera was elected as the new President of Malawi after winning 2,604,043 votes,
which is 59.34 percent of the total valid votes. On 23rd June 2020, the Presidential Elections of
Malawi was conducted after the annulment of the results of the 2019 Presidential Elections by the
Constitutional Court of Malawi earlier in the month of February 2020.
61. Ans-(b) It is generally a blue colour liquid used to make corrections that removes print marks
from paper.
The Tipp-Ex correction fluid is a white liquid. It is used for painting over mistakes in a piece of
writing. A brush (which was later replaced by a foam applicator) is attached to the cap, so when the
bottle is closed, the brush is immersed in the Tipp-Ex.
62. Ans- (c) Five or more
As the number of the judges has increased, they sit in smaller benches of two(division bench) or
three (full bench)—coming together in larger benches of five or more (referred to as a constitution
bench) when required to settle fundamental questions of law.
63. Ans- (d) Election of the President shall be held in accordance with the system of proportional
representation by means of single transferable vote and the voting at such election shall be by
secret ballot by an electoral college.
The President is elected by an Electoral College, which consists of the elected members of both
Houses of Parliament and the elected members of the Legislative Assemblies of all the States and
also of NCT of Delhi and the Union Territory of Puducherry. [Article 54 of the Constitution of India].
64. Ans- (a) SaulosChilima
Saulos Klaus Chilima (born 12 February 1973, Ntcheu) is a Malawian economist and politician who
is the current vice president of the Republic of Malawi
65. Ans – (d) KirenRijiju
KirenRijiju is an Indian lawyer and politician from Arunachal Pradesh. A member of the Bharatiya
Janata Party, Rijiju is the current Minister of State of the Ministry of Youth Affairs and Sports and
Minister of State in the Ministry of Minority Affairs of India.
66. Ans – (d) 2014
The Target Olympic Podium Scheme is a flagship program of the Ministry of Youth Affairs and
Sports which is an attempt to provide assistance to India‘s top athletes. In order to realize India‘s
Olympic medal dream, at the 2016 (Rio) and 2020 (Tokyo) Olympics, the Ministry of Youth Affairs
and Sports (MYAS) established the Target Olympics Podium Scheme (TOPS) in September 2014.
67. Ans – (c) Los Angeles
The 2028 Summer Olympics, officially known as the Games of the XXXIV Olympiad, and commonly
known as Los Angeles 2028 or LA 2028, is a forthcoming international multi-sport event that is
scheduled to take place from July 21 to August 6, 2028, in Los Angeles, California, United States.
68. Ans – (c) P.V Sindhu, Sunil Chhetri
A total of six sessions will be organized until 14th July. Leading sports celebrities will participate in
these Talk sessions to share their life experiences from failures struggles and to success, also will
be sharing fitness tips. In the first session on 3rd July P.V Sindhu and Sunil Chhetri participated.

QUANTITATIVE TECHNIQUES

69. Ans: (d)


Total number of gold bangles sold by store R in June,
July and August together = 35% of (87+105+130)
= 35% of 322 = .35100×322=112.7
≈ 113

70. Ans: (c) Request difference = (121 + 145)- (89 + 133) = 266 – 222 = 44
71. Ans: (c)
Request average = 129+87+1653=3813=127

72. Ans: (a)


Requested ratio = (S+T)August(S+T)September
= 114+129220+131=243351=913=9:13

73. Ans: (d)


Reqd % increase = 177−120120×100=57120×100=57×56=47.5%

74. Ans: (b)


People buying chocolate flavor in 2002 and 2003 together = 400+ 300 =700
People buying vanilla flavor ice-cream in = 2005 and 2006 = 350+300= 650
Required percentage = 50650×100=7.69 %

75. Ans: (d)


People buying vanilla flavor in 2002 except children = 375 – 150= 225

76. Ans: (b) People buying vanilla flavor in 2003 = 375


People buying all three flavor in 2005 = 350+250+275 = 875
Required percentage = 875−375875×100 ~ 57%

77. Ans: (a)


people buying vanilla flavor ice-creams in 2001, 2004 and 2006 = 400 + 250 + 300 = 950
People buying strawberry flavor overall years
= 300 + 350 + 325 + 300 +275 +175 = 1725
Required ratio = 9501725=3869

78. Ans: (a)


X = 5502=275
Number of people buying chocolate in 2007 = (100+ 10011 ) % 275
= 120011×100×275=300

79. Ans: (c)


Number of students playing Carrom and Hockey together from school P = 220 + 140 = 360
Number of students playing Carrom and Hockey together from school R = 200 + 320 = 520
Required % = 360520×100=69313%

80. Ans: (d)


Total number of students in schools S = 260 + 320 + 160 = 740
Total number of students in school Q = 240 + 180+ 260 = 680
Required Difference = 115100×740−95100×680
= 851 – 646 = 205

81. Ans: (c)


Number of students playing cricket from,
School Q = 180
School S= 320
School T = 240
Required Students = 40100×180+35100×320+45100×240
= 72 + 112 + 108 = 292

82. Ans: (d)


Total number of students playing Hockey from all school = 140+260+320+160+180=1060
Total number of students playing cricket from all school = 360+180+240+ 320+240= 1340
Required % = 10601340×100 ≈79%

83. Ans: (a)


Number of students playing Carrom from school P = 220
School P = 220
School T = 280
Required students = 25100×40100×220+23×45100×280
= 22+ 84 = 106
LOGICAL REASONING

84. Explanation: (d) Our speech can be a curse if it creates misunderstanding between the people.
Hence, it demands careful handling of words.
85. Explanation: (b) 'A slip of tongue' means an accidental and usually a trivial mistake in
speaking, but it is totally unintentional.
86. Explanation: (d) We should use simple words and common language, so that common man
may understand what we are saying.
87. Explanation: (d) According to the passage: Again different classes of people use different
vocabularies and the ordinary speech of an educated man may strike an uneducated listener
as pompous. Therefore it can be inferred that the educated man will be considered as arrogant by
the uneducated man.
88. Explanation : Correct answer is A
A. The passage contains a mixture of information about the aye-aye, both from a scientific and
cultural background. It gives an overview of the animal without giving a lot of detail in any one area.
Choice (B) is incorrect because the passage mentions evolution only briefly, at the very end. This
choice is too narrow. Choice (C) is incorrect because the style of the passage is too advanced for
young students. Choice (D) is incorrect because the information given is focused more on the aye-
aye itself than on the culture of Madagascar.
89. Explanation : . Correct answer is A
A. The author refers to the aye-aye as a ―superb example of life‘s variety.‖ Because this is a
positive statement, look for a positive answer. Choice (D) is negative. Choice (B) means confused.
Choice (C) is positive but is too extreme. Therefore, choice (A), admiring, is the best answer.
90. Explanation .A. Choice (A) is supported by the first paragraph. The classification of the aye-
aye changed, which demonstrates that such classifications are not absolute. Choice (C), however,
is not supported. Although the passage states that the ayeaye is in danger, it does not directly
discuss whether this is due to limited resources.
91. Explanation: C. Choice (C) is supported by the part of the passage dealing with the future of
the aye-aye. It states that the aye-aye is seen as an omen of death in the traditional religion of
Madagascar. Augury refers to the use of omens, so this statement must be true.
92. Explanation: C. Choice (C) is correct. The second half of the passage talks about its
adaptation and mentions 1 as well as 2.
93. Explanation : D. Choice (D) can be inferred from the first sentence of the passage. Choices
(A), (B) and (C) are extraneous.
94. Explanation: C. Choice (C) is correct as mentioned in the last sentence of the passage: loss of
large swathes of jungle habitat i.e. natural environment
95. EXPLANATION: Correct answer is C
The entire passage is primarily concerned with discussing the range or spread of the American
bison, so (C) should be the correct answer.
(A) ‗Ecology‘ is too broad a term and, though the author does mention this in passing, this is not the
main purpose of the passage.
(B) Conservation is never discussed in the passage.
96. EXPLANATION:Correct answer is D
It is not stated anywhere in the passage that the bison population fell drastically at any time. Hence,
(D) is the correct answer.
(A) This is implied by the last line of the first para.
(B) This is mentioned in the second para.
(C) This is implied by the last line of the first para.
97. EXPLANATION: Correct answer is B
The lines in the last paragraph ‗would have developed still longer hair‘ imply that the bison was
already a long-haired animal. Hence, (B) is the correct answer.
(A) The passage never states this.
(C) The passage does mention the Indian Gaur but not in this context.
(D) The passage never states either of these.
98. Explanations: (a) Statements 1 and 2 are clearly implicit from the passage, while statements 3
and 4 are not mentioned. Hence, option (a) is the correct answer.
99. Explanations: (d) Inclusive growth can be effected by focusing on delivering the basic services
to the deprived sections of the society. Hence, option (d) is the correct answer.
100. Explanations: (c) Statements 1 and 2 cannot be implied from the information given in the
passage while statements 3, 4 and 5 are correct according to passage. Hence, option (c) is the
correct answer.
101. Explanations: (d) Both the statements are wrong according to the passage. Hence, option (d)
is the correct answer.
102. Explanations: (d) The essential message conveyed by the author is the need to rethink the
role of the state in achieving the inclusive growth.
103. Explanations: (d). The argument introduces the problem of achieving inclusive growth. The
solution is to have an enabling state. Hence D
104. Explanation: (c). Statement 1 is correct. The economic case is made clearly in the second
sentence of the first paragraph. Statement 3 is also correct. Eutrophication due to discharge of
agricultural slurry is mentioned in the second last sentence of the second paragraph. In fact,
eutrophication has been elaborated upon later in the third paragraph. Statement 2 however is
incorrect.
While the last sentence of the first paragraph does mention that soil microorganisms can
decompose the organic components of agricultural slurry, nowhere it is mentioned that the same
microorganisms are not present in the watercourses. Thus, the right answer is option (c).
105. Explanation: (b). this answer should be clear from the last line of the first paragraph. Here, it is
mentioned that soil microorganisms decompose the organic components of slurry which makes
nutrients available to be absorbed by the vegetation. Thus, fertiliser refers to the decomposed
slurry. The previous sentence in the same paragraph also mentions that pollutant is converted into
fertiliser. Thus, before its decomposition by the soil microorganisms, the agricultural slurry was a
pollutant. Thus, the right answer is option (b).
106. Explanation: (b). Statement 1 is correct that agricultural slurry is a pollutant which is clear from
the first paragraph. It is the use of fertilisers which creates agricultural slurry. Statement 2 is
incorrect. Nowhere in the passage is the destruction of decomposed microorganisms mentioned.
Statement 3 is also correct. It is clear from the last paragraph that runoff of fertilisers causes
nutrient-enrichment of water bodies. Statement 4 is also correct. That algal blooms are created by
fertiliser use, mentioned in the last paragraph. Hence B
107. Explanation: (b). Statement 1 is correct. Ecosystem services are lost is clearly mentioned in
the last sentence of the second paragraph. Statement 2 is also correct. Loss of various flora and
fauna is mentioned in the first few sentences of the last paragraph. Statement 3 is incorrect. Loss of
mineral nutrients due to cultural eutrophication is not mentioned anywhere in the passage.
108. Explanation: (c). Option (a) is incorrect. Though the passage initially talks of legislation related
to agricultural slurry, it is not the central theme of the passage. Option (b) is also incorrect. The
passage does not talk of modern agriculture as a whole and neither does it talk of destruction of the
whole environment. Option (d) is also incorrect. The passage does not say that chemical fertilisers
are undesirable or should be gotten rid of, but instead mentions that they can be used smartly to
avoid negative effects on the environment. Option (c) is correct. The passage revolves around the
theme of disposal of agricultural slurry, its negative impact on aquatic ecosystems and their
avoidance.
109. Explanation: (a). Option A is true according to the first sentence of the passage. Option C is
incorrect as the passage mentions 150 sea areas and not 150 seas.
110. Explanation: Choice C. Only Option C is in accordance with the central theme (discussed in Q
5 above) of the passage. Hence C
111. Ans: (d)
Candidates from Science block courses – X, Y, Z. Candidates from commerce block courses- L, M,
N, P. Incompatible pairs – Y and L, Z and N, L and M.
In option (a), L and M cannot be together, In option (b), N and Z cannot be together, In option (c)
three persons from commerce block i.e. M, N and P are given, So, it is also not possible.
Only the team given in option (d) can be possible.

112. Ans: (b)


Candidates from Science block courses – X, Y, Z. Candidates from commerce block courses- L, M,
N, P. Incompatible pairs – Y and L, Z and N, L and M.
Option (a) is eliminated because M cannot be with L. Option (c) and (d) are also eliminated
because Z and N cannot be in same team. Only the team given in option (b) can be formed.

113. Ans: (b)


Candidates from Science block courses – X, Y, Z. Candidates from commerce block courses- L, M,
N, P. Incompatible pairs – Y and L, Z and N, L and M.
Option (a) is eliminated because M cannot be with L. Option (c) and (d) are also eliminated
because Z and N cannot be in same team. Only the team given in option (b) can be formed.

114. Ans: (d)


Date Day Exam Time Duration
2nd March Wednesday History 60 mins
3rd March Thursday Maths 50 mins
4th March Friday English 90 mins
5th March Saturday Hindi 100 mins
6th March Sunday Off Off
7th March Monday Economics 75 mins
8th March Tuesday Science 40 mins

115. Ans: (b)


Date Day Exam Time Duration
2nd March Wednesday History 60 mins
3rd March Thursday Maths 50 mins
4th March Friday English 90 mins
5th March Saturday Hindi 100 mins
6th March Sunday Off Off
7th March Monday Economics 75 mins
8th March Tuesday Science 40 mins

LEGAL REASONING

116. Answer- (b) When two or more person plan to commit an offence, they are said to have
committed criminal conspiracy. Option (c) is incorrect as there is no mention of defamation in the
given passage.

117. Answer- (a) all four of them shared a common intention of committing robbery and therefore all
four of them will be held liable.

118. Answer- (a) D had no knowledge about the robbery nor did he conspired with A, B and C for
the same. He was a mere taxi driver.

119. Answer- (d) C cannot seek relief that he was not a part of the conspiracy from its very
inception and therefore, should not punished for the same. A person who joins the conspiracy
knowing the objective even at later stage will be held liable.

120. Answer- (a) It is not essential that the accused person must do an overt act, and mere
agreement between two or more persons to commit an illegal act is sufficient to constitute the
offence of criminal conspiracy. It is also not necessary that the object of the conspiracy should have
been achieved for it to be considered as an offence.

121. Answer- (a) Option (a) cannot be referred from the passage though the statement is true.
122. Explanation: The answer is (c). This is because the rights enjoyed under copyright can be
exercised only by the owner of the copyright or by any other person who is duly licensed in this
regard by the owner of the copyright. These rights include the right of adaptation, right of
reproduction, right of publication, right to make translations etc. thereby making consent an integral
part in the instant case.

123. Explanation: The answer is (d). This is for the reason that until and unless the idea has not
been expressed into a concrete form it cannot fall under copyright infringement. The idea belonged
to X but the expression was undertaken by Y when he released the movie.

124. Explanation: The answer is (c). This is because certain scenes are common and intrinsic to a
genre in movies. Such scenes cannot be claimed under copyright as they tend to exist in every
movie of that category to generate a desired atmosphere and result. Therefore, such areas cannot
be considered anyone‘s original work.

125. Explanation: The answer is (d). This is because themes cannot be considered an original work
of anyone. The infringement begins in the expression of the theme. If the expression is different,
then it would not amount to infringement.

126. Explanation: The answer is (b). This is because the techniques or the accounting standards
are general in nature. They are used and performed by everyone in the same manner to assess
their financial position. Therefore, this cannot be copyrighted by anyone.

127. Answer- (c) Telling one‘s servant to do and not to do certain things for the business and
warning them against any bad customer will not qualify as defamation. The statement was made in
good faith.

128. Answer- (d) Questioning a women‘s character while she is involved in contractual fraud is
wrong. Mr. Unjust will not be protested under statutory authority as his statement were ultra vires
and had nothing to do with issue of contractual fraud. Therefore he will be liable for defamation.

129. Answer- (b) When the Standard initially made the statement that other newspapers exhibit
wrong information, the statement was made in general and not directing towards Birmingham Post
and therefore it will not amount to defamation.

130. Answer- (a) When the Standard initially made the statement that other newspapers exhibit
wrong information, the statement was made in general and not directing towards Daily Post or any
particular newspaper and therefore it will not amount to defamation.

131. Answer- (b) Birmingham Post in a retaliation act initiated a campaign against The Standard
maligning its reputation and therefore will be held liable for defamation.

132. Explanation: The answer is (a). This is because Amita went inside and consented for saving
the child. Since she had the knowledge of the risk she did consent to get harmed by the fumes.
Unless we are prescribed rescue case as an exception, we cannot apply it.

133. Explanation: The answer is (c). This is because the injuries so sustained are too excessive to
be considered a part of the game. Therefore, in the instant case Manoj neither had the knowledge
of auch injuries nor did he consent to it.
134. Explanation: The answer is (b). This is because of the reason that the risk and danger were
blatantly visible and her harder effort to pull out the lid of boiling vat shows the voluntary
acceptance of the risks involved.

135. Explanation: The answer is (c). This is because Meena had the knowledge that the surgery will
lead to the removal of lumps so she consented to that particular segment. Moreover, even when the
doctor was acting in the best interest of Meena he had flimsy reasons to decide on the cause of
formation of the lumps.

136. Explanation: The answer is (c). This is because even when the tunnel was owned by the
railways they were aware of the public usage of it and accordingly the instructions were being given
to the drivers. So, if Amit had the knowledge of the risk of narrow tunnel but he did not have the
knowledge of negligent driving therefore not consenting to it.

137. Answer- (c) Mukesh Mehra put Om on fear and caused him threat by saying that he will put
Shanti on fire. Om who is pyrophobic (fear of fire) got scared and delivered his one lakh. Here
Muksh has committed extortion as while doing so he had mala fide intention.

138. Answer- (c) The three essential elements mentioned in the above passage are not fulfilled in
the given situation. The third ingredient of ―deliver property or valuable security‖ is not fulfilled and
therefore it will not amount to the case of extortion.

139. Answer- (b) Putting a person in fear of injury to body, property or reputation is an important
ingredient from causing extortion. The same is missing in the present case and therefore Rachel
will not be held liable.

140. Answer- (b) Whoever intentionally puts any person in fear of any injury to that person, or to
any other, and thereby dishonestly induces the person so put in fear to deliver to any person any
property or valuable security, or anything signed or sealed which may be converted into a valuable
security, commits ―extortion‖. As there was no demand for valuable security or property, this will not
amount to a case of extortion. Also option (a) is not correct as it mentions another offence of
outraging the modesty of a women which forms no mention in the passage.

141. Answer- (c) ―It is a non- bailable offence and cannot be compounded.‖ And therefore it cannot
be settled outside the court.

142. Answer- (c) The Apex Court also reiterated that fundamental rights (right to association here)
are applicable only to laws made by or administrative actions of the State and do not apply to
actions of private persons.

143. Answer- (d) from the above passage it can be referred that the fundamental right of freedom of
association also includes the right not to join an association. Here making compulsory to join
association will violate once right guaranteed under constitution.

144. Answer- (d) As per the information given in the passage, fundamental rights (right to
association here) are applicable only to laws made by or administrative actions of the State and do
not apply to actions of private persons.

145. Answer- (c) there is no reference to option c in the passage.


146. Answer- (a) in the given passage- the Supreme Court held- The Act violated the rights of the
original members of the society to form an association guaranteed under Art 19 (1) (c). Mohan will
succeed as right to form association is guaranteed under Article 19(1) (c) of the constitution.

147. Answer- (a) The above mentioned passage mentions the ordinance for the state of Uttar
Pradesh particularly and has no mention of any penal action against a person buying a cow for the
purpose of slaughtering it in another state.

148. Answer- (b) The answer will not remain the same as now Hamid Miyan will be penalized under
the law as Uttar Pradesh Prevention of Cow Slaughter (Amendment) Ordinance, 2020 considers
cow slaughter as an offence.

149. Answer- (a) Hardeep is just transporting someone else‘s consignment. He neither slaughtered
the cow nor is he in the meat business. Therefore UP police did not act as per the law given in the
passage.

150. Answer- (b) Question already mentions that the argument put forth by Manav is true, so option
(a) becomes irrelevant. Now that he is not involved in cow slaughter he will not be held liable under
the law passed by the UP government. Answer- (b) Question already mentions that the argument
put forth by Manav is true, so option (a) becomes irrelevant. Now that he is not involved in cow
slaughter he will not be held liable under the law passed by the UP government.
Legal Edge 02 Mock

Answers & Details

ENGLISH

1. Ans:(d)
a direct question. The first three statement are given in the passage. Classification is also done by
understanding the evolutionary history of the plant. This is known as phylogenetic system.

2. Ans:(a)
Anatomy is the study of structure whereas physiology is the study of function. The study of tissues
is known as histology and the study of diseases of the body is known as pathology. The correct
answer is option (a).
3. Ans:(d)
option (a) as Artificial system is a non- revolutionary system. i.e harldly any radical changes has
been seen. One of the principles of botanical system is phylogenetic system. Hence, the history of
plants is also an important characteristics. Evolutionary traits can be seen in the cladistic system.
The correct statement is option (D).
4. Ans:(b)
Angiosperms and gymnosperms are a part of vascular plants. The seed and seedless plants are
divided into 5 groups each. Hence, altogether Vascular plants have 10 different classes.
5. Ans:(d)
phenetic system is used to eliminate subjectivity by using described and measured characters.
Option (a) is incorrect because this is how phenetic system eliminates the subjectivity. Option (b)
(c) is a part of cladistic and natural system respectively. Phenetic system doesn‟t show the
revolutionary system which means it shows the similarities
6. Ans:(d)
The first para talks about why e- cigarettes are harmful to the human body. Option (a) is true as this
is one of the reason why e- cigarettes is harmful to the body. Option (b) is partly correct as we don‟t
know whether everytone switch to e-cigarettes. The last para is talking majorly about youngsters.
People is a much broader term. Option (c) is altogether incorrect. The passage says that e-
cigarettes are is less harmful than cigarettes but it‟s not harmless. The correct option is (d) which
gives us the major reason why e-cigarettes were banned.
7. Ans:(b)
Option (a) which states that Police officers cannot arrest him without warrant is when it‟s a
cognizable offence. Option (b) is correct. Option (c) also lies under Non- cognizable offences.
Option (d) bail will be given when it‟s a non- bailable offence.
8. Ans:(c)
An ordinance is described as the legislative power of the President, it is issued on the advice of the
Council of ministers and is hence considered to be a law made by the executive. Option (d) and (c)
is true whereas option (a) is also true as Article 123 of the Indian Constitution grants the President
certain law making powers to promulgate when either the two houses of parliament are not in
session. Hence, Option (c) is incorrect in regards to the passing of an ordinance.
9. Ans:(d)
Option (b) is correct. Option (c) is also correct, if the imports of e-cigarettes have increased in India.
We can infer that exports have also increased. We aren‟t given a specific state but overall the
exports have increased which is true. Option (a) might seem partly wrong as cigarettes are more
harmful but if we look at the first half it tells how e- cigarettes were used to aid smoker but later
introduces them to cigarettes. It‟s true that New York recently banned flavored e- cigarettes but we
can‟t say whether it was the largest consumer. New York is a part of USA which is the largest
consumer of e-cigarettes.
10. Ans:(a)
the second para talks about the nictotine addiction, evidences are given to make the reader
understand the harmful effects of nicotine. Hence, the last few lines states the ban of nicotine which
is harmful to the health. Hence, the above statement is true.
11. Ans:(b)
This is a very tricky question, to choose the answer it‟s better to understand the passage. One
might feel option (a) (b) makes sense, whereas (c) (d) can be eliminated as (c) is incomplete, ultra
nationalism of, about? Whereas option (d) is incorrect. The passage is talking about the liberation of
Auschwitz. The final solution does make sense but the passage is talking about the journey and
why we should not forget the Holocaust. Hence, the correct answer is option (b).
12. Ans:(d)
option (a) is false. The inmates were liberated on 27th January, 1945. Option (b) is partly correct. 6
Million Jews were killed in the prison . People can be of different religion. Holocaust had a lot to do
with the Jews. Option (c) is incorrect. Nazi plan was to institute an Homogeneous society. i.e. a
similar, like society, Uncanny means strange or mysterious society.
13. Ans:(a)
option (b) and (d) are the characteristics which we aren‟t sure of as they aren‟t given explicitly.
Option (c) is incorrect altogether. Nazi believed in EUGENICS WHEREAS Dysgenics is just the
opposite.
14. Ans:(c)
The entire RC talks, about how important it is to know and remember holocaust. Only then can we
ensure that it doesn‟t happen again.
15. Ans:(a)
option (c) is incorrect. Holocaust remeberance day is celebrated to commemorate the genocide
rather than obliviating it. Option (b) and (d) are incomplete. Doesn't make sense. The correct
answer is option(a).
16. Ans:(d)
The question states did the Nazi believed they could improve the Human race. Certainly they did
believe. The killing of Jews and the aim to unite Germans in a racially pure society shows the
intentions of the Nazi. Hence, option (c) and (d) can be eliminated. Option (a) is incorrect. Yes, they
did believe they can improve human race. Option (a) is confusing, state sponsored genocide was
the one of the main reasons why they could improve the human race. The statement is incomplete.
To improve human race was not state sponsored?
State sponsored genocide was a way to improve human race. The correct answer is option (d).
17. Ans:(a)
The correct answer is to unite Germans in historically German territory.
18. Ans:(a)
Auschwitz was liberated on 27th January 1945. Option (c) and (d) are partly correct. No specific
date is given.
19. Ans:(b)
Direct question, the passage starts with Turicum being the capital of Zurich
20. Ans:(d)
Zurich gained imperial immediacy
21. Ans:(d)
Aelmannic is the dialect used in Zurich
22. Ans:(d)
Zurich is known for Finance and Banking even though it‟s a small place.
23. Ans:(d)
the correct answer is option (d) Kunsthaus Art museum
24. Ans:(d)
Refer to para 3rd, it says that USSR could have been the victim of atomic attack that would have let
third would war.
25. Ans:(b)
Refer to para 2, line 3rd and 4th..
26. Ans:(d)
Refer to para 1st, line 1st.
27. Ans:(b)
Refer to para 4th, line 6th and 7th
28. Ans:(a)
Refer to para 1, line 2nd
29. Ans:(b)
Refer to para 2nd, last line (6th line)

GENERAL KNOWLEDGE
30. Ans:(d)
Kholongchhu Hydroelectric Project is located on the lower course of the Kholongchhu River in
Trashiyangtse District in Eastern Bhutan. It is a joint project between India and Bhutan.
31. Ans:(c)
The 600 MW run-of-the-river project envisages an underground powerhouse of four 150 MW
turbines with water impounded by a concrete gravity dam of 95 meters height.
32. Ans:(d)
The Mangdechhu hydroelectric project (HEP) is a 720MW run-of-river power plant built on the
Mangdechhu River in Trongsa Dzongkhag District, central Bhutan. It was officially inaugurated in
August 2019.
33. Ans:(b)
The Kholongchhu Joint Venture project will be constructed as a 50:50 joint venture between Satluj
Jal Vidyut Nigam (SJVN), and the Druk Green Power Corporation (DGPC). The project is expected
to be completed in the second half of 2025.
34. Ans:(c)
Ganga Prasad Chaurasia is an Indian politician who is currently the Governor of the state of Sikkim
and former Governor of Meghalaya. He was member of Bihar legislative council and served as
leader of opposition during the RJD regime.
35. Ans:(a)
Madhya Pradesh has the largest forest cover in the country followed by Arunachal Pradesh,
Chhattisgarh, Odisha and Maharashtra. In terms of forest cover as a percentage of their total
geographical area, the top five states are Mizoram (85.41 percent), Arunachal Pradesh (79.63
percent), Meghalaya (76.33 percent), Manipur (75.46 percent) and Nagaland (75.31 percent)
36. Ans:(b)
According to the 2019 report, the total forest cover of the country is 712,249 square kilometres
(21.67 percent of India‟s total geographical area) slightly up from 708,273 sq. km (21.54 percent) in
2017. The tree cover of the country is 95,027 sq. km (2.89 percent of the total area) again slightly
up from 93,815 sq. km. (2.85 percent) in 2017.
37. Ans:(a)
Maharashtra has had the highest increase in tree cover and a large part of that is due to horticulture
in the state.
38. Ans:(a)
Amongst the States, Gujarat has the largest area of wetlands within RFA in the country followed by
West Bengal.
39. Ans:(d)
Arunachal Pradesh, has the highest carbon stock in India ((1,051 million tonnes), and has reported
a loss of 276 sq. km. forest cover even as its carbon stock rises according to the report.
40. Ans:(a)
There has been a decrease of forest cover to the extent of 765 sq km (0.45%) in the region. Except
Assam and Tripura, all the States in the region show decrease in forest cover.
41. Ans:(a)
The principal aim of National Forest Policy, 1988 is to ensure environmental stability and
maintenance of ecological balance including atmospheric equilibrium which are vital for sustenance
of all life forms, human, animal and plant. The derivation of direct economic benefit must be
subordinate to this principal aim. The nation‟s tree and forest cover has largely hovered from 21-
25% and is short of the National Forest Policy, 1988, which envisages 33% to be under such cover.
42. Ans:(d)
It is the first renewable energy project to supply to institutional customers outside the state,
including Delhi Metro, which will get 24 per cent of energy from the project. The remaining 76 per
cent is being supplied to the state distribution companies of Madhya Pradesh.
43. Ans:(c)
The Rewa Solar project comprises three solar generating units of 250 MW each, totaling 750 MW,
located on a 500-hectare plot inside a solar park.
44. Ans:(b)
The International Finance Corporation (IFC) is an international financial institution that offers
investment, advisory, and asset-management services to encourage private-sector development in
less developed countries. It is a part of World Bank group and is headquartered in Washington DC.
Rewa Solar Project is the first project to get funding from The World Bank and Clean Technology
Fund in India.
45. Ans:(c)
The National Solar Mission was inaugurated by former Prime Minister Manmohan Singh on 11
January 2010 with a target of 20GW by 2022 which was later increased to 100 GW by the Narendra
Modi government in the 2015 Union budget of India. The original target of 20 GW was surpassed in
2018, four years ahead of the 2022 deadline.
46. Ans:(c)
Raj Kumar Singh is a former Indian bureaucrat and a current Minister of State (Independent
Charge) of the Ministry of New and Renewable Energy. Singh is a 1975 batch Bihar cadre Indian
Administrative Service officer and former Home Secretary of India.
47. Ans:(b)
Gaganyaan is an Indian crewed orbital spacecraft that is intended to send 3 astronauts to space for
a minimum of seven days by 2022, as part of the Indian Human Spaceflight Programme. Indian
Space Research Organisation chief K Sivan announced that India‟s third lunar mission,
Chandrayaan-3 is on and has been approved by the government. The Chandrayaan-3 mission is
slated to launch in 2021. To expand Isro's infrastructure further, a new launch pad is set to come up
soon in Tuticorin in Tamil Nadu. The Indian Space Research Organisation in its modern form was
created by Vikram Sarabhai in 1969. Under the Gaganyaan schedule, three flights will be sent in
orbit. Of the three, there will be two unmanned flights and one human spaceflight. The human
space flight programme, called the Orbital Module will have three Indian astronauts, including a
woman. Although scores of landers sent by Russia, the U.S. and the Chinese have explored the
moon‟s surface, so far, no other agency has landed in the southern hemisphere of the moon. ISRO
hopes to be the first to do so.
The Indian Space Research Organisation (ISRO) is gearing up to lift off Brazil‟s Amazonia-1
satellite next month onboard the Polar Satellite Launch Vehicle (PSLV). Amazonia-1 will be the first
satellite for Earth Observation that is designed, assembled and tested in Brazil.The launch of the
Brazilian satellite is confirmed, but there is not clarity on a date for the planned launch. The report
quoted a top diplomat as saying that it all depends on the ISRO as to when it is planning to blast off
the satellite.
In the 2019 Budget, the government had announced the setting up of a New Space India Limited
(NSIL), a public sector company that would serve as a marketing arm of ISRO. Its main purpose is
to market the technologies developed by ISRO and bring it more clients that need space-based
services.
48. Ans:(c)
Gaganyaan is an Indian crewed orbital spacecraft that is intended to send 3 astronauts to space for
a minimum of seven days by 2022, as part of the Indian Human Spaceflight Programme. Indian
Space Research Organisation chief K Sivan announced that India‟s third lunar mission,
Chandrayaan-3 is on and has been approved by the government. The Chandrayaan-3 mission is
slated to launch in 2021. To expand Isro's infrastructure further, a new launch pad is set to come up
soon in Tuticorin in Tamil Nadu. The Indian Space Research Organisation in its modern form was
created by Vikram Sarabhai in 1969. Under the Gaganyaan schedule, three flights will be sent in
orbit. Of the three, there will be two unmanned flights and one human spaceflight. The human
space flight programme, called the Orbital Module will have three Indian astronauts, including a
woman. Although scores of landers sent by Russia, the U.S. and the Chinese have explored the
moon‟s surface, so far, no other agency has landed in the southern hemisphere of the moon. ISRO
hopes to be the first to do so.
The Indian Space Research Organisation (ISRO) is gearing up to lift off Brazil‟s Amazonia-1
satellite next month onboard the Polar Satellite Launch Vehicle (PSLV). Amazonia-1 will be the first
satellite for Earth Observation that is designed, assembled and tested in Brazil.The launch of the
Brazilian satellite is confirmed, but there is not clarity on a date for the planned launch. The report
quoted a top diplomat as saying that it all depends on the ISRO as to when it is planning to blast off
the satellite.
In the 2019 Budget, the government had announced the setting up of a New Space India Limited
(NSIL), a public sector company that would serve as a marketing arm of ISRO. Its main purpose is
to market the technologies developed by ISRO and bring it more clients that need space-based
services.
49. Ans:(d)
Gaganyaan is an Indian crewed orbital spacecraft that is intended to send 3 astronauts to space for
a minimum of seven days by 2022, as part of the Indian Human Spaceflight Programme. Indian
Space Research Organisation chief K Sivan announced that India‟s third lunar mission,
Chandrayaan-3 is on and has been approved by the government. The Chandrayaan-3 mission is
slated to launch in 2021. To expand Isro's infrastructure further, a new launch pad is set to come up
soon in Tuticorin in Tamil Nadu. The Indian Space Research Organisation in its modern form was
created by Vikram Sarabhai in 1969. Under the Gaganyaan schedule, three flights will be sent in
orbit. Of the three, there will be two unmanned flights and one human spaceflight. The human
space flight programme, called the Orbital Module will have three Indian astronauts, including a
woman. Although scores of landers sent by Russia, the U.S. and the Chinese have explored the
moon‟s surface, so far, no other agency has landed in the southern hemisphere of the moon. ISRO
hopes to be the first to do so.
The Indian Space Research Organisation (ISRO) is gearing up to lift off Brazil‟s Amazonia-1
satellite next month onboard the Polar Satellite Launch Vehicle (PSLV). Amazonia-1 will be the first
satellite for Earth Observation that is designed, assembled and tested in Brazil.The launch of the
Brazilian satellite is confirmed, but there is not clarity on a date for the planned launch. The report
quoted a top diplomat as saying that it all depends on the ISRO as to when it is planning to blast off
the satellite.
In the 2019 Budget, the government had announced the setting up of a New Space India Limited
(NSIL), a public sector company that would serve as a marketing arm of ISRO. Its main purpose is
to market the technologies developed by ISRO and bring it more clients that need space-based
services.
50. Ans:(c)
Gaganyaan is an Indian crewed orbital spacecraft that is intended to send 3 astronauts to space for
a minimum of seven days by 2022, as part of the Indian Human Spaceflight Programme. Indian
Space Research Organisation chief K Sivan announced that India‟s third lunar mission,
Chandrayaan-3 is on and has been approved by the government. The Chandrayaan-3 mission is
slated to launch in 2021. To expand Isro's infrastructure further, a new launch pad is set to come up
soon in Tuticorin in Tamil Nadu. The Indian Space Research Organisation in its modern form was
created by Vikram Sarabhai in 1969. Under the Gaganyaan schedule, three flights will be sent in
orbit. Of the three, there will be two unmanned flights and one human spaceflight. The human
space flight programme, called the Orbital Module will have three Indian astronauts, including a
woman. Although scores of landers sent by Russia, the U.S. and the Chinese have explored the
moon‟s surface, so far, no other agency has landed in the southern hemisphere of the moon. ISRO
hopes to be the first to do so.
The Indian Space Research Organisation (ISRO) is gearing up to lift off Brazil‟s Amazonia-1
satellite next month onboard the Polar Satellite Launch Vehicle (PSLV). Amazonia-1 will be the first
satellite for Earth Observation that is designed, assembled and tested in Brazil.The launch of the
Brazilian satellite is confirmed, but there is not clarity on a date for the planned launch. The report
quoted a top diplomat as saying that it all depends on the ISRO as to when it is planning to blast off
the satellite.
In the 2019 Budget, the government had announced the setting up of a New Space India Limited
(NSIL), a public sector company that would serve as a marketing arm of ISRO. Its main purpose is
to market the technologies developed by ISRO and bring it more clients that need space-based
services.
51. Ans:(c)
Gaganyaan is an Indian crewed orbital spacecraft that is intended to send 3 astronauts to space for
a minimum of seven days by 2022, as part of the Indian Human Spaceflight Programme. Indian
Space Research Organisation chief K Sivan announced that India‟s third lunar mission,
Chandrayaan-3 is on and has been approved by the government. The Chandrayaan-3 mission is
slated to launch in 2021. To expand Isro's infrastructure further, a new launch pad is set to come up
soon in Tuticorin in Tamil Nadu. The Indian Space Research Organisation in its modern form was
created by Vikram Sarabhai in 1969. Under the Gaganyaan schedule, three flights will be sent in
orbit. Of the three, there will be two unmanned flights and one human spaceflight. The human
space flight programme, called the Orbital Module will have three Indian astronauts, including a
woman. Although scores of landers sent by Russia, the U.S. and the Chinese have explored the
moon‟s surface, so far, no other agency has landed in the southern hemisphere of the moon. ISRO
hopes to be the first to do so.
The Indian Space Research Organisation (ISRO) is gearing up to lift off Brazil‟s Amazonia-1
satellite next month onboard the Polar Satellite Launch Vehicle (PSLV). Amazonia-1 will be the first
satellite for Earth Observation that is designed, assembled and tested in Brazil.The launch of the
Brazilian satellite is confirmed, but there is not clarity on a date for the planned launch. The report
quoted a top diplomat as saying that it all depends on the ISRO as to when it is planning to blast off
the satellite.
In the 2019 Budget, the government had announced the setting up of a New Space India Limited
(NSIL), a public sector company that would serve as a marketing arm of ISRO. Its main purpose is
to market the technologies developed by ISRO and bring it more clients that need space-based
services.
52. Ans:(c)
Gaganyaan is an Indian crewed orbital spacecraft that is intended to send 3 astronauts to space for
a minimum of seven days by 2022, as part of the Indian Human Spaceflight Programme. Indian
Space Research Organisation chief K Sivan announced that India‟s third lunar mission,
Chandrayaan-3 is on and has been approved by the government. The Chandrayaan-3 mission is
slated to launch in 2021. To expand Isro's infrastructure further, a new launch pad is set to come up
soon in Tuticorin in Tamil Nadu. The Indian Space Research Organisation in its modern form was
created by Vikram Sarabhai in 1969. Under the Gaganyaan schedule, three flights will be sent in
orbit. Of the three, there will be two unmanned flights and one human spaceflight. The human
space flight programme, called the Orbital Module will have three Indian astronauts, including a
woman. Although scores of landers sent by Russia, the U.S. and the Chinese have explored the
moon‟s surface, so far, no other agency has landed in the southern hemisphere of the moon. ISRO
hopes to be the first to do so.
The Indian Space Research Organisation (ISRO) is gearing up to lift off Brazil‟s Amazonia-1
satellite next month onboard the Polar Satellite Launch Vehicle (PSLV). Amazonia-1 will be the first
satellite for Earth Observation that is designed, assembled and tested in Brazil.The launch of the
Brazilian satellite is confirmed, but there is not clarity on a date for the planned launch. The report
quoted a top diplomat as saying that it all depends on the ISRO as to when it is planning to blast off
the satellite.
In the 2019 Budget, the government had announced the setting up of a New Space India Limited
(NSIL), a public sector company that would serve as a marketing arm of ISRO. Its main purpose is
to market the technologies developed by ISRO and bring it more clients that need space-based
services.
53. Ans:(a)
Dharmendra Pradhan is currently the Minister of Petroleum & Natural Gas and Minister of Steel in
the Second Modi Ministry.
54. Ans:(c)
Indian Oil, SDMC and NTPC came together to develop a Waste-to-energy Plant demonstration
using Gasification technology at Okhla landfill in Delhi.
55. Ans:(a)
In order to promote biofuels in the country, a National Policy on Biofuels was made by Ministry of
New and Renewable Energy during the year 2009. Globally, biofuels have caught the attention in
last decade and it is imperative to keep up with the pace of developments in the field of biofuels.
Biofuels in India are of strategic importance as it augers well with the ongoing initiatives of the
Government such as Make in India, Swachh Bharat Abhiyan, Skill Development and offers great
opportunity to integrate with the ambitious targets of doubling of Farmers Income, Import
Reduction, Employment Generation, Waste to Wealth Creation. Biofuels programme in India has
been largely impacted due to the sustained and quantum non-availability of domestic feedstock for
biofuel production which needs to be addressed.
56. Ans:(c)
Anil Baijal is a retired Indian Administrative Service officer and is the 21st Lieutenant Governor of
Delhi. He took over office on 31 December 2016 after the sudden resignation of Najeeb Jung.
57. Ans:(a)
Madhya Pradesh, which became the first state in the country to achieve 100 per cent door-to-door
garbage collection in its urban areas in February 2018, won the best-performing state in the
SwachhSurvekshan 2019 which evaluate the performance cities in achieving the cleanliness
standards under the Swachh Bharat Mission.
58. Ans:(b)
With a population of over a billion, there are about 500 million active web users and India‟s online
market is second only to China.
There are three categories of data in the Bill – "Personal data" is defined to mean any
characteristics, trait, attribute or other feature of the identity of a natural person, while "sensitive
personal data" relate to financial data, health data, genetic and biometric data, caste, religious or
political belief or affiliation etc. "Critical personal data" has not been defined and left for the DPAI to
do so.
Personal data is data which pertains to characteristics, traits or attributes of identity, which can be
used to identify an individual. The Bill categorises certain personal data as sensitive personal data.
This includes financial data, biometric data, caste, religious or political beliefs, or any other category
of data specified by the government, in consultation with the Authority and the concerned sectoral
regulator. The 2019 Bill doesn‟t have any provision for a Parliamentary law to be framed for the
purpose and tests of what is "necessary" or "proportionate" to the objectives being sought for
breaching privacy.
According to the Supreme Court in the Puttaswamy judgement (2017), the right to privacy is a
fundamental right and it is necessary to protect personal data as an essential facet of informational
privacy, whereas the growth of the digital economy is also essential to open new vistas of socio-
economic growth.
59. Ans:(b)
With a population of over a billion, there are about 500 million active web users and India‟s online
market is second only to China.
There are three categories of data in the Bill – "Personal data" is defined to mean any
characteristics, trait, attribute or other feature of the identity of a natural person, while "sensitive
personal data" relate to financial data, health data, genetic and biometric data, caste, religious or
political belief or affiliation etc. "Critical personal data" has not been defined and left for the DPAI to
do so.
Personal data is data which pertains to characteristics, traits or attributes of identity, which can be
used to identify an individual. The Bill categorises certain personal data as sensitive personal data.
This includes financial data, biometric data, caste, religious or political beliefs, or any other category
of data specified by the government, in consultation with the Authority and the concerned sectoral
regulator. The 2019 Bill doesn‟t have any provision for a Parliamentary law to be framed for the
purpose and tests of what is "necessary" or "proportionate" to the objectives being sought for
breaching privacy.
According to the Supreme Court in the Puttaswamy judgement (2017), the right to privacy is a
fundamental right and it is necessary to protect personal data as an essential facet of informational
privacy, whereas the growth of the digital economy is also essential to open new vistas of socio-
economic growth.
60. Ans:(b)
With a population of over a billion, there are about 500 million active web users and India‟s online
market is second only to China.
There are three categories of data in the Bill – "Personal data" is defined to mean any
characteristics, trait, attribute or other feature of the identity of a natural person, while "sensitive
personal data" relate to financial data, health data, genetic and biometric data, caste, religious or
political belief or affiliation etc. "Critical personal data" has not been defined and left for the DPAI to
do so.
Personal data is data which pertains to characteristics, traits or attributes of identity, which can be
used to identify an individual. The Bill categorises certain personal data as sensitive personal data.
This includes financial data, biometric data, caste, religious or political beliefs, or any other category
of data specified by the government, in consultation with the Authority and the concerned sectoral
regulator. The 2019 Bill doesn‟t have any provision for a Parliamentary law to be framed for the
purpose and tests of what is "necessary" or "proportionate" to the objectives being sought for
breaching privacy.
According to the Supreme Court in the Puttaswamy judgement (2017), the right to privacy is a
fundamental right and it is necessary to protect personal data as an essential facet of informational
privacy, whereas the growth of the digital economy is also essential to open new vistas of socio-
economic growth.
61. Ans:(d)
With a population of over a billion, there are about 500 million active web users and India‟s online
market is second only to China.
There are three categories of data in the Bill – "Personal data" is defined to mean any
characteristics, trait, attribute or other feature of the identity of a natural person, while "sensitive
personal data" relate to financial data, health data, genetic and biometric data, caste, religious or
political belief or affiliation etc. "Critical personal data" has not been defined and left for the DPAI to
do so.
Personal data is data which pertains to characteristics, traits or attributes of identity, which can be
used to identify an individual. The Bill categorises certain personal data as sensitive personal data.
This includes financial data, biometric data, caste, religious or political beliefs, or any other category
of data specified by the government, in consultation with the Authority and the concerned sectoral
regulator. The 2019 Bill doesn‟t have any provision for a Parliamentary law to be framed for the
purpose and tests of what is "necessary" or "proportionate" to the objectives being sought for
breaching privacy.
According to the Supreme Court in the Puttaswamy judgement (2017), the right to privacy is a
fundamental right and it is necessary to protect personal data as an essential facet of informational
privacy, whereas the growth of the digital economy is also essential to open new vistas of socio-
economic growth.
62. Ans:(a)
With a population of over a billion, there are about 500 million active web users and India‟s online
market is second only to China.
There are three categories of data in the Bill – "Personal data" is defined to mean any
characteristics, trait, attribute or other feature of the identity of a natural person, while "sensitive
personal data" relate to financial data, health data, genetic and biometric data, caste, religious or
political belief or affiliation etc. "Critical personal data" has not been defined and left for the DPAI to
do so.
Personal data is data which pertains to characteristics, traits or attributes of identity, which can be
used to identify an individual. The Bill categorises certain personal data as sensitive personal data.
This includes financial data, biometric data, caste, religious or political beliefs, or any other category
of data specified by the government, in consultation with the Authority and the concerned sectoral
regulator. The 2019 Bill doesn‟t have any provision for a Parliamentary law to be framed for the
purpose and tests of what is "necessary" or "proportionate" to the objectives being sought for
breaching privacy.
According to the Supreme Court in the Puttaswamy judgement (2017), the right to privacy is a
fundamental right and it is necessary to protect personal data as an essential facet of informational
privacy, whereas the growth of the digital economy is also essential to open new vistas of socio-
economic growth.
63. Ans:(b)
With a population of over a billion, there are about 500 million active web users and India‟s online
market is second only to China.
There are three categories of data in the Bill – "Personal data" is defined to mean any
characteristics, trait, attribute or other feature of the identity of a natural person, while "sensitive
personal data" relate to financial data, health data, genetic and biometric data, caste, religious or
political belief or affiliation etc. "Critical personal data" has not been defined and left for the DPAI to
do so.
Personal data is data which pertains to characteristics, traits or attributes of identity, which can be
used to identify an individual. The Bill categorises certain personal data as sensitive personal data.
This includes financial data, biometric data, caste, religious or political beliefs, or any other category
of data specified by the government, in consultation with the Authority and the concerned sectoral
regulator. The 2019 Bill doesn‟t have any provision for a Parliamentary law to be framed for the
purpose and tests of what is "necessary" or "proportionate" to the objectives being sought for
breaching privacy.
According to the Supreme Court in the Puttaswamy judgement (2017), the right to privacy is a
fundamental right and it is necessary to protect personal data as an essential facet of informational
privacy, whereas the growth of the digital economy is also essential to open new vistas of socio-
economic growth.
64. Ans:(d)
PAO S. P. Korolev Rocket and Space Corporation Energia in Russia announced on June 25, 2020
that it is planning on taking the first tourist on a spacewalk in 2023, under the terms of a new
contract with a United States partner „Space Adventures‟.Energia has signed a contract with Space
Adventures. The contact promises to take two space tourists to the International Space Station
(ISS) in 2023.
65. Ans:(b)
On July 25, 1984, Savitskaya became the first woman to spacewalk, conducting EVA outside the
Salyut 7 space station for 3 hours and 35 minutes, during which she cut and welded metals in
space along with her colleague Vladimir Dzhanibekov. Alexei ArkhipovichLeonov (30 May 1934 –
11 October 2019) was a Soviet and Russian cosmonaut, Air Force major general, writer, and artist.
On 18 March 1965, he became the first person to conduct a spacewalk, exiting the capsule during
the Voskhod 2 mission for 12 minutes and 9 seconds.
66. Ans:(c)
The Roscosmos State Corporation for Space Activities, commonly known as Roscosmos, is a state
corporation responsible for the wide range and types of space flights and cosmonautics programs
for the Russian Federation.
67. Ans:(c)
Space Exploration Technologies Corp., trading as SpaceX, is an American aerospace manufacturer
and space transportation services company headquartered in Hawthorne, California. It was founded
in 2002 by Elon Musk with the goal of reducing space transportation costs to enable the
colonization of Mars.
68. Ans:(d)
Dennis Anthony Tito is an American engineer and entrepreneur, most widely known as the first
space tourist to fund his own trip into space. In mid-2001, he spent nearly eight days in orbit as a
crew member of ISS EP-1, a visiting mission to the International Space Station. He is said to have
paid $20m for his eight days in space.

QUANTITATIVE TECHNIQUES

69. Ans:(c)
Number of male artists = 160
Number of female artists = 240

Required answer = 60 + 48 = 108


70. Ans:(b)
Number of male artists = 160
Number of female artists = 240
Required difference = 60 – 36 = 24
71. Ans:(d)
Number of male artists = 160
Number of female artists = 240

Required ratio = 36 : 45 = 4 : 5
72. Ans:(a)
Number of male artists = 160
Number of female artists = 240
Required answer = (20 + 45) + (60 + 96)
73. Ans:(d)
Number of male artists = 160
Number of female artists = 240

Required ratio = 35 : 48

74. Ans:(c)

75. Ans:(d)

76. Ans:(d)
77. Ans:(d)

78. Ans:(c)

79. Ans:(d)
The year 1993 there was an increase of 10% population of AP. And 12% of Bihar
The ratio is = 370000* 110/100 : 500000* 112/100 = 407 : 560
80. Ans:(d)
Total no of woman in AP is = 370000 ×610 = 222000
Total no of woman in HP is = 280000 ×47 = 160000
Required percentage = (222000/160000)*100 = ~126
81. Ans:(b)
The total no of literate under graduates population in Assam = 300000 ×(79×30100) = 70000
82. Ans:(a)
If 70% of the females are literate and 75% of the males are literate
Female illiterate Haryana = (450000*4/9)*(30/100) = 60000
Male illiterate Haryana = (450000*5/9)*(25/100) = 62500
Total no of illiterate population in Haryana = 122500
83. Ans:(c)
the ratio of literates in Assam to the literates in Bihar
= 300000*7/9 : 500000*1/5 = 21 : 9
=7:3
LOGICAL REASONING

84. Ans:(a)
Inference Statement: The positive manner in which the medical profession has been described,
with pure dedication towards improvement of human life, suggests that certain fixed principles and
guidelines are followed by its practitioners. Hence, it can be reasonably inferred that there must be
certain guidelines to ensure „acceptable‟ treatment.
85. Ans:(c)
Weakening Statement: This statement depicts the medical profession in negative light, whereas the
original argument expresses this profession very positively. Hence, this statement weakens the
original argument that medicine as a profession is only for the good of the public.
86. Ans:(d)
None of the Above. This statement is an example of another profession, astrology. However unlike
the original statement which depicts the benefits for the general good as a prerequisite for a true
profession, this statement only states that it is bound by a set of fixed rules (like medicine) and that
it is desired by many, but does not suggest how it works for the public good. Hence, it is not parallel
to the original argument, and neither does it strengthen or weaken the same. It is thus a statement,
irrelevant to the original argument.
87. Ans:(c)
Parallel Reasoning Statement. This statement states that law is an honourable profession as it is
bound by a set of rules and works towards the general good of the people. These are the very
parameters set for medicine being a noble or true profession. Hence, it is Parallel Reasoning
Statement, giving an example following the same logic for defining a noble profession.
88. Ans:(a)
Strengthening Statement. By giving a specific example of how medicine has proved beneficial to
man, this statement strengthens the argument that medicine can be deemed to be a true
profession, as it works for the general good of the people.
89. Ans:(d)
None of the Above. Unlike the original argument where medicine is defined as a true profession
because it works for the general good, acting is depicted as being noble profession because it
depicts reality, but does not state how this depiction is for the general good. It thus does not follow
the logic parallel to the original argument. It also does not strengthen or weaken the original
argument as it does not refer to the medical profession at all. Hence, it is irrelevant to the original
argument.
90. Ans:(d)
Options (a) and (b) are not mentioned in the passage. Option (c) is a general statement based on
the information given in the passage. None of the options explains 'natural selection in action‟
Hence D
91. Ans:(b)
Statement 1 states 'all' but 'many' is mentioned in the passage. So, Statement 1 is incorrect.
Options (a), (c) and (d) contain Statement 1, so we are left with statement 3, which is mentioned in
the passage. So, option (b) is correct.
92. Ans:(d)
It is mentioned in the passage that the ratio of cost to benefit has remained in the favour of
pesticides. None of the options states this. So, option (d) is correct.
93. Ans:(a)
Statement 3 is an incorrect interpretation of what is mentioned in the first paragraph. Statement 1 is
mentioned in the passage as 'pesticides resistance is simply natural selection in action.' Statement
2 is incorrect as instead of 'pest do possess‟ „pest may possess' is there. So, option (a) is correct.
94. Ans:(c)
Statements 1 and 3 are not mentioned in the passage. Statement 2 is justified by the first
paragraph of the passage.
95. Ans:(a)
the last paragraph states that, 'sustainability depends upon continually developing new pesticides
that keep at/east one step ahead of the pests.' So, option (a) is correct.
96. Ans:(d)
From the lines „If the pesticide is applied repeatedly, each successive generation of the pest will
contain a larger portion of resistant individuals‟ and from DDT resistance example, D can be
inferred.
97. Ans:(a)
The last para mentions “Pinckney received only 59 electoral votes, finishing third to Adams and
Jefferson”
98. Ans:(b)
Hamilton planned option (a) (Para 3, sentence 2), (c) (Para 3, sentence 3) and (d) (Para 3,
sentence 2- “attacking Adams for his monarchical tendencies” ). “South Carolina's loyalty to
Thomas Pinckney” prompted Hamilton to devise the plan.
99. Ans:(b)
Penultimate para mentions “deliberately threw away their second votes to men” Therefore, each
voter had two votes to cast
100. Ans:(c)
Logical Structure question. The purpose of referring to the election procedure at that time is to
inform the reader that it was different from that of modern times.

101. Ans:(c)
A series of events of the election of 1796 is explained. Choice (a) is incorrect, because nothing has
been refuted. Choice (b) is incorrect there is no documentation of any thesis. Option (d) is incorrect
because no solution is suggested.
102. Ans:(d)
Application question. The structure of the question and option (d) is similar – instead of utilising
your resources, destroying them so that the opponent is not benefitted.
103. Ans:(a)
Hamilton planned to conquer by dividing the Federalists. Hence (a.). Option (b.) is incorrect
because there was no retreat by Hamilton
104. Ans:(c)
Both in the cases of options (a) and (b) we see, persons who were not great performers
themselves, but were good mentors have been selected. So this is Sakshi‟s reasoning.
Karan‟s reasoning is well-followed in option (c), where we see a good actor but a novice director
Amitabh Bachchan getting picked for the job. Here we see a good performer but an unproven
mentor getting picked. Hence, the answer is (c).
105. Ans:(b)
Option (a) is not correct as Karan has not made this connection that a good player knows how to
handle pressure.
Karan assumes that many current generation players might not know how to handle pressure and
hence, coaches are required to teach the players to tackle pressure. So option (b) is correct.
Karan has made this assumption that all international players have the required skill-set and he has
enumerated that in his argument, so it is not a hidden assumption. So, option (c) is not the answer.
Karan tries to establish the fact that handling pressure is a totally different ball-game than the skill
of playing the sport, but he doesn‟t say that it is much more difficult than the latter. So, option (d) is
incorrect.
Hence option (b) is the answer.
106. Ans:(c)
Sakshi says that coaching is all about knowing the skills of a person about the sport. He need not
have been a good player in his playing days, but the coach needs to know about the tricks and
rules of the game. So, option (c) is correct.
107. Ans:(c)
Sakshi believes that a good player might not necessarily be a good coach, but to be a good coach,
what is necessary is that the person must have in-depth knowledge of the game and good coaching
skills. Hence option (c.) is the perfect conclusion for Sakshi‟s statements.
108. Ans:(c)
Explanations:
There is no requirement for drawing a family tree to solve this question. Since Divya is the
grandmother of Aman and Aman and Ravi are cousins (from the first statement). Ravi should be a
grandson to Divya.
Hence, option (c) is the correct answer.
109. Ans:(a)
Explanations:
A # B indicates A is the mother in the family
B $ C indicates B is the brother of C.
C * D indicates C is the daughter of D.
Since B and C are siblings and A is the mother of B and D is the father of C, D is the father in the
family.
Hence, option (a) is correct.
110. Ans:(c)
Explanation:
7 × 2 - 4 = 10
10 × 2 - 4 = 16
16 × 2 - 4 = 28
28 × 2 - 4 = 52
52 × 2 - 4 = 100
111. Ans:(c)
Explanation:

LEGAL REASONING

112. Ans:(d)
{Explanation- The woman merely staying still doesn‟t amount to resistance or an attempt to arrest.
She simply couldn‟t understand what the officer was saying. Thus, the officer couldn‟t use 46 (2) to
justify his actions.}
113. Ans:(c)
{Explanation- The inspector is liable despite the circumstances in which she made the arrest were
grave. This is because principle provides for no such exception and what she did was contrary to
the mentioned section}
114. Ans:(b)
{Explanation- SDM isn‟t judicial magistrate first class and hence his permission under the
mentioned case is of no use. Option A isn‟t the right answer because passage has remained silent
on whether SDM has ability to grant such permission or not. Hence, b is the best option.}
115. Ans:(b)
{Explanation- 46 (3), irrespective of the fact whether her video was seditious or not, X indeed was
accused of sedition, an offence punishable by death. And thus, police couldn‟t be held liable for her
death under the provisions provided in the passage.}
116. Ans:(c)
{Explanation- The content of video doesn‟t matter to the question. X was accused (sec 46 (4)) of
sedition, an offence punishable by death, and it was for an arrest to be made under the same
offence which resulted in her death.}
117. Ans:(a)
Explanation: Passage says for more than 7 years consent is not required thus in this case consent
was required. Therefore, correct answer is a
118. Ans:(b)
Explanation: According to PDP Bill, consent is required. Therefore, answer would be unchanged.
119. Ans:(d)
Explanation: Passage says. “The DNA bill thus needs to replicate the consent provision in the
PDP”, therefore a can‟t be correct answer b can also not be correct answer because of last
paragraph of passage. Similarly, c can also not be correct answer as passage mentions “.
Involuntary consent, tantamount to coerced consent, seems to be integral to the DNA bill”.
Therefore, correct answer is d
120. Ans:(b)
Explanation: Correct answer is b and it could be interpreted from the lines “The DNA bill and the
PDP bill have to be read concomitantly and harmonized in a manner which would ensure robust
and effective privacy protection for DNA information.”
121. Ans:(a)
Explanation: Offences with punishment of more than seven years of imprisonment or death penalty
does not require consent thus correct answer option is a
122. Ans:(a)
Explanation: According to PDP bill, consent is required every time when person is asked for DNA.
Thus, police is not right and thus answer would be a.
123. Ans:(a)
Ex: This is a case of trespass. The tangible object involved is the dried and felling tree leaves that
interfere with the physical possession of Abhilaksh‟s land and he cannot maintain his garden
therefore.
124. Ans:(b)
Ex: This is a case of nuisance for the bad smell is not a tangible object which is interfering with the
right of the plaintiff. This is an intangible object interfering with the accessory rights of the plaintiff.
125. Ans:(a)
Ex: The passage states that ordinarily only 1 public nuisance suit is permitted. However, if a plaintiff
suffers more than what the general public is suffering then he is allowed to also file a private action
for nuisance. Since Shuk was allergic and developed skin rashes, he shall be allowed to file a
private action.
126. Ans:(d)
Ex: The passage states that for private action to be filed in addition to a public nuisance action, the
injury must be independent and not merely consequential. Shuk‟s injury however is only
consequential to the injury that is being caused to the general public. The private action thus cannot
be allowed.
127. Ans:(d)
(Explanation: They are the consumers not because they are parties to the contract of insurance but
because they are the beneficiaries of the policy taken out by the insured. Moreover, they have the
implicit approval of Sunshine Freezers)
128. Ans:(d)
(Explanation: The passage talks about the approval of the person who buys a good or a
consideration on behalf of the consumer for availing the services.)
129. Ans:(d)
(Explanation: Smita is a consumer under the meaning of the Act. Even though she is not availing
the services but she bought the washing machine for the purpose of gifting)
130. Ans:(a)
(Explanation: The laptop is being used by the person. He is availing the services. The approval of
the person who may have bought the laptop is implicit as it is a scholarship meant for the scholar.
All the other options lack in the sense that either there is no approval from the original owner or
there is no consideration for the use of the services)
131. Ans:(c)
Explanation: The answer is (c). This is due to this reason that the Act came into force after the act
was committed. Therefore, past acts cannot be criminalized with retrospective effect making
Chitamann not guilty for the said act.
132. Ans:(a)
Explanation: The answer is (a). This is simply because Article 20 provides that any increased
punishment for the same act for which the punishment was different at the time of the commission
of the act are not constitutional in nature.
133. Ans:(b)
Explanation: The answer is (b). This is because the timeline of 6 months to fine the tax evaders is a
reasonable period. In addition to this, the fundamental right cannot extend to imposition of fines
when it is in the interest of public at large who is diligently paying all the taxes. Therefore, the
evaders have to be fined heavily to prevent them from avoiding the payment of taxes
134. Ans:(d)
Explanation: The answer is (d). This is because retrospective application is not justified in case of
increasing the punishment but it can be justified in reducing the punishment as it caters to benefit
the individual which can have a retrospective affect.
135. Ans:(a)
Explanation: The answer is (a). This is because offence of disproportionate assets is different from
the actual calculation of disproportionate assets. The prohibition is only on criminalizing from an act
which is not done in this case. Thus, there is no violation of Article 20.
136. Ans:(c)
Explanation: (reason: Under section 39 of the Motor Vehicle Act, 1988, no unregistered vehicle can
be driven in public places.)
137. Ans:(a)
Explanation: (reason: because Karan still had a month left to approach the registration authority for
assignment of new registration mark within whose jurisdiction the vehicle is. The cop didn‟t consider
this law and was therefore penalized.)
138. Ans:(d)
Explanation: (In case of joint ownership, the registration of vehicle can be applied by one of the
owners.)
139. Ans:(a)
Explanation: (Under section 48 of the motor vehicles act of 1988, the registering authority shall
furnish the applicant whose vehicle is refused registration, or whose application for renewal of the
certificate of registration is refused, a copy of such order, together with the reasons for such
refusal.)
140. Ans:(b)
Explanation: (by the law, Amit should have notified the concerned authority about the address
change within 30 days.)
141. Ans:(c)
Explanation: (as mentioned in the passage, the central motor vehicle rules, 1989, were set up to
allow exercising all the provisions of Motor vehicle act 1988, including registration of vehicles.)
142. Ans:(c)
Explanation: The answer is (c). This is because the conduct, language and expression of Sashi
were derogatory in nature forcing Ambika to commit suicide. Rajat would not be liable as he did not
take any active steps which prove that he had aided the result in the instant case.
143. Ans:(b)
Explanation: The answer is (b). This is because the offense of abetment by instigation relies on the
intention of the individual who abets and not upon the act which is finished by the individual who
has abetted. Therefore, Amit is liable for abetting Akhila.
144. Ans:(c)
Explanation: The answer is (c). This is because the act of abetment is not dependent on the end
result or the impact. It relies on the intention of the instigator and their act. Whether the abetment
has reached to its desired end is not a matter in question.
145. Ans:(c)
Explanation: The answer is (c). This is because abetment being an offence, the abetment of such
an offence is also an offence and would amount to abetment making Aditya liable for abetment of
abetment in the existing case.
146. Ans:(c)
Explanation: The answer is (c). This is because the words articulated in an angry state do not
possess the mens rea which is an essential condition for an offence to be categorized as abetment.
Therefore, such an utterance without intention does not amount to abetment.
147. Ans:(d)
Explanation: The answer is (d). This is because the rules based on the DPSP provide for the
minimum number of clients and not the maximum number for taking up cases of clients with
financial incapacity. Therefore, Mr. Tripathi is bound to accept the case.
148. Ans:(b)
Explanation: The answer is (b). This is because DPSP serve as a guideline for the government
policy formation and does not create a mandate on the part of the State unless it is made
enforceable through legislation. Therefore, in the absence of the mandate the State is exempted in
the instant case.
149. Ans:(d)
Explanation: The answer is (d). Heena could have secured employment on compassionate grounds
if she had met the minimum qualifications. Therefore, in that case enforceability of DPSP would
have taken a different shape as it derives the enforceability from the fundamental rights and welfare
State which are enforceable in nature. So, enforceability is the most relevant factor but with the
existing facts it cannot be made enforceable.
150. Ans:(c)
Explanation: The answer is (c). This is simply because it covers wide range social and economic
aspects inspired from preamble and not only socialist factors. Self –Explanatory.
Legal Edge 03 Mock

Answers & Details

ENGLISH

1. Ans: (d) (a) and (b) only. The words for example at the beginning of the sentence containing the
crowd analogy follow a sentence about the Pauli exclusion principle. This principle says that
fermions cannot inhabit the same fundamental state. Electrons, which are fermions, are likened to
members of a crowd in a stadium; the fact that electrons cannot circle the nuclei of atoms in
precisely the same orbits (just as crowd members cannot sit on top of one another) is a
―consequence‖ of the Pauli exclusion principle. Thus, the first statement is justified. These electrons
must occupy more and more distant locations; the crowd analogy certainly illustrates that behavior,
so the second statement is justified. As for the third, incorrect statement, while you do know that
electrons cannot occupy the same orbits as one another and must instead occupy more and more
distant locations, you do not know that those orbits are ―concentric‖ or ―evenly-spaced.‖

2. Ans: (b) The author begins by naming the two classes of subatomic particles, and then divides
the remainder of the passage into descriptions of each class and their relation to each other.
Regarding (a) and (d), the author explains both of these concepts within the passage, but it is
subordinate to the main idea of describing the two types of subatomic particles, and thus is not the
primary purpose of the passage. As for (c), the author‘s primary purpose in writing is not to provide
examples or to argue

3. Ans: (c) The author states that fermions, not bosons, are the constituents of ordinary matter. All
other answers are mentioned in the second paragraph of the passage.

4. Ans: (d) The passage states that Cooper pairs of electrons will flow in perfect harmony and with
zero resistance through the metal. As an example of the same phenomenon, you are told that a
swirl in a cup of superfluid helium will never dissipate. Therefore, it is correct to infer that a current
through a superconducting wire will never dissipate. If you were not certain that you could be sure
of this, you can still eliminate the other choices through process of elimination. As for (a), the
passage states that an even number of fermions (which, according to the first paragraph, include
electrons, protons, and neutrons) constitute a boson, but not an odd number (1/2 integer times an
odd will not give an integer). The last paragraph states that scientists argue for the existence of
skyrmions in a medium that might permit them to be formed, implying that they have not yet been
discovered, so eliminate (b). In (c), the author states that two electrons cannot circle a nucleus in
the same orbit, but they could spin in different orbits that are the same distance from the nucleus.

5. Ans: (a) (d) is the opposite of what the passage claims. Bosons have integral spin values and
fermions have half-integer spin. Answer (b) is incorrect because the passage does not discuss the
total number of particle types for bosons for fermions and answer (c) is incorrect because the
passage explicitly states that both fermions and bosons can exist in groups.

6. Ans: (a) The passage states that the Pauli principle prohibits any two particles from inhabiting
the same fundamental state. Further, you know that the Pauli principle should be applied to
fermions, which include electrons, protons, and neutrons, but not bosons (from the second
paragraph). Answer choice (a) discusses electrons, which are fermions, avoiding occupation of
identical energy levels, so (a) is relevant and thus the correct answer. As for (b), a charged particle
in a magnetic field neither provides the criteria for a fermion nor references inhabitance of the same
state.

7. Ans: (d) (a) and (c) only. According to the second paragraph, not knowing that they are seeing a
theater piece allows viewers to avoid the etiquette of theatergoing and engage with the action and
concepts of an unfolding drama asif these actions and concepts were real. This is a good match
with the first statement. The third paragraph refers to ―scripted characters‖ in invisible theater, so
you cannot infer that invisible theater is best described as improvised.(It may be possible in real life
for scripted characters to engage in improvised acting, but you cannot infer this from the passage.)
Another reference to dialogue … set up by invisible theater performers weighs against invisible
theater being improvisational. The third paragraph begins, Boal has documented various successful
instances of invisible theater in which non-performers … take unplanned public-minded action in
response to the dialogue and events set up by invisible theater performers. The last sentence of the
passage also states that …the goal of guerilla theater is to get people talking publicly. Thus, you
can infer that actions taken by the audience once the performance is over can be one measure of
success of a theater piece, and thus the third statement is true.

8. Ans: (a) In the first sentence, the author calls invisible theater and guerrilla theater two forms of
street theater with similar origins but very different approaches. This is a good match with (a). (b) is
incorrect because the passage doesn‘t say which form is more effective. (c) is too broad and the
evolution is not the focus. (d) fails because the passage states clearly — and describes in great
detail — the fact that ―invisible theater conceals its performative nature whereas guerrilla theater
flaunts it.‖

9. Ans: (d) (a) and (b) only. The first statement is true, as the third paragraph goes into great detail
about invisible theater‘s goal of encouraging public-minded talk, and the passage ends with ―the
goal of guerrilla theater is to get people talking publicly.‖ The second statement is justified because
you are told in the first paragraph that Both forms take exclusively in public places. The third choice
is only explicitly stated in regard to guerilla theater; invisible theater only said to involve the
audience

10. Ans: (c) The professors gave diplomas to the people who were the least able to answer
questions as a way to mock the university‘s decision. (a) does not have to be true because the
passage merely discusses the form of the professors‘ protest. (b) cannot be justified since it doesn‘t
have to be true that the professors believed that result would follow. Similarly, (d) is wrong because
it is not certain that the professors considered the legality of their actions

11. Ans: (d) I and III only. The first statement, if true, would contradict the assumption of invisible
theater that removing the boundary between performer and audience encourages involvement. The
second statement describes a very likely result, and a goal, of invisible theater, and thus would not
―undermine‖ the principle of invisible theater. The third statement describes a situation where the
goal of invisible theater — a lively debate about public issues — was happening, and invisible
theater ruined it! This would definitely undermine the principle of invisible theater.

12. Ans: (c)The tone of the passage is enthusiastic in its recommendation of the greyhound as pet
and, thereby, encourages people to adopt one. It does not give advice on transforming a greyhound
(choice a). Except to say that they love to run, the passage does not spend equal time on
describing the greyhound as racer (choice b). The author‘s tone is not objective (choice d), but
rather enthusiastic

13. Ans: (d.) See the last paragraph. The passage does not mention b or c. Choice a is clearly
wrong; the passage states the opposite

14. Ans: (a) See the first paragraph. Choices b, c, and d are not touched on in the passage

15. Ans: (d.) See the last paragraph. Choices a, b, and c are contradicted in the passage.

16. Ans: (d)


The enthusiastic tone of the passage seems meant to encourage people to adopt retired
greyhounds. Choice a is wrong because there is only one statistic in the passage (in the first
sentence), and it is not used to prove the point that greyhounds make good pets. Choice b is wrong
because the author substantiates every point with information. Choice c is wrong because the
passage does make the negative point that greyhounds do not make good watchdogs.
17. Ans: (d) The author has used many familiar images of a ruler, rail track, sea surface, airplane
and moving airplanes between any two locations to explain a difficult topic, related with core
scientific topic

18. Ans: (a) The words 'What distinguishes a continuum is the fact that the space between any two
points can be sub-divided into an infinite number of smaller divisions' indicate that one of the
significant features of continuum is the divisibility of the interval between any two points

19. Ans: (c) The whole passage revolves around the concept of dimensions and it starts as well as
ends with the mention of four-dimensions. The last sentences of the passage clearly indicate that
the fourth dimension is time

20. Ans: (d) The author is not trying to persuade the readers that the fourth dimension is time. He
is in fact putting up various evidences and logical proofs to show that the fourth dimension is time.
So, his mode is instructive. Hence, the answer is (d). He is not mentioning any information in
differential or candid way. So, options (b) and (c) can be ruled out

21. Ans: (b) The sentence 'Just indicating its position in space is not enough while describing any
physical event, which involves motion. How position changes in time also needs to be mentioned'
show that while portraying any physical event in which motion plays a role, one has to indicate how
position changes with time

22. Ans: (a) The author has mentioned that the sea surface is a two-dimensional continuum, which
means that this example is used to help the readers in understanding the concept of two-
dimensional continuum.

23. Ans: (c) The authors do not include massive stars in their list of possible explanations for
missing matter.

24. Ans: (c) An inference is drawn from the stated information. This question refers to the first
paragraph, where the authors explain that the critical density of matter needed to break the
expansion and close the universe is equivalent to three hydrogen atoms per cubic meter. If the
density is significantly less, then the universe will not 'close' but continue to expand indefinitely. So,
the correct answer is Statement III

25. Ans: (c) The authors did not change ideas about luminosity. However, the authors changed
their ideas about both rotational velocity and the distribution of matter. So, the correct answer is
option (c)

26. Ans: (b) Figuring out the author's primary concern depends on a careful review of the passage
as a whole. The author's primary purpose in this passage is to summarise the findings of their
research

27. Ans: (a) The region has similar rotational velocity and higher luminosity.

28. Ans: (d) (a), (b), and (c). You are told in this passage that widespread genomic changes would
wreak physiological havoc, such as cancer (statement (a)). You are also told in the second
paragraph that many organisms have also beneficial mechanisms to induce genetic change. In
short, some genetic changes are bad, but others are beneficial.
The third paragraph explains how genetic change is important to immune functioning (statement II),
and the last line of the passage tells you ―this process is regulated by T cells to prevent harmful
mutations,‖ so the third statement is also justified.

29. Ans: (c) The word seemingly indicates that the changes are not really haphazard. ―Seemingly
haphazard‖ refers to ―programmed genetic mutation.‖ The ―this‖ in this seemingly haphazard
process of programmed genetic mutation harks back to ―genetic recombination,‖ so you can be
sure that (c) is a match. Some of the other answers may be true— (d) certainly is — but do not
answer the question

30. Ans: (d) (a), (b), and (c). Pro-death signaling is given in a list of cellular mechanisms that
stymie genetic changes. Stymie means ―hinder.‖ Statement (a) is true. These cellular mechanisms
are called ubiquitous, which means ―existing everywhere,‖ so certainly they are ―very common.‖
Statement II is also true. In the final sentence of the paragraph, you are told, malfunctions in
molecular players that safeguard against mutagenesis, such as the protein p53, have been
implicated in diseases such as cancer. Since the malfunction of p53 may cause cancer, you can
infer that p53,when properly functioning, may work against cancer. That means Statement III is also
true.

31. Ans: (a) You are told that T cells need a large repertoire of receptors in order to be able to
recognize a wide variety of pathogens. Then: Relying only on a genetically encoded repertoire
would be disadvantageously limiting—analogous to having only a few dozen language phrases with
which to respond to the nearly infinite potential combinations of words in a conversation. Instead,
the repertoire is generated by a process of genetic recombination… In this analogy, the language
phrases are the repertoire of receptors; just as a speaker must respond to a nearly infinite body of
language combinations, T cells must also have a large repertoire so they can respond to a wide
variety of pathogens. You are told that the way this repertoire is increased is through genetic
recombination.
Note that (b) is out of scope, (c) is the exact opposite of what is being described, (d) is a distortion
based on another analogy in the passage
32. Ans: (a) (a) only. In the analogy referenced, the ―language phrases‖ are receptors that can
respond to various pathogens. The nearly infinite potential combinations of words is what a speaker
must respond to — the reason a speaker needs a wide repertoire of language. Similarly, the wide
variety of pathogens is the reason T cells need such a wide variety of receptors.

GENERAL KNOWLEDGE/CURRENT AFFAIRS

33. Ans: (b) is the correct answer.


The National Intelligence Grid (NATGRID) has signed a memorandum of understanding with the
National Crime Records Bureau (NCRB) to access the centralised online database on FIRs and
stolen vehicles. First conceptualised in 2009, NATGRID seeks to become the one-stop destination
for security and intelligence agencies to access database related to immigration entry and exit,
banking and telephone details of a suspect on a ―secured platform‖.
The current Director of NCRB is Ramphal Pawar (IPS). NCRB was set-up in 1986 to function as a
repository of information on crime and criminals so as to assist the investigators in linking crime to
the perpetrators.
NATGRID functions under the Home Ministry.
NCRB is headquartered in New Delhi and is part of the Ministry of Home Affairs (MHA),
Government of India. The current Director of NCRB is Ramphal Pawar (IPS). Samant Goel is the
current chief of RAW. Raghu Raman was the founding CEO of NATGRID.
34. Ans: (a) is the correct answer.
The National Intelligence Grid (NATGRID) has signed a memorandum of understanding with the
National Crime Records Bureau (NCRB) to access the centralised online database on FIRs and
stolen vehicles. First conceptualised in 2009, NATGRID seeks to become the one-stop destination
for security and intelligence agencies to access database related to immigration entry and exit,
banking and telephone details of a suspect on a ―secured platform‖.
The current Director of NCRB is Ramphal Pawar (IPS). NCRB was set-up in 1986 to function as a
repository of information on crime and criminals so as to assist the investigators in linking crime to
the perpetrators.
NATGRID functions under the Home Ministry.
NCRB is headquartered in New Delhi and is part of the Ministry of Home Affairs (MHA),
Government of India. The current Director of NCRB is Ramphal Pawar (IPS). Samant Goel is the
current chief of RAW. Raghu Raman was the founding CEO of NATGRID.
35. Ans: (b) is the correct answer.
The National Intelligence Grid (NATGRID) has signed a memorandum of understanding with the
National Crime Records Bureau (NCRB) to access the centralised online database on FIRs and
stolen vehicles. First conceptualised in 2009, NATGRID seeks to become the one-stop destination
for security and intelligence agencies to access database related to immigration entry and exit,
banking and telephone details of a suspect on a ―secured platform‖.
The current Director of NCRB is Ramphal Pawar (IPS). NCRB was set-up in 1986 to function as a
repository of information on crime and criminals so as to assist the investigators in linking crime to
the perpetrators.
NATGRID functions under the Home Ministry.
NCRB is headquartered in New Delhi and is part of the Ministry of Home Affairs (MHA),
Government of India. The current Director of NCRB is Ramphal Pawar (IPS). Samant Goel is the
current chief of RAW. Raghu Raman was the founding CEO of NATGRID.
36. Ans: (a) is the correct answer.
The National Intelligence Grid (NATGRID) has signed a memorandum of understanding with the
National Crime Records Bureau (NCRB) to access the centralised online database on FIRs and
stolen vehicles. First conceptualised in 2009, NATGRID seeks to become the one-stop destination
for security and intelligence agencies to access database related to immigration entry and exit,
banking and telephone details of a suspect on a ―secured platform‖.
The current Director of NCRB is Ramphal Pawar (IPS). NCRB was set-up in 1986 to function as a
repository of information on crime and criminals so as to assist the investigators in linking crime to
the perpetrators.
NATGRID functions under the Home Ministry.
NCRB is headquartered in New Delhi and is part of the Ministry of Home Affairs (MHA),
Government of India. The current Director of NCRB is Ramphal Pawar (IPS). Samant Goel is the
current chief of RAW. Raghu Raman was the founding CEO of NATGRID.
37. Ans: (a) is the correct answer.
The National Intelligence Grid (NATGRID) has signed a memorandum of understanding with the
National Crime Records Bureau (NCRB) to access the centralised online database on FIRs and
stolen vehicles. First conceptualised in 2009, NATGRID seeks to become the one-stop destination
for security and intelligence agencies to access database related to immigration entry and exit,
banking and telephone details of a suspect on a ―secured platform‖.
The current Director of NCRB is Ramphal Pawar (IPS). NCRB was set-up in 1986 to function as a
repository of information on crime and criminals so as to assist the investigators in linking crime to
the perpetrators.
NATGRID functions under the Home Ministry.
NCRB is headquartered in New Delhi and is part of the Ministry of Home Affairs (MHA),
Government of India. The current Director of NCRB is Ramphal Pawar (IPS). Samant Goel is the
current chief of RAW. Raghu Raman was the founding CEO of NATGRID.
38. Ans: Ans- c)
Reasoning: US president Donald Trump chose Mount Rushmore for his Independence Day speech
this year on 4th July.
39. Ans: Ans- c)
Reasoning: Mount Rushmore National Memorial is centered on a colossal sculpture carved into the
granite face of Mount Rushmore in the Black Hills in Keystone, South Dakota.
40. Ans: Ans- d)
Reasoning: It features 60-foot face carvings of four US Presidents George Washington, Thomas
Jefferson, Theodore Roosevelt and Abraham Lincoln.
41. Ans: Ans- a)
Reasoning: The idea to carve the granite face of Black Hills was conceived by historian Doane
Robinson in 1923, who thought it could promote tourism in the region.
42. Ans- d)
Reasoning: Trump's plans to kick off Independence Day with a showy display at Mount Rushmore
are drawing sharp criticism from Native Americans who view the monument as a desecration of
land violently stolen from them and used to pay homage to leaders hostile to native people.
43. Ans: (c) India has become the endorsed candidate from the Asia-Pacific States. The other
regional groups from which members are elected are African group, Eastern European Group, Latin
American and Carribean Group (GRULAC), Western European and Others Group (WEOG).

44. Ans: (c) Along with India, Ireland, Mexico and Norway also won the Security Council elections
held on Wednesday.

45. Ans: (a) Djibouti and Kenya vied for one seat allocated to the African and Asia-Pacific States
grouping and will head into the second round of voting on Thursday as each failed to secure the
required two-thirds majority, or 128 votes, to win a seat. Kenya won the seat finally.

46. Ans: (a) There are currently 193 UN Member States. Each of the Member States of the United
Nations has one seat in the General Assembly. South Sudan was the recent and became the 193rd
member of the United Nations in 2011.

47. Ans: (d) Apart from the Security Council elections, the General Assembly voted to elect veteran
Turkish diplomat and parliamentarian Volkan Bozkir as the President of the next session of the UN
General Assembly. He will preside over the landmark 75th session of the General Assembly, which
opens in September.

48. Ans: (d) Previously, India has been elected as a non-permanent member of the Council for the
years 1950-1951, 1967-1968, 1972-1973, 1977-1978, 1984-1985, 1991-1992 and most recently in
2011-2012.

49. Ans – b
The Foreign Investment Promotion Board (FIPB) was a national agency of Government of India,
with the remit to consider and recommend foreign direct investment (FDI) which does not come
under the automatic route. The Foreign Investment Promotion Board (FIPB) was housed in the
Department of Economic Affairs, Ministry of Finance. FIPB was abolished on 24 May 2017, as
announced by Finance Minister Arun Jaitley during 2017-2018 budget speech in Lok Sabha.
50. Ans: b
FDI is an important monetary source for India's economic development. Economic liberalisation
started in India in the wake of the 1991 crisis and since then, FDI has steadily increased in the
country. India, today is a part of top 100-club on Ease of Doing Business (EoDB) and globally ranks
number 1 in the greenfield FDI ranking.
51. Ans: b
FDI prohibition
There are a few industries where FDI is strictly prohibited under any route. These industries are
• Atomic Energy Generation
• Any Gambling or Betting businesses
• Lotteries (online, private, government, etc)
• Investment in Chit Funds
• Nidhi Company
• Agricultural or Plantation Activities (although there are many exceptions like horticulture, fisheries,
tea plantations, Pisciculture, animal husbandry, etc)
• Housing and Real Estate (except townships, commercial projects, etc)
• Trading in TDR‘s
• Cigars, Cigarettes, or any related tobacco industry
52. Ans:– b
2 routes are defined by the government: Automatic and government route. In first one no prior
approval of government or RBI is needed. In the latter one, it is needed. It is basically for the
security reasons.
53. Ans:– b
India recently revised its Foreign Direct Investment (FDI) policy with the objective of preventing
―opportunistic takeovers‖ of firms hit by the lockdown induced by the COVID-19 outbreak.
54. Ans: (b) is the correct answer.
The President of Turkey Recep Tayyip Erdogan has recently announced that the iconic
architectural place of the country Hagia Sophia would be made a mosque, which gained criticism
from across the world.After the top Turkish court revoked the museum status of the monument, the
President made this announcement.
Hagia Sophia is the former Greek Orthodox Christian patriarchal cathedral, later an Ottoman
imperial mosque and now a museum located in Istanbul, Turkey.
Built in AD 537 (Byzantine architecture), during the reign of Justinian 1, the Eastern Roman
emperor, it is famous for its large dome. In 1453, when Constantinople (capital city of the Roman
Empire) fell to Sultan Mehmet II‘s Ottoman forces, the Hagia Sophia was turned into a mosque. For
a long time, the Hagia Sophia was Istanbul‘s most important mosque. In 1934, Mustafa Kemal
Ataturk, the founder of the Republic of Turkey, converted the mosque into a museum in an attempt
to make the country more secular.
55. Ans: (a)is the correct answer.
The President of Turkey Recep Tayyip Erdogan has recently announced that the iconic
architectural place of the country Hagia Sophia would be made a mosque, which gained criticism
from across the world.After the top Turkish court revoked the museum status of the monument, the
President made this announcement.
Hagia Sophia is the former Greek Orthodox Christian patriarchal cathedral, later an Ottoman
imperial mosque and now a museum located in Istanbul, Turkey.
Built in AD 537 (Byzantine architecture), during the reign of Justinian 1, the Eastern Roman
emperor, it is famous for its large dome. In 1453, when Constantinople (capital city of the Roman
Empire) fell to Sultan Mehmet II‘s Ottoman forces, the Hagia Sophia was turned into a mosque. For
a long time, the Hagia Sophia was Istanbul‘s most important mosque. In 1934, Mustafa Kemal
Ataturk, the founder of the Republic of Turkey, converted the mosque into a museum in an attempt
to make the country more secular.
56. Ans: (a)is the correct answer.
The President of Turkey Recep Tayyip Erdogan has recently announced that the iconic
architectural place of the country Hagia Sophia would be made a mosque, which gained criticism
from across the world.After the top Turkish court revoked the museum status of the monument, the
President made this announcement.
Hagia Sophia is the former Greek Orthodox Christian patriarchal cathedral, later an Ottoman
imperial mosque and now a museum located in Istanbul, Turkey.
Built in AD 537 (Byzantine architecture), during the reign of Justinian 1, the Eastern Roman
emperor, it is famous for its large dome. In 1453, when Constantinople (capital city of the Roman
Empire) fell to Sultan Mehmet II‘s Ottoman forces, the Hagia Sophia was turned into a mosque. For
a long time, the Hagia Sophia was Istanbul‘s most important mosque. In 1934, Mustafa Kemal
Ataturk, the founder of the Republic of Turkey, converted the mosque into a museum in an attempt
to make the country more secular.
57. Ans: (b)is the correct answer.
The President of Turkey Recep Tayyip Erdogan has recently announced that the iconic
architectural place of the country Hagia Sophia would be made a mosque, which gained criticism
from across the world.After the top Turkish court revoked the museum status of the monument, the
President made this announcement.
Hagia Sophia is the former Greek Orthodox Christian patriarchal cathedral, later an Ottoman
imperial mosque and now a museum located in Istanbul, Turkey.
Built in AD 537 (Byzantine architecture), during the reign of Justinian 1, the Eastern Roman
emperor, it is famous for its large dome. In 1453, when Constantinople (capital city of the Roman
Empire) fell to Sultan Mehmet II‘s Ottoman forces, the Hagia Sophia was turned into a mosque. For
a long time, the Hagia Sophia was Istanbul‘s most important mosque. In 1934, Mustafa Kemal
Ataturk, the founder of the Republic of Turkey, converted the mosque into a museum in an attempt
to make the country more secular
58. Ans: (d)is the correct answer.
The President of Turkey Recep Tayyip Erdogan has recently announced that the iconic
architectural place of the country Hagia Sophia would be made a mosque, which gained criticism
from across the world.After the top Turkish court revoked the museum status of the monument, the
President made this announcement.
Hagia Sophia is the former Greek Orthodox Christian patriarchal cathedral, later an Ottoman
imperial mosque and now a museum located in Istanbul, Turkey.
Built in AD 537 (Byzantine architecture), during the reign of Justinian 1, the Eastern Roman
emperor, it is famous for its large dome. In 1453, when Constantinople (capital city of the Roman
Empire) fell to Sultan Mehmet II‘s Ottoman forces, the Hagia Sophia was turned into a mosque. For
a long time, the Hagia Sophia was Istanbul‘s most important mosque. In 1934, Mustafa Kemal
Ataturk, the founder of the Republic of Turkey, converted the mosque into a museum in an attempt
to make the country more secular.
59. Ans: d
Triple talaq, also known as talaq-e-biddat, instant divorce and talaq-e-mughallazah (irrevocable
divorce), was a form of Islamic divorce which has been used by Muslims in India, especially
adherents of Hanafi Sunni Islamic schools of jurisprudence. It allowed any Muslim man to legally
divorce his wife by uttering the word talaq (the Arabic word for "divorce") three times in oral, written
or, more recently, electronic form.
The use and status of triple talaq in India has been a subject of controversy and debate. Those
questioning the practice have raised issues of justice, gender equality, human rights and
secularism. The debate has involved the Government of India and the Supreme Court of India, and
is connected to the debate about a uniform civil code (Article 44) in India. On 22 August 2017, the
Indian Supreme Court deemed instant triple talaq (talaq-e-biddah) unconstitutional.
60. Ans: - d
On 22nd August 2017, the 5 Judge Bench of the Supreme Court pronounced its decision in the
Triple Talaq Case, declaring that the practise was unconstitutional by a 3:2 majority.
Majority: Rohinton Nariman J. and U.U. Lalit J.
Concurring: Kurian Joseph J.
Dissenting: CJI J.S. Khehar and Abdul Nazeer J.
Shayara Bano was married to Rizwan Ahmed for 15 years. In 2016, he divorced her through
instantaneous triple talaq (talaq -e biddat). She filed a Writ Petition in the Supreme Court asking it
to hold three practices – talaq-e-biddat, polygamy, nikah-halala – unconstitutional as they violate
Articles 14, 15, 21, 25 of the Constitution.
Talaq-e- bidat is a practise which gives a man the right to divorce his wife by uttering ‗talaq‘ three
times in one sitting without his wife‘s consent. Nikah Halala is a practise where a divorced woman
who wants to remarry her husband would have to marry and obtain a divorce, from a second
husband before she can go back to her first husband. And polygamy is a practice which allows
Muslim men to have more than one wife.
61. Ans - b
The Muslim Women (Protection of Rights on Marriage) Act, 2019 is an Act of the Parliament of
India criminalising triple talaq. In August 2019 the Supreme Court of India declared triple talaq,
which enables Muslim men to instantly divorce their wives, to be unconstitutional. The minority
opinion suggested the Parliament to consider appropriate legislation governing triple talaq in the
Muslim community.
In December 2017, citing the Supreme Court judgment and cases of triple talaq in India, the
government introduced The Muslim Women (Protection of Rights on Marriage) Bill, 2017. The bill
proposed to make triple talaq in any form—spoken, in writing, or by electronic means—illegal and
void. Punishment for breach of the law was proposed to include up to three years imprisonment for
the husband pronouncing triple talaq. The bill was passed by the Lok Sabha, the lower house of
the Parliament of India, on the same day, but was stalled by the opposition in the Rajya Sabha, the
upper house.
The bill was reintroduced and passed by the Lok Sabha and by the Rajya Sabha in July 2019.
Consequently, the bill received assent of the President of India. The act also entitles an aggrieved
woman to demand a maintenance for her dependent children[. It was subsequently notified as law
in the same month. The acts stands to be retrospectively effective from 19 September 2018.[
62. Ans: d
On 22nd August 2017, the 5 Judge Bench of the Supreme Court pronounced its decision in the
Triple Talaq Case, declaring that the practise was unconstitutional by a 3:2 majority.
Majority: Rohinton Nariman J. and U.U. Lalit J.
Concurring: Kurian Joseph J.
Dissenting: CJI J.S. Khehar and Abdul Nazeer J.
Shayara Bano was married to Rizwan Ahmed for 15 years. In 2016, he divorced her through
instantaneous triple talaq (talaq -e biddat). She filed a Writ Petition in the Supreme Court asking it
to hold three practices – talaq-e-biddat, polygamy, nikah-halala – unconstitutional as they violate
Articles 14, 15, 21, 25 of the Constitution.
Talaq-e- bidat is a practise which gives a man the right to divorce his wife by uttering ‗talaq‘ three
times in one sitting without his wife‘s consent. Nikah Halala is a practise where a divorced woman
who wants to remarry her husband would have to marry and obtain a divorce, from a second
husband before she can go back to her first husband. And polygamy is a practice which allows
Muslim men to have more than one wife.
63. Ans: a
Under Article 25 of the Constitution the state cannot take away the essential religious practice of a
person. Therefore, if a practice which is arbitrary and not an essential religious practice it will be hit
by the exception laid down u/a 25. Therefore, the whole issue was whether or not the practice is an
essential religious practice of Islam.
Therefore, as per majority it was held that the Triple Talaqor Talaq-e-biddatis not protected by the
exception laid down in Article 25 i.e. the court found the said practice not an essential element of
Islamic religion. The court justified its point of view in the sense that although it si practiced by the
Hanafi School but it is considered sinful in it.
64. Ans: (a) is the correct answer.
The Ministry of Shipping has issued the draft of Aids to Navigation Bill, 2020 for suggestions from
the stakeholders and the general public. he draft bill is proposed to replace the almost nine-decade
old, Lighthouse Act, 1927 to incorporate the global best practices, technological developments and
India's International obligations in the field of Aids to Marine Navigation.
Mansukh Mandaviya is the Minister of State (Independent Charge) for Ministry of Shipping and
Minister of State for Chemical & Fertilizers in Government of India.
The Directorate General of Lighthouses and Lightships is a subordinate office under the Ministry of
Shipping. Its headquarters is in Noida, Uttar Pradesh.
Shri. Ellappan Murthy took over charge as Director General of Lighthouses and Lightships, Ministry
of Shipping.
65. Ans: (c) is the correct answer.
The Ministry of Shipping has issued the draft of Aids to Navigation Bill, 2020 for suggestions from
the stakeholders and the general public. he draft bill is proposed to replace the almost nine-decade
old, Lighthouse Act, 1927 to incorporate the global best practices, technological developments and
India's International obligations in the field of Aids to Marine Navigation.
Mansukh Mandaviya is the Minister of State (Independent Charge) for Ministry of Shipping and
Minister of State for Chemical & Fertilizers in Government of India.
The Directorate General of Lighthouses and Lightships is a subordinate office under the Ministry of
Shipping. Its headquarters is in Noida, Uttar Pradesh.
Shri. Ellappan Murthy took over charge as Director General of Lighthouses and Lightships, Ministry
of Shipping.
66. Ans: (a)is the correct answer.
The Ministry of Shipping has issued the draft of Aids to Navigation Bill, 2020 for suggestions from
the stakeholders and the general public. he draft bill is proposed to replace the almost nine-decade
old, Lighthouse Act, 1927 to incorporate the global best practices, technological developments and
India's International obligations in the field of Aids to Marine Navigation.
Mansukh Mandaviya is the Minister of State (Independent Charge) for Ministry of Shipping and
Minister of State for Chemical & Fertilizers in Government of India.
The Directorate General of Lighthouses and Lightships is a subordinate office under the Ministry of
Shipping. Its headquarters is in Noida, Uttar Pradesh.
Shri. Ellappan Murthy took over charge as Director General of Lighthouses and Lightships, Ministry
of Shipping.
67. Ans: (a) is the correct answer.
The Ministry of Shipping has issued the draft of Aids to Navigation Bill, 2020 for suggestions from
the stakeholders and the general public. he draft bill is proposed to replace the almost nine-decade
old, Lighthouse Act, 1927 to incorporate the global best practices, technological developments and
India's International obligations in the field of Aids to Marine Navigation.
Mansukh Mandaviya is the Minister of State (Independent Charge) for Ministry of Shipping and
Minister of State for Chemical & Fertilizers in Government of India.
The Directorate General of Lighthouses and Lightships is a subordinate office under the Ministry of
Shipping. Its headquarters is in Noida, Uttar Pradesh.
Shri. Ellappan Murthy took over charge as Director General of Lighthouses and Lightships, Ministry
of Shipping.

QUANTITATIVE TECHNIQUES

68. Ans: (a) Selling price = 32,000 + 4000 = 36000


% of Profit = 400032000 = 12.5%

69. Ans: (c) HTC mobile selling price = 33,000


HTC mobile % of profit = 10%
Means 33,000 ------- 110%
? --------------100% (CP)
Cost price of HTC = 30,000
Micromax cost price = 3/5 * 30,000 = 18,000
Selling price = 22,000
Profit = 4,000
% of profit = (4000/18,000)*100 = 22 2/9%
70. Ans: (d) Profit on Samsung mobile = 3,500
From that profit on LG mobile = 3500 + 500 = 4000
Selling price of LG mobile = 32,000
% of profit on LG = (4000/28,000)*100 = 14 2/7%
71. Ans: (c)
(c) Cost price = 53,000
% of profit = 14%
53,000 ------- 100%
? ------------- 114 %
Selling price = 60,420
Profit = 60,420 – 53,000 = 7420
72. Ans: (d) Cost price = 35,000
Selling price = 35,000 + 3500 = 38500
Ratio = 35000 : 38500 = 10 : 11
73. Ans: (c) Number of students who failed in all the three subjects is
= 2000 – 1316 – 744 – 1180 + 868 + 252 + 332 = 212
74. Ans: (a) Number of students who failed in Bengali but not in Punjabi is = 1180 – 252 = 928

75. Ans: (a) Number of students who failed in Telugu but not in Bengali is = 1316 – 868 = 448

76. Ans: (d) Number of students who failed in Punjabi but not in Telugu is = 744 – 332 = 412

77. Ans: (c) Number of students who failed in Telugu or Bengali but not in Punjabi is = 2000 – 744
= 1256

78. Ans: (c) A is a triangle


So, area of A = 1/2 × 16 × 12 = 96 sqm
So, cost of flooring of A = 96 × 50 = Rs.4800
79. Ans: (a) Perimeter of B = 2 (10 + 20) = 60 m So, cost of fencing of B = 60 × 15 = 900
Perimeter of C = 4 × 15 = 60 m So, cost of fencing of C = 60 × 18 = Rs.1080
So, required difference = 1080 - 900 = Rs.180
80. Ans: (d) Area of D = Base × Height
= 20 × 12 = 240 m2
So, cost of flooring of D= 240 × 60 = Rs.14400
Perimeter of D = 2 (20 + 12) = 64 m
So, cost of fencing of D = 64 × 25 = Rs.1600
So, required ratio = 14400 : 1600 = 9 : 1
81. Ans: (d) Perimeter of E = 2πr = 2 × 22 / 7 × 10 = 440 / 7 m
Cost of fencing of E = 440 / 7 × 22 = Rs.1382.85
Area of C = (15)2 = 225 m2
So, cost of flooring of C = 225 × 40 = Rs.9000
So, required % = 1382.85 × 100 / 9000
= 15.36% of flooring cost of C.
82. Ans: (b) Fencing cost of C = Rs.1080
Fencing cost of D = Rs.1600
Required % = 1080 / 1600 × 100 = 67.5%
83. Ans: (d) Perimeter of B = 2( l+b) = 2 (10+20) = 60 m
Cost of fencing of B = 60*15 = Rs.900.

LOGICAL REASONING
84. Answer (C).
Option A is incorrect as the pandemic is not yet over and we cannot pass judgement without
analyzing the situation wholly. Option B is incorrect and makes a reverse assumption. Option C is
the right answer.
85. Answer (A).
Refer to, ―The government proved inept in not anticipating the adverse impact of the lockdown on
millions of migrant labourers, who were overnight deprived of their means of livelihood. While
extensions of the lockdown were rightly preceded by consultations with the State governments,
keeping the initiation of the lockdown a secret from all the stakeholders defied logic‖
86. Answer (A).
Refer to, ―The issue of spread of infection impacted communal harmony. The TablighiJamaat was
mercilessly flogged, especially by certain people and sections of the electronic media, in order to
profile Muslims adversely.‖ Option 2 and 3 are irrelevant.
87. Answer (A).
Refer to, ―While the protests against the CAA, NRC and NPR were snowballing, the pandemic hit
us and was soon followed by the lockdown.‖ Option B, being an extreme option is incorrect. Option
C cannot be proved conclusively as the crisis is not yet over. Option 4 is incorrect. Whether
organizing the jamat was right or not is nowhere mentioned.
88. Answer: (a)
The third paragraph of the passage verifies Statement 4. None of the other statements is
specifically mentioned anywhere , in the passage.

89. Answer: (b).


Statement 1 is correct as the passage states that a brain bank is like a neighbourhood bank.
Statement 2 is incoherent. Statement 3 has not been mentioned in the passage. Hence, (b) is the
correct answer.
90. Answer: (c)
Option (c) is clearly implied from the second paragraph of the passage. The other options are not
implied from the passage.
91. Answer: (a)
Only statements 2 and 3 are implied from the last paragraph of the passage. Statements 1 and 4
are not implied from the passage.
92. Answer: (d)
Option (a) is the effect of the 'new country' and not competition, hence, incorrect. Option (b) has a
negative connotation. Option (c) is incorrect as competition cures laziness according to the
passage. Hence, none of the statements is correct.
93. Answer: (b)
The author has criticized the Indian education system. Hence (b). Option (c) is too strong, hence
can be eliminated.
94. Answer: (c)
The argument claims that ‗In America, they found elbow room, books and facilities not available and
not likely to be available here.‘ i.e. poor infrastructure in India. Option D can be negated as it is not
mentioned that whether the service is by-product or main product. Hence (c)
95. Answer: (a)
The first is a solution suggested by the policy makers, but it is rejected in the first sentence of the
second paragraph. The second boldface is the solution provided by the argument. Hence (a)
96. Answer: (d)
This question covers the broad outline of the passage, so treat it as a global question and look to
your purpose and main idea. Choice (4) best describes the author‘s attitude toward McCarthyism
and Populism—although they were superficially similar, they were essentially dissimilar.
Choice (1) is an au contraire choice; it actually states the case against the author‘s views.
Choice (2) is too harsh. The first paragraph does draw some parallels between both movements.
Choice (3) is partly right, but partly wrong. Populism was, partially at least, a response to the
conditions of farmers, but nowhere is it suggested that McCarthyism was a response to industrial
conditions. The passage suggests quite the contrary: As described in the last paragraph, Rogin
showed that McCarthyism had little support from workers employed in industry.
97. Answer: (d)
The substance of Rogin‘s criticism of Lipset, Feuer, and Shils is explained in the last two
paragraphs. The second sentence of the third paragraph says what Rogin did: He ―critically
examined their assertions by the simple method of testing them against the evidence.‖ This really
tells you all you need to know to pick the right choice, choice (4).If you missed this, both of these
last two paragraphs are spent describing how Rogin showed that grass roots voting patterns in both
rural Wisconsin counties and more industrial Wisconsin counties failed to support the claims made
by Lipset, Feuer, and Shils about who really did and didn‘t support McCarthy. What Rogin and the
author are clearly suggesting is that Lipset, Feuer, and Shils failed to do their homework properly.
Choice (1) doesn‘t work, because Rogin never really quarrels with any descriptions of McCarthy.
The quarrel is with the misrepresentation of lower-class voters and the real nature of Populism.
Choice (2) doesn‘t work, since the passage never describes Rogin as disagreeing with the idea that
McCarthyism is linked to anti-intellectualism.
Choice (3) is wrong since it seems that Lipset, Feuer, and Shils seemingly placed too little
emphasis on the dual nature of Populism. They failed to perceive the continuity of the progressive
aspects of Populism among voters in Wisconsin.
98. Answer: (a)
The stem is asking why Rogin studied the class character of Wisconsin voting patterns. Primarily,
he did this to check to see if the voting patterns supported or contradicted the arguments of Lipset,
Feuer, and Shils. They argued that lower-class farmers and industrial workers provided crucial
voter support for both Populism and McCarthyism, and Rogin performed his study of voting patterns
in order to challenge their conclusions, by showing that lower-class farmers and workers in fact did
not give their votes to McCarthy. Statement I reflects this. Statement II is wrong since Rogin was
out to disprove the alleged links between Populism and McCarthyism, not explain them. Statement
III is way beyond the scope of the passage
99. Answer: (a)
Choice (1) - In the last half of the second paragraph the passage indicates that Lipset argued that
lower-class support of McCarthy included urban industrial workers, which he describes as ―the
lower classes, especially the workers.‖ The last paragraph describes the study in which Rogin
found that voters in counties with high industrial employment tended to vote against McCarthy.
Rogin used this study to support his counter assertion that McCarthy in fact did not enjoy extensive
support among the lower classes. Thus, an assumption made by both men is that voters among
industrial workers can be classified as lower-class voters.
Choice (2) is out; Rogin‘s overall thesis is to argue that lower-class voters voted more progressively
than Lipset, Feuer, and Shils admit.
Choice (3) clearly isn‘t an assumption of Rogin‘s, who suggests something quite different: McCarthy
lacked extensive support among lower classes.
Choice (4) is beyond the scope of the passage. There‘s not enough information to conclude that
either scholar assumed this.
100. Answer: (b)
This is a detail question, and since it asks you ―according to the passage,‖ it will be a paraphrase of
something that appears within it.
Correct choice (2) paraphrases the first sentence of the third paragraph and the concluding
sentence of the passage. Rogin‘s overall charge against Lipset, Feuer, and Shils is that they not
only reach faulty conclusions but are politically biased against the lower classes from the very
beginning. It‘s this latter charge that‘s summed up in choice (2).
Choice (1) goes too far. Rogin never charges deliberate falsification of evidence by the three men.
Choice (3) is out. Never does Rogin imply that the men‘s viewpoints will support attacks on civil
liberties.
Choice (4) puts things backwards. It‘s Rogin who makes scrupulous use of statistical analysis. This
is what Lipset, Feuer, and Shils failed to do.
101. Answer: (d)
For this global question, go to your purpose and main idea. The author‘s main purpose is to
describe the rebuff to the views on McCarthyism expressed in the second paragraph. The author
states that these conclusions were not based on real evidence, and suggested that they were
shaped by preconceived prejudices. These are flaws in historical methodology, choice (4).
Choice (1) uses the wrong verb. The author is not comparing the views; rather, he is supporting one
particular view, that of Rogin.
Choice (2) goes for a detail—Rogin‘s research methods. And choice (3) distorts the author‘s point
of view that the two movements were fundamentally dissimilar.
102. Answer: (c)
The author says that the professional schoolmaster was a match for people who tried to bring new
ideas and attitudes into education. This means that the schoolmaster succeeded in making the new
subjects dull. So, only Statement 4 is the correct
103. Answer: (a)
The author tells us that some pupils might have learned some facts but that they would not have
learned anything of the scientific method. He expresses himself very forcefully, but still not strongly
enough to make statements 2, 3 or 4 correct. Only statement 1 can be said to be correct.
104. Answer: (a)
The first sentence in the second paragraph gives what the author expects science education to do.
He goes on to say that practically no progress has been made towards the aim of helping the
student to think logically. The last sentence of the passage confirms that he would expect good
science education to enable students to use scientific knowledge. Thus, option (a) is the right
answer.
105. Answer: (d)
Conclusion 1 is verified by the phrase 'those privileged members of the community, who have been
through a secondary or public school education' in the second paragraph. Conclusion 2 is verified
by the phrase 'managed to make the understanding of chemical reactions as dull and as dogmatic'
near the end of the first paragraph. Conclusion 3 is verified by the phrase 'a certain limited success
has been reached in the first of these aims' in the second sentence of the second paragraph.
Conclusion 4 is verified by the phrase 'pupils not only ...to believe exactly what they are told and to
reproduce it when asked... non-sense to them or not' in the second sentence of the last paragraph.
106. Answer- (A)
The last paragraph clearly suggests how the British took over the booming Indian textile industry
and through its repressive policies, halted the progress India had made.
The passage nowhere talks about any predetermined hope of establishing monopoly. As is in para
1, they ruled out locals, French, and Dutch, but that‘s not the central idea of this passage. Also they
did have economic dreams to fulfil but initially it was not about monopoly.
107. Answer- (D).
Throughout the passage, the author is discussing option D. Options A, B and C are either side
points or outside the purview of the passage.
108. Answer - (d) - All of the above. All the statements find the presence in the passage. Statement
1, 2 and 3 can be inferred from Para 1; statement 4 from Para 2.

109. Answer- (B)


Strengthening statement. The claim in the passage about principle economic motive of the British
and how and why the great de-industrialization happened is evident from the passage. The
statement strengthens the author‘s earlier claims.
110. Answer- (B)
Statement 1 talks about the concluding fact that Britain‘s Industrial revolution rests on destruction of
Indian industries. Statement 2 provides the basis of the fact- it highlights how squeezing out foreign
buyers and cutting off export markets for Indian textile led to British manufacturing grow further.
111. Ans: (d) It can be seen from the above cases that C, D and G can be placed at the ends, but
F cannot be.
Case 1: _A_ _ G _ _ C _ _ B _ _ _ _ D_
Case 2: _G_ _ A _ _ C _ _ B _ _ _ _ D_
Case 3: _A_ _ G _ _ D _ _ B _ _ _ _ C_
Case 4: _G_ _ A _ _ D _ _ B _ _ _ _ C_
112. Ans: (a) A can only be seated next to G, C or D as can be seen from the 4 cases above.
Hence E and A cannot sit together.
Case 1: _A_ _ G _ _ C _ _ B _ _ _ _ D_
Case 2: _G_ _ A _ _ C _ _ B _ _ _ _ D_
Case 3: _A_ _ G _ _ D _ _ B _ _ _ _ C_
Case 4: _G_ _ A _ _ D _ _ B _ _ _ _ C_
113. Ans: (b)
1340
Amount Person
2517 Shahnaz
2234 Archana
1340 Helen
1193 Dhenuka
1139 (got by 2517-1378) Chelllamma

114. Ans: (a) 1139


Amount Person
2517 Shahnaz
2234 Archana
1340 Helen
1193 Dhenuka
1139 (got by 2517-1378) Chelllamma

115. Ans: (c)


Dhenuka
Amount Person
2517 Shahnaz
2234 Archana
1340 Helen
1193 Dhenuka
1139 (got by 2517-1378) Chelllamma

LEGAL REASONING

116. Answer: (c)


The above passage highlights the menace of lynching, with disproportionate targeting of Muslims
and Dalits, saying that it is a grim reminder of the fair distance that Indian democracy still has to
traverse to realize the promise of ‗constitutional citizenship‘ – in which one's identity is irrelevant to
the realization of rights and equal protection of the law. Apart from the majoritarian backlash,
another index for testing the equal citizenship claim is the state of civil and personal liberties in the
nation, in particular the freedom to dissent.
117. Answer: (a)
This claim was tested when the State arrested five human rights activists and critics of the State –
calling them ‗Urban Naxals'. These human rights activists had substantial experience working with
marginalized and disadvantaged communities. Further, they had often been critical of the
government in the past. This sudden arrest by the Pune Police was seen as an attempt to freeze
dissent by the heavy hand of state machinery.
118. Answer: (c)
The Court in a 2:1 judgment in Romila Thapar v. UOI rejected the plea for a Special Investigation
Team (SIT) to probe into the investigation, on the ground that the State had adduced sufficient
evidence for the possibility that they are members of a banned terrorist organization, CPI (M). Note
that the petitioners were not allowed to scrutinize this evidence, as it was submitted in sealed
covers – only the judges viewed it. The lone dissenting judge, DY Chandrachud, called for a court-
monitored probe as he recounted various procedural lapses in the arrest process, signalling States‘
selective targeting of critics.
[Extracted from ‗10 Cases that Shaped India in 2018‘. Published in IACL-AIDC Blog]
REBABACHINCHINCHU
119. Answer: (d)
This case forces one to re-examine the fragile nature of speech protection when it collides with the
state power. The standards of proof, required for successful conviction, need not be met to justify a
call for a probe at initial stages. A prima facie case is sufficient to merit investigation. Further,
should the power asymmetry between citizens and the State not be factored in, when such
brazenness is shown in arresting dissenters and critics? Rather than legitimizing sealed cover
jurisprudence, shouldn‘t the Court critically assess the government's account of the facts?
120. Answer: (a)
The menace of lynching, with disproportionate targeting of Muslims and Dalits, is a grim reminder of
the fair distance that Indian democracy still has to traverse to realize the promise of ‗constitutional
citizenship‘ – in which one's identity is irrelevant to the realization of rights and equal protection of
the law. Apart from the majoritarian backlash, another index for testing the equal citizenship claim is
the state of civil and personal liberties in the nation, in particular the freedom to dissent.
121. Ans: (c) To prove cheating it must be shown that there is a failure of the promise which was
made. It must be shown that there was no effort on the part of accused to perform his promise.

122. Ans: (a) Yes as he knowingly lied about completion of his duties, and took a considerable sum
against it.

123. Ans: (c) A intentionally induced the person so deceived to defraud him out of his property for a
wrongful gain.

124. Ans: (d) To establish cheating under section 415 and 420 o the IPC a person needs to fulfil all
the elements mentioned in the paragraph above.

125. Ans: (c) Anything that could be converted of monetary value can be considered property. For
this case too, he deceived someone through misrepresentation of his position to make a wrongful
gain.

126. Ans: (b) A criminal prosecutor must show beyond any reasonable doubt to convict an accused
person that the suspect actively and knowingly contributed in a crime that affected another person
or their property.

127. Ans: (c) [Agreed that the information was sensitive and non- public but Rachna eventually did
not benefit anything from this, which is an essential requirement of insider trading. An act will be
called insider trading if such information is provided to a third party who benefits from his
investment activities post receiving such information.]

128. Ans: (a) [Agreed that the information was sensitive and non- public but earlier Rachna did not
benefit anything from this, but now she did benefit which is an essential requirement of insider
trading. An act will be called insider trading if such information is provided to a third party who
benefits from his investment activities post receiving such information]

129. Ans: (a) [Mr Aarohi earlier managed the work of new business and had complete information
about it therefore thus was connected to it and acquired it was the virtue of this connection]
130. Ans: (d) [An act will be called insider trading if such information is provided to a third party who
benefits from his investment activities post receiving such information. In generic terms, insider
trading means buying and selling of stocks and shares based on significant information which is
publicly not available. This information resulted in investment in the same company which resulted
in an unfair advantage and benefitted the person]

131. Ans: (a) [The information was made public after Meghana had invested relying on it, therefore
when she invested it was still sensitive and gave her an unfair advantage]

132. Ans: (b) [Section15 (g) (i) only talks about making insider liable for insider trading, therefore
Aarohi. Meghana, as a third person may also be liable but in the passage only the information
revealed by the insider is considered as an offence. Thus, based on the passage only Aarohi
should be punished]

133. Ans: (a) (Section 498A is non-bailable. However, the police must provide healthcare facilities
to the accused and it‘s for the Magistrate to decide if further detention is necessary.)

134. Ans: (c) (On 2 July 2014, the Supreme Court of India in an order stopped automatic arrests
under Section 498a. The Court directed the police to use Section 41 of the Code of Criminal
Procedure, 1973, which contains a checklist, to decide whether an arrest is necessary. The Court
also stated that in all arrests the magistrate must examine whether further detention was
necessary.)

135. Ans: (d) (Under the Dowry Prohibition Act, there is no provision of withdrawing a complaint in
case of a reconciliation.)

136. Ans: (c) (The Section 113-B of the Indian Evidence Act, 1879 says that if a married woman
commits suicide within seven years of marriage, it must be assumed by the court that her husband
and his family abetted the suicide)

137. Ans: (a) (On 2 July 2014, the Supreme Court of India in an order stopped automatic arrests
under Section 498a. The Court directed the police to use the Section 41 of the Code of Criminal
Procedure, 1973, which contains a checklist, to decide whether an arrest is necessary.)

138. Ans: (d) (As of now, the Anti dowry laws take the wife's words against the husband. The case
can‘t be dismissed and he cannot be granted bail, but he can prove his innocence in the court of
law.)

139. Ans: (b) A doctor is not an insurer for the patient, inability to cure the patient would not
amount to negligence if the doctor had taken reasonable care.

140. Ans (a) - The consent given by the patient is for the risk that is involved in the operation and
not for any incident which took place because of the carelessness of the doctor.

141. Ans: (b) - The principle of res ipsa loquitur comes into operation only when there is proof that
the occurrence was unexpected, that the accident could not have happened without negligence and
lapses on the part of the doctor

142. Ans: (c) - In medical profession there may arise a situation where the doctor needs to make a
complicated yet important decision in order to save the life of the patient. In such situation the
doctor cannot be accused of negligence for any incident which occurs despite of taking all the
necessary precaution and care.

143. Ans: (b) - Doctors in India may be held liable for their services individually or vicariously
unless they come within the exceptions specified in the case of Indian Medical Association vs V P
Santha i.e. they treat patients for free.

144. Ans: (b) - that a doctor is not liable for negligence or medical deficiency if some wrong is
caused in her/ his treatment or in her/ his diagnosis if she/ he has acted in accordance with the
practice accepted as proper by a reasonable body of medical professionals skilled in that particular
art, though the result may be wrong.

145. Ans: (b) - (In case of citizens of India comes across a case of violation of fundamental rights,
he or she can approach the Supreme Court of India for ultimate justice.)

146. Ans: (a) (Because article 15 (2) & (3) talks about bias or restriction on the basis of race,
gender, place of birth, caste, or religion. However, any restrictions, strictly not based on the above-
mentioned things, can be levied.)

147. Ans: (d) - (Article 15 (5) exempts minority-run schools from making reservations in their
educational institutions.)

148. Ans: (b) - (Article 15 (3) makes it possible for the state to create special provisions for
protecting the interest of women and children.)

149. Ans: (a) - (under article 15 (5), state holds the right to make special provisions for the
betterment of the socially and economically backward sections of the society or for the SCs and
STs.)

150. Ans: (c) - (the correct option is a violation of Article 19 (1) (g), not article 15.)
Legal Edge 04 Mock

Answers & Details

ENGLISH

1. Ans:(c)
The first sentence of this paragraph defines Boal‘s work as a response to a culture of apathy. (a) is
incorrect— the paragraph describes a response, not an elaboration. (b) is incorrect because it is not
until the last paragraph that the author provides a rationale for the two theatres. (d) is beyond the
scope of the passage.
2. Ans:(d)
This is essentially a vocabulary question. ―Power‖ is one meaning of agency, and this is the only
meaning that makes sense in the context of creating ways to free themselves. (a) and (b) are other
meanings of agency that do not make sense in context. (c) might be related to agent but has no
relationship to agency or the passage here.
3. Ans:(d)
The last paragraph defines a ―spect-actor‖ as someone who simultaneously witnesses and creates
theater. In the second paragraph, the passage states that at image events everyone is at once
theater-maker and witness. (a) is incorrect, as theater is not mentioned. In (b), Boal specifically
says that catharsis keeps people passive (also, the audience member is not acting, which is crucial
to being a ―spectator‖). (c) is incorrect and, to an extent, backwards— the passage said that Boal
found that position analogous to that of a passive audience.
4. Ans:(a)
At the end of the first paragraph, the author paraphrases Boal: theater etiquette creates a kind of
culture of apathy where individuals do not act communally… and remain distanced from art. (c) can
be eliminated.
(b) and (d) are wrong because Boal states that traditional theater discourages political action by
providing catharsis.
(a)is correct because Boal states that actors do go into the audience, so they are not prevented
from doing so.
5. Ans:(d)
This choice is a characteristic of an Image workshop, not a Forum workshop. In the second
paragraph, the passage states that Forum workshops begin with a narrative skit (a) then the
facilitator — or mediator (b). Choice (c) is justified as the paragraph states that performances do not
always arrive at a satisfactory solution.
6. Ans:(a)
(a) Looks the best from the opening lines of the passage.
(b) This is only the purpose of the second paragraph of the passage.
(c)Again this is with specific reference to the Niagara Falls but the author's idea is to use Niagara
Falls as an example to arrive at a broader conclusion about natural wonders in general.
(d)This is only the purpose of the 3rd paragraph
The correct answer is (a).
7. Ans:(d)
(a) There is no comparison of the Niagara Falls with other natural waterfalls in the passage.
(b) 'Lied' is not the correct word since Hennepin most likely believed that he was stating the truth.
Incorrect estimation is not necessarily the same thing as lying.
(c)This is true but of no relevance to the height of the Niagara Falls.
(d) Clearly follows from the last line of the 2nd paragraph.
The correct answer is (d).
8. Ans:(d)
The 3rd para states that a person standing at the bottom of the Falls would hear a thunderous
sound, it is above the Falls that a person would experience complete silence. Thus (d) cannot be
true and is the correct answer.
(a) is mentioned in the 3rd paragraph.
(b) is implied by the opening lines of the passage.
(c) Is implied by the 3rd para.
The correct answer is (d).
9. Ans:(d)
We can start by looking for the option that agrees with the information in the passage and so at
which, anyone who has read the passage, would not be surprised. Alternatively, we could try to find
four options that a person who has read the passage would be surprised at i.e. four things that are
contrary to what is mentioned in the passage. The fifth option will be our answer. Either way we will
arrive at (d) as the answer. The last para mentions a river and the Falls clearly have to be a part of
the river.
(a)This would be surprising because the passage agrees with the view that the height of the falls is
closer to 150 feet.
(b)Again the 3rd para states that there are no mountains in the immediate vicinity of the falls so this
will be a surprising fact.
(c)The 3rd para states that there is a thunderous noise at the base of the falls.
The correct answer is (d).
10. Ans:(c)
The passage talks about mathematics, its history and the contributions of Indian mathematics.
Option (a) talks about a newspaper article lamenting the lack of scientific temper among Indian
historians. However, the passage does not pass any such arguments about the scientific temper of
Indian historians. Hence, (a) can be negated.
A blog essaying the history of mathematics from ancient times to the modern era would dwell upon
various mathematicians across different eras, their contributions and findings. However, the
passage only focuses on establishing the contribution of ancient Indian mathematicians to the field
of mathematics. Thus, option (b) can be negated.
Option (d) states that the passage is a part of an article published in a journal of mathematics
describing the contributions made by Indian mathematicians. However, the passage describes the
prevailing situation in the academic field and states that the contributions of Indian mathematics
have not been given prominence. Moreover, the language of the passage does not reflect that of a
mathematical journal. Hence, (d) can be ruled out. The first line of the last paragraph states that
'Keeping this in mind, the authors should be commended for endeavouring to take up this
prodigious task.' From this it becomes clear that the passage is introducing some authors who will
describe the 'concrete contributions made by Indian mathematicians in the initial advancement of
mathematics'. Thus, option (c), which states that the article is a preface or foreword in a book,
seems most appropriate. Hence, (c).
11. Ans:(d)
'Remonstrate' means 'to reason or plead in protest'. 'Adventitious' means 'something that happens
asa result of an external factor or chance rather than design or inherent nature'. 'Prodigious' means
'something which is of remarkably or impressively great in extent, size, or degree'. 'Momentous'
means 'of great importance or significance'. Thus, i - d, ii - c, iii - a, iv - b are the correct answers.
Option (d) gives this combination. Hence, (d).
12. Ans:(c)
The first option gives the title as 'Eurasia: The Cradle of History of Mathematics'. The term 'Eurasia'
is used to refer to the combined landmass of Europe and Asia. However, the passage clears states
that the contribution of Indian mathematics has been ignored. The figurative use of' cradle' means
'the place or region in which anything is nurtured or sheltered in its earlier stage'. As the whole of
Europe learned mathematics through the Greeks, it can hardly be called as the cradle of the history
of mathematics. Thus, option (a) can be ignored. Option (b) states that India and Greece were two
civilizations united by mathematics which were divided by destiny. However, this option joins the
contribution of India and Greece. The passage does not state any fact which shows that
contributions made by Indian mathematicians in the initial advancement of mathematics are related
to the developments that occurred in a later era in Greece. Also, the term 'divided by destiny'
means 'to be separated or torn between two extreme choices'. Even this cannot be concluded from
the passage. Thus, (b) cannot be inferred from the passage.
Option (d) states that the passage can be a perspective by western historians and Euro-centric
academics. However, the entire passage focuses on establishing a non-Western and Indian
viewpoint of history of Indian mathematics. Thus, (d) can also be ruled out.
The expression 'academic acrobatics' means 'establishing an academic position by trickery or by
usage of deception'. Also, 'negation' means 'the act or an instance of denying a truth'. The western
and Euro-centric academicians have consistently denied the contributions of Indian mathematics.
Moreover, by using their position of eminence, they have created an illusion that Indian
mathematics had very little to contribute to the history of mathematics. Thus, option (c) clearly
captures the message of the passage. Hence, (c).
13. Ans:(d)
The first option states that historians of Indian origin have a tendency to exaggerate claims of
historic contribution of Indian mathematics. The passage does not make such a judgment. Also, the
claim that Indian universities do not have the wherewithal to fund courses on history of Indian
mathematics is not mentioned in the passage. Thus, (a) can be ruled out.
The passage clearly surmises that the belief of Eurocentric academics which states that almost
everything in mathematics was done outside India is a false, motivated and fabricated opinion.
Hence, (b) is incorrect.
The passage states that some texts of Indian history have not been translated from their original
languages. However, this does not mean that they 'cannot be translated'. Thus, option (c) is not
correct.
The first part of option (d) states that the 'history of Indian mathematics is not even considered as a
part of the world history of mathematics due to the Eurocentric academics that have spread a
motivated version of the history of mathematics'. This is correctly stated in the third paragraph of
the passage. The second part of the option (d) states that 'the lack of multiple studies determining
positive and concrete contributions made by Indian mathematicians in the initial advancement of
mathematics'. The last paragraph mentions this point and states that some authors have begun to
work on this. Thus, (d) is true. Hence, (d).
14. Ans:(b)
The first paragraph states that some Indian mathematicians do not have the time to study about the
history of mathematics and the contribution of Indian mathematics to the history of mathematics.
However, the option (i) also states that they do not have the energy, resources or orientation to do
so. This is incorrect as it is not stated in the passage. Hence, (i) cannot be inferred from the
passage.
Option (ii) states that highlighting concrete contributions made by Indian mathematicians in the
initial advancement of mathematics and relating them to the developments that occurred in a later
era in Greece, Middle East, China and Japan will expose the deception spread by the Euro-centric
academicians about the contribution. This can clearly be inferred from the third and four
paragraphs. Hence, (ii) can be inferred from the passage.
The option (iii) states that unless a nation and a people work to own their place in history and
present it in an appropriate manner, they will be a slave of external projections of their histories.
This almost seems to be the summary of the passage. Hence, (iii) can also be inferred from the
passage.
The option (iv) states that, it is important for academics to move ahead from the deification of the
contribution of Indians to the history of mathematics and work practically on finding new innovations
in fields such as mathematics. This is in contrast to the message of the passage. Moreover, the
passage does not state that mathematics should work on finding new innovations in fields such as
mathematics. Hence, (iv) cannot be inferred from the passage.
The last option states that it is essential to find Western historians and Euro-centric academics of
history of mathematics as they have more credibility than Indian mathematicians. This is completely
opposite to the message of the passage. The passage states that Indian mathematicians must
focus on establishing the contribution of ancient Indian mathematics to the field of mathematics.
Moreover, the passage cites the bias of Western historians and Euro-centric academics and their
negation of Indian history. Thus, (v) cannot be inferred from the passage.
Thus, (ii) and (iii) are correct. Option (b) gives this combination. Hence, (b).
15. Ans:(d)
All the four words are synonyms of 'deadening'. However, the first three of them, 'injuring',
'destroying' and 'stunning' are used in the context of injuries. The passage states that the studies of
history of mathematics have suffered hopelessly and 'this has led to a deadening disposition‘.
It means that the prevailing state of academic studies have 'deadened' or 'dulled' one's frame of
mind with respect to studies on the history of mathematics in India. Thus, the closest meaning of
the word 'deadened' is 'dulling'. Hence, (d).
16. Ans:(c)
Understanding the author‘s purpose comes from a careful consideration of the whole passage. The
author begins by noting one explanation for the rise in temporary employment, but dismisses it,
finding another explanation more likely. The author closes the passage by making specific
recommendations to counter the problems caused by temporary employment.
A The author uses statistical analyses as the basis of an explanation, but the analyses act only as
support for the larger purpose of explaining a trend; no further studies are proposed.
B The author explores possible reasons for a recent development but recommends ways to curb or
change that development; the author does not predict the consequences if the situation is left
unchanged or the recommendations unmet.
C Correct. The author examines possible reasons for the rise in temporary employment and makes
specific recommendations to change the current situation.
D The use of the phrase several theories is enough to make this inaccurate. Two types of
explanation are suggested: employee preference or employer self-interest.
Hence, the answer is (c).
17. Ans:(b)
This question is based on information explicitly stated in the passage. The statistical analyses
suggest that factors affecting employers account for the rise in temporary employment.
A Some observers attribute the rise to the composition of the workforce; the passage does not
identify what most experts believe.
B Correct. The factors affecting employers may explain the rise in temporary employment.
C The passage suggests that these factors were more important than demographic changes in
explaining the rise.
D Although there is some suggestion in the passage that employers at some point experienced
difficulty from the cost of labor, the passage does not suggest that a sharp increase in that cost
occurred in the 1980s— and even suggests that labor costs may have decreased because of
labor‘s reduced bargaining strength.
Hence, the answer is (b).
18. Ans:(d)
Since the word suggests is used in the question, the answer is probably not directly stated in the
passage and therefore has to be inferred. The author believes that the rise in temporary
employment during the 1980s can be explained by two factors affecting employers: product
demand and labor‘s reduced bargaining strength. Temporary employment allows employers to
adapt their workforce to the fluctuating demand for their product. At this time, labor‘s reduced
bargaining strength left employers, not workers, in greater control of the terms of employment.
A This goes too far beyond the information provided in the passage. The passage neither says nor
implies anything about efficiency levels before the 1980s.
B The author says that demographic changes in the workforce did not correlate with variations in
the total number of temporary workers, ruling out this explanation.
C In 1992, the author recommended government-mandated policies because they did not exist.
D Correct. Labor‘s reduced bargaining power resulted in employers‘ increased control over the
terms of employment.
Hence, the answer is (d).
19. Ans:(d)
The question‘s use of the word suggests indicates that the answer is probably not directly stated in
the passage. The author says that the rise in temporary employment now far exceeds the level
explainable by recent workforce entry rates of groups said to prefer temporary jobs. Thus, the
number of workers employed on a temporary basis is far greater than the number of workers who
actually do prefer temporary employment.
A No evidence is presented that temporary jobs led to permanent positions.
B The passage grants that there was increased participation in the workforce by certain groups,
such as first-time or reentering workers. This suggests more rather than less demographic diversity.
C The role of temporary workers in labor unions is not discussed.
D Correct. The passage indicates that the number of workers in temporary jobs was higher than the
number of workers who stated a preference for temporary work.
Hence, the answer is (d).
20. Ans:(a)
These observers specifically attribute the growth of temporary employment to increased
participation in the workforce by certain groups … who supposedly prefer such arrangements. On
the basis of the passage‘s first sentence, any prediction these observers might make must be about
the relation between the number of workers in temporary employment and the preference of these
workers for temporary employment. No other issue is discussed. A rise in temporary employment
could be explained only by a rise in the number of new workers who prefer temporary jobs, and a
decline in temporary employment only by a decline in the number of new workers who prefer
temporary work.
A Correct. By this rationale, the only reason for a decline in temporary employment would be a
corresponding decline in the number of new workers who preferred temporary jobs.
B According to the observers, temporary employment would increase only if a greater number of
employers who preferred temporary jobs entered the workforce.
C These observers are not said to consider control over the terms of employment.
D These observers are not said to consider the relationship between product demand and
temporary employment.
Hence, the answer is (a).
21. Ans:(a)
In its context in this passage, the word excessive indicates a value-judgment by the author. The
author recommends that firms be discouraged from creating excessive numbers of temporary
positions on the basis of the statistical analyses, which show that the rise in temporary employment
now far exceeds the level explainable by recent workforce entry rates of groups said to prefer
temporary jobs. In the context of the passage, it is clear that the author believes that the large
expansion in temporary employment exclusively serves employer interests at the expense of
employee interests (including their preferences), and is, for that reason, excessive.
A Correct. An expansion of temporary employment that serves employer interests more than it
serves employee interests (such as preferences) is considered by the author to be excessive.
B It is not because the expansion in temporary employment allows employers to respond to
fluctuations in product demand that the author regards the expansion as excessive.
C The relation of temporary employment to the success of firms is not discussed.
D The relation of temporary employment to an expanding economy is not discussed.
Hence, the answer is (a).
22. Ans:(d)
The author closes the passage with a list of specific recommendations. Check that list against the
possible answers. By the process of elimination, choose the one recommendation the author does
not make. The author recommends that government policymakers consider:
1) mandating benefit coverage for temporary employees,
2) promoting pay equity between temporary and permanent workers,
3) assisting labor unions in organizing temporary workers, and
4) encouraging firms to assign temporary jobs primarily to employees who explicitly indicate that
preference.
A The author does recommend that firms assign temporary jobs to workers who prefer temporary
work.
B The author does recommend that pay equity between temporary and permanent workers be
encouraged.
C The author does recommend that labor unions be assisted in organizing temporary workers.
D Correct. The author does not recommend that such guidelines be established.
Hence, the answer is (d).
23. Ans:(b)
Paragraph 1 introduces readers to what the heart does and what it is made of. Paragraph 2
discusses the muscle that divides the heart into four chambers and how the muscle helps to pump
blood. Paragraph 3 explains how blood circulates between the right and left sides of the heart.
Paragraph 4 talks about usedbloodand how it is different from newly oxygenated blood. Paragraph
5 concludes the passage by explaining that the heart works harder than the body‘s other muscles.
The main idea, or thesis, is generally stated in the opening paragraph. Notice that the author states
the function of the heart and lists the mechanical attributes of the heart in the opening paragraph.
Choice (A) is a misused detail; pulmonary circulation is briefly discussed, but the passage contains
moreinformation about the heart other than pulmonary circulation. Choice (B) is a good match; this
answer best reflects the thesis stated in the opening paragraph. Choice(C) is too narrow; although
the heart's need for more blood than other organs need is mentioned, this is not the primary
concern of the passage. Choice (D) is a misused detail; the second paragraph is concerned with
the pumping of the heart; however, the other paragraphs explore other functions and aspects of the
heart.
24. Ans:(d)
Paragraph 1 introduces readers to what the heart does and what it is made of. Paragraph 2
discusses the muscle that divides the heart into four chambers and how the muscle helps to pump
blood. Paragraph 3 explains how blood circulates between the right and left sides of the heart.
Paragraph 4 talks about usedbloodand how it is different from newly oxygenated blood. Paragraph
5 concludes the passage by explaining that the heart works harder than the body‘s other muscles.
The question tells the reader that myositis are muscle cells. Look for a section of the passage that
discusses a muscle in the heart. The passage explains that the myocardium is composed of cells
called myositis, which work together to contract and expand the heart chambers. Choice (A) is out
of scope; the function of heart valves is not discussed in detail in this passage. Choice (B) is
distortion; although the myositis of the myocardium are responsible for pumping the blood to the
lungs, they are not responsible for oxygenation. Choice (C) is extreme; the heart never rests.
Choice (D) is a good match; myositis compose the myocardium, which contracts and relaxes the
heart chambers.
25. Ans:(d)
Paragraph 1 introduces readers to what the heart does and what it is made of. Paragraph 2
discusses the muscle that divides the heart into four chambers and how the muscle helps to pump
blood. Paragraph 3 explains how blood circulates between the right and left sides of the heart.
Paragraph 4 talks about usedbloodand how it is different from newly oxygenated blood. Paragraph
5 concludes the passage by explaining that the heart works harder than the body‘s other muscles.
Contraction and relaxation are both states of the heart‘s pumping cycle, and they are defined in this
sentence. Because relaxation is set in parentheses, the definition/context precedes it.Choice (A) is
distortion; the final paragraph states that the heart never rests. Choice (B) is an opposite;
contraction is the process of squeezing blood from the heart's chambers. Choice (C) is distortion;
relaxation is a part of the pumping process, but this does not define the term. Choice (D) Isa good
match for your prediction; the definition of relaxation in this context is stated prior to
the parentheses: expanding to allow blooding.
26. Ans:(b)
Paragraph 1 introduces readers to what the heart does and what it is made of. Paragraph 2
discusses the muscle that divides the heart into four chambers and how the muscle helps to pump
blood. Paragraph 3 explains how blood circulates between the right and left sides of the heart.
Paragraph 4 talks about usedbloodand how it is different from newly oxygenated blood. Paragraph
5 concludes the passage by explaining that the heart works harder than the body‘s other muscles.
Pulmonary circulation is mentioned. Read the following sentence for context. The author's use of
Thetis establishes that pulmonary circulation is about to be defined. In this case, the pumping of
blood through the lungs and oxygenation compose the process of pulmonary circulation. Choice (A)
is a misused detail; pulmonary circulation begins in the right heart chamber, but the circuit is
also composed of the lungs. Choice (B) is a good match for your prediction. Choice (C) is an
opposite; blood is pumped through the left side of the heart and the aorta following pulmonary
circulation. Choice (D) is extreme; the left heart chamber is not involved in pulmonary circulation.
27. Ans:(a)
Paragraph 1 introduces readers to what the heart does and what it is made of. Paragraph 2
discusses the muscle that divides the heart into four chambers and how the muscle helps to pump
blood. Paragraph 3 explains how blood circulates between the right and left sides of the heart.
Paragraph 4 talks about usedbloodand how it is different from newly oxygenated blood. Paragraph
5 concludes the passage by explaining that the heart works harder than the body‘s other muscles.
Reread the sentences before and after to further ascertain the context of the word oxygenated. It
mentions the blood is newly oxygenated; it says that the blood receives oxygen in the lungs. Choice
(A) is a good match for your prediction; pulmonary circulation supplies oxygen to the blood. Choice
(B) is an opposite; deoxygenated is when the blood is in need of oxygen. Choice (C) is a misused
detail; blood returning to the heart from the body is supplied with carbon dioxide, but it is in need of
oxygen. Choice (D) is distortion; although oxygenated blood is indeed bright red, the context does
not state this.
28. Ans:(d)
Paragraph 1 introduces readers to what the heart does and what it is made of. Paragraph 2
discusses the muscle that divides the heart into four chambers and how the muscle helps to pump
blood. Paragraph 3 explains how blood circulates between the right and left sides of the heart.
Paragraph 4 talks about usedbloodand how it is different from newly oxygenated blood. Paragraph
5 concludes the passage by explaining that the heart works harder than the body‘s other muscles.
What does blood deliver to the body, as it pertains to this passage? Used blood is returned to the
right side of the heart. Because the body has taken the oxygen it needs from the blood, it is
returned to the heart. Choice (A) is extreme; the passage never asserts that blood is considered
waste. Choice (B) is an opposite; the deoxygenated blood is dark bluish red. Choice (C) is
amis used detail; although the blood is indeed deoxygenated, it is not waste but instead will be
passed through the right heart and lungs to be oxygenated. Choice (D) is a good match for your
prediction; blood delivers nutrients to the organs and tissues of the body; once it circulates, it
is used and must return to the heart to be oxygenated and circulated again.
29. Ans:(c)
Paragraph 1 introduces readers to what the heart does and what it is made of. Paragraph 2
discusses the muscle that divides the heart into four chambers and how the muscle helps to pump
blood. Paragraph 3 explains how blood circulates between the right and left sides of the heart.
Paragraph 4 talks about usedbloodand how it is different from newly oxygenated blood. Paragraph
5 concludes the passage by explaining that the heart works harder than the body‘s other muscles.
The heart's mass and blood needs in proportion to the body are mentioned. The sentence begins
with the word Although, which sets the heart's mass (1 %) apart from its blood requirements (4%-
50/o).Choice (A) is a misused detail; the heart comprises less than 1 % of the body's weight.
Choice (B) is extreme; this is the opposite of what the paragraph is explaining. Choice (C) is a good
match for your prediction. Choice (D) is a misused detail; the heart does need a richer supply, but
the question asks how much.

GENERAL KNOWLEDGE/CURRENT AFFAIRS


30. Ans:(b)
Champions of the Earth is the United Nations highest environmental honour. The award celebrates
outstanding figures from the public and private sectors and from civil society whose actions have
had a transformative, positive impact on the environment.
The release of high-resolution climatic projections through the Coordinated Regional Climate
Downscaling Experiment (CORDEX) initiative provides unique opportunities to assess the risk of
hydrological extremes in various world regions. The Brahmaputra is the second largest river in
South Asia and is the main freshwater source of Bangladesh.
Padma Shri Jadav Payeng will be conferred with the Swami Vivekananda Karmayogi Award on 29
February in New Delhi at a function instituted by My Home India. He was awarded the 6th
Karmayogi Award for his consistent efforts in creating a real man-made forest through massive
reforestation. The award is composed of a trophy, a recitation and a reward of Rs 1 lakh.
Jadav Payeng is famously known as the Forest Man of India. Padma Shri Jadav Payeng is an
environmental activist and forest worker from Jorhat. In the past several years, he has planted and
tended trees on a sandbar of the river Brahmaputra turning it into a forest reserve.
Majuli is a river Island in the Brahmaputra River, Assam and in 2016 it became the first island to be
made a district in India. It had an area of 880 square kilometres at the beginning of the 20th
century, but having lost significantly to erosion it covers 352 square kilometres as at 2014.
Padma Shri is the fourth highest civilian award in the Republic of India, after Bharat Ratna, Padma
Vibhushan & Padma Bhushan. It is awarded by the Government of India, every year on India‘s
Republic Day.
31. Ans:(b)
Champions of the Earth is the United Nations highest environmental honour. The award celebrates
outstanding figures from the public and private sectors and from civil society whose actions have
had a transformative, positive impact on the environment.
The release of high-resolution climatic projections through the Coordinated Regional Climate
Downscaling Experiment (CORDEX) initiative provides unique opportunities to assess the risk of
hydrological extremes in various world regions. The Brahmaputra is the second largest river in
South Asia and is the main freshwater source of Bangladesh.
Padma Shri Jadav Payeng will be conferred with the Swami Vivekananda Karmayogi Award on 29
February in New Delhi at a function instituted by My Home India. He was awarded the 6th
Karmayogi Award for his consistent efforts in creating a real man-made forest through massive
reforestation. The award is composed of a trophy, a recitation and a reward of Rs 1 lakh.
Jadav Payeng is famously known as the Forest Man of India. Padma Shri Jadav Payeng is an
environmental activist and forest worker from Jorhat. In the past several years, he has planted and
tended trees on a sandbar of the river Brahmaputra turning it into a forest reserve.
Majuli is a river Island in the Brahmaputra River, Assam and in 2016 it became the first island to be
made a district in India. It had an area of 880 square kilometres at the beginning of the 20th
century, but having lost significantly to erosion it covers 352 square kilometres as at 2014.
Padma Shri is the fourth highest civilian award in the Republic of India, after Bharat Ratna, Padma
Vibhushan & Padma Bhushan. It is awarded by the Government of India, every year on India‘s
Republic Day.
32. Ans:(b)
Champions of the Earth is the United Nations highest environmental honour. The award celebrates
outstanding figures from the public and private sectors and from civil society whose actions have
had a transformative, positive impact on the environment.
The release of high-resolution climatic projections through the Coordinated Regional Climate
Downscaling Experiment (CORDEX) initiative provides unique opportunities to assess the risk of
hydrological extremes in various world regions. The Brahmaputra is the second largest river in
South Asia and is the main freshwater source of Bangladesh.
Padma Shri Jadav Payeng will be conferred with the Swami Vivekananda Karmayogi Award on 29
February in New Delhi at a function instituted by My Home India. He was awarded the 6th
Karmayogi Award for his consistent efforts in creating a real man-made forest through massive
reforestation. The award is composed of a trophy, a recitation and a reward of Rs 1 lakh.
Jadav Payeng is famously known as the Forest Man of India. Padma Shri Jadav Payeng is an
environmental activist and forest worker from Jorhat. In the past several years, he has planted and
tended trees on a sandbar of the river Brahmaputra turning it into a forest reserve.
Majuli is a river Island in the Brahmaputra River, Assam and in 2016 it became the first island to be
made a district in India. It had an area of 880 square kilometres at the beginning of the 20th
century, but having lost significantly to erosion it covers 352 square kilometres as at 2014.
Padma Shri is the fourth highest civilian award in the Republic of India, after Bharat Ratna, Padma
Vibhushan & Padma Bhushan. It is awarded by the Government of India, every year on India‘s
Republic Day.
33. Ans:(c)
Champions of the Earth is the United Nations highest environmental honour. The award celebrates
outstanding figures from the public and private sectors and from civil society whose actions have
had a transformative, positive impact on the environment.
The release of high-resolution climatic projections through the Coordinated Regional Climate
Downscaling Experiment (CORDEX) initiative provides unique opportunities to assess the risk of
hydrological extremes in various world regions. The Brahmaputra is the second largest river in
South Asia and is the main freshwater source of Bangladesh.
Padma Shri Jadav Payeng will be conferred with the Swami Vivekananda Karmayogi Award on 29
February in New Delhi at a function instituted by My Home India. He was awarded the 6th
Karmayogi Award for his consistent efforts in creating a real man-made forest through massive
reforestation. The award is composed of a trophy, a recitation and a reward of Rs 1 lakh.
Jadav Payeng is famously known as the Forest Man of India. Padma Shri Jadav Payeng is an
environmental activist and forest worker from Jorhat. In the past several years, he has planted and
tended trees on a sandbar of the river Brahmaputra turning it into a forest reserve.
Majuli is a river Island in the Brahmaputra River, Assam and in 2016 it became the first island to be
made a district in India. It had an area of 880 square kilometres at the beginning of the 20th
century, but having lost significantly to erosion it covers 352 square kilometres as at 2014.
Padma Shri is the fourth highest civilian award in the Republic of India, after Bharat Ratna, Padma
Vibhushan & Padma Bhushan. It is awarded by the Government of India, every year on India‘s
Republic Day.
34. Ans:(b)
Champions of the Earth is the United Nations highest environmental honour. The award celebrates
outstanding figures from the public and private sectors and from civil society whose actions have
had a transformative, positive impact on the environment.
The release of high-resolution climatic projections through the Coordinated Regional Climate
Downscaling Experiment (CORDEX) initiative provides unique opportunities to assess the risk of
hydrological extremes in various world regions. The Brahmaputra is the second largest river in
South Asia and is the main freshwater source of Bangladesh.
Padma Shri Jadav Payeng will be conferred with the Swami Vivekananda Karmayogi Award on 29
February in New Delhi at a function instituted by My Home India. He was awarded the 6th
Karmayogi Award for his consistent efforts in creating a real man-made forest through massive
reforestation. The award is composed of a trophy, a recitation and a reward of Rs 1 lakh.
Jadav Payeng is famously known as the Forest Man of India. Padma Shri Jadav Payeng is an
environmental activist and forest worker from Jorhat. In the past several years, he has planted and
tended trees on a sandbar of the river Brahmaputra turning it into a forest reserve.
Majuli is a river Island in the Brahmaputra River, Assam and in 2016 it became the first island to be
made a district in India. It had an area of 880 square kilometres at the beginning of the 20th
century, but having lost significantly to erosion it covers 352 square kilometres as at 2014.
Padma Shri is the fourth highest civilian award in the Republic of India, after Bharat Ratna, Padma
Vibhushan & Padma Bhushan. It is awarded by the Government of India, every year on India‘s
Republic Day.
35. Ans:(c)
Activities are climate-neutral or CO₂ neutral if they do not put a strain on the climate, i.e. if they do
not create greenhouse gases. Climate neutrality can be achieved if CO₂emissions are reduced to a
minimum and all remaining CO₂ emissions are offset with climate protection measures. This
stimulus deal is named Next Generation EU.
Ursula Gertrud von der Leyen is a German politician and the president of the European
Commission since 1 December 2019.
The green deal will work through a framework of regulation and legislation setting clear overarching
targets – a bloc-wide goal of net zero carbon emissions by 2050, and a 50%-55% cut in emissions
by 2030 (compared with 1990 levels) are supposed to be at the core – alongside incentives to
encourage private sector investment, with action plans for key sectors and goals such as halting
species loss, cutting waste and better use of natural resources.
While some countries like the ‗frugal four‘, consisting of the Netherlands, Denmark, Sweden and
Austria, have said they would want to fast-track their journey to achieve climate neutrality, others
like Poland have asserted it will go on its own pace.
36. Ans:(b)
Activities are climate-neutral or CO₂ neutral if they do not put a strain on the climate, i.e. if they do
not create greenhouse gases. Climate neutrality can be achieved if CO₂emissions are reduced to a
minimum and all remaining CO₂ emissions are offset with climate protection measures. This
stimulus deal is named Next Generation EU.
Ursula Gertrud von der Leyen is a German politician and the president of the European
Commission since 1 December 2019.
The green deal will work through a framework of regulation and legislation setting clear overarching
targets – a bloc-wide goal of net zero carbon emissions by 2050, and a 50%-55% cut in emissions
by 2030 (compared with 1990 levels) are supposed to be at the core – alongside incentives to
encourage private sector investment, with action plans for key sectors and goals such as halting
species loss, cutting waste and better use of natural resources.
While some countries like the ‗frugal four‘, consisting of the Netherlands, Denmark, Sweden and
Austria, have said they would want to fast-track their journey to achieve climate neutrality, others
like Poland have asserted it will go on its own pace.
37. Ans:(d)
Activities are climate-neutral or CO₂ neutral if they do not put a strain on the climate, i.e. if they do
not create greenhouse gases. Climate neutrality can be achieved if CO₂emissions are reduced to a
minimum and all remaining CO₂ emissions are offset with climate protection measures. This
stimulus deal is named Next Generation EU.
Ursula Gertrud von der Leyen is a German politician and the president of the European
Commission since 1 December 2019.
The green deal will work through a framework of regulation and legislation setting clear overarching
targets – a bloc-wide goal of net zero carbon emissions by 2050, and a 50%-55% cut in emissions
by 2030 (compared with 1990 levels) are supposed to be at the core – alongside incentives to
encourage private sector investment, with action plans for key sectors and goals such as halting
species loss, cutting waste and better use of natural resources.
While some countries like the ‗frugal four‘, consisting of the Netherlands, Denmark, Sweden and
Austria, have said they would want to fast-track their journey to achieve climate neutrality, others
like Poland have asserted it will go on its own pace.
38. Ans:(c)
Activities are climate-neutral or CO₂ neutral if they do not put a strain on the climate, i.e. if they do
not create greenhouse gases. Climate neutrality can be achieved if CO₂emissions are reduced to a
minimum and all remaining CO₂ emissions are offset with climate protection measures. This
stimulus deal is named Next Generation EU.
Ursula Gertrud von der Leyen is a German politician and the president of the European
Commission since 1 December 2019.
The green deal will work through a framework of regulation and legislation setting clear overarching
targets – a bloc-wide goal of net zero carbon emissions by 2050, and a 50%-55% cut in emissions
by 2030 (compared with 1990 levels) are supposed to be at the core – alongside incentives to
encourage private sector investment, with action plans for key sectors and goals such as halting
species loss, cutting waste and better use of natural resources.
While some countries like the ‗frugal four‘, consisting of the Netherlands, Denmark, Sweden and
Austria, have said they would want to fast-track their journey to achieve climate neutrality, others
like Poland have asserted it will go on its own pace.
39. Ans:(c)
Six major steps for the revival of the MSME sector
1. Collateral free loan of Rs 3 lakh crores for MSMEs. This will benefit 45 lakh units so that they can
resume work and save jobs.
2. For stressed MSMEs, Subordinate debt provision of Rs 20,000 cr has been announced for 2 lakh
MSMEs. It will benefit those which are NPAs or stressed MSMEs.
3. Rs 50,000 crore equity infusion through Mother fund-Daughter fund for MSMEs that are viable
but need handholding. A fund of funds with corpus of Rs 10,000 crore will be set up to help these
units expand capacity and help them list on Markets if they choose.
4. Definition of MSMEs has been revised to allow MSMEs to aim for expansion and not lose
benefits. Also, there'll be no distinction between manufacturing & services sector MSMEs.
5. Global tenders will be disallowed up to Rs 200 crore for government contracts.
6. Will ensure e-market linkages are provided across the board in the absence of non-participation
in trade fairs due to Covid. Govt of India and PSUs will clear all the receivables in next 45 days.
40. Ans:(c)
The Micro, Small and Medium Enterprises Development Act, 2006 is an Act of the Parliament of
India. According to the act, "any buyer who fails to make payment to MSMEs, as per agreed terms
or a maximum of 45 days, would be liable to pay monthly compounded interest at three times the
bank rate notified by RBI".
41. Ans:(a)
Finance Minister Nirmala Sitharaman on May 13 said that the definition of micro, small and medium
enterprises (MSME) will be changed to allow for relatively bigger companies to avail the benefits of
cheaper and priority loans.
42. Ans:(b)
An emergency credit line of ₹3 lakh crore for MSMEs from Banks and NBFCs. The credit line will be
up to 20% of all outstanding credit as on February 29, 2020. This scheme is available till October
31, 2020. There will be no guarantee free, no fresh collateral required. 45 lakh units are likely to
benefit from this move.
43. Ans:(c)
The Indian government will now disallow tenders up to ₹200 crores from foreign companies. The
government justified the decision by saying that the move is aimed at protecting MSMEs from the
unfair competition from foreign entities.
44. Ans:(c)
Kamov 226T light helicopters, manufactured in association with Russia, will be mainly used to move
troops and equipment to high-altitude locations like Siachen.
45. Ans:(a)
In October 2016, India and Russia finalised a broad agreement for the joint venture between
Hindustan Aeronautics Ltd. and two Russian defence majors for production of the helicopters in
India.
46. Ans:(a)
Coaxial rotors give a helicopter improvements in lift and payload capacity over conventional
choppers. This is especially advantageous in high-altitude environments such as the Himalayas
where an aircraft's performance at take-off tends to diminish due to the lower air density. The Indian
Navy has been using the Ka-25 and Ka-27 and also the Ka-31 airborne early-warning helicopter for
several years.
47. Ans:(d)
The 11th edition of the biennial mega defence exhibition, DefExpo 2020 was held in the capital city
of Uttar Pradesh, Lucknow from 5th February to 9th February 2020.
48. Ans:(b)
A private Indian firm has recently unveiled the indigenous air to ground missile ‗Khagantak‘, during
the DefExpo 2020
49. Ans:(c)
India‘s ranking in the Corruption Perceptions Index (CPI-2019) has slipped from 78 to 80 compared
to the year 2018.
The 2019 CPI draws on 13 surveys and expert assessments to measure public sector corruption in
180 countries and territories, giving each a score from zero (highly corrupt) to 100 (very clean).
50. Ans:(c)
Transparency International is a non-profit, non-governmental organisation dedicated to fighting
corruption. It was founded in 1993 and is based in Berlin, Germany.
51. Ans:(b)
First launched in 1995 by the Transparency International, the Index has been widely credited with
putting the issue of corruption on the international policy agenda.
52. Ans:(a)
New Zealand and Denmark, with scores of 87 each came on top, followed by Finland (86),
Singapore (85), Sweden (85) and Switzerland (85).
53. Ans:(c)
Transparency International has recommended a series of measures to combat rising corruption
across the world. Following are the recommendations:
Manage conflicts of interest.
Control political financing.
Strengthen electoral integrity.
Regulate lobbying activities.
Empower citizens.
Tackle preferential treatment.
Reinforce checks and balances.
54 Ans:(c)
Black carbon concentrations near the Gangotri glacier rose 400 times in summer due to forest fires
and stubble burning from agricultural waste, and triggered glacial melt, says a study by scientists at
the Wadia Institute of Himalayan Geology (WIHG).
55. Ans:(d)
Unlike CO2, which can stay in the atmosphere for years together, black carbon is short-lived and
remains in the atmosphere only for days to weeks before it descends as rain or snow.
56. Ans:(b)
India is the second largest emitter of black carbon in the world, with emissions expected to increase
dramatically in the coming decades, says an April 2019 study in the journal Atmospheric Research,
with the Indo Gangetic plains said to be the largest contributor
57. Ans:(b)
Gangotri Glacier is located in Uttarkashi District, Uttarakhand, India in a region bordering Tibet. This
glacier, one of the primary sources of the Ganges, is one of the largest in the Himalayas with an
estimated volume of over 27 cubic kilometers. The glacier is about 30 kilometres long and 2 to 4 km
wide.

58. Ans:(a)
Black carbon results from the incomplete combustion of fossil fuels and biomass. The fine particles
absorb light and about a million times more energy than carbon dioxide. It is said to be the second
largest contributor to climate change after CO2. But unlike CO2, which can stay in the atmosphere
for years together, black carbon is short-lived and remains in the atmosphere only for days to
weeks, before it descends as rain or snow.
59. Ans:(a)
The online NAM Contact Group Summit on the theme of ―United against COVID-19‖ was hosted by
the current Chairman of NAM, the President of Azerbaijan.
60. Ans:(d)
Ilham Heydar oglu Aliyev is an Azerbaijani politician who is the fourth president of Azerbaijan, in
office since 2003. He also functions as the Chairman of the New Azerbaijan Party, Non Aligned
Movement and the head of Azerbaijan's National Olympic Committee.
61. Ans:(a)
The NAM Contact Group Summit also commemorated the International Day of Multilateralism and
Diplomacy for Peace. The International Day of Multilateralism and Diplomacy for Peace observed
globally on 24 April. ‗International Day of Multilateralism and Diplomacy for Peace‘ was first
observed by the United Nations (UN) on April 24, 2019.
This day was established by the General Assembly of the United Nations, following the adoption of
resolution A/RES/73/127 of December 19, 2018 and NAM Member States played an important
initiative in this regard. Resolution 73/127 reaffirmed the Charter of the United Nations and its
purposes and principles, and especially the commitment to settle disputes through peaceful means
and the determination to save succeeding generations from the scourge of war. The Resolution
reaffirmed that the approach of multilateralism and diplomacy could reinforce the advancement of
the three pillars of the United Nations, namely, sustainable development, peace and security and
human rights, which are interlinked and mutually reinforcing, while observing respective mandates
and the Charter.
62. Ans:(b)
Six years after Bandung, the NAM was officially established, at the First Summit Conference of
Belgrade, 1961.

63. Ans:(a)
Today NAM is a forum of 120 developing world states that are not formally aligned with or against
any major power bloc. It has 17 observer countries and 10 observer organisations. It is the second
organisation after the UN with this big a membership.
64. Ans:(a)
UN moved to the permanent Headquarters of the United Nations in New York City at the start of its
seventh regular annual session, on 14 October 1952. This year‘s session was also supposed to
happen at the New York City.
65. Ans:(c)
Volkan Bozkır (born 22 November 1950) is a Turkish diplomat and politician who served as the
Minister of European Union Affairs from November 2015 to May 2016, having previously served
from August 2014 to August 2015. He concurrently served as the Chief Negotiator for Turkish
Accession to the European Union during the same time. Turkish diplomat Volkan Bozkir was
elected President of the 75th session of the UN General Assembly replacing Tijjani Muhammad-
Bande, the General Assembly previous President.
66. Ans:(b)
This will be the 75th session of the General Assembly, opening on Tuesday 15 September 2020,
will be presided over by H.E. Mr. Volkan Bozkir. Every year in September, all the Members meet in
this unique forum at Headquarters in New York.
67. Ans:(a)
The secretary-general of the United Nations (UNSG or just SG) is the head of the United Nations
Secretariat, one of the six principal organs of the United Nations. The secretary-general serves as
the chief administrative officer of the United Nations. The role of the United Nations Secretariat, and
of the secretary-general in particular, is laid out by Chapter XV (Articles 97 to 101) of the United
Nations Charter. As of 2020, the secretary-general is former Prime Minister António Guterres of
Portugal, appointed by the General Assembly on 13 October 2016.
68. Ans:(b)
The new Permanent Representative of India to the United Nations, T. S. Tirumurti. Prior to his
appointment, Mr. Tirumurti served as Secretary to India‘s Ministry of External Affairs beginning in
2018, in charge of the Persian Gulf, Arab world, Africa, development partnerships and economic
relations. He previously served in his country‘s embassy in Cairo, Egypt.

QUANTITATIVE TECHNIQUES
69. Ans:(c)
School A:
B+G = 10035
B-G = 373
Girls = 4831
In School B:
B+G = 10098
B-G = 640
G = 4729
Difference = 102
70. Ans:(c)
B+G = 10087
B-G = 285
G = 4901
Required % = (4901/10087)x 100%= 48.58%
71. Ans:(a)
School D:
B+G = 10081
B-G = 475B = 5278 ;
G = 4803
In School E:
B+G = 10009
B-G = 625B = 5317 ;
G = 4692
Ratio = (5278+4692):(4803+5317)= 9970:10120= 997:1012
72. Ans:(b)
B+G = 10045
B-G = 423
Boys = 5234
73. Ans:(d)
From above calculations:
School A: 4831
School B: 4729
School C: 4901
School D: 4803
School E: 4692
74. Ans:(a)

Participating in at least one house = 242 + 90 + 150 + 18 + 18 + 42 + 240 = 800


Who did not participating in any house = 2000 – 800 = 1200
Percentage of student = (1200 /2000 ) x 100 = 60%

75. Ans:(b)
Student who participate in only one house = 242 + 240 + 18 = 500
Percentage = (500/2000) x 100 = 25%
76. Ans:(d)

Student who participate in only Dalton house = 18


Who participate in only Raman and Dalton House = 90
Required ratio = 1 : 5
77. Ans:(a)

Student who participate in only one house = 242 + 240 + 18 = 500


78. Ans:(a)

Student who participate in all three houses = 150


Student who participate in two house = 90 + 42 + 18 = 150
Required percentage = (150 / 150) x 100 = 100%
79. Ans:(c)
Rice imported by India in 2017
= 120100×120=144 MT
Tea imported by India in 2017
= 1442×1=72 MT
Export of India in 2017 = 5000 +240= 5240 MT
Import of India in 2016 = 120 + 7240×100=300
Required sum = 5240 +300 = 5540
80. Ans:(a)
Total import of Japan in 2016
= 30100×2735=820.5 MT
Let import of Rice in 2016 in 2016 is ‗x‘ MT
So import of Tea in 2016  820.5 – x MT
ATQ,
180×x100×(820.5−x)=360347
X = 300
Rice imported by Japan in 2017 = 180100×300=540

81. Ans:(c)

82. Ans:(d)
Total export by US = 3860 MT
Total export by Brazil = 4210 MT
Required % = 4210− 38604210×100
= 3504210×100 = 8% (approx)
LOGICAL REASONING

83. Ans:(c)
This is a main idea question, so look to your scope and purpose. The second and third sentences
of paragraph 1 indicate that the unique identity of the Gullah cult is attributable to the combined
influences of African and U.S. elements.
Paragraphs 2 and 3 go on to consider both African and U.S. aspects of beliefs and practices.
Choice (a) focuses on a detail, a topic discussed only in paragraph 3.
Choice (b) is beyond the scope of the passage. The author never claims that ―heir‘s land‖ is the key
to Gullah culture.
Choice (d) hits on another detail.

84. Ans:(d)
For this question, go through each answer choice one at a time, looking for the place in the
passage that supports the relevant detail. Paragraph 3 is built on a contrast between elements of
Gullah religion that ―bespeak a U.S. heritage‖ and those that are African in origin. Immediately
following the statement that Gullah religious practices at least superficially reflect a U.S. heritage,
the author cites the role of Baptist and Methodist churches as ―essential social institutions.‖
Inferably, then, the social role of the churches is an American, not an African, element of the
culture. All the other choices are African elements in Gullah culture. The first sentence of paragraph
2 offers the extended family [choice (a)] as an example of West African influence on Gullah social
life. In paragraph 3, we‘re told that the Gullah conception of the ―body spirit‖ is similar to a West
African belief [choice (b)] As for choice (c), we‘re told in paragraph 2 that membership in the
extended family—itself an African element—―affects property rights;‖ the system of ―heir‘s land‖ is
cited as an example. While it‘s not clear that ―heir‘sland‖ existed in Africa, the method of acquiring
land was indirectly influenced by the African institution of the extended family.
85. Ans:(c)
Here‘s another inference question. The author states that ―heir‘s land‖ is a way for family members
to acquire land through an ―unwritten contract,‖ without paying for it. Any contract is by its nature an
agreement, and since this kind of agreement is not on paper, it follows that it is orally
communicated. Choice (a) is way beyond the scope of this passage. The only government grant
referred to is the original grant of the Sea Islands, after the Civil War. It‘s not implied that this said
anything about land tenure or individual acquisition of land.
Choice (b) distorts the passage by linking ―heir‘s land,‖ which operates within families, to marriage
agreements between families. It‘s never suggested that ―heir‘s land‖ has anything to do with
marriage.
The fact that ―heir‘s land‖ is defined as an unwritten contract eliminates choice (d).
86. Ans:(d)
In this application question, look for a social experience that parallels one described, probably in
more general terms, in the passage. The second paragraph states the importance of family ties and
illustrates this by discussing how family members can acquire land through these ties. This is
similar to acquiring a job, or any other benefit, through family connections. (Note that you don‘t
need to find evidence that the Sea Islanders get jobs in this way—the point is that choice (d) is
logically similar to an experience of the Sea Islanders, not that they might have this exact
experience.) Choice (a) is out, primarily because of its reference to court supervision. The only
reference to property (the third and fourth sentences of paragraph b) emphasizes exchanges within
the family with no such supervision. More generally, courts and other government institutions are
not mentioned anywhere in the passage. Choice (b) is certainly not like anything in traditional
Gullah life; it might be like an experience that has occurred since the ―encroachments of tourism
and commercialization,‖ except that we‘re never told what these encroachments have been. Choice
(c) relates to the importance of family units, but it is the extended family that is important to the
Gullah, whereas choice (c) involves unrelated adults.
87.
Ans:(a)
The author is relatively objective, but still favorable to Gullah culture, calling it ―unique‖ and ―vital,‖
and its heritage ―strong.‖ This makes the positive, but restrained, terms in choice (a) accurate.
Choice (b) describes a hesitant, wavering approach, one that would be characterized by mixed
feelings or conflicting ideas—this simply doesn‘t fit the author‘s approach. Given the relative
objectivity just mentioned, ―detached‖ in choice (c) is not too far off, but ―quizzical‖ (puzzled,
bemused) is totally out.
Choice (d) is the opposite of what you are looking for.
88. Ans:(d)
Jones-Jackson‘s views on the African influences on Gullah religious services appear in quotes in
the final paragraph. The ―classical, Ciceronian rhetorical style‖ and ―sophistic ornaments‖—
elements of the style of Gullah prayers and sermons—inspire the congregation to make ―raucous
and joyous replies.‖ Choice (4) focuses on this latter aspect.
Choice (a) misinterprets the mention of ―classical . . . style,‖ which refers to stylistic elements of the
prayers and sermons, not to any type of music, which is never mentioned.
Choice (b) mistakes Africa as an influence on Gullah services for Africa as a topic of the sermons;
sermon topics are never mentioned.
Choice (c) refers to a detail mentioned in paragraph 2; it is not clear what, if anything, this has to do
with religious services.
89. Ans:(c)
Statement 2 is mentioned in the passage, but not in the context as vulnerability. Statement 3 is not
mentioned in the passage. Statements 1 and 4 are mentioned in the last paragraph in the context of
vulnerability.
90. Ans:(d)
Low-carbon growth is mentioned in the passage in terms of energy consumption. None of the
options implies low-carbon growth. So, option (d) is correct.
91. Ans:(d)
Statement 1 is correct as it is directly mentioned in the passage. Statement 3 also finds support in
first paragraph of the passage. It is also mentioned that adopting and spreading technologies is
necessary for sustainable economic growth. So, all the statements are correct. Hence, option (d) is
the correct answer.
92. Ans:(d)
Statement 1 is opposite of what the passage says. Statement 2 cannot be inferred either because it
can be inferred that in water deficient areas adaptive methods should be adopted rather than giving
up farming entirely in such areas.
93. Ans:(d)
Assumption 1 finds support in the second paragraph as 'much of humankind's creative potential has
been directed at adapting to the changing world.' The examples of cotton cultivation plan of Soviet
Union and mangroves of Guinea give support to assumption 2. It is mentioned in the passage that
'the spread of economic prosperity has always been intertwined with ecological conditions.' So,
assumption 3 is also correct. Hence, option (d) is the correct answer.
94. Ans:(b)
Options (a), (c) and (d) are important, but these are not the central theme of the passage. The
second paragraph talks about the changing world and sustaining growth. So, option (b) is correct.
95. Ans:(a)
Explanation: Peter Parker says ―The less we depend on humans the lesser is the chance of
accidents.‖ So, option (a) goes perfectly with that logic. If we use robots instead of humans, then
there will be less chances of problems in the surgery. The success rates will be higher.
Peter Parker doesn‘t say that human-beings‘ intelligence has reduced due to use of technology. So
the option (b) doesn‘t match with his views. So neither option (b) nor (c) is the answer.
If Peter Parker would have said something like due to use of auto-pilot our pilots have got their IQ
levels down, then option (c) would have made sense. What Peter Parker means is that since
aeroplanes use more technology than roadways or railways, it actually helps the transportation
mode by making it safer. So Option (c) is not the answer.
So option (a) is the answer.
96. Ans:(c)
Explanation: Option (a) will strengthen the arguments of Peter Parker. He wants to prove that
airways are much safer than other modes of transport. Highly-trained pilots will add on to the safety
factor. So options (a) can‘t be the answer.
Option (b) is a hiccup that can be faced, i.e., air-pockets can be managed. It is not a very big
problem.
Option (c) puts in doubt Peter Parker‘s argument that airplanes use higher technology. It is a much
bigger problem than option (b). Peter Parker was depending on technology in his arguments, so if
his technology argument gets neutralized, he won‘t have much to defend his point. So option (c) is
the answer.
The food doesn‘t add anything to the safety factor. Hence, option (d) is not the answer
97. Ans:(c)
Explanation: Bruce Wayne is not saying that road-travel or rail-travel is safer than air-travel. He has
not countered all the arguments of Peter Parker. He is merely putting apprehensions on a part of
the argument of Peter Parker.
Thus option (a) which says that air travel is safer than road or rail travel is not correct.
Option (b) is not correct as Bruce Wayne has not pressed his case that air travel is unsafe as
compared to air or road travel. He barely gives any point of advantage for the roadways or railways.
Option (c) is much more suitable as Bruce Wayne just argues off one of the safety factors given by
Peter Parker. Bruce Wayne isn‘t sure about whether roadways or railways are safer than airways,
but airways can‘t be said to be safer than the two.
Thus, option (c) is correct.
98. Ans:(d)
Explanation: Options (a) proves Peter Parker‘s points that airways are safer, thus it doesn‘t bolster
Bruce Wayne‘s arguments.
Option (b) strengthens the argument of Peter Parker. It adds to the ―fact‖ that roadways are not
safer than airways. So option (b) can‘t be the answer.
So, neither statements strengthen Bruce Wayne‘s argument, hence, the answer will be option (d).
99. Ans:(c)
Explanation: Peter Parker believes that the less we depend on humans the lesser is the chance of
accidents. Based on this premise he establishes the argument that the auto-pilot technology
employed in a plane gives airplanes a huge advantage than rail or road transport to conclude that
airplanes are much safer. His conclusion is ―I feel that airplanes are much safer.‖
100. Ans:(d)
Explanation: Answer (D). Option 1 can be eliminated on the basis of its vagueness. The passage
talks about the beliefs that held by neuroscientists almost a century ago, while the option talks
about the beliefs of neurologists. Option 2 can be eliminated since it is extreme.
101. Ans:(d)
Explanation: Answer (D). Options 1, 2 ,3 talk about author‘s beliefs without stating why those beliefs
are wrong.
102. Ans:(d)
Explanation: Answer (D).
Option 1, 2, 3 provide evidence about successful application of the theory about hypnopaedia.
Option 4 is the correct answer.

103. Ans:(c)
Explanation: Answer (C). Refer to, ―The Psycho-Phone operated on the premise of an unproven
theory. There, recorded messages are used to train sleeping children in the values of a soulless
future society. A proud official in the book calls the new method, dubbed ‗hypnopaedia‘ by Huxley,
the ‗greatest moralising and socialising force of all time‘‖
104. Ans:(c)
Explanation: Answer (C). Option 1 is irrelevant as it fails to state the premise based on which
psycho phone operate. Option 2 is vague. Option 3 is the correct answer.
105. Ans:(b)
From the table, it is evident that Milk is drunk by Englishman. The correct choice is (b).
106. Ans:(b)

Table reveals that Hindi expert drinks coca-cola. The correct choice is (b).
107. Ans:(a)

The color of Hindi expert‘s house is yellow. The correct choice is (a).
108. Ans:(d)
Clearly option (d) gives the wrong information since French expert does not live next to Spanish
expert. The correct choice is (d).
109. Ans:(c)

110. Ans:(b)
Explanation:
The pattern of given series is :
5
14 = (5x3) - 1
45 = (14x3) + 3
130 = (45x3) - 5
(?) = (130x3) + 7 => (?) = 397
Thus, the missing number is 397
111. Ans:(d)
Explanation:
From all these three statements the gender of M can‘t be determined. Hence, we can‘t find out how
M is related to R.
LEGAL REASONING

112. Ans:(d)
[The agreement is void since both the parties were under the mistake as to a matter of fact
essential to the agreement.]
113. Ans:(b)
[The government‘s agent was deceived by the conduct of Ron and Hillary as to the person with
whom he was dealing, and hence no valid consent. The act of brothers prevented the real consent.
It means that an offer which is meant for one person cannot be accepted by another.]
114. Ans:(c)
[When anyone makes a contract in which the personality of the particular party contracted with is
important than no one else is at liberty to step in and maintain that he is the party contracted with.]
115. Ans:(c)
[Since no offer was made to North, it was made to Sir George Bullough, and consequently there
could be no contract with him. His right to ring was no more than that of a thief or a finder and he
could not convey a good title to Shyam.]
116. Ans:(b)
[There was no contract between Thomas and Blenkarn and, therefore he had no right to sell the
goods. Thomas intended to contract with Blenkiron & Co. and consequently no contract could have
arisen between him and Blenkarn.]
117. Ans:(a)
Ex: Kunjbihari was not aware of the disease having recovered. He was negligent and spread the
infection further. There is no malicious intention however and thus he shall be liable under Section
269 of IPC.
118. Ans:(b)
Ex: The bribing of police personnel to avoid screening despite suspecting positive report shows that
he had an intention of spreading the infection to multiple people in Delhi. Due to the presence of
malicious intention, he will be liable under Section 270 of IPC.
119. Ans:(d)
Ex: The passage states that Section 271 is non-bailable. The principle states that in the case of
non-bailable offences, bail cannot be sought as a right and is only discretionary.
120. Ans:(a)
Ex: The passage clearly states that there is a ban on the use of corporal force. In these facts there
is no corporal force involved and thus such punishments being given are justified by the police
personnel.
121. Ans:(b)
Ex: The passage clearly states that there is a ban on the use of corporal force. In these facts
however, there is corporal force being used and thus such punishments are not justified.
122. Ans:(a)
Ex: Watching cherry blossoms (even though a very very rare) occasion cannot be stated to be a
reasonable excuse at this time to not comply with the directions of a public servant.
123. Ans:(c)
Ex: The order of the NGT banning all construction is sufficient to constitute circumstances beyond
the control of JBM. They can take advantage of a force majeure clause.
124. Ans:(b)
Ex: The ban order was only on construction activities and the procurement of raw materials was not
prohibited by the NGT. Thus, JBM could have taken steps to mitigate the losses by timely
procurement of raw materials. They are only entitled to 2 months.
125. Ans:(d)
The passage clearly states that mere difficulty to perform is not enough to take the defence of force
majeure. There was availability of labourers, it was just that their services were more expensive
now.
126. Ans:(a)
Non-availability of any labourers in the city cannot be amounted to mere difficulty to perform. It did
constitute circumstances beyond the control of SUPERNOVA and constitutes force majeure
127. Ans:(b)
Ex: The passage states that mere difficulty to perform may constitute force majeure in case all
others situated in the same position as the plaintiff regarded the difficulty as an extraordinary
circumstance.
128. Ans:(d)
Ex: It needs to be found out whether all entities similarly placed are finding it significantly
inconvenient to perform the contract. Unlike the earlier question, the Local Dam Builders
Association of New Jersey is not situated in the same position as BB and thus BB cannot take
benefit of the same.
129. Ans:(d)
An individual‘s right to privacy is intrinsically a part of Article 21. Information regarding such
diseases also fall within that ambit and unless there is a compelling case to disclose the
information, it should not be done. In the present case a compelling case did not exist.
130. Ans:(d)
There has been a breach of privacy of all customers whose data has been exchanged. This is
clearly laid down in the passage that the information shall be used only for the purposes it is
provided for and consent shall be taken before tampering such information. Thus this amounts to a
breach of privacy.
131. Ans:(c)
The proportionality test provides for in the third limb that the measure should be least intrusive in
nature. However, publishing the name of individuals etc. can in no manner be considered as a least
intrusive measure as alternate measures could have been looked into for effective enforcement.
132. Ans:(a)
The information could only be accessed through the CLET database as can be reasonably
concluded by the given facts. (b) & (d) are not applicable as we do not know whether CLET is a
state entity and whether universities and coaching‘s in the limited information given in the passage
can be held liable or not.
133. Ans:(a)
In Mr. X v. Hospital Z the exact facts existed and the Court held that right to a healthy life will
outweigh the right to privacy.
134. Ans:(b)
The doctor while prescribing a wrong drug to a pregnant women breach his duty of care towards
her. Therefore the damage to the child in terms of physical disability will be an account of the doctor
and he will be liable for medical negligence.
135. Ans:(c)
―It was held in Calcutta Medical Research Institute v. Bimalesh Chatterjee that the onus of proving
negligence and the resultant deficiency in service was clearly on the complainant.‖ Here Mr. Batra
is a complainant (plaintiff) and onus of proving doctor and hospital‘s guilt lies with him.
136. Ans:(c)
Dr. Knight was supposed to discuss the possible outcomes of the surgery and also inform Mrs.
Menon about the level of risk involved. Such deeds form a part of duty, a doctor owes towards his
patients. Therefore Dr Knight will be held liable.
137. Ans:(b)
doctrine of vicarious liability will be applied here. As the act was done in the course of employment,
master (hospital) is liable for the act of its employees (Dr Umang and Dr Samara).
138. Ans:(c)
IV and V doesn‘t qualify as medical negligence.
139. Ans:(d)
Catching an infection which is extremely contagious cannot be regarded as a negligence by the
hospital. Also, facts are silent about any failed deed on the part of the hospital.
140. Ans:(b)
The children were of her class, hence couldn‘t be older than 12 as she was of 7. Also they did not
understand the nature of their activity as they were just playing as kids.
141. Ans:(c)
Epileptic fits are a serious mental condition that affect a person physically as well and hence can be
claimed against any such accident if they end in a criminal offence.
142. Ans:(b)
Regardless of his intent or ill-will, since he was under the age of 7 he cannot be convicted for any
offence.
143. Ans:(b)
Insanity is a medical or a clinical situation of the mind which the person has no control over. Rage
with intention is not a ground of insanity.
144. Ans:(c)
He was above the age of 12 with complete understanding of his actions and can be held liable for
criminal misappropriation.
145. Ans:(b)
Since anybody over the age of 14 can be held liable for an offence, she cannot claim lack of
understanding of laws, but kleptomania is a mental condition and she can use insanity as a
defense.
146. Ans:(c)
Explanation: The answer is (c). This is for the reason that the State can put a reasonable restriction
based on the aspect of interest of the public. Herein, no public interest has been violated thereby
making the restriction unreasonable.

147. Ans:(a)
Explanation: The answer is (a). This is because Article 25 provides individuals the freedom to
practice their religion subject to certain restrictions. This can include not being a part of the any
process if the religion demands so. Therefore, in the instant case it is not unreasonable of the three
students to not be a part of the process opted by the school.
148. Ans:(c)
Explanation: The answer is (c). This is because the no person in the name of religion can do any
act that is opposed to the public policy. Therefore, converting people out of force or allurement
disallows the converted people to practice their own religion making it against public policy. Also,
the fundamental right of one does not come at the cost of another.
149. Ans:(b)
Explanation: The answer is (B). This is because the restriction has only been on prayer offerings in
early and late hours due to disturbance to the nearby residents. At such instance, if the method of
the practice is regulated then it does not take away the right to practice such religion from the
community.
150. Ans:(c)
Explanation: The answer is (c). This is because Momina has a right to religion which allows her to
freely practice it even when it is in the form of wearing a headscarf for religious purposes. This
forms the external manifestations of the religion which are equally justifiable.
Legal Edge 05 Mock

Answers & Details

ENGLISH

1. Ans:(a)
Plato was of the view that the stories should aid in character building and also should provide good
role models and not lie. According to him Homer‟s writing are lie.
2. Ans:(b)
According to Platonic yardsticks, both the pairs are in the same boat as their work is not grounded
in reality and reason.
3. Ans:(b)
It is evident in the passage that Plato supports the State‟s guidelines for the artists and thinks that
the artists should follow these guidelines.
4. Ans:(a)
Exhortation means insistence. Hence, A is the correct answer.
5. Ans:(a)
Plato argues that this poet are imitators of virtue and are very far away from the reality. Hence, A is
the correct answer.
6. Ans:(c)
Ambedkar‟s criticism of Gandhi does not stem from hatred of Gandhi as a political opponent but
from his logical analysis and his disagreement over philosophical views.
7. Ans:(d)
All the three points highlight the disagreement of Ambedkar over the economic ideals of Gandhi.
Hence, D is the correct answer
8. Ans:(b)
Vaishyas under the varna system were the traders. Hence, B is the correct answer.
9. Ans:(b)
The Passage as a whole is Ambedkar‟s criticism of Gandhi‟s political, social as well as economic
ideals not based on Ambedkar‟s political rivalry with Gandhi but based on the rational analysis of
Gandhi‟s philosophy.
10. Ans:(d)
This Speaker of the line is Gandhi, “These being my views I am opposed to all those who are out to
destroy the caste system.” The lines suggest the firm decision on the part of Gandhi. Hence, D is
the correct answer.
11. Ans:(b)
Within two decades of liberalization, India has the lowest tariff rates in the world and the second
largest number of mobile phones in the world shows how successful telecom sector became.
Hence, B is the correct answer.
12. Ans:(a)
Scornful means feeling or expressing contempt or derision. Here Vidura was disgusted with the act
of Bhisma and hence expressed his contempt over the matter. Hence, A is the correct answer.
13. Ans:(b)
The author points that Singh had objected the stance of A. Raja earlier, however, did not follow up
on his stance and hence he was not determined. It leads to the conclusion that he himself is
intelligent and has integrity but he is not determined and hence failed leader. Hence, B is the
correct answer.
14. Ans:(a)
In the context of passage, the word driblet means „cheap‟. Hence, A is the correct answer.
15. Ans:(b)
Author has compared the situation in which both Singh and Bhisma choose to remain silent. Hence,
B is the correct answer.
16. Ans:(c)
Transmogrification means the act or process of being transformed into a different form. The
example in the passage are Egyptian sun disk became the halos of Catholic saints. Pictograms of
Isis nursing her miraculously conceived son Horus became the blueprint for our modern images of
the Virgin Mary nursing Baby Jesus. Hence, C is the correct answer.
17. Ans:(d)
Jesus has been a prophet whose influence was staggering. According to the passage he toppled
the kings, inspired millions of people and founded new philosophies. Hence, D is the correct
answer.
18. Ans:(a)
It was the acumen of Constantine that he realised the advent of Christianity and converted the
pagan religion into Christianity. Hence, A is the correct answe
19. Ans:(c)
The tone of the response from Sophie is puzzled.
20. Ans:(c)
Enigmatic means puzzling or inexplicable. Hence opposite is simple.
21. Ans:(d)
The boy in this passage has stressed on how the sheep used to trust him.
(a) The sheep did have a monotonous life. They went about their daily routine regularly, but that is
not what the boy is concerned about.
(b) The sheep did have minimalistic wants, regular feed and that‟s all. It‟s a part of what the boy is
thinking but not what he highlights.
(c) The sheep were naïve, but the boy was not much perturbed about their naivety.
(d) The sheep were trusting in nature. They trusted the boy and would even do that for a long time
even if turned into a monster. This is thus the correct answer.
22. Ans:(c)
The boy is having a mixed view about the sheep and he doesn‟t really have one single thought that
he can associate with them. He talks about their simple, monotonous and trusting lives. He then
starts associating them with themselves. Thus, the boy is just being reflective about the lives of the
sheep. Hence, the answer is option (c).
23. Ans:(c)
(a) The boy is thinking about the sheep but he is not concerned about them. He is more concerned
about himself. He thinks if he transforms into a monster, then too, the ship would trust him.
However, he then admonishes himself for the ungodly thought. So, it is himself that he is worried
about.
(b) Though he thinks it is the sycamore that is causing changes to his attitude, but he is not
concerned or worried about them.
(c) This is correct. Please see the above given explanation.
24. Ans:(c)
In the italicized line, „seamingly‟ is the wrong spelling. It should be „seemingly‟, which means
„apparently‟. Thus the answer is option (c).
25. Ans:(c)
“If I became a monster today” – In this line, the boy is substituting the concept of turning into a cruel
person with another term. This is metonymy. Thus the answer is option (c).
26. Ans:(c)
Err is to make error or mistake and fallible means someone who is likely to make error. Option c) is
the correct answer. Immaculate means clean or without error; impeccable means faultless or
errorless; and unimpeachable means someone or something that cannot be doubted or questioned.
.
27. Ans:(d)
In the third paragraph, the author argues that the report of the think tank commits error when it
argues for more investment in regulatory data generation and the government ownership of data.
Hence, option d) is the correct answer. Option a) and b) are just opposite of what report is
asserting. Option c) is also not correct because the report argues that data generation strengthens
the fight against air pollution but never claims that it can stop air pollution.

28. Ans:(d)
The author‟s main point is that everybody should have access to data and there should not be
gatekeepers that create a bottleneck at the gateway to the data. Hence, option d) is the correct
answer. Option a) can be a way to achieve democratization of data. Option b) might be an
argument in favour of data democratization. Option c) is beyond the scope of the passage.
29. Ans:(b)
This is the whole argument of the passage that if data is easily available to public, it can become a
shield against air pollution. Moreover, Whitefield case is the example given by the author to
emphasize the availability of data. Option a) contradicts the argument of the author that data can do
wonders if it is easily available. Option c) and d) are not touched upon by the author.
30. Ans:(a)
The main intention of the author behind giving this example is to show how monitors were shut
down, in order to avoid the real data being revealed, to serve the political purpose. Hence, it can be
said that Science is often pressed into the service of political agenda. Further, b), c) and d) are not
dealt with by the passage.
31. Ans:(b)
A new NEP usually comes along every few decades. India has had three to date. The first came in
1968 and the second in 1986, under Indira Gandhi and Rajiv Gandhi respectively; the NEP of 1986
was revised in 1992 when P V Narasimha Rao was Prime Minister. The third is the NEP 2020
released under the Prime Ministership of Narendra Modi. The incumbent government has set a
target of 2040 to implement the entire policy. NEP did not recommend to remove the PhD
32. Ans:(c)
A new NEP usually comes along every few decades. India has had three to date. The first came in
1968 and the second in 1986, under Indira Gandhi and Rajiv Gandhi respectively; the NEP of 1986
was revised in 1992 when P V Narasimha Rao was Prime Minister. The third is the NEP 2020
released under the Prime Ministership of Narendra Modi. The incumbent government has set a
target of 2040 to implement the entire policy. NEP did not recommend to remove the PhD.
33. Ans:(d)
A new NEP usually comes along every few decades. India has had three to date. The first came in
1968 and the second in 1986, under Indira Gandhi and Rajiv Gandhi respectively; the NEP of 1986
was revised in 1992 when P V Narasimha Rao was Prime Minister. The third is the NEP 2020
released under the Prime Ministership of Narendra Modi. The incumbent government has set a
target of 2040 to implement the entire policy. NEP did not recommend to remove the PhD.
34. Ans:(d)
A new NEP usually comes along every few decades. India has had three to date. The first came in
1968 and the second in 1986, under Indira Gandhi and Rajiv Gandhi respectively; the NEP of 1986
was revised in 1992 when P V Narasimha Rao was Prime Minister. The third is the NEP 2020
released under the Prime Ministership of Narendra Modi. The incumbent government has set a
target of 2040 to implement the entire policy. NEP did not recommend to remove the PhD.
35. Ans:(c)
A new NEP usually comes along every few decades. India has had three to date. The first came in
1968 and the second in 1986, under Indira Gandhi and Rajiv Gandhi respectively; the NEP of 1986
was revised in 1992 when P V Narasimha Rao was Prime Minister. The third is the NEP 2020
released under the Prime Ministership of Narendra Modi. The incumbent government has set a
target of 2040 to implement the entire policy. NEP did not recommend to remove the PhD
36. Ans:(a)
The NSCN (I-M) has demanded that Nagaland Governor RN Ravi should be removed as the
interlocutor for talks on the Naga accord. Greater Nagalim (sovereign statehood) i.e. redrawing of
boundaries to bring all Naga-inhabited areas in the Northeast under one administrative umbrella.
It includes various parts of Arunachal Pradesh, Manipur, Assam and Myanmar as well Naga
Yezabo (Naga Constitution) Naga national flag. The earliest sign of Naga resistance dates back to
1918, with the formation of the Naga Club. In 1929, the Club told the Simon Commission to leave
them alone to determine for themselves as in ancient times. In 1946, A Z Phizo formed the Naga
National Council (NNC), which declared Naga independence on August 14, 1947.
37. Ans:(d)
The NSCN (I-M) has demanded that Nagaland Governor RN Ravi should be removed as the
interlocutor for talks on the Naga accord. Greater Nagalim (sovereign statehood) i.e. redrawing of
boundaries to bring all Naga-inhabited areas in the Northeast under one administrative umbrella.
It includes various parts of Arunachal Pradesh, Manipur, Assam and Myanmar as well Naga
Yezabo (Naga Constitution) Naga national flag. The earliest sign of Naga resistance dates back to
1918, with the formation of the Naga Club. In 1929, the Club told the Simon Commission to leave
them alone to determine for themselves as in ancient times. In 1946, A Z Phizo formed the Naga
National Council (NNC), which declared Naga independence on August 14, 1947.
38. Ans:(a)
The NSCN (I-M) has demanded that Nagaland Governor RN Ravi should be removed as the
interlocutor for talks on the Naga accord. Greater Nagalim (sovereign statehood) i.e. redrawing of
boundaries to bring all Naga-inhabited areas in the Northeast under one administrative umbrella.
It includes various parts of Arunachal Pradesh, Manipur, Assam and Myanmar as well Naga
Yezabo (Naga Constitution) Naga national flag. The earliest sign of Naga resistance dates back to
1918, with the formation of the Naga Club. In 1929, the Club told the Simon Commission to leave
them alone to determine for themselves as in ancient times. In 1946, A Z Phizo formed the Naga
National Council (NNC), which declared Naga independence on August 14, 1947.
39. Ans:(c)
The NSCN (I-M) has demanded that Nagaland Governor RN Ravi should be removed as the
interlocutor for talks on the Naga accord. Greater Nagalim (sovereign statehood) i.e. redrawing of
boundaries to bring all Naga-inhabited areas in the Northeast under one administrative umbrella.
It includes various parts of Arunachal Pradesh, Manipur, Assam and Myanmar as well Naga
Yezabo (Naga Constitution) Naga national flag. The earliest sign of Naga resistance dates back to
1918, with the formation of the Naga Club. In 1929, the Club told the Simon Commission to leave
them alone to determine for themselves as in ancient times. In 1946, A Z Phizo formed the Naga
National Council (NNC), which declared Naga independence on August 14, 1947.
40. Ans:(a)
The NSCN (I-M) has demanded that Nagaland Governor RN Ravi should be removed as the
interlocutor for talks on the Naga accord. Greater Nagalim (sovereign statehood) i.e. redrawing of
boundaries to bring all Naga-inhabited areas in the Northeast under one administrative umbrella.
It includes various parts of Arunachal Pradesh, Manipur, Assam and Myanmar as well Naga
Yezabo (Naga Constitution) Naga national flag. The earliest sign of Naga resistance dates back to
1918, with the formation of the Naga Club. In 1929, the Club told the Simon Commission to leave
them alone to determine for themselves as in ancient times. In 1946, A Z Phizo formed the Naga
National Council (NNC), which declared Naga independence on August 14, 1947.
41. Ans:(a)
Border clashes in July killed at least 16 people, prompting the largest demonstration in years in the
Azerbaijani capital Baku, where there were calls for the region's recapture.
Reasons for Dispute:
Territorial: Nagorno-Karabakh region has 95% of the population as ethnically Armenian and is
controlled by them but it is internationally recognised as part of Azerbaijan.
Religious: Armenia is Christian majority, while Azerbaijan is Muslim majority country.
Domestic Politics: The leaders of both the nations have fueled the issue time and again for their
vested political interests.
India has supported Armenia while Azerbaijan has been supported by Pakistan. Pakistan was the
second country to recognize Azerbaijan‟s independence after Turkey. Also, Pakistan is the only
country that does not recognize Armenia as an independent state and fully supports Azerbaijan‟s
position. The INSTC was expanded to include eleven new members, namely: the Republic of
Azerbaijan, Republic of Armenia, Republic of Kazakhstan, Kyrgyz Republic, Republic of Tajikistan,
Republic of Turkey, Republic of Ukraine, Republic of Belarus, Oman, Syria, Bulgaria (Observer). In
April 2016, the region was particularly tense because of violent fighting between the two countries
in what came to be known as the Four-Day War. Since then, there have been many instances of
flare-ups in the region.
42. Ans:(d)
Border clashes in July killed at least 16 people, prompting the largest demonstration in years in the
Azerbaijani capital Baku, where there were calls for the region's recapture.
Reasons for Dispute:
Territorial: Nagorno-Karabakh region has 95% of the population as ethnically Armenian and is
controlled by them but it is internationally recognised as part of Azerbaijan.
Religious: Armenia is Christian majority, while Azerbaijan is Muslim majority country.
Domestic Politics: The leaders of both the nations have fueled the issue time and again for their
vested political interests.
India has supported Armenia while Azerbaijan has been supported by Pakistan. Pakistan was the
second country to recognize Azerbaijan‟s independence after Turkey. Also, Pakistan is the only
country that does not recognize Armenia as an independent state and fully supports Azerbaijan‟s
position. The INSTC was expanded to include eleven new members, namely: the Republic of
Azerbaijan, Republic of Armenia, Republic of Kazakhstan, Kyrgyz Republic, Republic of Tajikistan,
Republic of Turkey, Republic of Ukraine, Republic of Belarus, Oman, Syria, Bulgaria (Observer). In
April 2016, the region was particularly tense because of violent fighting between the two countries
in what came to be known as the Four-Day War. Since then, there have been many instances of
flare-ups in the region.
43. Ans:(a)
Border clashes in July killed at least 16 people, prompting the largest demonstration in years in the
Azerbaijani capital Baku, where there were calls for the region's recapture.
Reasons for Dispute:
Territorial: Nagorno-Karabakh region has 95% of the population as ethnically Armenian and is
controlled by them but it is internationally recognised as part of Azerbaijan.
Religious: Armenia is Christian majority, while Azerbaijan is Muslim majority country.
Domestic Politics: The leaders of both the nations have fueled the issue time and again for their
vested political interests.
India has supported Armenia while Azerbaijan has been supported by Pakistan. Pakistan was the
second country to recognize Azerbaijan‟s independence after Turkey. Also, Pakistan is the only
country that does not recognize Armenia as an independent state and fully supports Azerbaijan‟s
position. The INSTC was expanded to include eleven new members, namely: the Republic of
Azerbaijan, Republic of Armenia, Republic of Kazakhstan, Kyrgyz Republic, Republic of Tajikistan,
Republic of Turkey, Republic of Ukraine, Republic of Belarus, Oman, Syria, Bulgaria (Observer). In
April 2016, the region was particularly tense because of violent fighting between the two countries
in what came to be known as the Four-Day War. Since then, there have been many instances of
flare-ups in the region.
44. Ans:(d)
Border clashes in July killed at least 16 people, prompting the largest demonstration in years in the
Azerbaijani capital Baku, where there were calls for the region's recapture.
Reasons for Dispute:
Territorial: Nagorno-Karabakh region has 95% of the population as ethnically Armenian and is
controlled by them but it is internationally recognised as part of Azerbaijan.
Religious: Armenia is Christian majority, while Azerbaijan is Muslim majority country.
Domestic Politics: The leaders of both the nations have fueled the issue time and again for their
vested political interests.
India has supported Armenia while Azerbaijan has been supported by Pakistan. Pakistan was the
second country to recognize Azerbaijan‟s independence after Turkey. Also, Pakistan is the only
country that does not recognize Armenia as an independent state and fully supports Azerbaijan‟s
position. The INSTC was expanded to include eleven new members, namely: the Republic of
Azerbaijan, Republic of Armenia, Republic of Kazakhstan, Kyrgyz Republic, Republic of Tajikistan,
Republic of Turkey, Republic of Ukraine, Republic of Belarus, Oman, Syria, Bulgaria (Observer). In
April 2016, the region was particularly tense because of violent fighting between the two countries
in what came to be known as the Four-Day War. Since then, there have been many instances of
flare-ups in the region.
45. Ans:(a)
Border clashes in July killed at least 16 people, prompting the largest demonstration in years in the
Azerbaijani capital Baku, where there were calls for the region's recapture.
Reasons for Dispute:
Territorial: Nagorno-Karabakh region has 95% of the population as ethnically Armenian and is
controlled by them but it is internationally recognised as part of Azerbaijan.
Religious: Armenia is Christian majority, while Azerbaijan is Muslim majority country.
Domestic Politics: The leaders of both the nations have fueled the issue time and again for their
vested political interests.
India has supported Armenia while Azerbaijan has been supported by Pakistan. Pakistan was the
second country to recognize Azerbaijan‟s independence after Turkey. Also, Pakistan is the only
country that does not recognize Armenia as an independent state and fully supports Azerbaijan‟s
position. The INSTC was expanded to include eleven new members, namely: the Republic of
Azerbaijan, Republic of Armenia, Republic of Kazakhstan, Kyrgyz Republic, Republic of Tajikistan,
Republic of Turkey, Republic of Ukraine, Republic of Belarus, Oman, Syria, Bulgaria (Observer). In
April 2016, the region was particularly tense because of violent fighting between the two countries
in what came to be known as the Four-Day War. Since then, there have been many instances of
flare-ups in the region.
46. Ans:(b)
A security deal reached between the US and the militant Islamist group in February paved the way
for the current talks. According to the agreement signed between the US and the Taliban on
February 29, the majority of US troops are to withdraw from Afghanistan by the end of next
year.Recently, the much awaited intra-Afghan talks between the Taliban and the Afghan High
Council for National Reconciliation (representative of Afghan Government) opened in Doha, Qatar.
External Affairs Minister S Jaishankar attended the peace talks virtually and said that India was
committed to a country that is “Afghan-led, Afghan-owned and Afghan-controlled.” India had also
attended the signing of the US-Taliban agreement in February.
There were four objectives of the deal- an end to violence by declaring a ceasefire, an intra-Afghan
dialogue for a lasting peace, the Taliban cut ties with terrorist organisations such as al Qaeda and
U.S. troop withdrawal by April 2021.
Talks between the Afghan government and Taliban insurgents started in Qatar‟s capital Doha on
Saturday with the goal of bringing an end to nearly two decades of a conflict that has laid waste to
the country and killed tens of thousands of combatants and civilians. External Affairs Minister S
Jaishankar also attended the peace talks virtually.
On 11 September 2001, terrorist attacks in America killed nearly 3,000 people. Osama Bin Laden,
the head of Islamist terror group al-Qaeda, was quickly identified as the man responsible. Since
then, the US is fighting a war against the Taliban by deploying its forces.
47. Ans:(d)
A security deal reached between the US and the militant Islamist group in February paved the way
for the current talks. According to the agreement signed between the US and the Taliban on
February 29, the majority of US troops are to withdraw from Afghanistan by the end of next
year.Recently, the much awaited intra-Afghan talks between the Taliban and the Afghan High
Council for National Reconciliation (representative of Afghan Government) opened in Doha, Qatar.
External Affairs Minister S Jaishankar attended the peace talks virtually and said that India was
committed to a country that is “Afghan-led, Afghan-owned and Afghan-controlled.” India had also
attended the signing of the US-Taliban agreement in February.
There were four objectives of the deal- an end to violence by declaring a ceasefire, an intra-Afghan
dialogue for a lasting peace, the Taliban cut ties with terrorist organisations such as al Qaeda and
U.S. troop withdrawal by April 2021.
Talks between the Afghan government and Taliban insurgents started in Qatar‟s capital Doha on
Saturday with the goal of bringing an end to nearly two decades of a conflict that has laid waste to
the country and killed tens of thousands of combatants and civilians. External Affairs Minister S
Jaishankar also attended the peace talks virtually.
On 11 September 2001, terrorist attacks in America killed nearly 3,000 people. Osama Bin Laden,
the head of Islamist terror group al-Qaeda, was quickly identified as the man responsible. Since
then, the US is fighting a war against the Taliban by deploying its forces.
48. Ans:(c)
A security deal reached between the US and the militant Islamist group in February paved the way
for the current talks. According to the agreement signed between the US and the Taliban on
February 29, the majority of US troops are to withdraw from Afghanistan by the end of next
year.Recently, the much awaited intra-Afghan talks between the Taliban and the Afghan High
Council for National Reconciliation (representative of Afghan Government) opened in Doha, Qatar.
External Affairs Minister S Jaishankar attended the peace talks virtually and said that India was
committed to a country that is “Afghan-led, Afghan-owned and Afghan-controlled.” India had also
attended the signing of the US-Taliban agreement in February.
There were four objectives of the deal- an end to violence by declaring a ceasefire, an intra-Afghan
dialogue for a lasting peace, the Taliban cut ties with terrorist organisations such as al Qaeda and
U.S. troop withdrawal by April 2021.
Talks between the Afghan government and Taliban insurgents started in Qatar‟s capital Doha on
Saturday with the goal of bringing an end to nearly two decades of a conflict that has laid waste to
the country and killed tens of thousands of combatants and civilians. External Affairs Minister S
Jaishankar also attended the peace talks virtually.
On 11 September 2001, terrorist attacks in America killed nearly 3,000 people. Osama Bin Laden,
the head of Islamist terror group al-Qaeda, was quickly identified as the man responsible. Since
then, the US is fighting a war against the Taliban by deploying its forces.
49. Ans:(a)
A security deal reached between the US and the militant Islamist group in February paved the way
for the current talks. According to the agreement signed between the US and the Taliban on
February 29, the majority of US troops are to withdraw from Afghanistan by the end of next
year.Recently, the much awaited intra-Afghan talks between the Taliban and the Afghan High
Council for National Reconciliation (representative of Afghan Government) opened in Doha, Qatar.
External Affairs Minister S Jaishankar attended the peace talks virtually and said that India was
committed to a country that is “Afghan-led, Afghan-owned and Afghan-controlled.” India had also
attended the signing of the US-Taliban agreement in February.
There were four objectives of the deal- an end to violence by declaring a ceasefire, an intra-Afghan
dialogue for a lasting peace, the Taliban cut ties with terrorist organisations such as al Qaeda and
U.S. troop withdrawal by April 2021.
Talks between the Afghan government and Taliban insurgents started in Qatar‟s capital Doha on
Saturday with the goal of bringing an end to nearly two decades of a conflict that has laid waste to
the country and killed tens of thousands of combatants and civilians. External Affairs Minister S
Jaishankar also attended the peace talks virtually.
On 11 September 2001, terrorist attacks in America killed nearly 3,000 people. Osama Bin Laden,
the head of Islamist terror group al-Qaeda, was quickly identified as the man responsible. Since
then, the US is fighting a war against the Taliban by deploying its forces.
50. Ans:(c)
A security deal reached between the US and the militant Islamist group in February paved the way
for the current talks. According to the agreement signed between the US and the Taliban on
February 29, the majority of US troops are to withdraw from Afghanistan by the end of next
year.Recently, the much awaited intra-Afghan talks between the Taliban and the Afghan High
Council for National Reconciliation (representative of Afghan Government) opened in Doha, Qatar.
External Affairs Minister S Jaishankar attended the peace talks virtually and said that India was
committed to a country that is “Afghan-led, Afghan-owned and Afghan-controlled.” India had also
attended the signing of the US-Taliban agreement in February.
There were four objectives of the deal- an end to violence by declaring a ceasefire, an intra-Afghan
dialogue for a lasting peace, the Taliban cut ties with terrorist organisations such as al Qaeda and
U.S. troop withdrawal by April 2021.
Talks between the Afghan government and Taliban insurgents started in Qatar‟s capital Doha on
Saturday with the goal of bringing an end to nearly two decades of a conflict that has laid waste to
the country and killed tens of thousands of combatants and civilians. External Affairs Minister S
Jaishankar also attended the peace talks virtually.
On 11 September 2001, terrorist attacks in America killed nearly 3,000 people. Osama Bin Laden,
the head of Islamist terror group al-Qaeda, was quickly identified as the man responsible. Since
then, the US is fighting a war against the Taliban by deploying its forces.
51. Ans:(d)
US President Donald Trump, Israeli Prime Minister Benjamin Netanyahu and foreign ministers of
Bahrain and UAE signed the first ever peace agreement in 26 years. The deal is names as the
Abraham Accord. It took an Arab country three decades to recognise Israel. Egypt had signed the
first ever peace treaty with Israel. Jordan is the second Arab country to establish peace with Israel.
Jordan took 15 more years to establish the peace. 26 years after the Jordan, UAE signed peace
deal with Israel.
Likud officially the Likud – National Liberal Movement, is a centre-right to right-wing political party in
Israel. Benjamin Netanyahu is the leader of Likud.
52. Ans:(c)
US President Donald Trump, Israeli Prime Minister Benjamin Netanyahu and foreign ministers of
Bahrain and UAE signed the first ever peace agreement in 26 years. The deal is names as the
Abraham Accord. It took an Arab country three decades to recognise Israel. Egypt had signed the
first ever peace treaty with Israel. Jordan is the second Arab country to establish peace with Israel.
Jordan took 15 more years to establish the peace. 26 years after the Jordan, UAE signed peace
deal with Israel.
Likud officially the Likud – National Liberal Movement, is a centre-right to right-wing political party in
Israel. Benjamin Netanyahu is the leader of Likud.
53. Ans:(a)
US President Donald Trump, Israeli Prime Minister Benjamin Netanyahu and foreign ministers of
Bahrain and UAE signed the first ever peace agreement in 26 years. The deal is names as the
Abraham Accord. It took an Arab country three decades to recognise Israel. Egypt had signed the
first ever peace treaty with Israel. Jordan is the second Arab country to establish peace with Israel.
Jordan took 15 more years to establish the peace. 26 years after the Jordan, UAE signed peace
deal with Israel.
Likud officially the Likud – National Liberal Movement, is a centre-right to right-wing political party in
Israel. Benjamin Netanyahu is the leader of Likud.
54. Ans:(b)
US President Donald Trump, Israeli Prime Minister Benjamin Netanyahu and foreign ministers of
Bahrain and UAE signed the first ever peace agreement in 26 years. The deal is names as the
Abraham Accord. It took an Arab country three decades to recognise Israel. Egypt had signed the
first ever peace treaty with Israel. Jordan is the second Arab country to establish peace with Israel.
Jordan took 15 more years to establish the peace. 26 years after the Jordan, UAE signed peace
deal with Israel.
Likud officially the Likud – National Liberal Movement, is a centre-right to right-wing political party in
Israel. Benjamin Netanyahu is the leader of Likud.
55. Ans:(a)
US President Donald Trump, Israeli Prime Minister Benjamin Netanyahu and foreign ministers of
Bahrain and UAE signed the first ever peace agreement in 26 years. The deal is names as the
Abraham Accord. It took an Arab country three decades to recognise Israel. Egypt had signed the
first ever peace treaty with Israel. Jordan is the second Arab country to establish peace with Israel.
Jordan took 15 more years to establish the peace. 26 years after the Jordan, UAE signed peace
deal with Israel.
Likud officially the Likud – National Liberal Movement, is a centre-right to right-wing political party in
Israel. Benjamin Netanyahu is the leader of Likud.
56. Ans:(d)
Middle East, the lands around the southern and eastern shores of the Mediterranean Sea,
encompassing at least the Arabian Peninsula and, by some definitions, Iran, North Africa, and
sometimes beyond. By the mid-20th century a common definition of the Middle East encompassed
the states or territories of Turkey, Cyprus, Syria, Lebanon, Iraq, Iran, Israel, the West Bank, the
Gaza Strip, Jordan, Egypt, Sudan, Libya, and the various states and territories of Arabia proper
(Saudi Arabia, Kuwait, Yemen, Oman, Bahrain, Qatar, and the Trucial States, or Trucial Oman [now
United Arab Emirates]). Subsequent events have tended, in loose usage, to enlarge the number of
lands included in the definition. The three North African countries of Tunisia, Algeria, and Morocco
are closely connected in sentiment and foreign policy with the Arab states. In addition, geographic
factors often require statesmen and others to take account of Af.
57. Ans:(a)
The retrospective tax demand for India was Rs. 22,100 crores from Vodafone.
By 2014, all attempts by the telco and the Finance Ministry to settle the issue had failed. Vodafone
Group then invoked Clause 9 of the Bilateral Investment Treaty (BIT) signed between India and the
Netherlands in 1995. While the treaty was between India and the Netherlands, Vodafone invoked it
as its Dutch unit, Vodafone International Holdings BV, had bought the Indian business operations of
Hutchinson Telecommunicaton International Ltd. This made it a transaction between a Dutch firm
and an Indian firm. The BIT between India and the Netherlands expired on September 22, 2016.
One of the major factors for the Court of Arbitration to rule in favour of Vodafone was the violation of
the BIT and the United Nations Commission on International Trade Law (UNCITRAL). Article 3 of the
arbitration rules of UNCITRAL, among other things, says that ―constitution of the arbitral tribunal
shall not be hindered by any controversy with respect to the sufficiency of the notice of arbitration,
which shall be finally resolved by the arbitral tribunal‖.
Recently, the Supreme Court of India allowed telecom companies (telcos) 10 years‘ time to pay their
Adjusted Gross Revenue (AGR) dues to the government. AGR is a fee-sharing mechanism between
the government and the telcos who shifted to the 'revenue-sharing fee' model in 1999, from the 'fixed
license fee' model. The Permanent Court of Arbitration was set up in 1989. The victory of Vodafone
at PCA may provide it some relief as the telecom sector is already facing a number of issues and
challenges.
Capital gains tax is a levy assessed on the positive difference between the sale price of the asset
and its original purchase price. Capital gains can be reduced by deducting the capital losses that
occur when a taxable asset is sold for less than the original purchase price. The total of capital gains
minus any capital losses is known as the "net capital gains."
In May 2007, Vodafone had bought a 67% stake in Hutchison Whampoa for $11 billion. This
included the mobile telephony business and other assets of Hutchison in India.
58. Ans:(c)
The retrospective tax demand for India was Rs. 22,100 crores from Vodafone.
By 2014, all attempts by the telco and the Finance Ministry to settle the issue had failed. Vodafone
Group then invoked Clause 9 of the Bilateral Investment Treaty (BIT) signed between India and the
Netherlands in 1995. While the treaty was between India and the Netherlands, Vodafone invoked it
as its Dutch unit, Vodafone International Holdings BV, had bought the Indian business operations of
Hutchinson Telecommunicaton International Ltd. This made it a transaction between a Dutch firm
and an Indian firm. The BIT between India and the Netherlands expired on September 22, 2016.
One of the major factors for the Court of Arbitration to rule in favour of Vodafone was the violation of
the BIT and the United Nations Commission on International Trade Law (UNCITRAL). Article 3 of the
arbitration rules of UNCITRAL, among other things, says that ―constitution of the arbitral tribunal
shall not be hindered by any controversy with respect to the sufficiency of the notice of arbitration,
which shall be finally resolved by the arbitral tribunal‖.
Recently, the Supreme Court of India allowed telecom companies (telcos) 10 years‘ time to pay their
Adjusted Gross Revenue (AGR) dues to the government. AGR is a fee-sharing mechanism between
the government and the telcos who shifted to the 'revenue-sharing fee' model in 1999, from the 'fixed
license fee' model. The Permanent Court of Arbitration was set up in 1989. The victory of Vodafone
at PCA may provide it some relief as the telecom sector is already facing a number of issues and
challenges.
Capital gains tax is a levy assessed on the positive difference between the sale price of the asset
and its original purchase price. Capital gains can be reduced by deducting the capital losses that
occur when a taxable asset is sold for less than the original purchase price. The total of capital gains
minus any capital losses is known as the "net capital gains."
In May 2007, Vodafone had bought a 67% stake in Hutchison Whampoa for $11 billion. This
included the mobile telephony business and other assets of Hutchison in India.
59. Ans:(b)
The retrospective tax demand for India was Rs. 22,100 crores from Vodafone.
By 2014, all attempts by the telco and the Finance Ministry to settle the issue had failed. Vodafone
Group then invoked Clause 9 of the Bilateral Investment Treaty (BIT) signed between India and the
Netherlands in 1995. While the treaty was between India and the Netherlands, Vodafone invoked it
as its Dutch unit, Vodafone International Holdings BV, had bought the Indian business operations of
Hutchinson Telecommunicaton International Ltd. This made it a transaction between a Dutch firm
and an Indian firm. The BIT between India and the Netherlands expired on September 22, 2016.
One of the major factors for the Court of Arbitration to rule in favour of Vodafone was the violation of
the BIT and the United Nations Commission on International Trade Law (UNCITRAL). Article 3 of the
arbitration rules of UNCITRAL, among other things, says that ―constitution of the arbitral tribunal
shall not be hindered by any controversy with respect to the sufficiency of the notice of arbitration,
which shall be finally resolved by the arbitral tribunal‖.
Recently, the Supreme Court of India allowed telecom companies (telcos) 10 years‘ time to pay their
Adjusted Gross Revenue (AGR) dues to the government. AGR is a fee-sharing mechanism between
the government and the telcos who shifted to the 'revenue-sharing fee' model in 1999, from the 'fixed
license fee' model. The Permanent Court of Arbitration was set up in 1989. The victory of Vodafone
at PCA may provide it some relief as the telecom sector is already facing a number of issues and
challenges.
Capital gains tax is a levy assessed on the positive difference between the sale price of the asset
and its original purchase price. Capital gains can be reduced by deducting the capital losses that
occur when a taxable asset is sold for less than the original purchase price. The total of capital gains
minus any capital losses is known as the "net capital gains."
In May 2007, Vodafone had bought a 67% stake in Hutchison Whampoa for $11 billion. This
included the mobile telephony business and other assets of Hutchison in India.
60. Ans:(a)
The retrospective tax demand for India was Rs. 22,100 crores from Vodafone.
By 2014, all attempts by the telco and the Finance Ministry to settle the issue had failed. Vodafone
Group then invoked Clause 9 of the Bilateral Investment Treaty (BIT) signed between India and the
Netherlands in 1995. While the treaty was between India and the Netherlands, Vodafone invoked it
as its Dutch unit, Vodafone International Holdings BV, had bought the Indian business operations of
Hutchinson Telecommunicaton International Ltd. This made it a transaction between a Dutch firm
and an Indian firm. The BIT between India and the Netherlands expired on September 22, 2016.
One of the major factors for the Court of Arbitration to rule in favour of Vodafone was the violation of
the BIT and the United Nations Commission on International Trade Law (UNCITRAL). Article 3 of the
arbitration rules of UNCITRAL, among other things, says that ―constitution of the arbitral tribunal
shall not be hindered by any controversy with respect to the sufficiency of the notice of arbitration,
which shall be finally resolved by the arbitral tribunal‖.
Recently, the Supreme Court of India allowed telecom companies (telcos) 10 years‘ time to pay their
Adjusted Gross Revenue (AGR) dues to the government. AGR is a fee-sharing mechanism between
the government and the telcos who shifted to the 'revenue-sharing fee' model in 1999, from the 'fixed
license fee' model. The Permanent Court of Arbitration was set up in 1989. The victory of Vodafone
at PCA may provide it some relief as the telecom sector is already facing a number of issues and
challenges.
Capital gains tax is a levy assessed on the positive difference between the sale price of the asset
and its original purchase price. Capital gains can be reduced by deducting the capital losses that
occur when a taxable asset is sold for less than the original purchase price. The total of capital gains
minus any capital losses is known as the "net capital gains."
In May 2007, Vodafone had bought a 67% stake in Hutchison Whampoa for $11 billion. This
included the mobile telephony business and other assets of Hutchison in India.
61. Ans:(b)
The retrospective tax demand for India was Rs. 22,100 crores from Vodafone.
By 2014, all attempts by the telco and the Finance Ministry to settle the issue had failed. Vodafone
Group then invoked Clause 9 of the Bilateral Investment Treaty (BIT) signed between India and the
Netherlands in 1995. While the treaty was between India and the Netherlands, Vodafone invoked it
as its Dutch unit, Vodafone International Holdings BV, had bought the Indian business operations of
Hutchinson Telecommunicaton International Ltd. This made it a transaction between a Dutch firm
and an Indian firm. The BIT between India and the Netherlands expired on September 22, 2016.
One of the major factors for the Court of Arbitration to rule in favour of Vodafone was the violation of
the BIT and the United Nations Commission on International Trade Law (UNCITRAL). Article 3 of the
arbitration rules of UNCITRAL, among other things, says that ―constitution of the arbitral tribunal
shall not be hindered by any controversy with respect to the sufficiency of the notice of arbitration,
which shall be finally resolved by the arbitral tribunal‖.
Recently, the Supreme Court of India allowed telecom companies (telcos) 10 years‘ time to pay their
Adjusted Gross Revenue (AGR) dues to the government. AGR is a fee-sharing mechanism between
the government and the telcos who shifted to the 'revenue-sharing fee' model in 1999, from the 'fixed
license fee' model. The Permanent Court of Arbitration was set up in 1989. The victory of Vodafone
at PCA may provide it some relief as the telecom sector is already facing a number of issues and
challenges.
Capital gains tax is a levy assessed on the positive difference between the sale price of the asset
and its original purchase price. Capital gains can be reduced by deducting the capital losses that
occur when a taxable asset is sold for less than the original purchase price. The total of capital gains
minus any capital losses is known as the "net capital gains."
In May 2007, Vodafone had bought a 67% stake in Hutchison Whampoa for $11 billion. This
included the mobile telephony business and other assets of Hutchison in India.
62. Ans:(d)
The retrospective tax demand for India was Rs. 22,100 crores from Vodafone.
By 2014, all attempts by the telco and the Finance Ministry to settle the issue had failed. Vodafone
Group then invoked Clause 9 of the Bilateral Investment Treaty (BIT) signed between India and the
Netherlands in 1995. While the treaty was between India and the Netherlands, Vodafone invoked it
as its Dutch unit, Vodafone International Holdings BV, had bought the Indian business operations of
Hutchinson Telecommunicaton International Ltd. This made it a transaction between a Dutch firm
and an Indian firm. The BIT between India and the Netherlands expired on September 22, 2016.
One of the major factors for the Court of Arbitration to rule in favour of Vodafone was the violation of
the BIT and the United Nations Commission on International Trade Law (UNCITRAL). Article 3 of the
arbitration rules of UNCITRAL, among other things, says that ―constitution of the arbitral tribunal
shall not be hindered by any controversy with respect to the sufficiency of the notice of arbitration,
which shall be finally resolved by the arbitral tribunal‖.
Recently, the Supreme Court of India allowed telecom companies (telcos) 10 years‘ time to pay their
Adjusted Gross Revenue (AGR) dues to the government. AGR is a fee-sharing mechanism between
the government and the telcos who shifted to the 'revenue-sharing fee' model in 1999, from the 'fixed
license fee' model. The Permanent Court of Arbitration was set up in 1989. The victory of Vodafone
at PCA may provide it some relief as the telecom sector is already facing a number of issues and
challenges.
Capital gains tax is a levy assessed on the positive difference between the sale price of the asset
and its original purchase price. Capital gains can be reduced by deducting the capital losses that
occur when a taxable asset is sold for less than the original purchase price. The total of capital gains
minus any capital losses is known as the "net capital gains."
In May 2007, Vodafone had bought a 67% stake in Hutchison Whampoa for $11 billion. This
included the mobile telephony business and other assets of Hutchison in India.
63. Ans:(b)
The Chandra X-Ray Center (CXC) is operated for NASA by the Smithsonian Astrophysical
Observatory. The Chandra X-ray Observatory was launched by Space Shuttle Columbia in 1999.
The Chandra X-ray Observatory is part of NASA's fleet of "Great Observatories" along with the
Hubble Space Telescope, the Spitzer Space Telescope.
64. Ans:(c)
Cassiopeia A is a supernova remnant in the constellation Cassiopeia and the brightest extrasolar
radio source in the sky at frequencies above 1 GHz. The supernova occurred approximately 11,000
light-years away within the Milky Way.
65. Ans:(d)
The Galactic Centre is the rotational centre of the Milky Way galaxy. It comprises a collection of
celestial objects —
•Neutron and white dwarf stars,
•Clouds of dust and gas,
•A supermassive black hole called Sagittarius A*(weighs four million times the mass of the sun).
66. Ans:(d)
The Hubble Space Telescope is one of the largest and most versatile telescopes in service.It is a
space telescope that was launched into low Earth orbit (540km above Earth) in 1990.Hubble‘s four
main instruments observe in the near ultraviolet, visible, and near infrared spectra.
67. Ans:(b)
The term ‗black hole‘ was coined in the mid-1960s by American Physicist John Archibald Wheeler. It
refers to a point in space where the matter is so compressed as to create a gravity field from which
even light cannot escape. Black-holes were theorized by Albert Einstein in 1915.
68. Ans:(c)
The "X-ray universe" refers to the universe as observed with telescopes designed to detect X-rays.
X-rays are produced in the cosmos when matter is heated to millions of degrees. Such temperatures
occur where high magnetic fields, or extreme gravity, or explosive forces exist in space.
69. Ans:(b)

70. Ans:(d)
Let number of person in city C = x
Let number of person in city D = 3x
Required difference = 20100×3x-25100×x=588
= 0.6x – 0.25x = 588
X = 5880.35=1680
Total number of person in city D = 1680 ×3=5040
71. Ans:(d)
Required difference = (40 + 35)% - (35 + 25)% = 240\
15% = 240
Number of person who likes Apple in city E = 40% = 24015×40=640
72. Ans:(b)
Let,
Number of person is city D = x
And, number of person in city E = y
ATQ,
0.3x – 0.35y = 2100..(i)
And
0.45x + 0.25y = 4100……(ii)
On solving (i) and (ii)
X = 8000, y = 2000
Required % = 8000-20002000×100
= 60002000×100=300%
73. Ans:(c)
Let, number of person in city A = 2x
Number of person in city B = 3x
ATQ,
0.35 ×2x+0.3×3x=816
0.7x+0.9x = 816
=> x = 510
Total number of person in city B = 3 ×510
= 1530
74. Ans:(b)

Number of elements in set A only = 24

75. Ans:(c)
There are 10 elements in only set B, i.e, there are 10 odd numbers which are not primes.

76. Ans:(a)

There are 14 elements in B n C i.e, there are 14 odd prime numbers.


77. Ans:(b)

There is only one element in A n C i.e, 2 is the only even prime number.
78. Ans:(c)

Number of elements in C' = the elements which do not belong to the set (c) There are
(24 + 10) = 34 such numbers i.e, numbers are non primes.

79. Ans:(c)
80. Ans:(c)

Only P2 = 35
Only P3 = 28
Required Ratio = 35/28 = 5 : 4
81. Ans:(a)

Only P1 = 30
Only P1 + P3 = 16
Required % = 3016 X 100 = 187.5%

82. Ans:(d)

At most one paper = 30 + 28 + 35 = 93


Required % = 93200 x 100 = 46.5 %

83. Ans:(b)
Refer to, ―Searching for just one event, a decisive change in culture or brain structure, would
probably be a mistake. What caused our species to break out of the pattern set by archaic humans?‖
84. Ans:(a)
Refer to, ―. There is a mental ability we possess today that must have emerged at some point in our
history, and whose emergence would have vastly enhanced our ancestors‘ creative powers.‖
85. Ans:(a)
Refer to, ―I want to introduce two suggestions… Neither is directly about hypothetical thinking but,
combined, they offer a compelling picture of how humans acquired the capacity for it.‖
86. Ans:(a)
Refer to, ―Because it worked by sharing experience, it was limited to communication about things
that were on hand to be experienced.‖
87. Ans:(c)
This is a tough question. To analyse if the options are correct, we need to refer to ―hypothetical
thinking – the ability to detach one‘s mind from the here and now, and consciously think about other
possibilities.‖ That is we cannot be in the present, we need to think beyond it. A wholesome reason
of the passage suggests that the Archaic humans lacked the ability to think hypothetically. Reason
for the same could have been that they lived in the present, they didn‘t actively think up innovations
for themselves.
88. Ans:(c)
The only incorrect observation is in sentence 3 which incorrectly assume that ALL the energy it is
necessary for the day is gathered in the MORNING. The passage mentions ‗as much as it can
gather‘. Hence, option C is correct.
89. Ans:(c)
The author provides various examples to emphasize his point that long held notions of cognitive
sciences and memory seem to be disproved by plant behaviour. The description of basil is just one
of the many examples. Hence, option A cannot be the main Idea. The author does not mention that
cognition and memory is a distinction between humans and plants, rather he indirectly states that
plants have been long believed to not have cognition because they lack nervous system that
indicates a notion about plants, not a distinction between plants and humans.So option B is
incorrect. Option d is incorrect because there are no contrasting studies. Option C is correct.
90. Ans:(d)
The author refers to my corrhizal networks in the last paragraph for ―plants also communicate with
one another…‖ Hence, option D is correct.
91. Ans:(a)
The scientific belief in the passage relates necessity of nervous system with cognitive abilities.The
recent findings show presence of cognitive abilities in plants despite plants not having any nervous
system. Hence, option A is correct.
92. Ans:(b)
The information in the passage agrees with all of the answer choices except sentence 1.
Sentence 1 is wrong because of ‗The basil isn't an anomaly. Plants are not simply organic, passive
automata. We now know that they can sense and integrate information about dozens of different
environmental variables, and that they use this knowledge to guide flexible, adaptive behaviour.‘
Hence, option B is correct.
93. Ans:(a)
Although all three examples are given in the passage, the examples of Arabidopsis thaliana and
interplant network of fungus are given for cognition, not for memory. The only example for memory is
holy basil. Hence, option A is correct.
94. Ans:(c)
Sentence1 can be inferred from the author‘s emphasis in the second paragraph that what Adam
Smith did (Professor of Moral Philosophy) has nothing to do with what has been attributed to him
(economics). Sentence 2 can be inferred from last two lines of the second paragraph. Sentence 4
can be inferred from the last paragraph. Sentence 3 is an incorrect inference. Hence, option C is
correct.
95. Ans:(b)
According to the author, economics today has departed from its original theory but calling it
pseudoscience is incorrect. So option A is incorrect. Option C and option D are not given in the
passage. Option B is derived from superiority of markets over government planning in first
para. Hence, option B is correct.
96. Ans:(d)
The ideology of Political Right puts markets over government planning, not modern economic theory.
So option A is incorrect. Option B is unrelated to the passage. Option C is an exaggeration of what is
stated in the passage. Hence, option D is correct.
97. Ans:(b)
Option A is unrelated to the passage. Option C cannot be logically related to the passage. Option D
is an accusation the author does not make in the passage. Hence, option B is correct.
98. Ans:(c)
Option A is clearly incorrect. While Adam Smith's work was instrumental to economics, the author
criticises modern economic theory for departing from the way economics was engendered. So option
B is incorrect. Adam Smith was cynical of some of those who were eager to jump to his appreciation,
but that cannot be inferred as ―most of those‖. So option d is incorrect. Option C can be inferred from
the last two lines of last paragraph. Hence, option C is correct
99. Ans:(c)
Adam Smith's work was not understood by those who criticized him, but that does not make his work
―too difficult to understand‖. So option A is incorrect. Inferring that pretence was rampant, would be
an incorrect generalization. So option B is incorrect. Option d is clearly incorrect. Hence,option C is
correct.
100. Ans:(b)
The prediction is that the current high prices of domestic air travel in India will remain high and that a
price war among airlines is unlikely. [B] negates this by pointing out that private airlines owners
would like to reduce their prices. [A] does not comment on the possibility of a price war. [C] and [D],
by claiming that the private airlines cannot afford to lower their prices, confirm the prediction. Hence,
[B].
101. Ans:(b)
The author ends the passage with ―So why do otherwise rational people take an ―if I don‘t see it, it
isn‘t there‖ approach to symptoms that can have a devastating effect on their health?‖ This, clearly,
implies that he wants people to understand the seriousness of their illness. Option (a) trivializes this;
(c) with ―health insurance‖ is out of context; (b) is suitable. Hence (b).
102. Ans:(a)
The author talks about us ignoring our symptoms and delaying seeking help. Statement I supports
that. Statement II actually shows patients acting upon early signs which weakens the passage.
Hence (a).
103. Ans:(a)
The passage ends with a question - So why do otherwise rational people take an ―if I don‘t see it, it
isn‘t there‖ approach to symptoms that can have a perilous effect on their health? Option (a)
answers this aptly; (b) gives reasons for going to a doctor not refraining from; (c) is out of context
with ―economic conditions‖; (d) gives the physician‘s point of view and not the patients. Hence (a).
104. Ans:(d)
Option (d) weakens the argument made in the passage as it states that geriatrics (old people) go for
routine checks where actually early signs of serious illnesses can be recognized. Hence (d).
105. Ans:(c)
The author clearly states that we should not ignore early signs; we should not be in denial, and
should actually go and seek help. Option (a) gives an excuse for not taking care of one‘s health.
Option (b) is an example of denial. Option (d) contradicts the passage. Option (c) is apt with
―concern…professional advice.‖ Hence (c).
106. Ans:(a)
The passage begins with the viewpoint of Bacon and then the author drives the attention to talking
directly to us, the readers. Hence, A. Option B is eliminated because of ‗MANY‘. Option C is close,
but notice that nowhere he hints that readers should not have high expectations from him. Option D
is wrong because ‗the two introductions‘ will follow later, they‘re not explained here, in this passage.
107. Ans:(d)
The passage mentions the analogy (similarity in logical argument) between going to a function and
writing a book. Hence, D
108. Ans:(d)
The author is drawing an analogy between attending a function and beginning to write a book. He
compares the situation that just the way people do not know others in a party, similarly, it is equally
unpleasant for a writer to begin to write a book. Hence D. Option A – due to ‗often‘- and option B –
due to ‗do not enjoy‘ – are irrelevant.
109. Ans:(d)
Option A and option C are factually incorrect and against the information mentioned in the passage.
Option B is wrong because it is not the author‘s but Bacon‘s viewpoint. Option D can be inferred
from these lines: ‗this introduction is taking a more LONG-WINDED route in comparison with other
things you must have read, and I am going to continue in that style for a few pages by offering you
not one, but two introductions to Bacon FOR REASNS I will subsequently EXPLAIN‘
110. Ans:(a)
The passage mentions ‗PREFACE‘, hence, it must be a book. The passage focuses on
Bacon, hence, the book must be based on Bacon. Option B does not mention ‗Bacon‘. Option C
does not mention ‗book‘. Option D misses both, ‗Bacon‘ and ‗Book‘
111. Ans:(a)
N is taller to P. M is taller to R but shorter to P. At least one person is shorter to R. N>P>M>R>__ Q
is taller to M. The number of person taller to Q is same as shorter to O. So we get this-
Q>N>P>M>R>O.
O is the shortest in the group. Hence, option (a.).
112. Ans:(c)
Follow the numbers in the middle, the numbers are decreasing by 2. So next number has to 18
which is present only in 3rd option.
113. Ans:(b)
The passage states that communication of an acceptance is important to form a contract. In this
case, the letter of acceptance was never communicated to Grant and thus the same cannot
constitute a valid contract.
114. Ans:(d)
The passage states that expression of interest in making an offer must be made. In this case, the
words stated by Ravi only constitute an expression of interest as the word ―probably‖ was used by
Ravi
115. Ans:(b)
The passage states that it is important for an intention to offer to be present for an offer to be valid.
In this case, it cannot be said that there was an intention to offer since Aditi had only told the
minimum price for her car.
116. Ans:(b)
The passage states that a valid form of consideration is not doing a particular thing. Not participating
in a race aligns with such valid offers.
117. Ans:(a)
The passage states that acceptance may be done through a person‘s conduct. In this case, the
relevant conduct is Ashruth‘s resignation after he was made the offer by his employer which is
sufficient to prove that he had accepted the offer.
118. Ans:(b)
The passage states that there are three requirements of volentinon fit injuria are voluntary,
agreement and knowledge. In this case, as the two workers were fully aware that they were acting in
contravention of statutory provisions and against the instructions of the employer, they can be said
to fall within volenti non-fit injuria.
119. Ans:(c)
The passage states that knowledge is an important element for the applicability of volenti-non fit
injuria. In this case, the fact that the driver was smelling strongly of liquor is sufficient to establish
that Ashay was aware of the risks involved in asking the driver to drive fast.
120. Ans:(b)
The passage states that knowledge is an important element for the applicability of volenti-non fit
injuria. In this case, the fact that the secretary knew that disturbance would be caused is enough to
prove that he had consented to the risk.
121. Ans:(a)
The passage states that voluntary consent to risks is required for the defence to be applicable. In all
the options, there is influence of a superior. However, in Option A, the parties have only requested
the politician and thus consent may be assumed to undergo the risks.
122. Ans:(d)
The passage states that consent in cases of intoxicated state is dicey. It only depends on how
intoxicated the person consenting at the relevant time is for him to understand the consequences.
123. Ans:(d)
The passage states that consent in such cases can also be determined by conduct. However, in
cases wherein there is only silence about the landing, it cannot be said that he had consented to the
same.
124. Ans:(d)
The passage states the tort is only committed when the person is in lawful possession of the
property. Since in this case, the property was not in the lawful possession of Anand, Ahuja is not
liable for such trespass to chattel.
125. Ans:(a)
The passage states that mere depriving someone of their property is enough to show that the tort
has been committed. Thus, it is enough to state that property was thrown out of the house to the
road.
126. Ans:(c)
The passage states that trespass to chattel does not arise if such trespass is caused to real property
or land. Since in this case, the trespass was caused to a house, it cannot be said that trespass to
chattel is caused.
127. Ans:(c)
The passage states that a general intent to commit trespass to property is enough. There is no need
to have a specific intent to commit a tort against a specific person. Thus, liability arises.
128. Ans:(c)
The passage states that in cases where the consent is taken the tort of trespass of chattel is not
committed. However, since the property was taken without free consent due to the effect of the
medicine, it cannot be said that the tort was not committed.
129. Ans:(c)
The passage clearly states that a mistake of ownership is not a defence when it comes to trespass
to chattel. Mere possession is enough. Thus, A‘s taking away of B‘s book mistakenly shall not be a
valid defence.
130. Ans:(c)
K has induced Z to give him money, through dishonest means, regardless of Z giving him money or
not.K has committed extortion.
131. Ans:(c)
Z has not extorted D as he did not mean to gain any wrongful value from him when he asked him to
sit in his vehicle. But against his family members he would be guilty of the same.
132. Ans:(b)
A dishonest intention could a base for several crimes. It was extortion as fear was induced to get
hold of valuables in possession of Z, regardless of when the payment is due, or who is harmed in the
process to such gains.
133. Ans:(a)
134. Ans:(b)
Whoever intentionally puts any person in fear of any injury to that person, or to any other, and
thereby dishonestly induces the person so put in fear to deliver to any person any property or
valuable security, or anything signed or sealed which may be converted into a valuable security,
commits ―extortion‖.
135. Ans:(b)
Where the accused honestly believed that thecomplainant had taken the money belonging to him
(the accused), an attemptto get it back cannot be said to be with the intention of causing wrongful
loss tohim.
136. Ans:(b)
This is simply because of the alteration and curtailment of certain rights on account of being a
prisoner. Thus, it cannot put at par with other situations of forced labour under
normal circumstances.
137. Ans:(d)
This is because of the reason that no one would willingly work for less than the minimum wage
without some force or compunction and that compunction could include hunger or poverty.
Therefore, external pressure allows him it opt for a job without actually consenting to it.
138. Ans:(c)
This is because ―forced labor‖, prohibited by Article 23, includes not just physical force but also the
threat of imprisonment or fine. Also, even when it would amount to malicious prosecution but the
relevant aspect in the existing case for Article 23 is the consent of the individual.
139. Ans:(d)
This is because Mannu Singh was forced to work in return of grossly inadequate compensation as
he was constrained by the circumstances and he never voluntarily agreed to work at the farm.
140. Ans:(a)
This is due to the reason that the primary factor is the consent of the other individual which decides
whether the sexual intercourse would amount to rape or not. In the existing case, the absence
of consent makes Madhur liable for rape.
141. Ans:(c)
This is because every act has to be seen as a different or separate instance. Therefore, a consent
given to an earlier act would not extend to any future events. Therefore, if Disha did not consent to
the act it would amount to rape.
142. Ans:(c)
This is because even when she provided for a consent that was limited to a restricted sexual activity
so when she withdraws her consent for further acts; this absence of consent makes the intercourse
an act of rape. Option (c) would be more appropriate and relevant than option (d) since the former is
much closer to the question of rape due to the lack of consent.
143. Ans:(c)
This is because the consent of the minor cannot be considered a valid consent thereby making the
act the offence of rape. Therefore, the parents can file a case against Rahul for such an act against
their daughter.
144. Ans:(b)
This is because the idea of rape incorporates the aspect of penetration and such an act is not
possible for women therefore categorizing them only as the victims and not the aggressors. There
are other reasons as well but in the existing options (b) is the most relevant to the question.
145. Ans:(a)
This is because even when she failed to show an active resistance it was due to the threat exercised
by Robin making the consent an invalid consent thereby making him liable for the offence of Rape.
146. Ans:(b)
This is because he was under the impression that the gun is not loaded so the element of
negligence or great care cannot come in question. Therefore, such mistake of fact will exonerate him
from the liability.
147. Ans:(d)
This is because Chulbul was under a mistake of a fact which was not deliberate in nature. He acted
in good faith to follow the order after a detailed enquiry therefore exempting him of any liability.
148. Ans:(d)
This is because the letter was written without any due enquiry which highlights lack of reasonable or
due care making the actions not done under good faith and thus creating a liability on the President
of the Municipal office.
149. Ans:(c)
This is because when we take the whole factual information into consideration we realize that there
existed no circumstances which made Seeta to think that Geeta is the thief as the act of fetching
water does not determine that the utensils were same and the offence of theft has been committed.
Therefore, Seeta did not act in good faith and shall be considered liable.
150. Ans:(c)
This is because even when they acted in good faith the act shall fall under mistake of law as they
were importing non- edible sunflower oil under an open general license therefore no defense is
availed.
Legal Edge 06 Mock

Answers & Details

ENGLISH

1. Ans:(a)
No empire in Indian subcontinent fortified their boundaries so as to keep the invaders out like the
Chinese did with the great wall of China. Therefore, A is the correct answer.
2. Ans:(c)
Indians were too stubborn to follow the faith of foreign invaders and not amenable. Therefore, C
cannot be inferred from the passage and it is the correct answer.
3. Ans:(b)
The central idea is that even after the invasion by the Chinese tribes, Turks, Islamic rulers and
British invasion, Indian society has always remained heterogenic. Hence, B is the correct answer.
4. Ans:(c)
Both the statements are correct according to the passage. Hence, C is the correct answer.
5. Ans:(a)
"heterogeneous" may refer to a society or group that includes individuals of differing ethnicities,
cultural backgrounds, sexes, or ages. Diverse is the more common synonym in the context. Hence,
A is the correct answer.
6. Ans:(b)
Colonial historian give reason for calling it a black century as after the fall of Mughal empire, there
was anarchy all over and which boosted the colonial expansion in India. Therefore, B is the correct
answer.
7. Ans:(b)
Answer: It is clear in the passage that according to Bayly, the Indian political culture is dependent on
the legitimacy.
8. Ans:(c)
British were careful in changing with social dynamics and beliefs as their attempted social reforms
backfired and created anti-colonial sentiments among people. Hence, C is the correct answer.
9. Ans:(c)
The Chinese did not allow the Europeans to enter their empire so in order to gain commercial
privileges they had to attack the Chinese state. Hence, C is the correct answer.
10. Ans:(b)
Implausible means not seeming reasonable or probable; failing to convince. Hence, B is the correct
answer.
11. Ans:(c)
The passage never mentions Da Vinci as an astronomer or any of his astronomical works. Hence, C
is the correct answer.
12. Ans:(d)
All the three statements are correct as given in the first paragraph of the passage, hence, D is the
correct answer.
13. Ans:(d)
Both homosexuality and worshiping divine order of nature are against the Christianity and therefore,
Da Vinci been an awkward subject for historians. Hence, D is the correct answer.
14. Ans:(c)
Verbs are the action words in a sentence that describe what the subject is doing. In the passage, the
word is used in the context of action of Da Vinci. Hence, C is the correct answer.
15. Ans:(a)
Eerie means strange and frightening. Therefore, the most opposite in the opposite in the options is
comfortable.
16. Ans:(a)
Swashbuckling means to engage in daring and romantic adventures with bravado or flamboyance.
Hence, A is the correct answer.
17. Ans:(b)
Chandamama was collection of both original and retold stories. Therefore, B is not true and hence it
is the answer.
18. Ans:(b)
Vicissitude means changes or alternation. And perpetuity means indefinite or unchangeable.
Therefore, B is the correct answer.
19. Ans:(d)
Option A Synecdoche occurs when a part is represented by the whole or, conversely, the whole is
represented by the part. Hence, A is not the answer.
Option B, simile is a comparison between two unlike things using the words "like" or "as”, or “in”.
Hence, B is incorrect.
C, an oxymoron is two contradictory terms used together. Hence, it is not the answer.
Option D is thus correct, as none of the figures of speech applies in here.
20. Ans:(b)
In the twilight means the last years of someone's life. Hence, B is the correct answer.
21. Ans:(b)
Obloquy means strong public condemnation.
22. Ans:(c)
Sen argued that in a democracy, if food scarcity happens then it will be quickly reported and compel
the government to send quick help. Hence, C is the correct answer.
23. Ans:(c)
Only C is correct and is given in passage. Rest of the option cannot be inferred from the passage.
Hence, C is the correct answer.
24. Ans:(d)
The passage is not about criticizing any one government, it is about the restriction of free flow of
information leading to bad governance. Hence, D is the correct answer.
25. Ans:(c)
Verbs are the action words in a sentence that describe what the subject is doing. Here, the action
of Mohun Roy is shown by the usage of word urge. Hence, C is the correct answer.
26. Ans:(c)
It means to remove, take away, or withdraw gradually. Hence, C is the correct answer.
27. Ans:(a)
There is nothing about Uttar Pradesh police being corrupt in the passage while all the other
options are there. Hence, A is the correct answer.
28. Ans:(a)
The Crime is committed by the members of Thakur (upper caste) people on the lower caste Dalit girl.
It is a crime coming out of inequal society. Hence, A is the correct answer.
29. Ans:(b)
In a democracy, the state is granted the monopoly of violence, means the only actor in a democratic
nation to resort to violence is the State. Hence, B is the correct answer.
30. Ans:(b)
It means the disposal of a dead person's body by burning it to ashes, typically after a funeral
ceremony.

GENERAL KNOWLEDGE/CURRENT AFFAIRS


31. Ans:(a)
SARS-CoV-2 uses a human enzyme called ACE2 to enter into the cells of its host.
ACE2 lowers blood pressure in the human body and works against another enzyme known as ACE
(which has the opposite effect).
The virus causes the levels of ACE to fall in the lungs, and consequently pushes up the levels of
ACE2.
This happens as a chain reaction and increases the levels of the molecule bradykinin in the cells,
causing a bradykinin storm.
32. Ans:(c)
Solution: The bradykinin theory was outlined in a July research paper published in eLife, but it was
recently featured in a widely read article published on Medium's science website, Elemental.
33. Ans:(a)
Solution: The technologies provided by Israel include an Artificial Intelligence video-oriented and
voice-operated autonomous personal AI assistant robot. There is also an app that can be installed
on any mobile of COVID-19 staff, it will make the work of hospital staff inside the departments much
more effective and easier.
34. Ans:(c)
Solution: A subset of the infected patients develop severe COVID-19 because of an overreaction of
their immune systems, which triggers what is known as a “Cytokine Storm Syndrome” (CSS).
Normally, cytokines are part of the body's immune response to infection, but their sudden release in
large quantities can cause multisystem organ failure and death. Cytokine storms can be caused by a
number of infectious and non-infectious etiologies, especially viral respiratory infections such as
H5N1 influenza, SARS-CoV-1, and SARS-CoV-2 (COVID-19 agent).
35. Ans:(c)
Solution: Bradykinin is a compound that is related to pain sensation and lowering blood pressure in
the human body. SARS-CoV-2 uses a human enzyme called ACE2 to enter into the cells of its host.
ACE2 lowers blood pressure in the human body and works against another enzyme known as ACE
(which has the opposite effect).The virus causes the levels of ACE to fall in the lungs, and
consequently pushes up the levels of ACE2.
This happens as a chain reaction and increases the levels of the molecule bradykinin in the cells,
causing a bradykinin storm. The storm causes the blood vessels to expand and become leaky,
leading to swelling of the surrounding tissue.The levels of hyaluronic acid also increase.
36. Ans:(c)
Solution: Recently, the World Bank released the Human Capital Index (HCI) report for 2020. The
index benchmarks key components of human capital across countries. India has been ranked at the
116th position in the HCI 2020.
37. Ans:(d)
Solution: Recent Initiatives by India to Strengthen Human Capital:
Ayushman Bharat Yojana
National Education Policy
Atmanirbhar Bharat Yojana
TULIP: The Urban Learning Internship Program
Samagra Shiksha
38. Ans:(b)
Solution: The current President of the World Bank Group is David Malpass. He is the 13th President
of World Bank Group. He was selected by the Board of Executive Directors of the World Bank Group
on April 2019.
39. Ans:(b)
Solution: WTO or World Trade Organisation is not part of the World Bank community. World Bank
Group is comprised of the five institutions i.e. International Bank for Reconstruction and
Development (IBRD), the International Development Association (IDA), the International Finance
Corporation (IFC), the Multilateral Investment Guarantee Agency (MIGA) and the International
Centre for Settlement of Investment Disputes (ICSID).
40. Ans:(a)
Solution: The 2020 Human Capital Index update includes health and education data for 174
countries covering 98 percent of the world‟s population up to March 2020.
It provides a pre-pandemic baseline on the health and education of children, with the biggest strides
made in low-income countries.
The analysis shows that pre-pandemic, most countries had made steady progress in building the
human capital of children, with the biggest strides made in low-income countries. The country of
Singapore topped the score with 0.88 followed by Hong Kong at 0.81 and Japan at 0.80.
41. Ans:(c)
Solution: Indian Medical Council Act, 1956 is an Indian legislation. The Act provides for the
constitution of the Medical Council of India (MCI). The MCI regulates standards of medical
education, permission to start colleges, courses or increase the number of seats, registration of
doctors, standards of professional conduct of medical practitioners.
42. Ans:(a)
Solution:The NITI Aayog had recommended the replacement of MCI with NMC.
The decision was approved by most states and after its approval by the Prime Minister and NMC bill
was passed by parliament and approved by President on 8 August 2019.
National Medical Commission ordinance was brought in to replace Medical Council of India in early
2019 through an ordinance issued in January 2019 by the President of India.
43. Ans:(c)
Solution: The boards are: (i) the Under-Graduate Medical Education Board (UGMEB) and the Post-
Graduate Medical Education Board (PGMEB): These Boards will be responsible for formulating
standards, curriculum, guidelines, and granting recognition to medical qualifications at the
undergraduate and post graduate levels respectively. (ii) The Medical Assessment and Rating
Board (MARB): MARB will have the power to levy monetary penalties on medical institutions which
fail to maintain the minimum standards as laid down by the UGMEB and PGMEB. The MARB will
also grant permission for establishing a new medical college, starting any postgraduate course, or
increasing the number of seats. (iii) The Ethics and Medical Registration Board: This Board will
maintain a National Register of all licensed medical practitioners, and regulate professional
conduct. Only those included in the Register will be allowed to practice medicine. The Board will
also maintain a separate National Register for community health providers.
44. Ans:(b)
Solution: The Bill provides for just one medical entrance test across the country, the National Exit
Test (NEXT).
The NEXT would serve as -
the final MBBS exam, which will work as a licentiate examination
the screening test for foreign medical graduates
the screening test for admission to PG medical courses
[NEXT is different from the NEET which is to be taken before joining UG course.]
45. Ans:(b)
Solution: Delhi AIIMS‟ ENT head-neck surgery department, Professor Suresh Chandra Sharma,
was appointed as the chairman of National Medical Commission (NMC),.The NMC has been
mandated with framing policies for regulating medical institutions and medical professionals in the
country.
The Appointments Committee of the Cabinet has approved the appointment of Mr. Sharma for a
period of three years or till the age of 70 years.
46. Ans:(d)
Solution: The Life Insurance Corporation of India (LIC), General Insurance Corporation of India and
The New India Assurance Co have been identified as Domestic Systemically Important Insurers (D-
SIIs) for 2020-21 by insurance regulator, the Insurance Regulatory and Development Authority of
India (IRDAI).
The IRDAI would identify D-SIIs on an annual basis and disclose the names of such insurers for
public information.
47. Ans:(b)
Solution: Following the recommendations of the Malhotra Committee, in 1999 the Insurance
Regulatory and Development Authority (IRDA) was constituted to regulate and develop the
insurance industry and was incorporated in April 2000. Objectives of the IRDA include promoting
competition to enhance customer satisfaction with increased consumer choice and lower premiums
while ensuring the financial security of the insurance market.
48. Ans:(b)
Solution: IRDAI would identify D-SIIs on annual basis and disclose the names of such insurers for
public information. D-SIIs are perceived as insurers that are „too big or too important to fail‟ (TBTF).
D-SIIs refer to insurers of such size, market importance and domestic and global interconnectedness
whose distress or failure would cause a significant dislocation in the domestic financial system.
Thus, the continued functioning of D-SIIs is critical for the uninterrupted availability of insurance
services to the national economy.
49. Ans:(c)
Solution: The Life insurance Corporation of India was founded on September 1, 1956, when the
Parliament of India passed the Life Insurance of India Act that nationalized the insurance industry in
India. Over 245 insurance companies and provident societies were merged to create the state-
owned Life Insurance Corporation of India.
50. Ans:(a)
Income tax is a form of direct tax. Though the actual definitions vary between jurisdictions, in
general, a direct tax is a tax imposed upon a person or property as distinct from a tax imposed upon
a transaction, which is described as an indirect tax. The term may be used in economic and political
analyses, but does not itself have any legal implications.
51. Ans:(a)
ICJ is headquartered in The Hague, Netherlands and so is the International Criminal Court.
FAO‟s headquarters are in Rome, Italy. IMF is headquartered in Washington DC. International
Maritime Organisation is headquartered in London, UK.
52. Ans:(c)
The PCA was established in 1899, before the ICJ which came into existence in 1945 as an organ of
the United Nations. It is headquartered in The Hague, Netherlands. It is an intergovernmental
organization dedicated to serve the international community in the field of dispute resolution and to
facilitate arbitration and other forms of dispute resolution between States. It has a Financial
Assistance Fund which aims at helping developing countries meet part of the costs involved in
international arbitration or other means of dispute settlement offered by the PCA.
The PCA has a three-part organizational structure consisting of:
•Administrative Council - to oversee its policies and budgets,
•Members of the Court - a panel of independent potential arbitrators, and
•International Bureau - the Secretariat, headed by the Secretary-General.
The decision is published as a writ, along with any dissenting opinions. The writ is read to a public
session in the presence of the agents and lawyers of the parties to the case. The decision is binding
on the parties, and there is no mechanism for appeal.
53. Ans:(b)
UNCITRAL was established in 1966 as a subsidiary body of the United Nations General Assembly
(UNGA). It is the core legal body of the United Nations system in the field of international trade law.
Mandate: To further the progressive harmonization and modernization of rules on international
business and reform commercial laws.
It adopted the UNCITRAL Model Law on International Commercial Arbitration in 1985 and the
UNCITRAL Conciliation Rules in 1980. The UNGA has recommended the use of the said Model Law
and Rules in cases where a dispute arises in the context of international commercial relations and
the parties seek an amicable settlement of that dispute by recourse to conciliation. India has also
incorporated these uniform principles of Arbitration and Dispute Resolution (ADR) in the Arbitration
and Conciliation Act, 1996.
54. Ans:(d)
The tax levied on the profit or gain earned on selling capital assets is called capital gains tax.
Depending on the holding period, capital gains tax can be Long term Capital Gains Tax (LTCG) or
Short term Capital Gains Tax (STCG).
55. Ans:(b)
The 9.02 km-long-tunnel, built by the Border Roads Organisation (BRO), is the world‟s longest
highway tunnel and connects Manali to Lahaul-Spiti valley. It provides all-weather connectivity to the
landlocked valley of Lahaul-Spiti, which remains cut-off for nearly six months in a year as the
Rohtang Pass is uually snow-bound between November and April.
56. Ans:(a)
The tunnel reduces the distance by 46 km between Manali and Leh and the travel time by about 4 to
5 hours. It is expected to boost tourism and winter sports in the region. The tunnel, also significant
from the military logistics viewpoint, will provide better connectivity to the armed forces in reaching
Ladakh.
57. Ans:(c)
The Atal tunnel pierces through the Pir Panjal range above 10,000 feet, making it the longest
highway tunnel at such an altitude anywhere in the world.
58. Ans:(c)
The horseshoe-shaped tunnel has a telephone every 150 metres and a fire hydrant every 60 metres.
Ever 2.2 km there is a turning cavern while drivers can find an emergency exit every 500 metres.
There is also an air quality monitoring system after every kilometre.
59. Ans:(b)
Nathu La Pass is located in Sikkim whereas Rohtang Pass and Lipulekh Pass are located
in Himachal Pradesh and Uttarakhand respectively.
60. Ans:(b)
India successfully flight-tested its indigenously developed SMART torpedo system Monday, with
the Defence Research and Development Organisation hailing it as a “game changer” in anti-
submarine warfare.
61. Ans:(a)
Dr. Abdul Kalam Island, formerly known as Wheeler Island, is an island off the coast of Odisha,
India, approximately 150 kilometres (93 mi) east of the state capital Bhubaneswar. The Integrated
Test Range missile testing facility is located on the island. The island was originally named after
English commandant Lieutenant Hugh Wheeler. On 4 September 2015, the island was renamed to
honour the late Indian president, Dr. APJ Abdul Kalam.
62. Ans:(b)
Most of its flight in the air is covered at low altitudes with two-way data link from the warship or an
airborne submarine target detection system. It is also provided the exact location of the hostile
submarine to correct its flight path midway.
63. Ans:(b)
The Strait of Malacca is a narrow stretch of water, 890 km in length, between the Malay Peninsula
(Peninsular Malaysia) and the Indonesian island of Sumatra. As the main shipping channel between
the Indian Ocean and the Pacific Ocean, it is one of the most important shipping lanes in the world. It
is named after the Malacca Sultanate that ruled over the archipelago between 1400 and 1511, the
center of administration of which was located in the modern-day state of Malacca, Malaysia. It
separates the landmasses of Malaysia and Indonesia.

64. Ans:(d)
All are correctly matched.
The Port of Karachi is one of South Asia‟s largest and busiest deep-water seaports, handling about
60% of the nation‟s cargo located in Karachi, Pakistan. It is located on the Karachi Harbour, between
Kiamari, Manora, and Kakapir, and close to Karachi‟s main business district and several industrial
areas.
Chabahar Port is a seaport in Chabahar located in southeastern Iran, on the Gulf of Oman. It serves
as Iran's only oceanic port, and consists of two separate ports named Shahid Kalantari and Shahid
Beheshti, each of which has five berths.
The Hambantota Port was to be a maritime port in Hambantota, Sri Lanka. The first phase of the
port was opened on 18 November 2010, with the first ceremonial berthing of the naval ship "Jetliner"
to use the port facilities. It is named after former President Mahinda Rajapaksa.
65. Ans:(a)
Alfred Bernhard Nobel was a Swedish chemist, engineer, inventor, businessman, and philanthropist.
He held 355 different patents, dynamite being the most famous. The synthetic element nobelium
was named after him. He owned Bofors, which he redirected from its previous role as primarily an
iron and steel producer to a major manufacturer of cannon and other armaments. Having read a
premature obituary which condemned him for profiting from the sales of arms, he bequeathed his
fortune to institute the Nobel Prize. His name also survives in companies such as Dynamite Nobel
and Akzo Nobel, which are descendants of mergers with companies that Nobel established.
66. Ans:(b)
Poet, writer and thinker Rabindranath Tagore brought glory to the country when he became the first
Indian to win the Nobel Prize for the country. The 52-year-old Tagore was accorded the honour in
1913, 12 years after it made its debut.
67. Ans:(d)
Swedish chemist Alfred Nobel had drafted a will in 1895 where he reserved a large part of his estate
to establish Nobel Prizes after concerns of how the world would remember him. He wanted the
awards to be given to individuals (based on their achievements), annually, despite their nationality.
He died in 1896. It took nearly five years for the committee to set up, and the first set of awards for
Physiology or Medicine, Chemistry, Literature, Physics and Peace were awarded in 1901. After 67
years, Sweden's central bank with donation from donation from the Nobel Foundation, established
the Sveriges Riksbank Prize in Economic Sciences in Memory of Alfred Nobel in 1968.
68. Ans:(d)
Drs. Harvey J. Alter, Michael Houghton and Charles M. Rice had received the prize for their
discovery of the hepatitis C virus. The Nobel committee said the three scientists had “made possible
blood tests and new medicines that have saved millions of lives.”

QUANTITATIVE TECHNIQUES

69. Ans:(a)
Required number = (32% -22% ) of 1 lakh = 10% of 1 lakh = 10,000
70. Ans:(b)
Total crimes registered in WB in 2012 = 17% of 1 lakh = 17,000
Total Kidnapping registered in WB in 2012 = 15% of 17000 = 2550
∴ Total Kidnapping a registered per day WB
= 2550365≈7
71. Ans:(c)
Number of ITPA cases registered in TN = 12% of 9% of 1 lakh = 1080
Number of ITPA cases registered in UP = 18% of 9% of 1 lkah = 1620
∴ Required per cent = 1620-10801080×100=50%
72. Ans:(d)
Crimes registered against women in 2012 in Maharastra = 15% of 1 lakh = 15,000 and 10% of
15,000= 1500. Hence, our answer will be either ITPA or Dowry death, but ITPA is not given in
option. Hence, the answer is Dowry death
73. Ans:(b)

Required number = (43% - 40%) of 1 lakh = 3,000


74. Ans:(a)

Months Jan Feb March April May June


Who didn’t go
680 696 805 912 665 840
to Goa
Who went to
32% 40% 65% 52% 30% 60%
Goa
(100 – 40)%=
(100 – 32)%= 680
696
70% =
68% = 680 35% = 805 48% = 912 40% = 840
60% = 696 665
Total 100% = 680 × 100% = 100% = 100% =
100% = 696 × 100% =
100/68 2300 1900 2100
100/60 950
= 1000
= 1160
The number of
65% of 52% of 30% of 60% of
customers 32% of 1000 40% of 1160
2300 1900 950 2100
who went to = 320 = 464
= 1495 = 988 = 285 = 1260
Goa
The total number of customers in June = 2100
The total number of customers in Jan = 1000
The required difference = 2100 – 1000 = 1100
75. Ans:(d)

Months Jan Feb March April May June


Who didn’t go
680 696 805 912 665 840
to Goa
Who went to
32% 40% 65% 52% 30% 60%
Goa

(100 – 32)%= 680 (100 70% =


– 40)%= 35% = 805 48% = 912 40% = 840
Total 665
68% = 680 696
100% = 100% = 100% =
100% =
100% = 680 × 60% = 696 2300 1900 950 2100
100/68
100% = 696 ×
= 1000 100/60
= 1160
The number of
65% of 52% of 30% of 60% of
customers 32% of 1000 40% of 1160
2300 1900 950 2100
who went to = 320 = 464
= 1495 = 988 = 285 = 1260
Goa
In April, the total number of customers = 1900
The total number of males = 1900 x 35 = 1140
The total number of females = 1900 x 25 = 760
The total number of males who went to Goa = 40% of 1140 = 456
The total number of males who didn‟t go to Goa = 1140 – 456 = 684
Among the customers who didn‟t go to Goa, the number of females = 912 – 684 = 228
76. Ans:(b)

Months Jan Feb March April May June


Who didn’t go
680 696 805 912 665 840
to Goa
Who went to
32% 40% 65% 52% 30% 60%
Goa
(100 – 40)%=
(100 – 32)%= 680
696
70% =
68% = 680 35% = 805 48% = 912 40% = 840
60% = 696 665
Total 100% = 680 × 100% = 100% = 100% =
100% = 696 × 100% =
100/68 2300 1900 2100
100/60 950
= 1000
= 1160
The number of
65% of 52% of 30% of 60% of
customers 32% of 1000 40% of 1160
2300 1900 950 2100
who went to = 320 = 464
= 1495 = 988 = 285 = 1260
Goa
The number of customers who went to Goa in Jan and Feb together = 784
The number of customers who didn‟t go to Goa in March and April together = 1717
The required difference = 1717 – 784 = 933
77. Ans:(c)

Months Jan Feb March April May June


Who didn’t go
680 696 805 912 665 840
to Goa
Who went to
32% 40% 65% 52% 30% 60%
Goa
(100 – 40)%=
(100 – 32)%= 680
696
70% =
68% = 680 35% = 805 48% = 912 40% = 840
60% = 696 665
Total 100% = 680 × 100% = 100% = 100% =
100% = 696 × 100% =
100/68 2300 1900 2100
100/60 950
= 1000
= 1160
The number of
65% of 52% of 30% of 60% of
customers 32% of 1000 40% of 1160
2300 1900 950 2100
who went to = 320 = 464
= 1495 = 988 = 285 = 1260
Goa
The total number of customers in the first 3 months = 1000 + 1160 + 2300 = 4460
The total number of male customers in these months together = 4460 – 800 = 3660
78. Ans:(c)

Months Jan Feb March April May June


Who didn’t go
680 696 805 912 665 840
to Goa
Who went to
32% 40% 65% 52% 30% 60%
Goa
(100 – 40)%=
(100 – 32)%= 680
696
70% =
68% = 680 35% = 805 48% = 912 40% = 840
60% = 696 665
Total 100% = 680 × 100% = 100% = 100% =
100% = 696 × 100% =
100/68 2300 1900 2100
100/60 950
= 1000
= 1160
The number of 32% of 1000 40% of 1160 65% of 52% of 30% of 60% of
customers = 320 = 464 2300 1900 950 2100
who went to = 1495 = 988 = 285 = 1260
Goa
The total number of customers of GOGOAGONE Pvt ltd who had gone to Goa in the first six months
of the year 2018 = 320 + 464 + 1495 + 988 + 285 + 1260 = 4812
79. Ans:(c)
Total profit = 14% of Rs 120 cr = 16.8 cr.;India‟s share in profit = 3/6 × 16.8 cr = 8.4 crTax deduction
= 30% of 8.4 cr = 2.52 cr; Post tax profit = 8.4 - 2.52 = Rs 5.9 cr
80. Ans:(d)
Operational cost = 63% of 120 cr = 75.6 cr;25% to security = 75.6 × (25/100) = 18.9 crAvg. cost of a
security personnel = 1890 lakh / 17,000 = Rs 0.11 lakh.
81. Ans:(b)
Cash Inflow through Ticket Sales = 33% of 120 cr = 39.6 cr.;No. of Spectators = 760,000. Average
Realization per Spectator= 396000000 / 760,000 = Rs 521
82. Ans:(d)
Ticket sales at Rs 50/ticket = 50 × 14, 000 = Rs 7 lakhTicket sales at Rs 300/ticket = 300 × 14,000 =
Rs 42 lakhThe required difference = Rs 35 lakh

LOGICAL REASONING

83. Ans:(a)
Options (b), (c) and (d) form premise of the main idea of the author in the above passage. Therefore,
main idea of the author in the passage is that Government restriction has resulted into abandonment
of nuclear power plant projects in the United States. Hence, option (a) is the correct answer.
84. Ans:(b)
The main idea of the author in the above passage is that Government restriction has resulted into
abandonment of nuclear power plant projects in the United States and this based on the premise
that many companies have given up their plans to build nuclear power plants in the United
States. Therefore, option (b) is the correct answer.
85. Ans:(b)
In the above passage author says that federal government has imposed so many restrictions and
conditions on the building of nuclear power plants that it takes twice as long to build a nuclear power
plant in the United States as it does to build one in either Europe or Japan, and the result of this is
that many companies have given up their plans to build nuclear power plants in the United States.
Lower cost is not mentioned in the given passage. Therefore, correct answer is option (b).
86. Ans:(d)
The words „It is no coincidence‟ in the second sentence, and „clearly‟ in the last sentence, reveal the
author‟s assertion that government restrictions are directly responsible for the abandonment of
nuclear power station projects in the United States. So, he is asserting a causal connection between
two phenomena. Therefore, option (d) is the correct answer. Option (a) is not the correct answer
because the three sentences in the argument are very closely related, and no other explanation is
plausible and the author does not have to deny the possibility of any other point of view. Option (b) is
not correct because the author only states facts, and no assumptions have been made by him in the
argument. And no single case has been referred to in the passage. Hence, option(c) is wrong.
87. Ans:(d)
There‟s nothing in the passage to correlate the nuclear law in the two countries. Hence, (d)
88. Ans:(d)
Option (a) is incorrect as one student‟s view cannot be said to be the view of all the students. Option
(b) is wrong as we do not know who the respondents were. It need be students alone.
89. Ans:(c)
Explanation: Answer (c). Option (a) is irrelevant. Option (b) and (c) are close option choices but we
need to understand how option (b) is worded. It makes a comparison about how the changes in
higher education will be much more painful than seen elsewhere. The basis for such comparison
cannot be found in the passage. Hence we can eliminate option (b).
90. Ans:(c)
Option (a) and (b) are true, but are the subordinate ideas in the passage. Option (c) encapsulates
the main point that the author has raised in the passage and is the correct answer choice.
91. Ans:(b)
Option (a) cannot be inferred from the passage. Option (b) – refer to, “I need no convincing of the
value of campus life and in-classroom education.”
92. Ans:(a)
Option (a) is the correct answer choice as the core mission of the education is imparting skills as can
be inferred from the passage and employers now do not insist on formal degree for a person to be
eligible for a job. Option (b) is a confusing option choice, but can be eliminated as it doesn‟t consider
how MOOC courses fulfill the core mission of education.
93. Ans:(a)
Option (b) is extreme. Option (c) and (d) cannot be inferred from the passage. Option (a) can be
inferred from the following lines: „the regulatory process is unable to control profiteering in the form
of unaccounted donations‟. Hence, (a) is correct.
94. Ans:(d)
The passage is about the NEP 2020, so option (d) is correct. Options (a), (b) and (c) are side points,
not the main points.
95. Ans:(b)
The last sentence of the passage says that the idea of a NPST is bound to be disliked by states.
Therefore, option (b) should follow it as it tells another such instance where NEP will face opposition
from the states. Options (a), (c) and (d), though relevant to the argument, cannot immediately follow
the last sentence. They can be used in a better way, somewhere later in the analysis.
96. Ans:(c)
Option C weakens the author‟s argument in the sentence provided. Options A as well as option B
strengthen the author‟s argument. Option D is irrelevant to the given sentence. Hence, C is correct.
97. Ans:(c)
The introduction of National Education policy is a clear manifestation that the previous policy is now
regarded as unsatisfactory. Hence, (c) is correct. Option (a) or (b) do not find support from the
passage. Option D talks about „surety‟ i.e. it‟s certain to have academic success. However, passage
talks about „improvement‟ in learning outcomes. Hence, (d) is also wrong
98. Ans:(a)
The statement 1 talks about the status of the species now being „critically endangered‟. The
prospective solution to increase the number of species and save them from hunting can be
stricter regulations on fishing.
99. Ans:(c)
The concluding sentence of the paragraph arouses hope that human actions and conscience if work
towards the right direction might halt this hasty extinction march. Climate change is one part of the
cause of extinction BUT not a blanket basis to put up the statement.
100. Ans:(b)
The author has given no evidence to suggest that speed limitation was futile exercise. However he
does mention that due to their habit of remaining near land they get entangled easily and so the
fishermen need to use wires with weak breaking strength. Also the passage mentions about time not
being on whales‟ side as death outnumbers regulatory steps but that‟s not an explicit statement
about impossibility of saving them from extinction.
101. Ans:(b)
In the concluding paragraph the author talks about the possibility of things getting better slowly. It
might take long time he says, but things will improve.
But in the second last paragraph, the fact is contradicted by the above mentioned statement. Quite
clearly it weakens the author‟s very own argument.
102. Ans:(a)
Fishing regulations in response to critically endangering species would mean new, safe, and
advanced equipment for fishing. The burden will fall on fishermen‟s pocket and he will face even
more as the passage clearly states that the fish count will also go down. However, the regulations do
not support increased use of nets.
103. Ans:(b)
Low birth rate and lack of proper food happen to be the consequence of warming water in Gulf of
Maine and is mentioned in the passage. Sentence 4 is correct as it is mentioned that „climate
change has caused a lobster boom on the east coast of North America‟. Sentence 1 is wrong
because warming water is the result of climate change and not the other way round.
104. Ans:(c)
Lobster boom as the passage says is a direct consequence of climate change, while warming water
is another. Given in the passage is the fact that the whales look for cooler water, so evidently the
number of whales caught in warm water will be less than that of cold water.
105. Ans:(b)
The passage talks about lack of food and possibility of delivering off-springs in hot water. The lack of
both in hot water qualifies as best the reason to shift to cold water where birth and food both are
possible. Leisure cannot be inferred from the passage.
106. Ans:(a)
The correct option is (A) and the reasons for the loss of species count is enumerated in the passage
above. The revival of species may take steps that will give results in long-run, as long as 20 years is
mentioned in the last paragraph. With strict regulatory steps in action, we can reduce the rate at
which species is decreasing or quite possibly resolve it.
107. Ans:(a)
(1) and (2) statement
sim → mangoes.
(2) and (3) statement
tim → sweets.
(1) and (3) statementzoo → good.
108. Ans:(d)
According to given information the arrangement in the descending order of points will be
Divika>Shina‟s father >Ambar>Akash>Suhas>Suhas‟s niece
So, Shina has to be Suhas‟s niece and Snehal has to be Shina‟s father.
∴ the final arrangement will be as follows
Divika>Snehal>Ambar>Akash>Suhas>Shina
Shina is the lady in the group besides Divika.
109. Ans:(b)
According to given information the arrangement in the descending order of points will be
Divika>Shina‟s father >Ambar>Akash>Suhas>Suhas‟s niece
So, Shina has to be Suhas‟s niece and Snehal has to be Shina‟s father.
∴ the final arrangement will be as follows
Divika>Snehal>Ambar>Akash>Suhas>Shina
Divika won the game.
110. Ans:(c)
According to given information the arrangement in the descending order of points will be
Divika>Shina‟s father >Ambar>Akash>Suhas>Suhas‟s niece
So, Shina has to be Suhas‟s niece and Snehal has to be Shina‟s father.
∴ the final arrangement will be as follows
Divika>Snehal>Ambar>Akash>Suhas>Shina
Snehal stood second in the game.
111. Ans:(b)
The passage states that conspiracy occurs only when an illegal act occurs or an illegal act is sought
to be performed by illegal means. In this case, only import of camphor is banned prospectively. The
camphor stored in warehouses can still be legally sold. Thus, nothing illegal has been committed.
112. Ans:(b)
The passage states that conspiracy occurs only when an illegal act occurs or an illegal act is sought
to be performed by illegal means. In this case, Arnab and Rajput already had a lot of stock of masks
in their shops and this was not new produced stock and thus selling the same at higher prices would
not amount to a conspiracy.

LEGAL REASONING

113. Ans:(d)
The passage states that for a conspiracy to occur there is a requirement of 2 or more people. In this
case, it cannot be shown that there was any agreement between the 2 people as their means and
mode of operating were different. Thus, the two have acted independently and cannot be held liable
for conspiracy.
114. Ans:(b)
The passage states that an “agreement” is sine qua non for the occurrence of conspiracy. In this
case, only P would be guilty of murder because there was no “agreement” between P and her
friends that they would shoot at the bar mate. Hence, they cannot be held guilty of criminal
conspiracy.
115. Ans:(d)
The passage states that every person who partakes in the conspiracy is liable. The work may be as
small as delivering money to the goons. However, the relevant information is whether D knew about
the contents of the package which contained money for the goons. Thus, there is insufficient
information.
116. Ans:(a)
The passage states that a person is eligible to be a consumer when they do not buy something
for resale or for a commercial purpose. In the instant case, even if there was resale, the fact that the
original purpose of buying was domestic consumption is enough to prove that she is a consumer.
117. Ans:(b)
The passage states that a person is eligible to be a consumer when they do not buy something for
resale or for a commercial purpose. The very fact that Savita resold the packets for a profit, show the
involvement of a commercial purpose and thus she cannot be called a consumer.
118. Ans:(d)
The passage states that for a person to be a consumer, services must be hired for a consideration.
In this case, no consideration was paid for hiring the service. Thus, it cannot be said that Anuj is a
consumer.
119. Ans:(b)
The passage states that a company which has sold a defective part must replace the part with a part
of similar configuration or with the same part. In this case, the part was replaced with another part
that is dissimilar to the current engine. The replacement is not justified.
120. Ans:(b)
The passage states that the district forum has jurisdiction if the complaint is valued Rs 20 Lakhs or
less. The value of the complaint shall be 12 lakhs as only the engine is sought to be replaced and
not the car. Thus, the forum will have jurisdiction.
121. Ans:(d)
The passage states that in case a consumer is dissatisfied, they may approach the state
commission in 30 days. Thus, Rathore can definitely approach the commission within 20 days.
122. Ans:(d)
The passage states that a person may be given legal aid if he is poor and does not have the means
to afford a counsel. However, in this case, it is not clear whether Farid is poor or not. A person may
also choose not to get a counsel despite being well off
123. Ans:(c)
The passage states that a person may be given legal aid if he is poor and does not have the means
to afford a counsel. In the event such a person is not given legal aid, the trial is invalidated. In this
case, however, the fact that Vidisha did not ask for legal aid despite knowing her rights implies her
intent.
124. Ans:(a)
The passage states that a counsel must be given enough to time to prepare his defence. In this
case, as can be seen from the facts a time of only 1 day was given to prepare a defence which
cannot be stated to be in line with the rules of a fair trial.
125. Ans:(a)
The passage states that a counsel must be given enough to time to prepare his defence. In this
case, as can be seen from the facts a time of only 15 minutes were given for discussion with the
client which cannot be stated to be in line with the rules of a fair trial.
126. Ans:(b)
The passage states that a person is entitled to a public hearing before a court. However, the same is
only in the nature of an entitlement and cannot be said to be a mandatory requirement. Thus, the
closed doors hearing can be done.
127. Ans:(d)
The passage states that a person is entitled to a public hearing before a court. Even if the matter is
sensitive, it is per the laws of fair trial that the accused must be given a hearing in an open court.
128. Ans:(d)
The junior knew that there was a risk of theft happening at his home. The passage states that if
there is voluntary assumption of risk, the same is a defence to false imprisonment. Since, the junior
by not resigning stayed in Rajshekhar‟s chambers, there is a voluntary assumption of risk.
129. Ans:(b)
The principle mentions that any person responsible for security is also a police officer. Rohit, being
on deputation for Suryonshu is a police officer for the purpose. Thus, his acts in confining someone
unlawfully amount to false imprisonment.
130. Ans:(d)
The passage states that one must have lawful authority to confine. In this case, since Rohit was
working for Suryonshu, he had the lawful authority at that point in time. Since his actions were in line
with the penalty stipulated by the university he acted in authority and is not liable.
131. Ans:(a)
The passage states that the degree of imprisonment shall not matter. It thus does not matter that
only Asur‟s hands were held. The very fact that Dev did not have lawful authority to hold Asur‟s
hands means that he is liable for false imprisonment.
132. Ans:(b)
The passage states that a person‟s consent is a valid defence to the tort of false imprisonment. In
this case, it cannot be said that Asur had kept Dev captive. Dev had consented to being imprisoned
and thus there is a valid defence.
133. Ans:(d)
The passage clearly states that reasonable force may be used to escape from false imprisonment. In
this case, it cannot be said that killing someone with energy from your eyes is, reasonable enough to
justify self defence.
134. Ans:(c)
The passage states that intention is the most relevant factor. In this case since Hitesh did not intend
to commit battery, he cannot be said to have committed the offence.
135. Ans:(c)
The passage states that assault is assault even if the same was done by mutual consent. Thus, it
will not matter if the scuffle was from mutual consent.
136. Ans:(a)
The passage states that the police can arrest someone the moment a threat is made. It is not
required for the person to fulfill the crime of assault. Thus, the police are justified.
137. Ans:(c)
Ordinarily, the police did not take cognizance of empty threats for they are never intended to be
pursued. At the same time however, the police can arrest Mehta while they should not ordinarily.
138. Ans:(b)
The passage states that aggravated assault is constituted when the accused assaults a person with
a weapon. In this case, the knife was the weapon and thus attacking Mehta with the knife constitutes
aggravated assault.
139. Ans:(b)
Self explanatory
140. Ans:(c)
Had there been no mention of „unlimited‟ the bill might still hold water as it would only aid in the
investigation of crime. But, it‟s unrestricted therefore crosses the line of privacy.
141. Ans:(b)
This amounts to invasion of the dignity of the individual once he is discharged from the prison, he
has already completed his term and now subjection to this is only an invasion of his privacy.
142. Ans:(b)
self explanatory
143. Ans:(a)
such information does not form the part of a private information.
144. Ans:(d)
The customary or usual route via the Suez Canal being closed, John was bound to ship the
groundnuts by a reasonable and practical route and though John might be put to greater expense by
shipping the groundnuts via the Cape of Good Hope, that did not render the contract fundamentally
or radically different and there was not, therefore, frustration of the contract.
145. Ans:(a)
Under the circumstances she was not merely excused from playing, but she was also not at liberty to
play, if she was unfit to do so. The condition was clearly subject to the condition of her being well
enough to perform. The foundation of the contract was the performance of Y, failing which the whole
contract fails.
146. Ans:(d)
As the contract deed does not specify the supply of goods from Germany only, therefore there are
many other sources of supply, and the contract left the supplier with a free hand to procure the
goods from wherever it is possible. And hence Zoya‟s claim would not be entertained.
147. Ans:(b)
A will not be held liable for breach of contract since given circumstances were not under his control
and this hindered him from performing the contract, i.e., supplying the stock. The virus was a force
majeure event in the given circumstances which frees both the parties from contractual obligation.
148. Ans:(a)
The words „as and when the same may be received from the mills‟, should be construed as if they
were „if and when the same may be received from the mills. The words certainly regulates the
manner of performance, but they do not limit the sale to such goods as the mills might deliver.
149. Ans:(c)
The authenticity of a cultured pearl is compared to that of an apple grown in orchard. The similarity is
that both were deliberately grown. So if such an apple is not considered fake, then such a pearl
should also be not considered fake. Hence, (c).
150. Ans:(a)
Conclusion : winner of the 2018 election is truly the people‟s choice.
Supporting premise : voting in Maldives is compulsory ----------> over 99 percent of the eligible voters
voted
Missing piece of information : a voter always casts vote in favor of his/her election candidate/regional
representative.
A- Correct - Negate A: Voters are not allowed to vote for a candidate of their choice. Conclusion
breaks down.
B - not an assumption. it's a sequel to conclusion.
C- - not relevant
D - Maldives may have a Communist regime. But the leader can still be the people's choice.
Legal Edge 07 Mock

Answers & Details

ENGLISH

1. Ans:(b)
The word ‗without‘ as used in the first sentence refers to the weather outside. The sentence actually
means to say ―Outside, the night was cold and wet.‖
2. Ans:(c)
Options (a) and (d) although true in the context of the passage do not explain the meaning of the
given expression. Option (b) can probably be inferred but is not stated in the passage. Option (c) is
the best answer as ‗bog‘ refers to a marshy land and ‗torrent‘ also refers to a deluge.
3. Ans:(b)
The passage begins with a dark and stormy night and therefore the Whites would not want a guest
to wait outside for long. This is why Mr. White rose up with hospitable haste, in order to let in his
guest who was visiting them in such difficult weather. Option (b) is therefore the best answer.
4. Ans:(a)
Options (b) and (d) are extreme and therefore should be eliminated. Option (c) is far-fetched and not
suggested anywhere in the passage. Option (a) is the best answer as Mrs. White offered words of
condolence to her husband and then exchanged a knowing glance with her son because he would
probably defeat his father again.
5. Ans:(d)
Banging loudly and heavy footsteps are both denoting sound words and therefore they can be
classified as onomatopoeia.
6. Ans:(c)
The author mentions the line ―I shivered in that shadow as a boy growing up in a bull‘s eye of the
Cold War, and often doubted that I would live out a lifetime.‖ This would mean he used to live in a
highly affected region in the times of the Cold War. Since, he brings out instances of St. Louis Post
and talks about New Mexico, it must be said that he was in USA, which would mean he would have
been living in a place which was a target for the USSR. Thus, option (c) is the correct answer.
7. Ans:(a)
The author in the second paragraph has highlighted the casual approach of the journalists who have
unwittingly seen eras as disjointed entities, not connected to each other. The author feels that this
outlook is incorrect and casual. Perfunctory means an action carried out without real interest, feeling,
or effort. This seems to be the closest fit to the approach the author is describing. Hence, the answer
is option (a).
8. Ans:(d)
The author in the second paragraph is talking about how the journalists have shown a casual
attitude towards writing about the event, which has presented to the world a warped version of the
bomb. In the third paragraph the author talks about how the inception of the bomb has taken place
since long. Thus, we can say that the option (d) is the correct answer.
9. Ans:(b)
Stimson says though the technical proficiency for the creation of a nuclear bomb was achieved
during the World War II, the scientific basis was known to scientists to do the same was known to
scientists from a long time before. It was the urge to outrace the scientists of the Axis powers that
the Allied forces got the ―moral advancement‖ to create a nuclear bomb. Thus, option (b) is the
correct answer.
10. Ans:(a)
In option (a), we see the usage of a simile, as in here the author compares the events leading to the
creation of the nuclear bomb as a train running on rails with unstoppable momentum. In option (b),
we are talking about the Doomsday Clock, now for a clock, we use the words – hands. It‘s not a
personification, but a term attached to it. That warfare has consumed more than 100 million lives, is
not a hyperbole. Hence, option (c) is incorrect. Thus, option (a) is the correct answer.
11. Ans:(d)
Unmoored means ―not or no longer attached to a mooring‖. Thus, unattached is correct.
Inevitable means certain to happen; unavoidable. Predestined is a suitable meaning.
Abeyance means a state of temporary disuse or suspension. Thus suspension is correct.
Conflagration means an extensive fire which destroys a great deal of land or property. Thus inferno
is correct.
Thus, option (d) is correct.
12. Ans:(b)
The author would certainly agree to option (a), as he mentions that ―The only thing worse than
Hiroshima and Nagasaki was the unspeakable brutality that preceded them.‖ Thus, option (a) is
incorrect.
The author won‘t agree to option (b) fully as he though he mentions the fact that the Bomb has failed
to end all wars its menacing fear ―has spared the world from another industrial-
scale conflagration among great powers‖. Thus, option (b) is correct.
13. Ans:(b)
The author is trying to say that we should imbibe in us the attitude of Lawrence – ―Nothing is written‖
– that there is nothing as fate. This pandemic has left us damaged but we have many a futures
ahead of us and there‘s nothing that can hold us from taking positive steps – not even destiny. In
most of times, we find instances in history to follow, but, the situation that we are in has very less or
almost no instances. So, he is actually encouraging us to take in Lawrence‘s attitude and try things –
new things – as ―nothing is written‖. Option (b) is thus correct. Option (a) is wrong as he doesn‘t
criticize or compare the world‘s attitude to Lawrence‘s. Option (b) is thus, the correct answer.
14. Ans:(b)
To ―upend‖ means to ―set or turn (something) on its end or upside down‖. The author does mention
that ―The world that is being ushered in as a consequence of the covid-19 pandemic is new and
scary‖, which means there are almost no past instances of these kinds of pandemic and their
aftereffects happening in the world previously. This is novel. Thus, it has turned the society upside-
down. It has changed the fabric of the society. Thus, option (b) is correct.
We can‘t say that the author means to say that coronavirus pandemic has destroyed the fabric of
society as it is in our hands whether we let the pandemic destroy the society or not. Hence, option
(a) is incorrect.
Option (c) is wrong as the author doesn‘t say that the pandemic has given hope. Option (d) though is
in line with the author‘s thoughts, but this is not what he means in the above-given line.
15. Ans:(a)
The author mentions that we can go two-ways – either we ―could turn inward and embrace
nationalism and self-interest, or we could view these challenges — which cross all borders — as a
spur to global cooperation and action.‖ Thus, he means to have a global solution to global problem.
Hence, option (a) is correct.
The author mentions work-from-home as a change in the pandemic times, but it is not clear whether
he is in favour of that in the post-pandemic world or not. Thus, option (b) is not correct.
The author advocated for a sustainable growth. He doesn‘t mention about opening up the economy.
Thus, option (c) is incorrect.
Thus, option (a) is the answer.
16. Ans:(b)
The author in this phrase ―human agency‖ is indicating the social aspect. He says that it will be on
the people to change the direction of society. He goes on to say that the pandemic period has seen
people take up many positive changes to the lifestyle, that, if carried forward in the post-pandemic
world would benefit the society. Thus, option (b) is correct.
17. Ans:(c)
The author in this passage has exhibited a very positive and hopeful attitude, even in such times.
The world is in distress with the pandemic, but still the author maintains that this can be an
opportunity to bring in positive changes in the society. Sanguine means optimistic or positive,
especially in an apparently bad or difficult situation. Thus, option (c) is correct.
18. Ans:(c)
This will be the correct statement -
Think about the changes we have accepted in our own lives in response to the pandemic.
Thus, option (c) is correct.
19. Ans:(c)
The author in the first line is mentioning about the quarrel that is going on in between Arnab
Goswami and the rest of the TV news channels. He has said that this is like an ugly street brawl that
has broken out. This is a metaphor that has been used. Thus, option (c) is correct.
20. Ans:(d)
The italicized line actually says how the Arnab Goswami has actually in a way united all the other TV
channels, i.e. they are talking in the same lines. This has been mostly possible because everyone
has targeted Goswami and anyone who has opposed him in recent times have found support among
Arnab‘s other competitors or enemies. Thus, we can say option (d) can be used suitably.
21. Ans:(c)
Apocryphal means a story or statement of doubtful authenticity, although widely circulated as being
true. Hence, option (c) is correct.
22. Ans:(d)
The author says in the passage that the extras who are fighting against each other are actually the
journalists and anchors of the present-days. They were made to fight by the producer, who
represents the owners of the media-companies. Thus, the answer is option (d)
23. Ans:(a)
It stands for an attitude that reflects aggression and confidence. It also slightly hints at the careless
or rather carefree nature that the youth reflect. In the context of the passage it means the indifferent
attitude of the public.
24. Ans:(c)
The author says owning a media outlet gives a person intangible value, but sizeably more than the
economics of the media company. So, option (a) is not the answer.
The author says that the person owning the company gets in touch with ministers etc, from whom
they can reap out favours. So, option (b) is not the answer.
The author says that there are a set of owners and senior journalists who see themselves as
‗players‘ in politics and the game of power. They have to choose a side quickly, and almost always
the winning side. This means not everybody chooses the winning side but most of them do. So, this
is incorrect. Option (c) is the answer.
25. Ans:(d)
The author says that he won‘t say whether he thinks Arnab is wrong or not, because these are
polarizing times. Hence, option (a) is correct.
The author is the last line says ―When you see one television channel under pressure from the police
and rival media for allegedly faking its ratings, and within hours, statements appear in its support
from the president of the nation‘s dominant ruling party and the Union Information & Broadcasting
Minister, you know what is at stake.‖ Thus, he clearly indicates that there is some sort of an
association between Republic and the ruling-party that the latter come out in defence of the former.
Thus option (b) is correct.
Option (c) is also correct
26. Ans:(b)
Epoch signifies a long period of time (in the geological sense). Here also it has been used to denote
just that. Although the word ‗phase‘ seems tempting, it has a shorter connotation. Therefore ‗era‘ is
the best answer as life sciences have been ushered into a new era.
27. Ans:(c)
Read the question carefully. It says the answer has to be in the context of the passage. Now read
the relevant lines from the paragraph:
“It is being tried out in agriculture primarily to increase plant yield, quality, disease resistance,
herbicide resistance and domestication of wild species. The huge potential to edit genes using this
tool has been used to create a large number of crop varieties with improved agronomic performance;
it has also brought in sweeping changes to breeding technologies.”
Enhancing the yield and quality of plants and disease resistance are clearly stated in the passage.
‗Domestication of invasive species‘ is clearly wrong because the passage refers to wild species and
not invasive ones. Drought resistance, although true, is not mentioned in the passage and therefore
cannot be chosen as a part of the answer.
28. Ans:(c)
The answer is in the passage:
While the tool is most often used to make a cut in the DNA, newer approaches are being attempted
to add or make minor changes to the DNA. All these approaches may at some time in the future
make it easy to ―rewrite the code of life‖.
29. Ans:(c)
This is a personification as a human quality (aggressiveness) has been used to describe a
disease. Therefore the figure of speech is a personification.
30. Ans:(b)
The question asks for the incorrect statement. Therefore if we look at the options one by one:
(a)Correct, stated clearly in the passage.
(b)Incorrect as the passage states in the very beginning that there was no doubt as to who would
win the Nobel Prize so there wasn‘t room for much speculation.
(c)Correct as the passage states examples of agricultural applications.
(d)Correct, the passage states that it was used on a trial basis to treat an aggressive form of lung
cancer.
Therefore option (b) is the correct answer.

GENERAL KNOWLEDGE/CURRENT AFFAIRS

31. Ans:(c)
The International Day for Disaster Reduction (IDDR) is an international day that encourages every
citizen and government to take part in building more disaster-resilient communities and nations. The
United Nations General Assembly designated October 13 as the International Day for Natural
Disaster Reduction as part of its proclamation of the International Decade for Natural Disaster
Reduction. In 2009, the UN General Assembly decided to designate October 13 as the official date
for this day, and also changed the name to International Day for Disaster Reduction.
32. Ans:(d)
Data showed that Asia suffered the highest number of disasters in the past 20 years with 3,068 such
events, followed by the Americas with 1,756 and Africa with 1,192.In terms of affected countries,
China topped the list with 577 events followed by the United States with 467.The deadliest single
disaster in the past 20 years was the 2004 Indian Ocean tsunami, with 226,400 deaths, followed by
the Haiti earthquake in 2010, which killed some 222,000 people.
33. Ans:(c)
The 2004 Indian Ocean earthquake and tsunami (also known as the Boxing Day Tsunami and, by
the scientific community, the Sumatra–Andaman earthquake occurred at 07:58:53 in local time
(UTC+7) on 26 December, with an epicentre off the west coast of northern Sumatra, Indonesia. It
was an undersea megathrust earthquake that registered a magnitude of 9.1–9.3 Mw, reaching a
Mercalli intensity up to IX in certain areas. The earthquake was caused by a rupture along the fault
between the Burma Plate and the Indian Plate.
34. Ans:(a)
The National Disaster Management Authority (NDMA) is the apex statutory body for disaster
management in India.
The NDMA was formally constituted on 27th September 2006, in accordance with the Disaster
Management Act, 2005 with Prime Minister as its Chairperson and nine other members, and one
such member to be designated as Vice-Chairperson.
Mandate: Its primary purpose is to coordinate response to natural or man-made disasters and for
capacity-building in disaster resiliency and crisis response. It is also the apex body to lay down
policies, plans and guidelines for Disaster Management to ensure timely and effective response to
disasters.
35. Ans:(d)
The United Nations Office for Disaster Risk Reduction (UNDRR) was created in December 1999 to
ensure the implementation of the International Strategy for Disaster Reduction (General Assembly
(GA) resolution 54/219 . UNDRR (formerly UNISDR) is part of the United Nations Secretariat and it
supports the implementation & review of the Sendai Framework for Disaster Risk Reduction adopted
by the Third UN World Conference on Disaster Risk Reduction on 18 March 2015 in Sendai, Japan.
The Sendai Framework is a 15-year voluntary people-centered approach to disaster risk reduction,
succeeding the 2005-2015 framework.
36. Ans:(b)
Cas9 is a 160 kilodalton protein which plays a vital role in the immunological defense of certain
bacteria against DNA viruses and plasmids, and is heavily utilized in genetic engineering
applications. Its main function is to cut DNA and thereby alter a cell's genome.
37. Ans:(a)
FELUDA is an acronym for FNCAS9 Editor-Limited Uniform Detection Assay. It uses CRISPR-Cas
technology for the detection of genes specific to SARS-CoV-2 virus. Feluda, or Prodosh
Chandra Mitter, is a fictional Bengali private investigator starring in a series of Bengali novels and
short stories written by Indian Bengali film director and writer Satyajit Ray.
38. Ans:(c)
he Nobel Prize in Chemistry for 2020 which was announced on October 7, Wednesday, has two
women scientists as its recipients. That, in it, is a first in the sciences. Emmanuelle Charpentier, a
French microbiologist working at the Max Planck Unit for the Science of Pathogens in Berlin,
Germany, and Jennifer A. Doudna, a biochemist from the University of California, Berkeley, U.S.
shared the honour ―for the development of a method for genome editing‖.
39. Ans:(b)
The prokaryotic system, based on a region of DNA called Clustered Regularly Interspaced Short
Palindromic Repeats (CRISPR; is a largely stand-alone system that is capable of functioning in an
individual cell, a necessity for organisms that often display unicellular behavior. The CRISPR-Cas
system targets DNA or RNA as a way of protecting against viruses and other mobile genetic
elements.
40. Ans:(c)
DNA) is a molecule composed of two polynucleotide chains that coil around each other to form a
double helix carrying genetic instructions for the development, functioning, growth and reproduction
of all known organisms and many viruses. DNA and ribonucleic acid (RNA) are nucleic acids.
41. Ans:(c)
Under the Act, organisations require to register themselves every five years. As per the amended
FCRA rules, all NGOs registered or granted prior permission under FCRA are now required to
upload details of foreign contributions received and utilized by them every three months on their
website or the FCRA website. NGOs now need to file their annual returns online, with the hard copy
version dispensed with.
42. Ans:(d)
As defined in Section 2(1)(h) of FCRA, 2010, ―foreign contribution‖ means the donation, delivery or
transfer made by any foreign source.
Exemption: An article given to a person (includes Individual, HUF, Company, Association) as a gift
for his personal use and the market value of such article in India, on the date of such gift is not
exceeding Rs. 25,000/-.
43. Ans:(d)
Earlier in 2017, another powerful US-based Christian donor, Compassion International, was forced
to stop operations in India after the ministry found that it funded NGOs that encouraged religious
conversions.
the government seemed concerned that CI‘s funds were sent to local churches that ran the
children‘s services. And, as CI made clear, within the church context, children would be given
Christian teachings. In essence, the government was concerned that CI was giving their funds
exclusively to churches who could in turn encourage children to convert to Christianity.
44. Ans:(c)
Worldwide, the term ‗NGO‘ is used to describe a body that is neither part of a government nor a
conventional for-profit business organisation. NGOs are groups of ordinary citizens that are involved
in a wide range of activities that may have charitable, social, political, religious or other interests.. In
India, NGOs can be registered under a plethora of Acts such as the Indian Societies Registration
Act, 1860, Religious Endowments Act, 1863, Indian Trusts Act, etc. India has possibly the largest
number of active NGOs in the world, a study commissioned by the government put the number of
NGOs in 2009 at 33 lakh. Recently, the Union Ministry of Home Affairs has suspended licenses
under the Foreign Contribution Regulation Act (FCRA), 2010 of the six Non Governmental
Organisations (NGOs). Of the six NGOs, four are Christian associations, Ecreosoculis North
Western Gossner Evangelical in Jharkhand, the Evangelical Churches Association (ECA) in
Manipur, Northern Evangelical Lutheran Church in Jharkhand and New Life Fellowship Association
(NLFA) in Mumbai. The two other organisations whose licenses were suspended are the
Rajnandgaon Leprosy Hospital and Clinics, Chhattisgarh and the Don Bosco Tribal Development
Society, Tamil Nadu.
45. Ans:(b)
The Bajrang Dal, the youth wing of the Vishwa Hindu Parishad (VHP), had obstructed prayer
meetings organized by the NLFA in Mumbai in April and September last year. The Hindu religious
group alleged that the meeting was a pretext for religious conversion and later submitted a complaint
to the police.
46. Ans:(a)
Recep Tayyip Erdoğan is a Turkish politician serving as the 12th and current President of Turkey.
He previously served as Prime Minister of Turkey from 2003 to 2014 and as Mayor of Istanbul from
1994 to 1998.
47. Ans:(d)
At present, NATO has 30 members. In 1949, there were 12 founding members of the Alliance:
Belgium, Canada, Denmark, France, Iceland, Italy, Luxembourg, the Netherlands, Norway, Portugal,
the United Kingdom and the United States.
48. Ans:(c)
North Atlantic Treaty Organization (NATO) is a military alliance established by the North Atlantic
Treaty (also called the Washington Treaty) of April 4, 1949, by the United States, Canada, and
several Western European nations to provide collective security against the Soviet Union. NATO is
an international alliance that consists of 30 member states from North America and Europe.
49. Ans:(a)
The seven institutions of the European Union (EU) are seated in four different cities, viz. Brussels,
Frankfurt, Luxembourg and Strasbourg, rather than being concentrated in a single capital city. The
EU agencies and other bodies are located all across the union, but usually not fixed in the treaties.
50. Ans:(c)
RV MTA Oruç Reis is a Turkish research vessel owned by the General Directorate of Mineral
Research and Exploration in Ankara and operated by its division of Geophysical Directorate for
subsea geophysical exploration in shallow waters.
51. Ans:(b)
Crimean Congo hemorrhagic fever (CCHF) is one of the deadly hemorrhagic fevers that are
endemic in Africa, Asia, Eastern Europe, and the Middle East. It is a tick-borne zoonotic viral disease
caused by CCHF virus of genus Nairovirus (family Bunyaviridae).
52. Ans:(c)
Ticks are arthropods that suck blood from animals and humans. They occur around the world and
are important as vectors of a large number of diseases. Among the best-known human diseases
transmitted by ticks are tick-borne relapsing fever, Rocky Mountain spotted fever, Q fever and Lyme
disease. The Congo virus is transmitted through bite of Hyalomma tick, an external parasite, living
by feeding on the blood of mammals, birds etc.
53. Ans:(a)
They are named for Nairobi sheep disease virus, now considered a strain of Dugbe virus. There are
seven species recognized, all of which consist of multiple strains with distinct names. The viruses
are tick-borne although a few can also be transmitted by culicoid flies or mosquitoes.
54. Ans:(b)
The CCHF is a widespread disease caused by a tick- borne virus (Nairovirus) of the Bunyaviridae
family.
55. Ans:(d)
General supportive care with treatment of symptoms is the main approach to managing CCHF in
people. The antiviral drug ribavirin has been used to treat CCHF infection with apparent benefit.
There are no vaccines widely available for human or animal use.
56. Ans:(b)
The Artemis program is a US government-funded crewed spaceflight program that has the goal of
landing "the first woman and the next man" on the Moon, specifically at the lunar south pole region
by 2024.
57. Ans:(a)
The first moon landing occurred on July 20, 1969, on the Apollo 11 mission. The crew of Apollo 11
was Neil Armstrong, Michael Collins and Buzz Aldrin. Armstong and Aldrin walked on the lunar
surface while Collins remained in orbit around the moon.
58. Ans:(b)
ARTEMIS stands for 'Acceleration, Reconnection, Turbulence, and Electrodynamics of the Moon's
interaction with the Sun.
Artemis program that will return astronauts to the Moon for the first time (including the first woman
and the next man) in half a century, including eight scheduled launches and a mini-station in lunar
orbit by 2024.
As a result of Artemis, NASA will be able to establish a sustainable human presence on the Moon by
2028 to uncover new scientific discoveries, demonstrate new technological advancements, and lay
the foundation for private companies to build a lunar economy.
59. Ans:(c)
A ray of light from sun takes 8 minutes to reach earth. Light from moon takes only a second to reach
earth. The Moon is an astronomical body orbiting Earth and is the planet's only natural satellite. It is
the fifth-largest satellite in the Solar System, and by far the largest among planetary satellites
relative to the size of the planet that it orbit.
60. Ans:(d)
Iodic acid, HIO₃. It is a white water-soluble solid. Its robustness contrasts with the instability of
chloric acid and bromic acid. Iodic acid features iodine in the oxidation state +5 and is one of the
most stable oxo-acids of the halogens.
61. Ans:(b)
Arctic council has eight member countries: Canada, Denmark, Finland, Iceland, Norway, Russia,
Sweden, and the United States. Observer status is open to non-Arctic states approved by the
Council at the Ministerial Meetings that occur once every two years. China, India, Italy, Japan,
Singapore ,South Korea, Netherlands, Poland, Spain, United Kingdom, France, Germany are
observers of Arctic council.
62. Ans:(b)
Arctic council has eight member countries: Canada, Denmark, Finland, Iceland, Norway, Russia,
Sweden, and the United States. China, India, Italy, Japan, Singapore, South Korea, Netherlands,
Poland, Spain, United Kingdom, France, Germany are observers of Artic council.
63. Ans:(c)
El Niño is the warm phase of the El Niño–Southern Oscillation and is associated with a band of
warm ocean water that develops in the central and east-central equatorial Pacific, including the area
off the Pacific coast of South America
64. Ans:(d)
The Milne Ice Shelf, a fragment of the former Ellesmere Ice Shelf, is located in the Qikiqtaaluk
Region, Nunavut, Canada. It is the second largest ice shelf in the Arctic Ocean.
65. Ans:(c)
The Union Cabinet, chaired by the Prime Minister, has recently ratified the ban of seven Persistent
Organic Pollutants (POP‘s) listed under Stockholm Convention. With the decision, India is sending
out a positive message to the world that it is active in this area & do not tolerate health &
environmental hazard. After 12 years of negotiations, the Chemical Weapons Convention (CWC)
was adopted by the Conference on Disarmament in Geneva on 3 September 1992. The CWC is the
first disarmament agreement negotiated within a multilateral framework that provides for the
elimination of an entire category of weapons of mass destruction under universally applied
international control.
66. Ans:(a)
The Global Environment Facility was established on the eve of the 1992 Rio Earth Summit to help
tackle our planet's most pressing environmental problems.
67. Ans:(d)
Persistent organic pollutants, sometimes known as "forever chemicals" are organic compounds that
are resistant to environmental degradation through chemical, biological, and photolytic processes.
Because of their persistence, POPs bioaccumulate with potential adverse impacts on human health
and the environment.
68. Ans:(a)
The MoEFCC had notified the ‗Regulation of Persistent Organic Pollutants Rules, on March 5, 2018,
under the provisions of Environment (Protection) Act, 1986.
The regulation prohibited the manufacture, trade, use, import and export seven chemicals which
were already listed as POPs under Stockholm Convention-
(1)Chlordecone
(2)Hexabromobiphenyl
(3)Hexabromodiphenyl ether and Heptabromodiphenylether (Commercial octa-BDE)
(4)Tetrabromodiphenyl ether and Pentabromodiphenyl ether (Commercial Penta-BDE)
(5)Pentachlorobenzene
(6)Hexabromocyclododecane and
(7)Hexachlorobutadiene

QUANTITATIVE TECHNIQUES

69. Ans:(c)
Number of students who passed half-yearly exams in the school
= (Number of students passed in half-yearly but failed in annual exams) + (Number of students
passed in both exams) = (6 + 17 + 9 + 15) + (64 + 55 + 46 + 76)= 288.Also, Number of students who
passed annual exams in the school = (Number of students failed in half-yearly but passed in annual
exams) + (Number of students passed in both exams)= (14 + 12 + 8 + 13) + (64 + 55 + 46 + 76)=
288.Since, the number of students passed in half-yearly = the number of students passed in annual
exams. Therefore, it can be inferred that both the examinations had almost the same difficulty
level.Thus Statements (a), (b) and (d) are false and Statement (c) is true.
70. Ans:(d)
Since the classification of the students on the basis of their results and sections form independent
groups, so the total number of students in the class:
= (28 + 23 + 17 + 27 + 14 + 12 + 8 + 13 + 6 + 17 + 9 + 15 + 64 + 55 + 46 + 76) = 430.
71. Ans:(d)
Pass percentages in at least one of the two examinations for different sections are:
For Section A = (14+6+64)/(28+14+6+64) ×100 = 84/112×100 = 75%
For Section B = (12+17+55)/(23+12+17+55) ×100 = 78.5%
For Section C = (8+9+46)/(17+8+9+46) ×100= 78.75%
For Section D = (13+15+76)/(27+13+15+76) ×100= 79.39%
Clearly,the pass percentage is maximum for Section D
72. Ans:(a)
Total number of students passed in annual exams in a section
= [(No. of students failed in half-yearly but passed in annual exams) + (No. of students passed in
both exams) ] in that section
Success rate in annual exams in Section A= (14+64)/112 × 100 = 69.64%
Similarly, success rate in annual exams in:
Section B = (12+55)/107×100 = 62.62%
Section C = (8+46)/80×100 = 67.5%
Section D = 89/131×100 = 67.94%
Clearly, the success rate in annual examination is maximum for Section A.
73. Ans:(d)
Required difference = (128000 - 107000) = 21000.
74. Ans:(c)
From the line-graph it is clear that the productions of Company X in the years 1997, 1998, 1999,
2000, 2001 and 2002 are 119000, 99000, 141000, 78000, 120000 and 159000 and those of
Company Y are 139000, 120000,100000, 128000, 107000 and 148000 respectively.
Total production of Company X from 1997 to 2002
= 119000 + 99000 + 141000 + 78000 + 120000 + 159000 = 716000.
and total production of Company Y from 1997 to 2002
= 139000 + 120000 + 100000 + 128000 + 107000 + 148000 = 742000.
Difference = (742000 - 716000) = 26000.
75. Ans:(a)
Average number of vehicles manufactured by Company X
=1/6× (119000+99000+141000+78000+120000+159000)= 119333.
76. Ans:(d)
The difference between the productions of Companies X and Y in various years are:
For 1997 (139000 - 119000) = 20000.
For 1998 (120000 - 99000) = 21000.
For 1999 (141000 - 100000) = 41000.
For 2000 (128000 - 78000) = 50000.
For 2001 (120000 - 107000) = 13000.
For 2002 (159000 - 148000) = 11000.
Clearly, maximum difference was in 2000.
77. Ans:(b)
Required percentage= 128000/78000×100 = 164%
78. Ans:(a)

No. of students who are both female and excellent = none.


79. Ans:(b)

Proportion of good students who are male = 22/30 = 11/15 = 0.73


80. Ans:(b)
Proportion of female students who are good = 8/32= 1/4 = 0.25
81. Ans:(c)

No. of students who are both male and good = 22.


82. Ans:(d)

Among average students, the ratio of male to female = 16 : 24 = 2: 3


LOGICAL REASONING

83. Ans:(a)
Statements 1 is implicit (The aim must be to stay with the objective of inclusive growth that was laid
down by the founding fathers of the nation)
2 is also implicit from the passage (The most pressing requirement now is to have an empowered
government.),
Statements 3 and 4 are not mentioned.
Hence, option (a) is the correct answer.
84. Ans:(d)
Inclusive growth can be effected by focusing on delivering the basic services to the deprived
sections of the society (last two lines). Hence, option (d) is the correct answer.
85. Ans:(c)
Statements 1 and 2 cannot be implied from the information given in the passage while statements 3
(creates an empowered ethos for the market, so that individual enterprise can flourish), 4 (steps in to
help those who do not manage to do well for themselves, for there will always be individuals, no
matter what the system, 'who need support and help'.) and 5 (at the same time, plays an important
role in directly helping the poor by ensuring that they get basic education and health services and
receive adequate nutrition and food.) are correct according to passage. Hence, option (c) is the
correct answer.
86. Ans:(d)
Both the statements are wrong according to the passage. Hence, option (d) is the correct answer.
87. Ans:(d)
Main idea Question. The essential message conveyed by the author is the need to rethink the role of
the state in achieving the inclusive growth. Hence, D is correct.
Option A is correct, but it is NOT the central message of the passage. Hence, incorrect.
88. Ans:(d)
The argument introduces the problem of achieving inclusive growth. The solution is to have an
empowered state. Hence (d)
89. Ans:(a)
Sri Lanka is considered the leader and is seen as causing more than double the safe level of
warming (of 2°C)
90. Ans:(d)
Maldives is mentioned as having a rise of 3%.
91. Ans:(a)
Since the present policies have failed to arrest temperature rise.
92. Ans:(a)
Many countries are mentioned to be behind their targets and also the average temperature is set to
raise hence most countries are definitely not attaining their goals.
93. Ans:(b)
The paragraph mentions ―calamitous warming‖
94. Ans:(a)
The passage ends with Qs to be asked in such a scenario. Option A is the most logical sentence to
continue as it is also adding a vital Q in the list. Option B is close, but it is talking about ‗thousands‘
such cases, whereas the passage is focusing on a person committing the ‗rarest of rare crime‘.
Option C can appear somewhere later, but not immediately after the last sentence of this passage.
Option D is too general.
95. Ans:(b)
Option A is incorrect because of ‗only‘. The writer wants us to look for the reasons behind such
rimes.
Option C is against the views expressed in the passage (there can be no denying the fact such
heinous crimes have to be penalized in exemplary fashion).
Option D cannot be inferred.
Only Option C catches the spirit of the passage and the Qs that the author has raised.
Hence, C is correct.
96. Ans:(a)
The author is discussing the situations that make a person turn towards crime, hence, A is correct.
Option B and C are against the passage, as the author mentions that he is satisfied with the
quantum of punishment.
Option D is true as per passage, but is not the central idea of the passage.
97. Ans:(d)
Option A is incorrect as along with poverty, other factors are also mentioned.
Option B is and C are extreme.
Option D can be inferred from the given information. (such pauper juveniles who are robbed off their
childhood and are thrown in unfriendly and pathetic conditions by pernicious state of affairs and
destitution?)
98. Ans:(a)
The passage focuses on how poverty is an important factor in such crimes. Hence, A is correct.
Option B can be a solution, but is not implicit in the passage.
Option C and D are too general, hence incorrect.
99. Ans:(b)
Option (a) is irrelevant; (b) speaks of the same challenges as the passage – children
from disadvantaged backgrounds or no fault of theirs find it harder to get access to basic resources
option (c) is not the correct comparison. Option (d) while addressing an important issue of gender
pay gap loses out to option (b). Hence, (b).
100. Ans:(b)
Options (a), (c) and (d) negate the author‘s arguments. These options talk about a negative impact
on the GDP, about promoting the interest of the already privileged and rich. Option (b) supports the
idea that all children deserve a good education and backs it up with the positive impact that this will
have. Hence, (b).
101. Ans:(d)
The author seems to be a strong advocate of education of children and investment in education. He
states that the statistic going further is bad and that we need to do something to improve education
for all especially those in poor countries/communities. All of this is supported by option (d); (b) talks
about human capital (c) is vague. (a) does not address the universal problem of lack of educational
opportunities for all. Hence, (D).
102. Ans:(c)
The author supports children‘s education. He states that education helps not just the person and the
smaller system of the family but also the larger systems such as communities and countries. Only
option (c) weakens the argument by speaking about the advantage of dropping out of school. Hence,
(c).
103. Ans:(c)
Options A, B and D talk about ways that can help opportunity to education available to all.
Option (c) talks about providing support to students who already have academic advantage; this will
further the gap between those who have and those who do not. Hence, (c).
104. Ans:(d)
Main idea; This question concerns the main point of the passage. A careful examination of the
overall structure of the passage will reveal the main point. In the first paragraph, the author briefly
presents Armen Sarkissian‘s theory and explains why it could not be tested early on. In the second
and third paragraphs, the author describes how a new method allows testing of the theory and
shows how evidence from the testing supports the theory. While the final paragraph acknowledges
that other factors should be considered, the author‘s primary interest in this passage is in presenting
Sarkissian‘s theory and the recently discovered method for testing it.
A- A new research method is described, but no previous method is discussed.
B-As described in the passage, the new method tests and confirms the theory; there is no mention
that the theory is accepted or that the method casts doubt on it.
C-Nothing in the passage suggests that ―instability of data‖ is an issue.
D-Correct. The author presents Armen Sarkissian‘s theory and describes the oxygen isotope method
of testing it.
105. Ans:(c)
Application; The author‘s reaction to the statements about the Armen Sarkissian theory must be
based on how the author treats the theory in the passage. The first, second, and third paragraphs
describe the theory and the use of a new research method to test the theory. The passage states
that data from these tests have established a strong connection between variations in the Earth‘s
orbit and the periodicity of the ice ages, suggesting that the author of the passage believes the
theory is plausible. In the final paragraph, the author points to other factors that might be involved,
suggesting that the theory might not provide a complete explanation.
A-In the last paragraph, the author suggests that because there are still other untested factors that
may have effects on climate, other explanations are possible.
B-Though in the last paragraph the author points to other factors that may be involved, these are not
presented by the author as indicating limitations that diminish the plausibility of the theory—they are
acknowledged merely as possibilities that are not now understood—and nothing else in the passage
suggests that the theory is ―too limited.‖
C-Correct. The author‘s presentation of the theory and the tests of the theory show that the author
finds the theory plausible; the mention of other factors shows the author does not think that all other
explanations have been ruled out, even if they are as yet untested.
D-The theory was a plausible explanation from its beginning, but it was not testable until recently;
scientists would be unlikely to try to devise means to test a theory that did not strike them as
antecedently plausible.
106. Ans:(b)
Inference; To make an inference about the isotope record from ocean sediments, examine what the
passage says about that record. The third paragraph discusses that record and lists its two
advantages. First, it is a global record with remarkably little variation in samples from varied
locations. Second, it is more continuous than the record from rocks. If either of these advantages
were not true, then it is logical to infer that the record would be less useful.
A-According to the passage, the lighter isotope does predominate; this is part of the record and does
not affect its usefulness.
B-Correct. The author states that an advantage of the ocean record is that it is a more continuous
record than that taken from rocks on land. If this were not true, the ocean record would be less
useful
C-If the record were to show that the shifts did not occur every 100,000 years, Armen Sarkissian‘s
theory would be weakened. This impact on the theory does not make the isotope record less useful
to researchers. The record is useful precisely because it can offer evidence to confirm or refute such
theories.
D-This inconsistency would not affect the usefulness of the ocean-water record. Researchers would
simply need to accommodate the fresh-water inconsistency.
107. Ans:(c)
Supporting ideas; The phrase according to the passage suggests that the answer to the question is
most likely stated in the passage. Passage states that the relative volume of land ice can be
deduced from the ratio of oxygen 18 to oxygen 16 in ocean sediments.
A-There is no evidence in the passage about this point.
B-The ocean record is described as more continuous, so it is unlikely to be less reliable. In any case,
reliability is not discussed.
C-Correct. Passage explains that the land-ice volume for a given period can be deduced from the
ratio of two oxygen isotopes.
D-There is no evidence in the passage to support this statement.
108. Ans:(b)
Inference; Any inference about precipitation from evaporated ocean water needs to be based on
what the passage says. Heavier isotopes tend to be left behind when water evaporates from the
ocean surfaces. Therefore, the evaporated water would contain less oxygen 18 and the remaining
ocean water would contain more. It is logical to infer that precipitation formed from this evaporated
water would also contain less oxygen 18.
A.Passage says that the water remaining in the ocean after evaporation has more oxygen 18.
B.Correct. Since the heavier isotopes tend to be left behind, there will be less oxygen 18 in the
evaporated water and in the precipitation that forms from it.
C.The passage suggests that the ocean water evaporates and through subsequent precipitation
helps form the ice sheets, so the amount of oxygen 18 in the ice sheets should be similar to the
amount in the precipitation formed from the evaporated water.
D.The passage does not discuss precipitation formed from water on land.
109. Ans:(d)
Inference; Any inference about calcium carbonate shells needs to be based on what the passage
says about these shells. Lines 24–32 explain the role of these shells in forming sediments and
establishing a chronology for ice ages. The shells were constructed with oxygen atoms drawn from
the surrounding ocean. Lines 29–32 make it clear that if the sediments reveal a higher ratio of
oxygen 18, it is because more oxygen 18 had been left behind when the ocean water evaporated
and contributed to the growth of continental ice sheets. It can thus be inferred that the shells that
make up those sediments must reflect the proportion of oxygen 18 found in the ocean water at the
time they were formed.
A.The only mention of rocks in the passage is a comparison of ―gappiness‖ of the rock and
sedimentary specimen records in lines 38–39; this information does not allow any firm inference to
be made with respect to relative susceptibility to deterioration, though a more continuous record
might be the result of less susceptibility to deterioration.
B.The passage does not make any reference to the relative abundance of these shells during ice
ages; no such inference can be drawn.
C.The only information in the passage that might support this statement is found in lines 29–32, but
that information, about the correlation between oxygen ratios in sediment specimens and land ice,
describes a relation that implies nothing about distributions of such specimens.
D.Correct. The passage explains that oxygen atoms in the surrounding water are one of the building
blocks of calcium carbonate shells. The isotopic composition of the surrounding water changes
during the ice age cycles, so it is logical that the isotopic composition of the shells will change
depending on when they were formed.
110. Ans:(b)

Q faces centre and P is second to the left of Q.


111. Ans:(c)

T is third to the left of V


112. Ans:(d)

R, W, V and Y are sitting in the middle of the sides of the table. S is sitting at the corner.

LEGAL REASONING

113. Ans:(c)
Neha was responsible for the tailoring part and owed duty of care with respect to stitching. As there
is no negligence done with respect to stitching, Neha will not be held liable. Option (b) is incorrect
because Neha did owe a duty towards her client but not in respect to the quality of the fabric.
114. Ans:(b)
Last para defines neighbours as any person who is directly affected by person‘s act. Here, the
toddlers suffering from allergy is a direct resultant of high amount of harmful chemicals in the fabric
and therefore manufacturer will be held liable.
115. Ans:(b)
Aman‘s responsibility is only restricted to selling of the packet of bread and not in production of the
same.
116. Ans:(c)
Tempering with the expiry date and selling stale product will attract liability at Aman‘s end. Here Mr.
Duggal qualifies as Aman‘s neighbour to whom he owes duty of care.
117. Ans:(d)
It‘s the liability of the manufacturer and not the supplier, and Mahesh Babu had not breached his
duty of care. Therefore, Mahesh Babu will not be held liable.
118. Ans:(c)
Here only the seller will be held liable and not the manufacturer as it was sellers fault to make sure
that no damage occurs while the products are in transit.
119. Ans:(c)
This is because fingerprints fall under the exception as the individual is not being asked to provide
anything within their individual knowledge. Therefore, no violation of Article 20(3) in this case.
120. Ans:(d)
This is for the reason that conducting DNA profiling is not a testimonial act and if it does have a
testimonial character then there cannot be witnessing against self. Hence protection cannot be
granted under Article 20(3).
121. Ans:(d)
This is because search of the premises occupied by the accused without the accused being
compelled to be a party to such a search would not be violative of the constitutional guarantee
enshrined in Article 20(3).
122. Ans:(c)
This is because a confession is not violative of Article 20(3) especially when it is made in the court in
front of the magistrate. It would not be valid if it was done under police custody but is considered
acceptable in case of a magistrate.
123. Ans:(b)
as Ram wasn‘t compelled to produce the currency notes as no duress was applied on him.
Moreover, he wasn‘t even an accused at the time the currency notes were seized from him. Hence
in this case the scope of Article 20(3) was not applicable.
124. Ans:(d)
The father was the ostensible owner hence he has a right to alienate the property in anyone‘s favor,
family or outside relations, regardless, however section 41 does not allow the transfer of property
without consideration, hence the gift would be invalid and now lawful, making D the correct option.
125. Ans:(c)
Since Bunoo was the ostensible owner of the property, and at the time of the sale there was no
conflict, it was only after the will was read that such rights were found, Bunoo has every right to
alienate the property and execute the sale deed. And the transferee acted prudently and would have
found all facts in order too, since the burden is on them. Hence C is the answer.
126. Ans:(b)
Since the illegitimate heirs are not the ostensible owners in the first place, as given in the question
itself, it would not be a subject matter of section 41. The prerequisite to invoke section 41 is that the
transfer must be done by an ostensible owner and if the same is not fulfilled hence the court cannot
invoke this section.
127. Ans:(d)
Since the transferee cannot be held responsible for the husband‘s discussion to hold his wife as the
ostensible owner and she was even registered in the name of the owner, he was prudent and
efficient. And the husband cannot claim ownership after the sale as he is barred by estoppel, option
D is the correct answer.
128. Ans:(d)
Section 41 does not apply to land laws, or cases outside of Indian Territory and hence this dispute is
outside its jurisdiction and would not apply to them making D the correct answer.
129. Ans:(c)
By his conduct or consent or otherwise makes the other believe that a particular person has all the
powers over the property as that of the owner over the, including that of alienation, the transferor
should be shown to have been the ostensible owner with the express orimplied consent of the true
owner but the transfer itself need not be with the consent of the true owner. The consent includes
express or implied as well.
130. Ans:(c)
Article 19 and 21, both protect the right to access internet as a fundamental right. However, as each
fundamental right is not absolute and can have reasonable proportionate restriction, the same
clause is attached with the right to internet. Hence considering that there is no blanket restriction,
and they are in lieu of certain messages to curb a social menace of wrongful information that could
cause harm, this act is constitutionally valid.
131. Ans:(b)
The magistrate has exercised his legitimate power under Section 133 of CrPC. However, in terms of
Fundamental Right to internet, the same has to be exercised with proportionate restrictions. So, if
this ban on internet fails the definite time period then it would be unconstitutional as it already has
legitimate purpose.
132. Ans:(d)
The first part need to be addressed is if the law is constitutionally valid. The restriction on internet as
right has not been curbed. The restriction has been placed on certain information which are
misguided and against public interest and health. Hence the ban on hate messages and instigating
violence is a proportionate means. Secondly his act does not amount to hate speech, as it‘s not
a targeted message to instigate violence, but a news report stating facts. Hence option D is the
correct answer.
133. Ans:(b)
In Faheema Shirin v. State of Kerala, the right to internet access was recognized as a fundamental
right forming a part of the right to privacy and the right to education under Article 21 of the
constitution. However this interpretation refers to the right of accessing internet to facilitate
education. It does not make the state obligatory to give devices to access the internet. It merely
means that the state cannot restrict one‘s right to internet if it hampers their right to education with
them. Hence (b) is the correct option.
134. Ans:(b)
Even under the emergency regime, right to internet cannot be restricted indefinitely. It has to be
proportionate to the needs of the hour as an independent fundamental right, it means access to
actionable information, innovation best practices, access to finance and credit facilities, facilitation of
entrepreneurship and an enhanced employment market. In terms of social benefits, internet access
helps by facilitating and enabling an enhanced utility value of primary necessities, internet access
can prove a useful accelerator in all social development objectives.
135. Ans:(a)
Tax is devoid of quid pro quo because it is not imposed by state in return of rendering a specific
service or any benefit by the public authority but in common interest of all.
136. Ans:(c)
The levy goes to the Consolidated Fund of India. The money therein is used for various public
function and not only for maintaining the math. Therefore, there was no quid pro quo. Facts are
incomplete with respect to deciding proportionality.
137. Ans:(b)
The levy is being specifically charged for rendering the services of access to parks and gardens. It
need not be commensurate with the cost in exact arithmetical equivalence.
138. Ans:(b)
Unlawful trade does not fall within the ambit of Article 301. State can restrict in light of public interest.
139. Ans:(a)
Mihika will not succeed because such taxes falls under compensatory taxes which are levied to
facilitate trades. The toll tax and hotel tax are charged in return of services provided by state to
maintain the roads and providing the facility of stay if one is in the state of Uttar Pradesh.
140. Ans:(c)
Article 303(2) provides an exception to make discriminatory laws in case of scarcity in favour of
only Parliament and not state legislatures.
141. Ans:(c)
This is due to this reason that the Act came into force after the act was committed. Therefore, past
acts cannot be criminalized with retrospective effect making Chitamann not guilty for the said act.
142. Ans:(a)
This is simply because Article 20 provides that any increased punishment for the same act for which
the punishment was different at the time of the commission of the act are not constitutional in
nature.
143. Ans:(b)
This is because the timeline of 6 months to fine the tax evaders is a reasonable period. In addition to
this, the fundamental right cannot extend to imposition of fines when it is in the interest of public at
large who is diligently paying all the taxes. Therefore, the evaders have to be fined heavily to prevent
them from avoiding the payment of taxes.
144. Ans:(d)
This is because retrospective application is not justified in case of increasing the punishment but it
can be justified in reducing the punishment as it caters to benefit the individual which can have a
retrospective affect.
145. Ans:(a)
This is because offence of disproportionate assets is different from the actual calculation of
disproportionate assets. The prohibition is only on criminalizing from an act which is not done in this
case. Thus, there is no violation of Article 20.
146. Ans:(c)
This is because the conduct, language and expression of Sashi were derogatory in nature forcing
Ambika to commit suicide. Rajat would not be liable as he did not take any active steps which prove
that he had aided the result in the instant case.
147. Ans:(b)
This is because the offense of abetment by instigation relies on the intention of the individual who
abets and not upon the act which is finished by the individual who has abetted. Therefore, Amit is
liable for abetting Akhila.
148. Ans:(c)
This is because the act of abetment is not dependent on the end result or the impact. It relies on the
intention of the instigator and their act. Whether the abetment has reached to its desired end is not a
matter in question.
149. Ans:(c)
This is because abetment being an offence, the abetment of such an offence is also an offence and
would amount to abetment making Aditya liable for abetment of abetment in the existing case.
150. Ans:(c)
This is because the words articulated in an angry state do not possess the mens rea which is an
essential condition for an offence to be categorized as abetment. Therefore, such an utterance
without intention does not amount to abetment.
Legal Edge 08 Mock

Answers & Details

ENGLISH

1. Ans:(a)
Given all the data in the passage, it can be concluded that air pollution is now the biggest health risk
in India. The rest of the statements are true in the context of the passage but are incomplete in one
respect or another.
2. Ans:(d)
More polluting industries would mean a rise in pollution levels and this directly hits at the
government‟s claim that pollution levels are declining. The rest of the statements also do not support
the government‟s claim but the one that directly contradicts it is option (d).
Option (a) refers to the per capital pollution exposure whereas the government is referring to
average pollution levels, the domains of the two are different. Reopening of the lockdown and rising
pollution levels have nothing to do with the prevailing levels. Option (c) is also a general statement
which does not contradict the government‟s statement.
3. Ans:(c)
COVID-19, a disease for which people with heart and lung disease are particularly at risk of
infection and death, has claimed more than 1, 10,000 lives in India. Although the full links
between air pollution and COVID-19 are not yet known, there is clear evidence linking air
pollution and increased heart and lung disease, creating a growing concern that exposures to
high levels of air pollution during winter months in South Asian countries and East Asia
could worsen the effects of COVID-19.
The passage states that air pollution could worsen the effect of Covid 19. Therefore the most valid
assumption would be option (c).
4. Ans:(d)
The word neonatal refers to an infant who is just born, therefore (d) is the most appropriate answer.
5. Ans:(d)
Although there has been slow and steady reduction in household reliance on poor-quality fuels, the
air pollution from these fuels continues to be a key factor in the deaths of these youngest infants.
6. Ans:(a)
The main conclusion of the passage is one that summarises the essence of the passage gives it a
finality. Options (b), (c) and (d) are true in the context of the passage but not the „main
conclusion‟. The entire passage refers to a tussle between the Andhra State Government and the
High Court (the executive and the judiciary respectively). Option (a) is therefore the best response.
7. Ans:(b)
The allegations over the judiciary have led to acquire political overtones since it is being accused for
being biased against the current CM and his government.
8. Ans:(d)
His allegations would only be weakened if it is proved that the orders passed by the High Court are
good in law. Therefore option (d) is best response. Options (a) and (b) strengthen his argument and
option (c) is just a restatement from the passage, neither strengthens nor weakens his argument.
9. Ans:(b)
The answer lies in the following lines in the passage:
The right thing would probably be for the CJI to order an inquiry into the letter in accordance with the
apex court‟s internal procedure. Regardless of what happens, it may end the recriminations.
10. Ans:(a)
The final lines of the passage lead to this inference stated in option (a).
India can ill-afford a public perception that judges have strong political loyalties. For, that will
undermine faith in an independent judiciary.
If India, the world‟s largest democracy cannot afford to undermine people‟s faith in the judiciary then
there surely must be a link between an independent judiciary and a thriving democracy. Option (b) is
true but it is not too much of an assumption as the conflict is directly stated and there is little
reference to „wide powers‟ of the either sides. Option (c) is beyond the scope of the passage. Option
(d) is misleading and untrue.
11. Ans:(a)
The word derogatory implies something negative being said about a person or thing. Therefore the
most suitable antonym will be „flattering‟.-
12. Ans:(b)
The passage does not refer to malls, only to opening of wholesale markets which are thronged with
large crowds of people.
These have ranged from the opening of wholesale markets and political gatherings to big funerals;
many were infected when places of worship were allowed to be thronged.
13. Ans:(c)
All three are drawbacks in the government‟s strategy to handle Covid 19. They are mentioned
directly in the passage.
14. Ans:(b)
The visibility is clearly in reference to the opportunity to call out to the people and caution them
against the hazards of not wearing the mask.
Mr. Modi‟s appeal, which comes during the Navratri celebrations, and ahead of Dussehra, Deepavali
and other festivals, is to be welcomed, although the opportunity to caution the public was not
grasped early enough. Also, in spite of the call to “mask up” on October 8, as part of
a communication campaign he launched, its visibility has remained low.
15. Ans:(a)
The ultimate aim of the states is to increase economic activity which propels them to present a
picture of near normalcy and rapid recovery.
16. Ans:(b)
Double jeopardy refers to simultaneous disadvantage from two sources, like in this case it is fall in
income and increase in health insurance premium.
17. Ans:(d)
The word „efficacy‟ implies the effectiveness of a particular thing. Therefore „potency‟ is the best
synonym.
18. Ans:(d)
Indigenous refers to originating or occurring naturally in a particular place. Therefore (d) is the best
answer.
19. Ans:(d)
It is important to remember that although more than one option can be correct in the context of the
passage, it is important to pick the right answer. In this case, the passage states that Arce is
Morales‟ handpicked candidate. Therefore there is surely a mentor-mentee relationship between the
two, as is commonly observed in politics. Option (d) is the best response. Even though option (b) is
also correct but the sentence does not imply that.
20. Ans:(c)
Being socialistic means the party is working for the welfare of the poor people. Option (c) is the best
response.
21. Ans:(d)
The following sentence provides the correct answer:
Mr. Arce can keep the galvanised opposition at bay only by continuing MAS‟s socio-economic
“revolution” to expand its support base in a divided country.
22. Ans:(b)
The following sentence provides the answer:
Mr. Arce‟s biggest challenge would be to continue Mr. Morales‟s welfare policies, while keeping the
battered economy on track.
23. Ans:(d)
Legitimacy means validity or admissibility of a particular thing. Therefore „authority‟ is the best
response.
24. Ans:(c)
Coarse implies something which is rough; therefore the most suitable antonym will be smooth.
25. Ans:(d)
If Indian politics is a negotiation with entrenched power structures and multiple bigotries, both
instances underline how and why women continue to be at a disadvantage; and how easy it
is to turn on the tap of prejudice to discredit women in public life.
26. Ans:(a)
The central idea is accurately summarised by option (a) that all political parties use sexist slurs to
create an illusion of victory. The other options are true in the context of the passage but fail to
summarise the central idea.
27. Ans:(b)
The following lines provide us with the answer:
It‟s a trick as old as entitled masculinity: If you are up against a woman, run her down with innuendo
and the cheap shot, and swagger in the illusion that you have won the debate.
28. Ans:(a)
The answer that would be most appropriate is one that deals with consequence. Option (a) deals
with the fact that Kamal Nath, despite making that slur got away with it without any sort of action
taken against him, not even the slightest disapproval.
29. Ans:(a)
The author here means that the women politicians in India don‟t flout the patriarchal norms. They
don‟t cross that line. Hence, the answer will be option (a).
30. Ans:(b)
Deign means to do something that one considers to be beneath one's dignity. Thus, option (b)
seems appropriate and hence is our answer.

GENERAL KNOWLEDGE/CURRENT AFFAIRS

31. Ans:(d)
Solution :„Right of Passage‟, an 800-page study released in August 2017, authored by experts and
published by the Wildlife Trust of India (WTI) identifies and records details pertaining to 101 elephant
corridors across India.
32. Ans:(a)
Solution :Project Elephant was launched in 1992 by the Government of India Ministry of Environment
and Forests to provide financial and technical support to wildlife management efforts by states for
their free-ranging populations of wild Asian Elephants.
33. Ans:(b)
Solution :The elephant corridor is situated in the Masinagudi area near the Mudumalai National Park
in the Nilgris district.
34. Ans:(c)
Solution :World Elephant Day is an international annual event on August 12, dedicated to the
preservation and protection of the world's elephants.
35. Ans:(d)
Solution :It said that the government is fully empowered under the 'Project Elephant' of the Union
government as well as Article 51 A(g) of the Constitution to notify the elephant corridor in the state‟s
Nilgiris district.
Article 51 A(g): It shall be the duty of every citizen of India to protect and improve the natural
environment including forests, lakes, rivers and wildlife and to have compassion for living creatures.
36. Ans:(d)
Solution :The Global Environment Facility was established on the eve of the 1992 Rio Earth Summit
to help tackle our planet's most pressing environmental problems.The Global Environment Facility
(GEF) serves as a financial mechanism for the following conventions:
Convention on Biological Diversity (CBD)
United Nations Framework Convention on Climate Change (UNFCCC)
UN Convention to Combat Desertification (UNCCD)
Stockholm Convention on Persistent Organic Pollutants (POPs)
Minamata Convention on Mercury
37. Ans:(a)
Solution :The MoEFCC had notified the „Regulation of Persistent Organic Pollutants Rules, on March
5, 2018, under the provisions of Environment (Protection) Act, 1986.
38. Ans:(c)
Solution :In 2001, the Stockholm Convention originally covered the 12 POPs of greatest concern,
called the “dirty dozen:” aldrin, chlordane, DDT, dieldrin, dioxins, endrin, furans, heptachlor,
hexachlorobenzene, mirex, PCBs, and toxaphene.They have been banned by the Convention.
39. Ans:(d)
Solution : India had ratified the Stockholm Convention on January 13, 2006, as per Article 25 (4),
which enabled it to keep itself in a default “opt-out” position such that amendments in various
Annexes of the convention cannot be enforced on it unless an instrument of
ratification/acceptance/approval or accession is explicitly deposited with the UN depositary.
40. Ans:(d)
Solution : The regulation prohibited the manufacture, trade, use, import and export seven chemicals
which were already listed as POPs under Stockholm Convention-
(1)Chlordecone
(2)Hexabromobiphenyl
(3)Hexabromodiphenyl ether and Heptabromodiphenylether (Commercial octa-BDE)
(4)Tetrabromodiphenyl ether and Pentabromodiphenyl ether (Commercial Penta-BDE)
(5)Pentachlorobenzene
(6)Hexabromocyclododecane and
(7)Hexachlorobutadiene
41. Ans:(b)
Solution :The Department of Fertilizers (Ministry of Chemicals and Fertilizers) has been ranked 3rd
out of the 65 Ministries/Departments with a score 4.11 on a scale of 5 on Data Governance Quality
Index (DGQI).It has been ranked 2nd amongst the 16 Economic Ministries/Departments.
42. Ans:(a)
Solution :Ministries/Departments were classified in six categories: Administrative, Strategic,
Infrastructure, Social, Economic and Scientific.
43. Ans:(b)
Solution :The Development Monitoring and Evaluation Office (DMEO) was constituted in September
2015 by merging the erstwhile Program Evaluation Office (PEO) and the Independent Evaluation
Office (IEO). It is an attached office under NITI Aayog, aimed at fulfilling the organization‟s
monitoring and evaluation (M&E) mandate and building the M&E ecosystem in India.
44. Ans:(a)
Solution :Central sector schemes:
These schemes are 100% funded by the Central government.
Implemented by the Central Government machinery.
Formulated on subjects mainly from the Union List.
E.g.: Bharatnet, NamamiGange-National Ganga Plan, etc.
Centrally Sponsored Schemes are the schemes by the centre where there is financial participation
by both the centre and states.
Centrally Sponsored Schemes (CSS) are again divided into Core of the Core Schemes, Core
Schemes and Optional schemes.
45. Ans:(d)
Solution :NITI Aayog replaced Planning Commission of India.The NITI Aayog is a policy think tank of
the Government of India, established with the aim to achieve sustainable development goals with
cooperative federalism by fostering the involvement of State Governments of India in the economic
policy-making process using a bottom-up approach.
46. Ans:(b)
Solution :Uttar Pradesh reported the highest number of cases under IPC in 2019. Around 56% of the
cases in UP, i.e. 3.53 lakh cases were registered under IPC. In Maharashtra, two-thirds of the cases
registered are under IPC (3.41 lakh cases), closely followed by Delhi
47. Ans:(d)
Solution :The data for the report is collected by State Crime Records Bureaux (SCRBx) from the
District Crime Records Bureaux (DCRBx) and sent to NCRB at the end of every calendar year under
the reference.
48. Ans:(b)
Solution :The NCRB report uses the population estimates for 2019 on the basis of 2011 Census for
this calculation
49. Ans:(b)
Solution :NCRB, headquartered in New Delhi, was set-up in 1986 under the Ministry of Home
Affairs to function as a repository of information on crime and criminals so as to assist the
investigators in linking crime to the perpetrators.
50. Ans:(a)
Solution :This is the oldest and the most prestigious publication brought out by NCRB. The first
edition of 'Crime in India' pertains to the year 1953 and the latest edition of the report pertains to
the year 2019.
51. Ans:(b)
Solution :The first two assemblies were held in India in 2018 and 2019. The first Assembly of the ISA
was held from 2nd to 5th October 2018 in Greater Noida, National Capital Region, India.The event
was held in collaboration with Second Global REINVEST, ISA India partnership, Renewable Energy
and Investors Meet & Expo and was hosted by GOI
52. Ans:(a)
Solution :According to an ISA statement, representatives of Fiji and Nauru were chosen as the vice-
presidents for Asia Pacific Region of the first treaty-based international government organization
headquartered in India.
53. Ans:(b)
Solution : The launch of the International Solar Alliance (ISA) was announced by H.E. Mr.
NarendraModi, the Hon‟ble Prime Minister of India and H.E. Mr. Francois Hollande, former Hon‟ble
President of France on 30th November 201.
54. Ans:(a)
Solution :the Kalpana Chawla award, named after the American astronaut of Indian origin. The
award recognizes outstanding contribution of scientists and engineers working in the field of solar
energy. The inaugural award in this category was conferred to Dr. Bhim Singh from IIT Delhi (India),
and Dr. AaeshaAlnuaimi (UAE) from Dubai Electricity and Water Authority (DEWA).
55. Ans:(c)
Solution :Nicaragua, a Central American country is the 87th and the latest country to sign the
agreement.
Statement by H.E. VolkanBozkiron the occasion of International Day for the Total Elimination of
Nuclear Weapons– “Seventy-five years ago, the horrific consequences of nuclear weapons were
made undoubtably clear.
56. Ans:(c)
Solution :In 2013, the UN General Assembly (UNGA) declared 26th September to be the
International Day for the Total Elimination of Nuclear Weapons (Nuclear Abolition Day).
Objective: Total elimination of nuclear weapons through enhancing public awareness and education
about the threat posed to humanity by nuclear weapons and the necessity for their total elimination
57. Ans:(a)
Solution :India, Pakistan, Israel and South Sudan have not signed the treaty.North Korea acceded to
the NPT in 1985 and then withdrew in 2003. Thus, North Korea too hasn‟t signed this treaty. India
says that NPT is unfair to the developing nations who aim to solidify their defence and security from
external threat. NPT was brought in response to India‟s “Smiling Buddha” nuclear test in 1974.
58. Ans:(d)
Solution :The nuclear-weapon states (NWS) are the five states—China, France, Russia, United
Kingdom, and the United States—officially recognized as possessing nuclear weapons by the NPT.
59. Ans:(a)
Solution :After the 1998 nuclear test India also enunciated a doctrine of 'No First Use‟ (NFU) of
nuclear weapons.
The doctrine was formally adopted in January, 2003, and says that nuclear weapons will only be
used in retaliation against a nuclear attack on Indian territory or on Indian forces anywhere.
60. Ans:(b)
Volkan Bozkir is a Turkish diplomat and politician who served as the Minister of European Union
Affairs from November 2015 to May 2016, having previously served from August 2014 to August
2015. He has now recently been elected as the President of the United Nations General Assembly.
He is the first Turkish to hold this post.
61. Ans:(a)
Solution :the National Highways Authority of India (NHAI) has decided to use Infrastructure
Investment Trust(s) (InvIT) as a vehicle for mobilising funds for constructing road infrastructure.
62. Ans:(b)
Solution :Bharatmala Pariyojana is a centrally-sponsored and funded Road and Highways project of
the Government of India. The total investment for 83,677 km committed new highways is estimated
at ₹5.35 lakh crore, making it the single largest outlay for a government road construction scheme.
63. Ans:(d)
Solution :InvITs are regulated by the Securities and Exchange Board of India (SEBI) (Infrastructure
Investment Trusts) Regulations, 2014.
64. Ans:(d)
Solution : They have a trustee, sponsors, investment manager and project manager.
Trustee - Certified by SEBI, the trustee has the responsibility of inspecting the performance of an
InvIT.
Sponsors - They are promoters of the company that set up the InvIT.
In case of Public-private partnership (PPP) projects, it refers to the infrastructure developer or a
special purpose vehicle (SPV) holding the concession.
Investment manager is entrusted with the task of supervising the assets and investments of the
InvIT.
Project manager is responsible for the execution of the project.
65. Ans:(c)
At a time when private sector investment in the economy has declined, fund-raising by NHAI and
spending on infrastructure will not only provide a fillip to the economy, but will also crowd-in private
sector investment. So NHAI‟s InvIT offer, which is expected to come soon, is a way for the
government to tap alternative sources of financing to boost public spending in the roads and
infrastructure sector. FDI is not included in this step by the Government.
66. Ans:(b)
The Punjab Government has become the first state government to officially withdraw itself from the
Central Agricultural Bills.
67. Ans:(d)
The Essential Commodities (Amendment) Act, 2020 has been passed to remove cereals, pulses,
oilseeds, edible oils, onion and potatoes from the list of essential commodities.
68. Ans:(c)
The Bills aim to do away with government interference in agricultural trade by creating trading areas
free of middlemen and government taxes outside the structure of Agricultural Produce Market
Committees (APMCs). It will allow farmers an option to sell their produce directly to these new
zones, without going through the middlemen and paying levies such as mandi fees. It sought to
remove stock holding limits as well as curbs on inter-State and intra-State trade, and create a
framework for contract farming.
QUANTITATIVE TECHNIQUES

69. Ans:(c)
700+500×30100=360
70. Ans:(a)
Company P, R and T have more males than females employees,
So we don‟t calculate the ratios for them.
For company Q = 800900=0.89
For company S = 500750=0.67
71. Ans:(d)

72. Ans:(b)

73. Ans:(c)
= 700 + 800 + 650 + 500 + 800 + 500 + 900 + 450 + 750 + 600 = 6650
74. Ans:(c)

75. Ans:(d)
76. Ans:(b)

77. Ans:(a)

78. Ans:(d)
Saurabh‟s Speed = 95 km/hr
Sumit‟s Speed =50 km/hr
Required ratio: 95:50 = 19:10
79. Ans:(c)

Total student from X is 250 + 220 =470


Total student from is Y is 312 + 208 = 520
So required ratio is = 470:520 = 47:52
80. Ans:(a)

At X total student is = 470


At Y total student is =520
At Z total student is =198 + 262 = 460
So maximum student is at Y.
81. Ans:(d)

Total CLAT student from X and Y is 250 + 312 = 562


Total AILET student from Y and Z is 208 + 260 = 470
So the difference is 562 - 470 = 92
82. Ans:(b)
Total student at X is 470.
So total student at W is = ( 470*120/100) = 564

LOGICAL REASONING

83. Ans:(d)
Paragraph 1 - The first line "the underlying principles of the course of study known as historical
sociology" tells us we're about to learn more about a discipline. Towards the middle of the
paragraph, the author says that "sociologists‟ approach to the subject is usually to focus on only one
of these forms". Thus the answer is D. Kristalina wants to combine the two approaches taken by
sociologists in his discipline.
84. Ans:(d)
Towards the last sentence we have this line: "In order to capture the various facets of this mutual
interaction, Kristalina recommends a fourfold structure to which he believes the investigations of
historical sociologists should conform". The only logical answer to follow this would be "because X
(above), historical sociologists will be able to better portray the complex connections between
human agency and history." Hence, D.
85. Ans:(b)
No where in the paragraph does the author say contingency is a form of historical structuring.
Contingency is a condition that influences an individual‟s life in some degree.
It is identities and capacities which contingency impacts, not the social structuring.
86. Ans:(a)
Refer to: 'first, description of the event itself; second, discussion of the social context that helped
bring the event about and gave it significance; third, summary of the life history of the individual
agent in the event; and fourth, analysis of the consequences of the event both for history and for the
individual'
A historical sociologist MUST take into account the WHOLE picture. This includes not just the
subject at hand, but also the entire landscape (what‟s happening surrounding the subject, and what
the impacts are). Only A considers all of these factors.
87. Ans:(d)
In the first paragraph we are explaining what led to Kristalina's form of historical sociology. Hence
the author is giving us the BASIS for her theories.
88. Ans:(c)
Option C is the only case where definition of contingency doesn't hold. The rest are contingencies on
which the person had influence to varying degrees.
89. Ans:(d)
Refer to the first three lines: But, such a view of life, we are told, is intolerable and would destroy
the instinctive energy by which men persist. The way of escape that they have found is through
religion and philosophy. However alien and indifferent the outer world may seem, we are assured by
our comforters that there is harmony beneath the apparent conflict.
Option A- incorrect. Such a view of life will not destroy the energies, it will destroy the INSTINCTIVE
energies.
Option B – incorrect. The way of escape from such a view is through religion and philosophy. So
such a view does not support a life of religion and philosophy.
Option C – incorrect. The anthropocentric view could be the way to escape from „such a view of
life‟. „Such a view of life‟ is not anthropocentric.
Option D – correct. „life in reality is not harmonious, but is fraught with conflict at the fundamental
level‟ and therefore „we are assured by our comforters that there is harmony beneath the apparent
conflict.‟
90. Ans:(c)
Refer to the last sentence - No man is liberated from fear who dare not see his place in the world as
it is; no man can achieve the greatness of which he is capable until he has allowed himself to see his
own littleness.
91. Ans:(b)
Refer to the following sentences from the passage -
When the earth lost its central position, man, too, was deposed from his eminence, and it
became necessary to invent a metaphysic to correct the “crudities” of science.
All the long development from the original nebula is supposed to lead up to man as the
culmination of the process.
The way of escape that they have found is through religion and philosophy.
Hence, B
92. Ans:(c)
By giving the example of First Sailor (an unimportant character in the play, Hamlet), the writer wants
to show that we may not be aware, but our part in the universe may be very small, similar to this
character in Hamlet. Hence, C
93. Ans:(c)
This is an easy question that asks us to identify the implication of a sentence from the second
paragraph. The second paragraph paraphrased would mean that the West takes pride in their
having assimilated the cultures and arts of various other nations. So when the West looks at itself it
sees the Other (or the “reflection of the post war multicultural West”)
Option [A] is incorrect and far-fetched as freedom of expression and egalitarianism are irrelevant to
the context of the passage. Eliminate Option A.
Option [B] is incorrect. The passage talks about foreign writers embracing Western culture at least in
parts. But the reverse is not true. Hence “the west feels that it has embraced cultural values” from
anywhere is incorrect. We can eliminate option B.
Option [C] is likely to be correct. The „whiteness‟ quoted in the question refers to the presence of a
single culture, and paragraph 2, which mentions this term, further tells us that the West extols the
fact that this whiteness no longer exists, particularly with reference to art. Retain this option.
Option [D] is out of context as not only is it not mentioned in the passage, but it also seems to
contradict the flow of ideas in the passage. Paragraph 4 states, “ ... they are the mirror images that
make teh liberal West feel comfortable with itself...” That means “the west feels that it has failed...”
as option D states is incorrect.
Therefore, the correct choice is [C].
94. Ans:(d)
This is a question of medium difficulty. It asks us to identify the author‟s opinion of writers such as
Amish and Naipaul in the context of the West.
Option [A] can be immediately rejected Paragraph 3 the author expresses his view as, “These
(Amish and Naipaul) and may others like them are excellent writers, and people of much
independence of thought and posture.” , Option A is contrary to the passage and gets eliminated.”
Option [B] is incorrect as nowhere in the passage has the writer said anything to justify the option
which states that these authors have a “preference of Western shores to their homelands.” Besides,
in Paragraph 5 the author writes, “An Indian who writes in English does not abandon India per se,”
Thus option B gets eliminated.
Option [C] carries negative overtones because it uses the term „poster-boys‟ which can imply that
the author does not give serious recognition to these writers. This is also contradictory to the
passage.
Option [D], however, can be inferred from the last few lines of paragraphs 3. “ ... if they present
difference, they present just a different aspect of the West” and Paragraph 4, which states, “I repeat
my observation: they are a reflection of the new post-war multi-cultural West. They are the mirror
images that make the liberal West feel comfortable with itself, because it feels that in gazing on them
(and their works) it is reading and championing the Other.”
Therefore, option [D] is the correct choice.
95. Ans:(c)
This is a question of a moderate level of difficulty. It asks us to identify the author‟s major criticism
against the West. In a sense, it asks us to identify the main theme of the passage, since the passage
as a whole is about the author‟s criticism of the West. The main idea of the passage is beautifully
summed up by the writer in these words: “there it stands in front of gilded mirrors, gazing at itself in
admiration” (2nd Paragraph) and the last line of the passage, “And by chanting the mantra of „global‟
literature or multiculturalism, the West conveniently forgets that the reflections it sees in those
mirrors are, after all, its own.” In other words, what the West deludes itself in as „multiculturalism‟ is
merely its own obsession with itself and its own literature.
Option [A] can be ruled out, as it is neither the central theme of the passage nor mentioned
anywhere in it.
Option [B] is not a criticism made by the author per se, because the passage speaks about the
artistic prejudices of the West in general, and its literary prejudices in particular. It does not seek to
examine the definitions of greatness and mediocrity in general.
Option [C] constitutes the very essence of the author‟s criticism that the West is exclusivist and
hypocritical at least as far as literature is concerned and puts on a cloak of inclusiveness. Thus, this
option must be retained.
Option [D] is wrong because the passage does speak highly of writers that the West reveres, so it is
a contradiction to say that the standards of the West are mediocre.
Thus, option [C] is the right answer.
96. Ans:(b)
This is an easy question. Though it might seem difficult at first glance, as it asks us to imagine the
West as person, it is in fact fairly simple, which essentially just asks for a word to describe the West.
Option [A] says „hedonism‟; Hedonism is a philosophy which believes that pleasure is the most
important pursuit of human beings and paramount. The West, as described in the passage, cannot
be said to be hedonistic. So [A] is rejected.
Option [B] says „egocentrism‟, which means an attitude of self-obsession or self-absorption to the
exclusion of others. This is in tune with the way the passage depicts the West, „gazing at itself in
admiration‟. Retain option [B].
Option [C] says „indulgence‟, which means a tendency to be lenient or to gratify something. This also
cannot be inferred from the description of the West in the passage, so [C] is wrong.
Thus, option [B] is the correct choice.
97. Ans:(b)
(B) is essentially paraphrasing the paragraph, which says we know that all responsible scientists
admit that their theories may be incorrect. A scientist who thinks otherwise, such as Dr. Strange, is
not being responsible. Therefore, if you are studying guinea pig behavior (a la Strange) you cannot
responsibly conclude that your theory is undeniable.
(D) is definitely very tempting. But is it something that we could reasonably infer? Let's break down
the structure of the argument: All responsible neurosurgeons ALWAYS accept that their theories
may be incorrect. All X --->Y. (D), however, is saying All Y ---> X. This is not the same thing. Just
because a neurosurgeon accepts that their theories may be incorrect does not make that person
responsible. Surely, there are other criteria. For instance, a responsible neurosurgeon must also be
ethical and organized. On the other hand, if we know a neurosurgeon to be responsible (X), then
he/she will definitely accept that their respective theory may be incorrect (Y).
98. Ans:(c)
All options other than option (c) are exactly opposite to the information mentioned in passage.
C – As more teams work on short term projects, they need more coordination than long term
projects.
Hence, option (c) is correct.
99. Ans:(c)
Explanation: Catharsis, as defined in the paragraph, involves the purging of negative emotions
through watching or experiencing negative events.
There is no such cause-effect relation mentioned in option [A] – the two may be unrelated or the
cause-effect relation may be the other way around, i.e. people who are not nervous by nature may
be more inclined to watch horror movies.
Option [B] fails to state whether seeing violence or tragedy in real life does in fact result in catharsis,
so it is irrelevant.
While [D] does point out a possible loophole in the author‟s argument, it does not state whether
people actually do experience some positive emotion after watching horror or tragedy.
Only [C] directly weakens the argument, by showing that the cathartic effects of watching violence
may not be obvious in the short term, but they do show up in the long term.
Hence, [C].
100. Ans:(d)
In the passage it is stated that rubbish may be relative (second para, 1st line). If rubbish
(unimportant or valueless material) is relative then entire concept of hoarding may or may not exist.
Options A, B and C are irrelevant.
101. Ans:(a)
Refer to - 'there are many habits and behaviors that are unwise but we do not pathologise' while
mentioning why he is not entirely convinced about hoarding being a pathological issue. Option B and
C are individual examples that support but are not the reasons why hoarding is not a pathological
disorder.
102. Ans:(c)
Option A is incorrect because of the use of SURELY. Option B is against the passage.
103. Ans:(c)
The author states that 'rarity in itself cannot transform a pattern of behaviour from a bad habit into a
disorder'. So we can conclude that just because something is rare does not mean that it can be
branded into something new.
104. Ans:(b)
Refer to: 'Those who consider hoarding a disorder will likely point out that hoarding disorder is
associated with distinctive (i.e. unique) patterns of brain activity (i.e. cerebral functioning)'
Option D is against the passage.
105. Ans:(c)
The passage elaborates on the changed perception of the people of India in respect to women‟s
cricket team. It is highlighted through the statements that earlier, fans only spared a few minutes for
watching the game, but now it is an „appointment viewing‟. It has been successful in narrowing down
the pay gap between the men and women‟s teams. (a) is not implied; (b) and (d) are out of scope;
Hence (c).
106. Ans:(c)
The passage mentions that March 8 was an appointment viewing, unlike earlier times, when fans
had limited time for women‟s cricket. According to the author, „The battle may have been lost at the
MCG but the war has been won.‟ This war refers to the one which has begun since ages with the
society, against the predominating notions about cricket is gentleman‟s game, with very less
remuneration for women cricketers. But with March
8, this has changed- „with the best in the business on the mike in Australia and elsewhere it was
every bit built up as the spectacle that it deserved to be.‟ (a) is not the main idea, (b) is incorrect as
the main point of the passage, (d) misses „women‟s cricket‟ the focus of the argument and so it is
less properly worded than C. Hence (c).
107. Ans:(b)
The changed perception of the Indians shows that women cricketers are viewed with respect and
people give importance to the game equivalent to the one played by male cricketers. (a)
corroborates the same notion that women cricketers are held in high esteem, same with (c), and (d)
which connote the demand of the women in blue, whereas (b) rejects and threatens the notion by
stating that there is no craze or demand for the finale of women cricket teams.
Hence (b).
108. Ans:(c)
The existing situation of women‟s cricket is clarified in the statement „the tournament was treated as
a compulsion by broadcasters across the world‟; rather than broadcasting the matches played by
women on TV, the broadcasters avoided them and only aired them under pressure. (a) is incorrect
information, (b) is not a premise, but a repetitive conclusion, (d) is not effective. Hence (c).
109. Ans:(c)
The author supports women cricket throughout the passage. He praises the women cricket team for
displaying unwavering enthusiasm on the field, despite losing the match. (a) cannot be inferred from
the passage; (b) is out of scope as the author nowhere claims that the display of „valour‟ has earned
them recognition; (d) is incorrect as the author states that perception has already changed.
Hence (c).
110. Ans:(b)

111. Ans:(b)
Explanation:
•First let us take a quick scan on the statements
•Take Option A and compare it with conclusion I, All A can never be B. Some A is B. All B is C.
Some C is D (Option A is eliminated)
•Take Option C and compare it with conclusion II Any A which is a D is also C No A is C. All C are
B. Some C is D (Option C is eliminated)
•Now we have Option B and D
•Take Option B and compare with both the conclusions.
Some A are D. All A are C. No B is D

•From the Shaded region it is clear that All A can never be B and Any A which is D is also C. (Both
conclusions Follows, so Option B is the answer)
•In Exam You can stop once you find the answer.
•But, Now let us take Option D also and check.
All A are C. Some C are D. No D is B

•Here All A can never be B may or may not be true


•Therefore, option D doesn‟t follow.
112. Ans:(d)
Explanation:
Q is the son of R but R is not the mother. So, R is the Father of Q. P is married to R
So, P is the wife of R and the mother of Q, X is the daughter of P and hence of R and so she is the
Sister of Q, Y is the brother of R and Z is the brother of P.
Clearly. Y is brother of R who is a male. So, Y and R are a pair of brothers.

LEGAL REASONING

113. Ans:(a)
Explanation: In case of Rudal Shah v. State of Bihar, liability was imposed on the state for wrongful
detention. Similarly, in the present case compensation shall be paid to Deepak for wrongful
detention.
114. Ans:(c)
Explanation: As threatening in terms of penal provisions of law will not qualify as threatening which
is acknowledged by the Supreme Courts in its various judgments.
115. Ans:(d)
Explanation: As per Nandani Sathpathy v. P.L. Dani, a person while being examined under Section
161 of the CrPC is not required to answer those questions that have a tendency to self-incriminate.
Therefore, in the present case, if Rajmohan refuses to answer any question that would be self-
incriminatory in nature, he can do so as per law.
116. Ans:(c)
Explanation: As per the judgment of Supreme Court in Nilabati Behera v. State of Orissa, the safety
of the inmate must be maintained by the police authority, in custody. Therefore, despite the death
not caused due to any brutality or cruelty by Police Officers, the safety of Rizwaan was their
responsibility and hence makes them liable.
117. Ans:(d)
Explanation: The Supreme Court while deliberating upon the case of D.K. Basu v. State of Bengal
recognized the right of the prisoner against custodial torture deaths in police custody. So, in the
present even torture on terrorist is impermissible.
118. Ans:(d)
Explanation: There is no need to conduct a proper trial, as the magistrate just needs to ensure that
there is prima facie reason to presume that a triable offence has been committed by the accused to
proceed with the framing of charges. In the present case Manu Sharma is not convicted but only
being charged under relevant provisions of Indian Penal Provisions, for the court to proceed with the
trial.
119. Ans:(b)
Explanation: The court at the stage of framing of charges only needs to see whether a prima facie
case is made out against the accused to proceed with the trial, this is not a trial and thus no need to
enquire the case elaborately and decide on conviction.
120. Ans:(a)
Explanation: Section 227 of Code of Criminal Procedure provides for discharging of the accused
persons if there are insufficient grounds to proceed against them. Likewise, in the present case, if
the prosecution could not make out a prima facie case against the accused, the court can discharge
the accused under section 227.
121. Ans:(a)
Explanation: For framing the charges under Section 228 of the Code of Criminal Procedure, the
judge is not required to record detailed reasons. Hence, in the present case, the court is not required
to record reasons.
122. Ans:(d)
Explanation:The judge may proceed with framing the charges against an accused under Section
228 if, after perusal of the records and documents, there is prima facie reason to presume that a
triable offence may have been committed by the accused. The prima facie reason of Balu being
accused of triable crime has been made by the prosecution and there is no need to prove conviction
at this stage of criminal trial.
123. Ans:(a)
The passage states that a minority whether it be religious or linguistic has the right to establish and
administer their own schools. In this case however, it is not clear as to whether Charminder belongs
to such minority or not. Thus, it cannot be determined whether Charminder has the right to do so.
124. Ans:(d)
The passage states that a minority whether it be religious or linguistic has the right to establish and
administer their own schools. In this case however, it is not clear as to whether Charminder belongs
to such minority or not. Thus, it cannot be determined whether Charminder‟s school falls within
Article 30(2).
125. Ans:(d)
Explanation: The passage states that Article 30 should serve a purpose of protecting their religion,
language or culture i.e. the community which establishes such a school. In this case, the dance form
is not exclusive to the Muslim community and thus it should not fall under Article 30.
126. Ans:(d)
Explanation:The passage states that Article 30 should serve a purpose of protecting their religion,
language or culture i.e. the community which establishes such a school. In this case, the dance form
is not exclusive to the Muslim community and thus it should not fall under Article 30.
127. Ans:(c)
Ex: The passage states that Article 30 should serve a purpose of protecting their religion, language
or culture or the general development of their community‟s children. In this case, a school on English
language is good for development of Muslim community children and thus they fall under Article 30
128. Ans:(a)
Ex: The passage states that for a benefit to be availed under Article 30, a person has to belong to
a religious/linguistic minority. Even though Article 29 states “any section” the same is only valid for
Article 29 and cannot be extended to Article 30.
129. Ans:(c)
Explanation: Doctrine of frustration is effective when the contract does not stipulate a
force majeure clause but an impossible even happens making contract impossible to perform
whereas, in case of contingent contract, the future event on whose happening the contract is
dependent is mentioned in the contract itself.
130. Ans:(b)
Explanation: (b) Continuing the lease agreement for a period of 4 years did not become an
impossible act. The interruption was temporary and does not frustrate the lease for rest of
the 3 years.
131. Ans:(b)
Explanation: (b) Since both the parties had knowledge that license needs to be obtained for
the use of trawler, the license becomes foundation of the contract. But in this case, Maritime
Fish Co. itself excluded “Rosemary” from the license and hence, it is a frustration induced by
itself.
132. Ans:(d)
Explanation: (d) The article makes clear distinction between commercial difficulty and
impossibility and provided that former is not excused under Section 56. Just because
performance of contract becomes expensive and unprofitable, it does not render it
impossible.
133. Ans:(d)
Explanation: (d) Winter in Delhi in December is a certain event which is bound to happen.
Contingent contracts are those which are dependent on an “uncertain” future event.
134. Ans:(a)
Explanation: (a) The foundation of the contract was that the leased taxi stands would be used
by the taxi drivers and Hakim Singh would recover fees from them. But due to circumstances,
taxi stand wasn’t used and Hakim Singh can recover the amount from Municipal Committee
under Section 65.
135. Ans:(d)
Explanation: Mere exhibiting of nude body is not sufficient to attract the mischief of Section
509 I.P.C. It must be proved that such an act was committed with the intention to insult the
modesty of a woman. In the present case, facts are not sufficient to establish the intention.
136. Ans:(a)
Explanation: (a) Here, Street offences Act meant to prevent public from being solicited when
walking on road. The solicitation from windows and balconies fall under the mischief which
the Act wanted to rectify.
137. Ans:(c)
Explanation: (c) A sleeping woman is capable of possessing modesty. Pankaj did not have
intention to insult Madhurima’s modesty when he entered her house but he had the intention
when he saw her lying on bed and touched her legs.
138. Ans:(b)
Explanation: (b) Stalking is an offence which is established on constant approach and
monitoring of the use of internet. In this case, the facts are insufficient to establish that
Priyansh has been monitoring Madhuri’s use of internet. He had merely accessed her profile
to help him make his selection for the post of his private secretary.
139. Ans:(a)
Explanation: (a) The Act sought to prevent danger to anyone on highway due to driving under
the influence of alcohol. A drunk person riding is also capable of posing the danger to life of
others as well as his or hers. Therefore, applying mischief rule, Rose can be charged.
140. Ans:(a)
Explanation: An infant is also capable of possessing modesty as observed in the article.
When such gestures can be perceived as one which is capable of shocking the sense of
decency of a woman, it would amount to insult of modesty of woman even though the baby
itself is incapable of understanding it.
141. Ans:(c)
Explanation:Author opines that free criticism of judiciary would strengthen the institution as
an independent body. Therefore, most contradicting argument would be that free speech would
instill fear in the court which would affect its independence. None other argument is a strong as this
one, therefore Option C is the most appropriate answer.
142. Ans:(a)
Explanation: Darshika created an obstruction of administration of justice by threatening the
counsel of the other party. This would hinder him from appearing before the court and fairly
conduct the proceedings. It is not necessary that only judges of the court are affected for contempt
of court.
143. Ans:(a)
Explanation: Though strike by lawyers in itself is not criminal contempt of court but when it
prevented lawyers, litigating parties and judges form approaching the court, it amounts to
interference with administration of justice.
144. Ans:(d)
Explanation:Mr. Bhakti Bhoshan’s remark does not scandalize the court or him nor it
interfered with the administration of justice. It is merely a remark against him in his personal
capacity which is within the ambit of free speech.
145. Ans:(b)
Explanation: The court has clearly provided that freedom of speech and expression is not
absolute and cannot be used to scandalize court or its officers. Medha’s article clearly
scandalized both the court and its 3 judges by levelling allegations of personal vendetta and
intolerance.
146. Ans:(c)
The first remedy to any judgement given by the Supreme Court is to file an appeal for a review
petition of the same judgment. A curative petition is the last resort when such appeal and review also
fail to provide satisfactory and equitable verdict to the petitioner. Hence Shyam should first file for
review of the original verdict.
147. Ans:(c)
Despite the fact that Shyam and Sundar‟s case might fulfill other criterion of right to file for a curative
petition, it still missies on one major essential. They have not yet exhausted every remedy available
to them under the right to appeal and hence cannot claim the last resort to seek justice. A review
petition or a fresh case to change the judge would have the same effect of looking in to errors of
justice delivery before a curative petition.
148. Ans:(b)
The passage clearly states that such kind of petitions (curative) must be rare instead of regular. A
curative petition is a way to ask the court to review and revise their own decision even after a review
petition is dismissed or used. Hence the bench formed in C is not in question. There are no appeals
from apex court decision but there is a provision of review which can be exercised. Hence A is not
the option and B is the correct answer. D is again not the option as it speaks of fresh suit or a
reverse tier appeal which is not provided for.
149. Ans:(a)
As given in the passage, it would be heard by and reviewed by the three senior-most judges of the
court alongside the judges who passed the judgement, and if the majority feels that there was a
violation, the curative petition would be heard by the same bench. Hence A is the correct answer.
150. Ans:(d)
Raghav is raising a plea before the Supreme Court. Because it‟s an ordinary petition and not a
curative one, the above given essentials in the paragraph do not apply. Hence, D is the correct
answer.
Legal Edge 09 Mock

Answers & Details

ENGLISH

1. Answer: (b)
Explanation: In the very first line of the passage the author is making it clear that humans together
with computer can make great business. Hence, B is the correct answer.
2. Answer: (a)
Explanation: It is clearly mentioned in the passage that the author and his company survived the dot
com crash. Hence, A is the correct answer as it cannot be inferred from the passage.
3. Answer: (c)
Explanation: Author and his teams used the hybrid approach where the computer would flag the
most suspicious transactions, and human operators would make the final judgment as to their
legitimacy. Hence, C is the correct option.
4. Answer: (a)
Explanation: Bogus means fake, therefore, the most opposite word is real. Hence, A is the correct
answer.
5. Answer: (c)
Explanation: Alliteration is the repetition of the beginning sounds of neighbouring words.
Anaphora is a technique where several phrases or verses begin with the same word or words.
Hyperbole uses exaggeration for emphasis or effect. Here, Max has exaggerate himself to be the
Sherlock Holmes. Hence, C is the correct answer.
A simile is a comparison between two unlike things using the words "like" or "as."
6. Answer: (d)
Explanation: According to the passage under no circumstances the CEO of the early stage startup
should be paid more than $150,000 even if he was paid more than that in his early job or has a large
mortgage or have hefty private school tuition fee.
7. Answer: (c)
Explanation: According to the author the high pay of an early stage CEO will make him to maintain
status quo in the company along with maintaining his own salary and will not make him work on the
surface problems with the other workers. Hence, C is the correct answer.
8. Answer: (c)
Explanation: According to the author, cash bonus is better than cash salary because the former is
dependent upon the good work of an employee unlike salary which does not have any motivation.
Hence, C is the correct answer.
9. Answer: (a)
Explanation: An adverb is a word or an expression that modifies a verb, here, fix is the verb which is
being modified by the word aggressively. Hence, A is the correct answer.
10. Answer: (b)
Explanation: Emulate means to match or surpass (a person or achievement), typically by imitation.
Hence, B is the correct answer.
11. Answer: (a)
Explanation: According to passage, Shakespeare is superior guide because people become
obsessed with their competitors for career advancement. Then the firm themselves become
obsessed with their competitors in the marketplace. Amid all the human drama, people lose sight of
what matters and focus on their rivals instead. Hence, A is the correct answer.
12. Answer: (d)
Explanation: Marx believed that the ideology and perspective of people are different and therefore,
the actual fight is the fight of difference. Therefore, D is the correct answer.
13. Answer: (b)
Explanation: Retrospect means looking back on or dealing with past events or situations. Hence, B
is the correct answer.
14. Answer: (b)
Explanation: Author clearly mentions in the passage that competition is war and not business,
therefore, B cannot be inferred from the passage. Hence, it is the correct answer.
15. Answer: (c)
Explanation: Verbs are the action words in a sentence that describe what the subject is doing. In the
passage, the word is used in the context of action of Apple coming in between the other two and
earing more. Hence, C is the correct answer.
16. Answer: (b)
Explanation: According to the passage, ability of business to generate revenue depends on cash
flow and Twitter is expected to create monopoly in the future thereby expected to generate huge
cash flow in the future. Hence, B is the correct answer.
17. Answer: (b)
Explanation: Author is trying to say that escaping the competition by creating something new will
lead to monopoly but it is only good when the monopoly is going to survive for some time to reap its
benefit in the market. Hence, B is the correct.
18. Answer: (a)
Explanation: Tech companies always earn delayed revenues cause they take time to build value to
their product. Most of their value come after 10 to 15 years. Hence, A is the correct option.
19. Answer: (b)
Explanation: Bizarre means very strange or unusual. Therefore, ordinary is antonym.
20. Answer: (a)
Explanation: Use "a" or "an" if the title is not a specific title. Use "the" if a specific person has a title
or if only one person has a title. Don't Use "a," "an," or "the" if the person's name is given. Use "the"
if the name of the country is plural or indicates a group (of states, islands, etc.) A is only used in front
of consonants and an is used only before vowels. Hence, A is the correct answer.
21. Answer: (d)
Explanation: Personification gives human qualities to non-living things or ideas and A simile is a
comparison between two unlike things using the words "like" or "as." Both of them are used,
therefore, D is the correct answer.
22. Answer: (b)
Explanation: The option B is the third notice of the president and not the second notice. Hence, B is
the correct answer.
23. Answer: (a)
Explanation: Impresario means a person who organizes and often finances concerts. Hence, A is the
correct answer.
24. Answer: (a)
Explanation: Finally after the third notice the nawab executed as threatened means he ran over a
person after which when he sent the car to receive Jamila, nobody came into the way, and therefore
the nawab says, “'No trouble; the car is respected now. Progress has occurred. Hence, A is the
correct answer.
25. Answer: (a)
Explanation: Peremptory means insisting on immediate attention or obedience, especially in a
brusquely imperious way.
26. Answer: (b)
Explanation: The passage does not mentions if any one of the daughter was more beautiful than the
other. Hence, B is not true.
27. Answer: (a)
Explanation: According to the author, coconut is the first inhabitants of the city. Which means the
coconut work is still thriving in the city. Hence, A is the correct answer.
28. Answer: (c)
Explanation: Option C is related to Mumba Devi and not Ganesh, hence, C is the correct answer.
29. Answer: (c)
Explanation: According to the passage, whenever bad time came to Bombay, some nightwalker
would always report that he had seen the statue of Shivaji Maharaj moving. Hence, C is the correct
answer.
30. Answer: (b)
Explanation: Languid means having or showing a disinclination for physical exertion or effort. Hence,
B is the correct answer.
31. Answer: (a)
Explanation: Alliteration is the repetition of the beginning sounds of neighboring words. Here,
constancy of crab-catchers is alliteration. Hence, A is the correct answer.
Anaphora is a technique where several phrases or verses begin with the same word or words.
A metaphor makes a comparison between two unlike things or ideas.
A simile is a comparison between two unlike things using the words "like" or "as."
32. Answer: (b)
Explanation: Hurl means to throw or impel (someone or something) with great force.

GENERAL KNOWLEDGE/CURRENT AFFAIRS

33. Ans- c
Armenia and Azerbaijan have agreed to implement a ceasefire from 12pm (Moscow time)
on November 1, over two weeks after the two nations started clashes over the breakaway Nagorno-
Karabakh region. The ceasefire announcement was made by Russian Foreign Minister Sergei
Lavrov, who held negotiations with Armenian Foreign Minister Zohrab Mnatsakanyan and his Azeri
counterpart Jeyhun Bayramov, in Moscow.
34. Ans- a
Azerbaijan is a dialogue-partner of the Shanghai Cooperation Agreement or SCO.
35. Ans- a
Ilham Heydar oğlu Aliyev is the fourth president of Azerbaijan.
36. Ans- a
Verónica Michelle Bachelet Jeria is a Chilean politician who has served as United Nations High
Commissioner for Human Rights since 2018. She also previously served as President of Chile from
2006 to 2010 and 2014 to 2018 for the Socialist Party of Chile, she is the first woman to hold the
Chilean presidency.
37. Ans- d
Though the Transcaucasian region was annexed by USSR in 1917, the borders of Georgia,
Azerbaijan and Armenia were drawn up in 1920. It was in this year that Nagorno-Karabakh
Autonomous Region was established within Azerbaijan by the then Soviet Union.
38. Ans- c
Sudan has recently signed the peace-deal with Israel.
39. Ans- b
Abdalla Hamdok is a public administrator who became the 15th Prime Minister of Sudan. Prior to his
appointment, Hamdok served in numerous national and international administrative positions.
40. Ans- c
Mahmoud Abbas is the President of Palestine. Any peace deal signed by Israel with an Arab country
will be a disadvantageous position for Palestine. Hence, option (a) can‟t be correct. Jens Stoltenberg
is the Secretary General of NATO, hence, option (b) can‟t be correct. This deal might have helped
with some voters for Donald Trump during the presidential voting in US. Thus, option (c) is the
answer.
41. Ans- d
US President Donald Trump had hosted Israeli Prime Minister Benjamin Netanyahu and foreign
ministers of the United Arab Emirates and Bahrain for the signing of the historic Abraham Accord,
which is the first Arab-Israeli peace deal in 26 years.
42. Ans- b
Iraq is a member of the Persian Gulf. The Arab states of the Persian Gulf are the seven Arab states
which border the Persian Gulf, namely Bahrain, Kuwait, Iraq, Oman, Qatar, Saudi Arabia and the
United Arab Emirates.
43. Ans- c
The Permanent Court of Arbitration is an intergovernmental organization located in The Hague,
Netherlands.
44. Ans- c
Finance minister Pranab Mukherjee in 2012 retrospectively amended the Income Tax Act to
neutralise the apex court‟s verdict.
45. Ans – b
One of the major factors for the Court of Arbitration to rule in favour of Vodafone was the violation of
the BIT and the United Nations Commission on International Trade Law (UNCITRAL). Article 3 of the
arbitration rules of UNCITRAL, among other things, says that “constitution of the arbitral tribunal
shall not be hindered by any controversy with respect to the sufficiency of the notice of arbitration,
which shall be finally resolved by the arbitral tribunal”.
46. Ans- c
Hutchison Whampoa Limited was an investment holding company based in Hong Kong. It was a
Fortune Global 500 company and one of the largest companies listed on the Hong Kong Stock
Exchange.
47. Ans- d
In financial terms, the verdict is of limited consequence to either side as the aggregate tax liability of
about Rs 20,000 crore was not realised. But India‟s reputational loss is incalculable.
48. Ans- c
The writ petition was filed in the year 2017.
49. Ans- c
Although the dam is located in Kerala, it is operated by Tamil Nadu following an 1886 lease
indenture for 999 years (the Periyar Lake Lease Agreement) that was signed between the Maharaja
of Travancore and the Secretary of State for India for the Periyar Irrigation works.
50. Ans- d
The Periyar River is the longest river in the state of Kerala with a length of 244 km. It is also known
as „Lifeline of Kerala‟ as it is one of the few perennial rivers in the state.
51. Ans- b
The Central Water Commission, under the Government of India, conducted a study and
recommended lowering the water stored in the dam‟s reservoir to 136 feet from 142 feet.
52. Ans- a
The Dam was constructed between 1887 and 1895.
53. Answer: B
Solution: The test was carried out by the Defence Research and Development Organisation (DRDO)
from a defence facility off Odisha coast
54. Answer- C
Solution : The Defence Research and Development Organisation is an agency under the
Department of Defence Research and Development in Ministry of Defence of the Government of
India formed in 1958. Headquarter is DRDO Bhavan, New Delhi.
55. answer- B
Solution: The BrahMos (designated PJ-10) is a medium-range ramjet supersonic cruise missile that
can be launched from submarine, ships, aircraft, or land. It is the fastest supersonic cruise missile in
the world.
56. Answer: A
Solution: Shaurya is a land variant of short-range Submarine Launched Ballistic Missile (SLBM) K-15
Sagarika, which has a range of at least 750 kilometers.
It is capable of carrying payloads of 200 kg to 1000 kg.
It is a surface-to-surface tactical missile
57. answer- D
Solution: These submarines can not only survive a first strike by the adversary but also can launch a
strike in retaliation thus achieving Credible Nuclear Deterrence.
58. Answer- C
Solution: CarbonCopy and Respirer Living Sciences, two environmental organisations, have
released a dashboard which presents a comparative picture of particulate matter (PM) for 122 Indian
cities since 2016.
59. Answer- A
Solution: It is a National Air Quality Monitoring Programme (NAMP) based dashboard, built on data
from the Central Pollution Control Board‟s National Ambient Air Quality Monitoring (NAAQM)
Network which was started in 1984-85 and covers 344 cities/towns in 29 states and 6 UTs.
60. Answer- D
Solution: Noida ranked the worst with 119, followed by Agra, Delhi, Lucknow, Ghaziabad,
Muzzaffarpur, Kanpur, Chandigarh, Howrah and Kolkata.
61. Answer- B
Solution: Delhi ranked as the most polluted state on an average of 3 years‟ PM10 monitoring data,
followed by Jharkhand and Uttar Pradesh.
62. Answer- D
Solution: SAFAR: System of Air Quality and Weather Forecasting and Research, known as
"SAFAR", for greater metropolitan cities of India to provide location specific information on air quality
in near real time. It was introduced by the Ministry of Earth Science.
AQI: Air Quality Index (AQI) is a tool for effective communication of air quality status to people in
terms, which are easy to understand. There are six AQI categories, namely Good, Satisfactory,
Moderately polluted, Poor, Very Poor, and Severe. AQ sub-index has been evolved for eight
pollutants (PM10, PM2.5, NO2, SO2, CO, O3, NH3, and Pb) for which short-term (upto 24-hours)
National Ambient Air Quality Standards are prescribed.
63. Answer- A
Solution Mizoram borders Assam‟s Barak Valley and the boundary between present-day Assam and
Mizoram is 164.6 km long. Both states border Bangladesh.
64. Answer- C
Solution The Assam-Mizoram dispute stems from a notification of 1875 that differentiated Lushai
Hills from the plains of Cachar, and another of 1933 that demarcates a boundary between Lushai
Hills and Manipur.
65. Answer- D
Solution: Residents of Lailapur village in Assam‟s Cachar district clashed with residents of localities
near Vairengte in Mizoram‟s Kolasib district.
Earlier in October 2020, a similar clash took place on the border of Karimganj (Assam) and Mamit
(Mizoram) districts.
Mizoram civil society groups blame “illegal Bangladeshis” (alleged migrants from Bangladesh) on the
Assam side.
66. Answer- B
Solution: During British rule, Assam included present-day Nagaland, Arunachal Pradesh and
Meghalaya besides Mizoram, which became separate states one by one.
Currently, Assam has boundary problems with each of them.
67. Ans- a
The Barak River flows 900 kilometres through the states of Manipur, Nagaland, Mizoram and Assam
in India and into the Bay of Bengal via Bangladesh. Of its length 524 km is in India, 31 km on the
Indo–Bangladesh border and the rest is in Bangladesh.

QUANTITATIVE TECHNIQUES

68. Ans: (d)


When an employee aged 20 years joins, as the average age of the current employees is much more
than 20, the average age of the group will decrease. Similarly when a 55 years old employee retires,
as his age is more than the average age of the group, the average age of the remaining members
will drop.
∴ The new employee joined the finance department in 2009, the new employee joined the HR
department in 2011. the 55 years old employee retired from the marketing department in 2009 and
the new employee joined in 2010 and the new employee joined the operations department in 2009.
The total age of all the three persons on April 1 2008
=44.33×3=133
The age of Naveen on that day = 47
The age of Deepak on that day = 44
The age of the third person on that day = 42
∴ His age on April 1, 2013 = 47.
69. Ans: (c)
When an employee aged 20 years joins, as the average age of the current employees is much more
than 20, the average age of the group will decrease. Similarly when a 55 years old employee retires,
as his age is more than the average age of the group, the average age of the remaining members
will drop.
∴ The new employee joined the finance department in 2009, the new employee joined the HR
department in 2011. the 55 years old employee retired from the marketing department in 2009 and
the new employee joined in 2010 and the new employee joined the operations department in 2009.
The new employee joined the marketing department in 2010.
70. Ans: (c)
When an employee aged 20 years joins, as the average age of the current employees is much more
than 20, the average age of the group will decrease. Similarly when a 55 years old employee retires,
as his age is more than the average age of the group, the average age of the remaining members
will drop.
∴ The new employee joined the finance department in 2009, the new employee joined the HR
department in 2011. the 55 years old employee retired from the marketing department in 2009 and
the new employee joined in 2010 and the new employee joined the operations department in 2009.
The new employee joined the operations department on 1st April 2009
∴ His age on 1st April 2011 = 20 + 2 = 22 years.
71. Ans: (b) 2x/15 + 3x/20 + x/5 + 2x/15 + x/20 + x/4 +K=x
∴ 55x60 +K=x
∴ K = x - 55x/60 = 5x/60 = x/12
72. Ans: (a) Chemistry = x/5 + 3x/20 + x/20 + x/12 = 29x/60
Physics = 2x/15 + 3x/20 + x/20 + 2x/15 = 7x/15
∴Diff = 29x/60 - 7x/15 = 29x-28x/ 60 = x/60
∴ x/20 = 15∴ x = 300
So, Diff = x/60 = 300/60 =5
73. Ans: (d) Students who passed in exactly one paper
= 2x/15 + x/5 + x/4 = 7x/12
Students who passed in exactly two papers
= 3x/20 + 2x/15 + x/12 = 11x/30
∴ 11x/30 = 110 ∴ x = 300
So, 7x/12 = 175
74. Ans: (b) Students who passed in at least two papers
= 3x/20 + 2x/15 + x/12 + x/20 = 5x/12
Total Number of students = x
∴Reqd% = x 100 = 125/3 = 41/ 23 %
75. Ans: (d) Students who passed in only Maths = x4
Students who passed in all three papers = x20
Required % = (x/4- x20)x/20 x 100 = 4x/20 x 20/x x 100
= 400
76. Option C
Solution: Anant Vihar – Aligarh = 125 kmAligarh – Kanpur = 434-125 = 309 kmKanpur – Allahabad
=628-434 =194 kmAllahabad – Mughal Sarai = 782-628 = 154 km
77. Option C
Solution:Distance = 628-434 = 194 kmTime = 16:45 – 13:55 = 2 hours 50 minutes = 170 minsSpeed
= 194/170 *60 = 68.47 kmph
78. Option B
Solution:Departs from Aligarh at 9:21 , arrives at Kanpur at 13:45. Difference = 4 hour 24 minutes
79. Option D
Solution:Total passengers travelling from Aligarh-Kanpur = 400-200+251 = 451Hence ratio = 451 :
203
80. Option B
Solution: New Time=19:45 +28 -5 = 20:08

LOGICAL REASONING

81. Answer: (b)


Explanation: Option a) and c) strengthens the editors argument as they show that the company has
lost control and is mismanaged while option d) is not much relevant. Option b) weakens the editors
argument as the production is not slowed down because all the finished products in the inventory
been assigned to orders received from customers.
82. Answer: (a)
Explanation: Option b is a very far-fetched assumption and not right in the current context.
Option c and d are not the correct answer as well because the author cannot give the statement
about a particular incident based on the generality of the market.
Option a is the correct answer as it is possible that the editor is not aware about the day to day of the
company to understand the rising of the inventory of finished product.
83. Answer: (b)
Explanation: Option c and d are the assumptions, they are mere statements and irrelevant to the
problem at hand. Option a is will not lead to any assumption to claim mismanagement in the
company. Option b means that the inventory of finished product is rising and there is no stop at the
production which mean the company is producing more than it is required to, which is a sign of
mismanagement. Therefore, b is the correct answer.
84. Answer: (b)
Explanation: The company is mismanaged is a conclusion advanced by the editor in the form of a
claim supported by the reasoning. Hence, (b) is the correct answer. It can be understood that it is the
conclusion as the author uses the phrase “citing as evidence” the production has not slowed even in
the event of rise of inventory of finished product.
85. Answer: (d)
Explanation: Option a will weaken the editors argument as it leads to conclusion that the production
is going on at a decent rate. Option b and c is does not strengthen the argument either. Option d
means that the all the budgets is being pushed into production where is not demand and there is
budget cut in the other areas, which means the company is not managed properly. Hence, d is the
correct answer.
86. Answer: (d)
Explanation: Rationale- Option A is not correct because Ankit does not prefer anything but he is just
making his argument. Option B is not correct because according to Ronak war and peace are
mutually exclusive. Option C is not correct because nuclear arm treaty is to secure peace.
Therefore, option D is the correct answer.
87. Answer: (a)
Explanation: Rationale- Ankit assumes that Spending money on war research and arms means
preparation for war and to attack other countries. Therefore, option (A) is the correct answer.
88. Answer: (b)
Explanation: Rationale- Ankit argues that Russia and United States are preparing for both war and
peace simultaneously, and that is why both Russia and United States wants peace in the world, but
at the same time they are ready to go at war with other nations. Therefore, option (B) is the correct
answer.
89. Answer: (d)
Explanation: Rationale- Ankit argues that nations are spending vast amounts of money on war
research and arms while at the same time they are negotiating trade agreements and nuclear arms
treaties that are designed to secure peace, therefore he refutes Ronak‟s conclusion by showing that
preparing for war and preparing for peace are not mutually exclusive. Therefore, the correct answer
is option (D).
90. Answer: (a)
Explanation: Rationale- In option (A) we sees the proper reflection of the author‟s idea. Option (B)
forms the premise of the above passage. Option (C) and (D) is not the main idea of the author.
91. Answer: (c)
Explanation: Rationale- The author‟s main idea is that consumption of alcohol in train is as
dangerous as smoking in train. Thus the above given statement is just a premise to his main idea.
Thus option (C) is the answer.
92. Answer: (a)
Explanation: Rationale- Option (B) and (C) are not affecting the main idea of the author in the above
passage. Whereas, option (A) tells the reader that intoxication may not cause any harm to the public.
Therefore, option (A) is the most suitable answer.
93. Answer: (d)
Explanation: Rationale- Option (A), (B) and (C) is not the correct answer because the author didn‟t
talked about the effect of alcohol on health, neither about the own actions of the people, nor about
emotional effect of alcohol. Option (D) is the correct answer because author relies on the
comparison between effects of smoking and effects of alcohol while drawing his arguments.
94. Answer: (a).
Explanation: Rationale- In the above passage it is provided that “When intoxicated riders get off the
train, get in their cars, and drive, the public is exposed to at least as much danger as are
nonsmoking rail passengers who are forced to inhale cigarette smoke”, which means that drink in
public is a social problem and it may cause violence. On the basis of this statement the above
statement can be probably true. Therefore, option (A) is the correct answer.
95. Answer: (d)
Explanation: It is not a strengthening statement as it does nothing to explain or prove the author‟s
point any further than what is already mentioned in the passage.
96. Answer: (c)
Explanation: This is not definitely true as the author in the later part of the passage explains how this
makes economic sense to keep on stockpiling steel, even if there is a very low demand.
97. Answer: (d)
Explanation: Both, options A and B explain why India is exporting lower grade iron. Hence, C
98. Answer: (c).
Explanation: Both option 1 and 2 provide reasonable explanation as to why China is importing iron
ore from India, and producing large quantities of steel.
99. Answer: (d).
Explanation: Strictly sticking to the passage, we cannot say what will happen once the effects of
pandemic start waning as the passage does not mention about this at all
100. Answer: (d)
Explanation: The main argument is that the storage of energy is a big obstacle to the widespread
adoption of alternative sources of power, and magnesium can be used to extract energy in a
controlled fashion.
The first boldfaced portion states the problem involved in trying to use magnesium and the second
portion gives a possible solution. This solution becomes a solution to the whole problem of the
argument as well. Thus, choice D is correct.
Choices A, B and C are incorrect as the argument as a whole poses that storing energy is one of the
biggest obstacle and not the reactive nature of magnesium.
Thus, choice D is the correct answer.
101. Answer: (c)
Explanation: Option A is strengthening the argument.
Option B is unrelated to the purchase of Spyder.
Option D is not specifically about Spyder.
Only option C suggests that Spyder should not be purchased as it cannot be run on the present
computer system.
Hence C.
102. Answer: (a)
Explanation: The length of the description clearly indicates the author aims to abbreviate events that
unfolded over decades into a few paragraphs. But the description of what led to the war is unlikely to
continue, because the last second paragraph ends at the time immediately before the war. So option
D cannot be the answer.
Hence, option A is correct.
103. Answer: (a)
Explanation: While King James believed the king's authority and dignity is superior to all, the
reluctance to giving equal rights and principles has been cited to KING CHARLES in the passage.
Hence option A is correct.
Options B, C and D find mention in the first paragraph.
104. Answer: (b)
Explanation: Refer to the following lines: "So far the errors had been entirely upon the side of the
king". The author clearly blames King Charles for the deterioration in the relationship between the
Parliament and monarchy. Hence Option B is correct.
105. Answer: (c)
Explanation: According to the passage, King Charles CONTINUED with the SHIFTY (i.e.looking or
seeming dishonest) foreign policy, not a SHIFT (i.e. change) in the foreign policy.
The reasons stated in options A, B and D find mention in the passage. Hence, option C is correct.
106. Answer: (b)
Explanation: There is no evidence that King James was the king of Scotland, hence option 1 is
incorrect. The author mentions in the beginning "he brought with him from Scotland..." implying he
carried forth the ideologies of his native place. Hence, B is correct.
107. Answer: (c)
Explanation: Refer to: "thus there were on both sides..." Hence, option C is correct. The same
sentence implies that some earnest and intelligent men chose to leave the Parliament's side and
fight beside the king, so option 4 is incorrect. Option A is an incorrect generalization, while option B
is not supported by the passage.
108. Answer: (b)
From the given information, we get the following table.
T works in HR.
109. Answer: (d)
From the given information, we get the following table.

None of the given combinations is correct.


110. Answer: (a)
From the given information, we get the following table.

Three persons J, T and V work in organization Y,


111. Answer (d)
Explanation:
P1 = Pens, P2 = Pencils, G = Glasses
S = Sharpener, B = Book
112. Answer (d)
Explanation:

B = Bottle, F = Fluid, T = Train


T1 = Track, C = Compartment

LEGAL REASONING
113. Answer: (d)
Explanation: Motorola would not succeed because computer was actually used as an asset of the
company and not merely for personal use. It contributed to the customer service part of the business
and was also bought out of the business income of Motorola Pvt/ Ltd.
114. Answer: (a)
Explanation: The ads of Nidea Cellular focussed mainly on the different things that people can learn
by using NIN service. Therefore, it misleads public into believing that the service is like an institution
teaching various things and not merely an internet service provider.
115. Answer: (d)
Explanation: the terms of contract cannot be called unfair because they clearly give the option to opt
out of automatic renewal without any ambiguity. It does not matter whether it was already ticked in or
one has to tick it, because consumers have been clearly given the option to remove the tick instead
of making it hidden condition.
116. Answer: (c)
Explanation: Product is defective as it is not supposed to throw a wood piece in its 3 rd use. Moreover,
switching the switch off wasn‟t a requirement to use this gadget and merely a safe practice. He has
complied by the instructions. Therefore, no misuse on his part.
117. Answer: (c)
Explanation: Parimateria means dealing with same subject matter and in legal context, it means that
two laws be analysed in reference to each other.
118. Answer: (b)
Explanation: the man is not a consumer because he bought the jeep to use it for commercial
purposes.
119. Answer: (b)
Explanation: A clause referring parties to arbitration is treated as a separate agreement. Therefore,
any dispute pertaining to the validity of the contract itself, does not render the arbitration agreement
inoperative.
120. Answer: (a)
Explanation: The law that is applicable to the arbitration proceeding is the law of the juridical seat of
the arbitration. In this case, the seat was decided to be Paris and therefore that would be the
applicable law on arbitration agreement.
121. Answer: (d)
Explanation: According to BALCO judgement, there is a difference in Part I is applicable only when
the seat of arbitration is India. Here, the seat is ICC, Paris and therefore Rinz & Minz cannot
approach Indian courts to grant injunction.
122. Answer: (c)
Explanation: If “place” under Section 2(2) would be construed as “seat” then, Part I of 1996 would
only be applicable if seat is India, which is contrary to the judgement of Bhatia.
123. Answer: (d)
Explanation: As per Enercon case, the legal system where arbitration is conducted would help in
determining the law applicable to arbitration agreement.
124. Answer: (a)
Explanation: The Enercon case clarified that the legal system in which the arbitration is taking place
or the system which has the closest and most intimate connection with the arbitration proceedings is
relevant i.e the seat of the arbitration. Therefore, seat decides which courts would have jurisdiction.
125. Answer: (a)
Explanation: [When two or more persons make a joint promise, the promise may, in the absence of
express agreement to the contrary, compel one or more of such joint promisors to perform the whole
of the promise. Hence in this case C can be compelled to make the payment of even A and B.]
126. Answer: (b)
Explanation: [A joint promisor who has been compelled to perform the whole of the promise, may
require the other joint promisors to make an equal contribution to the performance of the promise.
Hence in the given fact situation A and B are liable to perform their part of promise.]
127. Answer: (c)
Explanation: [When a promise is performed on a certain day, and the promisor has undertaken to
perform it without application by the promise, the promisor may perform it at any time during the
usual hours of the business on such day and at the place at which the promise ought to be
performed. Since X did not deliver the goods during the usual hour hence he is liable.]
128. Answer: (c)
Explanation: [When a promise is made to more than one person jointly, the right to claim
performance rests with all of them jointly. If anyone of them dies, it rests with his legal
representatives jointly, with the survivors and after the death of the last survivor, with the
representatives of all jointly. Applying this principal B‟s representative have right to claim B‟ share of
money.]
129. Answer: (c)
Explanation: (The platform option c talks about is print media. IT Act is applicable only on online
activities.)
130. Answer: (b)
Explanation: (The Court did not strike down sections 69A and 79 of the Act, which deals with the
procedure and safeguards for blocking certain websites, but only section 66A.)
131. Answer: (a)
Explanation: (Under the original IT Act, Section 66A was observed to violate the freedom to speech
and expression guaranteed by the constitution of India.)
132. Answer: (b)
Explanation: (The charge for violating section 66A was dropped after the amendment, but the other
sections of IPC were still under effect.)
133. Answer: (b)
Explanation: (Because Ramesh‟s actions also constitute to hacking, which is still an offence by the
law.)
134. Answer: (d)
Explanation: (The acts of social media trolls and bloggers are deemed as their freedom to speech
and expression. After the strike down of section 66A, no one can demand action against it legally.)
135. Answer: (c)
Explanation: (Under the new law, buying or selling persons brings imprisonment of 7-10 years, and a
fine of at least Rs 1,00,000.)
136. Answer: (b)
Explanation: (According to the new law, the omission of Duty by an anti-trafficking authority for the
First-time attracts fine of minimum Rs 50,000)
137. Answer: (d)
Explanation: (according to the new law, if any public servant is found involved in an act of trafficking,
he/she shall receive life imprisonment with a fine.)
138. Answer: (a)
Explanation: (If a person is found guilty under the Bill and also under any other law, the higher
punishment will apply.)
139. Answer: (d)
Explanation: (According to the law, the owner/occupier of premises of trafficking may get up to 3
years imprisonment with fine of up to Rs 1,00,000 for the first-time conviction; Subsequent
conviction: at least 5 years with fine of up to Rs 2,00,000.)
140. Answer: (a)
Explanation: (Trafficking with the aid of the media brings imprisonment of 7-10 years, and fine of at
least Rs 1,00,000.)
141. Answer: (b)
Explanation: The correct answer is (b). If accused has created a reasonable doubt before the court
the liability of negation is on the prosecution. If only the alternate version of facts produced by
the accused is properly negated by the prosecution the accused can be held guilty.
142. Answer: (c)
Explanation: The correct answer is (c). As per Section 313(1) (b) of the Code of Criminal Procedure,
1973, after the prosecution closes its evidence and examines all its witnesses, the accused is given
an opportunity of explanation and hearing. So even if there were proofs against Vivek, he should be
given an opportunity to present his side of the case in the court.
143. Answer: (a)
Explanation: The correct answer is (a). The submission of Manu was a mere statement and was not
backed by enough evidence. Only a reasonable doubt created by the accused is needed to be
negated by the prosecution.
144. Answer: (a)
Explanation: The correct answer is (a). The prosecution needs to prove its case, and the accused
merely needs to create reasonable doubt or prove their alternate version by mere preponderance of
probabilities. So Sneha has to prove beyond reasonable doubt that her uncle raped her and so he is
guilty.
145. Answer: (b)
Explanation: The correct answer is (b). A trial court is required by law to apply its mind to the
defence presented by the accused magistrate‟s failure in doing so renders the entire exercise void.
146. Answer: (a)
Explanation: The correct answer is (a). As per Code of Criminal Procedure, the Court has to record
his satisfaction that the conditions laid down in the law for issuing warrant of arrest has been fulfilled
and the procedure has been complied with. In the present case, since the reasons are not recorded
the warrant cannot be issued and thus the arrest is unlawful.
147. Answer: (b)
Explanation: The correct answer is (b). In terms of Section 73 of CrPC, the court opined that a
Magistrate has jurisdiction and power to issue warrant of arrest. So, in the present case, there was
no need for a warrant being issued by the Sessions judge.
148. Answer: (c)
Explanation: The correct answer is (c). As per section 82 of CrPC, a person can be declared a
person as a proclaimed offender only after the Court is satisfied that the person is absconding, or is
concealing, and it is not possible to arrest him/ her. Megha Hatka gives regular attendance in the
police station and takes approval before leaving the town, so she does not satisfy any criteria for
being declared a proclaimed offender and thus her arrest was unlawful.
149. Answer: (d)
Explanation: The correct answer is (d). The Court has to record his satisfaction that the conditions
laid down in the law for issuing warrant of arrest has been fulfilled, as only being an accused of a
non-bailable offence is not a ground to issue warrant of arrest, as per the provisions of Section 73 of
the Code of Criminal Procedure.
150. Answer: (d)
Explanation: The correct answer is (d). As per section 73 of the Code of Criminal Procedure for an
arrest warrant to be issued two criteria shall be satisfied i.e. a non-bailable offence and the accused
is trying to evade arrest, both of which shall be met. In the present case, Rick and Morty have
committed a bailable offence in Biggum and thus cannot be arrested even if they are trying to evade
the arrest.
Legal Edge 10 Mock

Answers & Details

ENGLISH

1. Ans:(b)
Explanation: The author is talking about the opportunity which was lost as the Congress failed to
launch a big national movement for the commutation of the death sentence of Bhagat Singh and his
Comrades. If they could have seized the moment, history could have been different. Therefore, the
correct answer is B.
2. Ans:(a)
Explanation: Jatindra Nath Das died in the prison following the 63-days fasting and not because of
the beating he received. Therefore, A cannot be inferred from the passage.
3. Ans:(c)
Explanation: The author has argued that Bhagat Singh‘s young life is itself prove of his
achievement and the potential that he held. And in due course of time he would have been a stalwart
of the Global Socialist Movement. Therefore, C is the correct answer.
4. Ans:(b)
Explanation: The word Vociferous means expressing or characterized by vehement opinions; loud
and forceful. Therefore, B is the correct answer.
5. Ans:(c)
Explanation: Verbs are the action words in a sentence that describe what the subject is doing. In
the passage, the word is used in the context of action which the Congress should have taken.
Hence, C is the correct answer.
6. Ans:(b)
Explanation: Serendipity is when someone accidently finds something good. Therefore, B is the
correct answer.
7. Ans:(c)
Explanation: The authors means that the mother has no choice, if she will care for the child, people
will say that she is spoiling the child, and if she would not care much, people will say that she is a
bad mom. Therefore, the game is rigged and the correct answer is C.
8. Ans:(b)
Explanation: Having a feeling of melancholy is feeling sad and pensive. Therefore, B is the correct
answer.
9. Ans:(c)
Explanation: The central theme of the passage is Motherhood, the choices and the consequence of
the decisions a mother has to make. Therefore, C is the correct answer.
10. Ans:(c)
Explanation: The Constitutional Changes in Myanmar are not possible without the military‘s consent
and therefore the author is arguing that even if there is a large electoral transformation, it will not
lead to major constitutional change. Therefore, the correct answer is C.
11. Ans:(b)
Explanation: The international standing of Aung San Suu Kyi has taken hit because critics accuse
her of allowing widespread abuse of minority Rohingyas. Therefore, B is the correct answer.
12. Ans:(d)
Explanation: The Domination of the military might is not one of the issues mentioned in the passage
and therefore wrong. The correct option is D.
13. Ans:(c)
Explanation: Only I and II are correct, III is not mentioned in the passage. Therefore, the correct
answer is C.
14. Ans:(c)
Explanation: The proper sentence with the uses of the articles should be ―The constitution allows
the military to occupy 25% of parliamentary seats. Only serving military officers can lead the three
most powerful ministries – defence, home affairs and border affairs.‖ Therefore, the correct answer
is C.
15. Ans:(c)
Explanation: This is because the entire passage is trying to convey the idea as to how the
advertisement industry in today‘s world breaking all the myths and shaping the aspirations of the
people around. Other options do not convey the entire theme of the passage.
16. Ans:(d)
Explanation: The meaning of the term subliminal is below the threshold of sensation or
consciousness; perceived by or affecting someone's mind without their being aware of it.
17. Ans:(a)

18. Ans:(b)
Explanation: Option (a), (c) & (d) do not capture the entire theme of the passage.
19. Ans:(c)
Explanation: The correct usage of the article would be ―That‘s why Roland Barthes called
advertising the supreme mythmaker of our age and ads the closest modern approximation to ancient
myths.‖
20. Ans:(d)
Explanation: Option (a) & (b) just describe the instances used by the author to substantiate his
argument. Option (c) does not capture the main idea. Option (d) is the most appropriate answer.
21. Ans:(d)
Explanation: Option (d) is the most appropriate answer because it clearly states what new normal is
after the pandemic.
22. Ans:(c)
Explanation: The definition of the term amulet is an object worn because it is believed to protect
against evil, disease, or unhappiness.
23. Ans:(b)
Explanation: The meaning of the word ‗yore‘ is of long ago/former times. The author is trying to say
that despite all the awareness, people still believe that miracles can the pandemic.
24. Ans:(a)
Explanation: Here extrapolating is used to denote ―extending the application of DABTA on the
current Covid-19 pandemic‖ and hence it is a verb.
25. Ans:(d)
Explanation: The literal meaning of incendiary is something that causes fire/ tensing to stir up
conflict. Here, provocative is the most appropriate meaning.
26. Ans:(d)
Explanation: The author tries to convey that the perpetrator of violence create havoc amongst the
common masses if the media does not report the issues in a responsible fashion.
27. Ans:(c)
Explanation: The author describes honour killing as a mark of existence of patriarchy in the society.
It is a crime against the existence of one gender & hence it is an attack on everyone in the society.
28. Ans:(a)
Explanation: Here galloped is used to denote ―moving rapidly‖ and hence it is a verb.
29. Ans:(c)
Explanation: Option (c) captures the central theme of the passage in the most appropriate manner.

GENERAL KNOWLEDGE/CURRENT AFFAIRS

30. Ans:(b)
The World Bank has approved a $500 million Strengthening Teaching-Learning and Results for
States Program (STARS) to improve the quality and governance of school education in six Indian
states.
31. Ans:(c)
The National Education Policy 2020, which was approved by the Union Cabinet of India on 29 July
2020, outlines the vision of India's new education system. The new policy replaces the previous
National Policy on Education, 1986. STARS objectives are in line with this.
32. Ans:(b)
The states to be benefitted from this new project include Himachal Pradesh,
Rajasthan, Maharashtra, Madhya Pradesh, Kerala, and Odisha. The total cost of the project
supported by the World Bank is Rs 5718 crore. Thus Uttar Pradesh is not included in this
programme.
33. Ans:(a)
The project would be implemented as a new centrally sponsored scheme under the Department of
School Education & Literacy, Ministry of Education.
34. Ans:(d)
The role of assessments in education is finally getting the attention it deserves, as evidenced by the
National Education Policy (the NEP 2020). Its focus on assessments and board examination reform
is a step in the right direction—as they play a major role in ensuring students improve academically.
The proposal to set up a national assessment centre, the PARAKH (Performance Assessment,
Review, and Analysis of Knowledge for Holistic Development), as a standard-setting body under the
Ministry of Education indicates the government is keen to provide a regular check on the education
system.
35. Ans:(c)
The Rudram missile is launched from a Sukhoi-30 MKI. India's first indigenous anti-radiation missile,
Rudram, developed for the Indian Air Force, was successfully flight-tested from a Sukhoi-30 MKI jet
off the east coast. An anti-radiation missile can take out designated targets that emit radio
frequency, including radars and jammers.
36. Ans:(a)

37. Ans:(d)
The flight altitude range means the range of altitude in which the aircraft can operate. The same
for Rudram-1 is 500 m to 15 km.
38. Ans:(d)
The Rudram-1 has been jointly developed by Bharat Dynamics Limited (BDL) and Bharat Electronics
Limited (BEL).
39. Ans:(c)
Abhijit Banerjee is the latest Indian to win the Nobel Prize. In fact, the first Nobel Prize ever won by
an Indian was in 1913, 12 years after the prize made its debut. As many as 10 Indians have won the
Nobel Prize so far.
40. Ans:(d)
The Nobel Peace Prize is one of the five Nobel Prizes established by the will of Swedish industrialist,
inventor, and armaments manufacturer Alfred Nobel, along with the prizes in Chemistry, Physics,
Physiology or Medicine, and Literature. It was awarded in Oslo.
41. Ans:(a)
The Nobel Peace Prize 2020 was awarded to World Food Programme (WFP) "for its efforts to
combat hunger, for its contribution to bettering conditions for peace in conflict-affected areas and for
acting as a driving force in efforts to prevent the use of hunger as a weapon of war and conflict."
42. Ans:(d)
The 2020 Nobel Prize in Physiology or Medicine is awarded jointly to Harvey J. Alter, Michael
Houghton and Charles M. Rice ―for the discovery of Hepatitis C virus‖.
43. Ans:(c)
In October 2014, Malala, along with Indian children's rights activist Kailash Satyarthi, was named a
Nobel Peace Prize winner. At age 17, she became the youngest person to receive this prize.
Accepting the award, Malala reaffirmed that ―This award is not just for me.
44. Ans:(b)
By the terms of Alfred Nobel's will the Nobel Prize in Literature has been awarded by the
Swedish Academy since 1901. The Swedish Academy was founded in 1786 by Swedish King
Gustav III. The Academy is composed of 18 members whose tenure is for life.
45. Ans:(a)
The Council of Scientific and Industrial Research (CSIR) and the Central Mechanical Engineering
Research Institute (CMERI) have developed the world‘s largest solar tree in India.
46. Ans:(b)
The solar tree has been installed at the CSIR-CMERI Residential Colony, Durgapur. Each solar tree
will cost Rs.750,000 (~$9,595). In all, there are 35 solar photovoltaic (PV) modules in each tree with
a capacity of 330 W each. The tree is designed to ensure maximum sunlight exposure for each PV
module while creating the least amount of shadow beneath.
47. Ans:(c)
The installed capacity of the solar tree is above 11.5 kW with an annual capacity to generate 12,000-
14,000 kWh of clean power. The energy generated can be monitored either in real-time or daily.
48. Ans:(b)
The Government of India has set a target of installing of installing 175 GW of renewable energy
capacity by the year 2022, which includes 100 GW from solar, 60 GW from wind, 10 GW from bio-
power and 5 GW from small hydro-power.
49. Ans:(a)
The second largest tree has been set up in London and produces around 8.6kw.
50. Ans:(d)
The six STPs were inaugurated at Haridwar, Rishikesh, Muni-ki-Reti and Badrinath via video
conference.
51. Ans:(a)
PM Modi inaugurated the project of 'Ganga Avlokan' among six other projects under the 'Namami
Gange' programme in Uttarakhand. Ganga Avlokan will be a museum based on the significance of
river Ganga and will be located at Chandi Ghat.
52. Ans:(c)
The Prime Minister inaugurated nation‘s first museum on Ganga on Sept 29. It has been dedicated
to put light on the biodiversity, rejuvenation and culture in Ganga river. The museum is located at
Chandi Ghat, Haridwar.
53. Ans:(b)
The name of the book is National Mission for Clean Ganga and Wildlife Institute of India.
54. Ans:(d)
Jal Jeevan Mission, is envisioned to provide safe and adequate drinking water through individual
household tap connections by 2024 to all households in rural India.
55. Ans:(a)
The JSC Sukhoi Company was a major Soviet and now a Russian aircraft manufacturer,
headquartered in Begovoy District, Northern Administrative Okrug, Moscow, that designs both
civilian and military aircraft. It was founded by Pavel Sukhoi in 1939 as the Sukhoi Design Bureau.
Sukhoi fighter planes were acquired first by India from Russia.
56. Ans:(b)
The Dassault Rafale is a French twin-engine, canard delta wing, multirole fighter aircraft designed
and built by Dassault Aviation.
57. Ans:(d)
The NDA government had inked a Rs 59,000-crore deal on September 23, 2016 to procure 36
Rafale jets from French aerospace major Dassault Aviation after a nearly seven-year exercise to
procure 126 Medium Multi-Role Combat Aircraft (MMRCA) for the Indian Air Force did not fructify
during the UPA regime.
58. Ans:(c)
The Rafale jets also come with SCALP, the air-to-ground cruise missile with a range over 300 km. It
is a long-range deep strike missile.
59. Ans:(c)
The Congo Fever was first discovered in Crimea in 1944. It was then seen again in Congo in 1956.
In 1969, the disease was names Crimean Congo Hemorrhagic Fever (CCHF) or Congo fever.
60. Ans:(a)
The virus had first originated in India in the Valasad district of Gujarat.
61. Ans:(d)
The viral fever is transmitted through Hyalomma ticks.
62. Ans:(a)
The Nairovirus of the Bunyaviridae family causes Congo Fever.
63. Ans:(d)
The 2020 United States presidential election was the 59th quadrennial presidential election, held on
Tuesday, November 3, 2020.
64. Ans:(b)
It happens every Tuesday of the week.
65. Ans:(a)
The President of USA has be reelected once, i.e. a person can hold the post for a maximum of two
tenures.
66. Ans:(c)
Kamala Devi Harris is an American politician and attorney who has served as the junior United
States senator from California since 2017. She is the Democratic vice presidential nominee for the
2020 election.
67. Ans:(b)
There are currently 538 electors, and an absolute majority of electoral votes, 270 or more, is
required to win the election.
68. Ans:(d)
The president must: Be a natural-born citizen of the United States. Be at least 35 years old. Have
been a resident of the United States for 14 years.

QUANTITATIVE TECHNIQUES
69. Ans:(c)
Required answer
= 5800×35/100-3600×42/100
= 2030 – 1512 = 518
70. Ans:(d)
Required percentage =(15 × 36)/(16x 58)×100=58%
71. Ans:(a)
Required average =(24 × 58 - 28 × 36) + (11 × 58 - 14 × 36) + (18 × 58 - 21 × 36)/3
=(1392 – 1008 + 638 – 504 + 1044 – 756)/3
=268.67=269
72. Ans:(d)
Required percentage
=(18 × 58 -21 × 36) + (31 × 58 – 22 × 36)/(31 × 58+ 18 × 58)×100
=(1044-756)+(1798-792)/(1798+1044)×100
=(288+1006)/(1798+1044)×100=1294/2833×100=46%
73. Ans:(c)
Required ratio
= 18 x 58 : 21 x 36
= 58 : 42 = 29 : 21
74. Ans:(d)
Average of working hours of Shashi on Day 1 and Day 6 together = (8 + 4)/2 = 6 hours
Average of working hours of Neha on Day 5 and Day 6 together = (5 + 7)/2 = 6 hours
Required difference = 6 – 6 = 0
75. Ans:(b)
Shashi takes =7/2 hours
Neha takes = Shashi = 7/2 hours
And Ashok takes = 150/100 * 6 = 9 hours
So, ratio of their efficiency = 2/7: 2/7: 1/9 = 18: 18: 7
76. Ans:(a)
Time taken by them to complete thrice of the work :
= 3 * 1/(1/5 + 1/5 + ¼)
= 4 813 hours
77. Ans:(b)
On Day 4:
One-third of the work done by Shashi = 8/3 hours
One-third of the work done by Neha = 6/3 = 2 hours
And rest one-third of the work done by Ashok = 8/3 hours
So, total time = 8/3 + 2 + 8/3 = 223 = 7 1/3 hours
78. Ans:(c)
The total working hour of Shashi and Ashok together throughout the six days = 8 + 6 + 7 + 6 + 6 + 5
+ 8 + 8 + 5 + 4 + 4 + 7 = 74 hours
And total working hour of Neha throughout the six days = 7 + 4 + 4 + 6 + 5 + 7 = 33 hours
So, required ratio = 74: 33
79. Ans:(b)

80. Ans:(a)
Following common explanation, we get
The share of son A in total property = 4.6 lakhs
The share of son B in total property = 2.6 lakhs
The required difference = 4.6 – 2.6 = 2 lakhs

81. Ans:(d)
Following common explanation, we get
In 10 years 10 × 12 = 120 months
The person A had received highest share
He will pay to his father in 120 months = 120 × 2500 = 3 lakhs
The share of property he was left with = 4.6 – 3 = 1.6 lakhs
82. Ans:(b)
Following common explanation, we get
The share of son A in land = 3 lakhs
The share of daughter C in land = 1 lakhs
the reqd. % =(3-1) × 100/1=200%
83. Ans:(d)
Following common explanation, we get
The share of son A in total property = 4.6 lakhs
The share of son B in total property = 2.6 lakhs
The required ratio = 4.6 : 2.6 = 23 : 13
84. Ans:(d)
Writer in B is 48.which is 20% more than writer in A.
So writer in A is 48*100/120 = 40.
Let, painter in A is 7d and painter in b is 5d.
Dancer in a =5d-14 dancer in B =7d-28
According to question,
5d-14+7d-28=78,
Or, 12d=120, d=10
In A painter is 70, dancer is 36.
In B painter is 50, dancer is 42.
Now singer in A is 17c and singer in B is 13c
17c-13c = 51 ,c = 3
Singer in A is 51 and singer in B is 39.

CITTY WRITER PAINTER DANCER SINGER


A 40 70 36 51
B 48 50 42 39
Total people at A is 40+70+36+51 = 197
Total people at B is 48+50+42+39 =179
So respective ratio is 197:179.
85. Ans:(b)
Total painter is 70+50 =120
Total singer is 51+39 =90
Painter is 120-90 = 30 more than singer.
So respective percentage is = (30/90)*100= 33.33%

LOGICAL REASONING

86. Ans:(b)
Explanation: The reason the author suggests boundaries is because it will be tough on the child
and confuse the child with two different parenting styles. Option (b) strengthens this.
Hence (b).
87. Ans:(c)
Explanation: Option (a) talks about only mothers and mental health challenges, both of which are
not the crux of the passage.
Option (b) again speaks about parents and their children with respect to fatigue and the quality of the
relationship. This is not the focus of the passage.
Option (c) is apt with communication of decisions (read: boundary setting).
Hence (c).
88. Ans:(c)
Explanation: The passage talks about parenting being a learning process: ‗you, too, need your time
and space to make mistakes and understand the intricacies of parenthood‘.
Option (a) with ‗myth‘, option (b) with ‗exhaustion‘ and option (d) with ‗expensive‘ are out of context.
Hence (c).
89. Ans:(a)
Explanation: The passage says ‗You tend to feel awkward when you must assert such power over
the people who raised you.‘ The best option is option (a).
Hence (a).
90. Ans:(c)
Explanation: The passage in no way says that our parents are experts or on the contrary that we
should never go to them. It also does not talk about going to other parents for advice. The passage
says that we can rely on our parents for help and advice, but also be aware that our own journeys in
parenting may differ from theirs, so we must set boundaries. Hence (c).
91. Ans:(b)
Explanation: Option A is incorrect because of ‗Unsurity‘, similarly Option B is wrong because author
has not shown any dissatisfaction. Between option B and D, B is a much better choice because
author has not expressed hope (optimism), rather he has praised the groups to some extent.
Hence, B
92. Ans:(c)
Explanation: The author begins by talking about the structure of the group, then how the person
feels initially and then his final outcome i.e. he has explained about the group chronologically.
Hence, C
93. Ans:(b)
Explanation: The author has ended the paragraph with the benefits of the Encounter Groups, hence
most probably he will provide further evidence for the same by narrating personal experiences of
such beneficiaries.
Hence, B
94. Ans:(a)
Explanation: Within a week, the woman started loving her group-mates, this shows how quickly (or
rapidly) participants became comfortable.
Hence, A
95. Ans:(d)
Explanation: (D) can be easily inferred from ―So give up waiting as a state. When you catch
yourself slipping into waiting … snap out of it. Come into the present moment‖.
(A) is a passing observation.
(B) is not mentioned in the passage.
(C) is a recommendation.
Hence, (D)
96. Ans:(b)
Explanation: (B) - The author argues in the passage that the future is ―imaginary‖ and not ―real‖ in
the first paragraph.
(A) is hyperbole.
(C) is beside the point.
(D) is somewhat implied in health- and life-corroding insanity, but (B) is the concise answer.
Hence, (B)
97. Ans:(b)
Explanation: (B) can be safely inferred from ―you will continue to experience the inner condition of
lack, and deep down you will continue to feel unfulfilled‖. (A) is a technicality. (C) is a generalization.
(D) does not come to the point.
Hence, (B)
98. Ans:(c)
Explanation: (C) can be inferred from ―Come into the present moment. Just be, and enjoy being‖,
which will not make us wait for ―further physical or psychological gratification‖. (A) is not
substantiated. (B) is the opposite of the author‘s contention. (D) may or may not be true.
Hence, (C)
99. Ans:(d)
Explanation: According to the given argument, ‗X, if and only if Y; X being house and Y being
money and labour‘. Option
(A) does not follow the logic as the presence of the word ‗can‘ means existence of different
possibilities.
Option (B) also does not follow the logic because the second and the third statements do not follow
the same sequence as given in the question. Option (C) is a close answer but is wrong because the
conclusion states that he has no work experience. While according to the given statement, it should
have been good working experience. Option (D) is the correct answer as it follows exactly the same
logic as implied in the question. ‗An organization (X) will exist, if and only if excellent staff (Y) is
there.‘
Hence, D
100. Ans:(b)
Explanation: Option B is the correct answer. Option A is incorrect as the passage clearly states that
even though budget allocation has substantially increased, it had been of little use. Option C is
incorrect because sitting and wondering are not going to benefit in any way. Option D is incorrect
because equal distribution of funds is a total waste. Not all sectors of economy need the same
amount of money. There are some sectors that need more money, while some need less. Option B
seems to be the most plausible option as this alternative will increase efficiency to a great extent
101. Ans:(d)
Explanation: Option D is the correct answer. Option A is incorrect because it is far-fetched. Option
B is incorrect because the passage is silent on this. Option C is incorrect because the passage does
not contain any ratio or number of undernourished population. Option D follows as it has been
clearly stated that ―the nation is unable to provide access to food to a large number of people‖.
Hence, D
102. Ans:(c)
Explanation: C is the most appropriate option as it questions the sample survey method. Option A
is ambiguous and doesn‘t necessarily invalidate the given argument. Option B questions the
conclusion to some extent. D is beyond the scope of the argument.
Hence, C
103. Ans:(a)
Explanation: Refer to: "the PCB has RAISED the legal threshold for residues on some foods up to
300-fold above levels deemed SAFE in the 1990s" - second para, first sentence.
Hence A
104. Ans:(c)
Explanation: the 'agency' in option C would refer to the MP PCB. So, option C is the logical
corollary to the last sentence of the given passage. Other options can be included in other parts of
the passage.
105. Ans:(b)
Explanation: It is said in the passage that Dow's influence over the PCB has succeeded in
suppressing health concerns and also the governemnt has turned a blind eye for decades when it
comes to monitoring Aflatoxin. Hence, option A is only partially correct. Option D and C do not
represent the author's main point.
Hence, B
106. Ans:(d)
Explanation: Options A, B and C are in accordance with the psssage. However, Option D is
contrary to the given info. Refer to: "the government has TURNED A BLIND EYE (turn a blind eye -
to ignore something that you know is wrong) for decades when it comes to monitoring Aflatoxin"
Hence, D
107. Ans:(c)
Explanation: Option A contradicts the information given in the passage.
Option B and D are what they should do, but they don't do.
Option C can be inferred from "the government has turned a blind eye for decades when it comes to
monitoring Aflatoxin – failing to test for it on food and in our bodies." and "Numerous reports have
covered the internal company documents showing how Dow‘s influence over the PCB succeeded in
suppressing health concerns."
Hence, C
108. Ans:(d)
Explanation: Option A - a movie depicting an EERIE PLACE - Place is not eerie; the sound of the
roulette wheel is eerie.
Option B - It is not an ACTUAL PLACE, it is a movie
Option c - SHOT IN AN EERIE STYLE - the movie shooting is not eerie, but the the sound of the
roulette wheel is eerie.
Option D - Its a film (Refer to "THIS FILM is the latest in a series by the Wilsons" - Second para, last
sentence) that has two themes and two perspectives (2nd para).
Hence, D
109. Ans:(c)
Explanation: The main reason behind capturing the attention is that the imagery is personal and
relatable to everybody. Refer to - ". It's personal. We've all been there."
Hence, C
110. Ans:(b)
Explanation: Option A and option C cannot be inferred from the passage.
Option D is the style of Emin's contemporaries, not of Emin.
Option B can be inferred from "This art will move you whether you like it or not"
Hence, B
111. Ans:(d)
Explanation: Option A - Big Brother is not the viewer
Option B - WHILE - not mentioned that both actions are done simultaneously. People are
WATCHING BOTH.
Option C - Big Brother is not the subject.
Option D - Correct. "The machine becomes the watcher" and use of washing machines is an allusion
to the cleansing.
Hence, D.
112. Ans:(c)
Explanation: Option A - it is not an APPRAISAL of PREVALENT art forms.
Option B - It is not a CRITICAL analysis.
Option C - correct. The lack of coherence and randomly shifting the attention to various artists
makes the passage unclear. Probably the writer was attempting an objective view on them.
Hence, C
113. Ans:(c)
Explanation: The colour of the human blood is ‗red‘, and as it is given that ‗red‘ is called ‗yellow‘.
So, the colour of human blood is ‗yellow‘.
114. Ans:(b)
Explanation: From I.
Nothing is said about V. Hence 1 is not sufficient to answer the question.

Hence X sits on the immediate left of V. Hence statement II alone is sufficient to answer the
question.

LEGAL REASONING

115. Ans:(b)
(Self-explanatory)
116. Ans:(c)
(Explanation: Had there been no mention of ‗unlimited‘ the bill might still hold water as it would only
aid in the investigation of crime. But, it‘s unrestricted therefore crosses the line of privacy)
117. Ans:(b)
(Explanation: This amounts to invasion of the dignity of the individual once he is discharged from
the prison, he has already completed his term and now subjection to this is only an invasion of his
privacy)
118. Ans:(b)
(self-explanatory)
119. Ans:(a)
(Explanation: such information does not form the part of a private information)
120. Ans:(b)
Explanation: The passage clearly states that there must be a legally enforceable debt against
someone who issues a cheque. Further, the burden of proof lies on the person who issues such
dishonoured cheque. However, the first burden is proving that the cheque was issued for discharge
of liability. Thus, B is correct.
121. Ans:(c)
Explanation: The passage clearly states that a cheque must be for a legally enforceable debt.
However, it if is shown that the cheque was not issued for the legally enforceable debt but for some
other debt, then the same is a valid defence under the law. Thus, A must show that he had issued
the cheque for some other debt.
122. Ans:(a)
Explanation: The passage states that there must be a legally enforceable debt. The entire contract
amount seeks to be part of the legally enforceable debt. Thus, it cannot be said that the issuance of
the cheque was not for any legally enforceable debt.
123. Ans:(c)
Explanation: The passage states that for a person to be liable under the negotiable instruments act,
there must be a legally enforceable debt. In the event of illegal alcohol being smuggled, there cannot
exist a legally enforceable debt.
124. Ans:(b)
Explanation: The passage states that a cheque may be issued as security. However, proving that
the cheque amount exceeded any amount of liability that was outstanding, it is the best defence for
any person.
125. Ans:(d)
Explanation: The passage states that consideration for a contract may be in the past, present or
future. In this case, the promise to pay was made and the consideration was to be paid in the future.
There is thus a valid contract.
126. Ans:(b)
Explanation: The passage states that a consideration must be something that is over and above a
person‘s existing obligations. In this case, it cannot be said that the consideration from Navedia‘s
side was over and above his natural obligations.
127. Ans:(b)
Explanation: The passage states that a consideration must hold some value in the eyes of the law.
In this case, it cannot be said that the consideration would hold value in the eyes of the law and thus
there is no contract of agency.
128. Ans:(b)
Explanation: The passage states that a consideration must be something that is over and above a
person‘s existing obligations. It cannot be said that giving a person their share of monthly profits is
valid consideration for a contract to be formed.
129. Ans:(a)
Explanation: The passage states that a consideration must be something that is over and above a
person‘s existing obligations. Giving of monthly profits share in advance is a valid consideration and
thus there was a contract of agency.
130. Ans:(b)
Explanation: There is obviously no 'natural love and affection' in asking your wife to stay away from
you, and you're having a troubled marriage. Hence, there is no consideration, and the contract is not
a valid one.
131. Ans:(d)
Explanation: The terms of contract are uncertain because modern interior and furniture could be
different for Palak and Mehak. It is therefore uncertain what is meant by the term ―modern‖.
132. Ans:(b)
Explanation: As per the article, if the parties do not have absolute intention to refer the dispute to
arbitration at the time of entering into arbitration agreement, then is not enforceable.
133. Ans:(a)
Explanation: Clause 10 can be made effective by going with the interpretation where ―full amount‖
has certain meaning of being the expenditure incurred in erecting the system.
134. Ans:(b)
Explanation: in this case, there is clear uncertainty and there is no interpretation of this contract,
which would make it effective.
135. Ans:(a)
Explanation: It gives autonomy to the parties to decide if they want to refer the dispute to arbitration
or not. In case of sole option, the other party has agreed to let one party decide if they want to refer
the dispute to arbitration.
136. Ans:(b)
Explanation: The article lays the principle that criminal liability arises only when the wrongful act
has been committed with wrongful state of mind In this case, Mr. Mukherjee assumed himself to be a
widower and therefore remarried which implies he didn‘t have mens rea for the offence.
137. Ans:(a)
Explanation: Prashant was working as pump attendant when he started assaulting Mihir. Merely
because he ran to the forecourt to beat Mihir or was beating out of personal enmity does not take
him out of his course of employment. Therefore, Morrisson is liable as per the principle of vicarious
liability.
138. Ans:(c)
Explanation: In this case, Sakshi had drafted the representation. She is also the director and the
manager of the company and therefore can be called directing mind of the company. Therefore,
vicarious liability can be imposed on Motorla Inc.
139. Ans:(d)
Explanation: As per the 3 requirements of Respondent superior, Michael has committed a crime
While asking Jack to refrain from touching items, he was under the course of employment but while
losing temper and punching Jack even after Jack was ushered out of the shop, is not under course
of employment.
140. Ans:(c)
Explanation: Identification theory identifies the directing minds of the corporation and thereby hold
the corporation liable for the criminal acts committed by such directing minds because of absence of
a corporation‘s own mind.
141. Ans:(b)
Explanation: Rashi‘s mother falls under the definition of ―victim‖ though none of them can appeal
under Section 372 of CrPC.
142. Ans:(a)
Explanation: As per Section 372, a victim can file appeal against conviction for lesser offence and
Ramesh‘s father falls under definition of victim.
143. Ans:(c)
Explanation: Parvinder Kansal case has clarified that a victim cannot file appeal for enhancement of
sentence under Section 372.
144. Ans:(c)
Explanation: The author argues that plethora of cases where a victim is not satisfied with the
sentence and where the sentence is inadequate as compared to the offence, victim has no right
even though the proviso under Section 372 was brought with the aim to provide adequate rights to
the victim.
145. Ans:(c)
Explanation: Radhika is the accused in the case and therefore cannot appeal under S. 372 of
CrPC. Moreover, Rahul can only appeal against the conviction and not the sentence.
146. Ans:(a)
Explanation: As per S. 112, if a woman is married to a man and gives birth to a child, then such
child shall be presumed to be the legitimate child of the husband Only in cases of dissolution of
marriage, one would go into the question of non-access to each other and 280 days.
147. Ans:(c)
Explanation: As per S. 112, one must prove that the parties did not have access to each other ―at
any time‖ when the baby could have been begotten and not particularly on the exact date when the
baby was conceived.
148. Ans:(d)
Explanation: As per the passage, to refute the presumption of legitimacy of child during a valid
marriage, one has to prove that he/she did not have access to each other, which Rahul has
successfully proven.
149. Ans:(b)
Explanation: Nandlal Wasudev case held that scientific advancement proof will prevail over the
conclusive proof under law and therefore the DNA test would prevail over the presumption of
legitimacy.
150. Ans:(a)
Explanation: Nandlal case puts proof based on scientific advancement over the conclusive proof.
Therefore, problems associated with use of scientific advancement will weaken the argument. DNA
test being 99.9% accurate strengthen the argument whereas the fact that DNA tests are unregulated
and not uniform would weaken the argument.
Legal Edge 11 Mock

Answers & Details

ENGLISH

1. Ans:(d)
All the of options are correct as Trump in his pledge made on his inauguration day, laid down all
these reasons in reaching his conclusion that the American foreign policy is in dire straits.
2. Ans:(c)
Trump administration is leaving a very significant aspect of foreign policy reminder to both the
Democrats and Republicans that the Global Supremacy of the US cannot be sustained on the
unstable domestic economy of the US.
3. Ans:(b)
Only I is true as per the given passage. Both I and III are in stark opposition of what the passage has
been pointing towards. Therefore, A is the correct answer.
4. Ans:(c)
On the contrary to the option C, whether Biden has indicated that he would join the Paris Agreement
or not has not been mentioned in the passage.
5. Ans:(c)
Stagnation means lack of activity, growth, or development orthe state of not flowing or moving.
Therefore, C is the correct answer.
6. Ans:(a)
Only I is wrong, as the words accused and victim are used interchangeably. The sentence should be
―it is necessary that the victim is instigated by the accused‘. Therefore, A is the correct answer.
7. Ans:(c)
The SC in Sohan Raj Sharma v. the State of Haryana said that abetment is not only a mental
process of instigating a person but also involves intentionally aiding that person in committing the
act. Therefore, C is the correct answer.
8. Ans:(c)
According to the passage, a suicide note is a very strong and material evidence but it cannot solely
lead to conviction. Therefore, C is the correct answer.
9. Ans:(d)
Mens res is indispensable to the abetment as without it, the crime of abetment cannot be proved.
Therefore, D is the correct answer.
10. Ans:(a)
Material and documentary evidence is usually described as the clinching evidence. Therefore, A is
the correct answer.
11. Ans:(c)
According to the passage, ‗love jihad‘ entails ‗trapping‘ Hindu girls in love with the aim of converting
them for marriage. Therefore, C is the correct answer.
12. Ans:(b)
The claims of hindujagruti.org regarding the Islamic website turned out to be false. Therefore, B is
the correct answer as it cannot be inferred from the passage.
13. Ans:(c)
The proper sentence with the uses of the articles should be ‗The SC observed that matters of belief
and faith, including whether to believe, are at the core of constitutional liberty and the state or
‗patriarchal supremacy‘ could not interfere in her decision.‘
14. Ans:(a)
Proselytize means convert or attempt to convert (someone) from one religion, belief, or opinion to
another. Therefore, A is the correct answer.
15. Ans:(d)
According to the passage, option I is incorrect as right to propagate does not include right to
proselytize while option II is correct as Right to choose life partner is not affected by the matters of
faith.
16. Ans:(b)
The imprinted text or image on the t-shirt means nothing and it does not represent anything.
Therefore, B is not true.
17. Ans:(b)
Since, the t-shirt are unlike the traditional apparel which represent specific class, caste and power
structure, there is nothing to consider t-shirt as relevant text.
18. Ans:(c)
According to the passage, a t-shirt is flexible and ambiguous and not definite and precise. Therefore,
option C is wrong and does not contributes to the casualness of a t-shirt.
19. Ans:(b)
Ambiguous means open to more than one interpretation; not having one obvious meaning, the
opposite of which is definite or precise. Therefore, B is the correct answer.
20. Ans:(a)
Androgynous partly male and partly female in appearance; of indeterminate sex. Therefore, A is the
correct answer.
21. Ans:(b)
The rulers of Nurpur and Guler states were fighter from the Mughal Side in many battels and were in
the close association with the Mughals. Therefore, B is the correct answer.
22. Ans:(d)
It is clear from the passage that Mughal culture sought reflection not only in their attire but also in the
fine arts, lavish princely hobbies, and material luxuries. Therefore, D is the correct answer.
23. Ans:(c)
Imbibing means to absorb or assimilate (ideas or knowledge). Therefore, C is the correct answer.
24. Ans:(d)
All of the given options characterises the Pahari Paintings, therefore, D is the correct answer.
25. Ans:(d)
Option D is not only not in the passage but also is wholly irrelevant to the passage. Therefore, D is
the correct answer.
26. Ans:(c)
The press has a major informational role in disseminating knowledge and allowing critical
scrutiny. The second reason among the four reasons helps the critical scrutiny component of the
public reason.
27. Ans:(a)
Impede means delay or prevent (someone or something) by obstructing them; hinder. While
unrestrained means which does has any hinderance and therefore, A is the correct answer.
28. Ans:(d)
A verb is the action or state of being in a sentence. In the passage, ‗he argues‘, shows his action,
therefore, D is the correct answer.

GENERAL KNOWLEDGE/CURRENT AFFAIRS

29. Ans:(b)
Explanations: Recently, the Ministry of Electronics & Information Technology (MeitY) has released
the Draft Data Centre Policy, which aims to simplify clearances for setting up data centres in the
country.
Vision of the Draft Data Centre Policy:
•Making India a Global Data Centre hub,
•Promote investment in the sector,
•Propel digital economy growth,
•Enable provisioning of trusted hosting infrastructure to fulfil the growing demand of the country and
facilitate state of the art service delivery to citizens.
The policy will be Strengthening the Atmanirbhar Bharat initiative by identifying possible
opportunities of manufacturing of data centre equipment (IT as well as non-IT) in the country. The
size of the digital economy in India is estimated to grow from USD 200 billion in 2017-18 to USD 1
trillion by 2025.
30. Ans:(c)
Explanations: Recently, the Ministry of Electronics & Information Technology (MeitY) has
released the Draft Data Centre Policy, which aims to simplify clearances for setting up data centres
in the country.
Vision of the Draft Data Centre Policy:
•Making India a Global Data Centre hub,
•Promote investment in the sector,
•Propel digital economy growth,
•Enable provisioning of trusted hosting infrastructure to fulfil the growing demand of the country and
facilitate state of the art service delivery to citizens.
The policy will be Strengthening the Atmanirbhar Bharat initiative by identifying possible
opportunities of manufacturing of data centre equipment (IT as well as non-IT) in the country. The
size of the digital economy in India is estimated to grow from USD 200 billion in 2017-18 to USD 1
trillion by 2025.
31. Ans:(a)
Explanations: Recently, the Ministry of Electronics & Information Technology (MeitY) has
released the Draft Data Centre Policy, which aims to simplify clearances for setting up data centres
in the country.
Vision of the Draft Data Centre Policy:
•Making India a Global Data Centre hub,
•Promote investment in the sector,
•Propel digital economy growth,
•Enable provisioning of trusted hosting infrastructure to fulfil the growing demand of the country and
facilitate state of the art service delivery to citizens.
The policy will be Strengthening the Atmanirbhar Bharat initiative by identifying possible
opportunities of manufacturing of data centre equipment (IT as well as non-IT) in the country. The
size of the digital economy in India is estimated to grow from USD 200 billion in 2017-18 to USD 1
trillion by 2025.
32. Ans:(d)
Explanations: Recently, the Ministry of Electronics & Information Technology (MeitY) has
released the Draft Data Centre Policy, which aims to simplify clearances for setting up data centres
in the country.
Vision of the Draft Data Centre Policy:
•Making India a Global Data Centre hub,
•Promote investment in the sector,
•Propel digital economy growth,
•Enable provisioning of trusted hosting infrastructure to fulfil the growing demand of the country and
facilitate state of the art service delivery to citizens.
The policy will be Strengthening the Atmanirbhar Bharat initiative by identifying possible
opportunities of manufacturing of data centre equipment (IT as well as non-IT) in the country. The
size of the digital economy in India is estimated to grow from USD 200 billion in 2017-18 to USD 1
trillion by 2025.
33. Ans:(c)
Explanations: Recently, the Ministry of Electronics & Information Technology (MeitY) has
released the Draft Data Centre Policy, which aims to simplify clearances for setting up data centres
in the country.
Vision of the Draft Data Centre Policy:
•Making India a Global Data Centre hub,
•Promote investment in the sector,
•Propel digital economy growth,
•Enable provisioning of trusted hosting infrastructure to fulfil the growing demand of the country and
facilitate state of the art service delivery to citizens.
The policy will be Strengthening the Atmanirbhar Bharat initiative by identifying possible
opportunities of manufacturing of data centre equipment (IT as well as non-IT) in the country. The
size of the digital economy in India is estimated to grow from USD 200 billion in 2017-18 to USD 1
trillion by 2025.
34. Ans:(a)
Explanations: US Pharma giant Pfizer and German biotechnology company BioNtech, which are
testing four vaccine candidates for Covid-19 as part of Project Lightspeed.
During a clinical trial, three aspects are assessed: reactogenicity (ability to produce common,
adverse reactions), immunogenicity (ability to provoke an immune response), and safety. Pfizer‘s
Covid-19 vaccine is among the ones requiring storage at ultracold temperatures.
35. Ans:(a)
Explanations: US Pharma giant Pfizer and German biotechnology company BioNtech, which are
testing four vaccine candidates for Covid-19 as part of Project Lightspeed.
During a clinical trial, three aspects are assessed: reactogenicity (ability to produce common,
adverse reactions), immunogenicity (ability to provoke an immune response), and safety. Pfizer‘s
Covid-19 vaccine is among the ones requiring storage at ultracold temperatures.
36. Ans:(d)
Explanations: US Pharma giant Pfizer and German biotechnology company BioNtech, which are
testing four vaccine candidates for Covid-19 as part of Project Lightspeed.
During a clinical trial, three aspects are assessed: reactogenicity (ability to produce common,
adverse reactions), immunogenicity (ability to provoke an immune response), and safety. Pfizer‘s
Covid-19 vaccine is among the ones requiring storage at ultracold temperatures.
37. Ans:(c)
Explanations: US Pharma giant Pfizer and German biotechnology company BioNtech, which are
testing four vaccine candidates for Covid-19 as part of Project Lightspeed.
During a clinical trial, three aspects are assessed: reactogenicity (ability to produce common,
adverse reactions), immunogenicity (ability to provoke an immune response), and safety. Pfizer‘s
Covid-19 vaccine is among the ones requiring storage at ultracold temperatures.
38. Ans:(a)
Explanations: US Pharma giant Pfizer and German biotechnology company BioNtech, which are
testing four vaccine candidates for Covid-19 as part of Project Lightspeed.
During a clinical trial, three aspects are assessed: reactogenicity (ability to produce common,
adverse reactions), immunogenicity (ability to provoke an immune response), and safety. Pfizer‘s
Covid-19 vaccine is among the ones requiring storage at ultracold temperatures.
39. Ans:(b)
Explanations: Chabahar Port is located on the Gulf of Oman and is only 72 km away from
the Gwadar port in Pakistan which has been developed by China. The port serves as the only
oceanic port of Iran and consists of two separate ports named Shahid Beheshti and Shahid
Kalantari.]
Although the project has secured a special waiver from the USA, India is hesitant to deal with
the construction company which has links with the Islamic Revolutionary Guard Corps (IRGC)
and is under the sanctions.
Farzad-B Gas Field is located in Persian Gulf (Iran) and the contract for exploration of the field was
signed in 2002 by Indian consortium comprising Oil and Natural Gas Corporation Limited (ONGC)
Videsh, Indian Oil Corporation and Oil India.
40. Ans:(a)
Explanations: Chabahar Port is located on the Gulf of Oman and is only 72 km away from
the Gwadar port in Pakistan which has been developed by China. The port serves as the only
oceanic port of Iran and consists of two separate ports named Shahid Beheshti and Shahid
Kalantari.]
Although the project has secured a special waiver from the USA, India is hesitant to deal with
the construction company which has links with the Islamic Revolutionary Guard Corps (IRGC)
and is under the sanctions.
Farzad-B Gas Field is located in Persian Gulf (Iran) and the contract for exploration of the field was
signed in 2002 by Indian consortium comprising Oil and Natural Gas Corporation Limited (ONGC)
Videsh, Indian Oil Corporation and Oil India.
41. Ans:(b)
Explanations: Chabahar Port is located on the Gulf of Oman and is only 72 km away from
the Gwadar port in Pakistan which has been developed by China. The port serves as the only
oceanic port of Iran and consists of two separate ports named Shahid Beheshti and Shahid
Kalantari.]
Although the project has secured a special waiver from the USA, India is hesitant to deal with
the construction company which has links with the Islamic Revolutionary Guard Corps (IRGC)
and is under the sanctions.
Farzad-B Gas Field is located in Persian Gulf (Iran) and the contract for exploration of the field was
signed in 2002 by Indian consortium comprising Oil and Natural Gas Corporation Limited (ONGC)
Videsh, Indian Oil Corporation and Oil India.
42. Ans:(a)
Explanations: Chabahar Port is located on the Gulf of Oman and is only 72 km away from
the Gwadar port in Pakistan which has been developed by China. The port serves as the only
oceanic port of Iran and consists of two separate ports named Shahid Beheshti and Shahid
Kalantari.]
Although the project has secured a special waiver from the USA, India is hesitant to deal with
the construction company which has links with the Islamic Revolutionary Guard Corps (IRGC)
and is under the sanctions.
Farzad-B Gas Field is located in Persian Gulf (Iran) and the contract for exploration of the field was
signed in 2002 by Indian consortium comprising Oil and Natural Gas Corporation Limited (ONGC)
Videsh, Indian Oil Corporation and Oil India.
43. Ans:(a)
Explanations: Chabahar Port is located on the Gulf of Oman and is only 72 km away from
the Gwadar port in Pakistan which has been developed by China. The port serves as the only
oceanic port of Iran and consists of two separate ports named Shahid Beheshti and Shahid
Kalantari.]
Although the project has secured a special waiver from the USA, India is hesitant to deal with
the construction company which has links with the Islamic Revolutionary Guard Corps (IRGC)
and is under the sanctions.
Farzad-B Gas Field is located in Persian Gulf (Iran) and the contract for exploration of the field was
signed in 2002 by Indian consortium comprising Oil and Natural Gas Corporation Limited (ONGC)
Videsh, Indian Oil Corporation and Oil India.
44. Ans:(c)
Explanations: Recently, the Meteor lake at Lonar in Buldhana district of Maharashtra and the
Soor Sarovar at Agra have been declared Ramsar sites, a conservation status conferred
by International Ramsar Convention on Wetlands.
World Wetlands Day is celebrated every year on 2nd February. This day marks the date of the
adoption of the Convention on Wetlands on 2 February 1971, in the Iranian city of Ramsar.
Uttar Pradesh has the most number of Ramsar Sites in India. It has 7 Indian Wetlands. Chilika Lake
is the largest Ramsar Site of India. Chilika Lake (Orissa) and Keoladeo National Park (Rajasthan)
were recognized as the first Ramsar Sites of India. Renuka Wetland in Himachal Pradesh is the
smallest wetland of India.
45. Ans:(a)
Explanations: Recently, the Meteor lake at Lonar in Buldhana district of Maharashtra and the
Soor Sarovar at Agra have been declared Ramsar sites, a conservation status conferred
by International Ramsar Convention on Wetlands.
World Wetlands Day is celebrated every year on 2nd February. This day marks the date of the
adoption of the Convention on Wetlands on 2 February 1971, in the Iranian city of Ramsar.
Uttar Pradesh has the most number of Ramsar Sites in India. It has 7 Indian Wetlands. Chilika Lake
is the largest Ramsar Site of India. Chilika Lake (Orissa) and Keoladeo National Park (Rajasthan)
were recognized as the first Ramsar Sites of India. Renuka Wetland in Himachal Pradesh is the
smallest wetland of India.
46. Ans:(b)
Explanations: Recently, the Meteor lake at Lonar in Buldhana district of Maharashtra and the
Soor Sarovar at Agra have been declared Ramsar sites, a conservation status conferred
by International Ramsar Convention on Wetlands.
World Wetlands Day is celebrated every year on 2nd February. This day marks the date of the
adoption of the Convention on Wetlands on 2 February 1971, in the Iranian city of Ramsar.
Uttar Pradesh has the most number of Ramsar Sites in India. It has 7 Indian Wetlands. Chilika Lake
is the largest Ramsar Site of India. Chilika Lake (Orissa) and Keoladeo National Park (Rajasthan)
were recognized as the first Ramsar Sites of India. Renuka Wetland in Himachal Pradesh is the
smallest wetland of India.
47. Ans:(a)
Explanations: Recently, the Meteor lake at Lonar in Buldhana district of Maharashtra and the
Soor Sarovar at Agra have been declared Ramsar sites, a conservation status conferred
by International Ramsar Convention on Wetlands.
World Wetlands Day is celebrated every year on 2nd February. This day marks the date of the
adoption of the Convention on Wetlands on 2 February 1971, in the Iranian city of Ramsar.
Uttar Pradesh has the most number of Ramsar Sites in India. It has 7 Indian Wetlands. Chilika Lake
is the largest Ramsar Site of India. Chilika Lake (Orissa) and Keoladeo National Park (Rajasthan)
were recognized as the first Ramsar Sites of India. Renuka Wetland in Himachal Pradesh is the
smallest wetland of India.
48. Ans:(c)
Explanations: Recently, the Meteor lake at Lonar in Buldhana district of Maharashtra and the
Soor Sarovar at Agra have been declared Ramsar sites, a conservation status conferred
by International Ramsar Convention on Wetlands.
World Wetlands Day is celebrated every year on 2nd February. This day marks the date of the
adoption of the Convention on Wetlands on 2 February 1971, in the Iranian city of Ramsar.
Uttar Pradesh has the most number of Ramsar Sites in India. It has 7 Indian Wetlands. Chilika Lake
is the largest Ramsar Site of India. Chilika Lake (Orissa) and Keoladeo National Park (Rajasthan)
were recognized as the first Ramsar Sites of India. Renuka Wetland in Himachal Pradesh is the
smallest wetland of India.
49. Ans:(b)
Explanations: Recently, Defence Research and Development Organisation (DRDO) successfully
test-fired Quick Reaction Surface-to-Air Missile system (QRSAM). The test was conducted at the
Integrated Test Range, Chandipur off Odisha coast. The Test was in continuation of a series
of missile tests conducted by DRDO over the past two months. It has been designed for induction
into the Army and has a range of 25 to 30 km.
Satheesh Reddy, Head of the Defence Research and Development Organisation (DRDO), has
been given a two-year extension as the Secretary, Department of Defence Research and
Development and Chairman DRDO beyond his tenure of August 26, 2020
50. Ans:(a)
Explanations: Recently, Defence Research and Development Organisation (DRDO) successfully
test-fired Quick Reaction Surface-to-Air Missile system (QRSAM). The test was conducted at the
Integrated Test Range, Chandipur off Odisha coast. The Test was in continuation of a series
of missile tests conducted by DRDO over the past two months. It has been designed for induction
into the Army and has a range of 25 to 30 km.
Satheesh Reddy, Head of the Defence Research and Development Organisation (DRDO), has
been given a two-year extension as the Secretary, Department of Defence Research and
Development and Chairman DRDO beyond his tenure of August 26, 2020
51. Ans:(a)
Explanations: Recently, Defence Research and Development Organisation (DRDO) successfully
test-fired Quick Reaction Surface-to-Air Missile system (QRSAM). The test was conducted at the
Integrated Test Range, Chandipur off Odisha coast. The Test was in continuation of a series
of missile tests conducted by DRDO over the past two months. It has been designed for induction
into the Army and has a range of 25 to 30 km.
Satheesh Reddy, Head of the Defence Research and Development Organisation (DRDO), has
been given a two-year extension as the Secretary, Department of Defence Research and
Development and Chairman DRDO beyond his tenure of August 26, 2020
52. Ans:(c)
Explanations: Recently, Defence Research and Development Organisation (DRDO) successfully
test-fired Quick Reaction Surface-to-Air Missile system (QRSAM). The test was conducted at the
Integrated Test Range, Chandipur off Odisha coast. The Test was in continuation of a series
of missile tests conducted by DRDO over the past two months. It has been designed for induction
into the Army and has a range of 25 to 30 km.
Satheesh Reddy, Head of the Defence Research and Development Organisation (DRDO), has
been given a two-year extension as the Secretary, Department of Defence Research and
Development and Chairman DRDO beyond his tenure of August 26, 2020
53. Ans:(c)
Explanations: Recently, Defence Research and Development Organisation (DRDO) successfully
test-fired Quick Reaction Surface-to-Air Missile system (QRSAM). The test was conducted at the
Integrated Test Range, Chandipur off Odisha coast. The Test was in continuation of a series
of missile tests conducted by DRDO over the past two months. It has been designed for induction
into the Army and has a range of 25 to 30 km.
Satheesh Reddy, Head of the Defence Research and Development Organisation (DRDO), has
been given a two-year extension as the Secretary, Department of Defence Research and
Development and Chairman DRDO beyond his tenure of August 26, 2020
54. Ans:(c)
Explanation: Arunachal Pradesh recorded 1,084 females born per thousand males, followed by
Nagaland (965), Mizoram (964), Kerala (963). The worst sex ratio was reported in Manipur (757),
Lakshadweep (839) and Daman & Diu (877), Punjab (896) and Gujarat (896).
Registrar General of India was founded in 1961 by the Government of India under the Ministry of
Home Affairs. It arranges, conducts and analyses the results of the demographic surveys of India
including Census of India and Linguistic Survey of India. The position of Registrar is usually held by
a civil servant holding the rank of Joint Secretary.
Civil Registration System (CRS) in India is the unified process of continuous, permanent,
compulsory and universal recording of the vital events (births, deaths, stillbirths) and characteristics
thereof. The data generated through a complete and up-to-date CRS is essential for socio-economic
planning.
Prof. Amartya Kumar Sen, in his world famous article “Missing Women‟ has statistically proved
that during the last century, 100 million women have been missing in south Asia. This is due to
discrimination leading to death, experienced by them from womb to tomb in their life cycles.
55. Ans:(b)
Explanation: Arunachal Pradesh recorded 1,084 females born per thousand males, followed by
Nagaland (965), Mizoram (964), Kerala (963). The worst sex ratio was reported in Manipur (757),
Lakshadweep (839) and Daman & Diu (877), Punjab (896) and Gujarat (896).
Registrar General of India was founded in 1961 by the Government of India under the Ministry of
Home Affairs. It arranges, conducts and analyses the results of the demographic surveys of India
including Census of India and Linguistic Survey of India. The position of Registrar is usually held by
a civil servant holding the rank of Joint Secretary.
Civil Registration System (CRS) in India is the unified process of continuous, permanent,
compulsory and universal recording of the vital events (births, deaths, stillbirths) and characteristics
thereof. The data generated through a complete and up-to-date CRS is essential for socio-economic
planning.
Prof. Amartya Kumar Sen, in his world famous article “Missing Women‟ has statistically proved
that during the last century, 100 million women have been missing in south Asia. This is due to
discrimination leading to death, experienced by them from womb to tomb in their life cycles.
56. Ans:(b)
Explanation: Arunachal Pradesh recorded 1,084 females born per thousand males, followed by
Nagaland (965), Mizoram (964), Kerala (963). The worst sex ratio was reported in Manipur (757),
Lakshadweep (839) and Daman & Diu (877), Punjab (896) and Gujarat (896).
Registrar General of India was founded in 1961 by the Government of India under the Ministry of
Home Affairs. It arranges, conducts and analyses the results of the demographic surveys of India
including Census of India and Linguistic Survey of India. The position of Registrar is usually held by
a civil servant holding the rank of Joint Secretary.
Civil Registration System (CRS) in India is the unified process of continuous, permanent,
compulsory and universal recording of the vital events (births, deaths, stillbirths) and characteristics
thereof. The data generated through a complete and up-to-date CRS is essential for socio-economic
planning.
Prof. Amartya Kumar Sen, in his world famous article “Missing Women‟ has statistically proved
that during the last century, 100 million women have been missing in south Asia. This is due to
discrimination leading to death, experienced by them from womb to tomb in their life cycles.
57. Ans:(d)
Explanation: Arunachal Pradesh recorded 1,084 females born per thousand males, followed by
Nagaland (965), Mizoram (964), Kerala (963). The worst sex ratio was reported in Manipur (757),
Lakshadweep (839) and Daman & Diu (877), Punjab (896) and Gujarat (896).
Registrar General of India was founded in 1961 by the Government of India under the Ministry of
Home Affairs. It arranges, conducts and analyses the results of the demographic surveys of India
including Census of India and Linguistic Survey of India. The position of Registrar is usually held by
a civil servant holding the rank of Joint Secretary.
Civil Registration System (CRS) in India is the unified process of continuous, permanent,
compulsory and universal recording of the vital events (births, deaths, stillbirths) and characteristics
thereof. The data generated through a complete and up-to-date CRS is essential for socio-economic
planning.
Prof. Amartya Kumar Sen, in his world famous article “Missing Women‟ has statistically proved
that during the last century, 100 million women have been missing in south Asia. This is due to
discrimination leading to death, experienced by them from womb to tomb in their life cycles.
58. Ans:(d)
Explanation: Arunachal Pradesh recorded 1,084 females born per thousand males, followed by
Nagaland (965), Mizoram (964), Kerala (963). The worst sex ratio was reported in Manipur (757),
Lakshadweep (839) and Daman & Diu (877), Punjab (896) and Gujarat (896).
Registrar General of India was founded in 1961 by the Government of India under the Ministry of
Home Affairs. It arranges, conducts and analyses the results of the demographic surveys of India
including Census of India and Linguistic Survey of India. The position of Registrar is usually held by
a civil servant holding the rank of Joint Secretary.
Civil Registration System (CRS) in India is the unified process of continuous, permanent,
compulsory and universal recording of the vital events (births, deaths, stillbirths) and characteristics
thereof. The data generated through a complete and up-to-date CRS is essential for socio-economic
planning.
Prof. Amartya Kumar Sen, in his world famous article “Missing Women‟ has statistically proved
that during the last century, 100 million women have been missing in south Asia. This is due to
discrimination leading to death, experienced by them from womb to tomb in their life cycles.
59. Ans:(d)
The Regional Comprehensive Economic Partnership (RCEP) is a free trade agreement between the
Asia-Pacific nations of Australia, Brunei, Cambodia, China, Indonesia, Japan, Laos,
Malaysia, Myanmar, New Zealand, Philippines, Singapore, South Korea, Thailand, and Vietnam (15
countries).
60. Ans:(b)
The RCEP was first proposed at the 19th ASEAN meet in November 2011.
61. Ans:(c)
The RCEP was later also conceived by the ASEAN members and China as a response to the US-
led Trans-Pacific Partnership (TPP) — later renamed as Comprehensive and Progressive
Agreement for Trans-Pacific Partnership — after America opted out of this deal in 2017. The key
features of TPP included comprehensive market access, regional approach to commitments,
inclusive trade, regional integration, and addressing new trade challenges.
62. Ans:(a)
Many signatories of the RCEP already have FTAs with each other. The existing FTAs can be very
complicated to use compared to RCEP. This is because businesses with global supply chains might
face tariffs even within an FTA if their products contain components that are made elsewhere.
However, under RCEP, member nations would be treated equally. It would also incentivise
companies in member nations to look at the trade region for suppliers.
63. Ans:(d)
The RCEP is expected to eliminate a series of tariffs on imported products for the member nations in
the coming 20 years. The agreement also includes rules on intellectual property,
telecommunications, financial and professional services, and e-commerce.
QUANTITATIVE TECHNIQUES

64. Ans:(d)
Let the height of water in vessel D be h cm.
Volume of vessel E = Volume of water in vessel D
⇒ 23×Π×213=Π×282×h
⇒ h=63/8=7 7/8 cm
65. Ans:(b)
Capacity of vessel A = (length)3=353=42875 cm3
Capacity of vessel B = Area of bottom × height = 1260 × 25 = 31500 cm3
Required percentage = 42875-31500/31500×100 =36 1/9%
66. Ans:(a)
Radius of vessel C Height of vessel C=34
⇒ Radius28=34
Radius of vessel C = 21 cm
Slant height of vessel C = Radius2+Height2
=212+282
=35 cm
Ratio of lateral surface areas of vessel C and vessel E:
Lateral Surface Area of vessel C Lateral Surface Area of vessel E
=Π×Radius ×Slant Height2×Π×Radius2
=Π×21×352×Π×212
=56
=5:6
67. Ans:(c)
Let the radius of vessel F be r cm
Capacity of cylindrical vessel F = 10% more than capacity of vessel A
227×r2×49=1.1×35×35×35
⇒r=17.5 cm
Required percentage = 21-17.521=100
=1623%
68. Ans:(d)
Total Area to be painted
= Lateral Surface Area of vessel D + Lateral Surface Area+Area of the bottom of vessel A
=2×227×28×20+5×35×35
=3520+6125
=9645cm2
Total Expenditure =0.2×9645 =Rs.1929
69. Ans:(d)
Total number of students = 5000
College A
Arts = (7.2/100) * 5000 = 360
Science = (16.8/100) * 5000 = 840
College B
Arts = (18/100) * 5000 = 900
Science = 900
College C
Arts = (16/100) * 5000 = 800
Science = (24/100) * 5000 = 1200
Now in the table, College C,
Male Arts = 800; Female arts = 0
Male Science = 400; Female Science = 1200 - 400 = 800
College B
Arts = X + 2X = 3X = 900; X =300
Male Arts = 600; Female Arts = 300
Female Science = 400; Male Science = 900 - 400 = 500
College A
Male Arts = X - 100 = 200; Female Arts = 360 - 200 = 160
Female Science = X = 300; Male Science = 840-300= 540

Male Arts = 800; Female Science = 800, Ratio = 1: 1


70. Ans:(a)
College B (Arts + Science) = 900 + 900 = 1800 College A (Arts + Science) = 360 + 840 = 1200 B is
more than A;
Percentage = [(1800 - 1200)/1200] * 100 = 50% more
71. Ans:(a)
Total Number of Science Students = 840 + 900 + 1200 = 2940
Average = 2940/3 = 980
72. Ans:(d)
Male Students = 200 + 600 + 800 + 540 + 500 + 400 = 3040
Female students = 5000 - 3040 = 1960
Ratio = 3040: 1960 = 76: 49
73. Ans:(d)
College B Male Students = 600 + 500 = 1100
College B Female Students = 300 + 400 = 700
50% of Male + 75% of Female = (50/100) * 1100 + (75/100) * 700
Number of students who got job = 1075
Number of students who did not get job = 1800 - 1075 = 725
Ratio = 1075: 725 = 43: 29
74. Ans:(d)
Common Explanation:
Total number of students = 1800
Number of female students preparing for CLAT = 252
Number of students preparing for CLAT = 252 * 100/40 = 630
Number of male students preparing for CLAT = 630 - 252 = 378
Number of students preparing for AILET = 630 * 8/7 = 720
Number of male students preparing for AILET = (720 + 80)/2 = 400
Number of female students preparing for AILET = 720 - 400 = 320
Number of students preparing for MHCET = 1800 - 630 - 720 = 450
Total weight of students preparing for MHCET = 450 * 59 = 26550 kg
Total weight of male students preparing for MHCET = 20475 kg
Then, total weight of female students preparing for MHCET = 26550 - 20475 = 6075 kg
Now, number of male students preparing for MHCET = 630/2 = 315
Then, number of female students preparing for MHCET = 450 - 315 = 135
And, average weight of female students preparing for MHCET = 6075/135 = 45 kg
Average weight of female students preparing for AILET = 11520/320 = 36 kg
Average weight of female students preparing for MHCET = 45 kg
Therefore, ratio = 45: 36 = 5: 4
75. Ans:(b)
Common Explanation:
Total number of students = 1800
Number of female students preparing for CLAT = 252
Number of students preparing for CLAT = 252 * 100/40 = 630
Number of male students preparing for CLAT = 630 - 252 = 378
Number of students preparing for AILET = 630 * 8/7 = 720
Number of male students preparing for AILET = (720 + 80)/2 = 400
Number of female students preparing for AILET = 720 - 400 = 320
Number of students preparing for MHCET = 1800 - 630 - 720 = 450
Total weight of students preparing for MHCET = 450 * 59 = 26550 kg
Total weight of male students preparing for MHCET = 20475 kg
Then, total weight of female students preparing for MHCET = 26550 - 20475 = 6075 kg
Now, number of male students preparing for MHCET = 630/2 = 315
Then, number of female students preparing for MHCET = 450 - 315 = 135
And, average weight of female students preparing for MHCET = 6075/135 = 45 kg
Number of boys preparing for CLAT = 378
Number of boys preparing for AILET = 400
Therefore, percentage = (378/400) * 100 = 94.5%
76. Ans:(d)
Common Explanation:
Total number of students = 1800
Number of female students preparing for CLAT = 252
Number of students preparing for CLAT = 252 * 100/40 = 630
Number of male students preparing for CLAT = 630 - 252 = 378
Number of students preparing for AILET = 630 * 8/7 = 720
Number of male students preparing for AILET = (720 + 80)/2 = 400
Number of female students preparing for AILET = 720 - 400 = 320
Number of students preparing for MHCET = 1800 - 630 - 720 = 450
Total weight of students preparing for MHCET = 450 * 59 = 26550 kg
Total weight of male students preparing for MHCET = 20475 kg
Then, total weight of female students preparing for MHCET = 26550 - 20475 = 6075 kg
Now, number of male students preparing for MHCET = 630/2 = 315
Then, number of female students preparing for MHCET = 450 - 315 = 135
And, average weight of female students preparing for MHCET = 6075/135 = 45 kg
Total number of students = 1800
Number of female students = 252 + 320 + 135 = 707
Therefore, percentage = (707/1800) * 100 = 39% (approx.)
77. Ans:(c)
Common Explanation:
Total number of students = 1800
Number of female students preparing for CLAT = 252
Number of students preparing for CLAT = 252 * 100/40 = 630
Number of male students preparing for CLAT = 630 - 252 = 378
Number of students preparing for AILET = 630 * 8/7 = 720
Number of male students preparing for AILET = (720 + 80)/2 = 400
Number of female students preparing for AILET = 720 - 400 = 320
Number of students preparing for MHCET = 1800 - 630 - 720 = 450
Total weight of students preparing for MHCET = 450 * 59 = 26550 kg
Total weight of male students preparing for MHCET = 20475 kg
Then, total weight of female students preparing for MHCET = 26550 - 20475 = 6075 kg
Now, number of male students preparing for MHCET = 630/2 = 315
Then, number of female students preparing for MHCET = 450 - 315 = 135
And, average weight of female students preparing for MHCET = 6075/135 = 45 kg
Number of female students preparing for CLAT = 252
Number of male students preparing for MHCET = 315
Therefore, difference = 315 - 252 = 63
78. Ans:(d)

Person who like Golgape = 100 = A+B


Peroson who like Chicket Tica = 120 = B+C
Person who like none of these = 40
Person who like exactly one of two item = 100 = A+C
ATQ,
2(A+B+C) = 100+120+100 = 320
A+B+C = 160
A = 160 – 120 = 40
B = 160 – 100 = 60
C = 160 – 100 = 60
D = Person who do not like any of two = 40
People had like both the items = B = 60
79. Ans:(d)
Person who like Only GolGape A = 40
Person who like Only Chicken Tica = B = 60
Required %= 40-60/60 *100 = 33.33%

LOGICAL REASONING

80. Ans:(a)
Options (B), (C) and (D) just form the part of the idea of the author in the passage. But, option (A) is
the conclusion of the study conducted by U of W. Therefore, option (A) is the correct answer.
81. Ans:(b)
Option (A), (C) and (D) are further strengthening the conclusion in the above passage. Option (B) is
weakening the main idea of the study in the above passage by saying that there are many reasons
other than video games which are making teenagers violent. Therefore, option (B) is the correct
answer.
82. Ans:(a)
Option (B) and (C) are further weakening the main idea of the author in the above passage. Option
(A) is further strengthening the main idea of the author in the passage. Therefore, option (A) is the
correct answer.
83. Ans:(c)
The conclusion of the study in the above passage is that teenagers can be prevented from
committing violent acts by not being allowed to play violent video games, and this idea is based on
the premise that teenagers who have been playing violent video games commit a much greater
number of violent acts in their play than do the teenagers who have been playing nonviolent video
games. Therefore, option (C) is the correct answer.
84. Ans:(d)
Option (A) is not the correct answer because it is generalizing the main idea of the first para in the
above passage. Option (B) and (C) are not correct answer because they are irrelevant. Option (D) is
the correct answer because the study has assumed that there will not be any other reason which
may be responsible for the violent behavior of the teenagers.
85. Ans:(a)
In the last para, the author cites the example of his friend who faced a similar issue. It is thus
strengthening his argument. Option B is wrong because the conclusion is already stated in the first
para itself. Option C is wrong because there‘s no hint of a question arising. Option D is wrong
because the plight of the author already finds mention in the first para. Hence, A is correct.
86. Ans:(b)
Rationale- The premise that is missing in the above passage is that if an electronic device breaks
quickly than it was probably manufactured in Dharavi, which is leading the author towards the
conclusion that the thing was probably manufactured in Dharavi. Therefore, the correct answer is
option (B).
87. Ans:(c)
Rationale- Option (A) and (B) are not correct because they both are strengthening the conclusion of
the author. Option (D) is not the correct answer because it may or may not weaken the conclusion of
the author. Option (C) is the correct answer, if that statement is true it will definitely weaken the
conclusion of the author.
88. Ans:(a)
Rationale- Premises are the facts or evidence that support or lead to the conclusion. Therefore, the
above statement forms premise of the passage which leads to the conclusion. Hence, option (A) is
the correct answer.
89. Ans:(d)
Rationale- In the above passage the pattern of reasoning is that there is hidden assumption that has
been made by the author which is leading him towards the conclusion. In options (A), (B) and (C)
assumptions has been made by the author, but in option (D) the reasoning has been provided by the
author which is leading towards the conclusion. Therefore, the correct answer is option (D).
90. Ans:(c)
Option (b) is merely a repetition of the FDA inspector‘s recommendation in the passage, which is
said to be a logical fallacy. Options (a) and (d), by focusing only on every fourth egg, make the same
kind of logical error. Only option (c), which suggests checking all the eggs, not just every fourth one,
is a logically valid course of action.
Hence (c).
91. Ans:(a)
Options (c) and (d) can be ruled out at once, since, in the last sentence of the passage, the author
agrees with the quoted sentence and says that we have not gotten very far with the programme that
Premchand recommends. Since the author agrees with Premchand, it is clear that he is arguing in
favour of what Premchand suggests, not against it. Thus option (a) is correct and option (b) is wrong.
Hence (a).
92. Ans:(a)
Options (b) and (c) are both mathematically correct. The ‗100%‘ in option (d) is not meant as an
estimation of probability, but merely as a way of expressing certainty; so its mathematical validity is
irrelevant. Only option (a) makes the same mathematical mistake as the example of the forecaster,
who adds the two percentages together, rather than realizing that the overall probability averaged
over two days should remain 50%. Hence (a).
93. Ans:(b)
The FDA inspector‘s mistake in thinking is that she confuses a statistical probability of one in four
eggs being bad with the idea that literally every fourth egg in any given batch is bad. Options (a), (b)
and (d) make a similar mistake – they assume, respectively, that every fifth person in any group is
crazy, that accidents occur on that road literally every seventh day, and that every eighth person in
any given group is unable to finish college. Only option (b) is not an example of a mistake – if the
papers have to be in one of six drawers, and they are not in five of them, then they must be in the
sixth drawer.
Hence, (b).
94. Ans:(d)
According to the passage, what Ramanujan calls ‗innumeracy‘ is not a mere lack of aptitude for
mathematics or difficulty grasping particular mathematical concepts, but rather an inability to think in
statistical terms. The examples of innumeracy given in the passage further emphasize that the
phenomenon can be seen when adult professionals are unable to use simple statistical thinking in
everyday life. Thus, only option (d) is a case of ‗innumeracy‘; the rest are merely cases of students
having difficulty with numbers or maths as a subject.
Hence, (d).
95. Ans:(d)
Explanation: Option (D) is the correct answer which suggests that the Indian Railways has to walk a
tightrope, which means to deal with a difficult situation, especially one involving making a decision
between two opposing plans of action. Here, the Indian Railways has to "rationalise its workforce on
the one hand and scale up services to meet passengers' expectations on the other" in order to
recover its losses in the current scenario.
96. Ans:(c)
Explanation: It is not the overall essence of the passage; thus, option (A) cannot be the answer. It is
consistent with the passage as well, which makes option (B) incorrect. It is not refuted by the author
in the text. So, option (D) is incorrect. Option (C) is correct as it further leads to draw that the
situation of the Railways is very bad.
97. Ans:(a)
Explanation: From the line 'The nation's lifeline awaits a new lease of life', it could be inferred that
option (A) is true. Option (B) is incorrect since the passage states that the surplus is in decline, not
the sector's financial state as a whole. Option (C) is incorrect because it cannot be inferred that Lalu
was the best Railway Minister as there is no such information provided about him.
98. Ans:(c)
Explanation: Only option (C) includes a step that can be taken by the Railway Ministry to popularise
the Give Up scheme asking senior citizens to give up the concessions that they are provided for the
benefit of the Indian Railways. The text also states that there is lack of awareness among the senior
citizens among them which has not yielded any results. In order to improve awareness, option (C)
which talks about launching a nationwide campaign to create awareness could bring about the
result. Option (A) is for youngsters, while the scheme is for senior citizens. Option (B) is a forced
measure which is not consistent with the efforts to be taken by the Ministry in the text. Option (D) is
incorrect since it is something that can be done for those who give up the concession - how they will
be made aware about it has nothing to do with providing them a letter from the Railway Minister.
99. Ans:(b)
Explanation: Option (A) is neutral and it is insufficient to infer whether the situation is worsening or
improving. Option (B) states about the worsening financial condition of the Indian Railways. Option
(C) relates to the 'Give Up' scheme which is not consistent with the question. Option D is the
contradictory statement.
100. Ans:(c)
The passage raises the question, whether Charlie is the culprit or not. It indicates that the passage
intends to say that Charlie may not be the real culprit. Hence, (a) is not the answer. The statement
clearly indicated about the shortcomings on the part of Charlie. Hence, (b) is not the answer. The
passage does not intend to say that Charlie may be the main person. It is clear when he raises the
question. Hence, (c) is the answer. The passage believes that Charlie has a hand in the incident.
The passage does not find fault with arresting Charlie. It intends to say that there can be some other
people also behind the mishap. Hence, (d) is not the answer.
101. Ans:(c)
Option a) shows negligence on the part of the Charlie but it would be not the basis on which the
police can arrest him. Option b) is irrelevant as someone being notorious has nothing to do with the
boat being capsize. Option d) is a counter-allegation against the police and not an assumption police
would make. Option c) states that Charlie took bribe from boat owner and let more people on boat
than it is allowed which let to sinking of the boat. Charlie has the duty to check ticket and let only
limited people go on boat and therefore c) is the correct answer.
102. Ans:(a)
Author‘s statement that police arrested the department's ticket collector, Charlie, based on the
allegation that he did not check the tickets properly becomes his premise for the argument that the
justice is sold at the hands of the rich and police only arrest the innocent. Therefore, ―a‖ is the
correct answer.
103. Ans:(c)
The main idea of the passage is the system in which the culprit goes free while the innocent is
suffering on his behalf, therefore, option c is the correct answer. Option d is not correct because the
author does not directly say that the police is corrupt but only that the justice system is sold at the
hand of rich. It is self-explanatory why option a and b are not correct.
104. Ans:(b)
Option a, c and d does not resonate with the argument of the author. The author states that justice is
sold at the hands of the people with money and the real culprits are not punished but only innocent.
Only option b resonates with the theme of the passage. Therefore, b is the correct answer.
105. Ans:(a)
In case an arbitral award is challenged, release of monies awarded is subject to the production of
bank guarantee, as per the premise, hence A is the right answer.
106. Ans:(a)
Airport Authority of India is a government department. It has challenged the contractor/
Concessionaire. Following the ―if‖ conditional option A is the right answer
107. Ans:(b)
If the govt. or PSU challenge the decision, only then there is a restriction of 75 percent. If the Award
is not challenged, the other party MAY get full monies. Following the ―if‖ conditional option B is the
right answer
108. Ans:(d)
The given sequence of group of letter number follows the pattern:
For numbers:

For letters:
Third letter from F is I. Third letter from I is L. This logic will continue, so, required letter is U.
Therefore, missing group of letter number is = 252 U
109. Ans:(a)

Clearly, all the three brothers are not available at the same time on any day of the week.
110. Ans:(d)

There are four such days.


111. Ans:(d)
Clearly, Navin and Rajiv are available at home at the same time on Monday and Thursday.

LEGAL REASONING

112. Ans:(b)
Explanation: The Supreme Court in Selvi v. State held that compelling a person to undergo narco-
analysis, brain mapping and polygraph resulted in violation of Article 20(3). Therefore, the police
cannot force Hassan to go through brain mapping and it will be illegal.
113. Ans:(d)
Explanation: To be a witness means imparting knowledge in respect of relevant fact, by means of
oral statements or statements in writing, by a person who has personal knowledge of the facts to be
communicated to a court or to a person holding an enquiry or investigation.
114. Ans:(a)
Explanation: Section 53 of the Code of Criminal Procedure expressly says that ‗reasonable force‘
can be used for the medical examination of an accused by a police officer. Therefore, Jignesh can
be forced by the police to give any physical information to the police.
115. Ans:(b)
Explanation: State of Bombay v. Kathi Kalu Oghad held that taking blood samples of an accused
will not violate Article 20(3). The protection of Article 20(3) is available to matters within the
subjective consciousness of a person and not to his physical features.
116. Ans:(d)
Explanation: The Supreme Court in Selvi v. State held that compelling a person to undergo narco-
analysis, brain mapping and polygraph resulted in violation of Article 20(3) of the Indian Constitution.
117. Ans:(b)
Explanation: This is because no stranger to the consideration can take advantage of a contract.
Applying this in the existing case would disallow Vijay to sue the representatives as he is the
stranger to the agreement.
118. Ans:(a)
Explanation: This is because even if the wife is not a party to the agreement per se, the
arrangement so formed is falls under marriage settlement changing the dynamics of benefit and
interest. The changed dynamics due to marriage settlement makes her an indirect party and reap
benefits for the same.
119. Ans:(c)
Explanation: This is because in the existing case a position of trust was created in favor of Johan
for the specified amount and the village.Hence, he was entitled to file the claim for the same.
120. Ans:(b)
Explanation: This is because of the reason of admission of the obligation in front of the third party
which allows them to file a claim and get benefitted out of the said agreement.
121. Ans:(a)
Explanation: This is due to the reason that the principal is the real party to the contract while the
representation is concluded by his agent. Therefore, this allows the principal to file a suit against the
non-performance of contractual obligations by the other party.
122. Ans:(b)
Explanation: ‗a newly-wed wife being sexually harassed by her husband.‘ Because the convention
does not provide any commentary upon the abuse arising out of the marital relationships alone.
123. Ans:(d)
Explanation: in all the three situations mentioned, there exists gender-based violence arising out of
the workplaces. Hence all of the above situations are within the jurisdiction of the convention as
mentioned in the passage.
124. Ans:(b)
Explanation: Article 4 of the conventions stipulates for the members such as the
employer, government etc. to cull out more inclusive and gender responsive approaches that can
help curb the sexual harassment at workplaces. Whereas in option ‗b‘, the said policy only provides
a financial aid so as to ensure that the employee remains in the organisation. It does not address the
objective of article 4.
125. Ans:(c)
Explanation: ‗A trans women facing discrimination while filling out the recruitment application form
in a firm which does not allow for a column of her gender identity‘ because the third paragraph of the
passage mentions that the present convention does not cover violation of human rights (dignity and
self-determination).
126. Ans:(c)
Explanation: Both of the above situations are within the managerial powers of the employer and
hence do not place any undue or unreasonable burden upon the employer. Hence ‗d‘.
127. Ans:(d)
Correct Answer: The Supreme Court‘s ruling excluding the ―creamy layer‖ from reservation in
promotions for the Scheduled Castes and the Scheduled Tribes has predictably sparked a debate.
Alok Prasanna Kumar argues that the court erred by holding that the Scheduled Castes and the
Scheduled Tribes can avail of reservation in promotions only if they are backward, and challenges its
presumption that backwardness and untouchability will disappear if a generation or two of Dalits and
Adivasis get access to education and jobs. This presumption is also rebutted by PS Krishnan, who
points to instances of Dalits having been treated as untouchable even after occupying high office.
128. Ans:(a)
Correct Answer: The whole object of reservation is to see that backward classes of citizens move
forward so that they may march hand in hand with other citizens of India on an equal basis. This will
not be possible if only the creamy layer within that class bag all the coveted jobs in the public sector
and perpetuate themselves, leaving the rest of the class as backward as they always were
129. Ans:(c)
In the light of experience, here and elsewhere, the danger of ‗reservation‘, it seems to me, is three-
fold. Its benefits, by and large, are snatched away by the top creamy layer of the ‗backward‘ caste or
class, thus keeping the weakest among the weak always weak and leaving the fortunate layers to
consume the whole cake.
130. Ans:(d)
This line of reasoning, that the poorer among Dalits and Adivasis are losing out on reservation
benefits because the creamy layer corners all opportunities, is not backed by evidence. Consider the
central civil services: senior officials from Dalit and Adivasi communities are too few to even fill all
reserved positions. In fact, posts set aside for the Scheduled Castes and the Scheduled Tribes often
go unfilled and the government has to undertake special drives to fill backlog vacancies.The
―weakest among the weak‖ are not being deprived by the creamy layer, but by the same obstacles
that made reservation necessary in the first place.
131. Ans:(c)
The above passage argues that to exclude, through the creamy layer test, the people who have
overcome such hindrances to the extent they are able to avail of reservation goes against the very
purpose of reservation.
132. Ans:(a)
Explanation: Recording of statement of a rape victim by a Judicial Magistrate is a must and such
Magistrates cannot refuse to record the statement of a rape victim under Section 164 CrPC, for the
reason that she is not under any disability.
133. Ans:(b)
Explanation: Clause (a) of sub-section (5A) of Section 164 of the Code of Criminal Procedure
enjoins that the Judicial Magistrate shall record the statement of the person against whom the
offences of rape has been committed. Such victims may or may not include persons who are
mentally or physically disabled.
134. Ans:(c)
Explanation: The Supreme Court of India in State of Karnataka v. Shivanna directed that if there is
any delay exceeding 24 hours in taking the victim to the Magistrate, the Investigating Officer should
record the reasons for the same in the case diary and hand over a copy of the same to the
Magistrate.
135. Ans:(d)
Explanation: The Supreme Court of India in the State of Karnataka v. Shivanna directed that the
Investigating Officer shall make immediate steps to take the victim to any Metropolitan / preferably
Judicial Magistrate for the purpose of recording her statement under Section 164 CrPC.
136. Ans:(a)
Explanation: The Supreme Court of India in the State of Karnataka v. Shivanna directed that a copy
of the statement should be handed over to the Investigating Officer immediately with a specific
direction that the contents of such statement should not be disclosed to any person till
chargesheet/report under Section 173 CrPC is filed.
137. Ans:(a)
the court will not accept the allegations made by the doctor because they are evidence less and
cannot be proven in the court of law. The above incident is not that of grievous hurt and hence the
burden of proof is on prosecution i.e. the doctor.
138. Ans:(b)
Explanation: the court will not accept the allegations made by Tabrez because they are evidence
less and cannot be proven in the court of law. Tabrez is not a ‗healthcare service personnel‘. So, the
rules in passage does not apply to him and the burden of proof still remains on him.
139. Ans:(b)
Explanation: 19th June 2020. As per the passage, investigation needs to be completed within 30
days of filing of FIR. Confusions might rise as to it should be 20th June 2020, but a month and 30
days is different as May is a 31-day month.
140. Ans:(d)
Explanation: As per the passage, the burden of proof is on the accused and to prove their
innocence, they can prove any of the first two options in the court of law.
141. Ans:(c)
Explanation: 26th April 2021. As per the passage, trial has to be completed within 1 year of filing of
FIR.
142. Ans:(b)
Explanation: the possession was illegal as the BMC is not competent enough to do so. In the
context of the passage, there needs to be a legal backing to these kinds of actions of the executive.
143. Ans:(d)
Explanation: the possession will be illegal as the BMC is not competent enough to do so. The BMC
is still not competent as a quasi-legal proceeding does not give a proper backing to the executive
actions.
144. Ans:(b)
Explanation: the possession is illegal as the act doesn‘t amount to be that of public interest. As per
the facts, the possession does not seem to be in public interest as there was no dire need for the
same.
145. Ans:(c)
Explanation: the court will not give him remedies as it‘s not its jurisdiction to decide upon. Right to
property not being a fundamental right cannot be claimed under Article 32 of the constitution.
146. Ans:(a)
Explanation: the court will give him remedies as it‘s a clear case on merits and will issue a writ of
mandamus. Unlike the Supreme Court, the HC can pass remedies for violation of constitutional
rights as well.
147. Ans:(c)
Kollam District court had pecuniary, original and territorial jurisdiction over the matter and High
court heard the case as an appeal.
148. Ans:(b)
because no other court can hear such dispute which shows its exclusive jurisdiction.
149. Ans:(c)
because the Supreme Court does not have jurisdiction over the dispute arising out of any treaty
executed before the commencement of the constitution.
150. Ans:(b)
because the karnataka state commission did not have territorial jurisdiction over the matter as the
business was carried on from Uttar Pradesh. As the amount claimed was one crore,
state commission had the pecuniary jurisdiction over the matter.
Legal Edge 12 Mock

Answers & Details

ENGLISH

1. Ans:(a)
Solution: The expression actually implies that Johnsy is Joanna‘s nickname or in other words,
Johnsy is short for Joanna. Options (b) and (d) are totally off topic and option (c) is opposite to the
answer.
2. Ans:(d)
Solution:The author has provided a human quality to the streets (those of running, going crazy and
breaking) and therefore the figure of speech can be referred to as a Personification. The options (a)
and (b) seem tempting as there is some degree of comparison but Personification is more
appropriate in this case. Onomatopoeia is incorrect as there are no words conveying a sound.
3. Ans:(c)
Solution:The word “squatty” is used to describe something which is short and thick. The word
―stunted‖ comes closest in meaning to it. Towering is clearly an antonym. Broad is close but does
not appropriately describe something which is ―squatty‖. Narrow is also nowhere close in meaning to
―squatty‖.
4. Ans:(b)
Solution:If one refers to the following sentence:
Over on the east side this ravager strode boldly, smiting his victims by scores, but his feet trod
slowly through the maze of the narrow and moss-grown "places."
It is clear that the east side had victims taken in scores; therefore the east side was worse affected
than the ―places‖ as the spread of Pneumonia was very slow there. Option (a) is clearly wrong in this
regard. Option (c) is the opposite of what is stated in the passage. Option (d) fails to provide any
useful information for arriving at the answer. Therefore option (b) is the most appropriate answer.
5. Ans:(b)
Solution:The expression refers to the symptoms caused by Pneumonia, which are shortness of
breath and red hands and feet. Options (a) and (c) are completely off topic and almost ridiculous.
Option (d) seems like a tempting choice but the author‘s purpose is to describe the symptoms of
Pneumonia.
6. Ans:(a)
Solution:If one refers to the following sentence, it is amply clear that both Joanna and Sue had
similar interests.Options (c) and (d) give the right answer but only a part of it. Option (b) is not
mentioned in the passage. Therefore option (a) is the best answer.
“They had met at the table d'hôte of an Eighth Street "Delmonico's," and found their tastes in art,
chicory salad and bishop sleeves so congenial that the joint studio resulted.”
7. Ans:(d)
Solution:If one refers to the following lines:
The station, a small shack backed by heavy jungle, was a station in name only; for trains only
stopped there, if at all, for a few seconds before entering the deep cutting that led to the
tunnel. Most trains merely slowed down before taking the sharp curve before cutting.
This implies that the station served as a point where trains could slow down or just stop before
entering the tunnel. Options (b) and (c) seem correct but nowhere in the passage does it say that
very few trains passed through the station. It refers to the station as a small shack but that is not the
reason for it being ―a station in name only‖. The actual reason is the stoppage of trains or rather, the
lack of it.
8. Ans:(a)
Solution:The overland mail refers to the night train that passes through the station. Options (b) and
(c) cannot be arrived at as it does not mention whether it was a goods train or a passenger train.
Option (d) is not mentioned in the passage and is merely to confuse the readers as the word used in
the phrase is ‗mail‘.
9. Ans:(c)
Solution:„Shall I come too, Father?‟ asked Tembu sleepily, still lying in a huddle in a corner of the
hut.
The passage implies that Tembu would usually accompany his father to the station and this could be
attributed to the fact that the station was three miles away and going there in the middle of the night,
through a forested area and without proper light would be dangerous to be done alone. This is why
he asks to accompany his father.
The other options are just to mislead the reader.
10. Ans:(a)
Solution:Although, all of the options are correct for the word cutting, in some context or another. But
in the passage, the word ―cutting‘ has a geographical connotation and is used to describe a path that
abruptly turns from its course into tunnel. Therefore option (a) is the best answer.
11. Ans:(d)
Solution:The expression ―sawing‖ clearly refers to the sound made by a panther and the only
possible answer is Onomatopoeia.
12. Ans:(d)
Solution:Option (d) is the best answer because both the usage of a manual signal and an oil lamp
would imply lack of electrification which could mean that the passage is set in a time before the
modern day.
13. Ans:(b)
Solution:Although all the options are correct in some way or another, skipping school best
describes the phrase ―playing the wag‖. The other phrases are incomplete in one sense or another.
Running away, pretending to be sick and bunking a class could be possible answers but skipping
school is the best one of the four.
14. Ans:(b)
Solution:In the context of the sentence “goes out fishing on a sunny afternoon, with a bit of
string tied on to the end of a tree” the author is referring to a fishing rod used to catch fish. Twigs,
leaves and roots would be impractical in case of a fishing rod and a branch would be the best item to
make a fishing rod out of. Therefore option (b) is the best answer.
15. Ans:(c)
Solution:Options (b) and (d) could be correct about the legendary and historical fish and definitely
seem so given how much they marvel at the fish but these are not supported by the passage very
well. It also cannot be concluded that every story before this one had been false, which rules out
option (a). Option (c) is the most appropriate one and fits into the passage neatly as it can be
inferred that they were listening to several stories about the fish and then someone came to tell them
the real story of how the fish was caught.
16. Ans:(a)
Solution:The word astonishing is used to describe how marvelous or stunning the trout was.
Therefore option (a) is the best answer. Options (b), (c) and (d) are different words that imply the
same thing: something which is ordinary, run of the mill.
17. Ans:(a)
Solution: The word ‗fishy‘ can best be described as a pun because fishy implies both
meanings: relating to or resembling a fish and arousing feelings of doubt or suspicion.
The other options do not fit the meaning of the title in any case as there is no comparison (which
rules out metaphor and simile) and there is no irony conveyed through the title as well.
Pun or paronomasia refers to a joke exploiting the different possible meanings of a word or the fact
that there are words which sound alike but have different meanings. Therefore the given title is a pun
or play on the word ‗fishy‘.
18. Ans:(b)
Solution:Option (b) is the best answer as the author uses the technique of humor and surprise at
the end of the story. The fish is one being marvelled at and legends are being spun as to how it was
caught. At the end it is discovered that the fish was made of Plaster of Paris and was an inanimate
object since the beginning. This creates humour and surprise and therefore option (b) is the best
answer.
19. Ans:(b)
Solution:Option (a) is an overstatement because the passage does not imply that it is impossible to
strike a balance between the right to protest and the right to free movement. Option (c) can be an
inference but does not depict the central idea of the passage at all. Option (d) is just a restatement of
a sentence written in the passage and is incorrect in the context of passage. It also does not convey
the central idea of the passage. The central idea is properly conveyed by option (b) that an organic
protest should not automatically be quashed by the strong arm of the state. In fact, the passage
justifies the reasons as to why protests should be allowed as they are a form of expressing dissent
which is essential for a healthy democracy.
20. Ans:(c)
Solution:All of the options are correct. They are listed in the following passage:
After the pandemic led to the end of the protests, there was little left for adjudication, and the Court‘s
remarks might come across as a gratuitous offering to administrators looking to de-legitimise
protests. Following the earlier judgment that any ‗bandh‘ is illegal, courts routinely stayed sector-
wide strikes. Another aspect of the present ruling is the assertion that protests should be confined to
―designated places‖. Such judicial certitude may end up undermining the larger democratic need for
public expression of dissent in a manner and place that would be most effective.
21. Ans:(d)
Solution:Judicial certitude implies legal certainty, that is, the conviction with which a court gives a
decision. It does not imply permanence (since it can be changed by manner of appeal, revision or
review), or the certificate given by a court or the precedence of a court‘s ruling.
22. Ans:(c)
Solution:The word salutary here means helpful or beneficial. The author states that the principles
given by the court in the ruling are beneficial from an administrative point of view but that should not
mean that they are unquestionable.
23. Ans:(d)
Solution:The answer is in the following sentence:
Protests, by their very nature, are not always rooted in legality, but rather derive legitimacy from the
rightness of the underlying cause and the extent of public support.
Options (b) and (c) are completely off topic. Option (a) seems tempting but protestors cannot be
considered to give legitimacy to a protest. Therefore option (d) is the best answer as it refers to the
―rightness of the underlying cause.‖
24. Ans:(a)
Solution:The best answer is news channels influenced by the government. ‗Loudspeakers‘ is the
literal meaning of the word megaphones. Daily soaps and public television are incorrect as well.
25. Ans:(b)
Solution:Options (a) and (c) are out of context as compared to the passage. Option (d) is ridiculous
and is merely a twisted restatement of a sentence given in the passage. Option (b) is the most
appropriate as the author states that after the Sushant Singh Rajput case, the fact of involvement of
politicians in films and Bollywood is finally being exposed.
26. Ans:(b)
Solution:Option (a) is clearly incorrect as the phrase does not refer to the literal arming of the state
agencies. Option (c) is far-fetched and goes beyond the scope of the passage. Option (d) could be
true but is too narrow in its scope and does not properly define the intent of the phrase. Option (b) is
the most appropriate because given the context of the paragraph; it refers to manipulation of State
agencies by the government so that only that part is shown to the public that the government wants.
27. Ans:(c)
Solution:The government wants to colonise the film industry but here the word colonise is not used
in the literal sense but is used in the context of establishing control over a particular place. Therefore
option (c) is the best answer.
28. Ans:(b)
Solution:Here, the keyword in the question is ‗why‘. The question does not ask what is meant by the
phrase “standing up by seeking to intimidate”. If one approaches the options with the sole
purpose of answering the question ‗why‘, the only option that answers it perfectly is option (b).
Option (a) is a twisted restatement of what is already stated in the passage. Option (c) goes beyond
the scope of the passage as politicians are not the ones making derogatory remarks. Option (d)
might be true in the context of the passage but is a bit of an overstatement as it uses the word ‗only‘.
Therefore option (b) is the best answer.
29. Ans:(c)
Solution:Option (c) is the best answer. Option (a) and (b) sound true but are not correct in the
context of the passage. Option (d) is also beyond the scope of the passage. The reason why they
did not let her believe in Santa Claus is that they were poor and could not afford expensive presents
from ‗Santa Claus‘ every year, like other rich kids.
30. Ans:(a)
Solution:The word ‗pulse‘ implies a kind of oscillation, vibration or throbbing.Therefore throb is the
best answer. Glow and glare seem correct in the context of the stars but here the author does not
use the word pulse to refer to their glowing and glaring, she wants to convey the message of
pulsating light which can be understood through ‗throb‘.
31. Ans:(c)
Solution:The central idea of the passage is to teach children that even though material things bring
momentary joy, this ey do not last. The non-durable nature of material things should be understood.
They will not last but the knowledge gained by a person will definitely go a long way. That is why the
father makes them identify stars and teaches them a few things along with it because toys will be
broken but these stars will last forever. Option (a) is far-fetched and does not convey the central idea
of the passage. Option (b) is also a confusing restatement of what is stated in the passage. Option
(d) is again far-fetched and definitely not the central idea of the passage. Option (c) is the best
answer.
32. Ans:(b)
Solution:His attitude can definitely be described as positive as he behaves maturely in the face of
adversity. Despite losing his job, he does not project his grief or trouble to his children and instead
teaches them the value of knowledge the fleeting happiness one derives from material things like
toys. Option (b) is the best answer.

GENERAL KNOWLEDGE/CURRENT AFFAIRS

33. Ans:(a)
Explanations: Chapare hemorrhagic fever (CHHF) is a viral hemorrhagic fever caused by infection
with Chapare virus. The Chapare virus is in the arenavirus family. Arenaviruses are usually spread
to people through direct contact with infected rodents or indirectly through the urine or feces
(droppings) of an infected rodent.
There have been two documented outbreaks of CHHF to date. The first occurred in 2003 in Chapare
Province, Bolivia, which resulted in one fatal case. The second outbreak occurred in 2019 in
Caranavi Province, Bolivia and resulted in five confirmed cases— three of which were fatal.
Though the rodent reservoir of Chapare virus is unknown, similar arenaviruses are typically
transmitted either through direct or indirect contact with the saliva, urine, and droppings of infected
rodents. Examples of direct contact include bites and scratches by infected rodents. Examples of
indirect contact include breathing in the virus when it is stirred into the air or ingestion of food
contaminated with the urine, salvia, or droppings of infected rodents.
An infected person can then spread the illness to other people through contact with the patient‘s
body fluids, or during procedures in healthcare settings that can aerosolize (spray particles of) the
infected person‘s body fluids—such as during chest compressions, CPR, and intubation. Because
there are very few documented cases of Chapare in humans, more research is needed to
understand how the virus spreads and causes illness.
34. Ans:(c)
Explanations: Chapare hemorrhagic fever (CHHF) is a viral hemorrhagic fever caused by infection
with Chapare virus. The Chapare virus is in the arenavirus family. Arenaviruses are usually spread
to people through direct contact with infected rodents or indirectly through the urine or feces
(droppings) of an infected rodent.
There have been two documented outbreaks of CHHF to date. The first occurred in 2003 in Chapare
Province, Bolivia, which resulted in one fatal case. The second outbreak occurred in 2019 in
Caranavi Province, Bolivia and resulted in five confirmed cases— three of which were fatal.
Though the rodent reservoir of Chapare virus is unknown, similar arenaviruses are typically
transmitted either through direct or indirect contact with the saliva, urine, and droppings of infected
rodents. Examples of direct contact include bites and scratches by infected rodents. Examples of
indirect contact include breathing in the virus when it is stirred into the air or ingestion of food
contaminated with the urine, salvia, or droppings of infected rodents.
An infected person can then spread the illness to other people through contact with the patient‘s
body fluids, or during procedures in healthcare settings that can aerosolize (spray particles of) the
infected person‘s body fluids—such as during chest compressions, CPR, and intubation. Because
there are very few documented cases of Chapare in humans, more research is needed to
understand how the virus spreads and causes illness.
35. Ans:(b)
Explanations: Chapare hemorrhagic fever (CHHF) is a viral hemorrhagic fever caused by infection
with Chapare virus. The Chapare virus is in the arenavirus family. Arenaviruses are usually spread
to people through direct contact with infected rodents or indirectly through the urine or feces
(droppings) of an infected rodent.
There have been two documented outbreaks of CHHF to date. The first occurred in 2003 in Chapare
Province, Bolivia, which resulted in one fatal case. The second outbreak occurred in 2019 in
Caranavi Province, Bolivia and resulted in five confirmed cases— three of which were fatal.
Though the rodent reservoir of Chapare virus is unknown, similar arenaviruses are typically
transmitted either through direct or indirect contact with the saliva, urine, and droppings of infected
rodents. Examples of direct contact include bites and scratches by infected rodents. Examples of
indirect contact include breathing in the virus when it is stirred into the air or ingestion of food
contaminated with the urine, salvia, or droppings of infected rodents.
An infected person can then spread the illness to other people through contact with the patient‘s
body fluids, or during procedures in healthcare settings that can aerosolize (spray particles of) the
infected person‘s body fluids—such as during chest compressions, CPR, and intubation. Because
there are very few documented cases of Chapare in humans, more research is needed to
understand how the virus spreads and causes illness.
36. Ans:(d)
Explanations: Chapare hemorrhagic fever (CHHF) is a viral hemorrhagic fever caused by infection
with Chapare virus. The Chapare virus is in the arenavirus family. Arenaviruses are usually spread
to people through direct contact with infected rodents or indirectly through the urine or feces
(droppings) of an infected rodent.
There have been two documented outbreaks of CHHF to date. The first occurred in 2003 in Chapare
Province, Bolivia, which resulted in one fatal case. The second outbreak occurred in 2019 in
Caranavi Province, Bolivia and resulted in five confirmed cases— three of which were fatal.
Though the rodent reservoir of Chapare virus is unknown, similar arenaviruses are typically
transmitted either through direct or indirect contact with the saliva, urine, and droppings of infected
rodents. Examples of direct contact include bites and scratches by infected rodents. Examples of
indirect contact include breathing in the virus when it is stirred into the air or ingestion of food
contaminated with the urine, salvia, or droppings of infected rodents.
An infected person can then spread the illness to other people through contact with the patient‘s
body fluids, or during procedures in healthcare settings that can aerosolize (spray particles of) the
infected person‘s body fluids—such as during chest compressions, CPR, and intubation. Because
there are very few documented cases of Chapare in humans, more research is needed to
understand how the virus spreads and causes illness.
37. Ans:(b)
Explanations: Chapare hemorrhagic fever (CHHF) is a viral hemorrhagic fever caused by infection
with Chapare virus. The Chapare virus is in the arenavirus family. Arenaviruses are usually spread
to people through direct contact with infected rodents or indirectly through the urine or feces
(droppings) of an infected rodent.
There have been two documented outbreaks of CHHF to date. The first occurred in 2003 in Chapare
Province, Bolivia, which resulted in one fatal case. The second outbreak occurred in 2019 in
Caranavi Province, Bolivia and resulted in five confirmed cases— three of which were fatal.
Though the rodent reservoir of Chapare virus is unknown, similar arenaviruses are typically
transmitted either through direct or indirect contact with the saliva, urine, and droppings of infected
rodents. Examples of direct contact include bites and scratches by infected rodents. Examples of
indirect contact include breathing in the virus when it is stirred into the air or ingestion of food
contaminated with the urine, salvia, or droppings of infected rodents.
An infected person can then spread the illness to other people through contact with the patient‘s
body fluids, or during procedures in healthcare settings that can aerosolize (spray particles of) the
infected person‘s body fluids—such as during chest compressions, CPR, and intubation. Because
there are very few documented cases of Chapare in humans, more research is needed to
understand how the virus spreads and causes illness.
38. Ans:(a)
Explanations: The Kaleshwaram Lift Irrigation Project (KLIP) is a multi-purpose irrigation project on
the Godavari River in Kaleshwaram, Bhoopalpally, Telangana, India.Currently the world's largest
multi-stage lift irrigation project, its farthest upstream influence is at the confluence of the Pranhita
and Godavari rivers. The Pranahita River is itself a confluence of various smaller tributaries including
the Wardha, Painganga, and Wainganga rivers which combine to form the seventh-largest drainage
basin on the subcontinent, with an estimated annual discharge of more than 6,427,900 acre feet
(7,930 cubic hectometres) or 280 TMC. It remains untapped as its course is principally through
dense forests and other ecologically sensitive zones such as wildlife sanctuaries.
The Kaleshwaram Lift Irrigation Project is divided into 7 links and 28 packages spanning a distance
of approximately 500 km (310 mi) through 13 districts and utilizing a canal network of more than
1,800 km (1,100 mi). The project aims to produce a total of 240 TMC (195 from Medigadda Barrage,
20 from Sripada Yellampalli project and 25 from groundwater), of which 169 has been allocated for
irrigation, 30 for Hyderabad municipal water, 16 for miscellaneous industrial uses and 10 for drinking
water in nearby villages, with the remainder being estimated evaporation loss. The project aims at
increasing total culturable command area (the sustainable area which can be irrigated after
accounting for both upstream and downstream factors) by 1,825,000 acre⋅ft (2,251 hm3) across all
13 districts in addition to stabilizing the existing CCA.
On 21 June 2019, the project was opened by Telangana Governor E. S. L. Narasimhan and Chief
minister K. Chandrashekar Rao. National Green Tribunal declared the Scheme is constructed
without following the statuary provisions with regard to environmental aspects.
39. Ans:(d)
Explanations: The Kaleshwaram Lift Irrigation Project (KLIP) is a multi-purpose irrigation project on
the Godavari River in Kaleshwaram, Bhoopalpally, Telangana, India.Currently the world's largest
multi-stage lift irrigation project, its farthest upstream influence is at the confluence of the Pranhita
and Godavari rivers. The Pranahita River is itself a confluence of various smaller tributaries including
the Wardha, Painganga, and Wainganga rivers which combine to form the seventh-largest drainage
basin on the subcontinent, with an estimated annual discharge of more than 6,427,900 acre feet
(7,930 cubic hectometres) or 280 TMC. It remains untapped as its course is principally through
dense forests and other ecologically sensitive zones such as wildlife sanctuaries.
The Kaleshwaram Lift Irrigation Project is divided into 7 links and 28 packages spanning a distance
of approximately 500 km (310 mi) through 13 districts and utilizing a canal network of more than
1,800 km (1,100 mi). The project aims to produce a total of 240 TMC (195 from Medigadda Barrage,
20 from Sripada Yellampalli project and 25 from groundwater), of which 169 has been allocated for
irrigation, 30 for Hyderabad municipal water, 16 for miscellaneous industrial uses and 10 for drinking
water in nearby villages, with the remainder being estimated evaporation loss. The project aims at
increasing total culturable command area (the sustainable area which can be irrigated after
accounting for both upstream and downstream factors) by 1,825,000 acre⋅ft (2,251 hm3) across all
13 districts in addition to stabilizing the existing CCA.
On 21 June 2019, the project was opened by Telangana Governor E. S. L. Narasimhan and Chief
minister K. Chandrashekar Rao. National Green Tribunal declared the Scheme is constructed
without following the statuary provisions with regard to environmental aspects.
40. Ans:(b)
Explanations: The Kaleshwaram Lift Irrigation Project (KLIP) is a multi-purpose irrigation project on
the Godavari River in Kaleshwaram, Bhoopalpally, Telangana, India.Currently the world's largest
multi-stage lift irrigation project, its farthest upstream influence is at the confluence of the Pranhita
and Godavari rivers. The Pranahita River is itself a confluence of various smaller tributaries including
the Wardha, Painganga, and Wainganga rivers which combine to form the seventh-largest drainage
basin on the subcontinent, with an estimated annual discharge of more than 6,427,900 acre feet
(7,930 cubic hectometres) or 280 TMC. It remains untapped as its course is principally through
dense forests and other ecologically sensitive zones such as wildlife sanctuaries.
The Kaleshwaram Lift Irrigation Project is divided into 7 links and 28 packages spanning a distance
of approximately 500 km (310 mi) through 13 districts and utilizing a canal network of more than
1,800 km (1,100 mi). The project aims to produce a total of 240 TMC (195 from Medigadda Barrage,
20 from Sripada Yellampalli project and 25 from groundwater), of which 169 has been allocated for
irrigation, 30 for Hyderabad municipal water, 16 for miscellaneous industrial uses and 10 for drinking
water in nearby villages, with the remainder being estimated evaporation loss. The project aims at
increasing total culturable command area (the sustainable area which can be irrigated after
accounting for both upstream and downstream factors) by 1,825,000 acre⋅ft (2,251 hm3) across all
13 districts in addition to stabilizing the existing CCA.
On 21 June 2019, the project was opened by Telangana Governor E. S. L. Narasimhan and Chief
minister K. Chandrashekar Rao. National Green Tribunal declared the Scheme is constructed
without following the statuary provisions with regard to environmental aspects.
41. Ans:(c)
Explanations: The Kaleshwaram Lift Irrigation Project (KLIP) is a multi-purpose irrigation project on
the Godavari River in Kaleshwaram, Bhoopalpally, Telangana, India.Currently the world's largest
multi-stage lift irrigation project, its farthest upstream influence is at the confluence of the Pranhita
and Godavari rivers. The Pranahita River is itself a confluence of various smaller tributaries including
the Wardha, Painganga, and Wainganga rivers which combine to form the seventh-largest drainage
basin on the subcontinent, with an estimated annual discharge of more than 6,427,900 acre feet
(7,930 cubic hectometres) or 280 TMC. It remains untapped as its course is principally through
dense forests and other ecologically sensitive zones such as wildlife sanctuaries.
The Kaleshwaram Lift Irrigation Project is divided into 7 links and 28 packages spanning a distance
of approximately 500 km (310 mi) through 13 districts and utilizing a canal network of more than
1,800 km (1,100 mi). The project aims to produce a total of 240 TMC (195 from Medigadda Barrage,
20 from Sripada Yellampalli project and 25 from groundwater), of which 169 has been allocated for
irrigation, 30 for Hyderabad municipal water, 16 for miscellaneous industrial uses and 10 for drinking
water in nearby villages, with the remainder being estimated evaporation loss. The project aims at
increasing total culturable command area (the sustainable area which can be irrigated after
accounting for both upstream and downstream factors) by 1,825,000 acre⋅ft (2,251 hm3) across all
13 districts in addition to stabilizing the existing CCA.
On 21 June 2019, the project was opened by Telangana Governor E. S. L. Narasimhan and Chief
minister K. Chandrashekar Rao. National Green Tribunal declared the Scheme is constructed
without following the statuary provisions with regard to environmental aspects.
42. Ans:(d)
Explanations: The Kaleshwaram Lift Irrigation Project (KLIP) is a multi-purpose irrigation project on
the Godavari River in Kaleshwaram, Bhoopalpally, Telangana, India.Currently the world's largest
multi-stage lift irrigation project, its farthest upstream influence is at the confluence of the Pranhita
and Godavari rivers. The Pranahita River is itself a confluence of various smaller tributaries including
the Wardha, Painganga, and Wainganga rivers which combine to form the seventh-largest drainage
basin on the subcontinent, with an estimated annual discharge of more than 6,427,900 acre feet
(7,930 cubic hectometres) or 280 TMC. It remains untapped as its course is principally through
dense forests and other ecologically sensitive zones such as wildlife sanctuaries.
The Kaleshwaram Lift Irrigation Project is divided into 7 links and 28 packages spanning a distance
of approximately 500 km (310 mi) through 13 districts and utilizing a canal network of more than
1,800 km (1,100 mi). The project aims to produce a total of 240 TMC (195 from Medigadda Barrage,
20 from Sripada Yellampalli project and 25 from groundwater), of which 169 has been allocated for
irrigation, 30 for Hyderabad municipal water, 16 for miscellaneous industrial uses and 10 for drinking
water in nearby villages, with the remainder being estimated evaporation loss. The project aims at
increasing total culturable command area (the sustainable area which can be irrigated after
accounting for both upstream and downstream factors) by 1,825,000 acre⋅ft (2,251 hm3) across all
13 districts in addition to stabilizing the existing CCA.
On 21 June 2019, the project was opened by Telangana Governor E. S. L. Narasimhan and Chief
minister K. Chandrashekar Rao. National Green Tribunal declared the Scheme is constructed
without following the statuary provisions with regard to environmental aspects.
43. Ans:(d)
Explanations: China has begun work on a strategically significant railway line that will link Sichuan
province with Nyingchi in Tibet, which lies close to Arunachal Pradesh border.
This will be the second such route linking Tibet Autonomous Region (TAR) with mainland China.
oEarlier Qinghai-Tibet railway line connected Lhasa to the hinterland.
Implications on India:
Security Concerns:
•The railway line will largely improve the efficiency and convenience of military personnel and
material transportation and logistical supplies in the border area.
•In situations of direct standoff near Arunachal Pradesh border, as was seen during Doklam or
recent Ladakh standoff, China might be at an advantageous position.
Doklam Issue: Indian troops intervened to block the path of Chinese soldiers engaged in building
road-works on the Doklam plateau of Bhutan‘s territory that Beijing laid claim.
Ecological Concerns:
•The fragile ecological environment along the project line, may have ecological concerns for India.
Recent Steps Taken by India:
oIndia will spend 10% funds of the Border Area Development Programme (BADP) only to improve
the infrastructure along the China border.
The Border Roads Organisation (BRO) constructed the Daporijo bridge over Subansiri river in
Arunachal Pradesh in a record span of just 27 days.
•It links roads leading upto the Line of Actual Control (LAC) between India and China.
Recently the defence minister virtually laid the foundation of a tunnel at Nechiphu in West Kameng
district of Arunachal Pradesh.
•It will shorten travel time for troops till the LAC through Tawang, which China claims to be its
territory.
The BRO is already constructing an all-weather tunnel under the Se La pass in Arunachal Pradesh
which connects Tawang to the rest of Arunachal and Guwahati.
The state government of Arunachal Pradesh has advocated selection of 10 census towns along the
India-China border as pilot projects for infrastructure development in order to stop people living
along its international borders, specifically with China, from migrating to faraway urban centres in the
State.
oRecently, the Defence Minister inaugurated the Sisseri River Bridge located at lower Dibang Valley
in Arunachal Pradesh connecting Dibang Valley and Siang.
In 2019, the Indian Air Force inaugurated resurfaced runway at India‘s easternmost Village-
Vijaynagar (Changlang district) in Arunachal Pradesh.
In 2019, the Indian Army conducted exercise ‗HimVijay‘ in Arunachal Pradesh and Assam with its
newly created Integrated Battle Groups (IBG).
Bogibeel bridge, which is India‘s longest road-rail bridge connecting Dibrugarh in Assam to Pasighat
in Arunachal Pradesh was inaugurated in 2018.
•It will facilitate quicker movement of troops and equipment to areas near the India-China border.
44. Ans:(a)
Explanations: China has begun work on a strategically significant railway line that will link Sichuan
province with Nyingchi in Tibet, which lies close to Arunachal Pradesh border.
This will be the second such route linking Tibet Autonomous Region (TAR) with mainland China.
oEarlier Qinghai-Tibet railway line connected Lhasa to the hinterland.
Implications on India:
Security Concerns:
•The railway line will largely improve the efficiency and convenience of military personnel and
material transportation and logistical supplies in the border area.
•In situations of direct standoff near Arunachal Pradesh border, as was seen during Doklam or
recent Ladakh standoff, China might be at an advantageous position.
Doklam Issue: Indian troops intervened to block the path of Chinese soldiers engaged in building
road-works on the Doklam plateau of Bhutan‘s territory that Beijing laid claim.
Ecological Concerns:
•The fragile ecological environment along the project line, may have ecological concerns for India.
Recent Steps Taken by India:
oIndia will spend 10% funds of the Border Area Development Programme (BADP) only to improve
the infrastructure along the China border.
The Border Roads Organisation (BRO) constructed the Daporijo bridge over Subansiri river in
Arunachal Pradesh in a record span of just 27 days.
•It links roads leading upto the Line of Actual Control (LAC) between India and China.
Recently the defence minister virtually laid the foundation of a tunnel at Nechiphu in West Kameng
district of Arunachal Pradesh.
•It will shorten travel time for troops till the LAC through Tawang, which China claims to be its
territory.
The BRO is already constructing an all-weather tunnel under the Se La pass in Arunachal Pradesh
which connects Tawang to the rest of Arunachal and Guwahati.
The state government of Arunachal Pradesh has advocated selection of 10 census towns along the
India-China border as pilot projects for infrastructure development in order to stop people living
along its international borders, specifically with China, from migrating to faraway urban centres in the
State.
oRecently, the Defence Minister inaugurated the Sisseri River Bridge located at lower Dibang Valley
in Arunachal Pradesh connecting Dibang Valley and Siang.
In 2019, the Indian Air Force inaugurated resurfaced runway at India‘s easternmost Village-
Vijaynagar (Changlang district) in Arunachal Pradesh.
In 2019, the Indian Army conducted exercise ‗HimVijay‘ in Arunachal Pradesh and Assam with its
newly created Integrated Battle Groups (IBG).
Bogibeel bridge, which is India‘s longest road-rail bridge connecting Dibrugarh in Assam to Pasighat
in Arunachal Pradesh was inaugurated in 2018.
•It will facilitate quicker movement of troops and equipment to areas near the India-China border.
45. Ans:(b)
Explanations: China has begun work on a strategically significant railway line that will link Sichuan
province with Nyingchi in Tibet, which lies close to Arunachal Pradesh border.
This will be the second such route linking Tibet Autonomous Region (TAR) with mainland China.
oEarlier Qinghai-Tibet railway line connected Lhasa to the hinterland.
Implications on India:
Security Concerns:
•The railway line will largely improve the efficiency and convenience of military personnel and
material transportation and logistical supplies in the border area.
•In situations of direct standoff near Arunachal Pradesh border, as was seen during Doklam or
recent Ladakh standoff, China might be at an advantageous position.
Doklam Issue: Indian troops intervened to block the path of Chinese soldiers engaged in building
road-works on the Doklam plateau of Bhutan‘s territory that Beijing laid claim.
Ecological Concerns:
•The fragile ecological environment along the project line, may have ecological concerns for India.
Recent Steps Taken by India:
oIndia will spend 10% funds of the Border Area Development Programme (BADP) only to improve
the infrastructure along the China border.
The Border Roads Organisation (BRO) constructed the Daporijo bridge over Subansiri river in
Arunachal Pradesh in a record span of just 27 days.
•It links roads leading upto the Line of Actual Control (LAC) between India and China.
Recently the defence minister virtually laid the foundation of a tunnel at Nechiphu in West Kameng
district of Arunachal Pradesh.
•It will shorten travel time for troops till the LAC through Tawang, which China claims to be its
territory.
The BRO is already constructing an all-weather tunnel under the Se La pass in Arunachal Pradesh
which connects Tawang to the rest of Arunachal and Guwahati.
The state government of Arunachal Pradesh has advocated selection of 10 census towns along the
India-China border as pilot projects for infrastructure development in order to stop people living
along its international borders, specifically with China, from migrating to faraway urban centres in the
State.
oRecently, the Defence Minister inaugurated the Sisseri River Bridge located at lower Dibang Valley
in Arunachal Pradesh connecting Dibang Valley and Siang.
In 2019, the Indian Air Force inaugurated resurfaced runway at India‘s easternmost Village-
Vijaynagar (Changlang district) in Arunachal Pradesh.
In 2019, the Indian Army conducted exercise ‗HimVijay‘ in Arunachal Pradesh and Assam with its
newly created Integrated Battle Groups (IBG).
Bogibeel bridge, which is India‘s longest road-rail bridge connecting Dibrugarh in Assam to Pasighat
in Arunachal Pradesh was inaugurated in 2018.
•It will facilitate quicker movement of troops and equipment to areas near the India-China border.
46. Ans:(c)
Explanations: China has begun work on a strategically significant railway line that will link Sichuan
province with Nyingchi in Tibet, which lies close to Arunachal Pradesh border.
This will be the second such route linking Tibet Autonomous Region (TAR) with mainland China.
oEarlier Qinghai-Tibet railway line connected Lhasa to the hinterland.
Implications on India:
Security Concerns:
•The railway line will largely improve the efficiency and convenience of military personnel and
material transportation and logistical supplies in the border area.
•In situations of direct standoff near Arunachal Pradesh border, as was seen during Doklam or
recent Ladakh standoff, China might be at an advantageous position.
Doklam Issue: Indian troops intervened to block the path of Chinese soldiers engaged in building
road-works on the Doklam plateau of Bhutan‘s territory that Beijing laid claim.
Ecological Concerns:
•The fragile ecological environment along the project line, may have ecological concerns for India.
Recent Steps Taken by India:
oIndia will spend 10% funds of the Border Area Development Programme (BADP) only to improve
the infrastructure along the China border.
The Border Roads Organisation (BRO) constructed the Daporijo bridge over Subansiri river in
Arunachal Pradesh in a record span of just 27 days.
•It links roads leading upto the Line of Actual Control (LAC) between India and China.
Recently the defence minister virtually laid the foundation of a tunnel at Nechiphu in West Kameng
district of Arunachal Pradesh.
•It will shorten travel time for troops till the LAC through Tawang, which China claims to be its
territory.
The BRO is already constructing an all-weather tunnel under the Se La pass in Arunachal Pradesh
which connects Tawang to the rest of Arunachal and Guwahati.
The state government of Arunachal Pradesh has advocated selection of 10 census towns along the
India-China border as pilot projects for infrastructure development in order to stop people living
along its international borders, specifically with China, from migrating to faraway urban centres in the
State.
oRecently, the Defence Minister inaugurated the Sisseri River Bridge located at lower Dibang Valley
in Arunachal Pradesh connecting Dibang Valley and Siang.
In 2019, the Indian Air Force inaugurated resurfaced runway at India‘s easternmost Village-
Vijaynagar (Changlang district) in Arunachal Pradesh.
In 2019, the Indian Army conducted exercise ‗HimVijay‘ in Arunachal Pradesh and Assam with its
newly created Integrated Battle Groups (IBG).
Bogibeel bridge, which is India‘s longest road-rail bridge connecting Dibrugarh in Assam to Pasighat
in Arunachal Pradesh was inaugurated in 2018.
•It will facilitate quicker movement of troops and equipment to areas near the India-China border.
47. Ans:(c)
Explanations: China has begun work on a strategically significant railway line that will link Sichuan
province with Nyingchi in Tibet, which lies close to Arunachal Pradesh border.
This will be the second such route linking Tibet Autonomous Region (TAR) with mainland China.
oEarlier Qinghai-Tibet railway line connected Lhasa to the hinterland.
Implications on India:
Security Concerns:
•The railway line will largely improve the efficiency and convenience of military personnel and
material transportation and logistical supplies in the border area.
•In situations of direct standoff near Arunachal Pradesh border, as was seen during Doklam or
recent Ladakh standoff, China might be at an advantageous position.
Doklam Issue: Indian troops intervened to block the path of Chinese soldiers engaged in building
road-works on the Doklam plateau of Bhutan‘s territory that Beijing laid claim.
Ecological Concerns:
•The fragile ecological environment along the project line, may have ecological concerns for India.
Recent Steps Taken by India:
oIndia will spend 10% funds of the Border Area Development Programme (BADP) only to improve
the infrastructure along the China border.
The Border Roads Organisation (BRO) constructed the Daporijo bridge over Subansiri river in
Arunachal Pradesh in a record span of just 27 days.
•It links roads leading upto the Line of Actual Control (LAC) between India and China.
Recently the defence minister virtually laid the foundation of a tunnel at Nechiphu in West Kameng
district of Arunachal Pradesh.
•It will shorten travel time for troops till the LAC through Tawang, which China claims to be its
territory.
The BRO is already constructing an all-weather tunnel under the Se La pass in Arunachal Pradesh
which connects Tawang to the rest of Arunachal and Guwahati.
The state government of Arunachal Pradesh has advocated selection of 10 census towns along the
India-China border as pilot projects for infrastructure development in order to stop people living
along its international borders, specifically with China, from migrating to faraway urban centres in the
State.
oRecently, the Defence Minister inaugurated the Sisseri River Bridge located at lower Dibang Valley
in Arunachal Pradesh connecting Dibang Valley and Siang.
In 2019, the Indian Air Force inaugurated resurfaced runway at India‘s easternmost Village-
Vijaynagar (Changlang district) in Arunachal Pradesh.
In 2019, the Indian Army conducted exercise ‗HimVijay‘ in Arunachal Pradesh and Assam with its
newly created Integrated Battle Groups (IBG).
Bogibeel bridge, which is India‘s longest road-rail bridge connecting Dibrugarh in Assam to Pasighat
in Arunachal Pradesh was inaugurated in 2018.
•It will facilitate quicker movement of troops and equipment to areas near the India-China border.
48. Ans:(b)
Explanations: 20th Summit of SCO Council of Heads of State was held on November 10, 2020 (in
Video Conference Format). The Meeting was chaired by the President of the Russian Federation Mr.
Vladimir Putin. Prime Minister Shri Narendra Modi led the Indian delegation. Other SCO Member
States were represented by their Presidents, while India and Pakistan were represented at the level
of Prime Minister. Other participants were: Secretary General of the SCO Secretariat, Executive
Director of the SCO Regional Anti-Terrorist Structure, the Presidents of the four Observers
(Afghanistan, Belarus, Iran, Mongolia) of the SCO.
Prime Minister congratulated President Emomali Rahmon of the Republic of Tajikistan for assuming
the chairmanship of SCO next year and assured full cooperation from India.
49. Ans:(d)
Explanations: 20th Summit of SCO Council of Heads of State was held on November 10, 2020 (in
Video Conference Format). The Meeting was chaired by the President of the Russian Federation Mr.
Vladimir Putin. Prime Minister Shri Narendra Modi led the Indian delegation. Other SCO Member
States were represented by their Presidents, while India and Pakistan were represented at the level
of Prime Minister. Other participants were: Secretary General of the SCO Secretariat, Executive
Director of the SCO Regional Anti-Terrorist Structure, the Presidents of the four Observers
(Afghanistan, Belarus, Iran, Mongolia) of the SCO.
Prime Minister congratulated President Emomali Rahmon of the Republic of Tajikistan for assuming
the chairmanship of SCO next year and assured full cooperation from India.
50. Ans:(c)
Explanations: 20th Summit of SCO Council of Heads of State was held on November 10, 2020 (in
Video Conference Format). The Meeting was chaired by the President of the Russian Federation Mr.
Vladimir Putin. Prime Minister Shri Narendra Modi led the Indian delegation. Other SCO Member
States were represented by their Presidents, while India and Pakistan were represented at the level
of Prime Minister. Other participants were: Secretary General of the SCO Secretariat, Executive
Director of the SCO Regional Anti-Terrorist Structure, the Presidents of the four Observers
(Afghanistan, Belarus, Iran, Mongolia) of the SCO.
Prime Minister congratulated President Emomali Rahmon of the Republic of Tajikistan for assuming
the chairmanship of SCO next year and assured full cooperation from India.
51. Ans:(a)
Explanations: 20th Summit of SCO Council of Heads of State was held on November 10, 2020 (in
Video Conference Format). The Meeting was chaired by the President of the Russian Federation Mr.
Vladimir Putin. Prime Minister Shri Narendra Modi led the Indian delegation. Other SCO Member
States were represented by their Presidents, while India and Pakistan were represented at the level
of Prime Minister. Other participants were: Secretary General of the SCO Secretariat, Executive
Director of the SCO Regional Anti-Terrorist Structure, the Presidents of the four Observers
(Afghanistan, Belarus, Iran, Mongolia) of the SCO.
Prime Minister congratulated President Emomali Rahmon of the Republic of Tajikistan for assuming
the chairmanship of SCO next year and assured full cooperation from India.
52. Ans:(a)
Explanations: 20th Summit of SCO Council of Heads of State was held on November 10, 2020 (in
Video Conference Format). The Meeting was chaired by the President of the Russian Federation Mr.
Vladimir Putin. Prime Minister Shri Narendra Modi led the Indian delegation. Other SCO Member
States were represented by their Presidents, while India and Pakistan were represented at the level
of Prime Minister. Other participants were: Secretary General of the SCO Secretariat, Executive
Director of the SCO Regional Anti-Terrorist Structure, the Presidents of the four Observers
(Afghanistan, Belarus, Iran, Mongolia) of the SCO.
Prime Minister congratulated President Emomali Rahmon of the Republic of Tajikistan for assuming
the chairmanship of SCO next year and assured full cooperation from India.
53. Ans:(c)
54. Ans:(d)
Solution: On 26 January 2015, the U.S. President and Indian Prime Minister agreed, in a joint
statement, to upgrade exercise Malabar. India invited Japan to be a part of exercise, held in the Bay
of Bengal. Since 2007, India has only hosted Exercise Malabar when the US and India are the two
participating parties. Malabar 2009, 2011, and 2014 all took place off the coast of Japan and
included the Japanese Maritime Self-Defense Forces. The exercise was undertaken by the three
countries in the Bay of Bengal from 15 October 2015 till 19 October 2015.
55. Ans:(a)
Solution: The 2020 Exercise is expected to be held in the Bay of Bengal and the Arabian Sea. In
2019, the exercise was conducted off the coast of Japan.
Due to Covid-19 pandemic the exercise had been planned in a ‗non-contact - at sea‘ format.
Its objective is to enhance safety and security in the maritime domain.
56. Ans:(b)
Solution: It began as a bilateral naval exercise between India and the USA in 1992 and was
expanded into a trilateral format with the inclusion of Japan in 2015.
The Exercise is aimed to support free, open and inclusive Indo-Pacific and remain committed to a
rules based international order.
It is also aimed at interoperability with an emphasis on humanitarian assistance, surface war
manoeuvres, anti-submarines warfare, counter-terror operations, gunnery training and aerial
surveillance.
57. Ans:(c)
Solution: Australia and Singapore had been part of the expanded Malabar exercises along the
coast of Japan in April 2007. China had strongly objected to the exercise, which led both Australia
and Singapore to keep away. The Malabar exercises then continued to be bilateral between India
and the US, till Japan joined from 2015.
58. Ans:(c)
Solution: It is an informal strategic forum between the United States, Japan, Australia and India.
The idea of grouping was first conceptualised by the Japanese Prime Minister Shinzo Abe in 2007.
It has a shared objective of ensuring a free, open and prosperous Indo-Pacific region.
59. Ans:(a)
Solution: The PBR is the third in Madhya Pradesh to be included in the list after Pachmarhi and
Amarkantak.
Along with PBR, the Fuvahmulahand Addu Atoll in the Maldives has also been included in the
WNBR.1994: The Panna National Park got the status of Project Tiger Reserve as India‘s 22nd tiger
reserve.
2011: It was notified as a Biosphere Reserve by the Union Ministry of Environment, Forest and
Climate Change (MoEFCC).
2018: By 2018, it witnessed a remarkable turnaround in tiger population by increasing their numbers
remarkably from zero estimated a decade ago.
60. Ans:(b)
Solution: Panna - City Of Diamonds. The only Diamond City in India is Panna. Panna is beautifully
calm and serene : roll-on meadows dotted with evergreen trees, rocks, hills, forests Panna is famous
for its temples which strikes a very fine blend of Hindu and Muslim architecture.
61. Ans:(c)
Solution:The idea of the biosphere reserve was initiated by UNESCO in 1974 under the MAB with
the objective of obtaining international cooperation for the conservation of the biospheres.
The first biosphere reserve of the world was established in 1979 and since then the network has
increased to more than 600 in 119 countries across the world.
A scheme called Biosphere Reserve has been implemented by the Government of India since 1986
62. Ans:(c)
Solution:Launched by the UNESCO in 1971, it is an intergovernmental scientific programme that
aims to establish a scientific basis for the improvement of relationships between people and their
environments.
Every year UNESCO designates new Biosphere reserves and removes others to promote the
conservation of biodiversity and resolve man-animal conflict at that site and enable sustainable use
of natural resources.
MAB combines the natural and social sciences, economics and education to improve human
livelihoods and the equitable sharing of benefits, and to safeguard natural and managed
ecosystems, thus promoting innovative approaches to economic development that are socially and
culturally appropriate, and environmentally sustainable.
63. Ans:(b)
Solution: The Nilgiri Biosphere Reserve was the first biosphere reserve in India established in the
year 1986. It is located in the Western Ghats and includes 2 of the 10 biogeographical provinces of
India. Wide ranges of ecosystems and species diversity are found in this region. Thus, it was a
natural choice for the premier biosphere reserve of the country.
The Nilgiri Biosphere Reserve was established mainly to fulfill the following objectives:
To conserve insitu genetic diversity of species
To restore degraded ecosystems to their natural conditions
To provide baseline data for ecological and environmental research and education
To function as an alternate model for sustainable development
64. Ans:(d)
Solution: Chandrayaan-1, India's first mission to Moon, was launched successfully on October 22,
2008 from SDSC SHAR, Sriharikota. The spacecraft was orbiting around the Moon at a height of
100 km from the lunar surface for chemical, mineralogical and photo-geologic mapping of the Moon.
The spacecraft carried 11 scientific instruments built in India, USA, UK, Germany, Sweden and
Bulgaria.
After the successful completion of all the major mission objectives, the orbit has been raised to 200
km during May 2009. The satellite made more than 3400 orbits around the moon and the mission
was concluded when the communication with the spacecraft was lost on August 29, 2009.
65. Ans:(b)
Solution: The Artemis program is a US government-funded human spaceflight program that has the
goal of landing "the first woman and the next man" on the Moon, specifically at the lunar south pole
region by 2024. The program is carried out predominantly by NASA, U.S. commercial spaceflight
companies contracted by NASA, and international partners including the European Space Agency
(ESA), the Japan Aerospace Exploration Agency (JAXA), Canadian Space Agency (CSA), the Italian
Space Agency (ASI) the Australian Space Agency (ASA), the UK Space Agency (UKSA) and the
United Arab Emirates Space Agency (UAESA)
66. Ans:(a)
Solution: Data from the Mineralogy Mapper (M3), one of the instruments on Chandrayaan-1,
indicates the presence of hematite at the lunar poles.
Hematite (Fe2O3) is a mineral which is a form of iron oxide, or rust, produced when iron is exposed
to oxygen and water.
The sign of this finding is that even though the surface of the moon is known to have iron-rich rocks,
it is not known for the presence of water and oxygen, which are the two elements needed to interact
with iron to create rust.
Recently, the National Aeronautics and Space Administration (NASA) has found evidence of greater
quantities of metals such as iron and titanium on the moon‘s subsurface.
67. Ans:(c)
Solution: The spacecraft carried 11 scientific instruments built in India, USA, UK, Germany, Sweden
and Bulgaria. Eleven scientific instruments onboard Chandrayaan-1 spacecraft included 5 which
were are Indian and other 6 are from ESA (3), NASA (2), and Bulgarian Academy of Sciences (1).
Also, 2 of the ESA instruments have Indian collaboration.
68. Ans:(c)
Solution: the Global Innovation Index 2020 was released by the World Intellectual Property
Organisation (WIPO).India is at the 48th position in the list of top 50 innovative countries.
India occupied the 52nd position in 2019 rankings. India is one of the leading innovation achievers in
the central and southern Asian region, as it has shown a consistent improvement in its innovation
ranking for the last 5 years.
69. Ans:(a)
Solution: A key question is how the economic fallout from the COVID-19 crisis will impact start-ups,
venture capital, and other traditional sources of innovation financing. Many governments are setting
up emergency relief packages to cushion the impact of the lockdown and face the looming
recession. But the GII 2020 advises that further rounds of support must prioritize and then broaden
support for innovation, particularly for smaller enterprises and start-ups that are facing hurdles in
accessing rescue packages
70. Ans:(c)
Solution: Switzerland, Sweden, the USA, the UK and Netherlands are the top five countries.
With a group of Asian economies advancing up the rankings, the index indicates that "a gradual
eastward shift in the locus of innovation" is underway.
In Asian economies, China, India, the Philippines and Vietnam have made the most progress on the
index in recent years, with all four now among the top 50.
India is one of the leading innovation achievers in the central and southern Asian region, as it has
shown a consistent improvement in its innovation ranking for the last 5 years.
China, which is the only middle-income economy among the top 30, now holds the 14th position.
71. Ans:(d)
Solution: India improved the most in three pillars: Institutions (61st), business sophistication (55th),
and creative outputs (64th).
The consistent improvement in the index rankings is owing to the immense knowledge capital, the
vibrant startup ecosystem, and the amazing work done by the public and private research
organisations.
The India Innovation Index 2019 which was released by the NITI Aayog, has been widely accepted
as the major step in the direction of decentralisation of innovation across all the states of India.
GII provides detailed metrics about the innovation performance of 131 countries and economies
around the world.
Its 80 indicators explore a broad vision of innovation, including political environment, education,
infrastructure and business sophistication.
It is published annually by Cornell University, INSEAD and the WIPO.

QUANTITATIVE TECHNIQUES

72. Ans:(b)
In 2005 the average number of companies registered
= 5+10+15+35 ×10004 = 650004 = 16250
In 2006, the average number of companies registered
= 10+25+30+ +35 ×1000 4 = 25000
In 2007, the average number of companies registered
= 5+10+15+20 × 10004 = 12500
In 2008, the average number of companies registered
= (30+25+20+15)4 = 904 = 22.5 = 22500
In 2009, the average number of companies registered
= (5+20+25+35 ) ×10004 = 21250
In 2010, the average number of companies registered
= 10+20+25+30 ×1000 4 = 21250
In 2011, the average number of companies registered
= (5+15+25+35 ) ×10004 = 20000
In 2012, the average number of companies registered
= 10+15+30+35 × 10004 = 22500
Hence, in 2006, the average number of companies registered was the maximum.
73. Ans:(d)
Number of companies registered in 2012 = 90000
Number of companies registered in 2005 = 65000
∴ Required % = 90000-65000 65000 ×100 = 38.46%
74. Ans:(a)
Number of companies registered in japan during 2005 to 2012
= (10 + 35 + 10 + 25 + 20 + 30 + 35 + 35) × 1000 = 20000
Number of companies registered in China during 2005 to 2012
= (35 + 30 + 15 + 30 + 35 + 25 + 5 + 30) × 1000 = 205000
Required ratio = 20000 : 20500 = 40 : 41
75. Ans:(b)
Average number of companies registered in France
= (15 + 25 + 20 + 15 + 25 + 10 + 15 + 10) ×10008
= 135 ×10008 = 1687.5
Average number of companies registered in Germany
= (5 + 10 + 5 + 20 + 5 + 20 + 25 + 15) ×10008
= 105 ×10008 = 1312.5
Average number of companies registered in Japan = 200008 = 2500
Average number of companies registered in China = 205008 = 2562.5
Clearly, average number of companies registered in china is the maximum.
76. Ans:(c)
Total number of companies registered in France = 135000
Total number of companies registered in china = 20500
Therefore, 20500-13500 20500× 100 = 34.14%
Less than the number of companies registered in China.
77. Ans:(d)
Explanation: We are not given enough information regarding their relative positions and so there is
insufficient information to calculate the distance between Srilanka to Finland.
78. Ans:(c)
Explanation: There are 5 stormy days and 25 non-stormy days during the period. This gives the
ratio 5 : 25, which expressed in its simplest form is 1 : 5.
79. Ans:(a)
Explanation: From a previous question the total number of passages was calculated to be 2,450.
The total for the mouths June-September is 1,200 which is slightly less than half the total for the
year.
80. Ans:(b)
Explanation: The passages are the same distance and both would experience adverse currents
one way and favourable currents the other, so the effect of the currents is cancelled out. Therefore
you would expect the passages to take die same amount of time.
81. Ans:(d)
Explanation: There were a total of 2,450 sailings. To find 12% of this total, divide by 100 to get 1%
and then multiple by 12(2450÷100=14.5×12=294).
82. Ans:(a)
Explanation: The distance is 1,000 Nmiles and the speed that counts is the speed over the ground
(the speed through the water will be higher because of the counter current)
The answer is therefore 1,000 ÷ 8 = 125.
83. Ans:(d)
P(Win first game) x P(Win second game)= 0.4 x 0.3=0.12
84. Ans:(d)
P(won at least 1 game)= 1- P(won no games)
=1- [P(lost 1st game) x P(lost second game)] =1- [(1-0.4) x (1-0.2)] (0.2) in the second bracket
because after losing the first game the probability of wining the second match is 0.2. So 1-0.2 is the
probability of losing that game too.
85. Ans:(d)
0.4 x 0.3 x 0.3= 0.036
86. Ans:(d)
This problem can be solved in three parts
Part 1- India wins first game and loses second and third
part 2= Lose + Win + Lose
Part 3= Lose + Lose+ Win
P (Part 1)= India wins first game x India loses second game x India loses third game
= 0.4 x (1-0.3) x (1-0.2)= 0.4 x 0.7 x 0.8 = 0.224
P (Part2)= India loses first game x Wins second game x Loses third game
= (1-0.4) x 0.2 x (1-0.3)= 0.6 x 0.2 x 0.7= 0.084
P (Part 3)= L x L x W = (1-0.4) x (1-0.2) x 0.2= 0.6 x 0.8 x 0.2= 0.096
P= P1+P2+P3= 0.404
87. Ans:(b)
Part 1= Won first x Lost Second= 0.4 x (1-0.3)= 0.4 x 0.7=0.28
Part 2= Lost First x Won second = (1-0.4) x 0.2= 0.6 x 0.2=0.12
P= 0.28+0.12=0.40

LOGICAL REASONING
88. Ans:(d)
Religious guy said to the atheist that ―Well then you must believe now,‖ because ―After all, here you
are, alive.‖ And the religious guy believes in god. Therefore, option (D) is the most appropriate
answer.
89. Ans:(a)
Religious guy assumes that it was the god who came to save an atheist guy in the form of Eskimos.
Therefore, option (A) is the correct answer.
90. Ans:(d)
Option (A) is not the correct answer, because atheist guy said that ―No, man, all that was a couple
Eskimos happened to come wandering by and showed me the way back to camp‖. Hence, atheist
concludes that God do not exist, and it is humans who help each other. Therefore, option (D) is the
correct answer. Option (B) and (C) are not correct because they are not relevant in the above
passage.
91. Ans:(a)
After atheist guy got caught in terrible blizzard a month ago, religious guy said to the atheist that
―Well then you must believe in god now, after all, here you are, alive.‖ Because he thinks that it was
the god who came to help an atheist guy. Therefore, option (A) is the correct answer.
92. Ans:(a)
The main idea of the author in the above passage is that God exists and in difficult situations God
will come to help you. In option (A) even after asking for help from God he didn‘t get any help and
eventually died. Therefore, option (A) is the correct answer because it is actually weakening the
main idea of the religious guy in the passage. Option (B) is not correct because it is further
strengthening the main idea of the religious guy.
93. Ans:(a)
Atheist guy‘s main idea in the passage is that he doesn‘t believe in existence of god. The reason for
his argument in the passage is that, instead of getting help from god he got help from couple of
Eskimos. Therefore, option (A) is the correct answer.
94. Ans:(c)
―While this forced almost everyone to be included in numerous databases, it gave very little in return
by way of greater accountability or transparency of the State.‖
From the above statement in the passage (C) can be inferred. The author does not comment on the
utility of NPR, neither does he mean or say that the focus on digitalisation has been futile; infact, he
states that the government has been able to centralise citizens‘ data through digitalisation. Therefore
(A) and (B) are wrong. Option (D) is wrong because the author comments on the ‗coercive methods‘
employed for digitalisation and not digitalisation itself while saying „not seen in any nation with a
functioning democracy‟.
95. Ans:(b)
According to the author the government initiated digitalisation by ―relying on coercive methods not
seen in any nation with a functioning democracy‖. If it is true that a vast majority of people
participated willingly in the process of digitalisation then it weakens the author‗s claim.
96. Ans:(a)
The author says that digitalisation leads to hyper–centralisation which may have adverse effects on
federalism – digitalisation as the cause and hyper centralisation as its effect. An option which implies
or states that digitalisation does not necessarily lead to centralisation will weaken the author‗s
argument on digitalisation and centralisation.
Option (A) states that the technologies allow decentralised options also and therefore digitalisation
can be achieved without affecting federalism. Therefore, it is the correct option.
97. Ans:(a)
Refer to the explanation of the previous question. Only Option (A) strengthens the author‗s argument
that that digitalisation leads to hyper–centralisation which may have adverse effects on federalism.
98. Ans:(b)
Option B is the most apt answer.
Options A, C as well as D are too generalized, and hence do not suit the question.
99. Ans:(b)
The passage implies that real equality may not operate on the ground level despite claiming it, and
laws can be manipulated by those who are in power.
Other options don't have a direct connection with what is being said by the author.
100. Ans:(a)
It clearly establishes that a group of people misuse power for their own benefits. And the author
wants to focus more on the concept of equality rather than animalism, or the major's death.
101. Ans:(b)
Some elite groups may influence the democratic structure by using the privileges they already have,
and have greater joys than the others. No other option strengthens the author's argument.
102. Ans:(a)
Corporates have a lot of financial power and they use that to influence governments in their favour.
Option b is also correct as a statement but has less to do with the passage. The passage has
nothing to do with religious government and nor does it state that all governments are surely corrupt.
103. Ans:(d)
Option (d) is the most suitable because the passage talks about how equality, even though it exists,
can be flawed and manipulated.
104. Ans:(c)
Explanation: In the last few years we continually see the dramatic increase in the number of clinical
trials. There is a need of a proper structure for regulating the clinical trials as India lacks a proper
mechanism in the clinical trial industry. With experienced clinical professionals and technological
advances, India will be sure to give best as a major favourite destination for the clinical researches in
the near future. Both the options provide a base and conclusion for the article. Hence the answer is
option (C).
105. Ans:(a)
Only option A provides advantages of Clinical trials.
Option B and C weaken the argument by mentioning the negative consequences of Clinical Trials.
106. Ans:(c)
Explanation: Both A and B are clear indications of what might happen if clinical trials go awry.
Hence, both weaken the argument.
107. Ans:(d)
Explanation: Though there are advantages of clinical trials the consent of a well-informed participant
is a necessary factor. Therefore, an unethical trail, falsified data, fraud in trials, non-compliance with
GCP and the over execution of trials without the approval of DCGI are few kinds of risk involved.
Hence the answer is option (D).
108. Ans:(d)
Explanation: Option is (D) more elaborative in explanation than option (C). Here the consent of the
participant in the clinical trial plays the crucial role which denotes that the patients who are getting
involved in the respective process should be informed or be made aware of the trial being conducted
on him/her, as the process involved the negative and positive side effects, etc. There are many
countries which have no specific law to tackle the misconduct that is being carried out on the
patients who are not aware that they are being tested. Hence the answer is option (D).
109. Ans:(c)
The last paragraph of the passage states ‗I think the best takeaway for Americans, when looking at
the Japanese, is that if we restrict our intake of refined sugar, industrial seed oils, and increase [our]
intake of marine omega-3s, then we might be able to tolerate eating more rice‘.
This suggests that currently American people eat a lot of refined sugar and not enough healthy fats.
Hence (c)
110. Ans:(a)
The passage talks about the Japanese diet being very healthy even though it includes a lot of carbs.
The fact that the country is being researched for its good food habits means that the Japanese
people have good health.
Hence (a).
111. Ans:(c)
Option (a) and option (b) speak of the goodness of unprocessed foods and the Japanese diet, both
of which support information in the passage. Option (d) also supports the Japanese health-lifestyle
model. Option (c), which says that Japanese people are eating less rice, is a warning sign.
Hence (c).
112. Ans:(d)
Both A and B support the passage which talks about the overall health of the Japanese people
because of their good food choices – high carbs, low fat and good fats from fish, etc. Hence (d).
113. Ans:(b)
Option (b) best represents the main message of the given passage, as it talks about both health
aspects practised by the Japanese: good food and exercise. Hence (b).
114. Ans:(c)
Here also code is direct letter to another letter coding, we can decode this as common letters have
same letter as its code.

With the help of above table, the code for GUEST is FQSMN.
115. Ans:(d)
If a day before yesterday was Tuesday, then yesterday was Wednesday. So today is Thursday and
tomorrow will be Friday. Hence Saturday will be on day after tomorrow.

LEGAL REASONING

116. Ans:(b)
Explanation: The passage states that it is a prime requirement of the tort for an outrageous act to
occur. In this case, walking by Sugriv‘s home of Bali cannot be construed to be an act that the
defendant has done. Thus, he is not liable for IIED.
117. Ans:(b)
Explanation: The passage states that an outrageous act that scares someone must be done. In this
case, the goon was merely standing with a sunflower in his hand as no one except Rajesh‘s family
knew that he was allergic to sunflowers. Thus, it cannot be said that he acted outrageously.
118. Ans:(a)
Explanation: The passage states that an outrageous act that scares someone must be done. In this
case, Anand‘s father being family of Anand would have known about his allergy to sunflower pollen
and thus it can be said that Anand‘s father ‗s act is an act outrageous enough to cause emotional
distress to Anand.
119. Ans:(d)
Explanation: The passage states that an act may be considered in context to understand if the
same is outrageous in nature. In this case, understanding from the context, it can clearly be seen
that the carrying of the beer bottle was not an act outrageous in nature and could not have
reasonable caused distress.
120. Ans:(d)
Explanation: The passage states that possible defences to the act of IIED is that merely a society
finding it offensive is not enough. In this case, the objection to breastfeeding picture is merely based
on societal morality and decency. However, the same should not be classified as outrageous.
121. Ans:(c)
Explanation: The passage states that mere knowledge of the fact that an act is too dangerous and
will cause loss of life is enough to constitute murder. Thus, he can argue that he did not know that
the glucose was too dangerous to human health.
122. Ans:(c)
Explanation: The passage states that there must be an intentional act that causes the death of a
person. Such an act also includes an omission within. Thus, the doctor‘s omission of not treating
Amish shall render him liable for murder.
123. Ans:(b)
Explanation: The passage states that for constituting the offence of murder, a person must have the
intention to kill. In this case, Amish died only because the doctor was busy seeing other patients. It
cannot be said that he had an intention to kill Amish.
124. Ans:(b)
Explanation:The passage states that a person doing a reckless act which is very dangerous to the
society must realize the same. However, despite throwing Molotov cocktails, Aftab cannot be held
liable as the same did not explode. Aftab is thus not liable.
125. Ans:(a)
Explanation:The passage states that a person doing a reckless act which is very dangerous to the
society must realize the same. However, Aftab should have anticipated that cars may explode and
thus, he is liable for the murder.
126. Ans:(c)
Explanation:PAX-12 has not spread worldwide which is the definition of the world as laid down by
WHO. Thus, the same cannot be called a pandemic.
127. Ans:(c)
Explanation: The state has to formulate regulations. Act doesn‘t give any powers by itself. Thus, the
emergency team shall only be justified in checking them if the state has formed appropriate
regulations.
128. Ans:(b)
Explanation: The answer is self-explanatory. Gurpreet has taken off from Mumbai.
129. Ans:(d)
Explanation: Powers under the Epidemics Act must be used only when the existing laws are
insufficient to deal with the prevention. Thus, the state of Tripura is unjustified in imposing the ban
for existing laws are sufficient to prevent harm.
130. Ans:(c)
Explanation: The passage states that the ban can only be imposed on a class of persons and not
on 1 person specifically. Thus, L&L shall be given relief for Epidemic Act allows to place such a ban
only on a class of persons.
131. Ans:(a)
Explanation: The Supreme Court, in Suresh Singhal v. State, has held that a person, who is in
imminent and reasonable danger of losing his life, may inflict harm on his assailant. In the present
case, there is no imminent threat to life from the little boy which warranted an action, that too as
grave as killing him.
132. Ans:(b)
Explanation: As per the judgment of the Supreme Court in Amjad Khan Vs the State, the right to
private defence extends to defending either body or property. In the present case, Kabir threw stone
at Karan, to protect her bike, which constitutes a valid private defence.
133. Ans:(a)
Explanation:The right of private defence cannot be claimed by an accused, if the facts suggest that
the parties had involved in a free fight between them, and it could not be determined which party
was the first aggressor. In the present case, Guddu and Bablu entered in a free fight with Munna,
which was decided mutually by both the parties, thus there was no aggressor of the fight. Therefore,
Guddu does not get a right to private defence against Munna.
134. Ans:(b)
Explanation: As per the judgment of the Supreme Court in Amjad Khan Vs the State, the right to
private defence extends to defending either body or property. In the present case, when Dimpy shot
the thief, there was a threat neither to the body of a person nor any property. The thief had already
escaped when Dimpy shot him, so there was no threat that needed defending from.
135. Ans:(c)
Explanation: The Supreme Court, in Suresh Singhal v. State, has held that a mere reasonable
apprehension of danger to life is enough to put the right of self-defence into operation. In the present
case, after listening to a gunshot when Seema saw the man approaching her with a gun, she was
under an imminent apprehension of threat to life. This was reason enough for her to act in private
defence.
136. Ans:(d)
Explanation: Section 438 of the Criminal Procedure Code deals with ‗grant of bail to a person
apprehending arrest‘ and empowers the High Court or the Court of Sessions to grant anticipatory
bail upon exercising discretion. Thus, the court can exercise jurisdiction while deciding a bail
application.
137. Ans:(a)
Explanation:In Siddharam Mhetre v. State of Maharashtra (2011), SC held that custodial
interrogation should be avoided in cases where the accused has joined investigation and is fully
cooperating with the investigating agency and is not likely to abscond. In the present case, Sushil is
not likely to abscond as he knew that he has acted in self defence and is already cooperating with
investigation, he should have been granted bail by the magistrate.
138. Ans:(b)
Explanation: As per the Supreme Court in Nandini Satpathy v. PL Dani, the police are required to
permit the advocate of the accused to be present at the time he is examined. In the present case,
Abu Mallick has not been examined yet, so denying the request to lawyer breaches no fundamental
rights.
139. Ans:(c)
Explanation: Section 438 of the Criminal Procedure Code deals with ‗grant of bail to a person
apprehending arrest‘ and empowers the High Court or the Court of Sessions to grant anticipatory
bail upon exercising discretion. In the present case, the arrest has already been made, hence an
application for anticipatory bail could not be filed at this stage.
140. Ans:(c)
Explanation:In Siddharam Mhetre v. State of Maharashtra (2011), the SC held there is no
requirement to make out a ‗special case‘ for exercise of power under Section 438, only if legal
requisites such not absconding, tampering with evidence and cooperating with the investigation is
satisfied, anticipatory bail can be granted.
141. Ans:(b)
Explanation: The passage states that yelling and refusal of normal household chores is not cruelty.
However, a reasonable construction of the same must be made and yelling once is not sufficient to
get the benefit of domestic violence laws.
142. Ans:(a)
Explanation: The passage states that for cruelty to occur, there has to be an illegal act of mental or
physical torture. In this case, the demand of property (stolen laptop) was illegal. Beating up wife for
refusal of demand is physical torture.
143. Ans:(b)
Explanation: The passage states that liability extends to Husband or relative of husband. In this
case however, the liability does not extend friend of the husband. Thus, there is no cruelty. Though
Panna can be held liable for defamation and other offences mentioned under IPC.
144. Ans:(b)
Explanation:The law states that beating children is cruelty only when it is done with the intent of
harassing the mother. This however is not the case here as the mother is merely complaining. Thus,
the acts cannot constitute cruelty.
145. Ans:(b)
Explanation:The Court explained that, "the term ―cruelty‖ implies harsh and harmful conduct with
certain intensity and persistence. It covers acts causing both physical and mental agony and torture.
In order to hold that the acts amount to cruelty, it must be shown that such acts amount to
unbearable, continuous, repeated acts of brutality. Cancelling movie dates does not qualify as
harmful conduct done intentionally.
146. Ans:(c)
Explanation: In the absence of an identity certificate from the district magistrate Avanti cannot be
rightfully vested with legal rights pertaining to transgender people)
147. Ans:(d)
Explanation: Because the passage does not specify whether the government will be subsidizing or
providing the services for free or not.)
148. Ans:(c)
Explanation: Refer to line 6 of the para 1 of the passage. She does not have the identity certificate
from the magistrate. In such a situation she continues to be a female on a legal footing. ‗b‘ is not the
right answer because had her certificate of identity been valid she would have not been able to avail
to reservation as a woman any longer regardless of whether she took admission as a female or not)
149. Ans:(c)
Explanation: The fact that she later changed her gender has got nothing to do with her admission.
During that time, she was a female and therefore could avail the reservation.)
150. Ans:(b)
Explanation: Because all the special rights are conferred upon a trans by the said certificate.
Legal Edge 13 Mock

Answers & Details

ENGLISH

1. Ans:(a)
The first option is the most appropriate answer. The other options are unrelated.
2. Ans:(d)
The most appropriate meaning of farcical is senseless
3. Ans:(c)
Option (d) & (b) are unrelated. Option (A) is the opposite.
4. Ans:(a)
The phrase ―a blemish of a carbon footprint‖ means that the gifts that are passed on in Diwali are too
old and haven‘t been used.
5. Ans:(d)
It is an adjective qualifying the noun ‗stupidity‘
6. Ans:(c)
The author likes to partake in gifts if they are meaningful, thus, option (a) is wrong. The author
says she doesn‘t believe that Lord Ram had entered Ayodhya amidst fireworks, hence option (b) is
wrong. She feels festivals have become and not are a farce of what they formerly used to be, hence,
option (d) is wrong. Option (c) is correct as the author does mention this. Hence, the answer is
option (c).
7. Ans:(c)
The author mocks the concept of fireworks for celebrating Lord Ram‘s return. This is a form of satire.
It‘s not sarcasm, because no ironical concept is present in this. It will fall under the vast scope of
satire. Thus, the answer is option (c).
8. Ans:(d)
E.S. is a humble, dedicated & a nationalist person.
9. Ans:(c)
Obscure here means to conceal or hide something.
10. Ans:(b)

11. Ans:(b)

12. Ans:(a)
Haughty means arrogantly superior. Supercilious is looking as though one thinks one is superior to
others. Self-deprecating is being critical of oneself.
13. Ans:(c)
Option (c) captures the entire theme of the passage in the most appropriate manner.
14. Ans:(b)
Vocational means directed at a particular occupation and its skills.
15. Ans:(b)
Our here refers to people at large.
16. Ans:(d)
First paragraph clearly states that dyslexic children have a problem in coping up with the entire
environment and that is the real trauma for them.
17. Ans:(c)
The correct usage of the article would be ―Perhaps these are the traits that helped Albert Einstein
and Agatha Christie and Steve Jobs, all dyslexics, to make their mark in the world‖
18. Ans:(d)
The second paragraph clearly mentions all of the above options as straightforward consequences.
19. Ans:(c)
The third paragraph clearly mentions that women are affected by the pandemic more because of the
increased unpaid household work & stark increase in domestic violence cases.
20. Ans:(d)
The passage above clearly states that ―Aligning these recovery packages with India‘s commitments
on climate change by investing in green jobs will improve lives and make our planet healthier‖
21. Ans:(c)
The word lever here denotes ‗a means of pressurizing someone into doing something‘
22. Ans:(b)
All the other options only capture the limited essence of the passage.
23. Ans:(d)
Trudge means walking slowly and with heavy steps, typically because of exhaustion or harsh
conditions.
24. Ans:(d)
All of the above mentioned words mean ―exhaustion/tiring‖
25. Ans:(c)
Ilk here means of the same kind/variety or specie
26. Ans:(a)
The author here is trying to convey that it is very easy for humans to do some small gestures of
kindness towards birds, like providing them with easy access to water.
27. Ans:(d)
The author is trying to draw comparison between the women & crows and trying to convey that it
very difficult for the crows to quench their thirst in scorching summers.
28. Ans:(d)
The meaning of the word ‗sedentary‘ is tending to spend much time seated.
29. Ans:(d)
The second paragraph clearly points out all the issues associated with sending children to school on
a bicycle.
30. Ans:(c)
Promoting gender equality is not a direct advantage for children. It is more of a societal advantage.
31. Ans:(a)
Option (b), (c) & (d) are unrelated.
32. Ans:(a)
All the other options do not find a mention in the above passage.

GENERAL KNOWLEDGE/CURRENT AFFAIRS

33. Ans:(a)
Explanation:
•Overall Performance:
With a score of 27.2,India has a level of hunger that is ―serious‖.
It ranks 94 out of 107 countries in the Index. In 2019, India‘s rank
was 102 out of 117 countries.
•Performance on the Indicators:
Undernourishment: 14% of India‘s population is undernourished
(2017-19). It was 16.3% during 2011-13.
Child Wasting: 17.3% (2015-19), it was 15.1% in 2010-14.
Child Stunting: 34.7%, it has improved significantly, from 54% in 2000
to less than 35% now.
Child Mortality: 3.7%, it was 5.2% in 2012
34. Ans:(c)
Explanation: India has been ranked at 94 among 107 countries in the Global Hunger Index (GHI)
2020.
Comparison with Other Countries:
India features behind Nepal (73), Pakistan (88), Bangladesh (75),
Indonesia (70) among others.
Out of the total 107 countries, only 13 countries fare worse than
India including countries like Rwanda (97), Nigeria (98), Afghanistan
(99), Liberia (102), Mozambique (103), Chad (107) among others.
35. Ans:(c)
Explanation: Concern Worldwide is Ireland's largest aid and humanitarian agency. Since its
foundation 50 years ago it has worked in 50 countries. According to its latest annual report, Concern
helped 28.6 million of the world's poorest and most vulnerable people in 2019, while responding to
82 emergencies in 24 countries.
36. Ans:(d)
Explanation: It was first produced in 2006. It is published every October. The 2020
edition marks the 15 edition of the GHI.
Aim: To comprehensively measure and track hunger at the global, regional, and
country levels. (GHI) is a tool designed to comprehensively measure and track hunger at global,
regional, and national levels. GHI scores are calculated each year to assess progress and setbacks
in combating hunger.
37. Ans:(b)
Explanation: The report scores the countries based on four indicators such as undernourishment,
child wasting, child stunting and child mortality. According to the report, undernourishment is the
share of population whose caloric intake is not sufficient. Child wasting is the children who have low
weight for their height. Child stunting is the children who have low height for their weight. For this the
children of age below 5 are considered. Based on the values of the four indicators, the GHI
determines hunger on a
100-point scale where 0 is the best possible score (no hunger) and 100 is
the worst.
Each country‘s GHI score is classified by severity, from low to extremely
alarming.
38. Ans:(a)
Explanation: Hepatitis refers to an inflammatory condition of the liver.
It‘s commonly caused by a viral infection, but there are other possible causes of hepatitis like
autoimmune responses, medications, drugs, toxins, and alcohol.
There are 5 main hepatitis viruses, referred to as types A, B, C, D, and E.
World Hepatitis Day is observed each year on 28th July to enhance awareness of viral hepatitis.
Hepatitis C is caused by the Hepatitis C Virus (HCV).
39. Ans:(a)
Explanation: The award is popularly but incorrectly known as Nobel Prize in Economic Sciences,
(as it is not one of the five Nobel prizes that Alfred Nobel established in his will in 1895, it is not a
Nobel Prize).
It was created in 1968 by a donation from Sweden's central bank Sveriges Riksbank to the Nobel
Foundation to commemorate the bank's 300th anniversary and includes a 10 million Swedish kronor
award money (₹8.33 crores roughly).
Milgrom and Wilson improved the auction theory and invented new auction formats for auctioning off
many interrelated objects simultaneously, on behalf of a seller motivated by broad societal benefit
rather than maximal revenue.
40. Ans:(c)
Explanation: Emmanuelle Charpentier of France and Jennifer A Doudna of the USA have been
awarded the 2020 Nobel Prize in Chemistry for developing CRISPR/Cas9 genetic scissors, one of
gene technology‘s sharpest tools.
It is for the first time a Nobel science prize has gone to a women-only team.
The CRISPR/Cas9 genetic scissors can be used to change the deoxyribonucleic acid (DNA) of
animals, plants and microorganisms with extremely high precision.
The CRISPR/Cas9 tool has already contributed to significant gains in crop resilience, altering their
genetic code to better withstand drought and pests.
The technology has contributed to new cancer therapies.
41. Ans:(b)
Explanation: Since 1901, the Nobel Prizes have been presented to Nobel Laureates at ceremonies
on 10 December, the anniversary of Alfred Nobel‘s death.
As stipulated in Nobel‘s will, the Nobel Prizes in Physics, Chemistry, Physiology or Medicine and
Literature are awarded in Stockholm, Sweden, while the Nobel Peace Prize is awarded in Oslo,
Norway.
Since 1969 an additional prize has been awarded at the ceremony in Stockholm - the Sveriges
Riksbank Prize in Economic Sciences in Memory of Alfred Nobel.
42. Ans:(b)
Explanation: Marie Curie was the first woman to be awarded in 1903 (Physics Prize), followed by
Bertha von Suttner who was awarded in 1905 (Peace Prize). As of 2019, the Nobel Prize have been
awarded to women 53 times (Marie Curie is awarded twice). In 2009, the year when most women
were awarded in a single year, five women were awarded.
43. Ans:(b)
Economics was not included in the first edition of the Nobel Prizes. The first Nobel Prize in
Economics was awarded to Ragnar Frisch and Jan Tinbergen for their development and application
of dynamic models for the analysis of economic processes in 1969.
44. Ans:(a)
Explanation: Venus is the hottest planet of the solar system. It is also called Earth‘s Twin. The
surface temperatures of Venus reach 880 degrees Fahrenheit. It has highly dense clouds of 65 miles
that puts the atmospheric pressure 90 times as felt on the Earth‘s surface. There are no moons or
rings around Venus.
45. Ans:(b)
Explanation: VIRAL is Venus Infrared Atmospheric Gases Linker instrument that was co-developed
with Russian Federal Space Agency Roscosmos and LATMOS atmospheres. After its Mars orbiter
mission, Mangalyaan and moon mission Chandrayaan-1 and Chandrayaan-2, ISRO has now set its
eyes on Venus.
France is one of the three countries with whom India collaborates in strategic sectors such as space,
nuclear and defence. The two countries are US and Russia.
46. Ans:(d)
Explanation: Indian Space Research Organisation (ISRO) has resumed its work on Gaganyaan -
India's human space flight programme. ISRO chairman Dr K Sivan indicated that the Indian human
space flight is set for August 2022, although some small shift may take place owing to disruptions
due to the ongoing coronavirus pandemic.
The Gaganyaan mission aims to send a three-member crew to space for a period of five-seven days
by 2022 when India completes 75 years of its Independence. India has chosen four members of the
Indian Air Force for the mission and their training is in progress.
47. Ans:(c)
Explanation: Akatsuki is a Japanese orbiter mission currently studying the planet Venus. ... On
Venus, while the planet rotates at 6 km/h at the equator, the atmosphere spins around the planet at
300 km/h. Other experiments are designed to confirm the presence of lightning and to determine
whether volcanism occurs currently on Venus
48. Ans:(a)
Explanation: On September 30, 2020, the French Space Agency CNES announced that it is to
participate in ISRO‘s Venus Mission, SHUKRAYAAN that is to be launched in 2025. The ISRO
chairman and his French Counter held talks to review the areas of cooperation between the
countries.
49. Ans:(b)
Explanation: Under the 14th five-year plan that is to be implemented from 2021, China has
proposed hydro power project on Brahmaputra river in Tibet. The project has been proposed by the
Central Committee of the ruling Communist Party of China. The Committee prepared the fourteenth
five-year plan, 2021 to 2025 and the long-term goals through 2035 of China.
The riparian states of the Brahmaputra river such as Bangladesh and India have raised concerns
against the proposal.
50. Ans:(d)
Explanation: The Brahmaputra, called Yarlung Tsangpo in Tibet, Siang/Dihang River in Arunachal
Pradesh and Luit, Dilao in Assam, is a trans-boundary river which flows through Tibet, India and
Bangladesh. It is the ninth largest river in the world by discharge, and the 15th longest. With its origin
in the Manasarovar Lake region, near the Mount Kailash, located on the northern side of the
Himalayas in Burang County of Tibet as the Yarlung Tsangpo River, it flows along southern Tibet to
break through the Himalayas in great gorges (including the Yarlung Tsangpo Grand Canyon) and
into Arunachal Pradesh (India)
51. Ans:(a)
Explanation: In 2015, China operationalised its first hydropower project at Zangmu in Tibet, while
three other dams at Dagu, Jiexu and Jiacha are being developed, all on the upper and middle
reaches of the river. The Zangmu Dam is a gravity dam on the Yarlung Zangbo/Brahmaputra River 9
km northwest of Gyaca in the Tibet Autonomous Region of China. This dam is built a few kilometers
from the Bhutan-India border. The purpose of the dam is hydroelectric power production using run-
of-the-river technology.
52. Ans:(d)
Explanation: It is a perennial river and has several peculiar characteristics due to its geography and
prevailing climatic conditions.
It is flooded twice annually. One flood is caused by the melting of the Himalayan snow in summer
and the other due to the monsoon flows.
The frequency of these floods have increased and are devastating due to climate change and its
impact on high and low flows.
These pose a concern for the population and food security in the lower riparian states of India and
Bangladesh.
The river is in itself dynamic as frequent landslides and geological activity force it to change course
very often.
53. Ans:(a)
Explanation: The state-owned hydropower company POWERCHINA signed a strategic cooperation
agreement with the Tibet Autonomous Region (TAR) government to implement hydropower
exploitation in the downstream of the Yarlung Zangbo river as part of the new Five Year Plan (2021-
2025).
This will be the first time the downstream sections of the river will be tapped. However, the location
of the planned project has not been mentioned anywhere.
The Great Bend of the Brahmaputra and the Yarlung Zangbo Grand Canyon in Medog county,
where the river turns sharply to flow across the border into f Arunachal Pradesh could be the
potential spot for the project.
This 50 km section alone offers a potential of developing 70 million kilowatt hours (Kwh).
54. Ans:(d)
Explanation: Nokia's network equipment will be installed remotely on the Moon's surface using a
lunar hopper built by Intuitive Machines in late 2022."The network will self-configure upon
deployment," the firm said in a statement, adding that the wireless technology will allow for "vital
command and control functions, remote control of lunar rovers, real-time navigation and streaming of
high definition video."The 4G equipment can be updated to a super-fast 5G network in the future
55. Ans:(c)
Explanation: The Artemis program is a US government-funded human spaceflight program that has
the goal of landing "the first woman and the next man" on the Moon, specifically at the lunar south
pole region by 2024. The program is carried out predominantly by NASA, U.S. commercial
spaceflight companies contracted by NASA, and international partners including the European
Space Agency (ESA), the Japan Aerospace Exploration Agency (JAXA), Canadian Space Agency
(CSA), the Italian Space Agency (ASI)[6] the Australian Space Agency (ASA), the UK Space Agency
(UKSA) and the United Arab Emirates Space Agency (UAESA).
56. Ans:(a)
Explanation: The $14.1 million contract, awarded to Nokia‘s US subsidiary, is part of Nasa‘s
Artemis programme which aims to send the first woman, and next man, to the moon by 2024. The
astronauts will begin carrying out detailed experiments and explorations which the agency hopes will
help it develop its first human mission to Mars. it would distribute $370 million to 14 companies to
supply "Tipping Point" technologies for its mission, which include robotics and new methods of
harvesting the resources required for living on the moon, such as oxygen and energy sources.
57. Ans:(b)
Explanation: The Finnish company will partner with Texas-based private space craft design firm
Intuitive Machines to deliver the network equipment to the moon on their lunar lander.
Intuitive Machines, LLC is a private American company headquartered in Houston, Texas. It was
founded in 2013 by Steve Altemus, Kam Ghaffarian, Tim Crain to provide autonomous systems for
industrial systems, drones, spacecraft and spacesuit modeling and simulation services.
58. Ans:(a)
Explanation: Nokia Corporation (natively Nokia Oyj, referred to as Nokia; Finnish: is a Finnish
multinational telecommunications, information technology, and consumer electronics company,
founded in 1865. Nokia's headquarters are in Espoo, Finland, in the greater Helsinki metropolitan
area. In 2018, Nokia employed approximately 103,000 people across over 100 countries, did
business in more than 130 countries, and reported annual revenues of around €23 billion
59. Ans:(b)
Explanation: PM unveils ‗Statue of Peace‘ to mark 151st Birth Anniversary celebrations of
Jainacharya Shree Vijay Vallabh Surishwer Ji MaharajPM also requested Spiritual leaders to
promote AatmNirbharta by going vocal for local.
Mentioning the educational institutions established by the Jainacharya, the Prime Minister praised
his efforts to make the country AatmNirbhar in the field of education as he established many
institutes imbued with indian values in states like Punjab, Rajasthan, Gujarat, Maharashtra and Uttar
Pradesh.
PM said that these institutions have given so many industrialists, judges, doctors, and engineers
who have given yeoman service to the nation.
The Prime Minister also underscored the debt that the country owes to the efforts of these
institutions in the field of women's education as these institutions kept the flame of female education
alive in those difficult times.
60. Ans:(d)
Explanation: The Statue unveiled in the honour of the Jain acharya, has been named as ‗Statue of
Peace‘.
The 151 inch tall statue has been made from Ashtadhatu(octo-alloy) i.e. 8 metals, with Copper being
the major constituent, and is installed at Vijay Vallabh Sadhana Kendra, Jetpura, in Pali, Rajasthan.
The alloy is mainly used in casting metallic idols in Hindu and Jain temples.
The alloy is made of metals such as gold, copper, silver, tin, zinc, iron, tin and antimony (or
mercury). The metals are mixed in equal proportions to create the idol.
61. Ans:(a)
Explanation: The Statue of Unity is a colossal statue of Indian statesman and independence activist
Vallabhbhai Patel (1875–1950), who was the first Deputy Prime Minister and Home minister of
independent India and adherent of Mahatma Gandhi during the non-violent Indian Independence
movement. Patel was highly respected for his leadership in uniting 562 princely states of India with a
major part of the former British Raj to form the single Union of India. The statue is located in the
state of Gujarat, India. It is the world's tallest statue with a height of 182 metres (597 feet). It is
located on the Narmada River in the Kevadiya colony, facing the Sardar Sarovar Dam 100
kilometres (62 mi) southeast of the city of Vadodara and 150 kilometres (93 mi) from Surat.
62. Ans:(d)
Explanation: He was a Jain monk born on October 26, 1870.
He was a disciple of Vijayanand Suri.
He was the founder of Shri Parshwanath Jain Vidyalaya in Pali district of Rajasthan.
Also, he established the Mahavir Jain Vidyalaya in Mumbai, Pune and Vadodara.
He supported non-violent independent movements of Mahatma Gandhiji.
The Jains are one of the six centrally notified minorities in India. The other five are Buddhists,
Muslims, Christians, Sikhs and Parsis.
63. Ans:(a)
Explanation: Acharya Vijayanand Suri, also known as Atmaramji of Gujranwala, was the first
Swetambar Murtipujaka Jain monk in modern times to receive the title of Acharya. Born and raised
in Punjab, he was initiated as Sthanakvasi monk who later joined Murtipujaka tradition
64. Ans:(c)
Explanation: The software acquired from INTERPOL uses various mechanism such as detecting
nudity in images and to recognise the age of the person through their facial structures. It has an
inbuilt algorithm that will look for keywords such as child pornography. The algorithm will then track
the forums that indulge in these crimes using the keywords.
65. Ans:(d)
Explanation: The TRACE unit was set up as a part of a larger campaign against the Child Sexual
Abuse Material (CSAM) across the country since 2019. The team recently trained around 270
policemen across Maharashtra in the use of this software to track these cases.
The Maharashtra cyber cops have begun to forward the complaints to the districts where FIRs are
registered. So far around 50 persons have been placed under arrest.
66. Ans:(b)
Explanation: ‗Operation Blackface‘ is part of the larger action taken against CSAM across the
country. Acting on the tip off provided by NCRB, Maharashtra Cyber cops started forwarding
complaints to the districts where FIR‘s were registered against accused persons. In the current year
alone, there have been above 100 FIR‘s registered in cases linked to CSAM and nearly 50 persons
placed under arrest. The officer said that with their team being trained by Interpol it better equips
them to deal with CSAM instances and should overall bring down the number of such occurances
that has Maharashtra as its source.
67. Ans:(a)
Explanation: The International Criminal Police Organization (official abbreviation ICPO; French:
Organisation internationale de police criminelle), commonly known as INTERPOL is an international
organization that facilitates worldwide police cooperation and crime control. Headquartered in Lyon,
it has seven regional bureaus worldwide and a National Central Bureau in all 194 member states,
making it the world's largest police organization.
INTERPOL originated with the first International Criminal Police Congress in 1914, which brought
officials from 24 countries to discuss cooperation on law enforcement matters. It was founded in
1923 as the International Criminal Police Commission (ICPC), adopting many of its current duties
throughout the 1930s. After coming under Nazi control in 1938, the agency was effectively moribund
until after the Second World War. In 1956, the ICPC adopted a new constitution and the name
INTERPOL, derived from its telegraphic address used since 1946
68. Ans:(b)
Explanation: Researchers from the US Centers for Disease Control and Prevention (CDC) have
recently discovered a rare Ebola-like illness that is believed to have first originated in rural Bolivia in
2004.The virus is named Chapare after the province in which it was first observed.
Chapare, is a rural province in the northern region of central Bolivia.
69. Ans:(b)
Explanation: The recent biggest outbreak of the ‗Chapare virus‘ was reported in 2019, when three
healthcare workers contracted the illness from two patients in the Bolivian capital of La Paz. Bolivia
is a country in central South America, with a varied terrain spanning Andes Mountains, the Atacama
Desert and Amazon Basin rainforest. At more than 3,500m, its administrative capital, La Paz, sits on
the Andes‘ Altiplano plateau with snow-capped Mt. Illimani in the background. Nearby is glass-
smooth Lake Titicaca, the continent‘s largest lake, straddling the border with Peru
70. Ans:(d)
Explanation: Chapare Virus belongs to the same Arenavirus family that is responsible for illnesses
such as the Ebola virus disease (EVD). It causes Chapare Hemorrhagic Fever (CHHF).
Chapare virus is much more difficult to catch than the coronavirus as it is not transmissible via the
respiratory route. Instead, Chapare spreads only through direct contact with bodily fluids.
New sequencing tools will help develop an RT-PCR test — much like the one used to diagnose
Covid-19 to help detect Chapare.
71. Ans:(a)
Explanation: Chapare virus are generally carried by rats and can be transmitted through direct
contact with the infected rodent, its urine and droppings, or through contact with an infected person.
A disease vector is any agent which carries and transmits an infectious pathogen into another living
organism.
Virus can spread from person to person.
Chapare spreads only through direct contact with bodily fluids.
Sexually transmission:
Researchers also found fragments of Ribonucleic acid (RNA) associated with Chapare, in the semen
of one survivor 168 days after he was infected.

QUANTITATIVE TECHNIQUES

72. Ans:(b)
Explanation: The value of maximum exports = 6045.
The value of minimum imports = 87.
Therefore, the required ratio (6045/87) = 69.48 = 69 (approximately).
73. Ans:(b)
Explanation: Out of a total of 12 countries, 8 showed a deficit while 4 showed a surplus.
74. Ans:(b)
Explanation: Sum of exports - Sum of imports = deficit(11286).
75. Ans:(d)
Explanation: Visually its clear that L has the highest trade deficit.
76. Ans:(b)
Explanation: I has a ratio of 4002/2744 = 1.45, which is the highest.
77. Ans:(c)
Explanation: is the correct answer.

Years Australia England Others


2015 (1200) 450 150 600
2016 (1600) 750 400 450
2017 (1400) 300 400 700
The total runs scored in 2016 were 1600.
78. Ans:(b)
Explanation: The total runs scored against England in all three years = 150 + 400 + 400 = 950
79. Ans:(d)
Explanation: The total runs scored against Australia in 2015 = 450
The total runs scored against England in 2017 = 400
So, required ratio = 450 : 400 = 9 : 8
80. Ans:(a)
Explanation: The total runs scored against Others in 2015 = 600
The total runs scored against Others in 2016 = 450
So, required difference = 600 – 450 = 150
81. Ans:(c)
Explanation: The total runs scored against Australia in 2016 = 750
The total runs scored against Australia in 2017 = 300
so, Required % = 750300×100=250%
82. Ans:(b)
Explanation: Total Secretaries in Finance Ministry = 960 ÷ 48=20
Number of male Secretaries in finance Ministry = 33+2×20=12
Number of male officers from Communication and information Technology Ministry
= 88+5×520=320
Therefore, required percentage = 12320×100=380×100=3.75%
Hence, option (b)
83. Ans:(b)
Explanation: Number of Secretaries in Agriculture Ministry = 1080 ÷ 36 = 30
Number of Secretaries in Human Resource Development Ministry = 720 ÷45=16
Number of Secretaries in Panchayati Raj ministry = 560 ÷40=14
Therefore, required average = 30+16+143=603=20
Hence, option (b)
84. Ans:(c)
Explanation: Total number of officers from Human Resource development Ministry = 720
Total number of Officers from Panchyati Raj and Communication and information
Technology Ministry = 560 + 520 = 1080
Therefore, required percentage = 7201080×100=69×100=66.67%
Hence, Option (c)
85. Ans:(d)
Explanation: Number of Secretaries in Agriculture Ministry = 1080 ÷36=30
Number of Secretaries in Human Resource Development Ministry = 720 ÷45=16
Number of Male Secretaries in Agriculture Ministry = 88+7×30=16
Number of female Secretaries in Agriculture ministry = 30 – 16 = 14
Number of male Secretaries in Human Resource development Ministry = 55+3×16=10
Number of females Secretaries in human Resource department = 16 – 10 = 6
Male Officers in Agriculture Ministry = 55+4×1080=600
Therefore, female Officers in Agriculture Ministry = 1080 – 600 = 480
Number of male officers in human Resource development Ministry = 77+5×720=420
Therefore, female Officers in Human Resource development Ministry = 720 – 420 = 300
So, required difference = (16 + 10 + 600+420)- (14+ 6 + 480 + 300) = 1046 – 800 = 246
Hence, option (d)
86. Ans:(b)
Explanation: Total number of students in Youth Ministry = 1080+7202=900
So, total number of secretaries = 900 ÷60=15
Hence, Option (b).
LOGICAL REASONING

87. Ans:(d)
Explanation: Justin Trudeau‘s comments resonate with the attitude shown by the likes of Modi,
Trump and Cameron of appealing and playing the populism card to the constituencies to boost their
respective vote-share back home. Thus, the answer is option (d).
88. Ans:(a)
Explanation: This line of the passage ―Indian diplomacy has come a long way since the
“foreign hand” bogey…..‖ shows that Indian governments used to try to impress global leaders
about democracy in India rather than the diaspora. Thus, option (a) is correct and (b) is wrong.
Option (c) is out of the scope of this passage, thus, the answer is option (a).
89. Ans:(c)
Explanation: The author means to say that world leaders have tried to just impress the Indian
diaspora because they are a major force in determining the elections in India and many other
countries. Trudeau might have appeared to have taken a different step from Modi, Trump or
Cameron, but, his motive is to placate the Sikh or Indian diaspora in his country to get electoral
gains. Thus, option (c) is the answer.
90. Ans:(c)
Explanation: The reason for this is the huge population that India has. India‘s population and its
diaspora population is so huge that it has become a determining factor in the elections of many
countries. World leaders come to India, placate Modi when he is having a popular surge to score
some brownie-points with Indian diaspora voters in their respective countries. Trudeau sided with the
farers to win Sikh votes in Canada. Modi placates the Indian diaspora to get rewards from the
families of diaspora who reside in India. Thus, the answer is option (c).
91. Ans:(b)
Explanation: The author wants the farmers to stay away from the support of foreign hands and
especially Canada because Canada has a record of its support to the Khalistanis. It can be
understood from this line ―Canada has its own share of tragic memories, and political and
security failures on the “Khalistan” issue.‖ Thus, any foreign support from Canada and the
farmers would get likened to the Khalistanis. Thus, the answer is option (b).
92. Ans:(b)
Explanation: The first paragraph explains what ‗economic reasoning‘ means. Option B summarizes
the paragraph, and is the best option. Option A is factually correct but uses the word ‗scarcity‘, which
is unwarranted. Option C is a distractor. It is verbose and merely states that the choice is irrational –
which is contrary to ‗economic reasoning‘. Option D, similarly, is contrary to ‗economic reasoning‘.
Hence, [B].
93. Ans:(a)
Explanation: The answer is based on the first two sentences of the second paragraph: ―A more
efficient allocation is one that increases the net value of resources. Efficiency in the
allocation of resources is distinguished from equity, which is concerned with justice in the
distribution of wealth.‖ Options B, C and D depict situations where the net value of resources is
maximized. In the case depicted in option A, those who benefit from reservation would value it more
than others who are not under reservation. The latter may feel they are unduly deprived. Hence, [A].
94. Ans:(c)
Explanation: The second paragraph states, ―It (moral optimality) works with utilitarianism
because the idea of wealth maximization intuitively translates into more utility.‖ In other words,
in utilitarianism there is no other consideration than maximization of wealth. Option C states the
same as a preoccupation with outcomes, not considering whether something is good or bad. Options
A and B ascribe moral dimensions to the exchange, hence do not relate to utilitarianism. Option D is
stated in the paragraph in connection with Kantian theory of morality. Hence, [C].
95. Ans:(a)
Explanation: Under the Kaldor-Hicks efficiency test, an outcome is efficient if those who are made
better off could in theory compensate those who are made worse off and so produce a Pareto
efficient outcome. So, it follows that it produces more benefits than costs. Option B is not correct as
pareto improvement leaves no one worse off, but Kaldor – Hicks efficiency can. Option C is incorrect
as the eventual endpoint of pareto superior moves is termed pareto optimality, and not Kaldor-
Hicks efficiency. Option D is contradictory; as per Kaldor-Hicks efficiency, the gains after an
economic exchange are large enough that the winners could, if they had to, compensate the losers
in the new allocation of goods and still remain better off. Hence, [A].
96. Ans:(b)
Explanation: ―The most central assumption in economics is that human beings are rational
maximizes of their individual satisfactions, and, in turn, respond to incentives….‖ The
passage is then developed fully from this main idea. The central idea is negated by option B. In
effect option B states that ―…people's behaviour changes in unpredictable ways,‖ or that they
are not rational. The other options too characterize rational behaviour. Option D is an example
rational behaviour –the ability to recognize costs other than that of money – opportunity cost being
one of them. Hence, [B]
97. Ans:(a)
Explanation: The central idea of Lata‘s argument is the comparison between the hygienic condition
of hospitals in last century and this century. Therefore, A is the correct answer.
98. Ans:(b)
Explanation: If b is true than it clearly points that the reason for improvement is not improvement in
hygienic conditions but the improvement in medical science therefore, b is the correct answer.
99. Ans:(b)
Explanation: Option A weakens her argument as it shows that action is taken against erring
doctors.
Option b strengthens Simran‘s argument as it shows that the hygiene conditions have not yet
improved in the hospitals. Therefore, b is the correct answer.
100. Ans:(b)
Explanation: The last sentence forms her conclusion, rest of the sentences form the premise.
The given line forms the conclusion to the Lata‘s argument. The correct answer is b.
101. Ans:(c)
Explanation: Option d is the conclusion itself, and Simran has used option c as the premise to form
her conclusion. Therefore, c is the correct answer.
102. Ans:(d)
Explanation: Difficult Passage: Refer to the line, ―the processes of reproduction, birth, and
nurturance, in one form or another provide the essential foundation of kinship.”
1)It states only part of the view held by Gellner, et al., as it ignores birth and nurturance.
2)Passage does not mention about social class anywhere.
3)It is an indirect inference; passage does not mention surrogate ties.
4)Correct. It paraphrases the lines above, which describe the view, held by those who believed in
the biological framework for analyzing kinship.
Thus, choice D is the correct answer.
103. Ans:(a)
Explanation: Refer to paragraph 2 ―Furthermore the biological ........ for comparison”.
1)Correct. The lines above clearly state that biological processes are constant and culture is specific
and hence cannot be used.
2)Not mentioned in the passage.
3)Nothing is mentioned about quantifiability of culture.
4)There should be a constant for comparison so that comparative analysis can be done, whether this
constant should only be objective is not mentioned and cannot be inferred from the passage.
Thus, choice A is the correct answer.
104. Ans:(c)
Explanation:
1)Schneider does not imply ―marital relationships‖ vary because of culture, but just states that using
biology to analyze them is an ethnocentric construct. This can at best be a very indirect inference.
2)Passage does not talk about the evolution of human relationships.
3)Correct. It cannot be used universally because the notion of birth and family being the foundation
of kinship is Eurocentric and hence specific. It is mentioned in the third paragraph, last sentences.
4)Attaching importance to culture does not mean that for them kinship is defined by culture rather
than biology.
Thus, choice C is the correct answer.
105. Ans:(d)
Explanation: Refer to the lines in the third paragraph, ―They countered Gellner’s position by
arguing that kinship was primarily a matter of culture. It was the interpretation of the
processes of reproduction and not simply the processes in themselves that constituted
kinship.”
1)Biology does play a role in kinship, at least in Europe.
2)Eurocentric notions of kinship are different not biological processes.
3)Does not talk about the relationship between marriage and kinship. According to Schneider, the
relationship between biological framework for analyzing and marriage and family relationships is a
strongly is a European notion.
4)Correct. According to the above lines, Kinship is a matter of culture, and it should be analyzed
using a cultural framework. It is constituted by the interpretation of the processes of reproduction.
Hence, it can be inferred that it is culture that offers this interpretation, which is different for different
cultures.
Thus, choice D is the correct answer.
106. Ans:(c)
Explanation: Refer to the last paragraph. Most philosophy has been a reaction against
scepticism whereby nominally rational arguments had to be invented to secure social
coherence. This has led to deep insincerity infecting most philosophy. None of the other
options can be inferred from the passage.
Hence, [C].
107. Ans:(a)
Explanation: Refer to the second paragraph: ―But philosophy, in the historically usual sense,
has resulted from the attempt to produce a synthesis of science and religion.‖ Option B is said
in connection with ethics and politics. Option C is slightly tricky- last paragraph states that
―philosophers, when they have tackled the problem of preserving social coherence, have
sought solutions less obviously dependent upon dogma than those offered by official
religions. Most philosophy has been a reaction against scepticism…‖ So, option C is true about
certain or ‗most philosophy‘. But, with the specific mention of philosophy in the ―historically usual
sense‖, option C has to be rejected in favour of option A, which provides a precise answer.
Hence, [A].
108. Ans:(b)
Explanation: Option [A] is in opposition to the central idea of the passage. Option [C] is not even
stated by the author. Philosophy was largely sceptical of dogmas, but it didn't 'merely' concentrate
on that. So, [D] is an exaggeration. Refer to paragraph 2: ―sometimes ethical motives influenced
the philosopher's views as to the nature of the universe, sometimes his views as to the
universe led him to ethical conclusions.” Hence, [B].
109. Ans:(c)
Explanation: Options [A] and [B] are categorically stated in the passage. Refer Paragraph 2. ―….
Almost all types of philosophy were invented by the Greeks, and the controversies of our
own day were already vigorous among the pre-Socratics”. Refer to the first two sentences of
paragraph 1: option [D] too is clearly stated. Option [C] is contrary to the passage. Thus, the author
is least likely to agree with [C].
Hence, [C].
110. Ans:(a)
Explanation: Medium difficulty level question
1)Correct. Explains the rationale behind the rise and fall in share prices of Jump Networks and Kavit
Industries respectively and shows that it is not illogical and irrational.
2)Holding professional degrees cannot be assumed to result in people acting rationally
3)There is no information which indicates that unpredictability and volatility are inversely proportional
to the investor buying and selling rationally
4)It doesn‘t weaken the author‘s conclusion as share prices have changed irrespective of ups and
downs in profit.
Thus, choice A is the correct answer.
111. Ans:(a)
Explanation: Medium difficulty level question
The argument presumes that if adults keep sanitizing their children‘s friendships, the children will not
be able to develop close friendships and hence, will not be able to develop healthy adult
relationships. Hence, it is flawed, as correctly identified in choice A.
The factor of secret friendships, as mentioned in choice B, is irrelevant to the reasoning of the above
argument.
The argument is concerned with the emotional effects of affection and rejection in adult relationships
and childhood friendships and not whether they are otherwise similar. Thus, choice C is incorrect.
The argument does not assume that affection and rejection does not affect a child. In fact it implies
that affection and rejection that affects the child will prepare it emotionally for similar situations in its
adult life. Therefore, D is not incorrect.
Thus, choice A is the correct answer.
112. Ans:(c)

113. Ans:(c)

114. Ans:(d)

O>M>P>L>Q>N
LEGAL REASONING

115. Ans:(b)
Explanation: The passage states that for a contract to be vitiated on grounds of undue influence,
the very first requirement is the establishment of a dominating position. In this case, Rajesh being a
school student has been heavily dependent on the peon for his sneaking in activities. Thus, the peon
is in a dominating position.
116. Ans:(d)
Explanation: The passage states that a contract may be voidable for reasons of undue influence if
one person exerts influence over the other due to reasons of mental distress. The fact that despite
being 15 years of age, Rajesh is still in the 4th standard goes on to indicate that Rajesh does not
have normal mental capacity.
117. Ans:(d)
Explanation: The passage states that the burden of proof shall lie on the party which plays the
dominating role in a particular contract. In this case, by virtue of the fact that the employee controls
all of Patil‘s privileges, he is the dominating party in the contract. Thus, the burden of proof lies on
him.
118. Ans:(c)
Explanation: The passage states that there is a presumption of the use of a dominating position if
the presence of the same has been established. The same can be rebut if some other object is
shown. The employee quitting the department would mean that there was no exercise of his
dominant position on Patil anymore.
119. Ans:(b)
Explanation: The passage states that any undue influence occurs when the parties are in a
fiduciary relationship and one party abuses their dominant position for their unjust enrichment. In this
case, the lawyer in suggesting mediation as a way out cannot be said to be unjustly enriching
himself for mediation is the most effective.
120. Ans:(a)
Explanation: The passage states that any undue influence occurs when the parties are in a
fiduciary relationship and one party abuses their dominant position for their unjust enrichment. In this
case, it can be seen that the lawyer suggested the least effective remedy so that he could derive the
most wealth.
121. Ans:(d)
Explanation: The question requires assumption of authority of the law laid in the case of Shafhi
Mohammad. The case has clearly laid that ―the requirement of a certificate under Section 64B(4),
being procedural, can be relaxed by the Court wherever the interest of justice so justifies, and one
circumstance in which the interest of justice so justifies would be where the electronic device is
produced by a party who is not in possession of such device, as a result of which such party would
not be in a position to secure the requisite certificate‖. The above question is in line with the situation
explained under the judgement, hence, option (d) is the correct option.
122. Ans:(a)
Explanation: The Supreme Court in the latest judgement clarified that ―…..the required certificate
under Section 65B(4) is unnecessary if the original document itself is produced.‖ Hence, only
option (a) is correct and the rest are incorrect.
123. Ans:(c)
Explanation: The most viable alternative is option (c). The Supreme Court in its latest judgment has
clarified that ―An application can always be made to a Judge for production of such a certificate from
the requisite person under Section 65B(4) in cases in which such person refuses to give it.‖
124. Ans:(b)
Explanation: A USB cannot be primary evidence as documents contained in it are a copy of original
documents; and secondary evidence requires production of a certificate for it to be admissible.
Hence, option (b) is the correct option.
125. Ans:(c)
Explanation: Both acts of the defendant- not appearing before the court and not honoring the ex-
parte judgement passed by the court constitutes civil contempt of court according to the definition of
civil contempt.
126. Ans:(a)
Explanation: The school did not honor the writ issued by the SC. This constitutes civil contempt of
court regardless of it being a private or a govt. school. Hence, option D is not correct.
Also, the school denying admission to only girls constitutes violation of fundamental rights regardless
of the age. Hence, option B is not correct.
Anyone can file a case in the court when a fundamental right is violated. Hence, option D is not
correct.
127. Ans:(b)
Explanation: In order to exercise its power to punish the contemnor the court has to be satisfied
beyond reasonable doubt that he/she has willfully, deliberately and intentionally violated the court‘s
order. The court in the above case cannot infer beyond reasonable doubt that the defendant violated
the court‘s order willfully, deliberately and intentionally.
128. Ans:(d)
Explanation: The undertaking given by Raja is valid as it was incorporated in the court order. Raja
failed to honor his undertaking by the end of the year. This constitutes willful breach of undertaking
as he was benefitted from the order passed based on the undertaking.
129. Ans:(c)
Explanation: The answer is (c). This is because initiating a prosecution against an actual crime is
not forbidden by law. Therefore, it does not fall under the purview of unlawful. The case would have
been different if there could have been a threat of initiating a false prosecution. Therefore, the
contract is valid.
130. Ans:(c)
Explanation: The answer is (c). This is because the terms of the contract kept both the parties at an
unequal position where one could terminate out of will while Ghanshyam could not. In addition to
this, it has to be seen that he was a young song writer so this allows the company to dominate and
exploit him. Therefore, there existed some pressure even when he was at a liberty not to enter in it.
131. Ans:(b)
Explanation: The answer is (b) because the act was voluntary without any pressure exercised by
Zen. The reason can be the mutual trust and care developed among the two which made Mona to
gift the house as good gesture. This also makes (b) more appropriate than (c) as the former is the
effect of the latter and (b) is more close and relevant to the question.
132. Ans:(d)
Explanation: The answer is (d). This is because Monica is unemployed so she is not in a position to
create dominance or pressure but could have been subjected to it by the other party. Therefore, no
duress is exercised by Monica. Also, (d) is much closer to (c) as more than the gift, the position of
Monica is more relevant.
133. Ans:(a)
Explanation: The answer is (a). This is because the unlawful act or threat can be initiated from
anywhere. Also, the act need not be committed against the party but the act against the family or
relative creates an influence on the party. Therefore, the claim is justified and Harman does not have
to fulfill any obligations.
134. Ans:(b)
Explanation: The answer is (b). This is because he was under the impression that the gun is not
loaded so the element of negligence or great care cannot come in question. Therefore, such mistake
of fact will exonerate him from the liability.
135. Ans:(d)
Explanation: The answer is (d). This is because Chulbul was under a mistake of a fact which was
not deliberate in nature. He acted in good faith to follow the order after a detailed enquiry therefore
exempting him of any liability.
136. Ans:(d)
Explanation: The answer is (d). This is because the letter was written without any due enquiry
which highlights the lacks of reasonable or due care making the actions not done under good faith
and thus creating a liability on the President of the Municipal office.
137. Ans:(c)
Explanation: The answer is (c). This is because when we take the whole factual information into
consideration, we realize that there existed no circumstances which made Seeta to think that Geeta
is the thief as the act of fetching water does not determine that the utensils were same and the
offence of theft has been committed. Therefore, Seeta did not act in good faith and shall be
considered liable.
138. Ans:(c)
Explanation: The answer is (c). This is because even when they acted in good faith the act shall fall
under mistake of law as they were importing non- edible sunflower oil under an open general license
therefore no defense is availed.
139. Ans:(d)
Explanation: The Consumer Protection Act defines a consumer as someone who acquires goods or
services for direct use or ownership rather than for resale or use in production and manufacturing. In
this case, Mr A bought the cotton for further production and not for direct use. Thus, he is not a
consumer and cannot make the seller liable under the Consumer Protection Act. He can sue under
the sale of goods act, but not under the COPRA. Option b may be correct but does not goes with the
above passage and is not the primary reason.
140. Ans:(c)
Explanation: Now Mr A is a consumer as he brought it for direct usage. Therefore, he can sue the
seller under COPRA. The passage does not mention that customer should beware. And given that
the goods sold were of substandard quality the seller will be liable. Between a and c, option c is
more relate `d to the passage as it does talk about consumer rights and fair trade.
141. Ans:(a)
Explanation: Mr B is a consumer as he purchased the watch for direct use and not for resale. State
Consumer Dispute Redressal Commission, known as State Commission, deals with complaints
involving costs and compensation higher than Rs. Twenty Lakh and less than Rs. One Crore. The
amount to be considered is compensation claimed not the cost price. Thus, the answer is (a).
142. Ans:(d)
Explanation: COPRA defines a consumer as someone who acquires goods or services for
consideration for direct use or ownership rather than for resale or use in production and
manufacturing.
He/She must purchase the goods for consideration. In the present case, Meghana didn‘t pay any
consideration for the cream it was a free sample. Thus, Rhea cannot be made liable under the
COPRA, this does not mean she cannot be made liable under any other Act.
143. Ans:(c)
Explanation: [The place to file a complaint can be the place of residence of the opposite party,
place of business or place of the branch office. Therefore, the answer is C. The place of residence of
the complainant is not included under COPRA.]
144. Ans:(c)
Explanation:
A- no consideration, therefore not a consumer
B- Though he buys it but resells it, no direct usage, no consumer
C- Consideration is there, online payment is considered as consideration- therefore consumer]
145. Ans:(d)
Explanation: [The facts do not tell whether there was malice and unreasonable cause with which
the case was filed. Nor does they tell anything about the damage caused. Therefore, the best option
is d. We cannot say all the elements are satisfied or not.]
146. Ans:(a)
Explanation: [The plaintiff must prove that the proceeding complained was terminated in favour of
the present plaintiff. The claimant must show that the prosecution ended in his favour, but so long as
it did, it is of no importance how this came about. In the end she was acquitted and thus, the
proceedings ended in her favour. All the other elements are present in the facts except the damage
caused one. Since there is no option of insufficient data, option a is the most accurate. The fact that
they are mother and son is irrelevant.]
147. Ans:(d)
Explanation: [Though the prosecution initiated against Sarthak was initiated with malice and without
a cause, it did not end in his favour. He was convicted for conspiracy for committing the murder.
Thus, all the essentials are not satisfied.
148. Ans:(b)
Explanation: [Prosecution starts when the case reaches the court, before that it is not prosecution.
Thus, option b. D is not correct because that data is not relevant to answer the question. Even if
there was no reason, malice and damage the also it won‘t stand for malicious procession because
YG was not prosecuted]
149. Ans:(d)
Explanation: [Though he was acquitted, there was a reasonable cause to suspect him. He was a
petty thief and present at the crime scene just before the crime scene. There was a lack of malice or
unreasonable cause, thus no malicious prosecution.]
150. Ans:(b)
Explanation: [Monetary loss is not an essential condition there must be damage of any kind- loss of
reputation, work, etc. All, the others are essential].
Legal Edge 14 Mock

Answers & Details

ENGLISH

1. Ans:(b)
Explanation: Option (b) is the correct answer because all the other options only give half the
answer without a reasonable explanation.
2. Ans:(c)
Explanation: Option (a) is irrelevant and (b) & (d) are incomplete answers.
3. Ans:(d)
Explanation: Calloused here means hardened.
4. Ans:(d)
Explanation: Varicose veins means (of a vein, especially in the leg) swollen, twisted, and
lengthened, as a result of poor circulation.
5. Ans:(d)
Explanation: All of the above options are synonyms of sturdy.
6. Ans:(a)
Explanation: Vent means to let out and hence it is a verb here
7. Ans:(d)
Explanation: All of the above words mean ‗an action, carefully planned, to deceive someone‘
8. Ans:(c)
Explanation: The whole passage is based on the incidents of racial discrimination.
9. Ans:(d)
Explanation: All the above statements are a valid conclusion that can be drawn from the above
passage.
10. Ans:(a)
Explanation: Option (a) captures the true essence of the above passage in every possible way.
11. Ans:(b)
Explanation: Magnifies here means intensifies which is a verb.
12. Ans:(d)
Explanation: The fourth paragraph mentions all the problems faced by the poor farmers
13. Ans:(c)
Explanation: The author was very concerned about the fact that the villager‘s faith was making him
impatient and the guilt pangs of his helplessness to be not able to do anything was making him
worried.
14. Ans:(b)
Explanation: In the last paragraph the author clearly mentions that he is poor because he has
nothing to give them back
15. Ans:(d)
Explanation: The meaning of the word woes is sorrows/ pain. Alleviations means making something
less severe
16. Ans:(d)
Explanation: The above mentioned statements capture the entire essence of the passage. The
author first of all explains how he related everything to its smell and then goes on to discuss that the
masks being the need of the hour, act as a hindrance to his reminiscing.
17. Ans:(a)
Explanation: ‗Led by the nose‘ aptly describes how the author is connected to everything through
his olfactory senses.
18. Ans:(d)
Explanation: All of the above mentioned words mean stinking/foul smelling
19. Ans:(b)
Explanation: Redolence is always used in reference of smell.
20. Ans:(a)
Explanation: Here brimming with enthusiasm means overflowing with excitement.
21. Ans:(b)
Explanation: The word avian means relating to birds.
22. Ans:(b)
Explanation: The author is trying to say that the coordination among both the genders of the bird
sets a great example for humans to learn.
23. Ans:(a)
Explanation: Weaning is the process of gradually introducing an infant human or another mammal
to what will be its adult diet while withdrawing the supply of its mother's milk. Here weaning is used
in the sense to denote Popo‘s learning of the art of flying.
24. Ans:(d)
Explanation: Here the most appropriate meaning of perched would be to alight/rest on something
25. Ans:(b)
Explanation: Eat like a bird means eating in small portions.
26. Ans:(b)
Explanation: In literal sense, etched means to cut a pattern, picture, etc. into a smooth surface.
However, here it is used to denote a distinct incident which created an impact on the family.
27. Ans:(c)
Explanation: Wafting means pass gently through the air.
28. Ans:(a)
Explanation: Doe here is a female deer.
29. Ans:(c)
Explanation: If you are fighting a losing battle, you are trying to achieve something but are not going
to be successful.
30. Ans:(d)
Explanation: The doe was so startled that I could not move. This is the idea that the author is trying
to convey by using the phrase ‗stood rooted to the ground‘.

GENERAL KNOWLEDGE/CURRENT AFFAIRS

31. Ans:(d)
Explanation: A Western Disturbance, labelled as an extra-tropical storm originating in the
Mediterranean, is an area of low pressure that brings sudden showers, snow and fog in northwest
India.
32. Ans:(b)
Explanation: During La Niña, the easterly trade winds strengthen and cold upwelling along the
equator and the West coast of South America intensifies.
Sea-surface temperatures along the equator can fall as much as 7 degrees F below normal.
33. Ans:(c)
Explanation: Himalayas do not pass through Uttar Pradesh.
34. Ans:(a)
Explanation: The Tropic of Cancer passes through India, through 6 states.
35. Ans:(b)
Explanation: India and Bangladesh share a boundary of 4096 km.
36. Ans:(d)
Explanation: All the three statements are correct with respect to El Nino.
El Niño is the warm phase of the El Niño–Southern Oscillation (ENSO) and is associated with a
band of warm ocean water that develops in the central and east-central equatorial Pacific (between
approximately the International Date Line and 120°W), including the area off the Pacific coast of
South America. The ENSO is the cycle of warm and cold sea surface temperature (SST) of the
tropical central and eastern Pacific Ocean. El Niño is accompanied by high air pressure in the
western Pacific and low air pressure in the eastern Pacific. El Niño phases are known to last close to
four years, however, records demonstrate that the cycles have lasted between two and seven years.
During the development of El Niño, rainfall develops between September–November. The cool
phase of ENSO is La Niña, with SSTs in the eastern Pacific below average, and air pressure high in
the eastern Pacific and low in the western Pacific. The ENSO cycle, including both El Niño and La
Niña, causes global changes in temperature and rainfall. Developing countries that depend on their
own agriculture and fishing, particularly those bordering the Pacific Ocean, are usually most
affected. In Spanish, the capitalized term El Niño means "the boy". In this phase of the Oscillation,
the pool of warm water in the Pacific near South America is often at its warmest about Christmas.
The original phrase, El Niño de Navidad, arose centuries ago, when Peruvian fishermen named the
weather phenomenon after the newborn Christ. La Niña, chosen as the "opposite" of El Niño, is
Spanish for "the girl".
37. Ans:(c)
Explanation: Uighurs are a Muslim minority community concentrated in the country‘s northwestern
Xinjiang province.They claim closer ethnic ties to Turkey and other central Asian countries than to
China, by brute — and brutal — force.
38. Ans:(a)
Explanation: In the western sector, India shares about 2152 km long border with China. It is
between Union Territory of Ladakh (erstwhile state of Jammu and Kashmir) and Xinjiang Province of
China. In this sector, there is a territorial dispute over Aksai Chin. India claims it as part of erstwhile
Kashmir, while China claims it is part of Xinjiang.
39. Ans:(d)
Explanation: The dispute over Aksai Chin can be traced back to the failure of the British Empire to
clearly demarcate a legal border between China and its Indian colony. During the time of British rule
in India, two borders between India and China were proposed- Johnson‘s Line and McDonald Line.
The Johnson‘s line (proposed in 1865) shows Aksai Chin in erstwhile Jammu and Kashmir (now
Ladakh) i.e. under India‘s control whereas McDonald Line (proposed in 1893) places it under
China‘s control. India considers Johnson Line as a correct, rightful national border with China, while
on the other hand, China considers the McDonald Line as the correct border with India.
40. Ans:(d)
Explanation: Four states viz., Himachal Pradesh, Uttarakhand, Sikkim and Arunachal Pradesh and
a Union Territories of Ladakh (erstwhile state of Jammu & Kashmir) share a border with China. The
Sino-Indian border is generally divided into three sectors namely: Western sector, Middle sector, and
Eastern sector.
41. Ans:(a)
Explanation: The Andhra Pradesh Legislative Assembly Friday passed the Andhra Pradesh Disha
Bill, 2019 (Andhra Pradesh Criminal Law (Amendment) Act 2019). The bill provides for awarding
death sentence for offences of rape and gangrape and expediting trials of such cases to within 21
days.
42. Ans:(d)
Explanation: The Justice Verma Committee, which was formed days after the horrific Nirbhaya
gangrape case in Delhi in December 2012 to review criminal law related to sexual assault, batted for
enhanced punishment.
43. Ans:(b)
Explanation: It is a general principle that if there is a conflict between a central law and a law
enacted by the state legislature, the central law shall prevail.
44. Ans:(a)
Explanation: The IPC provides for death penalty to rapists after the Criminal Law Amendment Act,
2013 which was enacted in the wake of the horrific Nirbhaya Rape Case.
45. Ans:(a)
Explanation: Anthrax is a disease caused by Bacillus anthracis, a spore-forming bacteria. It affects
animals such as cattle, sheep, and goats more often than people.
46. Ans:(c)

47. Ans:(b)
Explanation: Namdapha Wildlife Sanctuary is located in Arunachal Pradesh. Pobitora and Manas
are in Assam.
48. Ans:(c)
Explanation: Measles is caused by a virus whereas plague and tuberculosis are both caused by
bacteria.
49. Ans:(d)
Explanation: India dropped one spot to 131 among 189 countries in the 2020 human development
index, according to a report released by the United Nations Development Programme (UNDP).
Human Development Index is the measure of a nation's health, education, and standards of living.
Life expectancy of Indians at birth in 2019 was 69.7 years while Bangladesh has a life expectancy of
72.6 years and Pakistan 67.3 years, the 2020 Human Development Report said. India, Bhutan (129),
Bangladesh (133), Nepal (142), and Pakistan (154) were ranked among countries with medium
human development
50. Ans:(a)
Explanation: The Paris Agreement is an agreement within the United Nations Framework
Convention on Climate Change, dealing with greenhouse-gas-emissions mitigation, adaptation, and
finance, signed in 2016. India pledged to reduce the emission intensity of its GDP from the 2005
level by 33-35 percent by 2030 and to obtain 40 percent of electric power capacity from non-fossil
fuel sources by 2030.
51. Ans:(c)
Explanation: Norway topped the index, followed by Ireland, Switzerland, Hong Kong and Iceland,
the report showed. Norway, which tops the HDI, falls 15 places if this metric is used, leaving Ireland
at the top of the table. In fact, 50 countries would drop entirely out of the ―very high human
development group‖ category, using this new metric PHDI.
52. Ans:(a)
Explanation: Planetary pressures–adjusted HDI (PHDI): HDI value adjusted by the level of carbon
dioxide emissions and material foot print per capita to account for excessive human pressures on
the planet. It should be seen as an incentive for transforma tion.
Adjustment factor for planetary pressures: Arithmetic aver age of the carbon dioxide emissions
index and the material footprint index, both defined below. A high value implies less pressure on the
planet
53. Ans:(b)
Explanation: The United Nations Development Programme is the United Nations' global
development network. It promotes technical and investment cooperation among nations and
advocates for change and connects countries to knowledge, experience and resources to help
people build a better life for themselves.
54. Ans:(a)
Explanation: Of these seven, the contract for three frigates was awarded to GRSE while the
contract for another four frigates was awarded to Government-owned Mazagon Docks Limited
(MDL) which is based in Mumbai.
These frigates will come armed with advanced state-of-the-art sensors and boast of top-notch
stealth features.
They will represent the most advanced class of major surface warships for the Indian Navy in a
decade, also featuring BrahMos supersonic surface-to-surface missiles.
These will also have torpedoes and rockets to hit submarines and rapid-fire guns to destroy anti-ship
missiles as well as a heavy main gun to engage ships and coastal target.
55. Ans:(a)
Explanation: The Project 17A ships are guided missile frigates. Each of these ships are 149 metres
long. They have displacement of approximately 6,670 tonnes and a speed of twenty-eight knots. The
ships being built under the project uses the latest integrated construction methodology. They have
reduced build periods and enhanced pre-outfitting to augment quality. The ships are built using
indigenously developed steel. They are integrated with indigenous sonar system and weapons such
as BrahMos missile, LRSAM.
56. Ans:(b)
Explanation: The Garden Reach Shipbuilders and Engineers (GRSE) recently launched a new
Project 17A ship called Himgiri for the Indian Navy. The ship was launched into the waters of
Hooghly river. It is a Nilgiri-class Stealth Frigate. They are the updated class of Leander class.
Frigate is a type of warship that has different roles over time. Also, frigates are of various sizes. The
Nigiri class Frigates of Indian Navy are Himgiri, Dunagiri, Udaygiri, Vindhyagiri and Taragiri. The
ships have been named after the hills in India.
57. Ans:(d)
Explanation: The project was approved by the Government of India in 2015. Under the project, a
total of seven advanced frigates are to be delivered to the Indian Navy. Four of these are to be
constructed by the Mazagon Dock Shipbuilders Limited and three are to be launched by GRSE. The
cost of the project was estimated as 7 billion USD. The project led to creation of employment
opportunities for more than 2,000 Indian companies and MSMEs.
58. Ans:(b)
Explanation: It is the first of the three ships built by GRSE. The Project 17A ships are fitted with
advanced sensors, indigenous weapons and is equipped with gas turbine propulsion. Earlier Himgiri
was commissioned into the Indian Navy in 1974 and decommissioned in 2005. A new ship is now
being launched in 2020.
In 1976, Himgiri was the first ship of Indian Navy to shoot a pilotless aircraft.
59. Ans:(a)
Explanation: Klaus Martin Schwab, 5'9, is a German engineer and economist best known as the
founder and executive chairman of the World Economic Forum. His wife and first collaborator, Hilde,
co-founded the Schwab Foundation for Social Entrepreneurship with him.
60. Ans:(b)
Explanation: The WEF organisers released a statement saying that the World Economic Forum will
convene the Special Annual Meeting 2021 in Singapore from May 13-16, 2021. The forum will return
to Davos-Klosters, Switzerland, for the Annual Meeting 2022.
According to WEF president Borge Brende, the health and safety concerns linked to the COVID-19
outbreak in Europe made it impossible to organise the meeting in Lucerne-Buergenstock as it
was initially planned. Singapore was chosen as the new venue for WEF 2021 after careful
consideration, as it has been largely successful in dealing with the pandemic
61. Ans:(b)
Explanation: The World Economic Forum (WEF 2021) will be moved from Davos in Switzerland to
Singapore due to COVID-19 concerns. The organisers announced the decision on December 7,
2020.
The annual gathering will be moved out of Europe next year, as the organisers feel that the COVID-
19 pandemic will make it challenging to host the event safely in Europe. The WEF is an annual
gathering of political and business leaders.
This is only the second time since its inception in 1971 that the event, which is usually held in
January at Davos, will be hosted outside Switzerland. It will be held in Asia for the first time.
62. Ans:(c)
Explanation: Davos is a town in the Swiss Alps, within the canton of Graubünden. It‘s a popular ski
resort with a conference center that hosts the annual World Economic Forum. Downhill and cross-
country ski areas include Jakobshorn, Pischa, Rinerhorn and Parsenn. Summer activities include
swimming and sailing on Lake Davos, hiking and mountain biking
63. Ans:(d)
Explanation: The World Economic Forum, based in Cologny, Geneva Canton, Switzerland, is an
international NGO, founded in 1971. The WEF's mission is stated as "committed to improving the
state of the world by engaging business, political, academic, and other leaders of society to shape
global, regional, and industry agendas".
64. Ans:(d)
Explanation: The Uttarakhand state was the first state to launch Organic Farming Policy in the year
2000 and Sikkim was the first state to become 100 per cent organic.
What is Organic Farming? Organic farming us a farm design & management system that develops
an ecosystem of agriculture production without using any form of synthetic external inputs like
pesticides, synthetic hormones, chemical fertilizers or genetically modified organisms.
65. Ans:(c)
Explanation: Sikkim has become a pioneer in sustainable development. It is also India‘s first state
to achieve 100 per cent sanitation coverage and the only state that is open defecation free. They are
on the road to attaining almost full literacy, and the only state to increase its forest cover – about
half of the state area. In 2016, Sikkim declared itself as fully organic – the first and only state in
India.
66. Ans:(d)
Explanation: 1/3rd area of the island was certified as organic in 2012. Since then, the agriculture
department was working to bring 100 per cent area under the organic certification.
Now, the Union Territory has acquired the ―100% organic‖ certification with the help of the central
government‘s Paramparagat Krishi Vikas Yojana (PKVY) and the financial assistance given to the
Union Territory under the scheme.
67. Ans:(a)
Explanation: ―Paramparagat Krishi Vikas Yojana‖ is an elaborated component of Soil Health
Management (SHM) of major project National Mission of Sustainable Agriculture (NMSA). Under
PKVY Organic farming is promoted through adoption of organic village by cluster approach and PGS
certification.
68. Ans:(d)
Explanation: Participatory Guarantee Systems, as defined by IFOAM, are "locally focused quality
assurance systems. They certify producers based on active participation of stakeholders and are
built on a foundation of trust, social networks and knowledge exchange. They represent an
alternative to third party certification,[2] especially adapted to local markets and short supply chains.
They can also complement third party certification with a private label that brings additional
guarantees and transparency. PGS enable the direct participation of producers, consumers and
other stakeholders.

QUANTITATIVE TECHNIQUES

69. Ans:(b)
On Monday
Downstream: Upstream = 5:4.
Speed of boat = downstream + upstream speed /2
4+5/2 = 4.5 unit = 45
1 unit = 10
Downstream speed is = 5*10=50 and Upstream speed = 4*10 = 40
240/d +300/u =y
240/50+300/40=12.3
70. Ans:(c)
On Tuesday
Down-stream speed = 70 & upstream speed = 30
On Wednesday
down-stream speed = 30 & upstream speed = 20
Average Down speed = (70+30)/2 = 50 & Average upstream speed = (30+20)/2 = 25
Now, %more or less = 50-25 / 25 *100 = 100%
71. Ans:(d)
On Thursday - 250/D = 250/60 = 25/6
On Friday – (250+150)/U = 400/10 = 40
Now, total time = 25/6+40 = 265/6
72. Ans:(b)
On Friday
x/40 +x/10 = 10
x= 80
while on Tuesday, Total distance is same as Friday=2X=160
distance covered Down : UP = 5:3 = 100&60km.
Now, time taken is = 100/70 + 60/30 = 24/7 hr.
73. Ans:(a)
Distance covered Downstream on Friday = 4/3 * Distance covered Downstream on Wednesday
Let, (distance covered on Friday =4x and on Wednesday 3x)
Given,
4x/40 + 3x/30 = 14
X=70 = difference between distance covered (4x-3x)
74. Ans:(d)

Batsman Total match Average run Total run


V 143 27 3861
W 133 47 6251
X 129 43 5547
Y 124 37 4588
Z 123 29 3567
Batsman score highest run in all matches: W
75. Ans:(d)
Total run score by V and W is =3861 + 6251 = 10112
Total run score by Y and Z is = 4588 + 3567 = 8155
So required percentage is = (10112/8155)*100 = 124%
76. Ans:(c)
Total run of X after next match is: 5547 + 34 = 5581
New average of X is: 5581/130 = 43
Total run of Y after next match is: 4588 + 94 = 4682
New average of Y: = 4682/125 = 37
Total run of Z after next match: 3567 + 62 = 3629
New average of Z is: 3629/124 = 29
Total sum of new average is = 43+37+29 =109
So average is =109/3=36.33
77. Ans:(b)
Total run score by V, X, Z is: 3861 + 5547 + 3567 = 12975
Total run score by W, Y, Z is: 6251+4588+3567=14406
So the difference is =14406 – 12975 = 1431
78. Ans:(a)
Total run score by W and X is =11798
So average of W and X is = 11798/ (133+129) = 45
Total run score by Y and Z is = 8155
So average is = 8155/ (124+123) =33
So required percentage is = (45/33)*100 = 136%
79. Ans:(a)
COMMON SOLUTION
Total number of students=1600.
Number of students in Dentistry= 45% of 1600= 720.
Number of students in Homeopathy= 1600-720=880.
Ratio of boys and girls studying Dentistry= 5:3.
Number of boys studying Dentistry= (5/8) * 720= 450.
Number of girls studying Dentistry= (3/8) * 720= 270.
Ratio of boys and girls studying Homeopathy= 4:7.
Number of boys studying Homeopathy= (4/11) * 880= 320.
Number of girls studying Homeopathy= (7/11) * 880= 560.
Now make a diagram to put these values.

Number of students studying Dentistry know only either English or Hindi


16% of 450 + 10% of 450 = 26% of 450 = 117.
20% of 270 + 10% of 270 = 30% of 270 = 81.
On adding these two, we get 198.
Note: In case of either or, we use addition concept.
80. Ans:(b)
12% of 450 + 10% of 320 + 10% of 560 + 10% of 270
= 12% of 450 + 10% (320+560+270) =169.
81. Ans:(d)
(50% of 270 + 50% of 560)/(270 +560)×100=50%
82. Ans:(c)
(70% of 320)/(60% of 560)=224/336=2/3=2:3

LOGICAL REASONING

83. Ans:(b)
China‘s response to India is that India is following a discriminatory attitude towards China in its
internet regime. Options (a), (b) and (c) would definitely weaken China‘s stand in its response, but
(a) and (c) won‘t be ironical in its nature. What is ironical is that China has itself maintained a tightly
controlled and heavily censored internet for a long time. Thus, option (b) is the answer.
84. Ans:(d)
The question asks that why does the author think that the step taken is only beneficial to India in the
short run. The author mentions in the passage that first and foremost China is too big a player to be
sidelined and secondly, there is a risk that moves such as blocking apps would be perceived
adversely by global investors and Internet companies. Thus, if we study the options, then it is clear
that the answer is option (d).
85. Ans:(d)
Option (a) is correct as India is indeed trying to bring down China by hurting its economic interests
by banning its apps. Option (b) is correct as India‘s stance is reflecting payback against China‘s
aggressive stance at the border. Option (c) is correct as India‘s apps have not done well when there
were Chinese apps; the author does mention that. Option (d) can‘t be inferred as the author says
that India‘s internet ecosystem needs to be regulated. Hence, the answer is option (d).
86. Ans:(c)
The author is in support of this stance by India. He simply wants to remain cautious and not employ
this method in the long run so as not to harm the investor-confidence and paint a bad picture of
India‘s internet-regulation. So, option (a) is wrong. Option (b) is wrong as the author though
mentions that this method has given Indian apps an opportunity to fare better, but he
isn‘t crusading (crusade -to make an effort to achieve something which you believe in strongly) it.
Option (c) is correct and thus, is the answer.
87. Ans:(c)
The author says that “While it is true that there has been some push back against Chinese
companies and technology globally…‖, hence, this would mean India might get some support
from the other countries. Hence statement 1 is correct. The author does mention that this move
might harm bilateral relations, thus the talks between India and China might not make any heads
after this reaction by India. Hence, statement 2 is correct. Thus, option (c) is the answer.
_
88. Ans:(a)
Option b states that the trolls became the reason of death, it is a bad effect of internet. Option c also
is an effect of internet and about the preference of people. Option d is irrelevant to the context of the
passage.
Option ‗a‘ tells us that most of the people‘s interests are molded by the internet and their lifestyle is
the product of advertisement that see on a regular basis on internet. We try to look at the society
from the lens of internet and decide what is necessary for us and what is not. Hence option ‗a‘ is the
correct answer.
89. Ans:(a)
Author‘s main point of the passage is that the internet controls the form and scale of human
association and changes us as individual and society but the people who debate over internet only
debate about the content, if it is good or bad. The statement becomes the premise of the argument
on which the further argument of the author is built. Therefore, the correct answer is a.
90. Ans:(c)
Option a, b and d are the basis not for the content of the internet but the form of internet which
changes the people, society and the human associations. Therefore, Option c is the correct option,
the point made by the author is that the people only discuss the good and bad impact of the
internet.
91. Ans:(a)
Option a is the correct answer as the argument of the author that the internet controls the scale and
form of human association and delivers a new form of human being, whose qualities are suited to it
is also an effect of internet which can be categorize as bad impact and therefore the debaters are
already debating upon this issue too.
Option b and c are some of the good impacts of internet and does not strengthen or weakens the
author‘s argument.
Option d is a suggestion and it does not weaken the author‘s argument as the author‘s argument is
precisely that the internet impacts our psyche and delivers a new human being.
92. Ans:(c)
This passage contends that while debating the effects of the Internet, we tend to only discuss the
content it delivers, while ignoring the larger effect the medium has in shaping what we see and how
we interact. The Internet controls the scale and form of human association and in fact shapes us into
a new form of human being, whose qualities are suited to it. Option C summarizes this idea best.
93. Ans:(c)
It is mentioned in the passage that Anglo-Indian press gloated over Tilak‘s conviction and the ruling
was criticised in England. No specific reason for the same is mentioned, and hence we cannot say
with full certainty that Strachey‘s interpretation of ―disaffection‖ was unacceptable in England.
94. Ans:(d)
Option (a) is wrongly stereotyping the judiciary. Option (b) (ALWAYS proves counterproductive) is an
extreme answer choice and can be easily eliminated.
95. Ans:(a)
The statement encapsulates the main idea of the passage. Refer to the last part of the passage and
the author‘s concluding remarks.
96. Ans:(d)
Refer to, “it is almost unbelievable that an article on the killing of Afzal Khan by Shivaji could
be part of the foundation of a case of sedition.‖ Since the article was the part of foundation of the
sedition case, it is probably true that Tilak would have tried to justify the murder and in doing so, he
supported something that was supposedly against the law.
97. Ans:(a)
Option (a) can be inferred from the fact that Tilak was not allowed to present the correct version of
the English translation. Option B is an extreme option choice and can be eliminated.
98. Ans:(a)
Option (c) and option (d) are not only out of context but also scientifically erroneous. If we are not in
a position to help others, there is no point in ―helping‖ and threatening our own loves and those in
contact with us. Option (b)sounds ungrateful. The entire passage narrates the bravery of the medical
professionals. Hence (a).
99. Ans:(b)
The passage talks about the bravado of the medical community during the coronavirus pandemic.
Option (a) detracts from this and talks about dangers spreading among the medical community.
Option (c) is too generic. Option (b) is in line with the ―give thanks to the medical professionals‖
sentiment held by the passage. Hence (b).
100. Ans:(b)
Statement 'a‘ shows care being taken and ethical practices from the doctor to the patient. This
strengthens the argument made in the passage. Statement b weakens the passage with ―glitches in
accuracy.‖ Hence (b).
101. Ans:(c)
A pandemic is an epidemic that is spread out throughout the world. The passage explains this
through examples as well. Option (a) does not talk about the disease, option (b)is too vague, option
(c) is apt. Option (d) is out of context with ―economies.‖ Hence (c).
102. Ans:(b)
We do not know about the symptoms of coronavirus nor is it discussed. The passage talks about the
hard work of nurses and doctors and mentions the concerns and deaths of nurses and doctors
respectively. Options (c) and (d) are ruled out. Option (a) has no corroboratory evidence. We know
that the patients are large in number. Option (b) is apt. If it weren‘t contagious it wouldn‘t have been
a pandemic and there wouldn‘t be such a massive global crisis. Hence (b).
103. Ans:(c)
Explanation: Since the person whose statistic would have helped to alert people about cholera
himself died, the early signs of the disease were ignored, and hence cholera was detected late.
Thus, C is the right answer.
(a) states the causes of spread of the disease and not the detection.
(b) mentions how the disease came to Egypt.
(d) mentions the canal but not the mention why cholera was detected later.
Thus, choice (c) is the correct answer.
104. Ans:(d)
(a) Visiting traders brought the cholera with them, hence (a) was a reason.
(b) The early death of the principal barber meant late detection, hence it is also a reason.
(c) It states the water in mosques which caused the rapid spread of cholera.
(d) However, it just states of proximity to the Nile which is not a reason for the cholera.
Thus, choice (d) is the correct answer.
105. Ans:(a)
The passage tells us that since the early death of the principal barber delayed the detection of
cholera, A is a valid assertion. If there is just one principal barber and something happens to him, the
information he has will be lost.
(b) is not right since cutting off all trading ties is not necessary and there is no guarantee that
diseases won‘t come from anywhere else.
(c) is not valid since the spread can be stopped even while continuing to pray in mosques, but by
avoiding use of water.
(d) is wrong since the canal itself is not the problem, but the spread of disease through it is.
Thus, choice A is the correct answer.
106. Ans:(c)
Option (a) is incorrect as the passage is silent on India‘s past. So it cannot be assumed if
democracy was a failure in the past or not. Because of the same reason option (d) is incorrect.
Option (b) is incorrect as can be seen from the last sentence of the given argument. It clearly states
that women need to assert themselves in all spheres of life. Option (c) is the most appropriate
assumption as immediately after mentioning how democracy can be success in India; the
passage elaborates on the bias between men and women and insists on its removal.
107. Ans:(a)
According to the given argument, ‗X, if and only if Y; X being a soccer team‘s participation and Y
being 11 players‘. Option (b) also does not follow the logic because the second and the third
statements do not follow the same sequence as given in the question. Option (c) does not follow the
logic as the presence of the word ‗can‘ means existence of different possibilities. But according to
the question statement it needs to be a compulsion. Option (d) is a close answer but is wrong
because the conclusion states that she can never appear for the exam. While according to the given
statement, it should have been that for now she can not appear for the exam. Option (a) is the
correct answer as it follows exactly the same logic as implied in the question. ‗A soccer team will be
allowed to participate (X), if and only if 11 players (Y) are there.‘
108. Ans:(c)
The conclusion of the argument is that there will be no change in business despite using inferior
material. This is most strengthened by option (c) which states that most people do not pay attention
to quality as long as the book is readable.
Hence, (c)
109. Ans:(a)
C is the grandson of E
110. Ans:(d)
111. Ans:(d)
S > (Q,U) > P > T > R
112. Ans:(b)
I is a possibility as

II – All P are J is a possibility, so II does not follow. (as seen in above venn, if all P are J, then also
all statements agree, so it is a possibility)
III – All H are J is not a possibility, so III follows. (if all H are J, some F will be J which is not possible)
IV is not definite

LEGAL REASONING

113. Ans:(a)
Explanation: The answer is (a). This is due to the reason that it is based on happening and non-
happening of an event. Also, the parties agree to the condition that the rights will be enforced and
the obligations will be due on the happening of the contingency on the contracting of the valid
contract.
114. Ans:(c)
Explanation: The answer is (c). This is due to the reason that the contract is a wagering agreement.
The only interest that the parties have is monetary in nature and is not situated on the happening
and non-happening of the event.
115. Ans:(c)
Explanation: The answer is (c). This is due to the reason that as the promise was followed by a
consideration of donation of Rs 20lakh. Therefore, it is a lawful consideration which Badri would now
be liable to pay.
116. Ans:(c)
Explanation: The answer is (c). This is for the reason that if consent is obtained through
misrepresentation then it is not free in its nature. However, if we see the other elements which are
mistake of law and unilateral mistake, these do not involve a manipulation by the parties thereby
making the consent valid obtained under these.
117. Ans:(c)
Explanation: The answer is (c). This is because the agreement was collateral to a
wagering agreement. So, the wager does not impact the collateral contract making it valid and
allowing Easypay to claim the amount.
118. Ans:(b)
One of the qualifiers under the present special marriage act is one of the to be spouse has to be a
resident of the district, of the registrar in whose office they applied for a marriage. Since they both
eloped and have not been residing in the city since 30 days they don‘t qualify for the essentials
under SMA yet.
119. Ans:(d)
Other than the argument that their religion permits for objection, hence their religious freedom is not
hampered. They could still file a petition on grounds of violation of fundamental rights like privacy
and article 14. The paragraph states the grounds for challenging this law, it does not say if the
verdict is out or if such grounds would be upheld. So challenging the petition is their right, but its
consequence is undecided.
120. Ans:(b)
The fundamental argument against Section 6 is that it violates the right to privacy. Since SMA is not
a personal law under any religion, it doesn‘t have to abide by the religious protocol per se. However
such publication of their notice of marriage left for public scrutiny violates their right to privacy, their
right to choose a spouse and dignity under Article 21.
121. Ans:(d)
All of the above are valid arguments which are fundamental to one‘s right to life and personal liberty
and can be argued against the existing provisions other than those laid under article 14 of the
constitution.
122. Ans:(b)
Section 6 mandates that all notices received shall be entered in the marriage notice book and the
Marriage Officer shall publish a notice by affixing a copy thereof. The provisions also allow anyone to
submit objections to the marriage and empower the Marriage Officer to inquire into such objections.
123. Ans:(c)
This would be an invalid plea bargaining as it does not apply to offences with punishment 7 years or
less of prison time.
124. Ans:(b)
He does not have a choice as the plea bargaining once agreed to cannot be appealed off before any
other court of law.
125. Ans:(d)
Yes Raghav can confess and ask for a plea bargaining deal as his offence was punishable with less
than 7 years of imprisonment.
126. Ans:(d)
This would be an invalid plea bargaining as it does not apply to offences with punishment 7 years or
less of prison time.
127. Ans:(a)
It is applicable in respect of those offences for which punishment is up to a period of 7 years only.
128. Ans:(a)
It is bargaining done by the accused of a serious and severe offence, with the authority for a lighter
punishment in lieu of a full- fledged trial.
129. Ans:(b)
This is against the Article 20 of the Indian Constitution which does not permit retrospective operation
of laws which do not explicitly mention such usage.
130. Ans:(c)
Yes, as it would be operated prospectively only the date of effect is explicitly mentioned in the Act.
131. Ans:(d)
Art. 20 states that no penal offence can have a retrospective operation as it would be against the
fundamental rights.
132. Ans:(c)
It would be an unconstitutional law as it violates Article 20 of the Indian Constitution which is a
fundamental right.
133. Ans:(c)
Art 20 clearly states that people would only be penalized with the current law in force to the time
when the offence was committed.
134. Ans:(a)
There is no question of retrospective or prospective operation as the statute does not mention
anything but the issue of who would be deemed to be penalized according to the law in force.
135. Ans:(c)
The author in the above passage argues that the decision of Justice Arun Mishra to not recuse
himself from the Land Acquisition Case runs deeper to the questions of propriety and impropriety in
the judiciary. As far as the role of lawyers questioning the involvement of Justice Arun Mishra is
concerned, the author only explores the various possibilities involved.
136. Ans:(a)
In the paragraph it has been mentioned that the controversy regarding his involvement is because,
he admittedly had strong views stated in a Supreme Court judgment on the subject of the manner of
payment of compensation to be awarded in certain classes of cases under the old Land Acquisition
Act 1894. Therefore, there was an apprehension of conflict of interest and prejudice in deciding the
said matter.
137. Ans:(d)
As per the passage, Justice U.U. Lalit decided to recuse himself from the case after he was
reminded that he had appeared in an earlier criminal case concerning Babri as evidenced by the
orders showing his appearance in the criminal case. Just by bringing this to the judge‘s notice
and without any allegation of bias, he recused himself as a matter of propriety.
138. Ans:(d)
The passage says that there is no one thumb rule as to whether or not the lawyers should object to a
judge hearing the matter. However, after considering the different possibilities it concludes that if
there are bonafide apprehensions arising from the involvement of a judge in a given matter, the
same should be raised by the lawyers. It also puts in a word of caution to say that it should only be
limited to ‗bonafide‘ matters and not give a platform for unscrupulous shopping.
139. Ans:(b)
The question relates to one of the unusual cases where the judge shifts the onus to analyse the
propriety of the matter to the lawyers. The author argues that the lawyers usually when the onus is
put on them, let the judge decide the matter because they do not want to attack the judge for
unfairness, or prejudice their client‘s case, or anger the judge in respect of future appearances
before him or her.
140. Ans:(c)
Explanation: The husband has clearly withdrawn from the society of Mridula cutting off all the ties.
Thus, Mridula has the grounds to file for RCR)
141. Ans:(b)
Explanation: The only reason the petition is not maintainable is because there no valid marriage
between Atul and his mistress.)
142. Ans:(c)
Explanation: The petition will as there is reasonable excuse on part of the wife to withdraw from the
society of her husband. Moreover, they had consensually decided.)
143. Ans:(d)
Explanation: Refer to the para. Even option c may be true however the same cannot be inferred
from the passage)
144. Ans:(d)
Explanation: Attachment of property happens to the non-complying party. The option c talks about
the ‗complying party‘
145. Ans:(b)
(under Art.19 (6), the state reserves the right to modify its policies for the betterment of the public.
Hence cannot be held responsible, even for the delay.)
146. Ans:(d)
(a citizen whose occupation of a place is unlawful cannot claim fundamental right to carry on
business in such place since the fundamental rights cannot be availed in the justification of an
unlawful act or in preventing a statutory authority from lawfully discharging its statutory functions.)
147. Ans:(c)
(The right to carry on a business also includes the right to close it, at any time depending upon the
desire of the owner.)
148. Ans:(a)
(by not issuing the legitimate bills and skipping the sales tax, Karan is already in illegal trade
practice. Under Art. 19, a fundamental right cannot be claimed when an operation is illegal)
149. Ans:(a)
(Article 19 (1) (g) of Constitution of India provides Right to practice any profession or to carry on any
occupation, trade or business to all citizens.)
150. Ans:(b)
(Article 19 (1) (g) of Constitution of India provides Right to practice any profession or to carry on any
occupation, trade or business to all citizens.)
Legal Edge 15 Mock

Answers & Details

ENGLISH

1. Ans:(b)
Explanation: Options (a), (c) and (d) although true in the context of the passage do not represent
the complete summary of the passage which is only represented by option (b).
2. Ans:(a)
Explanation: Indictment refers to a formal statement accusing someone of a crime or a wrong,
therefore charge is closest in meaning to it.
3. Ans:(d)
Explanation: Among the committee‘s observations is the absence of insurance cover for many and
oversight on hospitals to ensure that patients are not turned away in a crisis such as COVID-19.
4. Ans:(c)
Explanation: The following lines in the passage direct us to the answer:
Acknowledging these distortions, and the inadequacy of existing legal frameworks, the panel has
called for an omnibus law that will curb profiteering during such crises and provide robust
cashless health insurance.
5. Ans:(b)
Explanation: The first paragraph refers to the imbalance between government and private
healthcare facilities in India which led to most of the citizens being at the mercy of private sector
hospitals. Therefore option (b) is the best answer.
6. Ans:(a)
Explanation: The only answer is that government is concerned with the wellbeing of its citizens but
the private sector on the other hand has no such responsibility. Therefore option (a) is the best
answer.
7. Ans:(c)
Explanation: Options (a), (b) and (d) although true in the context of the passage are incomplete in
their own respects. Option (c) represents the underlying message in the passage which is reiterated
throughout the passage.
8. Ans:(c)
Explanation: Abeyance refers to a state of suspension, therefore remission is the best response.
9. Ans:(d)
Explanation: The following lines in the passage direct us to the answer:
It had indicated that if things did not improve by April-end, the suspensions of certain
sections of the IBC for six months could be considered to prevent companies at large from
being forced into the insolvency process for a ‘force majeure’ default.
This indicates that this was something not in anyone‘s control and so unforeseeable that it would
lead to breach of a contract.
10. Ans:(a)
Explanation: The following lines in the passage direct us to the answer:
Stretching the IBC’s abeyance, for one, does not square up with the government’s
proclamations of a firm, V-shaped economic recovery.
11. Ans:(a)
Explanation: The following lines in the passage direct us to the answer:
One reason could be that the suspension also cuts off businesses’ ability to voluntarily enter
insolvency — for many, post-COVID-19 operations may not seem viable. Denying them an
exit route so as to cut their losses, while their assets shed value is a lose-lose proposition for
both borrower and lender. A more nuanced approach would have been better for banks,
businesses and the economy.
12. Ans:(b)
Explanation: The word ‗blanket‘ is used to describe the suspension, in that it is all encompassing
and it covers everything. This is a comparison as the suspension is said to be like a blanket, and
therefore it is a metaphor.
13. Ans:(c)
Explanation: The author wants to give make the word sound ironic, as the statements that are an
end result of the irony do not show that they have been discussed. They are more like unilateral
statements from the side of the Prime Minister.
14. Ans:(d)
Explanation: The word spurn is closest in meaning to boycott, since boycott refers to shunning or
avoiding something.
15. Ans:(a)
Explanation: The following lines in the passage direct us to the answer:
Farm leaders said Tuesday that they have asked British-Punjabi leaders to appeal to UK Prime
Minister Boris Johnson to not come to India as the chief guest for next year‘s Republic Day event till
the Centre repeals its ―black laws‖.
16. Ans:(d)
Explanation: The word scrap as used in the passage means to reject or remove, therefore the most
suitable antonym for it will be preserve.
17. Ans:(a)
Explanation: The Prime Minister of Britain is concerned about Indian politics given a huge
population of Indians in Britain and his support will strengthen the resolve of the farmers.
18. Ans:(d)
Explanation: The passage refers to the protest against the farmers‘ bill; therefore the answer should
incorporate that context in itself. Therefore option (d) is the best response. Option (c) comes close
but it cannot be concluded from the passage as it is a little far-fetched.
19. Ans:(c)
Explanation: The following lines in the passage lead us to the answer:
That moment, when the protests became a character in their own right, was missed by the BJP-led
government at the Centre. That‘s where it will need to go back to, to find a resolution. What is more,
as The Indian Express found in a week-long journey through villages and cities across Punjab‘s
three regions of Malwa, Doaba and Majha, talking to scores of farmers and their families, the image
of the un-seeing Centre is compounded by the spate of name-calling.
20. Ans:(a)
Explanation: The author wishes to highlight that the farmers are not carrying out a conspiracy but
actually just trying to voice their concerns and demands, and the unjust treatment meted out to them
by the government.
21. Ans:(d)
Explanation: The word refers to seizing something or taking command over something, therefore
seize is the best answer.
22. Ans:(c)
Explanation: The author wishes to convey the message that the centre is forcing its laws on the
farmers in Punjab, which is a state that is known to oppose central policies and dogmas.
23. Ans:(a)
Explanation: To resolve something is to settle and sort something out, therefore the most suitable
antonym would be question.
24. Ans:(a)
Explanation: The following lines in the passage direct us to the answer:
There must have been a moment, a point in the ongoing agitation led by Punjab farmers,
when a line was crossed. When men and women began talking less about the three Central farm
laws that sparked small dharnas at petrol pumps and toll plazas across the state and more about
those who started travelling to Delhi‘s doorstep to huddle together in the cold. That moment, when
the protests became a character in their own right, was missed by the BJP-led government at
the Centre. That’s where it will need to go back to, to find a resolution.
25. Ans:(d)
Explanation: The following line in the passage directs us to the answer:
As a hard year closes out in the US, and the electoral college voting seals Joe Biden‘s win, the oft-
ignored power that individuals possess in a democracy has once more been highlighted.
26. Ans:(b)
Explanation: The following lines in the passage direct us to the answer:
In this and other ways, officers and ―electors‖ across the US overcame pressures and did their duty.
In so doing, public servants in states like Georgia and Arizona probably destroyed their political
futures, at least for the short term. But they saved their country‘s prestige.
27. Ans:(d)
Explanation: The following lines in the passage direct us to the answer:
In India, too, doing our duty in the teeth of pressures and inducements is not a small matter, whether
we are public servants, or members elected to a local, state or national council, or just citizens.
28. Ans:(b)
Explanation: The following lines in the passage direct us to the answer:
Biden‘s dislodging of Trump is, therefore, not the former‘s personal feat. It is the outcome of duty
done by a great many.
29. Ans:(a)
Explanation: Brazen refers to someone who is bold, therefore brash is the best synonym of the
word.
30. Ans:(a)
Explanation: These ―electors‖ could have cheated and voted against their party, that is, against
their state‘s verdict.

GENERAL KNOWLEDGE/CURRENT AFFAIRS

31. Ans:(a)
Explanation: India has lashed out at Organisation of Islamic Cooperation (OIC) (formerly
Organization of the Islamic Conference) and rejected unwarranted references about Jammu &
Kashmir made in resolutions adopted at the 47th Council of Foreign Ministers (CFM) session.
India has said that OIC has ―no locus standi in matters strictly internal to India, including that of
Union Territory of Jammu and Kashmir‖.
32. Ans:(b)
Explanation: It is an international organization founded in 1969, consisting of 57 member states.
It is the second largest inter-governmental organization after the United Nations.
The organisation states that it is ―the collective voice of the Muslim world‖ and works to ―safeguard
and protect the interests of the Muslim world in the spirit of promoting international peace and
harmony ―.
33. Ans:(a)
Explanation: OIC was established on the 25th of September 1969 when an Islamic Conference
took place with 24 members. The catalyst that led to this conference was a fire on the Al-Aqsa
Mosque in Jerusalem.
The OIC was officially founded in 1972.
It is headquartered in Jeddah, Saudi Arabia.
It has 57 members spread over 4 continents.
Its official languages are Arabic, English and French.
The membership spans roughly 1.81 million people.
34. Ans:(c)
Explanation: What is the OIC‘s stand on Kashmir?
It has been generally supportive of Pakistan‘s stand on Kashmir and has issued statements
criticizing India.
These statements over the last three decades became an annual ritual, of little significance to India.
Last year, after India revoked Article 370 in Kashmir, Pakistan lobbied with the OIC for their
condemnation of the move.
To Pakistan‘s surprise, Saudi Arabia and the UAE — both top leaders among the Muslim countries
— issued nuanced statements, and were not as harshly critical of New Delhi as Islamabad had
hoped.
Since then, Islamabad has tried to rouse sentiments among the Islamic countries, but only a handful
of them — Turkey and Malaysia — publicly criticised India.
35. Ans:(d)
Explanation: Individually, India has good relations with almost all member nations. Ties with the
UAE and Saudi Arabia, especially, have looked up significantly in recent years.
The OIC includes two of India‘s close neighbours, Bangladesh and Maldives.
Indian diplomats say both countries privately admit they do not want to complicate their bilateral ties
with India on Kashmir but play along with OIC.
What lies ahead with OIC?
India now sees the duality of the OIC untenable, since many of these countries have good bilateral
ties and convey to India to ignore OIC statements.
But these countries sign off on the joint statements which are largely drafted by Pakistan.
South Block feels it is important to challenge the double-speak since Pakistan‘s campaign and
currency on the Kashmir issue has hardly any takers in the international community.
36. Ans:(d)
Explanation: At present, the Parliament (Prevention of Disqualification) Act, 1959, bars an MP, MLA
or an MLC from holding any office of profit under the central or state government unless it is
exempted.
However, it does not clearly define what constitutes an office of profit.
Legislators can face disqualification for holding such positions, which bring them financial or other
benefits
37. Ans:(a)
Explanation: The Supreme Court in Pradyut Bordoloi vs Swapan Roy (2001) outlined the four
broad principles for determining whether an office attracts the constitutional disqualification.
First, whether the government exercises control over appointment, removal and performance of the
functions of the office
Second, whether the office has any remuneration attached to it
Third, whether the body in which the office is held has government powers (releasing money,
allotment of land, granting licenses etc.).
Fourth, whether the office enables the holder to influence by way of patronage.
38. Ans:(c)
Explanation: A petition was filed seeking disqualification of Vijaysai Reddy as a member of the
Upper House of Parliament alleging that the post of special representative of the Andhra Pradesh
government at the Andhra Pradesh Bhawan being held by the YSR Congress Party MP was an
office of profit.
However, the EC had held that since no pecuniary gain was derived from the said office and Reddy
was not entitled to any other perks or remuneration other than enjoying the status of a ‗state guest‘
during his travels to Andhra Pradesh in connection with performance of his duties as special
representative, he did not incur disqualification under Article 102(1) (a) of the Constitution of India‖.
39. Ans:(a)
Explanation: The expression ―office of profit‖ has not been defined in the Constitution or in the
Representation of the People Act, 1951.
It is for the courts to explain the significance and meaning of this concept. Over the years, courts
have decided this issue in the context of specific factual situations.
But, articles 102 (1) and 191(1) which give effect to the concept of office of profit prescribe
restrictions at the central and state level on lawmakers accepting government positions.
40. Ans:(a)
Explanation: The officials of the law ministry are of the view that defining an office of profit could
lead to the filing of a number of cases with the Election Commission and the courts.
Also, once the definition is changed, one will also have to amend various provisions in the
Constitution including Article 102 (1) (a) and Article 191(1) (a) that deal with the office of profit.
It will have an overarching effect on all the other sections of the Constitution.
41. Ans:(c)
Explanation: A virtual meeting of G20 Education Ministers was held recently to discuss and share
experiences of member countries in the three identified areas of Education – Continuity in Times of
Crises, Early Childhood Education and Internationalization in Education.
The Kingdom of Saudi Arabia is hosting G-20 Leaders‘ Summit in 2020.
42. Ans:(a)
Explanation: Charles Yves Jean Ghislaine Michelborn 21 December 1975) is a Belgian politician
serving as President of the European Council since 2019. He previously served as Prime Minister of
Belgium between 2014 and 2019.
Michel became Minister of Development Cooperation in 2007, and remained in this position until
elected the leader of the Francophone liberal Reformist Movement (MR) in February 2011. He led
MR to the 2014 federal election, where they emerged as the third-largest party in the Chamber of
Representatives. After coalition negotiations, Michel was confirmed as Prime Minister of a MR-N-VA-
OVLD-CD&V government. He was sworn in on 11 October 2014, becoming the youngest Belgian
Prime Minister since 1845.
43. Ans:(c)
Explanation: The G20 is also working with international organisations to complement these efforts,
including multilateral development banks.
TOTAL FUNDS RAISED : G20 countries have contributed over $21 billion to support health systems
and the hunt for a vaccine and injected an unprecedented $11 trillion to safeguard the global
economy.
INDIA AS A GLOBAL PARTNER : Arabs see India as an important partner in this journey to ensure
a better tomorrow for all. India is carrying out ambitious reforms to modernise the country
44. Ans:(a)
Explanation: The Group of Seven is an intergovernmental organization consisting of Canada,
France, Germany, Italy, Japan, the United Kingdom and the United States. The heads of
government of the member states, as well as the representatives of the European Union, meet at the
annual G7 Summit.
45. Ans:(b)
Explanation: The Paris Agreement (French: l'accord de Paris) is an agreement within the United
Nations Framework Convention on Climate Change (UNFCCC), dealing with greenhouse-gas-
emissions mitigation, adaptation, and finance, signed in 2016. The agreement's language was
negotiated by representatives of 196 state parties at the 21st Conference of the Parties of the
UNFCCC in Le Bourget, near Paris, France, and adopted by consensus on 12 December 2015.As of
November 2020, all 196 members of the UNFCCC have signed the agreement and 188 remain party
to it. Of the eight countries which are not party to the law, the only significant emitters are Iran,
Turkey, and the United States
46. Ans:(a)
Explanation: Ethiopia is on the brink of a civil war. A domestic conflict is going on in the country‘s
northern Tigray region. Eritrea may be hardest hit, due to its proximity to Tigray.
If the violence and conflict spills outside Ethiopia‘s borders, it may potentially destabilize the Horn of
Africa region.
The US and China have several strategic military bases in that region, the closest being Djibouti. If
these military bases were to be impacted by the disturbances in any way, it may cause foreign
powers to get militarily involved in the region and the conflict.
47. Ans:(b)
Explanation: one of the TGE's main aims was to establish a devolution of political power down to
ethnic divisions, arguing that it was an essential move if the country wanted to lessen conflicts
across ethnic lines, ensure a fairer distribution of resources across the country, and increase
efficiency within the public sector. By 1994, the ethnically-based regions of Afar, Tigray, Somali,
Amhara, Benishangul-Gumuz, Oromia, Harari, Gambela, and the Southern Nations, Nationalities,
and Peoples Region (SNNPR), a diverse region inhabited by at least 40 different ethnic groups,
were formally established. Addis Ababa, defined as a federal district, and later Dire Dawa were
declared to be ethnically diverse chartered cities
48. Ans:(c)
Explanation: A ―full-scale humanitarian crisis‖ is unfolding in Ethiopia, the United Nations refugee
agency said , with more than 27,000 now having fled heavy fighting to Sudan.
Ethiopians fled the ongoing fighting in Tigray region, prepare to cross the Setit River on the Sudan-
Ethiopia border in Hamdait village in eastern Kassala state
The pace of the exodus, some 4,000 a day, may also indicate ―massive displacement‖ within the
Tigray region.
Ethiopia‘s prime minister warned that a deadline for rebel northern forces to lay down arms had
expired, paving the way for a final push on the Tigray region‘s capital in a two-week conflict
destabilising the Horn of Africa.
49. Ans:(b)
Explanation: The TPLF was founded in 1975 as a resistance army of the Tigrayan people against
the military dictatorship, which was called the Derg.
The leftist Derg, which was established in 1974, would change its title in 1987 but practically
remained in power till it was ousted by the armed rebels in 1991.
The TPLF played a crucial role in ousting the junta and they were welcomed as national heroes in
1991.
TPLF leader Meles Zenawi took over as the interim President in 1991 and became the first elected
Prime Minister in 1995.
He is largely seen as the architect of the country‘s ethno-federal system and remained in power till
2012.
50. Ans:(a)
Explanation: Ethiopia, in the Horn of Africa, is a rugged, landlocked country split by the Great Rift
Valley. With archaeological finds dating back more than 3 million years, it‘s a place of ancient
culture. Among its important sites are Lalibela with its rock-cut Christian churches from the 12th–
13th centuries. Aksum is the ruins of an ancient city with obelisks, tombs, castles and Our Lady Mary
of Zion church.
51. Ans:(d)
Explanation: former captain of the Indian hockey team and two-time Olympian Pargat Singh has
also decided to return the Padma Shri award he received in 1998. The former hockey player who is
now a Congress member will join former athletes in returning their prestigious awards to the Centre,
as a mark of protest against the three contentious agrarian laws and express solidarity with the
farmers' agitation. Singh, who led the Indian team at the 1992 and 1996 Olympics, is also a recipient
of the Arjuna Award which he received in 1989.
Apart from Pargat Singh, Arjuna awardees Sajjan Singh Cheema and Rajbir Kaur, Padma Shree
awardee Kartar Singh, Olympic medallist Gurmail Singh and former cricket coach Rajinder Singh
have also decided to return their awards back to the Union Government. The athletes have stated
that they will proceed to Delhi on December 5 and place their awards outside the Rashtrapati
Bhavan.
52. Ans:(c)
Explanation: All India Kisan Sangarsh Coordination Committee is a pan-Indian umbrella
organisation comprising 250 farmers' organisations. The committee was formed with the integration
of 130 farmers' organisation after the death of six farmers in alleged police firing in Mandsaur,
Madhya Pradesh in June 2017
53. Ans:(a)
Explanation: APMCs are regulated by states through their adoption of a Agriculture Produce
Marketing Regulation (APMR) Act.[1]
Until 2020, the first sale of agriculture produce could occur only at the market yards (mandis) of
APMC. However, the Farmers‘ Produce Trade and Commerce (Promotion and Facilitation) Act,
which came into effect in 2020, allowed farmers to sell outside APMC mandis in India.
54. Ans:(c)
Explanation: Much of government procurement at minimum support prices (MSP) — of paddy,
wheat and increasingly pulses, cotton, groundnut and mustard — happens in APMC mandis.
In a scenario where more and more trading moves out of the APMCs, these regulated market yards
will lose revenues.
They may not formally shut, but it would become like BSNL versus Jio.
And if the government stops buying, farmers will be left with only the big corporates to sell to.
55. Ans:(a)
Explanation: Arhatiyas, as they are called in Punjab and Haryana, are commission agents. They
have been in business for around six decades, since the passage of Agricultural Produce Markets
Act, 1961. Over the years, arhatiyas have come to dominate the agriculture marketing and credit
system, and even command substantial political and social clout.
56. Ans:(c)
Explanation: It must be noted here that the election commission is not concerned with the elections
to panchayats and muncipalities in the states. For this, the Constitution of India provides for a
separate State Election Commission
57. Ans:(c)
Explanation: Federal countries also include Austria, Belgium, Ethiopia, Germany, Malaysia, Mexico,
Nigeria, Pakistan, Switzerland, the United Arab Emirates, and Venezuela, among others. The
governmental structures and political processes found in these federal systems show great variety.
58. Ans:(c)
Explanation: Article 324 of the Constitution has made the following provisions with regard to the
composition of election commission: 1. The Election Commission shall consist of the chief election
commissioner and such number of other election commissioners, if any, as the president may from
time to time fix. 2. The appointment of the chief election commissioner and other election
commissioners shall be made by the president.
59. Ans:(c)
Explanation: Delhi - February 2020
Bihar - November 2020
60. Ans:(b)
There are 12 schedules under Indian constitution
61. Ans:(c)
Explanation: It is an international treaty for the conservation and wise use of wetlands.
It is named after the Iranian city of Ramsar, on the Caspian Sea, where the treaty was signed on 2
February 1971.
62. Ans:(b)

63. Ans:(b)
Explanation: The Lonar Lake in Maharashtra and Sur Sarovar, also known as Keetham lake, in
Agra, have been added to the list of recognised Ramsar sites.
64. Ans:(a)
Explanation: The Montreux Record was established by Recommendation of the Conference of the
Contracting Parties (1990).
Sites may be added to and removed from the Record only with the approval of the Contracting
Parties in which they lie.
Currently, two wetlands of India are in Montreux record: Keoladeo National Park (Rajasthan) and
Loktak Lake (Manipur).
Chilka lake (Odisha)was placed in the record but was later removed from it.
65. Ans:(d)
Explanation: Formally the Jammu and Kashmir State Land (Vesting of Ownership to the
Occupants) Act, 2001, it was passed by the then National Conference government led by Farooq
Abdullah to give ownership to people in possession of state land, with a cut-off of 1990, and against
payment as determined by the government. Since the aim was to generate resources for
hydroelectric power projects, it was called Roshni (Light) Act.
66. Ans:(c)

67. Ans:(a)
Explanation: In a later amendment, the Ghulam Nabi Azad government set the premium at 25% of
the market rate and the cut-off date at 2007. The government gave free ownership rights on
agricultural land to farmers occupying it, who only needed to pay Rs 100 per kanal of land as
documentation fee.
68. Ans:(d)
Explanation: In October 2018, then Governor Satya Pal Malik repealed the Roshni Act
prospectively. ―All pending proceedings under the Act shall stand cancelled immediately… any
action taken under the provisions of the repealed Act shall not be invalid,‖ his order read.

QUANTITATIVE TECHNIQUES

69. Ans:(b)
The first question itself requires us to calculate the visibility of all the brands across the stores.
Hence, before we start solving the questions, we can look at the visibility for each brand across the
stores.
Visibility of Astute brand
= 111450+ 48440+ 91280+ 30350+80480 ≈0.94
Visibility of Supreme brand
= 128450+ 55440+ 79280+ 111350+65480 ≈1.13
Visibility of Paramount brand
= 69450+ 116440+ 50280+ 101350+105480 ≈1.1
Visibility of Smash brand= 85450+ 137440+ 30280+ 60350+108480 ≈1.06
Visibility of ultimate brand= 57450+ 84440+ 30280+ 48350+122480 ≈0.82
Supreme.
70. Ans:(a)
The first question itself requires us to calculate the visibility of all the brands across the stores.
Hence, before we start solving the questions, we can look at the visibility for each brand across the
stores.
Visibility of Astute brand
= 111450+ 48440+ 91280+ 30350+80480 ≈0.94
Visibility of Supreme brand
= 128450+ 55440+ 79280+ 111350+65480 ≈1.13
Visibility of Paramount brand
= 69450+ 116440+ 50280+ 101350+105480 ≈1.1
Visibility of Smash brand= 85450+ 137440+ 30280+ 60350+108480 ≈1.06
Visibility of ultimate brand= 57450+ 84440+ 30280+ 48350+122480 ≈0.82
Astute in Store 4 has a visibility that is 30/350 < 0.1.
No other brand in any other store shows such low visibility
71. Ans:(a)
The first question itself requires us to calculate the visibility of all the brands across the stores.
Hence, before we start solving the questions, we can look at the visibility for each brand across the
stores.
Visibility of Astute brand
= 111450+ 48440+ 91280+ 30350+80480 ≈0.94
Visibility of Supreme brand
= 128450+ 55440+ 79280+ 111350+65480 ≈1.13
Visibility of Paramount brand
= 69450+ 116440+ 50280+ 101350+105480 ≈1.1
Visibility of Smash brand= 85450+ 137440+ 30280+ 60350+108480 ≈1.06
Visibility of ultimate brand= 57450+ 84440+ 30280+ 48350+122480 ≈0.82
Total number of T-Shirts of size M = 22100 x 2000 = 440
Total number of T-shirts of size M in Stores 1, 2, and
5= 10% of 1370= 137
Remaining T-shirts of size M to be distributed across
Stores 3 and 4 = 440 - 137 = 303.
To minimize the size M T-shirts in Store 4, we need to maximize size M T-shirts in Store 3. The
total number of T-shirts in Store 3 are 280 and hence this is the maximum number of T-shirts of size
M, that could be in Store 3.
Hence, the minimum possible number of T-shirts in Store 4 = 303 - 280 = 23.
72. Ans:(b)
The first question itself requires us to calculate the visibility of all the brands across the stores.
Hence, before we start solving the questions, we can look at the visibility for each brand across the
stores.
Visibility of Astute brand
= 111450+ 48440+ 91280+ 30350+80480 ≈0.94
Visibility of Supreme brand
= 128450+ 55440+ 79280+ 111350+65480 ≈1.13
Visibility of Paramount brand
= 69450+ 116440+ 50280+ 101350+105480 ≈1.1
Visibility of Smash brand= 85450+ 137440+ 30280+ 60350+108480 ≈1.06
Visibility of ultimate brand= 57450+ 84440+ 30280+ 48350+122480 ≈0.82
Share of Supreme brand in all stores together = 128+55+79+111+65 2000 × 100 ≈ 22%
73. Ans:(d)
The first question itself requires us to calculate the visibility of all the brands across the stores.
Hence, before we start solving the questions, we can look at the visibility for each brand across the
stores.
Visibility of Astute brand
= 111450+ 48440+ 91280+ 30350+80480 ≈0.94
Visibility of Supreme brand
= 128450+ 55440+ 79280+ 111350+65480 ≈1.13
Visibility of Paramount brand
= 69450+ 116440+ 50280+ 101350+105480 ≈1.1
Visibility of Smash brand= 85450+ 137440+ 30280+ 60350+108480 ≈1.06
Visibility of ultimate brand= 57450+ 84440+ 30280+ 48350+122480 ≈0.82
Required percentage = 420-341 341 x 100 = 23.16%.
74. Ans:(a)
Required ratio 800+1200+500+1400+700+2500+600+100+1100+500
1800+500+400+2000+1200+1600+1000+1500+700+1400
= 10300 : 12100 = 1:1.2.
75. Ans:(a)
By checking the options we can see that the difference between DI and FI is highest for Technology
sector.
76. Ans:(a)
Since, the denominator while calculating the average DI is the same for all the four years, we can
get the answer simply by adding the column for DI for the four years given in the options. Totals for
2009,2010, 2011 and 2013 are respectively 12400, 8100, 14500 and 16000. Hence, the average
would be the highest for 2013.
77. Ans:(a)
Consumer Cyclical at 4700 is the second lowest after Energy at 3700.
78. Ans:(b)
66300 : 78400 2: 2.36
79. Ans:(c)
We can easily solve this problem by checking the options. Average annual expenses for A
= 105+185+100+120+1105 = 124.
Average annual expenses for B = 135+115+130+125+1355 =128
Average annual expenses for C = 165+155+190+100+1005 = 142
Average annual expenses are the maximum for company C.
80. Ans:(b)
By checking the options we can see that average annual revenue is the maximum for the year 2012.
81. Ans:(c)
Revenue of company C in 2012 = 180
Revenue of company C in 2015 = 120
Percentage decline in the revenue = 180-120180 ×100=33.33%
82. Ans:(c)
Average revenue of A in 2011, 2012, 2013 = 120+135+1653 = 140
Average revenue of B in 2013, 2014, 2015 = 165+150+1653 = 160
Difference = (160 - 140) × 1000 = 20,000.

LOGICAL REASONING

83. Ans:(c)
Explanations(c):-The author wants to say that pre-CAA India had been a messy democracy, but
still a democracy, which have been brought to its path again by the Constitution. If we go by that
logic then option (a) does not weaken the author‘s view. The reason is that the Rohingyas were
immigrants and not citizens (the author has mentioned that she is concerned about the Indian
identity, meaning citizenship), so this point neither strengthens nor weaken the author‘s take. Option
(b) actually strengthens the author‘s view, as we see the government acting upon its citizens (Indian-
Chinese) in a wrongful way, but then the SC deciding and judging against the government decision.
This is a classic example of a messy democracy being saved by a pillar of the Constitution. Thus
option (c) is the answer.
84. Ans:(d)
Explanations(d):-The author says ―Unlike a pure theocracy where the political head is also the
religious head, or at least the religious leader assumes a leading role in the state, a
constitutional theocracy operates within the bounds of the constitution.” This means there has
to be a façade of constitutionalism residing on the body of a communal and theocratic structure of
administration. Options (a) and (b) are too much in the open of a religious kind. Option (c) is
something that the author has herself denied. Option (d) fits the description properly. Thus, the
answer is option (d).
85. Ans:(b)
Explanations(b):-The second way as has been described by the author is more subtle and less
transparent than creating or amending statutes, which is the first way. Thus, less transparent
would mean translucent. Thus, option (b) is correct.
86. Ans:(c)
Explanations(c):-The option has to be such that there has to be oxymoronic words and it should
describe a chaotic and unjust order. Option (a) doesn‘t have oxymoronic words, same as option (d).
Thus, they are not the answer. However, option (b) and (c) have oxymorons, but option (b) doesn‘t
denote a chaotic political culture, but rather an economic one. Option (c) has oxymoronic words and
they refer to a chaotic order.
Thus, the answer is option (c).
87. Ans:(b)
Explanations(b):- Option (a) is incorrect as the author does mention that India has been a messy
democracy plagued by notions of race, caste, creed, religion etc, however, our secular-democratic
ethos have always been rescued by the Constitution. Hence, option (a) is not the answer.
Option (b) is correct as the author does mention – “it (CAA) elliptically and insidiously does is to
cast a shroud of doubt on many legal, valid Muslim citizens of India”. Thus, option (b) is the
answer.
Option (c) is incorrect as the author says that India might become a constitutional theocracy. This is
because CAA doesn‘t favour only a specific religion but it biases against one specific religion –
Islam.
Option (d) is incorrect as CAA has in a way included an exclusionary tactic for the inclusion of
people into Indian citizenship. It doesn‘t favour the majoritarian religion only.
88. Ans:(a)
Explanations(a):-The first way of becoming a constitutional theocracy is to have the force of statute
and laws being passed to achieve it. Option (a) fits the description perfectly. Option (b), (c) and (d)
pursues the second way – the use of non-statutory and less political or non-political ways to become
a constitutional theocracy.
Thus, the answer is option (a)
89. Ans:(d)
Explanations(d):-Supporting Ideas; The passage ascribes to Romila Thapar the claim that there
was ―a surge of educational opportunities for women in India‖ after the Indian Independence, and
that this surge resulted from a new ideology of ―republic motherhood.‖ According to the passage,
Thapar argued that the nation's leaders advocated education for women to equip them, in their
family role, to raise politically virtuous sons.
A-The passage attributes no claim to Thapar concerning a demand by women for education.
B-The passage attributes no claim to Thapar concerning a new ideology calling for equality between
women and men.
C-Thapar's argument as represented in the passage did not claim that an increase in education
opportunities for women resulted from a belief that such an increase was required for successful
reform of the Indian educational system.
D-Correct. According to the passage, Thapar argued that educational opportunities for women
increased because the nation's leaders believed that successful democratic government would
require that women raise politically virtuous sons within their families, and that women could do so
only if they had access to education themselves.
90. Ans:(d)
Explanations(d):- Supporting Ideas; According to the passage, Guha's work was a ―notable
exception‖ as contrast to the work of other educational historians, who ―barely mentioned women
and girls.‖
A-Other historians prior to Guha's 1979 work may have used newspaper advertisements as
evidence, but the passage provides no information as to whether this was so.
B-The passage is silent as to whether educational historians besides Guha concentrated on the
period before the Indian Independence.
C-The passage does not provide information as to the extent to which either Guha or other
historians focused on educational attitudes as opposed to educational practices.
D-Correct. As stated above, the passage describes Guha's work as a ―notable exception,‖ i.e.,
atypical, with respect to his discussion of education for girls.
91. Ans:(a)
Explanations(a):- Application; According to the passage, Guha characterized ―An Essay on
Woman‖ (1943) as reflecting a shift in view, and the context indicates that this shift concerned new
attitudes that accompanied increased opportunities after 1940 for girls to become educated women.
A-Correct. Based on the passage, this is a claim with which Guha would likely have agreed.
B-The passage represents Guha as claiming that ―An Essay on Woman‖ reflected changes that had
already occurred around 1940. The passage does not indicate whether Guha would have agreed
with this claim about a persuasive effect on educators.
C-Nothing in the passage represents Guha as thinking that ―An Essay on Woman‖ had ideas about
women's education that did not come to fruition until after the Indian Independence. The tenor of
Guha's thinking, as the passage represents it, is that the essay reflected changes already occurring.
D-The passage indicates that Guha characterizes ―An Essay on Woman‖ as ―reflecting‖ a view that
had already gained some currency; so it is unlikely that Guha saw the essay as offering any highly
original arguments in favor of women's education.
92. Ans:(b)
Explanations(b):- Evaluation; The passage represents Thapar as claiming that the Indian
Independence led to a surge in educational opportunities for women because the nation's leaders
believed women needed to be educated if they were to raise politically virtuous sons. Guha,
however, is represented as claiming that there was a significant increase in such opportunities and
significant advocacy for women's education well before the Independence.
A-The passage does not represent either Thapar or Guha as addressing the extent to which women
were interested in pursuing educational opportunities in the twentieth century.
B-Correct. The passage attributes to Guha the view that ―practical education for females had many
advocates before the Independence,‖ notably in the 1940s, and that the Independence at most
accelerated an earlier trend of changing attitudes. This is contrary to the views attributed to Thapar.
C-The passage gives no information as to whether Thapar or Guha addresses this issue, nor does it
discuss to what extent, if any, such resistance may have occurred.
D-The passage indicates that Thapar and Guha hold that there was a change in attitudes toward
women's educational opportunities during the twentieth century, disagreeing, however, as to whether
the most significant change occurred before or after the Independence.
93. Ans:(c)
Explanations(c):-Supporting Ideas; The passage attributes to Thapar the claim that the
nation's leaders believed a virtuous citizenry was essential to the success of the nation's republican
form of government, and that women would play a primary role in raising future citizens who would
be politically virtuous.
A-According to the passage, Thapar indicates that the nation's leaders believed religious places and
schools, as well as families, would work to imbue political virtue, though they emphasized the crucial
role of families.
B-Thapar argues that the educational system underwent reform in the sense that educational
opportunities for women increased; but does not claim that schools or families would change focus
to imbue girls with political virtue.
C-Correct. Thapar argues that political leaders emphasized the family as the primary means by
which future citizens would be imbued with political virtue.
D-Thapar does not claim the nation's leaders proposed that the family would take over functions
previously fulfilled by schools and religious places.
94. Ans:(c)
Explanations(c):- Choice ‗C‘ is the main claim in the passage. This claim is supported by the entire
passage which acts as a premise. (The Iraqi example of Yazidis.)
95. Ans:(a)
Explanations(a):-The passage essentially states that social distancing will result in discrimination,
while option 'A' gives data to the contrary. Option 'B' strengthens the author's argument. The other
options neither strengthen nor weaken the main argument.
96. Ans:(d)
Explanations(d):-Total number of cases cannot be determined.
Hence, 'D'
97. Ans:(b)
Explanations(b):- Option 'B' is correct as the passage is argumentative in nature. The first
paragraph builds on the claim "unfettered promotion of social distancing can reinforce existing social
prejudices driven by different forms of social exclusion"
The second paragraph acts as a premise to support this claim.
Hence, 'B'
98. Ans:(c)
Explanations(c):- Option 'C' is correct as the Iraq example gives us the disadvantage of social
distancing. Hence, this is a valid conclusion.
99. Ans:(d)
Explanations(d):- Rationale- The reason for minister‘s recommendation is most likely because of an
incident or incidents which might have occurred in the prisons due to snapping of tensions among
the prisoners. This reason is best captured in option (D). Therefore, option (D) is the correct answer.
100. Ans:(a)
Explanations(a):- Rationale- The minister in charge of prisons while making a recommendation to
the Government about an amnesty policy has assumed that this policy will lead to a decrease of
inmates inside jails and then the tension will not snap inside prisons. Therefore, option (A) is the
correct answer.
101. Ans:(b)
Explanations(b):- Rationale- In the above passage, while recommending that minor crimes should
be decriminalized, they assumed that decriminalization of minor crimes will lead to less number of
crimes and then the prisons will not get overcrowded anymore. Therefore, option (B) is the correct
answer.
102. Ans:(a)
Explanations(a):- Rationale- Option (B) is irrelevant. Option (C) is further strengthening the solution
of minister and attorney general. Option (A) is the correct answer because it provides that
decriminalization will lead to more offences and then prisons will be overcrowded, therefore, option
(A) is the correct answer.
103. Ans:(c)
Explanations(c):- The introduction of reservation for women in the top management was to
encourage them towards excellence and create gender equality. This purpose of the policy is
defeated if it encourages poor performance as is evident by the drop in the performance of women
managers. Thus, choice C is evidence for the flawed reasoning of the company.
Choice ‗B‘ is inappropriate because it claims that there existed a bias against women in the company
and this is exactly what the policy aimed to eradicate.
Choice ‗A‘ doesn‘t directly disprove the argument because it provides no evidence against the policy.
Also, the policy would effectively take the percentage even higher than 33% and therefore would
provide motivation all the same.
Choice ‗D‘ validates the policy introduced by the company and is thus, inappropriate.
Thus, choice ‗C‘ is the correct answer.
104. Ans:(a)
Explanations(a):- Rationale- Option (D) is irrelevant. Option (B) and (C) cannot be inferred on the
basis of information provided in the above passage. Option (A) is the correct answer because it can
be best inferred from the above passage, on the basis of information provided in the above passage.
105. Ans:(c)
Explanations(c):- Rationale- In the above passage it is provided that ―If marijuana were legalized in
states, the states could start assessing tax on the drug, increasing state revenues.‖ Therefore, it can
be best concluded from the arguments of Marijuana advocate that Legal marijuana could mean a big
push for state economies and big bucks for the states. Hence, option (C) is the correct answer.
106. Ans:(b)
Explanations(b):- Rationale- The Solicitor General points out data from other countries, suggesting
that things will not as rosy as the marijuana advocate suggests. Therefore, the correct answer is
option (B). The arguments of Solicitor General does not give any direct proof, it only provides for the
analogy of different countries and state.
107. Ans:(a)
Explanations(a):- Rationale- One of the argument of the Marijuana advocate is that ―Since sales
would be legal, the criminal culture supporting the drug would vanish; as crimes cease, the state
would save money on fighting crime.‖ Option (A) is further strengthening the above argument of the
MA. Therefore, option (A) is the correct answer. Option (B) and (C) are irrelevant to the arguments of
Marijuana advocate
108. Ans:(d)
105 is correct number.
Explanations
10 = 4 × 2 + 2
23 = 10 × 2 + 3
50 = 23 × 2 + 4
(105)104 = 50 × 2 + 5
216 = 105 × 2 + 6
439 = 216 × 2 + 7
Hence, 104 is the wrong number in the series and 105 should be in place of 104.
109. Ans:(c)
Explanation
Applying condition 2 the relation represented below

Which represent I is nephew of Q


110. Ans:(b)
Explanations
One man is facing south, thus one lady will be facing north (as persons opposite to each other are of
opposite sex.) Further, as no lady is facing east, another lady will face west and another man will
face east. Thus, the ladies are facing north & west.
111. Ans:(c)
Explanations
See video solution.
112. Ans:(a)
Explanations
See video solution.

LEGAL REASONING

113. Ans:(d)
Explanations: Promissory estoppel is not applicable as the relief is granted only when the person
has acted upon the promise. Furthermore, it cannot be the sole reason for a cause of action.
114. Ans:(d)
Explanations: Promissory Estoppel is applicable in the present case as the customer acted upon
the promise of the vendor and bought the jeans. The vendor cannot go back on his promise.
Promissory Estoppel can be used as a defence here.
115. Ans:(c)
Explanations: Self-explanatory
116. Ans:(d)
Explanations: The doctrine of legitimate expectation is not applicable as it is only applicable to
public bodies.
117. Ans:(d)
Explanations: The retrospective enforcement would be allowed as the doctrine of legitimate
expectation can be used as sword and not just a shield. Even though option (a) is also correct,
option (d) is more ‘likely’.
118. Ans:(b)
Explanations: The lady acted upon a promise made to her by a public body. In such cases the
doctrine of legitimate expectation is applicable.
119. Ans:(d)
Explanations: Self explanatory.
120. Ans:(b)
Explanations: Even though the facts of both the questions are same, the former refers to the
likelihood while the latter refers to conclusive truth. It is likely that the taunts were not issued by the
relatives but the society; however it is not the conclusive truth. The society may include the relatives
as well.
121. Ans:(a)
Explanations: The offence of dowry death requires death through unnatural means.
122. Ans:(d)
Explanations: The passage does not mention the burden of proof but only states that the
presumption is in favour of the prosecution.
123. Ans:(b)
Explanations: If, soon before the death, the deceased wife must have been at the receiving end of
cruelty or harassment, on account of demand for dowry; it is described as dowry death.
124. Ans:(d)
Explanations: The period after which death has occurred is not mentioned, ruling out option (a) and
(b). Further, the passage only talks about presumption of guilt and not the burden of proof.
125. Ans:(a)
Explanations: Answer is (A) it is very clear from the passage that the court of Singapore has yet to
accept or decriminalize homosexual activity. According to the prevailing law anyone can be
prosecuted under §377A for indulging in the homosexual activity, therefore the facts and law stands
beside prosecution.
126. Ans:(d)
Explanations: Answer is (D) because the current case is in respect to the law of Singapore which
very specifically held that sexual orientation is not their concern but the act of indulging in ‗gross
indecency‘ as comprehended by the society of Singapore, best recourse is to challenge the
judgement and decriminalize §377A. Not (a) and (b) because they deal with the law in India, option
(c) is eliminated because it is not an advice.
127. Ans:(c)
Explanations: Answer is (C) homosexual act is punishable, sodomy with a person of opposite
gender is not a punishable offence, even thou the same act is considered gross indecency. Not (a)
and (d) because neither is mensrea being talked about nor is the act of sodomy punishable, at least
wrt. the passage. Not (b) because it is not a homosexual act.
128. Ans:(b)
Explanations: Answer is (B) most important words in the question are unverified evidence and
decide the recourse, because Paul produced unverified document giving an ultimatum is not the
right recourse whereas option (b) talks about prosecution. Not (a) because factually wrong any
sexual activity b/w 2 people of same sex is considered homosexual activity. Not (c) because it is an
ultimatum. Not (d) because the question asks to decide the recourse whereas (d) only states opinion
and is not a decision.
129. Ans:(b)
Explanations: As given in the passage, Section 25 of the lays a bar on proving any confessions
made to a police officer against a person accused of an offence, and hence Bob‘s statement,
which is of the nature of a confession, and is made to the police officer, cannot be proved]
130. Ans:(c)
Explanations: Section 27 lifts the ban on proving a confession made to a police officer, as far as is
related to proving a fact discovered distinctly related to the fact in issue. The location of the murder
weapon revealed by Bob helped the police discover it, and hence it can be proved]
131. Ans:(c)
Explanations: As given in the passage, Section 25 of the lays a bar on proving any confessions
made to a police officer against a person accused of an offence. In the present case, the police
merely overheard the conversation between David and the journalist, and the statements which
amount to a confession cannot be said to have been made to a police officer, but to the
journalist. Hence, this should be allowed to be proved.]
132. Ans:(c)
Explanations: As given in the passage, Section 25 of the lays a bar on proving any confessions
made to a police officer, against a person accused of an offence. In the present case, Raju is not
under trial for Domestic Violence, but is the defendant in a divorce suit. Hence the bar under section
25 does not apply. (Option is self-explanatory)]
133. Ans:(b)
Explanation: ‗the lawyers to be allowed the immunity to hold representations against the directions
of the state in a completely free manner‘ holds true as both the other measures only strengthen the
existing directive of the state of Arizona to compel the lawyers to execute the death sentence of their
clients. Whereas option ‗b‘ stipulates a narrative which goes against a state. Hence in light of the
passage, the measure in option ‗b‘ is something that the state of Arizona shall not seek to
implement.
134. Ans:(d)
Explanation: Compelling the lawyers to act on the directive of the state which imply acting partiality
in favour of the state‘s amounts to professional misconduct on the part of lawyers. Hold true as the
passage clearly states in paragraph three that the direction issued by the government prevents a
lawyer to act independently and impartially in providing legal services but the directive is asking the
lawyer to be a slave to the aims of government, hence resulting in professional misconduct.
135. Ans:(a)
Explanation: ‗For importing life-threatening drugs to the US for the purpose of killing their clients‘
has been clearly laid down as a potential reason which might make the lawyers of the state of
Arizona land up in jail.
136. Ans:(b)
Explanation: ‗Lawyers not being able to fulfil their professional ethics and still functioning on the
state orders which fall contrary to their duties‘ shall still remain as the author‘s concern as the state,
even after implementing such a provision be requiring the lawyers to act on its directions,
jeopardizing the professional ethics of the lawyers.
137. Ans:(a)
Explanation: ‗the author asks the readers to ponder over the recent changes that the Arizonian
state has imposed. And in addition to this analyse as to whether through these changes the lawyers
would lose to profess their professional duties at the cost of benefit of the state.‘ Because by posing
such a question the author implores the readers to understand the seriousness of how the state is
taking away the professional ethics of the lawyers.
138. Ans:(d)
Explanation: the author mentions his concern about the compromise of accused‘s rights that has
been effectuated by the state legislation amidst the extraordinary situation. Both the options a & b
have been talked about by the author in the 4th and the 5th paragraph of the passage respectively.
139. Ans:(d)
Explanation: both the options mentioned in a & b are agreed upon by the author in the passage.
Therefore none of the options fits correctly.
140. Ans:(d)
Explanation: both ‗all the sexual assault victims in Australia used to provide their testimony on
virtual courts‘ and ‗only the child sexual assault victims were allowed to give pre-recorded testimony‘
reflect the state of affairs before the implementation of the current emergency legislation.
141. Ans:(b)
Explanation: only option ‗b‘, ‗she shall not have to face the potential abuse from the accused in the
courtroom thus not resulting in any form of additional trauma‘ fits correctly as the author in the last
paragraph states, in cases of child sex abuse, pre-recorded evidence has been found to reduce the
trauma and abuse such victims may endure if required to give evidence in court.
142. Ans:(c)
Explanation: third paragraph of the passage clearly states that the primary objective of the
legislation NSW was to implement social distancing and hence option ‗c‘, ‗to adopt an approach
which ensures social distancing ‗holds true.
143. Ans:(b)
Explanations: Ravish only had the permission to admit or remove members from the group, he was
not involved in any manner with regard to the uploading of content and neither does he have the
right to authenticate content on the group. Thus, he is no responsible for the infringement of
Mr. Goenka‘s copyright.
144. Ans:(d)
Explanations (d): Manish permitted Jatin to publish malicious details about other members without
supporting evidence, knowing it was defamation. Thus he was aware of an illegal act and permitted
the same.
145. Ans:(b)
Explanations (b): Knowingly adding an ultra vires act on their Platform has led to their losing the
intermediary immunity they could claim under Section 79 of IT Act,2000.
146. Ans:(d)
Explanations (d): In the present case Insta as an intermediary had the liability to take corrective
action upon receiving complaint of the infringement of copyright. Upon failing to do so Insta cannot
claim safe harbour under Section 79 of the IT Act,2000 as explain in the paragraph above.
147. Ans:(c)
Explanations: [c] Option (c) is the only one that takes into account the objective of the measure and
then proceeds to classify it as not overly-restrictive. As stated in the passage above, the correctness
of the decision does not matter (option (a) and (b) are ruled out). The reasoning needs to have taken
into account the two conditions. The conclusion does not matter if the test has been appropriately
followed.
148. Ans:(b)
Explanations: [b] Option (b) considers the purpose of the measure imposed and then classifies
whether the measure was restrictive to achieve that purpose. Therefore, this is a possible reasoning
that can be taken up by the court.
149. Ans:(c)
Explanations: [c] Option (c) analyses how the measure taken upon by the state, overly-restricts the
fundamental rights of the people. Thus, it could be a possible reasoning taken up by the court to
apply the proportionality test.
150. Ans:(d)
Explanations: [d] (Even though both options (b) & (c) lead to different conclusions, the reasoning
adopted by both of them follow the two-prong test of proportionality and hence, right.)
Legal Edge 16 Mock

Answers & Details

ENGLISH

1. Ans:(a)
A coup is an overthrow of the democratically elected government in a State usually by means of
military force. In Nepal, it was not done through military force but after adhering to the provisions of
the Constitution. Therefore it has been described as a “constitutional coup”.
2. Ans:(b)
The following line from the passage directs us to the answer:
Though it was decided that power will be shared between leaders of the two parties that merged, Oli
was reluctant.
3. Ans:(d)
Although there are several references in the passage that indicate Oli is not willing to share his
power and is engaged in a game of one-upmanship but it cannot be concluded that he is tyrannical
and does things only for his benefit. It is a vague and an exaggerated statement. Therefore option
(d) is the best response.
4. Ans:(a)
Option (a) is the only statement that summarises what his stance should have been according to the
author. Instead of ordering fresh elections, Oli should have accommodated his rivals. The other
options are also true as to what Oli could have done after coming to power but the questions is very
specific and asks what he should have done instead of dismissing the House of Representatives.
5. Ans:(a)
A clean slate is an idiom, it means taking a fresh start onto something.
6. Ans:(c)
The word internecine refers to something marked by slaughter or destruction. Therefore the most
suitable antonym is „restorative‟.
7. Ans:(c)
“One simple arithmetic” is a reference to basic underlying fact and therefore option (c) is the best
answer.
8. Ans:(d)
The author wants to cite an example of how a greater power like Germany was defeated when
Britain and France came together (which was also an unlikely combination at that time). He
compares it to defeating BJP in Bengal which will require two parties to come together. Therefore
option (d) is the best answer.
9. Ans:(b)
Behemoth refers to a monstrous creature therefore option (b) is the best answer.
10. Ans:(d)
The following lines from the passage direct us to the answer:
An alliance will also cut down on cost, disincentivise defection and allegations of
dynasticism in TMC. It would also minimise the political violence that Bengal routinely
witnesses before polls.
11. Ans:(a)
The following lines in the passage direct us to the answer:
Third, a coalition in power will keep Mamata’s monocracy in check, end CPM’s long isolation
from governance; and create a pool of political progeny for the future.
12. Ans:(a)
Turning the tables refers to a situation where one puts himself in a position of advantage especially
after he was perceived to be in a disadvantageous position.
13. Ans:(b)
The author begins the passage with two well-known maxims but they are both contradictory to each
other. Therefore option (b) is the best answer.
14. Ans:(d)
It cannot be concluded, just by her actions of joining the TMC that she wanted to divorce her
husband. That is something caused in the course of her actions.
15. Ans:(c)
As is cited in the passage that a couple is getting divorced because one of them decided to switch
political parties, therefore it can be concluded that switching political parties has greater implications
than merely switching jobs.
16. Ans:(a)
The author draws out one similarity between politics and Bollywood but it cannot be concluded that
both are inherently similar. Therefore option (a) is the correct answer.
17. Ans:(d)
A feud refers to a fight or a quarrel and therefore option (d) is the closest in meaning to it.
18. Ans:(b)
Ruling the roost means to be in complete control over something.
Drawing a line means to limit something.
To stop at the water‟s edge means to limit something within a demarcated area.
19. Ans:(a)
The following lines in the passage direct us to the answer:
After all, how could the PLA mobilise and deploy a significant number of troops from its interior for
ingress across a heavily militarised border without a timely Indian counter-response? All signs point
to an intelligence failure.
20. Ans:(a)
Better information is concerned with decision making and therefore option (a) is the best answer.
21. Ans:(d)
The author states that intelligence failures are seldom the result of shortcomings in tasking or data
collection to prove his point that merely curing operational and organisational infirmities do not help
in solving the intelligence failure issue. Therefore it can be inferred that shortcomings in data
collection are part of operational infirmities.
22. Ans:(a)
Intelligence failure occurred at the analysis stage and the committee might not have helped with that.
Therefore option (a) is the best answer.
23. Ans:(a)
To mobilise is to summon or marshal, therefore option (a) is the best answer.
24. Ans:(a)
Rather, they are mainly produced due to mistaken analysis on behalf of intelligence analysts or
political elites.
25. Ans:(a)
India had not acquired a secular colour just after independence, given the horrific partition that
happened on communal lines. Therefore Maulana Azad felt that his work would be more appreciated
after Hindus and Muslims had learnt to live with each other. .
26. Ans:(b)
The following lines in the passage direct us to the answer:
While a Muslim chief minister is no longer even a distant dream, the “secular” intellectuals of our
country are now complaining that Owaisi even contesting the elections in Bihar led to the
polarisation of the electorate. It is a strange animal, this electorate of their imagination.
27. Ans:(a)
The sentence is only to point out the hypocrisy as secular Indians do not seem to notice all the anti-
religious and factionist efforts being taken by the government or other fundamentalist organisations,
but they only say that the electorate will be polarised when a Muslim contests elections.
28. Ans:(a)
The following lines in the passage direct us to the answer:
Maulana Azad, however, made a gross error of judgment in his assessment. India would still not be
ready for this insight 30 years after his death.Eighty-three years after the election of the first
autonomous provincial government in Bihar, nothing seems to have changed. While a Muslim chief
minister is no longer even a distant dream, the “secular” intellectuals of our country are now
complaining that Owaisi even contesting the elections in Bihar led to the polarisation of the
electorate.
29. Ans:(a)
Slaughter means to take someone‟s life, therefore butcher is the closest in meaning to it.
30. Ans:(a)
The correct antonym is truth. Option (d) seems tempting but „reality‟ would be an antonym for
imagination and not „real‟.
GENERAL KNOWLEDGE/CURRENT AFFAIRS

31. Ans:(a)
Explanations: The four global GNSS systems are – GPS (US), GLONASS (Russia), Galileo (EU),
BeiDou (China). Additionally, there are two regional systems – QZSS (Japan) and IRNSS or NavIC
(India). The operational name of IRNSS is NavIC.
It covers India and 1500 km of region covering India. It has eight constellations in its orbit.
Antrix Corporation Limited is a government-owned company under the administrative control of the
Department of Space. It was incorporated in September 1992 commercially exploiting space
products of ISRO, providing technical consultancy services and transferring technologies to industry.
32. Ans:(c)
Explanations: The four global GNSS systems are – GPS (US), GLONASS (Russia), Galileo (EU),
BeiDou (China). Additionally, there are two regional systems – QZSS (Japan) and IRNSS or NavIC
(India). The operational name of IRNSS is NavIC.
It covers India and 1500 km of region covering India. It has eight constellations in its orbit.
Antrix Corporation Limited is a government-owned company under the administrative control of the
Department of Space. It was incorporated in September 1992 commercially exploiting space
products of ISRO, providing technical consultancy services and transferring technologies to industry.
33. Ans:(c)
Explanations: The four global GNSS systems are – GPS (US), GLONASS (Russia), Galileo (EU),
BeiDou (China). Additionally, there are two regional systems – QZSS (Japan) and IRNSS or NavIC
(India). The operational name of IRNSS is NavIC.
It covers India and 1500 km of region covering India. It has eight constellations in its orbit.
Antrix Corporation Limited is a government-owned company under the administrative control of the
Department of Space. It was incorporated in September 1992 commercially exploiting space
products of ISRO, providing technical consultancy services and transferring technologies to industry.
34. Ans:(b)
Explanations: The four global GNSS systems are – GPS (US), GLONASS (Russia), Galileo (EU),
BeiDou (China). Additionally, there are two regional systems – QZSS (Japan) and IRNSS or NavIC
(India). The operational name of IRNSS is NavIC.
It covers India and 1500 km of region covering India. It has eight constellations in its orbit.
Antrix Corporation Limited is a government-owned company under the administrative control of the
Department of Space. It was incorporated in September 1992 commercially exploiting space
products of ISRO, providing technical consultancy services and transferring technologies to industry.
35. Ans:(c)
Explanations: The death anniversary of Dr Babasaheb Ambedkar, which falls on December 6, is
dubbed as Mahaparinirvana Divas. On this day each year, millions of people throng to Dr
Ambedkar‟s Samadhi in Mumbai, which is called Chaitya bhoomi to pay respect to the father of
Indian Constitution.
Dr. Ambedkar was born on 14 April 1891 in Mahu, Madhya Pradesh, which is now known as Dr.
Ambedkar Nagar.
The name of Dr. Ambedkar's memorial site is Chaitya Bhoomi, which is in Mumbai, Maharashtra.
Bharat Ratna was given to Dr. Ambedkar by the government of Vishwanath Pratap Singh in 1990.
36. Ans:(a)
Explanations: The death anniversary of Dr Babasaheb Ambedkar, which falls on December 6, is
dubbed as Mahaparinirvana Divas. On this day each year, millions of people throng to Dr
Ambedkar‟s Samadhi in Mumbai, which is called Chaitya bhoomi to pay respect to the father of
Indian Constitution.
Dr. Ambedkar was born on 14 April 1891 in Mahu, Madhya Pradesh, which is now known as Dr.
Ambedkar Nagar.
The name of Dr. Ambedkar's memorial site is Chaitya Bhoomi, which is in Mumbai, Maharashtra.
Bharat Ratna was given to Dr. Ambedkar by the government of Vishwanath Pratap Singh in 1990.
37. Ans:(d)
Explanations: The death anniversary of Dr Babasaheb Ambedkar, which falls on December 6, is
dubbed as Mahaparinirvana Divas. On this day each year, millions of people throng to Dr
Ambedkar‟s Samadhi in Mumbai, which is called Chaitya bhoomi to pay respect to the father of
Indian Constitution.
Dr. Ambedkar was born on 14 April 1891 in Mahu, Madhya Pradesh, which is now known as Dr.
Ambedkar Nagar.
The name of Dr. Ambedkar's memorial site is Chaitya Bhoomi, which is in Mumbai, Maharashtra.
Bharat Ratna was given to Dr. Ambedkar by the government of Vishwanath Pratap Singh in 1990.
38. Ans:(d)
Explanations: The death anniversary of Dr Babasaheb Ambedkar, which falls on December 6, is
dubbed as Mahaparinirvana Divas. On this day each year, millions of people throng to Dr
Ambedkar‟s Samadhi in Mumbai, which is called Chaitya bhoomi to pay respect to the father of
Indian Constitution.
Dr. Ambedkar was born on 14 April 1891 in Mahu, Madhya Pradesh, which is now known as Dr.
Ambedkar Nagar.
The name of Dr. Ambedkar's memorial site is Chaitya Bhoomi, which is in Mumbai, Maharashtra.
Bharat Ratna was given to Dr. Ambedkar by the government of Vishwanath Pratap Singh in 1990.
39. Ans:(a)
Explanations: The death anniversary of Dr Babasaheb Ambedkar, which falls on December 6, is
dubbed as Mahaparinirvana Divas. On this day each year, millions of people throng to Dr
Ambedkar‟s Samadhi in Mumbai, which is called Chaitya bhoomi to pay respect to the father of
Indian Constitution.
Dr. Ambedkar was born on 14 April 1891 in Mahu, Madhya Pradesh, which is now known as Dr.
Ambedkar Nagar.
The name of Dr. Ambedkar's memorial site is Chaitya Bhoomi, which is in Mumbai, Maharashtra.
Bharat Ratna was given to Dr. Ambedkar by the government of Vishwanath Pratap Singh in 1990.
40. Ans:(b)
Explanations: The G20 summit founded in 1999 is the premier forum for international cooperation
on the most important aspects of the international economic and financial agenda. It brings together
the world‟s major advanced and emerging economies.
In 2009, during the third Summit of G20, the leaders designated the grouping as a premium forum
for international economic cooperation. The G20 summit were held semi-annually until 2010 and
annually from 2011. The G20 countries account to 80 percentage of World Trade and holds two third
of World Population.
41. Ans:(b)
Explanations: The G20 summit founded in 1999 is the premier forum for international cooperation
on the most important aspects of the international economic and financial agenda. It brings together
the world‟s major advanced and emerging economies.
In 2009, during the third Summit of G20, the leaders designated the grouping as a premium forum
for international economic cooperation. The G20 summit were held semi-annually until 2010 and
annually from 2011. The G20 countries account to 80 percentage of World Trade and holds two third
of World Population.
42. Ans:(b)
Explanations: The G20 summit founded in 1999 is the premier forum for international cooperation
on the most important aspects of the international economic and financial agenda. It brings together
the world‟s major advanced and emerging economies.
In 2009, during the third Summit of G20, the leaders designated the grouping as a premium forum
for international economic cooperation. The G20 summit were held semi-annually until 2010 and
annually from 2011. The G20 countries account to 80 percentage of World Trade and holds two third
of World Population.
43. Ans:(d)
Explanations: The G20 summit founded in 1999 is the premier forum for international cooperation
on the most important aspects of the international economic and financial agenda. It brings together
the world‟s major advanced and emerging economies.
In 2009, during the third Summit of G20, the leaders designated the grouping as a premium forum
for international economic cooperation. The G20 summit were held semi-annually until 2010 and
annually from 2011. The G20 countries account to 80 percentage of World Trade and holds two third
of World Population.
44. Ans:(d)
Explanations: The G20 summit founded in 1999 is the premier forum for international cooperation
on the most important aspects of the international economic and financial agenda. It brings together
the world‟s major advanced and emerging economies.
In 2009, during the third Summit of G20, the leaders designated the grouping as a premium forum
for international economic cooperation. The G20 summit were held semi-annually until 2010 and
annually from 2011. The G20 countries account to 80 percentage of World Trade and holds two third
of World Population.
45. Ans:(c)
Explanations: Arundhati Roy won the prestigious Booker Prize in 1997 for her first novel The God
of Small Things. Two other Indian-origin authors had won it before her – Salman Rushdie's second
novel, Midnight's Children won in 1981 and Sir V.S. Naipaul
Hilary Mantel is the first woman and the first British author to win the prize twice. JM Coetzee was
the first person to win twice, in 1983 and again in 1999, when he described the Booker as 'the
ultimate prize to win in the English speaking world'. Peter Carey won first in 1988 and then in 2001.
The Best of the Booker is a special prize awarded in commemoration of the Booker Prize's 40th
anniversary. Eligible books included the 41 winners of the Booker Prize since its inception in 1968. It
was awarded to Salman Rushdie.
46. Ans:(a)
Explanations: Arundhati Roy won the prestigious Booker Prize in 1997 for her first novel The God
of Small Things. Two other Indian-origin authors had won it before her – Salman Rushdie's second
novel, Midnight's Children won in 1981 and Sir V.S. Naipaul
Hilary Mantel is the first woman and the first British author to win the prize twice. JM Coetzee was
the first person to win twice, in 1983 and again in 1999, when he described the Booker as 'the
ultimate prize to win in the English speaking world'. Peter Carey won first in 1988 and then in 2001.
The Best of the Booker is a special prize awarded in commemoration of the Booker Prize's 40th
anniversary. Eligible books included the 41 winners of the Booker Prize since its inception in 1968. It
was awarded to Salman Rushdie.
47. Ans:(b)
Explanations: Arundhati Roy won the prestigious Booker Prize in 1997 for her first novel The God
of Small Things. Two other Indian-origin authors had won it before her – Salman Rushdie's second
novel, Midnight's Children won in 1981 and Sir V.S. Naipaul
Hilary Mantel is the first woman and the first British author to win the prize twice. JM Coetzee was
the first person to win twice, in 1983 and again in 1999, when he described the Booker as 'the
ultimate prize to win in the English speaking world'. Peter Carey won first in 1988 and then in 2001.
The Best of the Booker is a special prize awarded in commemoration of the Booker Prize's 40th
anniversary. Eligible books included the 41 winners of the Booker Prize since its inception in 1968. It
was awarded to Salman Rushdie.
48. Ans:(a)
Explanations: Arundhati Roy won the prestigious Booker Prize in 1997 for her first novel The God
of Small Things. Two other Indian-origin authors had won it before her – Salman Rushdie's second
novel, Midnight's Children won in 1981 and Sir V.S. Naipaul
Hilary Mantel is the first woman and the first British author to win the prize twice. JM Coetzee was
the first person to win twice, in 1983 and again in 1999, when he described the Booker as 'the
ultimate prize to win in the English speaking world'. Peter Carey won first in 1988 and then in 2001.
The Best of the Booker is a special prize awarded in commemoration of the Booker Prize's 40th
anniversary. Eligible books included the 41 winners of the Booker Prize since its inception in 1968. It
was awarded to Salman Rushdie.
49. Ans:(c)
Explanations: Arundhati Roy won the prestigious Booker Prize in 1997 for her first novel The God
of Small Things. Two other Indian-origin authors had won it before her – Salman Rushdie's second
novel, Midnight's Children won in 1981 and Sir V.S. Naipaul
Hilary Mantel is the first woman and the first British author to win the prize twice. JM Coetzee was
the first person to win twice, in 1983 and again in 1999, when he described the Booker as 'the
ultimate prize to win in the English speaking world'. Peter Carey won first in 1988 and then in 2001.
The Best of the Booker is a special prize awarded in commemoration of the Booker Prize's 40th
anniversary. Eligible books included the 41 winners of the Booker Prize since its inception in 1968. It
was awarded to Salman Rushdie.
50. Ans:(a)
Explanations: Assam has won the award among Hilly and North East States. The best performing
state award among inland states is to be provided to Uttar Pradesh and Odisha.
The Government of India had launched the Pradhan Mantri Matsya Sampada Yojana at Rs 20,050
crores. The programme aims to achieve 22 million tonnes of fish production by 2024-25. Also, it is
expected to create employment opportunity to 55 lakh people
Ministry of Fisheries, Animal Husbandry and Dairying is a newly formed ministry in India. It was
formed in May 2019 by Modi government from the department of same name under Ministry of
Agriculture and Farmers Welfare. Giriraj Singh is the minister responsible.
51. Ans:(c)
Explanations: Assam has won the award among Hilly and North East States. The best performing
state award among inland states is to be provided to Uttar Pradesh and Odisha.
The Government of India had launched the Pradhan Mantri Matsya Sampada Yojana at Rs 20,050
crores. The programme aims to achieve 22 million tonnes of fish production by 2024-25. Also, it is
expected to create employment opportunity to 55 lakh people
Ministry of Fisheries, Animal Husbandry and Dairying is a newly formed ministry in India. It was
formed in May 2019 by Modi government from the department of same name under Ministry of
Agriculture and Farmers Welfare. Giriraj Singh is the minister responsible.
52. Ans:(a)
Explanations: Assam has won the award among Hilly and North East States. The best performing
state award among inland states is to be provided to Uttar Pradesh and Odisha.
The Government of India had launched the Pradhan Mantri Matsya Sampada Yojana at Rs 20,050
crores. The programme aims to achieve 22 million tonnes of fish production by 2024-25. Also, it is
expected to create employment opportunity to 55 lakh people
Ministry of Fisheries, Animal Husbandry and Dairying is a newly formed ministry in India. It was
formed in May 2019 by Modi government from the department of same name under Ministry of
Agriculture and Farmers Welfare. Giriraj Singh is the minister responsible.
53. Ans:(a)
Explanations: Assam has won the award among Hilly and North East States. The best performing
state award among inland states is to be provided to Uttar Pradesh and Odisha.
The Government of India had launched the Pradhan Mantri Matsya Sampada Yojana at Rs 20,050
crores. The programme aims to achieve 22 million tonnes of fish production by 2024-25. Also, it is
expected to create employment opportunity to 55 lakh people
Ministry of Fisheries, Animal Husbandry and Dairying is a newly formed ministry in India. It was
formed in May 2019 by Modi government from the department of same name under Ministry of
Agriculture and Farmers Welfare. Giriraj Singh is the minister responsible.
54. Ans:(c)
Explanations: Assam has won the award among Hilly and North East States. The best performing
state award among inland states is to be provided to Uttar Pradesh and Odisha.
The Government of India had launched the Pradhan Mantri Matsya Sampada Yojana at Rs 20,050
crores. The programme aims to achieve 22 million tonnes of fish production by 2024-25. Also, it is
expected to create employment opportunity to 55 lakh people
Ministry of Fisheries, Animal Husbandry and Dairying is a newly formed ministry in India. It was
formed in May 2019 by Modi government from the department of same name under Ministry of
Agriculture and Farmers Welfare. Giriraj Singh is the minister responsible.
55. Ans:(b)
Explanations: After independence, Azad focused on imparting information in modern sciences. He
started a weekly magazine in Urdu called Al-Hilal that criticised the British Government and its
policies. After it was banned, he started another magazine called the “Al-Bagah”. Following this, the
British Government banned him under Defence of India Regulations, 1916.
India wins freedom is an account of the Partition from the Maulana Azad‟s perspective. It includes
his personal experiences when India became independent, and his ideas on freedom and liberty.
In 1923, at an age of 35, he became the youngest person to serve as the President of the Indian
National Congress.
56. Ans:(b)
Explanations: After independence, Azad focused on imparting information in modern sciences. He
started a weekly magazine in Urdu called Al-Hilal that criticised the British Government and its
policies. After it was banned, he started another magazine called the “Al-Bagah”. Following this, the
British Government banned him under Defence of India Regulations, 1916.
India wins freedom is an account of the Partition from the Maulana Azad‟s perspective. It includes
his personal experiences when India became independent, and his ideas on freedom and liberty.
In 1923, at an age of 35, he became the youngest person to serve as the President of the Indian
National Congress.
57. Ans:(a)
Explanations: After independence, Azad focused on imparting information in modern sciences. He
started a weekly magazine in Urdu called Al-Hilal that criticised the British Government and its
policies. After it was banned, he started another magazine called the “Al-Bagah”. Following this, the
British Government banned him under Defence of India Regulations, 1916.
India wins freedom is an account of the Partition from the Maulana Azad‟s perspective. It includes
his personal experiences when India became independent, and his ideas on freedom and liberty.
In 1923, at an age of 35, he became the youngest person to serve as the President of the Indian
National Congress.
58. Ans:(b)
Explanations: After independence, Azad focused on imparting information in modern sciences. He
started a weekly magazine in Urdu called Al-Hilal that criticised the British Government and its
policies. After it was banned, he started another magazine called the “Al-Bagah”. Following this, the
British Government banned him under Defence of India Regulations, 1916.
India wins freedom is an account of the Partition from the Maulana Azad‟s perspective. It includes
his personal experiences when India became independent, and his ideas on freedom and liberty.
In 1923, at an age of 35, he became the youngest person to serve as the President of the Indian
National Congress.
59. Ans:(b)
Explanations: After independence, Azad focused on imparting information in modern sciences. He
started a weekly magazine in Urdu called Al-Hilal that criticised the British Government and its
policies. After it was banned, he started another magazine called the “Al-Bagah”. Following this, the
British Government banned him under Defence of India Regulations, 1916.
India wins freedom is an account of the Partition from the Maulana Azad‟s perspective. It includes
his personal experiences when India became independent, and his ideas on freedom and liberty.
In 1923, at an age of 35, he became the youngest person to serve as the President of the Indian
National Congress.
60. Ans:(b)
Explanations: The Portuguese colonized several parts of India in 1510. However, by the end of
19th century their colonies were limited to Daman, Dadra, Goa, Diu, Nagar Haveli.
On December 18, 1961, India launched a 36-hour military operation called Operation Vijay. The
operation involved attacks by Indian Army, Indian Air Force and Indian Navy.
India reclaimed the Goan territories on December 19, 1961. The then Governor of Goa Manuel
Antonio Vassalo e Silva signed the certificate of surrender bringing the Portuguese rule to an end.
Goa was granted statehood in 1987. The Goa Statehood Day is celebrated on May 30th.
61. Ans:(b)
Explanations: The Portuguese colonized several parts of India in 1510. However, by the end of
19th century their colonies were limited to Daman, Dadra, Goa, Diu, Nagar Haveli.
On December 18, 1961, India launched a 36-hour military operation called Operation Vijay. The
operation involved attacks by Indian Army, Indian Air Force and Indian Navy.
India reclaimed the Goan territories on December 19, 1961. The then Governor of Goa Manuel
Antonio Vassalo e Silva signed the certificate of surrender bringing the Portuguese rule to an end.
Goa was granted statehood in 1987. The Goa Statehood Day is celebrated on May 30th.
62. Ans:(b)
Explanations: The Portuguese colonized several parts of India in 1510. However, by the end of
19th century their colonies were limited to Daman, Dadra, Goa, Diu, Nagar Haveli.
On December 18, 1961, India launched a 36-hour military operation called Operation Vijay. The
operation involved attacks by Indian Army, Indian Air Force and Indian Navy.
India reclaimed the Goan territories on December 19, 1961. The then Governor of Goa Manuel
Antonio Vassalo e Silva signed the certificate of surrender bringing the Portuguese rule to an end.
Goa was granted statehood in 1987. The Goa Statehood Day is celebrated on May 30th.
63. Ans:(a)
Explanations: The Portuguese colonized several parts of India in 1510. However, by the end of
19th century their colonies were limited to Daman, Dadra, Goa, Diu, Nagar Haveli.
On December 18, 1961, India launched a 36-hour military operation called Operation Vijay. The
operation involved attacks by Indian Army, Indian Air Force and Indian Navy.
India reclaimed the Goan territories on December 19, 1961. The then Governor of Goa Manuel
Antonio Vassalo e Silva signed the certificate of surrender bringing the Portuguese rule to an end.
Goa was granted statehood in 1987. The Goa Statehood Day is celebrated on May 30th.
64. Ans:(a)
Explanations: The Portuguese colonized several parts of India in 1510. However, by the end of
19th century their colonies were limited to Daman, Dadra, Goa, Diu, Nagar Haveli.
On December 18, 1961, India launched a 36-hour military operation called Operation Vijay. The
operation involved attacks by Indian Army, Indian Air Force and Indian Navy.
India reclaimed the Goan territories on December 19, 1961. The then Governor of Goa Manuel
Antonio Vassalo e Silva signed the certificate of surrender bringing the Portuguese rule to an end.
Goa was granted statehood in 1987. The Goa Statehood Day is celebrated on May 30th.
65. Ans:(c)
Explanations: IOM was established in 1951 and is the leading inter-governmental organization in
the field of migration. IOM entered into an agreement with the United Nations in 2016 and became
one of its specialized agencies.
Every year, the International Migrants Day is celebrated on December 18 by the United Nations and
several other international organizations. This year, the International Migrants Day is celebrated
under the following theme
Theme: Reimagining Human Mobility.
66. Ans:(a)
Explanations: IOM was established in 1951 and is the leading inter-governmental organization in
the field of migration. IOM entered into an agreement with the United Nations in 2016 and became
one of its specialized agencies.
Every year, the International Migrants Day is celebrated on December 18 by the United Nations and
several other international organizations. This year, the International Migrants Day is celebrated
under the following theme
Theme: Reimagining Human Mobility.
67. Ans:(b)
Explanations: IOM was established in 1951 and is the leading inter-governmental organization in
the field of migration. IOM entered into an agreement with the United Nations in 2016 and became
one of its specialized agencies.
Every year, the International Migrants Day is celebrated on December 18 by the United Nations and
several other international organizations. This year, the International Migrants Day is celebrated
under the following theme
Theme: Reimagining Human Mobility.
68. Ans:(c)
Explanations: IOM was established in 1951 and is the leading inter-governmental organization in
the field of migration. IOM entered into an agreement with the United Nations in 2016 and became
one of its specialized agencies.
Every year, the International Migrants Day is celebrated on December 18 by the United Nations and
several other international organizations. This year, the International Migrants Day is celebrated
under the following theme
Theme: Reimagining Human Mobility.
QUANTITATIVE TECHNIQUES

69. Ans:(c)
Average expenses of P

70. Ans:(a)
Amount spent by Rahul on Entertainment

71. Ans:(d)
Average expenses of Rohit
72. Ans:(a)
Required difference

73. Ans:(b)
Required percentage

74. Ans:(d)
5x + 5 + 6x + 2 = 95
x=8
Ram: 6x + 2 = 50
Bala: 5x + 5 = 45
75. Ans:(b)
20/(x - 5) - 20/(x + 5) = 1
X2 = 225
x = 15
2x = 30 kmph
76. Ans:(c)
Let us take cost price of the bicycle be Rs.100
MP = 100 * 125/100 = Rs.125
SP = 125 * (100 - 10)/100 = Rs.112.5
Required percentage = (112.5 - 100)/100 * 100
= 12.5%
77. Ans:(d)
Amount = P (1 + R/100)2
43264 = P (1 + 4/100)2
43264 * 100/104 * 100/104 = 40000
Simple interest = PNR/100
= (40000 * 15 * 2)/100 = Rs.12000
78. Ans:(a)
Profit ratio of Ram and Bala = (900 * 6)/(x * 12)
= 15/13
x = 390
79. Ans:(d)
88% ----- 22,000
100% ----- 22000×10080=25000
Total salary = 25,000
Savings = 25,000-22,000 = 3,000
Ratio = 25,000 : 3,000 = 25:3
80. Ans:(c)
Expenditure = 32,000 - 5,500 = 26,500
100% ---- 26,500
20% ---- 26500×20100=5300
For Education he spends = 5,300
81. Ans:(b)
C salary = 27,000
C expenditure = 27,000*88/100 = 23,760
Rent = 23,760*10/100 = 2,376
82. Ans:(b)
A's Savings = 45,000 - 37,500 = 7500
After increasing his salary become = 120*45000/100 = 54,000
Expenditure = 37,500*110/100 = 41,250
A's new savings = 54,000 - 41,250 = 12,750
Difference = 12,750 - 7,500 = 5,250
LOGICAL REASONING

83. Ans:(b)
Explanation: Rationale - The poem is a son‟s plea to a dying father. His purpose is to show his
father that all men face the same end, but they fight for life, nonetheless. “Old age should burn and
rave at close of day,” (line 2) is almost the thesis for this poem. The other options are too specific
compared to the larger theme.
Hence, [B].
84. Ans:(d)
Explanation: Rationale - After classifying men into four categories and stating that whatever what
one was in one‟s lifetime, one has a reason to live – wise men‟s „words had forked no lightning‟, so
they do not go gentle into that good night. Good men‟s „deeds might have danced… etc. Wild men
„learn too late … etc.‟ Grave men „ see with blinding sight… etc.” give indication that for all of them
there is reason to live. Hence option „D‟ is the best reason for the classification. Option „A‟ is not
related to the classification. Option „B‟ is incorrect as the poet does not suggest specifically that his
father was a wise man. Option 3 merely states that everyone does, undermining the reason for the
classification.
Hence, [D].
85. Ans:(d)
Explanation: Rationale - “Wild men who caught and sang the sun in flight,/ And learn, too late, they
grieved it on its way”. „Caught and sang the sun in flight‟ is an exaggeration which implies that now
they realize that the life they lived was not real. The option that most closely expresses this idea is
„D‟. All the other options merely imply the way they might have lived but they are not reasons enough
to warrant grieving at the end.
Hence, [D].
86. Ans:(d)
Explanation: Rationale - Both option 1 and 2 are twisted option choices. Option 1 cannot be inferred
from the passage. The report talks about the rate at which India‟s carbon emission is rising and does
not talk in terms of an absolute quantity. Option 2 is also wrong. Passage mentions that average
Indian contributes only minuscule amounts of transport-related carbon dioxide (CO2) emissions.
Hence, „D‟
87. Ans:(c)
Explanation: Rationale - The statement is probably true based on the information provided in the
passage that average Indian contributes only minuscule amounts of transport-related carbon dioxide
(CO2) emissions to global climate change and, “India‟s carbon emissions are rising more than two
times as fast as the global rise in 2018”.
Hence, „C‟
88. Ans:(c)
Explanation: Rationale - Both option 1 and 2 are correct. Refer to, “Globally, the transport sector
accounts for a quarter of total emissions, out of which three quarters are from road transport.
Reducing CO2 emissions of road transport leverages multiple co-benefits, for example, improving air
quality and increasing physical activity, which are critical for well-being”
Hence, „C‟
89. Ans:(d)
Explanation: Rationale - Option 2 is outrightly wrong. Option 1 is a confusing choice, but we need to
consider if the option logically supports the argument that emissions vary with spatial context. The
options fails to do that and hence, it is an incorrect answer choice.
Hence, „D‟
90. Ans:(c)
Rationale: Options (A) and (B) find mention in the passage. Option (D) can be inferred from the
statement “Another great achievement of the protests has been the secularisation of the Muslim
ghetto” Option (C) cannot be inferred from the passage. The author does not seem to have a
favourable opinion on the two leaders but nothing connects the event of the Prime Minister inviting
Jait Bolsonaro as the chief guest of the Republic Day and the protests or protestors.
Hence, „C‟.
91. Ans:(d)
Explanation: Rationale: Based on Jair Bolsonaro„s visit, the author says that “It must, therefore, be
assumed that the Prime Minister nurses him as a model”. Both options weaken the author„s claim. In
both the options the Prime Minister has been shown to invite people who have dissimilar ideology to
either that of the Prime Minister or Jair Bolsonaro, which the author suggests is the same.
Hence, „D‟.
92. Ans:(d)
Explanation: Rationale: Option (A) is more an over–generalisation. Also, though the author
mentions the point on secularisation of Muslim ghetto, he does that to illustrate the achievement of
the protests and how it has brought together communities against the government. It is not the main
point but it lends support to the main point, which is that protests have shaken the ruling party
somewhat and caused their leaders “into such glorious absurdities”. Option (A), therefore, can be
eliminated. Option (B) is wrong firstly because the passage only talks about Muslim women who are
part of protests and not Muslim women in general and secondly empowerment is a broad term and
the passage has not covered it in detail. Option (C) is also wrong as the passage doesn„t talk about
that. Option (D) is correct as it is what the author wants to emphasise and provides claims to support
it.
Hence, „D‟.
93. Ans:(a)
Explanation: Rationale: “The dilemma of today's liberals' face: they cannot give up on capitalism
(therefore imperialism) even in its post globalization, post 2008 avatar.” Option (A) can be easily
inferred.
Option (B) is wrong because the author says that the liberal often end up siding with the
establishments that redirect popular resentment against inequality towards issues of immigration,
ethnic identities. He does not comment or imply that the liberals themselves redirect popular
resentment against inequality towards issues of immigration, ethnic identities.
Hence, „A‟.
94. Ans:(a)
Explanation: Rationale: Option (C) is an overgeneralised statement. The author only says that
homebound and docile Muslim women are first time seen in their public avatar. They can still be
homebound and docile and be public and vocal about their opinions when needed. Secondly, the
author refers to Muslim women as a whole or as a group. It may not apply to all the Muslim women
in the group. Something that is true of the whole/group may not be true for all its individual members.
Therefore, Option (C) is wrong. Option (D) is wrong. The reference to bindi in the passage is
metaphorical to signify the coming together of two communities in the protests.
95. Ans:(d)
Explanation: Rationale- Conclusion of the author is that to increase job satisfaction of the police
officers, the belief that control which police officers have over their job assignments should be
increased and argument that police officers have many other reasons to be dissatisfied with their
jobs would seriously undermine the author‟s conclusion.
Therefore, option (D) is the most appropriate answer.
96. Ans:(b)
Explanation: Rationale- The pattern of reasoning in the above passage by the author is that State
Government is providing something which police officers didn‟t had to increase their job satisfaction.
The same reasoning has been followed in option (B).
Therefore, option (B) is the most appropriate answer.
97. Ans:(c)
Explanation: Rationale- Option (B) and (D) is not the correct answer the above statement is not the
conclusion of the above passage. Conclusion is the main idea of the passage. Option (A) is not the
correct answer because his statement that many police personals are not satisfied with their jobs in
itself forms premise to the authors‟ conclusion. Option (C) is the correct answer because it forms
premise to the authors‟ conclusion.
Hence, „C‟.
98. Ans:(c)
Explanation: Rationale- Author in the passage provides that many police personals in certain States
are not satisfied with their jobs, which means that many but not all police officers are dissatisfied with
their jobs in certain states. Therefore, option (C) must be the true statement
99. Ans:(d)
Explanation:
A. The argument talks about being prone to illnesses, not their severity.
B. Does not bring out any objective parameters by which the argument can be proved or disproved
C. We would need additional information about the two groups with respect to following precautions
and their eating habits.
D. Correct. The author uses the fact that more teenagers were treated, to say that they are more
susceptible or that they were more likely to fall ill, which is similar to saying that the probability of
falling ill is higher in the case of teenagers. This cannot be stated unless we know what percentage
of each group fell ill. If the percentage of the sexagenarians out of all people in that age bracket who
had fallen ill was greater than the percentage of youngsters then it clearly shows that the author is
wrong and vice-versa.
Thus, choice „D‟ is the correct answer.
100. Ans:(b)
Explanation: The artificial manure gives a greater yield of wheat. Thus, Diljit will grow more wheat
when he uses artificial manure. Hence, answer choice B strengthens the argument.
According to choice A, the artificial manure doesn‟t work in water and the wheat requires water for
growth. Thus, this weakens the argument.
Choice C talks about the type of insecticides used by the farmer which doesn‟t strengthen or weaken
the argument in any way.
Choice D gives us information about low availability of organic manure. If organic manure was
favored by the farmer he would use at least as much organic manure as is available in the market.
Therefore, this cannot be the reason for using „only‟ artificially synthesized manure.
Thus, answer choice B is correct.
101. Ans:(c)
Explanation: The argument states that the people from lower income group spend more on medical
facilities than those from the higher income group. Choice C mentions that the people from lower
income group utilize free medical facilities, which contradicts the argument.
Choice A doesn‟t mention anything about the expenses incurred, though it mentions the hygiene
conditions. Hence, choice A is inappropriate.
Choice B does not weaken the argument.
Choice D does not weaken the argument.
Thus, choice C is the correct answer.
102. Ans:(d)
Explanation: A. Not enough information to ascertain whether the employees have reached their
upper limit.
B. Though there might be employees who do not report personal life related stress, the management
has devised the workshop for those who have reported the same.
C. Does not weaken the management‟s strategy to improve the productivity of stressed out
employees. Would have been the right answer if the management‟s strategy was to improve
productivity in general and not stress related productivity. Also we have no information to determine
if office space comforts and employee relationships at Amazon.com are not satisfactory.
D. Correct. The workshop is a five-day four-step method to tackle stress, but according to Option D
this would be ineffective because people suffering from personal life related stress cannot be treated
by this method. The management has assumed that both types of stresses can be cured by the
same method.
Thus, choice D is the correct answer.
103. Ans:(a)
Explanation: The argument the author presents is that traits which currently seem counterintuitive
or redundant in living creatures, actually have a basis in evolution - examples in the passage include
the attention seeking feathers of birds that attract mates, the human chin that has carried over from
larger muscles and jaws for intensive chewing, etc. These examples help the author make
arguments that “…strengthen(s) the theory of evolution in that these seemingly weaker individuals
are actually showing how well they can do in the face of adversity.” (Paragraph 2 - sentence 5) The
author makes the same argument about vestigial too, proving that evolution has a role to play in
traits that appear quirky or useless, if one studies them closely. This is also why option „B‟ is not the
correct choice. Besides option „B‟ is the other end of the extreme, dismissing the role of evolution in
any biological trait or characteristic altogether and is therefore not compatible with the views of the
author in the passage. Option „C‟, is not the central argument, and is not true based on the
information of the passage - in paragraph 2, the author describes how sexual selection is the reason
some birds have ostentatious plumage. There is no mention of the role of sexual selection being
larger than it is deemed to be. Option „D‟ is not correct because not only is it not the central premise
of the passage but it is also incoherent with the information in the passage- “As human diets
changed, the bones and muscles in our jaws became smaller so we didn‟t waste energy on them but
we were left with a protruding bone at the bottom of the face…Although the chin throws a spandrel in
the works, there is nearly always a reason or, at least, an explanation for the myriad traits we see
across biology.” The author means therefore, that the chin is not a quirk that exists for its own sake
but can actually be traced back to its evolutionary purpose.
Hence, [A].
104. Ans:(c)
Explanation: Sentence 1 & 2 of paragraph 2 make this clear, whereby the author states that despite
some species having traits that help them disappear into their surroundings, birds of paradise
actually want to be noticed by potential mates. “Many species invest heavily in camouflage and other
means of blending into the surroundings to avoid predators. So the physically heavy and downright
ostentatious plumage of birds of paradise, peacocks and many other birds seems like a clear
invitation to be eaten. But crucially they help these birds pass on their genes because they increase
their chances of attracting a mate.” Thus although their feathers are used for survival, they are not
used for camouflage. Option „A‟ is true based on sentence 3 of paragraph 4, “An evolutionary
spandrel is a physical structure or behavioural characteristic that is a by-product from some other
functional adaptation.” Option „B‟ is true based on the passage, because it is a mere re-statement of
a line from the passage, Paragraph 3, sentence 3, “If we could embody evolution as a person, then
he or she would be creative but inherently lazy.” Option 4 is not the correct answer because it is true
based on sentence 6 of paragraph 3, “Male pythons have little claw-like structures towards the tail,
which, although they aid courtship, are all that remain of their hind limbs.” This sentence makes it
clear that the claws behind male pythons, originated from their once hind limbs. Therefore Option „D‟
is true.
Hence, [„C‟].
105. Ans:(d)
Explanation: The central argument is that vestigial traits and spandrels are only apparently useless,
but they have an evolutionary function. Option „D‟ implies that evolution may not eliminate useless
features that are harmless. This means that there are features that are clearly useless and have no
evolutionary significance. This makes the entire argument of the passage invalid. Option „A‟ is
certainly assumed by the author. This can be deduced from sentence 5 of paragraph 1, “As
important as the concept of survival of the fittest is to evolution, there are many examples that seem
to undermine this idea.” This sentence is based on the inherent assumption that survival of the fittest
is perceived as a concept that is important to the theory of evolution. Option „B‟ is an assumption
made in paragraph 2, whereby sexual selection in birds becomes a means of perpetuating their
genes. Option „C‟ supports the main idea that vestigial traits are useful.
Hence, [„D‟].
106. Ans:(c)
Explanation: Option „A‟ is factually incorrect. Option „B‟ describes spandrels incorrectly. Option „D‟ is
incorrect as both are only apparently useless – hence it cannot be inferred that one or the other may
be discarded. The third paragraph states about vestigial structures: This is a feature that no longer
does whatever made it advantageous enough to evolve in the first place. The fourth paragraph
explains a spandrel: An evolutionary spandrel is a physical structure or behavioural characteristic
that is a by-product from some other functional adaptation.
Hence, [C].
107. Ans:(b)
Explanation: Option „A‟ is not sustainable because implicitly vestigial structures and spandrels
reinforce the idea of evolution (as subtle proofs for the process). Hence option 2 is correct. Option
„C‟ is contrary to the passage – the last sentence states that, “Although the chin throws a spandrel in
the works, there is nearly always a reason or, at least, an explanation for the myriad traits we see
across biology.” Option „D‟ is incorrect. The author states it quite explicitly in the first paragraph that
“In reality, evolution is a mindless, plan-free phenomenon, driven into endless possibilities by
random mutations, the most successful of which win out.” Hence, [B].
108. Ans:(d)
Explanation:

109. Ans:(c)
Explanation:

110. Ans:(a)
Explanation: According to their age= B>D>F(81)>C>E(62)>A
111. Ans:(d)
Explanation: According to their age= B>D>F(81)>C>E(62)>A
112. Ans:(c)
Explanation: According to their age= B>D>F(81)>C>E(62)>A

LEGAL REASONING

113. Ans:(c)
EXPLANATION: Gogo Foods, even though a subsidiary, is not identical to its parent as it has
entered into its own contract of a different nature. Clearly, it has a mind of its own and therefore c is
the only suitable answer.
114. Ans:(a)
EXPLANATION: The answer is a. The parent company has made significant changes to company‟s
structure of governance and business. Launching new products in the same domain is not per se an
individual or independent venture. Therefore, d cannot be the answer. B and c don‟t make any
sense.
115. Ans:(b)
EXPLANATION: New Horizon is a newly established entity but the joint venture is experienced. Due
consideration must be given to the constituents experience to keep up with the realities of the
situation.
116. Ans:(a)
EXPLANATION: A very simple yet conceptual question. The CEO‟s personal assets cannot be
attached, no matter how reckless he was. He was an employee of the company and the loan was
taken in the company‟s name and not his. Therefore, the answer is a.
117. Ans:(b)
EXPLANATION: b is the only correct answer. The words “sold its share in the Indian company”
should be underlined here. Therefore, it was a share sale of an Indian company. Two foreign entities
buying and selling shares is clearly outside the Indian tax authorises jurisdiction. What is to be seen
here is whether the Indian subsidiary, which is a separate entity, is benefiting from the deal or not.
118. Ans:(c)
EXPLANATION: - Dodaphone, and Dodaphone India are 2 separate entities specially after the
former sold their majority share to foreign entities. Therefore prosecuting Dodaphone for the act of
Dodaphone India is inherently wrong.
119. Ans:(a)
EXPLANATION: the authority does have a discretionary power but such power cannot be used to
deny the request when other reasonable restrictions are met with. Not (b) and (c) because the
reason is so wide that they cannot be applied on facts unless there is any immediate threat.
120. Ans:(d)
EXPLANATION: - because the source of the traffic jam is still not clear, whether it happened
because of protest or is a regular occurrence in that area therefore no such reason is able to be
upheld or reject the decision of authority.
121. Ans:(a)
EXPLANATION: according to the author such limitations are wrong and destroy the essence of
expression of dissent in a self-ruled democracy.
122. Ans:(a)
EXPLANATION: Right to dissent can only be exercised peacefully use of farming tools in a rally is
excessive, the threatening effect of traditional farming tools is very well known throughout. Not (d)
because nothing about occupying a public place was mentioned rallies are a moving mass of people
and not an absolute occupation of a place.
123. Ans:(b)
EXPLANATION: - because the farmers changed the tactic and will now use fake tools to remove the
apprehension of danger, the authorities must allow them the permission.
124. Ans:(b)
Only his right to legal counsel under Article 22(1) is being violated. He was produced before a
magistrate within 24 hours, so Article 22(2) is not being violated. Hence option (a) and (d) are not
applicable. Article 22(5) is only applicable to detenues. Johnny is not arrested under any preventive
detention law so option (c) is incorrect.
125. Ans:(d)
The question brings up concerns related to Articles 22(2) and 22(6). However, both of these
concerns are mitigated because the Public Safety Act allows preventive detention under which 22(2)
is taken away so option (a) and consequently option (c) in not applicable. The concern under Article
22(6) is mitigated because the Act deals with public order so not informing Thomas about the
grounds for his arrest does not violate any of his fundamental rights.
126. Ans:(c)
Protection under Article 22 is available to citizens and non-citizens which includes Indian Citizens,
foreigners and non-resident Indians. It is not available to persons arrested under a preventive
detention law.
127. Ans:(a)
The correct answer is option (a) because the police informed him that he violated he „Cleanliness
Act‟ at the time of his arrest. Hence, option (b) is incorrect. Option (c) and (d) are not applicable
because „duty to be aware about the laws under which one is being arrested‟ is not a criterion as per
the passage.
128. Ans:(c)
The correct answer is option (c) because Article 21 is available to both citizens and non-citizens.
Option (b) is incorrect because even if there was no „amputation‟ or „mutilation‟, the functioning of his
ear deteriorated and he was deprived of the same because of the torture constituting a violation of
his right to life and personal liberty. Option (d) is incorrect because the villager was not a citizen of
Liberalia. The two countries maintain a peaceful relation, so he was not an enemy alien hence,
option (a) is incorrect.
129. Ans:(d)
The correct option is (d) since the enemy soldier from Liberalia is an „enemy alien‟ and the protection
of fundamental rights is not advanced to enemy aliens. Hence, both (b) and (c) are correct. Option
(a) is incorrect because even if he is a non-citizen, as per the question he is an enemy alien of
Leftistan.
130. Ans:(c)
The correct answer is option (c) because it is a case of abduction. Since Eleanor is not a minor, the
question is not concerned with the offence of kidnapping. Hence, option (a) is incorrect. By
voluntarily calling Chidi so that the two of them could run away and get married, Eleanor consented
to the act. This waives off abduction so options (b) and (d) are incorrect.
131. Ans:(b)
The correct answer is option (b) because the stranger kidnapped Jason. The consent of the person
being kidnapped is irrelevant and so is the fact that the two were friends. By taking Jason away from
lawful guardianship, the stranger committed the offence of kidnapping.
132. Ans:(d)
The correct answer is option (d) because it is a case of kidnapping since Blue Ivy was of unsound
mind at the time of the incident. The man took her away from her mother. Since she was of unsound
mind, she was not in a position to go willingly or unwillingly. It is not a case of abduction since there
is no element of deceit, even if she was not a minor.
133. Ans:(a)
The correct answer is option (a) because they used force to take Mala from one place to another.
Options (b) and (d) are incorrect because Mala was taken forcefully. Option (c) is incorrect because
Mala was not compelled but taken by brute force.
134. Ans:(d)
The correct answer is option (d) because the presence of consent can condone abduction. Shanaya
was not induced or compelled to go from one place to another. She did so willingly to complete the
treasure hunt. Hence, options (a), (b) and (c) are incorrect.
135. Ans:(c)
The correct answer is option (c). Options (a), (b) and (d) are not relevant in light of the passage.
Option (c) is the best answer because the passage clearly defines the word „take‟ to include the
action of escort someone. The mental attitude of the minor is not relevant in the course of taking. So,
it doesn‟t matter if it was David he wanted to get away.
136. Ans:(c)
The correct answer is option (c) because this game involves betting. The game involves soliciting of
bets on the cups with maximum coins and distribution of prize money to the winner. Under the
Ordinance, wagering or betting is deemed to include collection or soliciting of bets, and the receipt or
distribution of winnings or prizes.
137. Ans:(a)
The correct answer is option (a). The fact that his data is being recorded only makes it a „game‟
within the purview of the Ordinance hence, option (b) is incorrect. Options (c) and (d) are irrelevant
because Sujan is committing an offence because the Ordinance criminalizes the game of rummy
itself.
138. Ans:(b)
The correct answer is option (b) because the Ordinance only criminalizes those online games which
involve wagering or betting such as rummy and poker. Options (c) and (d) are not supported by the
given facts. The entire website is not prohibited because it houses a collection of other games also.
139. Ans:(d)
The correct answer is option (d) because the passage mentions that “the Ordinance comes in the
wake of the observations of the Madras High Court regarding the addictive nature of online games
and its negative effect on the masses”. All of these effects are a result of the negative and addictive
nature of such games.
140. Ans:(b)
The correct answer is option (b) because all games involving skill are explicitly excluded from the
scope of the Ordinance so it doesn‟t matter if it involves betting. Option (c) is incorrect because
horse racing as a game involves both: runners racing the horses and people placing bets in the
derby. The two are not severable.
141. Ans:(c)
The correct answer is option (c) because the new law takes away the fundamental right ofn peaceful
assembly by way of an arbitrary legislative action. The validity of the law in terms of how this
arbitrariness is exercised is not a concern of the passage. Even if it was passed with a majority it is
violative of Article 19. Hence, options (a) and (b) are incorrect. Option (d) is incorrect because the
law does not put reasonable restrictions but takes away the right to protest in entirety.
142. Ans:(b)
Parliament road is a public space, not a designated protest area and thus, cannot be occupied
indefinitely for a protest. Hence, options (a) and (c) are not applicable. Option (d) is irrelevant to the
question.
143. Ans:(a)
The correct answer is option (a) because the restriction was in the interest of public safety. In light of
the given fact, it is a reasonable restriction that does not violate the right to protest hence, options
(b), (c) and (d) are incorrect.
144. Ans:(d)
The correct answer is option (d) because a terrorist threat endangers security of the state and
therefore, constitutes a reasonable restriction. Consequently, options (a) and (b) are incorrect
because this advisory of the government is not a violation of Article 19(3) via arbitrary exercise of
power. The question is not concerned with public order hence, option (c) is not applicable.
145. Ans:(b)
The correct answer is option (b) because the Court had clearly ordered the students to clear the
protest site and to still continue to protest would amount to contempt of Court. Options (a), (c) and
(d) are irrelevant to the question.
146. Ans:(c)
The correct answer is option (c) because the people were protesting peacefully and it is their
fundamental right to do so. Firing of rubber bullets to break the protest is a violation of their
fundamental rights. Thus, option (b) is incorrect. Sings songs of unity and fraternity cannot be
reasonably assumed to incite people to turn violent and hence, option (a) is not applicable. Option
(d) is neither relevant nor correct as per the passage.
147. Ans:(c)
The correct answer is option (c) because this ordinance only criminalizes conversion when it was the
sole purpose behind the marriage. Options (b) and (d) are irrelevant to the question. Option (a) is
incorrect because the ordinance is concerned with the conversion of both men and women.
148. Ans:(b)
The correct answer is option (b) because Murad coerced Sybil into converting to Maradonianism by
threatening to hit her if she did not convert. Options (a) and (c) are incorrect because Sybil
converted due to coercion. Option (d) is incorrect because it is a case of coercion and not undue
influence. Even if Murad was her husband, he threatened to hurt her and she converted out of fear
to prevent the same.
149. Ans:(d)
The correct answer is option (d) because Murad facilitated Sybil‟s conversion. As per the passage,
the burden to prove that a conversion was not done through misrepresentation, force, undue
influence, coercion, allurement, fraudulent means or for marriage would be on the person who
caused the conversion or the person who facilitated it.
150. Ans:(a)
The correct answer is option (a) because by promising to bear the marriage expenses if Moeena
converted he allured her to unlawfully convert before marriage. Bearing the marriage expenses
amounts to allurement. Hence, options (b) and (d) are incorrect. Option (c) is not directly
substantiated by the given facts.
Legal Edge 17 Mock

Answers & Details

ENGLISH

1. Ans:(b)
Denial of a right will involve active prevention of women from accessing the convenience facilities.
Therefore locking women‘s toilets at night will lead to denial, whereas making more toilets for men
(if adequate toilets are made for women) and making women feel unwanted will not lead to an active
denial of the right to women.
2. Ans:(a)
The following lines in the passage direct us to the answer:
A new ―period room‖ in a Mumbai slum, the first of its kind in a public toilet, therefore, suggests a
refreshing reimagination of the public space.
3. Ans:(d)
The word replicate refers to copying; therefore the most suitable antonym would be to vary.
4. Ans:(b)
A private toilet is a common feature in most homes in the city but in slums it is an extravagant
feature. In such a situation, if a toilet is converted to an exclusive period room the resultant is
definitely an ironic situation.
5. Ans:(d)
This statement is not within the scope of Period Rooms. They are essentially made to promote
menstrual hygiene and provide women with a safe place for cleaning up. If women‘s toilets remain
locked in the evenings, the only solution would be to unlock them and open them up for women.
6. Ans:(c)
Making an exclusive space for women to use period rooms in community toilets, it is important to
dispel the taboo surrounding menstruation as community toilets are located in residential areas. It
will normalize the condition of menstruation if people are made familiar with public
amenities providing for women.
7. Ans:(c)
The move against Trump is very small compared to the damage caused by the siege and the
violence already committed by his supporters. Mere suspension of social media accounts will not
help in any way.
8. Ans:(b)
Zuckerberg was denying the fact that social media affects voters‘ decisions and therefore the only
statement that will support his statement is that social media is not given too much importance by
voters.
9. Ans:(d)
The following lines in the passage direct us to the answer:
Trump and his falsehoods, though, are only a part of a larger digital ecosystem that has devalued
facts, created ―communities‖ of conspiracy theorists and, at times, even contributed to violence.
10. Ans:(b)
The following lines in the passage direct us to the answer:
In essence, the algorithm is meant to keep users glued to the screen and get more of the same. For
example, someone watching ―flat earth‖ videos, or ―the history of forced conversion in India‖ or, more
recently, the bizarre conspiracy theories about a film star‘s suicide, could well be led to something
like a QAnon page (a group that believes Trump is saving the US government from a corporate
―deep state‖) or to fake news that whips up prejudice against minority groups.
11. Ans:(a)
The word despot refers to someone who is an autocratic leader, therefore dictator is the best
answer.
12. Ans:(d)
Culpable means someone who is guilty, therefore the most suitably antonym is someone who is
innocent.
13. Ans:(b)
The following statements direct us to the answer:
Jaishankar‘s message, that India still sees the 1987 amendment — the constitutional basis for the
provincial councils — as central to addressing Tamil political aspirations came within days of a
statement by the DMK‘s T R Balu, just as Tamil Nadu hits its election season, asking Prime Minister
Narendra Modi to ensure that the provincial council system remains intact. Sri Lankan Tamils
themselves no longer set that much store by India‘s or Tamil Nadu‘s pronouncements on their
problems.
14. Ans:(a)
Sri Lankan leaders do not want to address Tamil political aspirations in Sri Lanka as Tamils will
participate in the elections for the provincial councils and acquire influence over the community.
15. Ans:(d)
India has clearly not accepted its waning influence over Sri Lanka because they still want it to remain
a secret, albeit an open one. They are trying to control Tamil interests in Sri Lanka by stressing on
the validity of the 13th amendment and also trying to persuade the leaders to think according to
them.
16. Ans:(d)
Options ((a) and ((d) are confusing, but the question asks what can be used as a bargaining chip
and the logical answer would be the agreement and not Sri Lanka‘s need for the agreement.
17. Ans:(c)
Waning refers to reduction or decline and waxing refers to increase and so it is the best answer.
18. Ans:(a)
Reconcilation means a settlement of a dispute and therefore option (a) is the best answer.
19. Ans:(c)
A careful reading of the passage clearly indicates that the reference to prime minister and president
is very clearly to the PM and President of Sri Lanka, that is, the Rajpaksha brothers.
20. Ans:(d)
USA has just managed to save itself, therefore it has escaped danger very closely. Option (d) will be
the best answer.
21. Ans:(d)

22. Ans:(b)
Demagoguery refers to political activity or practices that seek support by appealing to the desires
and prejudices of ordinary people rather than by using rational argument.
23. Ans:(d)
The siege has led to shock and mayhem and dilution of faith in free and fair elections. When Biden
came to power through a democratic process, Trump should have stepped down gracefully but he
has undermined the sanctity of the process by indulging in this siege.
24. Ans:(c)
The following lines direct us to the answer:
It came with trademark Trumpisms though—on how ‗facts bore out‘ the reasons why he ‗disagreed
with the outcome of the elections‘. Or him calling the rioters as ―very special‖, the fact that there were
ones who were still mesmerised by Trumpism, still willing to believe that his first term was the
‗greatest‘ in US history!
25. Ans:(d)
Trump refers to rioters as very special because they are supporting him despite his defeat and
refusing to concede to Biden as the new President.
26. Ans:(a)
The word jab means an injection, therefore option (a) is the best answer. A safety jab is essentially
something that will protect one from the virus and therefore the injection of the vaccine is the most
appropriate answer.
27. Ans:(b)
The following lines from the passage direct us to the answer:
In this context, Adar Poonawalla, CEO of Serum Institute, which is partnering to produce the
AstraZeneca vaccine in India, had named three vaccines as efficient, while labelling others safe ―just
like water‖. Taking umbrage, Bharat Biotech CMD Krishna Ella hit back at ―doubters‖ in a press
conference, claiming its vaccine Covaxin was ready for use.
28. Ans:(c)
The question specifically asks ‗how‘ they buried the hatchet. The simple answer is that they issued a
joint statement promising to work together in favour of public good. Therefore option (c) is the best
answer.
29. Ans:(b)
The following lines in the passage direct us to the answer:
The health department would do well to be slow rather than sloppy in green-lighting ‗emergency-use‘
vaccines. Lives are at stake and the country‘s mass immunisation programme is too big to go wrong.
30. Ans:(c)
The spat cost us precious time, means that the war has led to wastage of time that could have been
used for more productive pursuits.
31. Ans:(d)
The word ‗umbrage‘ refers to offense and therefore reconciliation is the most suitable antonym.
32. Ans:(d)
The government and Serum Institute had a deadlock over pricing issues to which Serum Institute
finally yielded.
The following lines in the passage direct us to the answer:
Pricing issues too between the government and the Serum Institute, which is going to market
AstraZeneca‘s Covishield, have caused delays and it appears a large stock of 70 million doses has
built up as the stalemate continued.

GENERAL KNOWLEDGE/CURRENT AFFAIRS

33. Ans:(b)
In July 2019, the European Council nominated Ursula von der Leyen to succeed Jean-Claude
Juncker, and she was elected the 13th President of the European Commission by the
European Parliament on 16 July.
34. Ans:(c)
The EU and European citizenship were established when the Maastricht Treaty came into force
in 1993.
35. Ans:(b)
The European Union (EU) is a political and economic union of 27 member states that are located
primarily in Europe. Before the exit of Britain there were 28 member states.
36. Ans:(d)
Brussels (Belgium) is considered the de facto capital of the European Union, having a long history of
hosting a number of principal EU institutions within its European Quarter. The EU has no official
capital, and no plans to declare one, but Brussels hosts the official seats of the European
Commission, Council of the European Union, and European Council, as well as a seat (officially the
second seat but de facto the most important one) of the European Parliament.
37. Ans:(b)
The EU and European citizenship were established when the Maastricht Treaty came into force in
1993.[19] The EU traces its origins to the European Coal and Steel Community (ECSC) and the
European Economic Community (EEC), established, respectively, by the 1951 Treaty of Paris and
1957 Treaty of Rome.
38. Ans:(d)
The United Kingdom is made up of England, Scotland, Wales and Northern Ireland.
39. Ans:(c)
The budget session is held in February to april every year. During this period, the Budget comes
before Parliament for its deliberation, voting and approval; the departmentally related standing
committees consider the Demands for Grants of ministries/departments and report on the same to
the Houses of Parliament.
40. Ans:(d)
Pralhad Venkatesh Joshi is an Indian politician who is the current union Minister of Coal,
Parliamentary Affairs and Mines of India in the central cabinet under the Prime Minister Modi since
from May 2019.
41. Ans:(a)
The Constitution of India provides for the joint sitting of the Parliament‘s two Houses, the Lok Sabha
and the Rajya Sabha, in order to break any deadlock between the two. Article 108 of the Constitution
talks about a joint Parliament session.
42. Ans:(b)
The winter session of Parliament is held in mid-November to mid-December every year. It is the
shortest session of all.
43. Ans:(b)
Money Bill is defined in Article 110 of the Indian Constitution. Money bills are concerned with
financial matters like taxation, public expenditure, etc. They can be only introduced in Lok Sabha.
44. Ans:(b)
The current president of Nepal is Bidhya Devi Bhandari, who was first elected in October 2015. She
is the first female head of state of Nepal. The president is formally addressed as "The Right
Honourable".
45. Ans:(d)
Khadga Prasad Sharma Oli, more commonly known as K. P. Sharma Oli is a Nepalese politician and
the current Prime Minister of Nepal. He previously served as Prime Minister from 11 October 2015 to
3 August 2016 and was the first elected PM under the newly adopted Constitution of Nepal.
46. Ans:(a)
On May 28, 2008, the newly-elected Constituent Assembly declared Nepal a Federal Democratic
Republic, abolishing the 240-year-old monarchy. The motion for the abolition of the monarchy was
carried by a huge majority: out of 564 members present in the assembly, 560 voted for the motion
while 4 members voted against it.
47. Ans:(b)
Vinay Mohan Kwatra is the Indian Ambassador to Nepal - From March 2020 till date.
48. Ans:(d)
Sashastra Seema Bal is a border patrol organization of India deployed along its border with Nepal
and Bhutan. It is one of the Central Armed Police Forces under the administrative control of the
Ministry of Home Affairs
49. Ans:(a)
Doha is the capital and most populous city of the State of Qatar. It has a population of 956,460. The
city is located on the coast of the Persian Gulf in the east of the country, North of Al Wakrah and
South of Al Khor.
50. Ans:(d)
Operation Enduring Freedom (OEF) was the official name used by the U.S. government for the
Global War on Terrorism. On 7 October 2001, in response to the September 11 attacks, President
George W. Bush announced that airstrikes targeting Al Qaeda and the Taliban had begun in
Afghanistan.Operation Enduring Freedom primarily refers to the War in Afghanistan.
51. Ans:(b)
Osama bin Laden, was a founder of the pan-Islamic militant organization al-Qaeda, designated as a
terrorist group by the United Nations Security Council, the North Atlantic Treaty Organization
(NATO), the European Union, and various countries.
52. Ans:(d)
Osama Bin Laden was shot dead by the Navy Seals of US on 2nd May 2011 in the city of
Abbottabad, Pakistan. The operation was code named as Operation Neptune Spear.
53. Ans:(a)
Ashraf Ghani Ahmadzai is an Afghan politician, academic, and economist who is serving as
President of Afghanistan. He was first elected on 20 September 2014 and was re-elected in the 28
September 2019 presidential election.
54. Ans:(d)
After years of scattered protests, a full-scale revolt broke out in March 1959, and the Dalai Lama
was forced to flee as the uprising was crushed by Chinese troops. On March 31, 1959, he began a
permanent exile in India, settling at Dharamsala, where he established a democratically based
shadow Tibetan government.
55. Ans:(c)
Tibet has been occupied and ruled by China since 1951 in ―a calculated and systematic strategy
aimed at the destruction of their national and cultural identities.‖
56. Ans:(b)
The Nobel Peace Prize 1989 was awarded to The 14th Dalai Lama (Tenzin Gyatso) "for advocating
peaceful solutions based upon tolerance and mutual respect in order to preserve the historical and
cultural heritage of his people."
57. Ans:(c)

58. Ans:(b)
The renminbi is the official currency of the People's Republic of China, and translates to ―people's
money.‖ Its international symbol is CNY (or CNH in Hong Kong; but abbreviated RMB, with the
symbol ¥).
59. Ans:(b)
Explanations: There is currently widespread farmers' protest going on against the three reforms —
Farmers' Produce Trade and Commerce (Promotion and Facilitation) Act, Farmers (Empowerment
and Protection) Agreement on Price Assurance and Farm Services Act, and the Essential
Commodities (Amendment) Act.
Food Processing Industries Minister, Harsimrat Kaur Badal of Shiromani Akali Dal resigned from her
post in protest against these Bills on 17 September 2020.
An Agricultural Produce Market Committee (APMC) is a marketing board established by the State
Governments in India to safeguard the farmers from the exploitation by large retailers, ensuring that
the farm to retail price does not reach excessively high levels.
60. Ans:(b)
Explanations: There is currently widespread farmers' protest going on against the three reforms —
Farmers' Produce Trade and Commerce (Promotion and Facilitation) Act, Farmers (Empowerment
and Protection) Agreement on Price Assurance and Farm Services Act, and the Essential
Commodities (Amendment) Act.
Food Processing Industries Minister, Harsimrat Kaur Badal of Shiromani Akali Dal resigned from her
post in protest against these Bills on 17 September 2020.
An Agricultural Produce Market Committee (APMC) is a marketing board established by the State
Governments in India to safeguard the farmers from the exploitation by large retailers, ensuring that
the farm to retail price does not reach excessively high levels.
61. Ans:(d)
Explanations: There is currently widespread farmers' protest going on against the three reforms —
Farmers' Produce Trade and Commerce (Promotion and Facilitation) Act, Farmers (Empowerment
and Protection) Agreement on Price Assurance and Farm Services Act, and the Essential
Commodities (Amendment) Act.
Food Processing Industries Minister, Harsimrat Kaur Badal of Shiromani Akali Dal resigned from her
post in protest against these Bills on 17 September 2020.
An Agricultural Produce Market Committee (APMC) is a marketing board established by the State
Governments in India to safeguard the farmers from the exploitation by large retailers, ensuring that
the farm to retail price does not reach excessively high levels.
62. Ans:(a)
Explanations: There is currently widespread farmers' protest going on against the three reforms —
Farmers' Produce Trade and Commerce (Promotion and Facilitation) Act, Farmers (Empowerment
and Protection) Agreement on Price Assurance and Farm Services Act, and the Essential
Commodities (Amendment) Act.
Food Processing Industries Minister, Harsimrat Kaur Badal of Shiromani Akali Dal resigned from her
post in protest against these Bills on 17 September 2020.
An Agricultural Produce Market Committee (APMC) is a marketing board established by the State
Governments in India to safeguard the farmers from the exploitation by large retailers, ensuring that
the farm to retail price does not reach excessively high levels.
63. Ans:(b)
Explanations: There is currently widespread farmers' protest going on against the three reforms —
Farmers' Produce Trade and Commerce (Promotion and Facilitation) Act, Farmers (Empowerment
and Protection) Agreement on Price Assurance and Farm Services Act, and the Essential
Commodities (Amendment) Act.
Food Processing Industries Minister, Harsimrat Kaur Badal of Shiromani Akali Dal resigned from her
post in protest against these Bills on 17 September 2020.
An Agricultural Produce Market Committee (APMC) is a marketing board established by the State
Governments in India to safeguard the farmers from the exploitation by large retailers, ensuring that
the farm to retail price does not reach excessively high levels.
64. Ans:(b)
Explanation: Plasma Therapy or Convalescent Plasma Therapy is a clinical trial in which blood is
transfused from recovered COVID-19 patients to a coronavirus patient who is in critical condition.
Due to their crown-like projections on the surfaces coronaviruses got their name. The virus
resembles a crown when viewed under an electron microscope. "Corona‖ in Latin means "halo" or
"crown".
The first case of novel coronavirus was identified in Wuhan, Hubei Province, China.
As per several scientists and researchers, the new strain of the virus is not more deadly but is
70% more transmissible compared to the previous mutations.
65. Ans:(a)
Explanation: Plasma Therapy or Convalescent Plasma Therapy is a clinical trial in which blood is
transfused from recovered COVID-19 patients to a coronavirus patient who is in critical condition.
Due to their crown-like projections on the surfaces coronaviruses got their name. The virus
resembles a crown when viewed under an electron microscope. "Corona‖ in Latin means "halo" or
"crown".
The first case of novel coronavirus was identified in Wuhan, Hubei Province, China.
As per several scientists and researchers, the new strain of the virus is not more deadly but is
70% more transmissible compared to the previous mutations.
66. Ans:(a)
Explanation: Plasma Therapy or Convalescent Plasma Therapy is a clinical trial in which blood is
transfused from recovered COVID-19 patients to a coronavirus patient who is in critical condition.
Due to their crown-like projections on the surfaces coronaviruses got their name. The virus
resembles a crown when viewed under an electron microscope. "Corona‖ in Latin means "halo" or
"crown".
The first case of novel coronavirus was identified in Wuhan, Hubei Province, China.
As per several scientists and researchers, the new strain of the virus is not more deadly but is
70% more transmissible compared to the previous mutations.
67. Ans:(c)
Explanation: Plasma Therapy or Convalescent Plasma Therapy is a clinical trial in which blood is
transfused from recovered COVID-19 patients to a coronavirus patient who is in critical condition.
Due to their crown-like projections on the surfaces coronaviruses got their name. The virus
resembles a crown when viewed under an electron microscope. "Corona‖ in Latin means "halo" or
"crown".
The first case of novel coronavirus was identified in Wuhan, Hubei Province, China.
As per several scientists and researchers, the new strain of the virus is not more deadly but is
70% more transmissible compared to the previous mutations.
QUANTITATIVE TECHNIQUES

68. Ans:(d)
Average annual growth rate = 1950-300300×4×100=165012=137.5%
69. Ans:(d)
Nova‘s turnover in 1998 = 14×300=Rs. 75 crores
Nova‘s turnover in 2002 = Rs. 110 crores
∴ Percentage growth = 110-7575×100=35×10075=46.7%
70. Ans:(c)
At 25% per annum, the number of distributors increases by 56.25% in two years at 20% per annum,
the
Turnover increases by 44% in two years.
∴ Turnover- to –number of distributors for Balife in 2000
= 78×100 lakh1.440.2 lakh1.5625=78001.44×1.56250.2=42318
71. Ans:(c)
Nothing can be inferred about statement 1 as data is given for only top 4 companies.
Statement 2 is definitely true as top 4 companies have approximately 85% of total number of
distributors.
Total turnover of top 3 companies = 472 + 165+ 110 = Rs. 747 crores
Turnover of all other companies = 1950 – 747 = Rs. 1203 crores
Number of companies in the direct selling business will be minimum if all companies other than top 3
companies have turnover almost equal to the turnover of fourth company, i.e. Balife
∴ Minimum number of companies (excluding top 3) = 120378=15.4=16
∴ Minimum number of companies = 16+ 3 = 19
∴ Statement 3 is definitely false.
72. Ans:(b)

Year Ratio(approx.) Percentage increase


1998 25000 -
1999 11500 -ve
2000 15600 4100/ 115> 35%
2001 17100 1500/156 < 10%
2002 17400 300/17100< 10%
73. Ans:(d)
In 2005-06, K = 100-50100 = 12
In 2006-07, K = 150-100150 = 13
In 2007-08, K = 400-200400 = 12
In 2008-09, K = 550-250550 = 611
In 2009-10, K = 700-400700 = 37
74. Ans:(a)
Ref. Q. 70
75. Ans:(c)
Percentage increase in the number of girls = 400-5050 x 100% = 700%
Percentage increase in the number of boys = 300-5050 x 100% = 500%
Difference in percentage point = 700% - 500% = 200%
76. Ans:(c)
Total profit of ‗S‘ in the given week =
(55 + 62.5 + 65.3 + 56.2 + 49.8 + 40) x 1000 = Rs. 3,28,800
Average profit per day = 3,28,800/6 = Rs. 54,800
77. Ans:(c)
On Tuesday, Wednesday and Saturday, the profit is less when compare with previous day.
Profit on Friday compared with Thursday = (14.35 - 12.3) x 1000 = 2050
Profit % = 2050/12300 x 100% = 16.67%
Profit on Thursday compared with Wednesday = (12.3 – 7.6) x 1000 = 4700
Profit % = 4700/7600 x 100% = 61.84%
78. Ans:(c)
Total profit earned by ‗Q‘ and ‗R‘ on Monday, Tuesday and Friday
= [(5.6 + 3.8 + 5) + (32.4 + 28.8 + 49.5)]1000 = 1,25,100
Total profit earned by ‗T‘ and ‗U‘ on Monday, Tuesday and Friday
= [(21.7 + 35.5 + 30) + (10.5 + 12 + 22.3)]1000 = 1,32,000
Required ratio = 125100/132000 = 417/440
79. Ans:(b)
R‘s profit on Wednesday = Rs. 51000
T‘s profit on Saturday = Rs. 15,650
R‘s profit is more than the T‘s profit by Rs. (51000 - 15650) = Rs. 35,350
Required % = 35350/15650 x 100% = 225.88% approx.
80. Ans:(d)
LOGICAL REASONING

81. Ans:(b)
Bruce talks about how people‘s movements have toppled governments and then populist
governments have come in power. Robin Hood didn‘t come to power or anything like that is not
mentioned in the option. Thus, option (a) is not correct. Bruce says that ―Democracy sometimes
needs a shock to the system. Concern that democracy is being eroded by populists can push decent
people to re-engage with politics again.‖ Thus, he believes in a shock-theory for spreading
awareness. Hence, option (b) is the answer.
82. Ans:(a)
Wayne says that populism invites people to demonise ―others‖. Trump was elected President on a
surge of populism of his call to ―Make America Great Again‖ and ―Build a Wall‖ etc. This certainly can
strengthen his argument. Option (b) strengthens Bruce‘s idea about how people coming from elite
class who run the government have little touch with the people. Option (c) has no scope of usage in
Wayne‘s arguments. Option (d) can strengthen Bruce‘s argument. Thus, option (a) is the answer.
83. Ans:(d)
Both options (a) and (b) are incorrect and in fact are opposite to what they actually should be. Bruce
is more concerned towards how a populist government is formed after people get disenchanted from
a non-populist one and he uses examples to validate his arguments. Wayne on the other hand, is
concerned about how populist governments falter in their economic duties, demonise opponents and
uses the observation of Luddites to describe the tenets to populism to disfavor the latter. Thus,
option (d) is the answer.
84. Ans:(a)
The definition certainly strengthens Bruce‘s argument as the rights, wisdom, or virtues of the
common people are something that any democracy will always look up to. Hence, option (a) is the
answer.
85. Ans:(c)
Wayne discredits the Luddites and likens them to populists. He makes it clear that just how the
Luddites were unsuccessful in protesting against the Industrial Revolution, the populists will be
unsuccessful in protesting against globalization. Thus, it can be said that he thinks that populists
always support localization. Statement 1 is correct.
Bruce thinks that a positive populist government will not be corrupt and not just any populist
government. Hence, statement 2 is wrong.
Bruce believes in shock-theory. He believes a populist government can renew the people‘s interest
in democracy thereby renewing the same. Hence, statement 2 is correct.
Thus, option (c) is the correct answer.
86. Ans:(d)
Quote 1 is in support of Wayne‘s argument. It discredits the populists‘ attitude. Quote 2 discredits
populism as having to deal with matters having or showing very little intelligence or judgement. Thus,
Wayne can use this.
Quote 3 is an utter discredit of populism that it doesn‘t reason but just create noise. Wayne can use
this.
Hence, option (D) is the answer.
87. Ans:(c)
The passage starts thus: ―Raindrops might be pumping bacteria into the sky, according to a study
published today in the journal Nature Communications.‖ The part that follows this excerpt mentions
that which was already known (prior to the study) regarding the topic—this includes the fact that
atmospheric bacteria transmit diseases. But how they entered the atmosphere was yet unknown—
and, this was explained in the aforementioned study. Refer to paragraph 4: ―… when a rain drop hits
soil, small bubbles are formed. When the bubbles burst, they send soil into the air with them. And
those tiny bits of soil carry even tinier bacteria…a single rain drop can transfer 0.01 per cent of
bacteria on the soil surface to the atmosphere.‖ Option [C] correctly phrases this. Options 2 and 4
talk of the secondary findings of the study.Refer to paragraph 2: ―This new study builds on previous
research that looked at how droplets interact with porous surfaces.‖ Option [A] is incorrect as it
states the objective of the previous studies from which the current study evolved. Hence, [C].
88. Ans:(d)
Paragraph 7 talks about the experiment with different soil types. The preceding paragraph points to
the reason as to why the experiment was conducted— to know how the amount of bacteria released
differed depending on soil types. Option [A] is factually incorrect as the paragraph clearly states that
Paragraph 7 states that sandy clay soils have ideal properties for aerosol generation. Option [B] can
be discarded as we don‘t have enough information to judge how the density of soil affects aerosol
generation. Option [C] can‘t be taken to be a solid conclusion as researchers don‘t have a valid
reason yet to prove the association between temperature and aerosol generation; refer to paragraph
7: ―Warmer temperatures also seem to propel bacteria more efficiently, though the researchers are
not sure why.‖ Option [D] is the correct option: refer to paragraph 7: ―Buie's team found that sandy
clay soils have best properties for aerosol generation.‖ Aerosol generation refers to the amount of
bacteria sent to the atmosphere. Hence, [D].
89. Ans:(b)
Option [A], though factually correct, doesn‘t answer the question that concerns the role of rainfall
in the suspension of bacteria-filled particles in the air. Refer to paragraph 3: ―Scientists knew that
incidents of people getting this infection went up after a rainfall, but they weren't sure exactly how
rain was involved.Buie‘s study suggests a mechanism—bioaerosolization, or the suspension of
particles that contain living organisms like bacteria.‖ Option [C] too doesn‘t answer the question as it
almost restates what is already known—it doesn‘t state the reason as to why the amount of bacteria
in the atmosphere increases with the frequency of rainfall. Option [D] is incorrect—simply an
interaction between porous surfaces and water doesn‘t release the bacteria into air. Refer to
paragraph 4: ―On certain surfaces, when droplets hit at kind of the right speed the droplets would
trap air.‖ Option [A] is the correct option as it clearly relates the results of the study with the
increased incidence of the disease immediately following a rain. Hence, [B].
90. Ans:(d)
Refer to paragraph 7: "To get this result, Buie took five different soil types (to account for differences
in density)…" Though the passage doesn‘t say how soil density contributes to the amount of bacteria
released into air, we can conclude it to be a contributing factor as it is the only (specified) factor that
distinguishes between the various soil types. Option [B] is clearly a contributing factor—both rainfall
and warm climate are observed (though the latter hasn‘t been proved) to be leading to increased
aerosol generation. Refer to paragraph 4: ―On certain surfaces, when droplets hit at kind of the right
speed the droplets would trap air.‖ Therefore, option [C] is also a factor influencing the phenomenon.
There is just one instance where the size of particles is mentioned, paragraph 7: ―…and treated
them with particles of various sizes (including some benign bacteria).‖ However, we are not informed
as to whether these particles got released into air along with the bacteria, or not. If yes, size can‘t be
a contributing factor. In other words, we have insufficient evidence to prove that size of the bacteria
influences the process. Hence, [D].
91. Ans:(a)
The passage states in the entire passage that unfortunately attractive people have more
opportunities laid out for them as people treat them favourably. This is communicated well in option
(a). Hence (a).
92. Ans:(d)
Both of the statements weaken the arguments given in the passage. The passage talks about the
benefits of being attractive – how beautiful people are treated better and are perceived as having
positive qualities. Neither of the statements is true. Hence (d).
93. Ans:(c)
The author clarifies in the beginning that people are treated differently based on looks. Option (d) is
ruled out.
Option (a) has no corroboratory evidence. Option (b) is out of the purview of the passage. Option (c)
is apt as the passage lists all those benefits. Hence (c).
94. Ans:(d)
The passage starts by saying ―Attractive people are treated differently than the rest of us‖ this means
that all of us are not equally attractive. Option (a) contradicts the passage.
Option (b) if true would mean that all people would be treated equally. Option (d), in fact, contradicts
the passage as people who are attractive tend to have more opportunities and biases that work in
their favour. Hence (d).
95. Ans:(d)
The statement by Lauren Human that we assume attractive people have positive qualities that have
nothing to do with their physical attractiveness, which is sometimes called the halo effect supports all
the given examples. Hence (d).
96. Ans:(b)
The passage actually says that people have too many assumptions of people only on the basis of
their looks. They think good looking people are happier or more successful and generally better
people. Option (b) breaks this by asking us to challenge our assumptions.
Hence (b).
97. Ans:(a)
This is straightforward. The author mentions the presence of genetic material in viruses but does not
say anything about their presence or absence in bacteria. Therefore Option (B) is wrong. The author
also compares viruses to two extreme cases; rock and bacteria– living and non–living. He tries to
place viruses somewhere in between; on the borderline. Hence (A) can be inferred from the
argument.
98. Ans:(a)
Can easily be inferred from the arguments of the two persons.
99. Ans:(c)
Sampras uses the following two independent premises to support his conclusion that Viruses are
non–living. 1. The requirement of being a living thing is that it must be made of cells. 2. Living things
grow.
Options (A) and (B) use only one premise to draw their conclusions. Option (D) concludes by saying
―bacteria can be considered alive‖, which is not as resolute as author‗s conclusion ―Therefore,
viruses are not alive‖. Option (C) also uses two premises to conclude that fungi are living. Therefore,
it parallels the pattern of reasoning used in Sampras‗s argument.
100. Ans:(d)
The main task in doing these questions is to identify what the two speakers overlap on. The right
answer is going to be something they disagree on, everything else isn‗t. Agassi surely believes that
independent evolution should be considered a sufficient condition and not a necessary condition.
Sampras, though, does not talk about evolution in his argument at all. Therefore, Option (A) can be
eliminated. Option (B) also brings forth the issue of independent evolution, which Sampras does not
talk about. Option (C) talks about independent replication, which both of them believe does not occur
in viruses. So, they agree on it.
―Life is the manifestation of a coherent collection of genes that are competent to replicate within the
niche in which they evolve. Viruses fulfil this definition.‖
Agassi‗s argument can be analysed as follows:
Replicate within the niche they evolve --> independent evolution --> Living.
Whereas, Sampras says ―even though they replicate, viruses are more like androids‖ (non–living).
Therefore, they disagree on whether replication in viruses can be considered proof of them being
alive.
101. Ans:(a)
Options (B), (C) and (D) are either mentioned or can be implied from Agassi‗s argument. Agassi
says that recognising viruses primarily as pathogens is wrong; he doesn‗t say that viruses are not
pathogens.
Hence, A.
102. Ans:(d)
Explanation: The game is expected to make the users aware of the problems faced by people in
developing countries. Therefore, if the challenges have no resemblance to the real problems faced
by people in developing countries, the main aim will be defeated. Therefore, choice ‗D‘ is the correct
answer.
‗A‘ throws light on a specific demerit of the introducing online gaming, but does not create any doubt
in achievement of the main goal of spreading awareness amongst people. Hence, it is incorrect.
‗B‘ states of the lowest ratios for number of internet connections per million people but we don‘t know
how much it will affect the online gaming.
‗C‘ shows the game is also earning revenue through users. This has no relevance to the extent of
achievement of the aim of the game.
Thus, choice D is the correct answer.
103. Ans:(c)
We need to undermine the conclusion that people living in cold regions are healthier than those in
hot ones. Option (a) simply strengthens it. Option (b) is out of scope and neither strengthens nor
weakens. Option (d) simply denies the point without providing any rationale. In option (c), the reason
cited for the poor health is not the climate of the region, but the lack of resources, which correctly
weakens the conclusion. Hence (c).
104. Ans:(a)
Option (b) doesn‘t explain how men are fitter than women. And option (c) doesn‘t explain how
population figures affect health and fitness of people. Option (d) would weaken the conclusion. Only
option (a) states how colder regions encourage activities for better fitness levels, thus strengthening
the conclusion. Hence (a).
105. Ans:(a)
The study compares health and fitness levels of people in different regions. Rather than focusing on
the region, it stresses on people from the region. It doesn‘t cover exercise, so (b) can be eliminated.
Both (c) and (d) talk of climates being easier, so they can be eliminated too. Since the study is about
the people, (a) is the correct conclusion. Hence (a).
106. Ans:(c)

107. Ans:(a)
The pattern followed is as under:
In the first step, the word which comes first in the dictionary is placed at the first place and the
remaining words are written in a reverse order.
In the second steep, the word which comes second in the dictionary is placed at the second place
and all words except the first and the second are written in a reverse order. The process continues in
the same manner to give the pass codes for the subsequent batches.
Input : four of the following five form a group
Batch I (10 a.m. to 11 a.m.) : a group form five following the of four
Batch II (11 a.m. to 12 noon) : a five four of the following form group
Batch III (12 noon to 1 p.m.) : a five following group form the of four
Batch IV (1 p.m. to 2 p.m.) : a five following form four of the group
Rest hour (2 p.m. to 3 p.m.)
Batch V (3 p.m. to 4 p.m.) : a five following form four group the of
108. Ans:(c)
The pattern followed is as under:
In the first step, the word which comes first in the dictionary is placed at the first place and the
remaining words are written in a reverse order.
In the second steep, the word which comes second in the dictionary is placed at the second place
and all words except the first and the second are written in a reverse order. The process continues in
the same manner to give the pass codes for the subsequent batches.
Clearly, Batch IV starts at 1 p.m. Thus, in the pass code for Batch IV, first four words are arranged in
alphabetical order. So, as per the pattern, we ought to place the word which comes fifth in the
dictionary at the fifth place and then write all the words except the first five, in reverse order, to get
the pass code for the batch at 3:00 p.m., i,e,. Batch V.
Batch IV: back go here people who settle want to
Batch V: back go here people settle to want who
109. Ans:(d)
Explanations:
Fraction of male persons =70100=710
27 th of 710=710×27=15 males are married
Fraction of married persons =30100=310
Fraction of single females =15310=23
110. Ans:(b)
For selection, Candidate must have a post-graduate degree/diploma in marketing management not
in finance.
111. Ans:(d)
The statement can be restated as All soldiers serve their country. Therefor (d) is correct. Option (a)
may or may not be true.
112. Ans:(d)
LEGAL REASONING

113. Ans:(c)
The correct answer is option (c) because this was a case of false and misleading advertising. Option
(b) is incorrect. Options (a) and (d) are not directly supported by the given facts.
114. Ans:(a)
The correct answer is option (a). The passage holds celebrity endorsers and manufacturers or
service providers accountable to prevent false or misleading advertisements. Future Retail is just the
seller and hence, not liable for false or misleading advertising. Only Bluebull Co. i.e. the
manufacturer and Kirat i.e. the endorser, are liable.
115. Ans:(b)
The correct answer is option (b). The term consumer includes those who make purchases online
and food falls within the purview of the Act. Hence, online food platforms are liable for deficiency in
services. Since Trimato did not refund Birbal for serving him with the wrong order, Birbal can sue
Trimato. Option (c) is subsumed under Option (b)
116. Ans:(d)
The correct answer is option (d) because the purpose of the Act is ―to provide for better protection of
the interests of consumers‖. Farida can sue the institute for a refund because she dropped out of the
course due to the deficient and subpar services and hence, the institute is liable to refund the
remaining amount. The institute did provide medical coaching which makes it a service provider.
Options (b) and (c) are not supported by the given facts or passage.
117. Ans:(c)
The correct answer is option (c). The passage says that ―manufacturers and sellers of products or
services have been made responsible to compensate for any harm caused to a consumer by
defective products, manufactured or sold, or for deficiency in services‖. Hence, both the café and
manufacturer are liable.
118. Ans:(d)
The correct answer is option (d). By refusing to allow H a refund on his payment and discontinuing
the deficient services, ELF indulged in unfair trade practices. The failure to apply for the certificate in
more than 3 years amounts to a deficiency in service. All this makes H entitled to seek refund and
compensation.
119. Ans:(a)
The correct answer is option (a). This is because defection occurs when a legislator defies the party
whip on any issue. Although he has a right to exercise his vote, he cannot go against the party. It is
reasonable to expect that if his party brings out a bill, he must vote in favour of it. Defying this
amounts to defection.
120. Ans:(b)
The correct answer is option (b) because the passage clearly mentions that a legislator is deemed to
have defected when he disobeys the party leadership on a vote. Option (a) is subsumed under
option (b). Moreover, it does not matter if Jayanti Lal has violated party policy because the concern
of disqualification arises due to his failure to comply with party directives in the vote on the bill.
121. Ans:(c)
The correct answer is option (c). The passage states that in "the absence of a formal resignation by
the member, the giving up of membership can be inferred by his conduct". The fact that he started
attending INP meeting tells us that he voluntarily gave up his membership in DJP. Options (a) and
(b) are subsumed in option (c). Option (d) is irrelevant to the question.
122. Ans:(b)
The correct answer is option (b) because if the legislators disagree with the party policy and wish to
join another party to represent the concerns of their voters better, they can face disqualification.
Option (d) is incorrect because voicing the concerns, even if they are not accepted by the party, is
something that the legislators can do. Giving a contrary opinion in party meets does not mean that
they are defying the party in the Parliament.
123. Ans:(a)
The correct answer is option (a). The question does not reveal any additional facts to support that
his coughing, collapse and subsequent death could be attributed to his occupation of mining. Option
(c) by itself is incomplete because it does not provide a direct causal connection to establish a legal
claim. It is subsumed under option (a). The remaining options rely on additional knowledge to prove
a link between Soham‘s occupation and death.
124. Ans:(b)
The correct answer is option (b). The cause of his death could be attributed to his employment i.e.
mining which caused long-term exposure to such compounds. Option (c) is close however, option (b)
is more suited to the principle propounded in the passage i.e. compensation based on connection of
death to the employment. Option (c) only mentions the demerits of mining and does not directly
apply it to Soham‘s death to fasten liability on the company.
125. Ans:(d)
The correct answer is option (d). X was injured while he was working at the construction site he was
employed in. Even if the rod fell from an adjoining construction site, it does not change the fact that
his leg injury was causally connected to the work he was engaged in. Option (c) is incorrect because
it was not the nature of work that caused him an injury, rather it was the fact that he was working at
that site at that point of time.
126. Ans:(b)
The correct answer is option (b). The distinguishing factor in this case is that there was a sufficient
cause behind the delay in filing for compensation. The question indicates that X was not in a position
to file for the claim due to his injury and subsequent treatment and, that he was the only one who
could have filed for a claim. The passage reasons that there must be a sufficient cause and
evidence (reports and receipts in this case) to condone delay, both of which are present in the
question.
127. Ans:(c)
The correct answer is option (c). A legal claim for compensation from the employer arises only if the
accident was causally connected to his employment as a traffic police officer. Mohan met with an
accident after his shift was over and hence, there is no direct link between his accident and
employment to claim compensation. Option (d) is incorrect because it only says that the traffic police
department is not liable for his accident but does not explain why.
128. Ans:(c)
The correct answer is option (c). The basement was a dark place with no electricity, windows or
washroom. They do not have adequate living conditions including basic sanitation. Hence, their right
to life, which include right to shelter is being violated. Option (d) is included in option (c).
129. Ans:(b)
The correct answer is option (b). Their right to shelter is being violated which is a part of Article 21.
Option (a) is incorrect because it only tells us that it is a violation of Article 21 but does not tell us
how the right is being violated. Option (d) is incorrect because they can be asked to vacate a public
space however, they must be provided with alternative housing so that they have a shelter to protect
their life and limb.
130. Ans:(d)
The correct answer is option (d). By promising alternative housing, the Municipality is protecting their
right to shelter. However, right to life also includes the right to livelihood. The Municipality must make
arrangements to ensure that the people have an adequate opportunity to earn a livelihood. By
moving them to a remote, undeveloped part of town, were there will be no opportunity to earn, they
are being deprived of their right to life.
131. Ans:(b)
The correct answer is option (b). Right to life guarantees personal liberty and protects against
exploitation. Although, right to shelter is an implied fundamental right under it, a main ingredient of
this right is the exploitation of poor and vulnerable groups. There is no such exploitation happening
in the question. In fact, by imparting them with life skills, the residents were equipped with the
necessary skills to be able to earn a livelihood and provide a roof over their heads. Therefore, asking
the older residents to vacate the protection home is not a violation of their personal liberty.
132. Ans:(a)
The correct answer is option (a). The question mentions that the river lands are the only source of
their income. Hence, by ordering the farmers to vacate, the government is violating their right to earn
a livelihood. Even if they are being compensated for the same, they will still be deprived of their
ability to earn a living.
133. Ans:(c)
The correct answer is option (c). Right to shelter provides for a home where he has opportunities to
grow physically, mentally, intellectually and spiritually. It includes adequate living space, safe and
decent structure, clean and decent surroundings, sufficient light and other civic amenities, all of
which have been provided to him. A personal laptop is not necessary/essential for adequate living
conditions, that too for attending an online course.
134. Ans:(c)
Reason – The concept of allowing the users to choose players based on their real life performance
in real matches requires skill.
135. Ans:(d)
Reason – Betting on horses in India is legal and even a regulated industry in certain states since it
involves a lot of skills to place bet on horses. The performance of the Horse and the Jockey have to
be kept in mind.
136. Ans:(d)
Reason – Playing Rummy for stakes is not prohibited by law since it requires the player to use
his/her skills to win the game. It is not dependent on chance.
137. Ans:(b)
Reason – The user‘s place bets on a game which is highly dependent on chance making it fall under
gambling which is prohibited in India.
138. Ans:(d)
Reason – Delhi government does provide ‗indirect substantial funding‘ to the DCA since it arranges
the travel and transportation of not only the players but managers also for matches across the
country.
139. Ans:(d)
Reason – The NGO is not receiving ‗substantial direct or indirect‘ funding from the government.
Subsidies in Water bill and Electricity bill do not qualify as substantial funding when compared with
the cost of running the elder care.
140. Ans:(d)
Reason – The trust did receive indirect funding from the government in the form of the 150 acre land
allocated for construction of the Hospital. It was a substantial funding by the government, hence it is
a public authority under the act.
141. Ans:(d)
Reason – The co-operative society even though not formed by an act/law is still registered under the
Maharashtra Cooperative Societies Act, 1960 which has power to some extent with respect to
enquiry and audits, hence it is a public authority.
142. Ans:(d)
Ex: The principle states that consideration is an essential element of the contract. In this case,
however, the contract has no consideration and thus cannot be stated to be complete. Thus, there
does not exist a contract of bailment.
143. Ans:(d)
Ex: The passage states that consideration is an important essential of a contract. In this case
however, it cannot be stated that the considerations were exchanged upon payment of the money
also for it was not as part of agreed terms of a contract. Thus, the essentials of a valid contract have
not been fulfilled.
144. Ans:(a)
Ex: The passage states that past consideration is a valid consideration. Thus, in this case, it can be
said that Radhey giving umbrella earlier was a valid past consideration. There can exist a contract of
bailment.
145. Ans:(a)
Ex: The passage states that a person must take care of the goods as a reasonable person would
have taken. It cannot be said that a reasonable person would have repaired the handle by himself.
Hence Jeevan is liable.
146. Ans:(d)
Ex: The original contract of bailment was only to repair the umbrella‘s handle in exchange for
Raone giving the umbrella to Jeevan. Thus, the cloth repair may not be paid for.
147. Ans:(a)
Ex: The video was shot at a kitchen which was based in Hyderabad. Thus, the violation of the law
occurred at Hyderabad. It will not matter that the sweets are being sold in Chennai for the law
prescribes measures to safely prepare sweets and not sell them.
148. Ans:(a)
Ex: As far as the drug peddling by Rohan is concerned, the same occurred in Delhi and not
Bangalore as the package was received in Delhi. Thus, the Delhi HC should have jurisdiction.
149. Ans:(c)
Ex: NDPS Act is a national legislation and the Supreme Court of India is eligible to look into all
national legislations and concerned violations. Thus, the petition is maintainable at the Supreme
Court of India.
150. Ans:(b)
Ex: The legislation of extra 50% duty is passed by the State assembly and the same is thus liable to
be challenged before the Rajasthan HC.
Legal Edge 18 Mock

Answers & Details

ENGLISH

1. Ans:(a)
Its main theme is ‗love‘. Browning elevates love by praising it for the purity of the feeling that it is.
2. Ans:(c)
Browning has written the poem in the form of a sonnet. The poem has two quatrains (four line
stanzas) and one sestet (six line stanza)
3. Ans:(c)
Through the words ‗let it be for nought‘, the poet beseeches her lover to love her for nothing else, but
love‘s sake alone. She doesn‘t wish to be loved for her physical attributes like her smile, her looks,
and her way of thinking that her lover finds bewitching.
4. Ans:(d)
The mentioned line means love should not be based on pity. A man should not love a woman
because he feels sorry for her pitiable state and wishes to wipe her tears or rid her of her woes. it‘s
not enough to be taken care of by a lover; sympathy, too, is subject to change. If the speaker‘s lover
is moved to love by her tears, and comforts her, and she stops crying—what then?
5. Ans:(a)
The speaker‘s starts by rejecting all the common reasons one might give for being in love
6. Ans:(b)
The speaker‘s starts by rejecting all the common reasons one might give for being in love. While it‘s
a poetic commonplace to observe that physical beauty doesn‘t last, the speaker here suggests that
any kind of reason for loving someone is vulnerable to time. Even being emotionally close or
intellectually simpatico won‘t necessarily last forever; the speaker, the lover, or both may change.
7. Ans:(b)
Earth has not anything to show more fair." This statement is not speaking of nature, but of the city.
The poet goes on to list the beautiful man-made entities therein, such as "Ships, towers, domes,
theatres and temples.
8. Ans:(a)
Personification is the attribution of a personal nature or human characteristics to something non-
human, or the representation of an abstract quality in human form.
9. Ans:(a)
In this poem, the speaker appreciates the beauty of the city, which looks serene early in the
morning, especially since he is seeing the city removed from the usual noisiness and activity
10. Ans:(c)
The speaker declares that he has found the most beautiful scene on earth. You'd have to be
someone with no spiritual sense, no taste for beauty, to pass over the Westminster Bridge that
morning without stopping to marvel at the sights.
11. Ans:(c)
Given how busy and full of activity this major commercial city and center of empire (London) usually
is, it's no wonder Wordsworth is surprised at the calm. If, however, you have ever been in a city or
town in the early morning on a clear, sunny day, before the bustle and activity has begun, you know
what Wordsworth is talking about. What he most marvels at is that this sort of serenity can exist in
an urban area, not just in nature.
12. Ans:(c)
Acute conditions are severe and sudden in onset. This could describe anything from a broken bone
to an asthma attack. A chronic condition, by contrast is a long-developing syndrome, such as
osteoporosis or asthma.
13. Ans:(a)
The following line tells us the same: With the demographic transition underway, from 9% in 2011, the
60-plus population is likely to go up to 20% in 2050.
14. Ans:(d)
The following lines indicates to the same: The Centre‘s National Programme for the Health Care of
Elderly in 2011 promises preventive, curative and rehabilitative care, specialised training of health
professionals, and aims to ensure full participation of the elderly in society. To meet this objective,
geriatric health care services must be made part of the primary health care services.
15. Ans:(d)
The following line indicates to the same:
Traditional support systems have weakened over the years due to fewer children in each family,
increased employment opportunities for women, and the rise of nuclear families.
16. Ans:(c)
The following lines indicate towards the same: ―But the difference in the US is that the violence was
instigated by the President himself.‖
The violence was instigated by Trump, who is the president and guardian of democracy in the
country.
17. Ans:(b)
The following lines indicate towards the same : ―He chose to overlook, and often demonise, those
who came from different places, including native Americans, African Americans and Hispanics.‖ This
shows Trump‘s intolerance towards non native Americans
18. Ans:(d)
There is nothing in the passage which indicates to option D, thus, the correct answer is (d).
19. Ans:(a)
The following lines indicate the same:
―Politicians across the world need to realise that the genie released by false social media information
is out of the bottle. The US must lead by example given the resilience of its democratic institutions in
battling this scourge‖
20. Ans:(a)
Espouse means to adopt of support, thus, A is the right answer
21. Ans:(a)
The poet paints a somber picture of the world. The mood feels lonely and meditative, the speaker
watching as a silent bystander leaning upon the coppice gate — a gate that opens onto the woods.
22. Ans:(d)
Hyperbole is an exaggerated statement or claim not meant to be taken literally. Hence, the correct
answer here is D.
23. Ans:(c)
The poet says that the land‘s sharp features seemed like the century‘s corpse on display.
24. Ans:(a)
The poem often talks about the century‘s corpse, thus the poet is lamenting the century‘s passing
25. Ans:(c)
The poet means that was no cause for such joyful singing—at least no cause was evident in the
world around him. So he thought the bird's happy song carried some secret and holy hope,
something that the thrush knew about but he didn‘t.
26. Ans:(a)
The speaker is walking through a forest whose leaves have turned yellow in autumn.
27. Ans:(b)
The speaker takes the other path, judging it to be just as good a choice as the first, and supposing
that it may even be the better option of the two, since it is grassy and looks less worn than the other
path. Thus, the correct answer is B
28. Ans:(c)
The speaker realizes that both the roads are equally worn and hold the same merit. He exclaims that
he or she is in fact just saving the first road, and will travel it at a later date. Thus, C is the right
answer
29. Ans:(a)
The poet opined that both the roads were same. They were equally travelled and there was no
difference between them. So he got confused. It was difficult to decide which road would be more
convenient and comfortable. He wanted to choose the one that would take him to his intended
destination.

GENERAL KNOWLEDGE/CURRENT AFFAIRS

30. Ans:(b)
Explanation: Benjamin Netanyahu (born 21 October 1949) is an Israeli politician serving as Prime
Minister of Israel since 2009, and previously from 1996 to 1999. Netanyahu is also the Chairman of
the Likud – National Liberal Movement. He is the longest-serving prime minister in Israeli history and
the first pm to be born in Israel after the establishment of the state.
31. Ans:(d)
Explanation: The term "Knesset" is derived from the ancient Knesset HaGdola or "Great Assembly",
which according to Jewish tradition was an assembly of 120 scribes, sages, and prophets, in the
period from the end of the Biblical prophets to the time of the development of Rabbinic Judaism.
32. Ans:(a)
Explanation: Mossad is the national intelligence agency of Israel. It was formed on December 13,
1949, as the Central Institute for Coordination on the recommendation of Prime Minister David Ben-
Gurion.
33. Ans:(b)
Explanation: Jerusalem is a city in the Middle East, on a plateau in the Judaean Mountains
between the Mediterranean and the Dead Sea. It is one of the oldest cities in the world, and is
considered holy to the three major Abrahamic religions—Judaism, Christianity, and Islam. Both
Israel and the Palestinian Authority claim Jerusalem as their capital.
34. Ans:(c)
Explanation: Hebrew is Israel‘s official language, and almost the entire population speaks it either
as native speakers or proficiently as a second language. Arabic is used mainly by Israel's Arab
minority which comprises about one-fifth of the population. Russian is spoken by about 20% of the
Israeli population, mainly by the large immigrant population from the former Soviet Union, and
English is a known foreign language by a significant proportion of the Israeli population.
35. Ans:(b)
Explanation: The Shanghai Cooperation Organisation Charter, formally establishing the
organisation, was signed in June 2002 and entered into force on 19 September 2003. The
original five nations, China, Kazakhstan, Kyrgyzstan, Russia, Tajikistan were previously members of
the Shanghai Five group, founded on 26 April 1996.
36. Ans:(c)
Explanation: India and Pakistan joined Shanghai Cooperation Organization as full members on 9th
June 2017 at a summit in Astana, Kazakhstan.
37. Ans:(c)
Explanation: The Shanghai Cooperation Org has its headquarters in Beijing, China.
38. Ans:(a)
Explanation: There are 8 members in SCO, Russia, China, Kyrgyzstan, Kazakhstan, Tajikistan,
Uzbekistan, India, and Pakistan.
39. Ans:(c)
Explanation: The original five nations, with the exclusion of Uzbekistan, were previously members
of the Shanghai Five group, founded on 26 April 1996. Uzbekistan joined in 2001. Since then, the
organisation has expanded its membership to eight countries when India and Pakistan joined SCO
as full members on 9th June 2017 at a summit in Astana, Kazakhstan.
40. Ans:(d)
Explanation: Assam has been excluded because the National Register of Citizens (NRC) exercise
has already been conducted in the state.
41. Ans:(b)
Explanation: The NPR is being prepared under provisions of the Citizenship Act 1955 and the
Citizenship (Registration of Citizens and Issue of National Identity Cards) Rules.
42. Ans:(a)
Explanation: NPR will be conducted by the Office of the Registrar General of India (RGI) under the
Home Ministry.
43. Ans:(a)
Explanation: A usual resident for the purposes of NPR is a person who has resided in a place for
six months or more and intends to reside there for another six months or more
44. Ans:(d)
Explanation: The Abraham Accord between Israel, the United Arab Emirates and Bahrain is
mediated by the USA. It is the first Arab-Israeli peace deal in 26 years.
45. Ans:(a)
Explanation: The President is also the Commander-in-Chief of the UAE Armed Forces. The current
President is Khalifa bin Zayed Al Nahyan, who became president on 3 November 2004, following the
death of his father, Zayed bin Sultan Al Nahyan.
46. Ans:(b)
Explanation: There were the only two peace deals between Israel and the Arab States in more than
a quarter of a century. Egypt was the first Arab State to sign a peace deal with Israel in 1979. Jordan
signed a peace pact in 1994.
47. Ans:(c)
Explanation: In the Islamic tradition, the Kaaba in Mecca is considered the holiest site, followed by
the Prophet's Mosque in Medina, and Al-Aqsa Mosque in Jerusalem.
48. Ans:(a)
Explanation: Manama is the capital and largest city of the state and emirate of Bahrain.
49. Ans:(c)
Explanation: There are 54 countries in Africa today, according to the United Nations.
50. Ans:(b)
Explanation: The Sahara in North Africa is the largest hot desert in the world. However, Antarctica
is the largest desert in the world.
51. Ans:(b)
Explanation: Rabat is a city and capital of Morocco. One of the country's four imperial cities, it
is located on the Atlantic coast at the mouth of the Wadi Bou Regreg, opposite the city of Sale.
52. Ans:(c)
Explanation: Western Sahara is a territory in Northern Africa, bordered by the North Atlantic Ocean.
53. Ans:(d)
Explanation: The Strait of Gibraltar connects the Atlantic Ocean with the Mediterranean Sea and
separates Spain on the European continent from Morocco on the African continent
54. Ans:(c)
Explanation: The B.1.1.7 lineage which is found to be more transmissible strain of the coronavirus
has been identified in United Kingdom.
55. Ans:(b)
Explanation: In the case of SARS-CoV-2, which is a Ribonucleic acid (RNA) virus, a mutation
means a change in the sequence in which its molecules are arranged.
56. Ans:(d)
Explanation: Coronavirus disease 2019 (COVID-19) is a contagious disease caused by severe
acute respiratory syndrome coronavirus 2 (SARS-CoV-2). The first case was identified in Wuhan,
China, in December 2019. It has since spread worldwide, leading to an ongoing pandemic.
57. Ans:(a)
Explanation: United Kingdom Prime Minister Boris Johnson canceled his visit to India, who was
scheduled to be the chief guest at India's Republic Day parade on January 26.
58. Ans:(b)
Explanation: COVAXIN, India's indigenous COVID-19 vaccine by Bharat Biotech is developed in
collaboration with the Indian Council of Medical Research (ICMR) - National Institute of Virology
(NIV).
59. Ans:(d)
Explanation: In September 2020, President Ram Nath Kovind gave his assent to the three
'Agriculture Bills' that were earlier passed by the Indian Parliament. These Farm Acts are as follows:
Farmers' Produce Trade and Commerce (Promotion and Facilitation) Act, 2020, Farmers
(Empowerment and Protection) Agreement on Price Assurance and Farm Services Act, 2020,
Essential Commodities (Amendment) Act, 2020.
60. Ans:(a)
Explanation: An Agricultural Produce Market Committee (APMC) is a marketing board established
by state governments in India to ensure farmers are safeguarded from exploitation by large retailers,
as well as ensuring the farm to retail price spread does not reach excessively high levels.
61. Ans:(a)
Explanation: The arhtiyas, who are extending critical support to the ongoing farmers‘ agitation, are
the commission agents of Punjab.
62. Ans:(c)
Explanation: Article 246 of the Constitution places ―agriculture‖ and ―markets and fairs‖ in the State
List.
63. Ans:(b)
Explanation: At present, MSP is provided for 23 crops.
7 Cereals: paddy, wheat, maize, sorghum, pearl millet, barley and ragi.
5 Pulses: gram, tur, moong, urad, lentil.
7 Oilseeds: groundnut, rapeseed-mustard, soyabean, seasmum, sunflower, safflower, nigerseed 4
Commercial Crops: copra, sugarcane, cotton and raw jute.
64. Ans:(d)
Explanation: Narendra Singh Tomar is an Indian politician and a member of the Lok Sabha. He is
the Minister of Agriculture & Farmers Welfare, Minister of Rural Development and Minister of Food
Processing Industries.
65. Ans:(a)
Explanation: Dr. M. S. Sahoo, an acclaimed thought leader in the area of securities markets and a
distinguished public servant, currently serves as Chairperson of the Insolvency and Bankruptcy
Board of India.
66. Ans:(c)
Explanation: The Central government introduced the Insolvency and Bankruptcy Code (IBC) in
2016 to resolve claims involving insolvent companies. This was intended to tackle the bad loan
problems that were affecting the banking system.
67. Ans:(c)
Explanation: Moratorium period of 180 days (extendable up to 270 days) for the Company is
provided under IBC. For Startups and small companies the resolution time period is 90 days which
can be extended by 45 days.
68. Ans:(b)
Explanation: Nirmala Sitharaman is an Indian economist and politician serving as the current
Minister of Finance and Corporate Affairs of India. She is a member of the Rajya Sabha, upper
house of the Indian Parliament, since 2014.
QUANTITATIVE TECHNIQUES

69. Ans:(b)
Explanation:

So, length of platform is 130 m


So, total time taken by train A to cross the platform is = (260+130)/ (36*(5/18)) =39 sec
Total time taken by train B is = (130+220)/ (40*5/18) =31.5
So required difference is =39-31.5=7.5
70. Ans:(c)
Explanation:
Time taken by train A to cross post is 260/(36*5/18) =26
Time taken by train E to cross post is 270/(72*5/18) =13.5
So average time taken by two train is = 39.5/2 = 19.75
Time taken by train B to cross a man is = 220/(40*5/18) =19.8
Time taken by train D to cross a man is =280/(54*5/18) =18.66
So average time taken is =38.46/2=19.23
So required difference is =0.52
71. Ans:(b)
Explanation:
Time taken by A and E is = [(260+270)/ {(72-36)*5/18}] =53sec
Time taken by C and B is = [(240+220)*18/85*5] =19.48sec
So total sum = 53+19.48 =72.48sec
72. Ans:(a)
Explanation:
Average speed of A and B is = (36+40)/2=38
Average speed of D and E is = (54+72)/2=63
So required ratio is =38:63
73. Ans:(b)
Explanation:
Production of A company = 15100 × 25 = 3.75 crore
Production of C company = 22100 22 × 25 = 5.5 crore
The total cost of the production of product 1 by companies A and C together- = 25 × 3.75
+ 45 × 5.5 ⇒1.5+4.4 = 5.9crores
74. Ans:(d)
Explanation:
The production of D = 8100 × 25 = 2 crores
The profit earned by company 'D' on product II ⇒ 2 × 58 × 25100 ⇒ 0.3125crore
75. Ans:(a)
Explanation:
Cost of production of product I by company F = 5100 × 25 × 15 1 = 0.25 crore
Cost of production of product II by company D = 8100 × 25 × 58 = 1.25
According to question, 0.251.25 × 100 = 20%
76. Ans:(b)
Explanation:
Profit earned by company G for product I & II together: 12100 × 25
× 46 × 30100 + 12100 × 25 × 26 × 24100
= 0.6 + 0.24 = 0.84 crore = 84 lakh
77. Ans:(d)
Explanation:
Common Explanation:
Let the sum of money he invested in Axis bank = 100x
Then at the end of one year
Amount = 100x×1×20100 + 100x = 120x
The CI of 2 years = 57600
The CI of 1 year = 24000
Difference = 57600 – 24000 = 33600
Now, 33600 – 24000 = 9600
At R% per annum, 24000 gives compound interest of Rs. 9600
24000×R100 = 9600
R = 40% per annum
Following the common explanation,
we get At 40% per annum, 120x gives compound interest of 57600 in two years or Rs. 24000 in one
year
C.I.=P(1+R100)N-P
120x (1 + 40100 ) – 120x = 24000
120x × 1.4 – 120x = 24000
168x – 120x = 48x = 24000
x = 2400048=500
The sum of money he had invested in Axis bank = 100x = 100 × 500 = Rs. 50000
78. Ans:(b)
Explanation:
From the common explanation, we get
The interest, Krishna received from Axis bank = 20x = 20 × 500 = 10,000
The interest from Bandhan bank = 57600
The required sum = 10,000 + 57600 = 67600
79. Ans:(a)
Explanation:
From the common explanation, we get
P = 50000 , R = 40%
1st year = 40% per annum
SI Next 2 years = 20% per annum CI
Amount at the end of 1st year i.e. received from the Axis bank = 50000 + 40% of 50000 = 70000
SI = 70000 – 50000 = 20000
From the Bandhan bank
C.I.=P(1+R100)N-P
C.I.=70000(1+20100)2-70000
CI = 30800
Total interest = 20000 + 30800 = 50800
The interest, Krishna received from Axis bank = 20x = 20 × 500 = 10,000
The interest from Bandhan bank = 57600
The required sum = 10,000 + 57600 = 67600
The required difference = 67600 – 50800 = 16800
80. Ans:(b)
Explanation:
From the common explanation, we get
P = 50000
SI at the end of 3 years = 50000×20×3100 = Rs. 30,000
81. Ans:(c)
Explanation:
From the common explanation, we get
P = 50,000
Let the interest received from the Axis bank = Rs. x
then the first year‘s interest at Bandhan bank = 40% of (50000 + x) = x
20000 + 0.4x = x
0.6x = 20000

82. Ans:(c)
Explanation:
Common Explanation:
Total teacher in college A is 300.
Out of this 70% are asst professors
So asst professors at A are 210, and lab asst is 90.
Total teacher at B is 150.
In B 80% is asst professor
So asst professor at B is 120
So lab asst is 30.
Lab asst in C is 30.
Total teacher at C is 210-10 =200
So asst professors at C are 170.
IN college D, lab asst is 40, which is 25% of total teacher.
So total teacher is 40*100/25 =160
Asst professor at D is 120.

Total asst professor at A and B is 210+120=330


Total lab asst at C and D is 40+30 =70
So the difference is 330-70=260.
83. Ans:(a)
Explanation:
60% asst professor is female at (d)
So female asst professor at D is = 120*60/100 =72
20% lab asst at D is femal(e)
Female lab asst is 40*20/100 =8
Total female teacher is 80.
84. Ans:(d)
Explanation:
At B , out 120, phd holder is 3x and non phd holder
5x.
So , 8x=120
x=15
So total non phd holder is 5*15 =75
85. Ans:(b)
Explanation:
Lab asst at A is 90 and Lab asst at D is 40.
So the difference is 50.
So required percentage is 50/40*100 =125%

LOGICAL REASONING
86. Ans:(a)
The passage in Justin‘s part says that businesses have used the value of social networks. Thus it
would naturally mean that the people in general have great value of social networks in their lives and
this can only be possible because it has become an indispensable tool for them. Thus, option (a) is
correct. Options (b) and (c) are not really associated with the business point-of-view as per the
passage. Hence, option (a) is the answer.
87. Ans:(d)
Statement (1) has been mentioned as people can connect with old friends in Justin‘s argument.
Statement (2) vindicates Justin‘s view that it can benefit businesses. Statement (3) vindicates
Justin‘s view that it can be used to raise one‘s voice. Statement (4), strengthens Justin‘s views of
benefits of social networks, though it has not been mentioned by him that social networks can be
used in rescue operations.
88. Ans:(b)
A person being totally dependent on something means being addicted. Beiber says excessive usage
of social networks leads to a person being addicted to it and not simple usage. Hence option (a) is
wrong. Option (b) is correct as Beiber says that people have isolated their lives behind their online
identities. This means an identity duality is being created – a real life character and an online one.
Thus, option (b) is the answer.
89. Ans:(c)
This is such a statement which will strengthens both Justin and Beiber‘s arguments but weakens
nobody‘s. Justin says social networks have made it easy to communicate and raise ones voice, thus,
this point strengthens his argument. Beiber says that it is now easy to viral a fake news and spread
false information. This is due to easy reach to like-minded people. Thus, option (c) is correct.
90. Ans:(c)
The statement has no effect on the arguments put forth by Justin. Thus, options (a) and (b) are
incorrect. This statement strengthens the argument put forth by Beiber that it can be a distraction for
the employees. Hence, the answer is option (c).
91. Ans:(c)
The author means to say that a person who has seen life without social media will not fall addicted to
it. Thus, the logic is applicable on both the options (a) and (b). Hence, the answer is option (c).
92. Ans:(b)
The whole speech is about the place of dissent in a liberal democracy and within that is
encapsulated the concept of what is actually dissent. Mr. Mehta questions the basic tenets of the
dissent in the notion of dissent in a liberal democracy itself. The dissenters are not discussed in the
passage though they feature as a mere part in the whole question and so does the role of the state
to suppress dissent. So, option (b) is the answer.
93. Ans:(c)
Mr. Mehta makes it amply clear that dissent is not freestanding. He gives the example of Satyagraha
to elucidate the point. That Satyagraha was pegged on to the notion of truth, dissent must be
pegged on to the cause of the dissent. If we look from the dissenter‘s angle then dissent should be
pegged on to the motive or the rightness or the moral judgement of the dissenter. Hence, the answer
should be option (c).
94. Ans:(a)
Option (a) is correct as the author seeks to mean that if a dissent is freestanding then it will mean
that the dissent doesn‘t have any higher cause to a dissent. The people are protesting just for the
sake of it and then anyone can be called a dissenter. A fascist revolting against a democratic
establishment can also be called a dissenter. Option (b) is incorrect as it is the fear of the
establishment of dissent which is self-serving. Option (c) is incorrect as the author doesn‘t mention
that liberal democracies have criminalized dissent, but they are being made to look so. Hence,
option (a) is the answer.
95. Ans:(d)
Mr. Mehta says that there is an inherent self-serving fear among the authorities like the government
for instance that they might be replaced by dissent. Likewise even if there is a dissenting group there
will always be a self-serving fear among the dissenting authority that there can be some internal
dissent which might crop up and take their place. Thus, option (d) is correct and is the answer.
96. Ans:(b)
It is difficult to say Mr. Mehta doesn‘t not believe that democracy requires right to dissent. He
criticizes the lazy approach of people who believe in this notion in a hollow attitude. Thus, option (a)
is wrong. Mr. Mehta does mention that ―If you look at the range of laws that have been enacted in
the last 35-40 years….‖ Thus, option (b) is correct and hence, is our answer.
97. Ans:(b)
Mr. Mehta says that opposition dissenting for opposition‘s sake might be valuable. He gives the
reason that this makes them less easily manageable (combating docility) by the government. This is
the moral judgement or the cause of the opposition. Thus, it strengthens Mr. Mehta‘s statement that
dissent should not be freestanding. Hence, option (b) is the answer.
98. Ans:(c)
Refer to paragraph 2: ―The world‘s first atomic bomb, nicknamed the ―Gadget,‖ was scheduled to be
tested at a carefully selected site codenamed Trinity in a barren valley near Alamogordo, New
Mexico, 200 miles south of Los Alamos.‖ Hence, [c].
99. Ans:(a)
The President at the time of the test was Harry S. Truman as the last paragraph states: ―On July 17,
President Harry S. Truman, only a few months into office following the death of Franklin D.
Roosevelt, would begin meeting with Churchill and Stalin …‖ Hence, option 1 is untrue and is the
answer to the question. Options 2, 3 and 4 are stated directly at various places in the passage.
Hence, [a].
100. Ans:(a)
The correct answer is (a) because the passage refers to Peter Lax as one of the few people
associated with the Project, and states that ‗his memories of the time shed light on the challenge
facing the scientists‘: ―… There was a feeling of great urgency … At the outset, we did not know how
far along the Germans were with the bomb as it turned out, not very far at all. But we felt as if the
fate of the world was in our hands.‖ ‗How far … not very far at all…‘ means they were not close.
Hence, option A can be validly concluded. Option (b) is not correct because the passage says
‗Recruited for his already-evident mathematical prowess, Lax was far from a key player in the
development of the bomb …‖ Option (c) is not correct because the passage is about the nuclear test
and not the dropping of the weapons in war. Option (d) is not correct because the year is incorrect;
the letter was written in 1939 but the Manhattan project was ‗fully authorised in 1942.‘ It may have
existed in a different form prior to 1942. Hence, [A].
101. Ans:(a)
The correct answer is (a); the passage is on a historical event that the public has limited knowledge
about— the first nuclear test named Manhattan Project; throughout, it tries to give us a clearer
picture about the Manhattan Project through the perspective of Peter Lax. Option (b) is not as good
as option (a), as it does not emphasize Peter Lax‘s involvement and his perspective of the event that
has been shaped through his experiences. Options (c) and (d) form only a part of the passage.
Hence, [A].
102. Ans:(c)
Refer to the third paragraph: ―… Among them is Peter Lax, a 94-year-old mathematics genius and
retired professor at New York University, who at the time of the Trinity test was just a 19-year-old
corporal stationed at Los Alamos. … Lax was far from a critical player in the development of the
bomb, …‖ The only supported information is in option C.
Hence, [C].
103. Ans:(a)
This is an easy question.
Option [a] is correct. The last paragraph states, ‗Perhaps it is not yet time for alarm, but surely we
must learn to recognize leaders with autocratic tendencies before they attain power, before it is too
late.‘ Thus, it can be inferred that the strength of demagogues is likely to increase (A demagogue is
apolitical leader who seek support by appealing to popular desires and prejudices rather than by
using rational argument). Retain option [a].
Option [b] is incorrect. The author talks about religious and political groups without differentiating
between them. So [b] is eliminated.
Option [c] is incorrect. While the author cautions against the rise of autocratic leaders in society, he
does not assert that ‗no one benefits from such leaders‘. Eliminate option [c].
Option [d] is incorrect. The passage provides no information directly or implicitly about when ‗sacred
science‘ came into being, nor does it suggest that it is a recent phenomenon. Eliminate option [d].
Hence, the correct answer is option [a].
104. Ans:(b)
This is an easy title question.
Option [1] is inappropriate for the title. It is the name of a movement headed by Jerry Falwell used by
the author as an example. Eliminate option [a].
Option [b] is a relevant title. The passage discusses several autocratic movements and their leaders,
how they exerted power over the masses, the need to resist that power. The passage also cautions
us against the rise of such power in society. Retain option [b].
Option [c] is inappropriate. Though the passage admits that the tendency of religious, political, and
social movements to gain control over 'an ever-increasing base of followers' is widespread, it doesn‘t
suggest that dictators are universal entities. Eliminate option [c].
Option [d] is inappropriate. The relativity of truth is not the main concern of the passage. The
passage is chiefly concerned with explaining the rise of autocratic powers in society.
Hence, the correct answer is option [b].
105. Ans:(b)
This question is of medium difficulty.
Jonestown example is mentioned in Paragraph 3. ‗In either case, these movements begin out of
moral imperative, and change once they achieve power and control. Goodness is an early posture.
Later goodness is transformed into destruction as amply demonstrated by the Jonestown example.‘
Option [a] while not entirely incorrect, does not relate specifically to the context in which the
Jonestown example is mentioned. So option [a] can be ruled out.
Option [b] is correct. Jonestown example is quoted in context to show that the leaders of these
movements misrepresent themselves early on, and show their true colours only later. The writer then
says that the Jonestown example is such a case. Retain option [b].
Option [c] is incorrect, as the example does not imply a reason for the mass conning of the public.
Eliminate option [c].
Option [d] is a general statement which the Jonestown example does not represent. Eliminate
option [d].
Hence, the correct answer is option [b].
106. Ans:(d)
This question is of moderate difficulty. We are asked to identify the author‘s attitude towards his
subject.
Option [a] is incorrect. When we use the word 'pragmatic' to refer to an attitude, it denotes an earthy,
down to earth or practical approach. The word does not accurately describe the author‘s attitude
towards autocrats. Hence option [a] is eliminated.
Option [b] is incorrect as well. ‗Bias‘ would mean a tendency to believe that some people, ideas, etc.
are better than others that usually results in treating some people unfairly. We cannot say that the
writer is treating either the autocrats or the oppressed unfairly. So option [b] can be eliminated.
Option [c] says ‗rational criticism‘. The author‘s criticism of autocrats is based on sound reasoning
and hence justifiable. However, option [c] is inferior to option [d].
Option [d] is correct. It sums up the author‘s attitude towards autocrats, as can be seen in his harsh
criticism of autocrats. ‗They swallow large numbers of people, if their ambitions are realized, in
proselytizing campaigns designed to play on the common fears of the masses.‘ The writer also
exhorts us to stop the autocrats before it is too late.
Hence, the correct answer is option [d].
107. Ans:(b)
(a) Firms can hire and fire but there is no information in the passage to support this inference.
(b) Correct. Refer to the line "Job tenure has not declined and labor mobility did not increase despite
foreign competition, technological change and labor market deregulation". The use of the word
'despite' implies that decrease in job tenure and movement of labor was expected.
(c) No information to support this inference. The paragraph just mentions foreign competition.
(d) No information to support this inference.
108. Ans:(d)
(a) Not mentioned in the passage.
(b) Not mentioned in the passage.
(c) This has not been mentioned as a reason in the passage.
(d) Correct. Explicitly stated in ―they are still reluctant to change residence, let alone emigrate.‖
109. Ans:(c)
(a) Not mentioned in the passage.
(b) Not mentioned in the passage.
(c) Correct. The fears that there will be instability in the job market because of hiring and firing
policies under deregulation is being referred to as the reason why the productivity of the work force
has gone up.
(d) Not mentioned in the passage.
110. Ans:(a)
"It is safe to assume that collective bargaining led to increased wages and, thus, to less hiring and
less flexible labour markets. It is therefore surprising to note that despite the declining share
of unionized labor in two thirds of the OECD countries - unemployment remained stubbornly high."
Paragraph 2, first 3 lines.
(a) Correct. Unionized labor caused unemployment, which arose from higher wages causing "less
hiring" brought on by collective bargaining.
(b) No information to suggest that there was an economic downturn because of unemployment.
(c) It can be inferred to be detrimental only to the unemployed part of the workforce and not the
employed part because the wages were high.
(d) Not mentioned in the passage.
111. Ans:(c)
(a) Not mentioned in the passage.
(b) Not mentioned in the passage.
(c) Correct. Explicitly stated in paragraph 2.
(d) It is not because the agreements were equally good, but because the un-unionized were also
covered by agreements, that unemployment remained high.
112. Ans:(b)
(a) Though the author says that "it seems that it is much..."? in the last line of the passage, he does
not state or suggest that hybrid models do not work because they are contradictory.
(b) Correct. He states that both extreme models have shown positive results while the hybrid models
are yet to show conclusive results.
(c) Not mentioned in the passage.
(d) Not mentioned in the passage.
113. Ans:(b)

114. Ans:(b)
Using statement Venn diagram is formed as
By Venn diagram only conclusion II and III follows.

LEGAL REASONING

115. Ans:(d)
Explanation: The correct answer is (d) – The Sessions Judge was right in rejecting the petition as
the power to accept a petition of an aggrieved person for a refusal on the part of an officer in charge
of a police station to record the information in cases relating to custodial offences other than those
involving death of the victim is in the hands of Chief Judicial Magistrate, as per s.154A of CrPC.
116. Ans:(a)
Explanation: The correct answer is (a) – To avail the sanction under s.197, there must be a
reasonable connection between the act and the discharge of official duty in order to determine that
the act is committed in connection to the official duty. The police officer pushed him the lock up with
reasonable force because he was trying to attack the witness and so he must be absolved.
117. Ans:(d)
Explanation: The correct answer is (d) - S.176(1A) of CrPC deals with cases of death,
disappearance or rape in police custody. In these cases a parallel magisterial inquiry must be carried
out by a Judicial Magistrate.
118. Ans:(a)
Explanation: The correct answer is (a) - To avail the sanction under s.197, there must be a
reasonable connection between the act and the discharge of official duty. The connection here is
that he was doing his duty while stopping him from running from jail. His men rea cannot be proved.
119. Ans:(a)
Explanation: The correct answer is (a) – The word used in s.154A is ‗any person‘ which can include
Legal Aid Centre, or NGO, or any friend or relative. And if the police refuse to register FIR, they can
file a complaint to Sessions judge in case of death of the victim.
120. Ans:(d)
Explanation: Careful reading of the last point in order of preference gives us answer to the fact
specific situation in this question. Support of independent candidates will have to be proved by a
vote of confidence in the assembly. Option (b) is incorrect since anti-defection is not a point of
discussion in the passage. Option (c) refers to forming alliance with only ‗parties‘ and not
independent candidates, a concept alien to the guidelines in the passage.
121. Ans:(c)
Explanation: The passage clearly mentions an order of preference a Governor is required to follow
in case no single party / alliance attains majority. Thus, possibility of formation of a post poll alliance
upon break of a pre-poll alliance always exists. Governor ought to welcome a post poll alliance and
invite formation of government.
122. Ans:(c)
Explanation: Invitation for formation of government upon conclusion of election is of utmost priority
for the Governor. Delaying such invitation by ―prioritizing other official duties‖ over it logically opens
the window for formation/collapse of alliances. Thus, ulterior objective of governor becomes evident.
The overall context of the passage gives indication towards this answer (as to why the Punchhi
Commission guidelines came up)
123. Ans:(d)
Explanation: As per the passage, the Punchhi Commission has clearly delineated the duties of
Governor. The Governor having followed his duties correctly as per the guidelines, the loss of P and
Q in vote of confidence and subsequent government formation by R with support from other
candidates is beyond the control of Governor. The parties may form as many alliances as possible
as long as they are able to win the vote of confidence in the legislative assembly and Governor has
to only do his duties as prescribed.
124. Ans:(b)
Explanation: No, since both the individual are private parties and as per the law under the ‗The
Indian Telegraph Act,1885‘, No private individual has the right to incept the communication of
another individual, only the government of India has the power to take possession of any licensed
telegraphs in case of a public emergency or in the interest of public safety. Hence the argument here
by Razia is false and so is the evidence gained by her.
125. Ans:(c)
Explanation: Yes, since here in this situation Passive wiretapping was done that too with proper
orders which is completely legal because it can only be done by the government. Though some
loses were incurred by Shubham, but it gets countered by Section 5 states, where the government
can order interception of communication and reasonable restrictions can be imposed on the right to
privacy in the interests of the ―sovereignty and integrity of India, the security of the state, friendly
relation with foreign states or public order or for preventing incitement to the commission of an
offence‖.
126. Ans:(c)
Explanation: Both the centre and state have the power to initiate the proceeding. Central
Government or a State Government or any of its officers specially authorised by the Central
Government or the State Government, as the case may be to exercise powers of interception, under
the IT act 2000. Hence both of the governments.
127. Ans:(d)
Explanation: No, since tapping a telephone and recording a telephone call are two very different
things. Call recordings could be exercised by private parties. The whole situation revolves around
the call recording of Sanjay, which do not fall in the scope of under the Indian Telegraph Act,1885
and interception rules, 2009. Thus, the argument here before the court of law are grossly invalid.
128. Ans:(b)
Explanation: No, since firstly information encrypted end to end does not give direct access to the
any the third party and the scope of internet transmitting device do not come under the scope of
‗telegraph‘. Under some other law, this device could be defined under, but not under the
Indian Telegraph Act. Secondly the frequencies all are not under the prescribed source of section
3(1AA), which says that ‗radio waves‘ or ‗Hertzian waves‘ means electromagnetic waves that have
frequencies lower than 3,000 giga-cycles/second propagated in space without artificial guide.
129. Ans:(a)
Explanation: The correct answer is option (a) because Derek made all attempts to save Dalex but
the surgery was unsuccessful due to unforeseeable complications. This does not mean a failure to
take reasonable care hence, option (b) and (d) are incorrect. Option (c) is not substantiated in the
given facts.
130. Ans:(d)
Explanation: The correct answer is option (d). Sheetal is liable for medical negligence because it
was due to her inefficiency on the job that the bag burst causing the patient an infection. None of the
options address this hence, the answer is ‗none of the above‘. Option (c) is irrelevant because the
passage is concerned with medical negligence of professionals and does not mention vicarious
liability.
131. Ans:(c)
Explanation: The correct answer is option (c) because the Pharmacist failed to take the reasonable
standard of care by asking Mr. Dharma if he had lactose intolerance before selling him Z. A
pharmacist is a healthcare service professional as per the given passage but this fact is irrelevant
when asked if the Pharmacist in question was medically negligent. Hence, options (a) and (b) are
irrelevant. Option (d) is incorrect because even if he was unaware, it was his reasonable duty to ask
Mr. Dharma since the Pharmacist knew about the possible side effects of Z.
132. Ans:(b)
Explanation: The correct answer is option (b) because leaving a watch inside the patient‘s stomach
implies a failure in taking a reasonable standard of care. Option (a) is also correct but it is subsumed
in option (b). Options (c) and (d) are incorrect because these facts do not excuse the negligence of
the doctor.
133. Ans:(d)
Explanation: The correct answer is option (d) because the contract is not voidable since it was
caused by a mistake of law in force in India i.e. the Indian law of limitation. The contract is valid.
134. Ans:(a)
Explanation: The correct answer is option (a). A mistake as to a law not in force in India is a
mistake of fact which is void. French Law of Limitation is not a law in force in India.
135. Ans:(b)
Explanation: The correct answer is option (b). Mistake as to the identity of a person amounts to
mistake as to a fact essential to a contract. Mistake of fact is makes the contract void,
136. Ans:(d)
Explanation: The correct answer is option (d) because Jim and Pam did not consent to contract for
the sale of the same thing in the same sense because of a mistake of fact. This renders the contract
void.
137. Ans:(c)
Explanation: Reason – Section 354E added under the new draft bill punishes the acts which
causes ―a sense of danger, intimidation, and fear to a woman‖. The act of pulling and pushing in the
van caused her fear and threatened her.
138. Ans:(c)
Explanation: Reason – X was just holding her hand and had no intention to harm her or sexually
abuse her. Intention is important for the application of Section 354E as the section states ‗intentional
acts‘ creating a sense of danger, intimidation, fear to a woman will be punished.
139. Ans:(c)
Explanation: Reason - Consent is not implied under the existing law (IPC). Emma gave her consent
on the pretext of marriage. David was aware of the same and gave her the impression that he
wishes to marry her. He misrepresented facts to get her consent.
140. Ans:(d)
Explanation: Reason – Section 354E punishes intentional acts which create a sense of danger,
intimidation, fear to a woman. The act of threating by Stuart created a sense of danger for Amy.
141. Ans:(c)
Explanation: The correct answer is (c). If a reasonable complaint has been made against a person,
then it is necessary to arrest the person if it could be justified that the arrest was made to prevent
such person from causing the evidence of the offence to disappear or tampering with such evidence
in any manner.
142. Ans:(b)
Explanation: The correct answer is (b). In case of woman, the police officer shall not touch the
woman for making her arrest, unless the police officer is a female and the circumstances require.
143. Ans:(b)
Explanation: The correct answer is (b). If a reasonable complaint has been made, or credible
information has been received, or a reasonable suspicion exists against any person for committing a
cognizable offence, a police officer can arrest such person without any warrant or order of the
Magistrate, to prevent such person from committing any further offence; for proper investigation of
the offence; or to prevent such person from making any inducement, threat or promise to any person
acquainted with the facts of the case so as to dissuade him from disclosing such facts to the court or
to police officer.
144. Ans:(a)
Explanation: The correct answer is (a). Under Section 41-B every police officer while making an
arrest shall be under the colour of their office and bear an accurate, visible and clear identification of
their name.
145. Ans:(d)
Explanation: The correct answer is (d). To avoid arbitrary arrest, there is a statutory provision u/s.
41-B that every police officer while making an arrest will prepare a memorandum of arrest, attested
by a witness from arrestee‘s family or a respectable member of the locality, where the arrest is
made.
146. Ans:(d)
Explanation: Answer is (D) because every person has a fundamental right to Human dignity. Option
(a) and (b) suggests that foreigners do not have such fundamental rights and option (c) is irrelevant
as an answer demanded by the question.
147. Ans:(b)
Explanation: Answer is (b) Zudon is a human right activist and nothing suggests that he is a lawyer
by profession, on the contrary reason of incapability suggests that he is not a lawyer and precisely
why he is incapable of representing anyone much less Xiano.
148. Ans:(c)
Explanation: Answer is (c) because Xiano is under trial and the guideline to contact anyone about
convict‘s placement in jail was for a ‗convict‘ and Xiano is yet to be convicted. Therefore police owed
no duty to follow such guideline.
149. Ans:(d)
Explanation: Answer is (d) because no valid reason could be given to justify such negligence.
Request of a straw by a person suffering from Parkinson‘s disease is not unreasonable and is
essential to ensure right to life, personal liberty and human dignity, which is a duty of the state.
150. Ans:(b)
Explanation: Answer is (b) because torture is an act that personally destructs the individual, the
refusal to provide Stan with the basic necessity for him personally destructed him and landed him in
hospital.
Legal Edge 19 Mock

Answers & Details

ENGLISH

1. To bemuse means to ―puzzle, confuse, or bewilder (someone)‖. Thus, the correct answer is D.

2. The following line indicates the same: Sec 66A was found to be an impediment to free speech –
and therefore against the right to freedom of expression – as it penalised offensive messages online
while failing to define offensiveness.

3. The following line indicates the same:


―Sec 66A was found to be an impediment to free speech – and therefore against the right to freedom
of expression – as it penalised offensive messages online while failing to define offensiveness.‖
The section penalized offensive messages without defining the term offensive. Thus, it was
ambiguous.

4. The following lines indicate the same:


In the same vein as Sec 66A, the police have also been guilty of liberally using the draconian
National Security Act and sedition charges to curb dissent and curry favour with ruling parties and
social groups linked to them.
The lines suggest that police uses this section to curb free speech, not morally police citizens online.
Thus, the correct answer is D.

5. While all the other options are true statements, they all are statements directing towards the main
underlying theme of the passage, which is C.

6. The speaker does not think that the comparison is appropriate and is unworthy because his friend
is more beautiful and temperate than a summer‘s day. The second line of the sonnet tells us the
same:
―Thou art more lovely and more temperate‖

7. Here the word 'summer' refers to the beauty and youthfulness of the poet's beloved. Hence, the
correct answer is A.

8. The poem says that beauty is not permanent and it falls away even from the most beautiful
people.

9. Here the word ‗this‘ refers to the verse (Sonnet 18) written by William Shakespeare

10. The tone is romantic as it professes love and adoration for the poet‘s lover.

11. The poet in the last says that as long as this sonnet lives, it will give life to you i.e. The beloved.

12. Transient means fleeting. Eternal means ever-lasting. Thus, the correct answer is A.
13. The poem begins with the speaker asking a fearsome tiger what kind of divine being could have
created it: ―What immortal hand or eye/ Could frame they fearful symmetry?‖ Each subsequent
stanza contains further questions, all of which refine this first one. Thus, the correct answer is A

14. The tiger initially appears as a strikingly sensuous image. However, as the poem progresses, it
takes on a symbolic character, and comes to embody the spiritual and moral problem the poem
explores: perfectly beautiful and yet perfectly destructive.

15. The term "fearful symmetry" in "The Tyger" refers to the paradox that the Tyger is both beautiful
and frightening, using its beauty, balance, and grace to act as a ruthless predator. Thus, the correct
answer is C.

16. Throughout the poem, the poet‘s tone reflects of awe, fear and curiosity with respect to the tiger.
Thus, the correct answer is C.

17. Personification is to attribute human qualities to inanimate things. Blake has used personification
in these lines as he considers stars as humans.

18. The poem is full of symbolisms and keeps on reiterating the ―beautiful yet fearful‖ qualities of the
tiger and in extension, the human nature. The poet thus wonders how the same entity could create
the deadly tiger and the innocent lamb at the same time.

19. (d) is factually incorrect, hence, can be eliminated. From, (a) , (b) , & (c) , option (c) is an apt
representation of the young man’s dialogue.

20. Option (b) is the most suitable explanation of the sentence given in the question.

21. Blasphemy is an act of insulting or showing contempt or lack of reverence for god. However, by
definition, an innocuous act is an act not meant to cause harm or upset somebody. Therefore, (a) ,
(b) , (d) are examples of innocuous acts and hence, (c) is the correct answer.

22. Option (a) ‘animosity’ is the synonym of ‘belligerence’ which means ‘aggressive or war like
behaviour’ while options (b) , (c) and (d) are synonyms of ‘peaceful’ which is an antonym of
‘belligerence’.

23. From paragraph (i), we can understand from the young man’s speech that few writers are being
attacked due to their unconventional and critical thoughts reflecting in their work, disliked by the
extremists. Hence, (c ) is the correct answer.

24. Option (a) , (b) , (c) can be inferred from the passage. Hence, (d) is the correct answer.

25. Since, the ‘facebook moment’ is a juncture of younger generation moving from facebook to other
social media apps due to the onset of widespread use of facebook by the older generation. This
shows the prevalence of a generation gap in the society reflecting on social media.
26. Option (d) is the correct answer.
A metaphor is a figure of speech that describes an object or action in a way that isn’t literally true,
but helps explain an idea or make a comparison.
27. Option c) is the most appropriate replacement of [1] in the passage.

28. Option (c) is the correct answer as it gives a complete idea by briefly covering all aspects about
the current situation of Whatsapp given in the passage.
GENERAL KNOWLEDGE

29. The Inner Line Permit was established by the British government under the Bengal Eastern
Frontier Regulation, 1873 to safeguard tribal of eastern part of Bengal. This 1873 Regulation is also
known as ILR or ILP.

30. The ILP system is in force in four Northeastern states — Arunachal Pradesh, Nagaland, Mizoram
and now Manipur — and no Indian citizen can visit any of these states without permit unless he or
she belongs to that state, nor can he or she overstay beyond the period specified in the ILP.

31. Temporary Permit: It will be initially issued for 15 days to tourists, business representatives, or
those who visit for a short period subject to production of valid identity card. Any person applying for
Temporary Permit shall have to pay a fee of Rs 100 in the first instance and Rs 200 for renewal.

32. Union Home Minister Amit Shah has assured the civil society groups that the Citizenship Bill
would provide protection to such regions and states where the Inner Line Permit (ILP) is applicable,
and autonomous administration has been granted under the Sixth Schedule of the Constitution.

33. The Foreigners (Protected Areas) Order 1958 states that a Protected Area Permit (PAP) is
required for non-Indian citizens to visit certain areas in India (mainly in the North-East).Certain
requirements have to be fulfilled in order to get this permit. Indian citizens who are not resident in
these areas need an Inner Line Permit (ILP) to enter these places.

34. Uttarakhand is going to develop India‘s first Snow Leopard Conservation Centre in Uttarkashi
forests. This centre will be developed by the Uttarakhand Forest Department along with the United
Nations Development Program (UNDP). Objective The main objective of this park is conservation,
restoration of Himalayan ecosystems and creating livelihood.

35. The Human Development Report Office releases five composite indices each year: the Human
Development Index (HDI), the Inequality-Adjusted Human Development Index (IHDI), the Gender
Development Index (GDI), the Gender Inequality Index (GII), and the Multidimensional Poverty Index
(MPI).

36. The Historical Index of Human Development (HIHD) was created by Leandro Prados de la
Escosura, and provides a measure of human development from 1870 onwards.

37. Calculation of the index combines four major indicators: life expectancy for health, expected
years of schooling, mean of years of schooling for education and Gross National Income per capita
for standard of living. Every year UNDP ranks countries based on the HDI report released in their
annual report.

38. The least developed country globally with the lowest HDI is Niger, with an HDI of 377. Niger has
widespread malnutrition, and 44.1% of people live below the poverty line.

39. It was adopted by 196 Parties at COP 21 in Paris, on 12 December 2015 and entered into force
on 4 November 2016.

40. The cons of Paris Agreement are – difficult to enfore, impacts some energy-related jobs, different
rules for different countries, may not go far enough to slow global warning.

41. The Paris Agreement is an agreement within the United Nations Framework Convention on
Climate Change, dealing with greenhouse-gas-emissions mitigation, adaptation, and finance.
42. After a three-year delay, the US has become the first nation in the world to formally withdraw
from the Paris climate agreement. On 1 June 2017, US President Donald Trump announced that the
United States would withdraw from the agreement.

43. The initiative was launched by Prime Minister Narendra Modi at the India Africa Summit, and a
meeting of member countries ahead of the 2015 United Nations Climate Change Conference in
Paris in November 2015. The framework agreement of the International Solar Alliance opened for
signatures in Marrakech, Morocco in November 2016, and 200 countries have joined.

44. India is acquiring five Russian Almaz-Antei S-400 Triumf self-propelled surface-to-air (SAM)
systems for $5.5 billion

45. The United States has imposed sanctions on Turkey on Monday over Ankara‘s acquisition of
Russian S-400 air defence systems. Ankara acquired the Russian S-400 ground-to-air defenses in
mid-2019 and says they pose no threat to NATO allies.

46. Russia then annexed Crimea in 2014 following a referendum, and administers it as two federal
subjects of Russia, and claimed it to be 'fully integrated' in July 2015. Ukraine and the majority of
international governments continue to regard Crimea as an integral part of Ukraine.

47. Enacted in 2017, it is a US federal law that imposed sanctions on Iran, North Korea and
Russia.Includes sanctions against countries that engage in significant transactions with Russia‘s
defence and intelligence sectors.

48. It is a mobile, surface-to-air missile system (SAM). It is the most dangerous operationally
deployed modern long-range SAM (MLR SAM) in the world, considered much ahead of the US-
developed Terminal High Altitude Area Defense system (THAAD).

49. The formation of gender identity is not completely understood, many factors have been
suggested as influencing its development. In particular, the extent to which it is determined by
socialization (environmental factors) versus innate (biological) factors is an ongoing debate in
psychology, known as "nature versus nurture". Both factors are thought to play a role. Biological
factors that influence gender identity include pre- and post-natal hormone levels. Social factors
which may influence gender identity include ideas regarding gender roles conveyed by family,
authority figures, mass media, and other influential people in a child's life.

50. Androgynous: Having neither a clearly masculine or feminine appearance or blending masculine
and feminine.

51. The Union Minister of Social Justice & Empowerment will be Chairperson (ex-officio) and Union
Minister of State for Social Justice & Empowerment will be Vice-Chairperson (ex-officio).

52. The case was heard before a two-judge bench of the Supreme Court, composed of Justice K.S.
Panicker Radhakrishnan and Justice Arjan Kumar Sikri.

53. The rights granted to third gender are as follow - Legal Recognition for Third Gender, Legal
Recognition for people transitioning within male/female binary, Public Health and Sanitation, Socio-
Economic Rights.

54. Dec 2019: On 25 December the tunnel, which was known as Rohtang tunnel till then, was
officially renamed as the Atal Tunnel.
55. The project was announced by the then Prime Minister Atal Bihari Vajpayee on 3 June 2000.
The work was entrusted to BRO on 6 May 2002.
56. The electromechanical work in the construction of the 9 km-long Atal Tunnel in Himachal
Pradesh was undertaken by a Turkish company, Savronik. It was the company's first project in India.

57. The tunnel is at an elevation of 3,100 metres (10,171 ft) whereas the Rohtang Pass is at an
elevation of 3,978 metres (13,051 ft). It was inaugurated by Prime Minister Narendra Modi on 3
October 2020. The cost of the entire project is ₹3,200 crore (US$438 million).

58. The work was awarded to a joint venture of AFCONS Infrastructure Limited, an Indian
construction company of Shapoorji Pallonji Group, and STRABAG AG, Austria, in September 2009.
The drilling of the Rohtang Tunnel through the Himalayan ranges began on 28 June 2010 at the
South Portal, 25 km (16 mi) north of Manali.

59. The Ramsar Convention on Wetlands of International Importance Especially as Waterfowl


Habitat is an international treaty for the conservation and sustainable use of wetlands. It is also
known as the Convention on Wetlands. It is named after the city of Ramsar in Iran, where the
convention was signed in 1971

60. Harike Wetland and the lake were formed by constructing the head works across the Sutlej river,
in 1953.

61. Only two wetlands are registered under Montreux Record from India. The first is Loktak Lake and
the second is Keoladeo National park. Chilka lake was earlier a part if it but was later removed.

62. Located on the southwestern part of the delta, the Indian Sundarban constitutes over 60% of the
country's total mangrove forest area. It is the 27th Ramsar Site in India, and with an area of 4,23,000
hectares is now the largest protected wetland in the country.

63. Vembanad - Kol - Wetland – System, one of the three Ramsar sites in Kerala (November 2002),
is the largest estuarine system of the western coastal wetland systems.

QUANTITATIVE TECHNIQUES

64. According to question


Total work = 30 × (X + 20) = 45X
30X + 600 = 45X
X = 40
Total work = 40 × 45 = 1800 units
Desired number of days = 1800/75 = 24 days

65. According to question,


(Y – 5) × (Y + 5) = Y × (Y – 1)
Y2 – 25 = Y2 – Y
Y = 25
So, total amount of work = 25 × (25 – 1) = 25 × 24 = 600 units
Amount of work done by 0.6Y (15 people) in 20 days = 15 × 20 = 300 units
Amount of work left = 600 – 300 = 300 units
Desired time = 20 + 300/(15 + 5) = 20 + 15 = 35 days

66. According to question,


175 × Z2 = 252 × M2
(Z/M)2 = 252/175 = 36/25
Z/M = 6/5
5Z = 6M
Total number of workers = 5Z × 6M = 6M × 6M = 36M2
Desired time = (252M2)/(36M2) = 7 days

67. Common Explanation:


20x + 10x + 22x + 33x + 30x = 2300
Or, 115x = 2300
Or, x = 20

Let the cost price of the article be Rs. x


Therefore, selling price of the article = Rs. (x + 100)
According to the question,
(x + x + 100) = 5100
Or, 2x = 5000
Or, x = Rs. 2500
Therefore, cost price of the article = Rs. 2500
Selling price of the article = (2500 + 100) = Rs. 2600
Therefore, marked price of the article ‗A‘ = (2600 + 400) = Rs. 3000
Amount by which article ‗A‘ is marked up = (3000 – 2500) = Rs. 500
According to the question,
New amount by which article ‗A‘ is marked up = 0.8 × 500 = Rs. 400
New marked price of the article = 2500 + 400 = Rs. 2900
New selling price = 1.14 × 2500 = Rs. 2850
Required discount = 2900 – 2850 = Rs. 50

68. Let CP = x
SP = x - 200
CP + SP = 3000
x + x - 200 = 3000
2x = 3200
x = 1600
CP = 1600 Rs
SP = 1600-200 = 1400 Rs
D = 400 Rs.
MP 1400 + 440 = 1840 Rs.
So Marked up price = MP - SP
= 1840 - 1600
= 240 Rs.
According to the question
New cost price of the article = 1.25 × 1600 = Rs. 2000
New amount by which the article is marked up = 1.15 × 240 = Rs. 276
New marked price of the article = 2000 + 276 = Rs. 2276
New selling price = 2276 – 440 = Rs. 1836
Required loss percentage = {(2000 – 1836)/2000} × 100 = 8.2%

69. Cost price of article ‗F‘ = 2.5 × 660 = Rs. 1650


Selling price of article ‗F‘ = 3000 – 1650 = Rs. 1350
Discount offered on the article = 0.25 × 3850 = Rs. 962.5
Therefore, marked price of the article = 1350 + 962.5 = Rs. 2312.5
Amount by which article ‗F‘ is marked up = 2312.5 – 1650 = Rs. 662.5

70. Let the cost price of article ‗D‘ be Rs. x


Therefore, selling price of the article = (x + 240)
According to the question,
(x + 240 + x) = 6240
Or, 2x = 6000
Or, x = 6000/2 = Rs. 3000
Therefore, cost price of the article = Rs. 3000
Selling price of the article = 3000 + 240 = Rs. 3240
Marked price of the article = 3240 + 660 = Rs. 3900
Percentage by which the article is marked above its cost
price = {(3900 – 3000)/3000} × 100 = 30%
According to the question,
New discount = 660 – 390 = Rs. Rs. 270
New marked price = 3240 + 270 = Rs. 3510
Required cost price = 3510/1.3 = Rs. 2700

71. Let the cost price of the article be Rs. x


Therefore, marked price of the article = Rs. 1.2x
Selling price of the article = Rs. (1.2x – 200)
According to the question,
3850 – (x + 1.2x – 200) = 90
Or, 2.2x = 3960
Or, x = 3960/2.2 = Rs. 1800
Therefore, original selling price of the article = 3850 – 1800 = Rs. 2050
Original marked price = 2050 + 200 = Rs. 2250
Percentage by which article is marked up above its cost price = {(2250 – 1800)/1800} × 100 = 25%

72.

From the table, Vivo are used by 7000 people which is least
73. It can be easily observed from table that the required city is Assam.

74. The total number of persons using Oppos for transportation in Assam, Bihar, Gujarat and
Kerala= 1500 + 2000 + 3000 + 1000 = 7500The average = 7500/4 = 1875.

75. It is clear from the graph as well as table that in both Gujarat and Assam, 12.5% of the total
people use car as their mode of transport.

76. We can use District football player as DF, state football player as SF, District cricket player as
DC, state cricket player as SC, District Hockey player as DH, state hockey player as SH, District
volleyball player as DV, state Volleyball player as SV, District basketball player as DB, state
basketball player as SB. 30% of total student is play football= 2500*30/100 =750 Now DC =DB
Let total basket ball player is 3x and total cricket ball player is 5x.
10% hockey player, 250 student play Hockey,.
Total Volleyball player is 500
SO, 8x =2500 -750-500-250
X=1000
Total cricket player is 625 AND TOTAL Basketball Player is 375.
Now
DC – SH=DH, DC=SH+DH, DC=250[Total hockey player is 250]
DC=DB=250, so SB=125
Ds=625-250 =375
DV-SV=200 AND DV+SV =500
Solving these two DV=350, SV=150
DV=DF
DF=350 so DS= 400.

Let ,
DH =x SH=9x
So, 10x=250
x=25
DH=25 ,SC=375.
Than required percent is: (25/375)*100 =6.66%.
77. Total DF+DC =350+250=600
Total SB+SV=125+150= 275
So the ratio is 600:275 =24:11

78. Total Volleyball player participate in college sports is = 500*28/100 = 140


If one team has 7 players than total team is 140/7 =20

79. Total basketball player is 375. Total DF is 350.


25 more than DF.
So the required percentage = 25/350 x 100 = 7.14%

LOGICAL REASONING

80. Both A and B can be inferred directly from the passage. Therefore, C is the correct answer.

81. The central idea of the passage is that a pluralistic society is one in which different communities
are living together and therefore, it is necessary to sustain diverse views as different communities
have different customs and norms and restriction through censorship are only going to aid to the
singularity of communities.
Therefore, B is the correct answer.
82. A pluralist system is a system in which two or more states, groups, principles, sources of
authority, etc., coexist. Therefore, it will be an irony if people are not allowed to be liberal and instead
are forced to be conservative i.e. narrow minded. Hence, B is correct.

83. If it is true that in a society there are less or no censorship law and the people there are more
inclusive than in a pluralistic society, to be more inclusive it is better not to have censorship.
Therefore, D is the correct answer.

84. Option A is abstract and cannot become an assumption to support the argument. Option B and D
both lead to opposite conclusion to what the author is arguing and therefore cannot be the answer.
Option C states that if there are censorship laws than the artist tries to work within them and
therefore, succumbs to the arbitrary rules and starts to self-censor himself. Therefore, C is the
correct answer.

85. The central idea of the passage is clearly that the rights of women are more important than a
foetus since the foetus is not a conscious being with thoughts and emotions.
Therefore, C is the correct answer.
86. Only option C can be inferred from the passage as all the remaining options are
untrue. Consider the following lines from the passage: ―There are a few blurred lines (blurred -
difficult to understand or separate clearly) when it comes to the topic of abortion primarily when the
exact moment a foetus, becomes a "live human being".‖
87. If the foetus has the ability to think and feel pain then it becomes human and it would be wrong
to terminate it as it would be equivalent to kill a human and therefore, option A makes the authors
argument for abortion weak.

88. The assumption made by the author is that there is not scientific method to know when a foetus
becomes a baby and therefore, he used the words it is a blurred line. Therefore, A is the correct
answer.

89. The conclusion is that the women should have the right over her body to choose whether she
wants to keep or terminate the unborn person. Therefore, D is the correct answer.

90. Options C and D are out of context. Option B is exactly the opposite of what the author says in
the passage. The author, in the passage, speaks on the negative effects of agricultural trade
liberalization.

91. Refer to the last sentence of the passage and to "there is a tendency for low income countries to
forego investment in domestic production and they just rely on international trade to meet their
consumption needs" Hence, D

92. Refer to the comments made by the author regarding agricultural trade liberalization. She feels
that agricultural trade liberalisation has done more harm than good to low income countries. Hence,
it can be inferred that all policies related to trade practices may not always be good. Some may be
harmful. The remaining options cannot be inferred.

93. The passage ends with the reduction in investment. Option D explains the reduction in
investment further and hence, it is the appropriate continuation.

94. Refer to the sentence, "This is because high income countries have better means of productions
through their large-scale and capital intensive systems compared to developing nations' labor-
intensive means of production." The author believes that high income countries have funds and so,
are at an advantageous position vis-a-vis low income countries. If low income countries are provided
funds, they can effectively compete against high income countries in terms of agricultural
production.

95. The author‘s thesis needs to be clearly understood before you can attempt this question. The
thesis is the answer to question. ―Ant colonies possess memory, much like individual ants and most
other living species.‖ We need to look for an option that reinforces the idea that ants have individual
memories and the colony has a collective memory. Option C reinforces both – the individual ants
tracing different routes and returning to different positions reinforces individual memory. Yet the
colony returning to the original shape reinforces collective memory. Option A negates both individual
and collective memory. Options B and D are related to individual memory alone. Hence, [C].

96. The passage mentions something quite contrary to the option. According to the passage it is the
older ants that actually take the younger ones along the trails that are passed on from generation to
generation of ant colonies - ―When the ants emerge in the spring, an older ant goes out with a young
one along the older ant‘s habitual trail. The older ant dies and the younger ant adopts that trail as its
own, thus leading the colony to remember, or reproduce, the previous year‘s trails.‖ Option A is a
trait of red wood ants, paragraph 4, sentence 3- ―Their nests are enormous mounds of pine needles
situated in the same place for decades, occupied by many generations of colonies.‖ Option B is a
trait of the red wood ants, which can be observed from sentence 5 & 6 of paragraph 4; Option D is
also true since the passage mentions red wood ants feeding on the ―excretions of aphids‖ Paragraph
4, sentence 2. Hence, [C].
97. Although the author mentions in paragraph 3 that ―Past events can alter the behaviour of both
individual ants and ant colonies‖, he makes no mention of them having the exact same memory.
This can be further confirmed by the reference the author makes in paragraph 1, describing memory
in ants and ant colonies, ―Like a brain, an ant colony operates without central control. Each is a set
of interacting individuals, either neurons or ants, using simple chemical interactions that in the
aggregate generate their behaviour.‖ Thus, while ant colonies have collective memory, that
generates their behaviour, they are also ‗interacting individuals‘. Option A is incorrect because it is
true based on the passage - in paragraph 4, the author cites the case of the red wood ants, where
―an older ant goes out with a young one along the older ant‘s habitual trail. The older ant dies and
the younger ant adopts the trail as its own, thus leading the colony to remember, or reproduce, the
previous year‘s trails.‖ Option C is incorrect because it is true based on the passage 3 -―Past events
can alter the behaviour of both individual ants and ant colonies.‖ Option D is also true based on the
example of carpenter ants used in paragraph 3, sentence 2 - ―Individual carpenter ants offered a
sugar treat remembered its location for a few minutes; they were likely to return to where the food
had been.‖ Hence, [B].

98. The passage predominantly describes the role of memory in ant colonies and individual ants,
citing examples of redwood ant colonies, Sahara Desert ants and carpenter ants. The author
constructs this argument by building a larger argument around what memory is, and how it shapes
behaviour in trees, humans and ants. Option A is an incorrect option because although the passage
makes reference to the retention and retrieval of memory among ant colonies and individual ants, it
doesn‘t make any reference to codes and means of functioning. Option C can be eliminated because
the passage does not compare the red wood ants with the Sahara Desert Ant - In Paragraph 3 the
author describes the memory of the Sahara desert ant, and in paragraph 4, he describes the red
wood ant, without any comparison between the two. Option D is incorrect because the only
reference the passage makes to the memories of ants is at the opening of the passage, ―Like a
brain, an ant colony operates without central control. Each is a set of interacting individuals, either
neurons or ants, using simple chemical interactions that in the aggregate generate their behaviour.‖
There is no evidence in the passage of linked traces stored in the neurons. Hence, [B].

99. It is best to understand why the author has used this sentence by studying the context - This
sentence appears in paragraph 2 of the passage, where the author is explaining what memory is in
its simplest form. First, the paragraph cites the examples of trees and then the example of the
human body, in supporting the argument that, ―Any living being can exhibit the simplest form of
memory, a change due to past events.‖ (paragraph 2, sentence 1). Option B and C are not correct
options because the author is neither contradicting his argument, nor is he refuting what memory is
conventionally described as. If anything, he is confirming that this is an example of the simplest form
of memory. Option D is not the correct choice , because the author explains how memory involves
the brain and neurons, in paragraph 1 - ― …and brain as computer has led us to take ‗memory‘ to
mean something like the information stored on a hard drive. We know that our memory relies on
changes in how much a set of linked neurons stimulate each other; that it is reinforced somehow
during sleep; and that recent and long-term memory involve different circuits of connected neurons.‖
He then goes on to describe how these changes lead to changes in behaviour, and does not
distinguish between memory in the body v/s memory in the brain. Hence, [A].

100. Choice C is correct because it states that the hotel needs to compare the cost of replacement
of the core mechanism of both types of heating systems.
Choice A, though relevant to the hotel‘s decision to switch from one system to another, does not
have any bearing on the maintenance costs and hence do not answer the question.
Choices B and D are irrelevant to this decision.
Thus, choice C is the correct answer.
101. Methane and electricity are more environmental friendly than petrol or diesel and they causes
less pollution. By using methane in the cars, quantity of harmful emissions has been reduced. It has
reduced the overall pollution. Hence, Answer choice D strengthens the argument.
Choice A mentions the mileage of the cars which cannot be directly concluded to be the reason for
being less polluting. Hence it is incorrect.
The rise in the consumption of electricity doesn‘t reduce pollution if all other conditions remain the
same. Hence, choice B is also incorrect.
Choice C weakens the argument by mentioning that the electricity run cars have a higher carbon
foot print. Hence it is incorrect.Thus, choice D is the correct answer.

102. The passage talks about how our ethical choices are ―rarely black or white‖. As soon as we
confront such a situation (ambiguity), we make a convenient choice – ―selective enough to serve our
interests and yet plausible enough to convince others…‖ that we are not wrong. These decisions are
‗morally ambiguous‘ – not black or white. The paragraph is then properly closed by the name
philosophers give to such decisions. Hence, option B closes the paragraph without leaving loose
ends. ‗Breaking the speed limit‘ in option A does not aptly illustrate ―moral ambiguity.‖ Option C is an
exaggeration as it talks about ‗the culture of cheating spreading quickly.‘ Option D is irrelevant to the
topic, ‗ambiguity about judgment.‘ Hence, [B].

103. The question asks specifically the government‘s role in protecting or empowering the elderly.
Option (a) seems more like a family responsibility. Option (c) seems more like a community
mechanism to improve the wellbeing of elderly people. Option (d) seems to be the role of
educational institutions.
Option (b) with ―pension scheme‖ is a large scale policy that can be taken up at the national level by
the government. Hence, (b).
104. The passage talks about the abuse of the elderly especially those of old women due to various
factors.
Option (a) with ―relaxation and indulgence‖ is contradictory. Option (b) gives a very ideal situation of
old age. Option (d) speaks about the abuse of the younger generation by the old. Option (c) is apt
with ―mistreatment in one form or another.‖ Hence, (c).
105. The passage talks about abuse in old age especially for old women and the responsibility of the
government and families to keep them happy and less lonely. Option (a) contradicts this; (b) is more
pertaining to the lack of caregiver than the well-being of elderly; (c) is apt which shows older people
having further mental health challenges. (d) describes a problem faced by the elderly but loses out to
option (c) which is a stronger support to the argument. Hence, (c).

106. The passage speaks of the grave situation of the elderly in India. The activist presses on the
need of protection of interests of older persons at all levels. The projected data given in option (a) is
the most basic assumption made here. Option (b) with ―joint-family‖ is out of context. Option (c) talks
about care giving children and option (d) talks about India‘s general population increase. Hence, A

107. The passage states ―But more than the government, it ultimately comes down to the loved ones
of the aged who are truly responsible for keeping them happy and healthy.‖ The option that aptly can
be inferred from this is option (d). All options are close, but option (d) manages to capture the
essence by mentioning our responsibility to give love and respect to elders which in turn affects their
health. Hence (d).
108.

makeÞxy

plansÞ lo

nowÞfs

109.

finance Þmn

110.

good Þzt
111.
cl Þ'develop' or 'country

LEGAL REASONING

112. Because this was a domestic matter between a wife and husband and there was no intention to
create legal obligations. Option (a) in itself is not independently related to the given passage. Option
(c) is incorrect because there is no consideration flowing in the question.

113. Because there was no intention to create legal obligations between Sita and Gita. Meeting at
Starbucks to discuss notes does not indicate an intention to create legal obligations pursuant to any
contract.

114. Because the money was the consideration for the bike. Option (a) is incorrect because intention
to create legal obligations is looked at only in the absence of consideration as per the passage.
Here, the amount of Rs. 15,000 was the consideration due in exchange for the bike.

115. There is no real consideration that can be determined in the question however, there was an
intention to create legal obligations. Mrs Jones offering to pay for her daughter's education in return
of her coming to stay with her in England is thus, a valid contract. Even though the relationship is
that of mother-daughter, it does not imply that there is no intention to create legal obligations.

116. Because the Bill bans selling/disposing the cattle for slaughtering or intentionally killing the
cattle an offence. Option (c) is incorrect because even if he is slaughtering cattle for its meat, he is
indeed intentionally killing the cattle.

117. Y is the one who slaughters the cattle and purchases them with the intention of killing them. Z
raises young cows and buffaloes for selling them to Y and for his own consumption. Even if he does
so ethically, he has the intention to kill them. The restaurant owners sell the meat which they
purchase from Y, who butchers the animals only on order. Since the Bill prohibits selling/disposing
the cattle for slaughtering or intentionally killing the cattle an offence, all of them are liable.

118. Dom‘s conviction is invalid because he was merely transporting the cattle and there is nothing
to indicate an intention to slaughter/sell the cattle. Option (d) is incorrect because of the operative
‗would never have‘ which goes beyond the scope of the question.

119. Although John killed the calf, he only did so in good faith to protect the remaining cows and to
end the calf‘s suffering. Intention to kill the calf becomes irrelevant in this case because he was
acting in good faith to protect the other cows.

120. The Act allows killing of cattle, including buffaloes which are above 13 years of age and are
terminally-ill as determined by a veterinary doctor. The question clearly mentions that the buffaloes
were healthy. She cannot send them to the slaughterhouse merely because their upkeep had
become unprofitable.
121. The Bill has exempted cattle operated for any experimental or research purpose at any
government or recognised institute from its provisions. Hence, even if the buffaloes died, the institute
cannot be held liable for slaughter of cattle.

122. Post Emergency, the Parliament cannot infringe upon the fundamental rights guaranteed under
the Constitution, although reasonable restrictions are allowed. This protection does not extend to
other legal rights in the Constitution. They can be amended/impinged upon a valid purpose if they
are not a part of the basic structure. Right to property is not a part of the basic structure of the
Constitution and hence, land can be acquired for such purpose.

123. Self Explanatory

124. Patrika has the right to report freely and without censorship the proceedings in the Legislature.
Therefore, the government cannot ask Patrika to censor its article.

125. The Preamble is a part of the basic structure of the Constitution which cannot be amended by
the Legislature.

126. Article 15(3) is a fundamental right and fundamental rights are a part of the basic structure of
the Constitution and hence, cannot be derogated from or amended. The action is thus invalid.

127. Because kids weren‘t taken away with intent to destroy the DBZ on the contrary military training
is to be provided to increase their immunity; moreover, nothing in the facts suggests otherwise. Not
(d) because the definition is not at all compatible with the one provided in the passage.

128. Because the intent of an act can be proved by the events that follows. 100% of the kids lost
their faith, a whole generation of DBZ has been wiped out, 40% by death and 60% through trauma
and brainwashing moreover the kids were taken forcefully.

129. Killing or taking away the future of a whole community endangers their survival, legally
genocide doesn‘t need literal interpretation. (b) Does not throw more light on the generational
aspect. Not (a) because bargaining isn‘t really a point of genocide.

130. Because according the passage a country can prosecute people for international crimes
regardless of the place of commission and nationality of accused/victims. Not (b) because factually
wrong. Not (c) because (A) is more suitable and deals with a legal concept of universal jurisdiction.
Not (d) because international law does not require domestic law‘s approval.

131. Because the transfer of children was not from one group to another, it is to be noted that the
new government and military has equal representation and therefore the condition of transfer does
not follow. Not (a) because just a forceful taking of children does not constitute genocide. Not (b)
because technically all the children of DBZ were already taken earlier, only the children of
Shippuden were left.

132. amongst the three options (C) is most viable one, as particular conditions were put on DBZ
which destroyed generations of DBZ physical destruction in part.

133. as his words are not intended to incite violence or attack against the government.

134. Inducing people to cease to obey the law and lawful authority. Prime example was Civil
Disobedience movement, it has been considered sedition since Indians protested against the
government and disobeyed the law meanwhile making others do the same.
135. Because a mere slogan stating ‗freedom‘ cannot be construed to mean that a person is trying to
incite people against the government.

136. As second statement is provocative and can instigate violence amongst the people against the
people, as he is urging the people to take some violent and harmful action against the ruling party
and other people.

137. Because in a relationship the partners do not have authority over the body of the other. Since B
was rejecting A‘s advances and did not want to engage in sex that day, so A should not have
coerced her for the same. A does not have authority over B‘s body and cannot engage in sex without
her permission. Since he did do such an act, this can be considered as marital rape.

138. Because in any kind of relationship, be it marriage or a live-in relationship, there can be no
coercion to obtain sex and if there is no mutual consent, then that also serves as reason to not
engage in sex. Since B was rejecting A‘s advances, and also A had coerced B by threatening to
harm their child, thus this act comes under the ambit of marital rape.

139. Because the society believes that the husband has authority over his wife‘s body, and he can
even force her for sex. If the wife refuses the husband then this might destabilize the institution of
marriage.

140. Because it has been said clearly in the passage that forceful sex done only by a husband or a
live-in partner is marital rape. Since, Anuj and Raj were neither in relation to Sheetal, so their act
cannot be considered as marital rape, but can well come within the ambit of rape.

141. The State cannot validly restrict the usage or sale of any property as the same would be
violative of a person‘s fundamental right to property under Art. 19.

142. It is clear that there is no reasonable basis to discriminate between vodka and other forms of
alcohol as provided. It is still possible for the state to justify the imposition of restriction on
individuals.

143. By virtue of the legislation, the consumption of alcoholic liquids has only been limited in public
interest and not absolutely prohibited. Thus, it can be said that the restriction is only proportional in
nature.

144. The passage above states that India is not a military state and only privileges allowed to the
ordinary people should be applicable to the army. Thus, the AFSPA cannot be validly sustained as it
provides special powers to the Army.

145. The passage states that there must be dishonest intention for criminal breach of trust to occur.
In this case, however, criminal breach of trust cannot be constituted till an evidence of dishonest
intent is received.

146. The passage states that there must be dishonest intention for criminal breach of trust to occur.
In this case, however, criminal breach of trust cannot be constituted till an evidence of dishonest
intent is received. A mere phone call is no such evidence.

147. The passage states that there must be dishonest intention. This intention is now established
and thus the acts of X are sufficient to cause criminal breach of trust.
148. The passage states that entrusting means giving a person the possession for a specific
purpose. The peon in this case was entrusted with giving the documents to the superior and is thus
liable.

149. The passage states that there must be dishonest intention. Mere mistake is not criminal breach.
Thus, in this case the peon by spilling water and keeping file outside has committed a mere mistake.
His intention is not dishonest.

150. The passage states that mere taking away of a property with dishonest intention is not
sufficient. There must be some misuse of the property. In this case, till there is no link with the
accused of Anil, it cannot be said that there is a possibility of misuse of the file.
Legal Edge 20 Mock

Answers & Details

ENGLISH

1. The following lines indicate the same:


―Delhi police‘s arrest of young climate activist Disha Ravi on multiple charges like sedition, criminal
conspiracy and promoting enmity between groups – alleging her involvement in a farm agitation
toolkit publicised by Greta Thunberg – appears disproportionate.‖
Disha‘s involvement has just been ―alleged‖ i.e., speculated, without any concrete proof. Thus, the
correct answer is (b)
2. The following line indicates the same:
―Police sought Disha‘s custodial remand to ‗unearth‘ her purported links to a Khalistani group that
allegedly originated the toolkit.‖
3. Feeble means ―markedly lacking in strength‖, which is synonymous with frail. Thus, the correct
answer is (c)
4. The following line indicates the same:
―The police‘s task is to gauge if the yardsticks for invoking sedition like incitement to imminent
violence can be applied against a ragtag bunch of activists led by Disha.‖
The author says that ―incitement to imminent violence‖ is the yardstick for invoking sedition. Thus,
the correct answer is (c)
5. The following lines indicate the same:
―However, Delhi police has only itself to blame for the public outrage now. Disha‘s hush-hush arrest
with no clarity on whether a transit remand from a Bengaluru court was obtained and the remand
hearing in Delhi where she was represented by a legal aid counsel rather than a lawyer of her choice
lead to questions on whether her constitutional rights were prejudiced.‖
Thus, the correct answer is (c)
6. Option A best summarizes the passage and thus, is the right answer.
7. The poem‘s speaker asks an ancient question: why does an all-powerful, loving God allow for so
much suffering in the world? On the one hand, the speaker believes that God is inherently good and
has a plan for everything. On the other hand, he can‘t help but wonder why God would allow human
beings to struggle in ways that seem so never-ending and senseless.
8. The poet says that God‘s ways are inscrutable, i.e., impossible to understand or interpret. Thus,
the correct answer is (b)
9. The following line indicates the same:
―Why flesh that mirrors Him must some day die?
The poet questions as to why would God make humans so similar to himself, the speaker wonders,
and yet unlike himself in that they are doomed to a finite existence. Thus, the correct answer is (b)
10. The last line of the poem indicates the same:
―To make a poet black, and bid him sing!‖ thus, the correct answer is (d)
11. From the start, this poem is expressing something more complicated than either "God is good" or
"if God were good there wouldn''t be suffering." Instead, it''s trying to reconcile these two seemingly
opposite ideas: that God is good, and that he allows humans to suffer terribly.
12. Alliteration is a figure of speech in which the same sound repeats in a group of words.
The alliteration is in the first line, which also draws attention to the relationship between "God" and
"goodness". Thus, the correct answer is (a)
13. The above passage is about how Trump, even after being the mastermind behind the Capitol
riots, was not held accountable because of a lack of sufficient votes which point to a division in the
views of the Senate. Thus, the correct answer is A as it best captures the essence of the passage
14. To rebuke is to express sharp disapproval or criticism of (someone) because of their behavior or
actions, which is similar to the word reprimand. Thus, the correct answer is (a)
15. The following line indicates the same:
―Importantly, seven Republican senators joined their Democratic colleagues in finding Trump guilty,
highlighting bipartisan consensus about the former president‘s actions that led to the right-wing
insurrection‖
The author says that even Republicans held Trump responsible for his actions, which shows
consensus in both the parties. Thus, the correct answer is (c)
16. The idiom means a good news, thus the correct answer is (a)
17. A dictator is a ruler who has absolute power. Thus, the correct answer is (b)
18. a rendezvous is a meeting at an agreed time and place. Thus, the correct answer is (a)
19. As made clear by the title, ―I Have a Rendezvous with Death‖ is a poem about mortality and,
more specifically, the speaker‘s acceptance of death. Thus, the correct answer is (c)
20. the speaker seems to embrace the inevitability of death, seeing it as something as unavoidable
as the onset of spring. And though there is still some uncertainty regarding what the actual
experience of death will be like, the speaker never attempts to escape this fate. Thus, the correct
answer is (a)
21. the author throughout the passage says that his death will take place in Spring, thus the correct
answer is (b)
22. The following line indicates the same:
―And I to my pledged word am true,
I shall not fail that rendezvous‖
The author says that he has pledged to meet death for their rendezvous. Thus, the correct answer is
(c)
23. there is very little regret detectable on the speaker‘s behalf. Instead, the speaker simply states
that he will surely die very soon, and then he tries to imagine what this will be like, wondering if he
will be led into a ―dark land.‖ This is the only kind of uncertainty that characterizes the speaker‘s
thoughts about death, but it doesn‘t change the speaker‘s overall outlook on mortality.
Thus, the correct answer is (b)
24. A "rendezvous" is a meeting or appointment, and this line thus suggests that the speaker''s
encounter with death has already been planned out or decided upon, that the speaker''s death has
been somehow predetermined, though the time is not sure. Thus, the correct answer is (b)
25. Lines 3 and 4 associate the speaker''s upcoming death with the emergence of spring.
Consequently, a sense of juxtaposition arises between the imminent end of the speaker''s life and
the kind of rebirth and continuation that spring represents.
26. The speaker of "I Have a Rendezvous with Death" is a soldier who believes he will soon die on
the battlefield. This can be seen from the fact that the author imagines that he will die either on a
disputed barricade or scarred slope of battered hill or in some flaming town. Thus, the correct
answer is (a)
27. The author says that he has a rendezvous with death, meaning that he knows the imminence of
death. Thus, the correct answer is (c)
28. The author means that his body belongs to England, and he has always breathed English air.
England''s rivers cleansed him, and he was blessed by England''s sun. thus, the correct answer is
(a)
29. It is a deeply patriotic and idealistic poem that expresses a soldier''s love for his homeland—in
this case England, which is portrayed as a kind of nurturing paradise. Indeed, such is the soldier''s
bond with England that he feels his country to be both the origin of his existence and the place to
which his consciousness will return when he dies.
Thus, the correct answer is (c)
30. ―The Soldier‖ explores the bond between a patriotic British soldier and his homeland and is a
powerful expression of patriotic desire and belief in the bond between people and their homeland.
the poem no where mentions horrors of war. Thus, the correct answer is (d)
31. The speaker in this poem is, of course, the "soldier" of the title. The reader learns nothing
specific about this soldier''s circumstances, and that''s because this soldier is a kind of idealized
figure who represents an equally idealized way of considering nationhood and patriotism. Thus, the
correct answer is (b)
32. The speaker is contented with the idea of his death, even embracing it. That''s because he feels
that dying is a noble sacrifice, part of his way of returning the love that his country has showed him.
Thus, the correct answer is (b)

GENERAL KNOWLEDGE AND CURRENT AFFAIRS

33. Tata Electronics has signed a MoU with the Tamil Nadu government to set up a facility for
manufacturing mobile components.
34. It is an Indian multinational conglomerate. The company is headquartered in Mumbai. It was
Founded in the year 1868 by Jamsetji Tata. The company is one of the biggest and oldest industrial
groups of India. Each of the Tata company is operating independently under its own board of
directors and shareholders.
35. All of the above are the salient features of TN‘s Industrial Policy. The Government has in a way
stuck to the usual rhetoric that it will even out the regional industrial development. It will develop the
industrial infrastructure and higher participation of women is expected.
36. Tamil Nadu Ranks 2nd after Maharashtra in terms of GDP in all the Indian States.
37. Tamil Nadu aims to achieve 15% annual growth rate in the manufacturing sector.
38. The Union Defence Minister, Rajnath Singh, inaugurated 57,000 water structures under the
―Jalabhishekam Water Conservation Campaign‖ on February 11, 2021 virtually from New Delhi.
39. Under the campaign, 57,000 water structures inaugurated under the Mahatma Gandhi NREGA
and Pradhan Mantri Krishi Irrigation Scheme.
40. MGNREGA was started off from 2005. At that time Mr. Manmohan Singh was the PM of India.
41. The act was passed with the aim to guarantee the ‗right to work‘.
42. Bhitada village , Jhabua district of Madhya pradesh developed the unique pat system. This
system was devised according to the peculiarities of the terrain to divert water from swift-flowing hill
streams into irrigation channels called pats.
43. The portal, jointly developed by e-Gov Foundation, Bharat Electronics Limited (BEL), Directorate
General Defence Estates (DGDE) and National Informatics Centre (NIC).
44. From the portal, he residents of cantonment areas will be able to avail basic services like the
renewal of leases, application for birth & death certificates, water & sewerage connections, trade
licences, mobile toilet locators and payment of different types of taxes and fees, with just a click of a
button.
45. Kanpur Cantonment is the largest cantonment in India, both by area and population. The area of
the Cantonment is approx. 4243.0084 acres.
46. The ''e-Chhawani'' portal was launched in the presence of Chief of Defence Staff General Bipin
Rawat in Delhi.
47. A cantonment board is a civic administration body in India under control of the Ministry of
Defence. It takes care of mandatory duties such as the provision of public health, water supply,
sanitation, primary education, and street lighting etc.
48. Covaxin has been touted as the first Indian-made vaccine that has been manufactured by the
Bharat Biotech. [1] is Covaxin.
49. India to supply 5 lakh doses of Covid vaccine to Canada after Trudeau''s call to PM Modi.
50. New Delhi plans to supply 1 crore or 10 million vaccine dose Covid vaccine under GAVI''s
(Global Alliance for Vaccines and Immunisation) COVAX facility.
51. GAVI Created in 2000, Gavi is an international organisation - a global Vaccine Alliance, bringing
together public and private sectors with the shared goal of creating equal access to new and
underused vaccines for children living in the world''s poorest countries.
52. Covaxin vaccine is developed using Whole-Virion Inactivated Vero Cell derived platform
technology. Inactivated vaccines do not replicate and are therefore unlikely to revert and cause
pathological effects. They contain dead virus, incapable of infecting people but still able to instruct
the immune system to mount a defensive reaction against an infection.
53. German Watch Situated In Bonn, Germany Is an NGO That Releases Climate Risk Index.
54. Mario Draghi was a former European Central Bank chief. He formally sworn in as Italy‘s new
prime minister on February 13, 2021 amid the deadly coronavirus pandemic and crippling recession
in the country.
55. According to Climate Risk Index Mozambique, Zimbabwe and the Bahamas were the most
affected countries in 2019 followed by Japan, Malawi and the Islamic Republic of Afghanistan.
56. As per the index, India has been ranked 7th Global Climate Risk Report Index 2021.
57. In the index for 2020, Sweden led the ranking, followed by Denmark and Morocco. The last three
ranks were taken by the United States of America, Saudi Arabia and Chinese Taipei.
58. The 10th edition of the festival was held in Madhya Pradesh in October 2019.
59. The Rashtriya Sanskriti Mahotsav (RSM) flagship festival of this ministry organized since 2015
with active participation of seven zonal centre.
60. Prahlad Singh Mulayam Singh Patel is an Indian politician who is the current Minister of State for
Culture, Tourism of India.
61. It covers the fusion of folk art forms from 22 states.
62. The flagship and premier festival of Ministry of Culture, the Rashtriya Sanskriti Mahotsav is to be
conducted in Jabalpur, Madhya Pradesh on 14th October, 2019. It is a part of the Ek Bharat
Shreshta Bharat initiative that is being organized between 14th October, 2019 and 21st October
2019.
The festival began in 2015. It showcases rich cultural heritage of India. It includes classical and
contemporary handicrafts, painting, arts-folk, sculpture, etc. It covers the fushion of folk art forms
from 22 states and will offer the best chance to experience the emerging virtuosity.
It was announced by PM Modi in October 2015 on the occasion of 140th birth anniversary of Sardar
Vallabhbhai Patel. The following are the main objectives of the initiative:
To celebrate unity and diversity of the Nation
To promote the spirit of national integration
To show case the rich heritage and culture, traditions and customs of the country
63. The agency headquarters are in Hyderabad, Telangana, where it moved from Delhi in 2001.
64. Shubash Chandra Khuntia, former chief secretary to the Karnataka government, has been
appointed as the new chairman of the Insurance Regulatory and Development Authority of India
(Irdai)
65. DigiLocker is a cloud based document storage system of the Government of India. Using this
facility, one can store important documents such as PAN cards, mark sheets, insurance policies,
certificates, driving license, etc. in an electronic format.
66. DigiLocker is an Indian digitization online service provided by Ministry of Electronics and
Information Technology, Government of India under its Digital India initiative. It will work on both IOS
and Windows.
67. Launched in the year 2015, the novel digiLocker has been provided by the Ministry of Electronics
and Information Technology (MeitY).

LEGAL REASONING

68. Attorney-client privilege encourages clients to make "full and frank" disclosures to their attorneys
for proper representation. Option (c) is incorrect because it is not just that he was a victim of fraud
that is relevant, but the entirety of the facts. He can share this information with his lawyer because all
their communication will be privileged and not because of the mere fact that he was a victim of fraud.
It is entering into a client-counsel relationship that makes the disclosure confidential.
69. The passage clearly mentions that the communication between an attorney and client is
confidential and hence, cannot be disclosed to Baburao. The relationship of agency cannot be
determined on the basis of the given facts. Option (d) is irrelevant because each lawyer-client
relationship is separate and distinct. Merely hiring the same lawyer does not reduce the quantum of
confidentiality.
70. Even if Baburao engages Raju‘s lawyer, the information shared by Raju to his lawyer will still be
confidential because they form an independent client-attorney relationship.
71. because the passage clearly mentions that even the documents shared by the client with his
lawyer are protected and confidential as they fall within the scope of privilege under Section 126 of
the Evidence Act. Option (a) is incorrect because Louis Litt is not an in-house counsel of Anuradha
Enterprises because he was engaged only for this case as per the facts.
72. because the information disclosed between Mike and Harvey is confidential under attorney-client
relationship and cannot be forced out of Mike. Prison-in-charges are not exempt under the Evidence
Act.
73. Mujrim‘s disobedience was the result of some compelling circumstances i.e. the traffic jam which
was beyond his control as the question clearly mentions that he started on time. Due to the same,
his disobedience was not wilful or intentional.
74. as the passage clearly mentions that ―that a person does not commit contempt of court if during
the pendency of certain proceedings, he takes recourse to other judicial proceedings open to him,
even though the latter proceedings put the other party at a loss‖. Therefore, by settling outside court,
she did not commit contempt of court.
75. Dholu was discussing offers of sale and the question mentions that he urgently needed money to
pay for his wife‘s surgery. This forms a compelling circumstance for the Court to consider.
76. Since Dholu was a rich businessman, a fact expressly mentioned in the question, he could have
paid for his wife‘s surgery without selling the property. Therefore, his disobedience was wilful and
intentional and hence, the Court could execute the decree of a contempt proceeding.
77. Since Ranjhana was caught up in the mob and badly injured, he couldn‘t have possibly come to
court the next day. This forms a compelling circumstance in against punishing him for wilful
disobedience as his disobedience of the bail conditions was not actually wilful in nature.
78. There isn‘t any information given in the factual situation about the condition of insanity. Therefore
the defense of unsoundness will not sustain as he was in his senses and the act was done under the
fit of rage.
79. Gordon‘s action clearly shows that his mental condition was not of such a nature as to prevent
him from understanding what he had done was an act of violent nature and one that was clearly
prohibited by law.
80. Applying the information giving in the passage, it is clear that Ali was incapable of understanding
his act and therefore cannot be held to have committed murder as he was suffering from insanity.
81. Anay would not be liable as he committed the act under unsoundness of mind. Sonu would not
be liable as no information related to the negligence is given in the passage.
82. As per the given information, nothing is an offence which is done by a person who, at the time of
doing it, by reason of unsoundness of mind, is incapable of knowing the nature of the act, or that he
is doing what is either wrong or contrary to law. In the instant case, Rao was not in the sound state
of mind and he didn‘t know the nature of the act he has committed, therefore he is not liable for any
offence.
83. option A is in contradiction with the views presented in the passage. Option B is general and
does not answers the question directly. Option C does not provide the correct and relevant reason.
84. option has no legal argument hence is incorrect. Option C and D are inconsistent with the views
expressed in the passage and do not answer the question directly.
85. option A is irrelevant to the question asked. Option b is factually incorrect and also irrelevant.
Option is again not irrelevant and out of context.
86. option A and B are not expressed by the author. Even their contradictory statements have been
used.
87. the matter involves a question of law hence option A is correct. Option B is out of context option
C is in accordance with the passage but in contradiction with the factual situation presented in the
question.
88. option D is out of the scope, option B and C are contradictory to the views expressed in the
passage.
89. none of the statements have been expressly or impliedly are evident except the option (d)
90. in a private suit the plaintiff has a choice to file or not file a suit. Option C is incorrect as it is not a
legal reason and just a general statement.
91. any other options except option A, are not expressly supported by the authors.
92. there was no selective prosecution as there was no group to select from. Option B and C are
incorrect because they do not provide a sound legal reasoning as per the passage.
93. Since Amy was an earning member and ran a business together with Jake, Options A and B are
incorrect. Option D is irrelevant and hence incorrect. Hence, Option C is the correct answer.
94. Option A is incorrect because although the exclusion of such work from economic analysis has
been mentioned in the Hon‘ble Judge‘s observation, the reasons are not. Similarly, Option B and D
are incorrect because there is no mention of such ‗challenges‘, and neither does the paragraph
actually discuss the methods or criteria employed in calculating the notional income of home-
makers. Option C is clearly stated in the second paragraph, hence is the correct answer.
95. Option B is incorrect, as the passage does not mention additional incomes, or primary incomes.
Option C is partially correct, however not the correct choice as there is no indication in the passage
that the concept of ‗notional income‘ is limited only to Motor Accident Compensation Claims. Hence,
Option A is the correct answer.
96. Option A is although very similar to Option B, is not the correct answer as this criterion will only
apply to earning members in the case their income cannot be proved. Although Saroj ran a small
business that wasn‘t really catered to earn profits, it still gathered earnings and made her an earning
member and not a non-earning home-maker, hence Option C is also incorrect. Hence Option B is
the correct answer.
97. While the explanations offered in Options B, C and D might have been correct in deciding the
liability of an offense had adultery been an offense, the paragraph clearly states that Section 497 is
unconstitutional and hence, struck off. Hence, nobody can be made liable for the offence of adultery.
Thus, Option A is the correct answer.
98. As is evident by the last paragraph, it degrades women by reducing them to a commodity, hence
Option C is the option that corresponds the most with that idea. Option A is correct, but that does not
make it anti-woman. Option B is mentioned nowhere in the passage. Hence, Option C is the correct
answer.
99. Option A is incorrect, as the judgement also mentions that adultery can be a ground or
dissolution of marriages despite it being termed as unconstitutional. Option D is incorrect, as it does
not answer the question of the validity of the defence. Option B is incorrect as it is irrelevant. Hence,
Option C is the correct answer.
100. Meet will not be liable for adultery as it is no longer an offence, however he may be liable for
‗unbecoming conduct‘ under the Armed Forces Act. Hence, Option C is the correct answer.
101. The bill will not apply to anyone as currently it is only a bill and unless it is passed by both the
houses and given the presidential assent it shall not come into force in the union of India.
102. Since the bill only encompasses the data pertaining to natural persons which leads to their
identification either directly or indirectly, the other 3 options do not adhere so.
103. As mentioned above, the bill encompasses all the data pertaining to natural persons that
identify them either directly or otherwise. In the present question, Mrs Rai was involved as an
employee in ZXV making the data highly sensitive to her person.
104. As the Bill clearly encompasses protection of sensitive data, in the form of transparency of data
and reporting of all breaches, Foodpallate should inform the authorities about said breach and take
all the precautions otherwise under the ambit of their act.

LOGICAL REASONING

105. The media had proceeded on a loud discourse that had almost notched up to cacophonous
decibels of noise. The silence on the other side was the reflection of the other side resembling the
reflecting back of the ultrasonic sounds that gets reflected in the process of echolocation. This was
not an act of giving up, nor was it a representation of the voice gone silent. Hence, options (a) and
(b) are not the answers. Option (d) is not the answer. Option (c) is the most suitable answer.
106. The author maintains that though most of us were under physical lockdowns imposed to
contain the spread of the virus, our touch and vision restricted, we sensed and responded to
intensities from a distance. People responded publicly and at the same time. This was an
interruption. Thus option (d) is the answer.
107. Options C and D are part of cacophony described at the beginning of the last paragraph.
Hence, both are eliminated.
To imagine pandemic media as ‗echolocating‘ devices – which don‘t burden us with testimony but
equip us to move collectively towards safety and nourishment – may be key to a generative leap
away from what looks like a wasteland right now. Thus, echolocating (what the author is referring to)
doesn‘t mean a tit for tat attitude, but a reaction which can save the entity from a bad action. Thus,
option (b) is the answer.
108. This line in the passage lends support to the fact that the Janta Curfew was an interruption to
the way of life of 2020. The result of which was that
―Less than 72 hours later he appeared on television again, this time with the news that the nation wa
s going into a complete lockdown.” Thus, option (b) is the correct answer.
109. The passage is silent about the vision of bats. Hence, option C is incorrect.
The author says – ―The cacophony of ‗nationalism‘ and bullish demonstrations of ‗togetherness‘ –
duly amplified by television and social media – was an impediment to the possibilities of critical
hearing. This increased the distress of medical, migrant and contract workers.‖ It means a critical
hearing would have wanted to speak out for the plight of the medical, migrant and contract workers,
but they were ignored by the media.
Hence, the answer is option (b).
110. In this argument, the author compares the politics with the game like cricket or football and
wants to conclude there can be no democracy in two party systems. Option C strengthens the fact
as if it were true than the two party systems in a democracy would not be good as it limits the choice
of voters. Therefore, option (C) is the correct answer.
111. The contention of the argument is that there cannot be democracy in a two party system
because politics is game unlike the other traditional games. So, if it is true that even politics is played
like other games, then the argument is weakened.
112. That politics is not a game is mentioned in the premise. The premise says that there can be
democracy in a two party system if it is a game like Cricket. Its underlying assumption is that Cricket
is a game played by two teams. Therefore, option (C) is the correct answer.
113. Option (A), (C) and (D) are further weakening the argument for two party system. Option (B) is
the correct answer because it is providing a benefit of having a two party system in a country
114. Retrospect to politicians and corporate leaders, the passage clearly hints that they didn''t take
action on climate change out of fear of loss to economy. Thus, they turn their face away from an
imperative because of a potential loss. The same pattern of reasoning is exhibited in option C.
Hence, C
115. The passage states that ''Global air traffic decreased by 4.3% in February with cancellations of
tens of thousands of flights to affected areas. But Rob Jackson, the chair of Global Carbon Project,
said this would only be meaningful if it led to long-term behavioural change, particularly in aviation.''
116. Options B, C and D cannot be inferred from the passage.
The passage shows how the health crisis caused by COVID has compelled people (masses) to
change behaviour radically. Most of these changes are for the better and should have been done
much earlier.
Hence, A is correct.
117. It is mentioned that "hefty 25% carbon dioxide cuts can bring less traffic and cleaner air with
only a small reduction in economic growth" This shows that economy was used as a reason against
implementation of carbon reduction policies.
118. Passage says "More carbon savings will come from the cancellations of international
conferences" Hence, a decrease in international gatherings would dramatically save carbon
emmission. This also means that such gatherings in the past have contributed to climate change.
Option A is against the passage.
Option C is extraneous.
Hence, B.
119. This choice does not mean there will be a huge demand for electric cars.
(B) This option does not necessarily mean that people would buy electric cars.
(C) This selection does not explain the huge demand on which the argument is based.
(D) Correct. The argument is based on the assumption that high levels of awareness will translate
into people switching to electric cars so automobile manufacturers will have to start making electric
cars to maintain their market share.
120. (A) The passage is concerned with the ineffectiveness of the warning mechanisms, not the
sales of cigarettes or any effort to declare them illegal.
(B) Correct. This choice properly completes the passage as it states that new mechanisms to warn
smokers should be devised since the current methods are ineffective.
(C) There is no reference in the passage to smokers being unconcerned about the damage to their
health.
(D) The passage says that 78.7% of respondents find the warnings ineffective. From this, option (D)
concludes that 21.3% (the rest of the respondents) notice the "warnings". This is incorrect because
finding the warnings "ineffective" does not mean "not noticing them"
121. (A) This information does not weaken the argument because there is no information whether
Singapore will retain the Jet Air brand name after the takeover.
(B) This information strengthens the argument.
(C) This option presents information that is irrelevant to the passage and the question.
(D) Correct. The author makes the argument that Singapore ticket sales will go up after it acquires
Jet Air based on the evidence that its ticket sales had risen by 60% after it acquired Kingfisher
Airways. This option says that, last year, Singapore Airways offered the lowest possible airfares by
giving a 50 percent discount on all its tickets. This information suggests that the rise in ticket sales is
related to the drop in prices rather than the acquisition of Kingfisher Airways.
122. Only C follows.
(A) does not follow since only children may not be maimed.
(B) does not mention landmine as a reason.
(C) Ambarish Rai may not necessarily be a kid.
123. The 1st statement talks about the fact relating to the pandemic i.e. - it is an infectious agent
with a difference. And the statement 2 presents the basis of this difference i.e. - the power of virus
(non-humans) over humans.
124. The statement is an inference based on the prior statement by the author - ―They are powerful
because they are participants in the network. They become influential in their association with the
larger network of things including humans and vice versa.‖ It means as long as they are in a broad
network, they are powerful and lesser when in small network.
125. The coronavirus is different because it demands that we revisit the idea of life and life sciences
to get the idea of it, and its microbiology and microchemistry at large.
126. In paragraph 4, the author mentions how in ANT it is not only humans who make a difference
but also viruses and airplanes. On that note, it is inferred that viruses and pandemics can spread
their feet through ANT.
127. None of the other options are either directly mentioned or can be inferred from the passage at
any point.
128. Viruses are non-human, and are social. Together they are actors, constituting a part of ANT.
They bring about change in the network of the social on a global level too and are hence globalisers
bigger than MNC‘s.
129. The correct option is (b) and it is mentioned in the passage. Since non-human also are a part of
ANT, organizations and institutions too would by default be a part of it
130. Option (a) and option (c) strengthen the passage and show the seriousness and dangers of the
possibility of doctors contracting coronavirus. Option (d) also supports the idea that coronavirus is
serious. Option (b) weakens the passage by stating that doctors have a low risk of contracting
coronavirus. Hence (b).
131. The author of the passage talks about medical workers as ‗we‘, implying that he/she is one too.
Hence (a).
132. The passage talks about the tough job medical workers are doing, as they attempt to handle
the coronavirus pandemic while feeling as confused, upset and anxious about it themselves as their
patients. All of the options are in line with what the author describes. Hence (d).
133. The author provides examples of how hospitals can ‗mobilize in a crunch‘ and make special
efforts during medical emergencies. Options (a), (b) and (c) are all also examples of such
emergencies. Hence (d).
134. Statement A is right as the passage mentions many illnesses and doctors being at the heart of
all of these. Statement B with ‗positive about being able to help everyone walking in the door with a
cough‘ contradicts the medical workers‘ concerns mentioned in the passage. Hence (a).

QUANTITATIVE TECHNIQUES

135. Total number of people affected by different disease = 160000


Typhoid = 160000*25% = 40000
Dengue = 160000*32% = 51200
Corona = 160000 * 12% = 19200
Cancer = 160000 * 15% = 24000
TB = 160000* 16% = 25600
Total number of children affected from different disease is
Typhoid = 40000 – (16000+14000) = 10000
Dengue = 51200 – (25000+15000) = 11200
Corona = 19200 – (10000+6000) = 3200
Cancer = 24000 – (8000+10000) = 6000
TB = 25600 – (7200+9600) = 8800
Total number of people affected by Cancer and TB = 24000 + 25600 = 49600
Total number of Children affected by Typhoid = 10000
% more = 49600-10000/10000 * 100 = 396%
136. Total number of people affected by Corona in 2020 is increased by 50% as compared to 2019,
=19200 * 100/150 = 12800
Required % = 12800/9600 * 100 = 133.33%
137. Find the ratio of number children affected by Typhoid and Corona taken together in 2020 to the
number children affected by TB in 2020
Required ratio = (10000+3200) : (8800) = 13200:8800 = 3:2
138. The average number of men affected by Corona and
Cancer taken together = (6000+10000)/2 = 8000
Average number of women affected by Cancer and TB taken together = (8000+7200) = 7600
Required difference = 8000 – 7600 = 400
139. Required average = (10000+11200+3200+6000)/4 = 7600

140.

141.

142.

143.
144.

145.

146.

Total run scored against Australia and Pakistan together = 1000+700= 1700
20% of total run in T20 means Rest 80% in ODIs = 320
Total run is score in T20 against South Africa = 320/80*100 = 400
Total run scored against Pakistan and South Africa = 700 + 400 = 1100
Difference = 1700-1100=600

147. Total runs scored against India and Australia together = 300+1000=1300
Total number of runs scored against Pakistan in ODIs = 525
Required fraction = 1300/525 = 2 10/21

148. Against USA, total run scored in ODI is 80% more than total run scored in T20s, which is 20 %
more than the total run scored in ODIs against India
USA (ODI) = 180/100 * USA (T20) = 120/100 * run scored in ODIs against India
USA (ODI) = 180/100 * USA (T20) = 135* 120/100 =162
USA (ODI) = 162
USA (T20) = 162*100/180 = 90
Total run scored = 162 + 90 = 252

149. 4x = ODI + T20


320 = 4/3 * T20 so, T20 = 320*3/4 = 240
Total runs = 4x = 320+240 = 560
hence x = 560/4 =140
And x2+2x-50 = 19830

150. Total run scored in T20s against (India, Australia and Pakistan together) = 165+350+175 = 690
Average run scored in ODIs against all the team = 135+650+520+320)/4 = 406.25
Difference = 690 – 406.25 = 283.75
Legal Edge 21 Mock

Answers & Details

ENGLISH

1. The Delhi HC order has granted bail to Disha, and nothing more has been mentioned. Thus, the
correct answer is (b)

2. The following line indicates the same:


“In my considered opinion, citizens are conscience keepers of government in any democratic
nation.”
Thus, the correct answer is (a)

3. The following line indicates the same:


“With police and prosecution harbouring no compunction presenting dissent as crime, the
safeguards against sedition are failing”. Thus, the correct answer is (c)

4. The line means that dissent is not a crime, but is being treated like one by the police, which shows
extremism on the police‟s i.e. the State‟s end. Thus, the correct answer is (c)

5. The following lines indicate the same:


“On Disha collaborating with Greta Thunberg and Canadian pro-Khalistani activist Mo Dhaliwal, the
judge noted that India‟s 5,000 year civilisation was never averse to ideas from varied quarters. “In
my considered opinion, the freedom of speech and expression includes the right to seek a global
audience. There are no geographical barriers on communication,” said Rana, quoting the Rig Veda
verse: “Let noble thoughts come to me from all directions.” To be liberal means to be willing to
respect or accept behaviour or opinions different from one''s own; open to new ideas. Thus, the
correct answer is B
6. Incarceration means imprisonment. Thus, the correct answer is (a)

7. Equitable means “fair and impartial”. Reservation is based on equity. Seeing the context, the
correct answer is (b)

8. The following line indicates the same:


“Turn to states for answers as India‟s reservation history has distinct regional patterns.” Thus, the
correct answer is (a)
9. The following lines reflect the same:
“People are trapped in narrow group identities, with fresh reservation demands representing benefits
for one group at the expense of another.” The author says that reservation is benefits given to one
group at the expense of another. Thus, the correct answer is (b)
10. Option C tells us the innumerous castes existing in India, thereby implying India‟s diversity. Thus,
the correct answer is (c)
11. The task given to the committee was to sub-categorize the central list into smaller parts. Thus, in
the context of the passage, it means dividing the central list into smaller parts. Thus, the correct
answer is (b)

12. The author in the first sentences says “If a democracy‟s health were to be judged by how many
people exercise the vote…”, which does not imply any of the options A, B, or C thus, the correct
answer is (d)

13. The author partially blames Trump and his administration for the dysfunction in the passage.
Further, the most logical option is (a) Thus, the correct answer is (a)

14. All the above-mentioned adjectives can be used to describe this year‟s election turnout. Thus,
the correct answer is (d)

15. The line says that Trump has been stoking misinformation – and potentially violence- indicating
and undermining of norms and institutions. Thus, the correct answer is (b)

16. The following lines indicate the same:


“They may well begin by addressing the gulf between the citizens of colour, a majority of whom
voted for them, and the white people who didn‟t.” the author says that there is still a gap between
people of colour and whites. Thus the correct answer is (b)
17. The following lines indicate the same:
“Biden has already won more overall votes than any other presidential candidate in US history. Of
course Al Gore and Hillary Clinton also won the popular vote, only to fade into electoral college
coma. This is the fate Democrats have been determined to avert this time.”
“They may well begin by addressing the gulf between the citizens of colour, a majority of whom
voted for them, and the white people who didn‟t.”
“The world also looks to the US to provide better leadership on issues ranging from climate change
to global security”
Thus, the correct answer is (d)
18. Phantom means “not real; illusory”. Further, in the next sentence, the author calls love jihad a
bogey. Thus, the correct answer is (b)

19. The following lines indicate the same:


“The horrific Ballabhgarh shooting that has stoked outrage in Haryana, and could be the driving force
behind chief minister ML Khattar‟s proposed law, must be recognised for what it is: Male entitlement,
stalking culture and VIP mentality that violently rejected a woman‟s agency to withhold consent.”
Thus the correct answer is (b)
20. Paternalism is the interference of a state or an individual with another person, against their will,
and defended or motivated by a claim that the person interfered with will be better off or protected
from harm. Thus, the correct answer is (b)

21. The following line indicates the same:


“Laws against love jihad under consideration by Uttar Pradesh and Haryana betray an obsession
with excessive legislation having no perceptible benefits and ample scope for harassment and rights
violations.” Thus, the correct answer is (a)
22. The passage implies all of the above. Thus, the correct answer is (d)

23. Option A reflects the essence of the passage the best. Thus, the correct answer is (a)

24. The following lines indicate the same:


“As Joe Biden wrote in an oped, it can make “all the difference from North Carolina and Virginia to
Pennsylvania and Michigan to Georgia and Texas”. In other words, it can give a critical nudge in the
swing states.”
25. While A and C are also reasons behind the voting, the primary reason is that majority of the
Indians tilt towards liberalism. Thus, the correct answer is (b)

26. All of the above are true, as can be seen from paragraph 2. Thus, the correct answer is (d)

27. The spotlight is unprecedented, i.e, never seen before. Thus, the correct answer is (b)

28. The following line indicates the same:


“Yet, it is noteworthy that it chose not to provide the same information when it had a legal obligation
to do so to a statutory body.”
Thus, the correct answer is (b)
29. To chastise means to severely rebuke. Thus, the correct answer is (b)

30. The passage goes on to talk about the importance of accountability, and then talks about how
there is not law regulating personal information as of today, further highlighting the importance of
RTI in this particular case. thus, the correct answer is (c)

31. The following lines reflect the same:


“Given this context, it doesn‟t help that CIC today is without a chief information commissioner and
only 50% of the sanctioned strength of central information commissioners have been filled. Over the
last six years CIC has had repeated gaps between the tenures of successive chiefs.”
32. Option A reflects the underlying theme of the passage, thus, is the correct answer.

GENERAL KNOWLEDGE AND CURRENT AFFAIRS

33. RT-PCR requires advanced infrastructure and up to 24 hours to deliver the results warrants a
better alternative. It employs the cutting-edge CRISPR-Cas9 technology which earned the
Emmanuelle Charpentier and Jennifer A. Doudna the Nobel Prize in Chemistry this year. Since the
groundbreaking work was published in 2012, the technique has successfully been applied in gene
editing and diagnostics.
CRISPR naturally occurs in bacteria and archaea as a defence system against invading viruses.
When bacteria survive a viral attack, they incorporate pieces of the virus‟s DNA into their genomes.
Those viral segments and the genetic pattern between them are the „Clustered Regularly
Interspaced Short Palindromic Repeats‟ (CRISPR). If the virus attacks again, the bacteria use those
CRISPR segments as a template to create strands of RNA that home in on the corresponding
sequence of the viral genome. The CRISPR RNA carries along a protein – Cas9 – to the target
location on the viral DNA and disables the virus by cutting its DNA.
CSIR Institute of Genomics and Integrative Biology is a scientific research institute devoted primarily
to biological research. It is a part of Council of Scientific and Industrial Research, India.
Source : The wire
34. RT-PCR requires advanced infrastructure and up to 24 hours to deliver the results warrants a
better alternative. It employs the cutting-edge CRISPR-Cas9 technology which earned the
Emmanuelle Charpentier and Jennifer A. Doudna the Nobel Prize in Chemistry this year. Since the
groundbreaking work was published in 2012, the technique has successfully been applied in gene
editing and diagnostics.
CRISPR naturally occurs in bacteria and archaea as a defence system against invading viruses.
When bacteria survive a viral attack, they incorporate pieces of the virus‟s DNA into their genomes.
Those viral segments and the genetic pattern between them are the „Clustered Regularly
Interspaced Short Palindromic Repeats‟ (CRISPR). If the virus attacks again, the bacteria use those
CRISPR segments as a template to create strands of RNA that home in on the corresponding
sequence of the viral genome. The CRISPR RNA carries along a protein – Cas9 – to the target
location on the viral DNA and disables the virus by cutting its DNA.
CSIR Institute of Genomics and Integrative Biology is a scientific research institute devoted primarily
to biological research. It is a part of Council of Scientific and Industrial Research, India.
Source : The wire
35. RT-PCR requires advanced infrastructure and up to 24 hours to deliver the results warrants a
better alternative. It employs the cutting-edge CRISPR-Cas9 technology which earned the
Emmanuelle Charpentier and Jennifer A. Doudna the Nobel Prize in Chemistry this year. Since the
groundbreaking work was published in 2012, the technique has successfully been applied in gene
editing and diagnostics.
CRISPR naturally occurs in bacteria and archaea as a defence system against invading viruses.
When bacteria survive a viral attack, they incorporate pieces of the virus‟s DNA into their genomes.
Those viral segments and the genetic pattern between them are the „Clustered Regularly
Interspaced Short Palindromic Repeats‟ (CRISPR). If the virus attacks again, the bacteria use those
CRISPR segments as a template to create strands of RNA that home in on the corresponding
sequence of the viral genome. The CRISPR RNA carries along a protein – Cas9 – to the target
location on the viral DNA and disables the virus by cutting its DNA.
CSIR Institute of Genomics and Integrative Biology is a scientific research institute devoted primarily
to biological research. It is a part of Council of Scientific and Industrial Research, India.
Source : The wire
36. RT-PCR requires advanced infrastructure and up to 24 hours to deliver the results warrants a
better alternative. It employs the cutting-edge CRISPR-Cas9 technology which earned the
Emmanuelle Charpentier and Jennifer A. Doudna the Nobel Prize in Chemistry this year. Since the
groundbreaking work was published in 2012, the technique has successfully been applied in gene
editing and diagnostics.
CRISPR naturally occurs in bacteria and archaea as a defence system against invading viruses.
When bacteria survive a viral attack, they incorporate pieces of the virus‟s DNA into their genomes.
Those viral segments and the genetic pattern between them are the „Clustered Regularly
Interspaced Short Palindromic Repeats‟ (CRISPR). If the virus attacks again, the bacteria use those
CRISPR segments as a template to create strands of RNA that home in on the corresponding
sequence of the viral genome. The CRISPR RNA carries along a protein – Cas9 – to the target
location on the viral DNA and disables the virus by cutting its DNA.
CSIR Institute of Genomics and Integrative Biology is a scientific research institute devoted primarily
to biological research. It is a part of Council of Scientific and Industrial Research, India.
Source : The wire
37. RT-PCR requires advanced infrastructure and up to 24 hours to deliver the results warrants a
better alternative. It employs the cutting-edge CRISPR-Cas9 technology which earned the
Emmanuelle Charpentier and Jennifer A. Doudna the Nobel Prize in Chemistry this year. Since the
groundbreaking work was published in 2012, the technique has successfully been applied in gene
editing and diagnostics.
CRISPR naturally occurs in bacteria and archaea as a defence system against invading viruses.
When bacteria survive a viral attack, they incorporate pieces of the virus‟s DNA into their genomes.
Those viral segments and the genetic pattern between them are the „Clustered Regularly
Interspaced Short Palindromic Repeats‟ (CRISPR). If the virus attacks again, the bacteria use those
CRISPR segments as a template to create strands of RNA that home in on the corresponding
sequence of the viral genome. The CRISPR RNA carries along a protein – Cas9 – to the target
location on the viral DNA and disables the virus by cutting its DNA.
CSIR Institute of Genomics and Integrative Biology is a scientific research institute devoted primarily
to biological research. It is a part of Council of Scientific and Industrial Research, India.
Source : The wire
38. The total area of the Nilgiri Biosphere Reserve is 5,520 sq. km. It is located in the Western Ghats
between 76°- 77°15''E and 11°15'' - 12°15''N. The Nilgiri Biosphere Reserve encompasses parts of
Tamilnadu, Kerala and Karnataka.
Keystone species, in ecology, a species that has a disproportionately large effect on the
communities in which it lives.
In 2011, the Madras HC upheld the validity of the Tamil Nadu government‟s notification (of 2010)
declaring an „Elephant Corridor‟ in the Sigur Plateau of Nilgiris District.
It said that the government is fully empowered under the ''Project Elephant'' of the Union government
as well as Article 51 A(g) of the Constitution to notify the elephant corridor in the state‟s Nilgiris
district.
„Right of Passage‟, an 800-page study released in August 2017, authored by experts and published
by the Wildlife Trust of India (WTI) identifies and records details pertaining to 101 elephant corridors
across India.
39. The total area of the Nilgiri Biosphere Reserve is 5,520 sq. km. It is located in the Western Ghats
between 76°- 77°15''E and 11°15'' - 12°15''N. The Nilgiri Biosphere Reserve encompasses parts of
Tamilnadu, Kerala and Karnataka.
Keystone species, in ecology, a species that has a disproportionately large effect on the
communities in which it lives.
In 2011, the Madras HC upheld the validity of the Tamil Nadu government‟s notification (of 2010)
declaring an „Elephant Corridor‟ in the Sigur Plateau of Nilgiris District.
It said that the government is fully empowered under the ''Project Elephant'' of the Union government
as well as Article 51 A(g) of the Constitution to notify the elephant corridor in the state‟s Nilgiris
district.
„Right of Passage‟, an 800-page study released in August 2017, authored by experts and published
by the Wildlife Trust of India (WTI) identifies and records details pertaining to 101 elephant corridors
across India.
40. The total area of the Nilgiri Biosphere Reserve is 5,520 sq. km. It is located in the Western Ghats
between 76°- 77°15''E and 11°15'' - 12°15''N. The Nilgiri Biosphere Reserve encompasses parts of
Tamilnadu, Kerala and Karnataka.
Keystone species, in ecology, a species that has a disproportionately large effect on the
communities in which it lives.
In 2011, the Madras HC upheld the validity of the Tamil Nadu government‟s notification (of 2010)
declaring an „Elephant Corridor‟ in the Sigur Plateau of Nilgiris District.
It said that the government is fully empowered under the ''Project Elephant'' of the Union government
as well as Article 51 A(g) of the Constitution to notify the elephant corridor in the state‟s Nilgiris
district.
„Right of Passage‟, an 800-page study released in August 2017, authored by experts and published
by the Wildlife Trust of India (WTI) identifies and records details pertaining to 101 elephant corridors
across India.
41. The total area of the Nilgiri Biosphere Reserve is 5,520 sq. km. It is located in the Western Ghats
between 76°- 77°15''E and 11°15'' - 12°15''N. The Nilgiri Biosphere Reserve encompasses parts of
Tamilnadu, Kerala and Karnataka.
Keystone species, in ecology, a species that has a disproportionately large effect on the
communities in which it lives.
In 2011, the Madras HC upheld the validity of the Tamil Nadu government‟s notification (of 2010)
declaring an „Elephant Corridor‟ in the Sigur Plateau of Nilgiris District.
It said that the government is fully empowered under the ''Project Elephant'' of the Union government
as well as Article 51 A(g) of the Constitution to notify the elephant corridor in the state‟s Nilgiris
district.
„Right of Passage‟, an 800-page study released in August 2017, authored by experts and published
by the Wildlife Trust of India (WTI) identifies and records details pertaining to 101 elephant corridors
across India.
42. The total area of the Nilgiri Biosphere Reserve is 5,520 sq. km. It is located in the Western Ghats
between 76°- 77°15''E and 11°15'' - 12°15''N. The Nilgiri Biosphere Reserve encompasses parts of
Tamilnadu, Kerala and Karnataka.
Keystone species, in ecology, a species that has a disproportionately large effect on the
communities in which it lives.
In 2011, the Madras HC upheld the validity of the Tamil Nadu government‟s notification (of 2010)
declaring an „Elephant Corridor‟ in the Sigur Plateau of Nilgiris District.
It said that the government is fully empowered under the ''Project Elephant'' of the Union government
as well as Article 51 A(g) of the Constitution to notify the elephant corridor in the state‟s Nilgiris
district.
„Right of Passage‟, an 800-page study released in August 2017, authored by experts and published
by the Wildlife Trust of India (WTI) identifies and records details pertaining to 101 elephant corridors
across India.
43. Based on the values of the four indicators, the GHI determines hunger on a 100-point scale
where 0 is the best possible score (no hunger) and 100 is the worst.
Each country‟s GHI score is classified by severity, from low to extremely alarming.
But this year, India‟s rank takes it below Pakistan and Bangladesh – ranked 88th and 75th,
respectively; in fact, India has fared better than only Afghanistan (99) in its neighbourhood.
The National Food Security Act, 2013 is in place to ensure food security for the most vulnerable
communities. Ration distribution through fair price shops, mid-day meal programmes at schools,
nutrition and maternity benefit programmes for children and pregnant mothers at anganwadis fall
within the Act.
The Constitution of India prohibits discrimination and recognizes all human rights. The right to life is
recognized as a directly justiciable fundamental right (art. 21), while the right to food is defined as a
directive principle of State policy
Source : New Indian Express
44. Based on the values of the four indicators, the GHI determines hunger on a 100-point scale
where 0 is the best possible score (no hunger) and 100 is the worst.
Each country‟s GHI score is classified by severity, from low to extremely alarming.
But this year, India‟s rank takes it below Pakistan and Bangladesh – ranked 88th and 75th,
respectively; in fact, India has fared better than only Afghanistan (99) in its neighbourhood.
The National Food Security Act, 2013 is in place to ensure food security for the most vulnerable
communities. Ration distribution through fair price shops, mid-day meal programmes at schools,
nutrition and maternity benefit programmes for children and pregnant mothers at anganwadis fall
within the Act.
The Constitution of India prohibits discrimination and recognizes all human rights. The right to life is
recognized as a directly justiciable fundamental right (art. 21), while the right to food is defined as a
directive principle of State policy
Source : New Indian Express
45. Based on the values of the four indicators, the GHI determines hunger on a 100-point scale
where 0 is the best possible score (no hunger) and 100 is the worst.
Each country‟s GHI score is classified by severity, from low to extremely alarming.
But this year, India‟s rank takes it below Pakistan and Bangladesh – ranked 88th and 75th,
respectively; in fact, India has fared better than only Afghanistan (99) in its neighbourhood.
The National Food Security Act, 2013 is in place to ensure food security for the most vulnerable
communities. Ration distribution through fair price shops, mid-day meal programmes at schools,
nutrition and maternity benefit programmes for children and pregnant mothers at anganwadis fall
within the Act.
The Constitution of India prohibits discrimination and recognizes all human rights. The right to life is
recognized as a directly justiciable fundamental right (art. 21), while the right to food is defined as a
directive principle of State policy
Source : New Indian Express
46. Based on the values of the four indicators, the GHI determines hunger on a 100-point scale
where 0 is the best possible score (no hunger) and 100 is the worst.
Each country‟s GHI score is classified by severity, from low to extremely alarming.
But this year, India‟s rank takes it below Pakistan and Bangladesh – ranked 88th and 75th,
respectively; in fact, India has fared better than only Afghanistan (99) in its neighbourhood.
The National Food Security Act, 2013 is in place to ensure food security for the most vulnerable
communities. Ration distribution through fair price shops, mid-day meal programmes at schools,
nutrition and maternity benefit programmes for children and pregnant mothers at anganwadis fall
within the Act.
The Constitution of India prohibits discrimination and recognizes all human rights. The right to life is
recognized as a directly justiciable fundamental right (art. 21), while the right to food is defined as a
directive principle of State policy
Source : New Indian Express
47. Based on the values of the four indicators, the GHI determines hunger on a 100-point scale
where 0 is the best possible score (no hunger) and 100 is the worst.
Each country‟s GHI score is classified by severity, from low to extremely alarming.
But this year, India‟s rank takes it below Pakistan and Bangladesh – ranked 88th and 75th,
respectively; in fact, India has fared better than only Afghanistan (99) in its neighbourhood.
The National Food Security Act, 2013 is in place to ensure food security for the most vulnerable
communities. Ration distribution through fair price shops, mid-day meal programmes at schools,
nutrition and maternity benefit programmes for children and pregnant mothers at anganwadis fall
within the Act.
The Constitution of India prohibits discrimination and recognizes all human rights. The right to life is
recognized as a directly justiciable fundamental right (art. 21), while the right to food is defined as a
directive principle of State policy
Source : New Indian Express
48. The water sharing dispute between India and Pakistan began in 1958.
Feni river, which forms part of the India-Bangladesh border, originates in the South Tripura district,
passes through Sabroom town.
Being developed as a corridor for trade and commerce between India''s North-East and Bangladesh,
the Feni bridge also known as Maitree Setu will connect Tripura of India with Chittagong port of
Bangladesh.
Source – Newsonair, https://economictimes.indiatimes.com/news/politics-and-nation/tripura-to-be-
gateway-of-indias-corridor-with-south-east-asia-nirmala-
sitharaman/articleshow/50468044.cms?from=mdr
49. The water sharing dispute between India and Pakistan began in 1958.
Feni river, which forms part of the India-Bangladesh border, originates in the South Tripura district,
passes through Sabroom town.
Being developed as a corridor for trade and commerce between India''s North-East and Bangladesh,
the Feni bridge also known as Maitree Setu will connect Tripura of India with Chittagong port of
Bangladesh.
Source – Newsonair, https://economictimes.indiatimes.com/news/politics-and-nation/tripura-to-be-
gateway-of-indias-corridor-with-south-east-asia-nirmala-
sitharaman/articleshow/50468044.cms?from=mdr
50. The water sharing dispute between India and Pakistan began in 1958.
Feni river, which forms part of the India-Bangladesh border, originates in the South Tripura district,
passes through Sabroom town.
Being developed as a corridor for trade and commerce between India''s North-East and Bangladesh,
the Feni bridge also known as Maitree Setu will connect Tripura of India with Chittagong port of
Bangladesh.
Source – Newsonair, https://economictimes.indiatimes.com/news/politics-and-nation/tripura-to-be-
gateway-of-indias-corridor-with-south-east-asia-nirmala-
sitharaman/articleshow/50468044.cms?from=mdr
51. The river Feni is not a tributary of any river. It flows into Bangladesh and then discharges into the
Bay of Bengal.
The water sharing dispute between India and Pakistan began in 1958.
Feni river, which forms part of the India-Bangladesh border, originates in the South Tripura district,
passes through Sabroom town.
Being developed as a corridor for trade and commerce between India''s North-East and Bangladesh,
the Feni bridge also known as Maitree Setu will connect Tripura of India with Chittagong port of
Bangladesh.
Source – Newsonair, https://economictimes.indiatimes.com/news/politics-and-nation/tripura-to-be-
gateway-of-indias-corridor-with-south-east-asia-nirmala-
sitharaman/articleshow/50468044.cms?from=mdr
52. The water sharing dispute between India and Pakistan began in 1958.
Feni river, which forms part of the India-Bangladesh border, originates in the South Tripura district,
passes through Sabroom town.
Being developed as a corridor for trade and commerce between India''s North-East and Bangladesh,
the Feni bridge also known as Maitree Setu will connect Tripura of India with Chittagong port of
Bangladesh.
Source – Newsonair, https://economictimes.indiatimes.com/news/politics-and-nation/tripura-to-be-
gateway-of-indias-corridor-with-south-east-asia-nirmala-
sitharaman/articleshow/50468044.cms?from=mdr
53. The Union Government has declared that India will decarbonize by 2050. India‟s already stated
target of producing 175 GW of renewable energy by the year 2022 has got a huge impetus along
with the National Hydrogen Mission when it was allocated Rs. 1500 crores in the Union Budget
2021-22. OIC has started the HCNG Buses pilot in Delhi in October 2020. Delhi became the first
state to launch such buses. State-run power generator NTPC has already signed an MoU with
Siemens for production of green hydrogen from the company‟s renewable energy plants and its use
in transportation. It has also planned pilot projects to run five hydrogen-cell electric buses and five
cars in Delhi and Leh. In its laboratories, NTPC is designing a prototype for hard/sea water
electrolysis and reactors for hydrogen production through the photo-electro-chemical process. In
developing countries like India, the power-plants are mostly coal-fired, so even though EVs (Electric
Vehicles) work on electricity, it needs to be charged and for that considerable electricity is required.
The whole objective of a cleaner transportation mode gets defeated when this electricity is obtained
from coal-fired power-plants. Hydrogen Fuel-Cell Electric Vehicles (HFCEVs) can be a good
alternative for this. Hydrogen is the lightest and first element on the periodic table. Since the weight
of hydrogen is less than air, it rises in the atmosphere and is therefore rarely found in its pure form,
H2. At standard temperature and pressure, hydrogen is a nontoxic, nonmetallic, odorless, tasteless,
colorless, and highly combustible diatomic gas. Hydrogen fuel is a zero-emission fuel burned with
oxygen. It can be used in fuel cells or internal combustion engines. It is also used as a fuel for
spacecraft propulsion. It is the most abundant element in the universe. The sun and other stars are
composed largely of hydrogen. Astronomers estimate that 90% of the atoms in the universe are
hydrogen atoms. Hydrogen is a component of more compounds than any other element. Water is
the most abundant compound of hydrogen found on earth.
Solutionlink: https://indianexpress.com/article/explained/electric-vehicle-technology-hydrogen-car-
national-hydrogen-mission-7193217/
https://indianexpress.com/article/explained/hydrogen-as-the-alternative-fuel-quixplained-7198064/
https://www.financialexpress.com/industry/national-hydrogen-energy-mission-govt-plans-green-
hydrogen-auction/2198308/
54. The Union Government has declared that India will decarbonize by 2050. India‟s already stated
target of producing 175 GW of renewable energy by the year 2022 has got a huge impetus along
with the National Hydrogen Mission when it was allocated Rs. 1500 crores in the Union Budget
2021-22. OIC has started the HCNG Buses pilot in Delhi in October 2020. Delhi became the first
state to launch such buses. State-run power generator NTPC has already signed an MoU with
Siemens for production of green hydrogen from the company‟s renewable energy plants and its use
in transportation. It has also planned pilot projects to run five hydrogen-cell electric buses and five
cars in Delhi and Leh. In its laboratories, NTPC is designing a prototype for hard/sea water
electrolysis and reactors for hydrogen production through the photo-electro-chemical process. In
developing countries like India, the power-plants are mostly coal-fired, so even though EVs (Electric
Vehicles) work on electricity, it needs to be charged and for that considerable electricity is required.
The whole objective of a cleaner transportation mode gets defeated when this electricity is obtained
from coal-fired power-plants. Hydrogen Fuel-Cell Electric Vehicles (HFCEVs) can be a good
alternative for this. Hydrogen is the lightest and first element on the periodic table. Since the weight
of hydrogen is less than air, it rises in the atmosphere and is therefore rarely found in its pure form,
H2. At standard temperature and pressure, hydrogen is a nontoxic, nonmetallic, odorless, tasteless,
colorless, and highly combustible diatomic gas. Hydrogen fuel is a zero-emission fuel burned with
oxygen. It can be used in fuel cells or internal combustion engines. It is also used as a fuel for
spacecraft propulsion. It is the most abundant element in the universe. The sun and other stars are
composed largely of hydrogen. Astronomers estimate that 90% of the atoms in the universe are
hydrogen atoms. Hydrogen is a component of more compounds than any other element. Water is
the most abundant compound of hydrogen found on earth.
Regarding Question 5, hydrogen is never available in its pure-form, so there is nothing called
hydrogen-reservoirs anywhere in the world. So, option (b) is redundant. Option (a) is a genuine and
there is a safety concern as hydrogen will be stored at high pressure in car-bellies. Thus, the answer
is option (a).
Solutionlink: https://indianexpress.com/article/explained/electric-vehicle-technology-hydrogen-car-
national-hydrogen-mission-7193217/
https://indianexpress.com/article/explained/hydrogen-as-the-alternative-fuel-quixplained-7198064/
https://www.financialexpress.com/industry/national-hydrogen-energy-mission-govt-plans-green-
hydrogen-auction/2198308/
55. The Union Government has declared that India will decarbonize by 2050. India‟s already stated
target of producing 175 GW of renewable energy by the year 2022 has got a huge impetus along
with the National Hydrogen Mission when it was allocated Rs. 1500 crores in the Union Budget
2021-22. OIC has started the HCNG Buses pilot in Delhi in October 2020. Delhi became the first
state to launch such buses. State-run power generator NTPC has already signed an MoU with
Siemens for production of green hydrogen from the company‟s renewable energy plants and its use
in transportation. It has also planned pilot projects to run five hydrogen-cell electric buses and five
cars in Delhi and Leh. In its laboratories, NTPC is designing a prototype for hard/sea water
electrolysis and reactors for hydrogen production through the photo-electro-chemical process. In
developing countries like India, the power-plants are mostly coal-fired, so even though EVs (Electric
Vehicles) work on electricity, it needs to be charged and for that considerable electricity is required.
The whole objective of a cleaner transportation mode gets defeated when this electricity is obtained
from coal-fired power-plants. Hydrogen Fuel-Cell Electric Vehicles (HFCEVs) can be a good
alternative for this. Hydrogen is the lightest and first element on the periodic table. Since the weight
of hydrogen is less than air, it rises in the atmosphere and is therefore rarely found in its pure form,
H2. At standard temperature and pressure, hydrogen is a nontoxic, nonmetallic, odorless, tasteless,
colorless, and highly combustible diatomic gas. Hydrogen fuel is a zero-emission fuel burned with
oxygen. It can be used in fuel cells or internal combustion engines. It is also used as a fuel for
spacecraft propulsion. It is the most abundant element in the universe. The sun and other stars are
composed largely of hydrogen. Astronomers estimate that 90% of the atoms in the universe are
hydrogen atoms. Hydrogen is a component of more compounds than any other element. Water is
the most abundant compound of hydrogen found on earth.
Regarding Question 5, hydrogen is never available in its pure-form, so there is nothing called
hydrogen-reservoirs anywhere in the world. So, option (b) is redundant. Option (a) is a genuine and
there is a safety concern as hydrogen will be stored at high pressure in car-bellies. Thus, the answer
is option (a).
Solutionlink: https://indianexpress.com/article/explained/electric-vehicle-technology-hydrogen-car-
national-hydrogen-mission-7193217/
https://indianexpress.com/article/explained/hydrogen-as-the-alternative-fuel-quixplained-7198064/
https://www.financialexpress.com/industry/national-hydrogen-energy-mission-govt-plans-green-
hydrogen-auction/2198308/
56. The Union Government has declared that India will decarbonize by 2050. India‟s already stated
target of producing 175 GW of renewable energy by the year 2022 has got a huge impetus along
with the National Hydrogen Mission when it was allocated Rs. 1500 crores in the Union Budget
2021-22. OIC has started the HCNG Buses pilot in Delhi in October 2020. Delhi became the first
state to launch such buses. State-run power generator NTPC has already signed an MoU with
Siemens for production of green hydrogen from the company‟s renewable energy plants and its use
in transportation. It has also planned pilot projects to run five hydrogen-cell electric buses and five
cars in Delhi and Leh. In its laboratories, NTPC is designing a prototype for hard/sea water
electrolysis and reactors for hydrogen production through the photo-electro-chemical process. In
developing countries like India, the power-plants are mostly coal-fired, so even though EVs (Electric
Vehicles) work on electricity, it needs to be charged and for that considerable electricity is required.
The whole objective of a cleaner transportation mode gets defeated when this electricity is obtained
from coal-fired power-plants. Hydrogen Fuel-Cell Electric Vehicles (HFCEVs) can be a good
alternative for this. Hydrogen is the lightest and first element on the periodic table. Since the weight
of hydrogen is less than air, it rises in the atmosphere and is therefore rarely found in its pure form,
H2. At standard temperature and pressure, hydrogen is a nontoxic, nonmetallic, odorless, tasteless,
colorless, and highly combustible diatomic gas. Hydrogen fuel is a zero-emission fuel burned with
oxygen. It can be used in fuel cells or internal combustion engines. It is also used as a fuel for
spacecraft propulsion. It is the most abundant element in the universe. The sun and other stars are
composed largely of hydrogen. Astronomers estimate that 90% of the atoms in the universe are
hydrogen atoms. Hydrogen is a component of more compounds than any other element. Water is
the most abundant compound of hydrogen found on earth.
Regarding Question 5, hydrogen is never available in its pure-form, so there is nothing called
hydrogen-reservoirs anywhere in the world. So, option (b) is redundant. Option (a) is a genuine and
there is a safety concern as hydrogen will be stored at high pressure in car-bellies. Thus, the answer
is option (a).
Solutionlink: https://indianexpress.com/article/explained/electric-vehicle-technology-hydrogen-car-
national-hydrogen-mission-7193217/
https://indianexpress.com/article/explained/hydrogen-as-the-alternative-fuel-quixplained-7198064/
https://www.financialexpress.com/industry/national-hydrogen-energy-mission-govt-plans-green-
hydrogen-auction/2198308/
57. The Union Government has declared that India will decarbonize by 2050. India‟s already stated
target of producing 175 GW of renewable energy by the year 2022 has got a huge impetus along
with the National Hydrogen Mission when it was allocated Rs. 1500 crores in the Union Budget
2021-22. OIC has started the HCNG Buses pilot in Delhi in October 2020. Delhi became the first
state to launch such buses. State-run power generator NTPC has already signed an MoU with
Siemens for production of green hydrogen from the company‟s renewable energy plants and its use
in transportation. It has also planned pilot projects to run five hydrogen-cell electric buses and five
cars in Delhi and Leh. In its laboratories, NTPC is designing a prototype for hard/sea water
electrolysis and reactors for hydrogen production through the photo-electro-chemical process. In
developing countries like India, the power-plants are mostly coal-fired, so even though EVs (Electric
Vehicles) work on electricity, it needs to be charged and for that considerable electricity is required.
The whole objective of a cleaner transportation mode gets defeated when this electricity is obtained
from coal-fired power-plants. Hydrogen Fuel-Cell Electric Vehicles (HFCEVs) can be a good
alternative for this. Hydrogen is the lightest and first element on the periodic table. Since the weight
of hydrogen is less than air, it rises in the atmosphere and is therefore rarely found in its pure form,
H2. At standard temperature and pressure, hydrogen is a nontoxic, nonmetallic, odorless, tasteless,
colorless, and highly combustible diatomic gas. Hydrogen fuel is a zero-emission fuel burned with
oxygen. It can be used in fuel cells or internal combustion engines. It is also used as a fuel for
spacecraft propulsion. It is the most abundant element in the universe. The sun and other stars are
composed largely of hydrogen. Astronomers estimate that 90% of the atoms in the universe are
hydrogen atoms. Hydrogen is a component of more compounds than any other element. Water is
the most abundant compound of hydrogen found on earth.
Regarding Question 5, hydrogen is never available in its pure-form, so there is nothing called
hydrogen-reservoirs anywhere in the world. So, option (b) is redundant. Option (a) is a genuine and
there is a safety concern as hydrogen will be stored at high pressure in car-bellies. Thus, the answer
is option (a).

Solutionlink: https://indianexpress.com/article/explained/electric-vehicle-technology-hydrogen-car-
national-hydrogen-mission-7193217/
https://indianexpress.com/article/explained/hydrogen-as-the-alternative-fuel-quixplained-7198064/
https://www.financialexpress.com/industry/national-hydrogen-energy-mission-govt-plans-green-
hydrogen-auction/2198308/
58. The New START Treaty which was about to expire on February 5, 2021 was renewed by both
US and Russia and extended for the next 5 years. Antony John Blinken is an American government
official and diplomat serving as the 71st United States secretary of state since January 26, 2021. He
previously served as deputy national security advisor from 2013 to 2015 and deputy secretary of
state from 2015 to 2017 under President Barack Obama. The New START Treaty is a treaty
between the United States of America and the Russian Federation on measures for the further
reduction and limitation of strategic offensive arms. It entered into force on 5th February, 2011 and
was signed in 2010. It is a successor to the START framework of 1991 (at the end of the Cold War)
that limited both sides to 1,600 strategic delivery vehicles and 6,000 warheads. The START
framework of 1991 (at the end of the Cold War) limited both sides to 1,600 strategic delivery vehicles
and 6,000 warheads. The New START Treaty continues the bipartisan process of verifiably reducing
U.S. and Russian strategic nuclear arsenals by limiting both sides to 700 strategic launchers and
1,550 operational warheads. The Intermediate-Range Nuclear Forces Treaty (INF Treaty, formally
Treaty Between the United States of America and the Union of Soviet Socialist Republics on the
Elimination of Their Intermediate-Range and Shorter-Range Missiles) is a 1987 arms control
agreement between the United States and the Soviet Union.
Solution link : https://www.thehindu.com/news/international/us-extends-new-start-nuclear-arms-
treaty-with-russia-secretary-of-state-antony-blinken/article33741681.ece
59. The New START Treaty which was about to expire on February 5, 2021 was renewed by both
US and Russia and extended for the next 5 years. Antony John Blinken is an American government
official and diplomat serving as the 71st United States secretary of state since January 26, 2021. He
previously served as deputy national security advisor from 2013 to 2015 and deputy secretary of
state from 2015 to 2017 under President Barack Obama. The New START Treaty is a treaty
between the United States of America and the Russian Federation on measures for the further
reduction and limitation of strategic offensive arms. It entered into force on 5th February, 2011 and
was signed in 2010. It is a successor to the START framework of 1991 (at the end of the Cold War)
that limited both sides to 1,600 strategic delivery vehicles and 6,000 warheads. The START
framework of 1991 (at the end of the Cold War) limited both sides to 1,600 strategic delivery vehicles
and 6,000 warheads. The New START Treaty continues the bipartisan process of verifiably reducing
U.S. and Russian strategic nuclear arsenals by limiting both sides to 700 strategic launchers and
1,550 operational warheads. The Intermediate-Range Nuclear Forces Treaty (INF Treaty, formally
Treaty Between the United States of America and the Union of Soviet Socialist Republics on the
Elimination of Their Intermediate-Range and Shorter-Range Missiles) is a 1987 arms control
agreement between the United States and the Soviet Union.

Solution link : https://www.thehindu.com/news/international/us-extends-new-start-nuclear-arms-


treaty-with-russia-secretary-of-state-antony-blinken/article33741681.ece
60. The New START Treaty which was about to expire on February 5, 2021 was renewed by both
US and Russia and extended for the next 5 years. Antony John Blinken is an American government
official and diplomat serving as the 71st United States secretary of state since January 26, 2021. He
previously served as deputy national security advisor from 2013 to 2015 and deputy secretary of
state from 2015 to 2017 under President Barack Obama. The New START Treaty is a treaty
between the United States of America and the Russian Federation on measures for the further
reduction and limitation of strategic offensive arms. It entered into force on 5th February, 2011 and
was signed in 2010. It is a successor to the START framework of 1991 (at the end of the Cold War)
that limited both sides to 1,600 strategic delivery vehicles and 6,000 warheads. The START
framework of 1991 (at the end of the Cold War) limited both sides to 1,600 strategic delivery vehicles
and 6,000 warheads. The New START Treaty continues the bipartisan process of verifiably reducing
U.S. and Russian strategic nuclear arsenals by limiting both sides to 700 strategic launchers and
1,550 operational warheads. The Intermediate-Range Nuclear Forces Treaty (INF Treaty, formally
Treaty Between the United States of America and the Union of Soviet Socialist Republics on the
Elimination of Their Intermediate-Range and Shorter-Range Missiles) is a 1987 arms control
agreement between the United States and the Soviet Union.

Solution link : https://www.thehindu.com/news/international/us-extends-new-start-nuclear-arms-


treaty-with-russia-secretary-of-state-antony-blinken/article33741681.ece
61. The New START Treaty which was about to expire on February 5, 2021 was renewed by both
US and Russia and extended for the next 5 years. Antony John Blinken is an American government
official and diplomat serving as the 71st United States secretary of state since January 26, 2021. He
previously served as deputy national security advisor from 2013 to 2015 and deputy secretary of
state from 2015 to 2017 under President Barack Obama. The New START Treaty is a treaty
between the United States of America and the Russian Federation on measures for the further
reduction and limitation of strategic offensive arms. It entered into force on 5th February, 2011 and
was signed in 2010. It is a successor to the START framework of 1991 (at the end of the Cold War)
that limited both sides to 1,600 strategic delivery vehicles and 6,000 warheads. The START
framework of 1991 (at the end of the Cold War) limited both sides to 1,600 strategic delivery vehicles
and 6,000 warheads. The New START Treaty continues the bipartisan process of verifiably reducing
U.S. and Russian strategic nuclear arsenals by limiting both sides to 700 strategic launchers and
1,550 operational warheads. The Intermediate-Range Nuclear Forces Treaty (INF Treaty, formally
Treaty Between the United States of America and the Union of Soviet Socialist Republics on the
Elimination of Their Intermediate-Range and Shorter-Range Missiles) is a 1987 arms control
agreement between the United States and the Soviet Union.

Solution link : https://www.thehindu.com/news/international/us-extends-new-start-nuclear-arms-


treaty-with-russia-secretary-of-state-antony-blinken/article33741681.ece
62. The New START Treaty which was about to expire on February 5, 2021 was renewed by both
US and Russia and extended for the next 5 years. Antony John Blinken is an American government
official and diplomat serving as the 71st United States secretary of state since January 26, 2021. He
previously served as deputy national security advisor from 2013 to 2015 and deputy secretary of
state from 2015 to 2017 under President Barack Obama. The New START Treaty is a treaty
between the United States of America and the Russian Federation on measures for the further
reduction and limitation of strategic offensive arms. It entered into force on 5th February, 2011 and
was signed in 2010. It is a successor to the START framework of 1991 (at the end of the Cold War)
that limited both sides to 1,600 strategic delivery vehicles and 6,000 warheads. The START
framework of 1991 (at the end of the Cold War) limited both sides to 1,600 strategic delivery vehicles
and 6,000 warheads. The New START Treaty continues the bipartisan process of verifiably reducing
U.S. and Russian strategic nuclear arsenals by limiting both sides to 700 strategic launchers and
1,550 operational warheads. The Intermediate-Range Nuclear Forces Treaty (INF Treaty, formally
Treaty Between the United States of America and the Union of Soviet Socialist Republics on the
Elimination of Their Intermediate-Range and Shorter-Range Missiles) is a 1987 arms control
agreement between the United States and the Soviet Union.

Solution link : https://www.thehindu.com/news/international/us-extends-new-start-nuclear-arms-


treaty-with-russia-secretary-of-state-antony-blinken/article33741681.ece
63. Jairam Ramesh was the head of the Science and Technology panel that was formed for the
evaluation of the DNA Technology (Use and Application) Regulation Bill, 2019.
Regarding the second question, we can‟t say that there is a huge belief of caste-based bias among
the police department. That will be a huge generalization amounting to a gross mis-statement. Thus,
option (a) can‟t be the answer. So, option (d) too can‟t be the answer. Option (c) can‟t be applicable
as this DNA profiling can happen only after the Bill is passed, which will form a DNA regulatory
board, hence, there will be sufficient control over DNA data misuse. Option (b) can be the answer as
in India, though the population of the SCs, STs, OBCs and Muslims are in a minority, they
unsymmetrically represent a huge chunk in India‟s jailed population. Thus, this is a seed for a caste-
based profiling potential. Thus, the answer is option (b). Regarding the third question Section 34
under Chapter VI lays bare that the records maintained may be utilised for many things- from
facilitating adjudication and prosecution in criminal matters to identification for civil matters. This is to
say that if someone has rendered their DNA for a civil matter, their DNA could be used at a later
stage for cross-referencing in a criminal matter.Thus, consent given for the collection of DNA for civil
matters cannot be understood to be blanket consent for identification in a criminal case.Having a
unified database blurs the line between guilt and innocence. The presumption of innocence is
undermined, somehow making people who have given their DNA for civil matters less innocent than
those whose DNA is not on the database. Thus, option (a) is a pertinent issue. This Bill is not only at
odds with the right to privacy but also with the Personal Data Protection Bill. “Sensitive personal
data” under the Data Protection Bill includes genetic data that has secure parameters for collection,
processing and requires informed and specific consent. However, if an individual which includes
guardians of minors and relatives of victims under does not consent to give his DNA for an
investigation, a magistrate can easily override this consent and can order the collection under S.23
of the DNA Bill. Thus, taking away an individual‟s right to make intimate decisions about themselves
guaranteed under Article 21 of the Constitution. Thus, article (b) is a pertinent concern. Another
example of security failure is S.35(b) of the Bill. As per this section, highly confidential DNA data will
be made available to the personnel of DNA laboratories for training purposes. DNA data is highly
contaminable and it runs the risk of a large-scale breach of security. The Bill does not make a
provision for consent to be taken from individuals for their DNA to be used to educate trainees. Thus,
article (c) is a pertinent concern. There are no concerns raised over the appointment of National
DNA Data Bank. Thus, option (d) is the answer. The Supreme Court had held in Selvi vs. State of
Karnataka that personal autonomy is of utmost importance and any interference with it violates the
right to privacy especially in circumstances where a person can face criminal charges or penalties.
Thus, a possible cross-referencing of DNA collected for civil and criminal matters would lead to self-
incrimination. In the same case, Justice Ranjana Desai had observed that even though retention and
collection of DNA samples are not unconstitutional, future use and development of DNA profiling for
testimonial purposes could face constitutional obstacles under Article 20(3) of the Constitution. The
overriding power of the magistrate to procure the DNA sample of a person without her/his consent
violates an individual‟s right to make intimate decisions about themselves guaranteed under Article
21 of the Constitution. The Bill allows the use of the technology to establish the identity of persons in
matters of crime, parentage dispute, emigration or immigration and transplantation of human organs.
It provides for establishment of national and regional DNA data banks and each databank will
maintain crime scene index, suspects‟ or undertrials‟ index and offenders‟ index separately.
Link to solution: https://www.drishtiias.com/daily-updates/daily-news-analysis/concerns-over-dna-
technology-regulation-bill
https://www.hindustantimes.com/india-news/address-fears-over-dna-technology-bill-in-parliament-
and-outside-panel-to-govt-101612340870918.html
https://www.thehindu.com/news/national/dna-bill-can-lead-to-castecommunity-based-profiling-
owaisi-viswam/article33677524.ece
https://www.bloombergquint.com/opinion/the-dna-bill-another-invasive-imperfect-database
https://www.theleaflet.in/prodding-privacy-why-indias-dna-regulation-will-be-curtains-for-autonomy/#
64. Jairam Ramesh was the head of the Science and Technology panel that was formed for the
evaluation of the DNA Technology (Use and Application) Regulation Bill, 2019.
Regarding the second question, we can‟t say that there is a huge belief of caste-based bias among
the police department. That will be a huge generalization amounting to a gross mis-statement. Thus,
option (a) can‟t be the answer. So, option (d) too can‟t be the answer. Option (c) can‟t be applicable
as this DNA profiling can happen only after the Bill is passed, which will form a DNA regulatory
board, hence, there will be sufficient control over DNA data misuse. Option (b) can be the answer as
in India, though the population of the SCs, STs, OBCs and Muslims are in a minority, they
unsymmetrically represent a huge chunk in India‟s jailed population. Thus, this is a seed for a caste-
based profiling potential. Thus, the answer is option (b). Regarding the third question Section 34
under Chapter VI lays bare that the records maintained may be utilised for many things- from
facilitating adjudication and prosecution in criminal matters to identification for civil matters. This is to
say that if someone has rendered their DNA for a civil matter, their DNA could be used at a later
stage for cross-referencing in a criminal matter.Thus, consent given for the collection of DNA for civil
matters cannot be understood to be blanket consent for identification in a criminal case.Having a
unified database blurs the line between guilt and innocence. The presumption of innocence is
undermined, somehow making people who have given their DNA for civil matters less innocent than
those whose DNA is not on the database. Thus, option (a) is a pertinent issue. This Bill is not only at
odds with the right to privacy but also with the Personal Data Protection Bill. “Sensitive personal
data” under the Data Protection Bill includes genetic data that has secure parameters for collection,
processing and requires informed and specific consent. However, if an individual which includes
guardians of minors and relatives of victims under does not consent to give his DNA for an
investigation, a magistrate can easily override this consent and can order the collection under S.23
of the DNA Bill. Thus, taking away an individual‟s right to make intimate decisions about themselves
guaranteed under Article 21 of the Constitution. Thus, article (b) is a pertinent concern. Another
example of security failure is S.35(b) of the Bill. As per this section, highly confidential DNA data will
be made available to the personnel of DNA laboratories for training purposes. DNA data is highly
contaminable and it runs the risk of a large-scale breach of security. The Bill does not make a
provision for consent to be taken from individuals for their DNA to be used to educate trainees. Thus,
article (c) is a pertinent concern. There are no concerns raised over the appointment of National
DNA Data Bank. Thus, option (d) is the answer. The Supreme Court had held in Selvi vs. State of
Karnataka that personal autonomy is of utmost importance and any interference with it violates the
right to privacy especially in circumstances where a person can face criminal charges or penalties.
Thus, a possible cross-referencing of DNA collected for civil and criminal matters would lead to self-
incrimination. In the same case, Justice Ranjana Desai had observed that even though retention and
collection of DNA samples are not unconstitutional, future use and development of DNA profiling for
testimonial purposes could face constitutional obstacles under Article 20(3) of the Constitution. The
overriding power of the magistrate to procure the DNA sample of a person without her/his consent
violates an individual‟s right to make intimate decisions about themselves guaranteed under Article
21 of the Constitution. The Bill allows the use of the technology to establish the identity of persons in
matters of crime, parentage dispute, emigration or immigration and transplantation of human organs.
It provides for establishment of national and regional DNA data banks and each databank will
maintain crime scene index, suspects‟ or undertrials‟ index and offenders‟ index separately.
Link to solution: https://www.drishtiias.com/daily-updates/daily-news-analysis/concerns-over-dna-
technology-regulation-bill
https://www.hindustantimes.com/india-news/address-fears-over-dna-technology-bill-in-parliament-
and-outside-panel-to-govt-101612340870918.html
https://www.thehindu.com/news/national/dna-bill-can-lead-to-castecommunity-based-profiling-
owaisi-viswam/article33677524.ece
https://www.bloombergquint.com/opinion/the-dna-bill-another-invasive-imperfect-database
https://www.theleaflet.in/prodding-privacy-why-indias-dna-regulation-will-be-curtains-for-autonomy/#
65. Jairam Ramesh was the head of the Science and Technology panel that was formed for the
evaluation of the DNA Technology (Use and Application) Regulation Bill, 2019.
Regarding the second question, we can‟t say that there is a huge belief of caste-based bias among
the police department. That will be a huge generalization amounting to a gross mis-statement. Thus,
option (a) can‟t be the answer. So, option (d) too can‟t be the answer. Option (c) can‟t be applicable
as this DNA profiling can happen only after the Bill is passed, which will form a DNA regulatory
board, hence, there will be sufficient control over DNA data misuse. Option (b) can be the answer as
in India, though the population of the SCs, STs, OBCs and Muslims are in a minority, they
unsymmetrically represent a huge chunk in India‟s jailed population. Thus, this is a seed for a caste-
based profiling potential. Thus, the answer is option (b). Regarding the third question Section 34
under Chapter VI lays bare that the records maintained may be utilised for many things- from
facilitating adjudication and prosecution in criminal matters to identification for civil matters. This is to
say that if someone has rendered their DNA for a civil matter, their DNA could be used at a later
stage for cross-referencing in a criminal matter.Thus, consent given for the collection of DNA for civil
matters cannot be understood to be blanket consent for identification in a criminal case.Having a
unified database blurs the line between guilt and innocence. The presumption of innocence is
undermined, somehow making people who have given their DNA for civil matters less innocent than
those whose DNA is not on the database. Thus, option (a) is a pertinent issue. This Bill is not only at
odds with the right to privacy but also with the Personal Data Protection Bill. “Sensitive personal
data” under the Data Protection Bill includes genetic data that has secure parameters for collection,
processing and requires informed and specific consent. However, if an individual which includes
guardians of minors and relatives of victims under does not consent to give his DNA for an
investigation, a magistrate can easily override this consent and can order the collection under S.23
of the DNA Bill. Thus, taking away an individual‟s right to make intimate decisions about themselves
guaranteed under Article 21 of the Constitution. Thus, article (b) is a pertinent concern. Another
example of security failure is S.35(b) of the Bill. As per this section, highly confidential DNA data will
be made available to the personnel of DNA laboratories for training purposes. DNA data is highly
contaminable and it runs the risk of a large-scale breach of security. The Bill does not make a
provision for consent to be taken from individuals for their DNA to be used to educate trainees. Thus,
article (c) is a pertinent concern. There are no concerns raised over the appointment of National
DNA Data Bank. Thus, option (d) is the answer. The Supreme Court had held in Selvi vs. State of
Karnataka that personal autonomy is of utmost importance and any interference with it violates the
right to privacy especially in circumstances where a person can face criminal charges or penalties.
Thus, a possible cross-referencing of DNA collected for civil and criminal matters would lead to self-
incrimination. In the same case, Justice Ranjana Desai had observed that even though retention and
collection of DNA samples are not unconstitutional, future use and development of DNA profiling for
testimonial purposes could face constitutional obstacles under Article 20(3) of the Constitution. The
overriding power of the magistrate to procure the DNA sample of a person without her/his consent
violates an individual‟s right to make intimate decisions about themselves guaranteed under Article
21 of the Constitution. The Bill allows the use of the technology to establish the identity of persons in
matters of crime, parentage dispute, emigration or immigration and transplantation of human organs.
It provides for establishment of national and regional DNA data banks and each databank will
maintain crime scene index, suspects‟ or undertrials‟ index and offenders‟ index separately.
Link to solution: https://www.drishtiias.com/daily-updates/daily-news-analysis/concerns-over-dna-
technology-regulation-bill
https://www.hindustantimes.com/india-news/address-fears-over-dna-technology-bill-in-parliament-
and-outside-panel-to-govt-101612340870918.html
https://www.thehindu.com/news/national/dna-bill-can-lead-to-castecommunity-based-profiling-
owaisi-viswam/article33677524.ece
https://www.bloombergquint.com/opinion/the-dna-bill-another-invasive-imperfect-database
https://www.theleaflet.in/prodding-privacy-why-indias-dna-regulation-will-be-curtains-for-autonomy/#
66. Jairam Ramesh was the head of the Science and Technology panel that was formed for the
evaluation of the DNA Technology (Use and Application) Regulation Bill, 2019.
Regarding the second question, we can‟t say that there is a huge belief of caste-based bias among
the police department. That will be a huge generalization amounting to a gross mis-statement. Thus,
option (a) can‟t be the answer. So, option (d) too can‟t be the answer. Option (c) can‟t be applicable
as this DNA profiling can happen only after the Bill is passed, which will form a DNA regulatory
board, hence, there will be sufficient control over DNA data misuse. Option (b) can be the answer as
in India, though the population of the SCs, STs, OBCs and Muslims are in a minority, they
unsymmetrically represent a huge chunk in India‟s jailed population. Thus, this is a seed for a caste-
based profiling potential. Thus, the answer is option (b). Regarding the third question Section 34
under Chapter VI lays bare that the records maintained may be utilised for many things- from
facilitating adjudication and prosecution in criminal matters to identification for civil matters. This is to
say that if someone has rendered their DNA for a civil matter, their DNA could be used at a later
stage for cross-referencing in a criminal matter.Thus, consent given for the collection of DNA for civil
matters cannot be understood to be blanket consent for identification in a criminal case.Having a
unified database blurs the line between guilt and innocence. The presumption of innocence is
undermined, somehow making people who have given their DNA for civil matters less innocent than
those whose DNA is not on the database. Thus, option (a) is a pertinent issue. This Bill is not only at
odds with the right to privacy but also with the Personal Data Protection Bill. “Sensitive personal
data” under the Data Protection Bill includes genetic data that has secure parameters for collection,
processing and requires informed and specific consent. However, if an individual which includes
guardians of minors and relatives of victims under does not consent to give his DNA for an
investigation, a magistrate can easily override this consent and can order the collection under S.23
of the DNA Bill. Thus, taking away an individual‟s right to make intimate decisions about themselves
guaranteed under Article 21 of the Constitution. Thus, article (b) is a pertinent concern. Another
example of security failure is S.35(b) of the Bill. As per this section, highly confidential DNA data will
be made available to the personnel of DNA laboratories for training purposes. DNA data is highly
contaminable and it runs the risk of a large-scale breach of security. The Bill does not make a
provision for consent to be taken from individuals for their DNA to be used to educate trainees. Thus,
article (c) is a pertinent concern. There are no concerns raised over the appointment of National
DNA Data Bank. Thus, option (d) is the answer. The Supreme Court had held in Selvi vs. State of
Karnataka that personal autonomy is of utmost importance and any interference with it violates the
right to privacy especially in circumstances where a person can face criminal charges or penalties.
Thus, a possible cross-referencing of DNA collected for civil and criminal matters would lead to self-
incrimination. In the same case, Justice Ranjana Desai had observed that even though retention and
collection of DNA samples are not unconstitutional, future use and development of DNA profiling for
testimonial purposes could face constitutional obstacles under Article 20(3) of the Constitution. The
overriding power of the magistrate to procure the DNA sample of a person without her/his consent
violates an individual‟s right to make intimate decisions about themselves guaranteed under Article
21 of the Constitution. The Bill allows the use of the technology to establish the identity of persons in
matters of crime, parentage dispute, emigration or immigration and transplantation of human organs.
It provides for establishment of national and regional DNA data banks and each databank will
maintain crime scene index, suspects‟ or undertrials‟ index and offenders‟ index separately.
Link to solution: https://www.drishtiias.com/daily-updates/daily-news-analysis/concerns-over-dna-
technology-regulation-bill
https://www.hindustantimes.com/india-news/address-fears-over-dna-technology-bill-in-parliament-
and-outside-panel-to-govt-101612340870918.html
https://www.thehindu.com/news/national/dna-bill-can-lead-to-castecommunity-based-profiling-
owaisi-viswam/article33677524.ece
https://www.bloombergquint.com/opinion/the-dna-bill-another-invasive-imperfect-database
https://www.theleaflet.in/prodding-privacy-why-indias-dna-regulation-will-be-curtains-for-autonomy/#
67. Jairam Ramesh was the head of the Science and Technology panel that was formed for the
evaluation of the DNA Technology (Use and Application) Regulation Bill, 2019.
Regarding the second question, we can‟t say that there is a huge belief of caste-based bias among
the police department. That will be a huge generalization amounting to a gross mis-statement. Thus,
option (a) can‟t be the answer. So, option (d) too can‟t be the answer. Option (c) can‟t be applicable
as this DNA profiling can happen only after the Bill is passed, which will form a DNA regulatory
board, hence, there will be sufficient control over DNA data misuse. Option (b) can be the answer as
in India, though the population of the SCs, STs, OBCs and Muslims are in a minority, they
unsymmetrically represent a huge chunk in India‟s jailed population. Thus, this is a seed for a caste-
based profiling potential. Thus, the answer is option (b). Regarding the third question Section 34
under Chapter VI lays bare that the records maintained may be utilised for many things- from
facilitating adjudication and prosecution in criminal matters to identification for civil matters. This is to
say that if someone has rendered their DNA for a civil matter, their DNA could be used at a later
stage for cross-referencing in a criminal matter.Thus, consent given for the collection of DNA for civil
matters cannot be understood to be blanket consent for identification in a criminal case.Having a
unified database blurs the line between guilt and innocence. The presumption of innocence is
undermined, somehow making people who have given their DNA for civil matters less innocent than
those whose DNA is not on the database. Thus, option (a) is a pertinent issue. This Bill is not only at
odds with the right to privacy but also with the Personal Data Protection Bill. “Sensitive personal
data” under the Data Protection Bill includes genetic data that has secure parameters for collection,
processing and requires informed and specific consent. However, if an individual which includes
guardians of minors and relatives of victims under does not consent to give his DNA for an
investigation, a magistrate can easily override this consent and can order the collection under S.23
of the DNA Bill. Thus, taking away an individual‟s right to make intimate decisions about themselves
guaranteed under Article 21 of the Constitution. Thus, article (b) is a pertinent concern. Another
example of security failure is S.35(b) of the Bill. As per this section, highly confidential DNA data will
be made available to the personnel of DNA laboratories for training purposes. DNA data is highly
contaminable and it runs the risk of a large-scale breach of security. The Bill does not make a
provision for consent to be taken from individuals for their DNA to be used to educate trainees. Thus,
article (c) is a pertinent concern. There are no concerns raised over the appointment of National
DNA Data Bank. Thus, option (d) is the answer. The Supreme Court had held in Selvi vs. State of
Karnataka that personal autonomy is of utmost importance and any interference with it violates the
right to privacy especially in circumstances where a person can face criminal charges or penalties.
Thus, a possible cross-referencing of DNA collected for civil and criminal matters would lead to self-
incrimination. In the same case, Justice Ranjana Desai had observed that even though retention and
collection of DNA samples are not unconstitutional, future use and development of DNA profiling for
testimonial purposes could face constitutional obstacles under Article 20(3) of the Constitution. The
overriding power of the magistrate to procure the DNA sample of a person without her/his consent
violates an individual‟s right to make intimate decisions about themselves guaranteed under Article
21 of the Constitution. The Bill allows the use of the technology to establish the identity of persons in
matters of crime, parentage dispute, emigration or immigration and transplantation of human organs.
It provides for establishment of national and regional DNA data banks and each databank will
maintain crime scene index, suspects‟ or undertrials‟ index and offenders‟ index separately.
Link to solution: https://www.drishtiias.com/daily-updates/daily-news-analysis/concerns-over-dna-
technology-regulation-bill
https://www.hindustantimes.com/india-news/address-fears-over-dna-technology-bill-in-parliament-
and-outside-panel-to-govt-101612340870918.html
https://www.thehindu.com/news/national/dna-bill-can-lead-to-castecommunity-based-profiling-
owaisi-viswam/article33677524.ece
https://www.bloombergquint.com/opinion/the-dna-bill-another-invasive-imperfect-database
https://www.theleaflet.in/prodding-privacy-why-indias-dna-regulation-will-be-curtains-for-autonomy/#
68. Israel used Arrow 2 missile against Syria. War - March 2017 Israel–Syria incident.
According to Dov Raviv, a senior developer dubbed "the father of the Arrow anti-ballistic missile", a
single Arrow interceptor has a 90 percent probability of destroying a target missile at the highest
altitude possible.
The first launch of the Arrow interceptor took place on August 9, 1990, designed to test the missile''s
control and guidance systems.
The Arrow-4 and David‟s Sling are meant to replace the existing MIM-104 Patriot batteries in service
Source: Wikipedia
69. Israel used Arrow 2 missile against Syria. War - March 2017 Israel–Syria incident.
According to Dov Raviv, a senior developer dubbed "the father of the Arrow anti-ballistic missile", a
single Arrow interceptor has a 90 percent probability of destroying a target missile at the highest
altitude possible.
The first launch of the Arrow interceptor took place on August 9, 1990, designed to test the missile''s
control and guidance systems.
The Arrow-4 and David‟s Sling are meant to replace the existing MIM-104 Patriot batteries in service
Source: Wikipedia
70. Israel used Arrow 2 missile against Syria. War - March 2017 Israel–Syria incident.
According to Dov Raviv, a senior developer dubbed "the father of the Arrow anti-ballistic missile", a
single Arrow interceptor has a 90 percent probability of destroying a target missile at the highest
altitude possible.
The first launch of the Arrow interceptor took place on August 9, 1990, designed to test the missile''s
control and guidance systems.
The Arrow-4 and David‟s Sling are meant to replace the existing MIM-104 Patriot batteries in service
Source: Wikipedia

LEGAL REASONING

71. The correct answer is option (b). The peculiar facts i.e. planning a terrorist conspiracy do fall in
the exceptional category. Hence, their arrest is not indiscriminate or irrational in nature but rather,
imperative in nature.

72. The correct answer is option (c). The arrest in the given facts seems irrational and indiscriminate
in nature. Since arrests impinge upon personal liberty, they should only be chosen as a last resort. In
this case, John was directly arrested without an opportunity to defend himself. Option (a) is incorrect
because of the use of the word „unrestricted‟.

73. The correct answer is option (b). The totality of the facts suggest that Maruqi was arrested even
before he made a joke and did not actually hurt religious sentiments. In any case, the investigation
or interrogation could have been done without curtailing his personal liberty as his case was not
exceptional enough to be result in arrest.

74. The correct answer is option (d). A consideration of the totality of circumstances at hand
suggests that curtailing HP‟s personal liberty was necessary to control him and his remarks. Thus,
his arrest was based on a rational basis and hence, valid. Option (a) is not incorrect by itself but
option (d) is a better answer.
75. The correct answer is option (d). Nora‟s father cannot threaten her with a case since she is not
being „forced‟ to convert her religion in order to make the provisions of the Act or Ordinance
applicable to her.

76. The correct answer is option (d). In the year 1975, the 2020 Ordinance was not applicable under
which a person concerned as well as his parents and family can also file a complaint to the police
and the collector in a case of forceful conversion. Only Babli can file a case under the 1968 Act and
not her father.

77. The correct answer is option (b). Babli is eligible to file a case against Daku Mangal Singh
because she was converted under pressure. Since the year is 2021, the Ordinance will be applicable
and not the Act of 1968.

78. The correct answer is option (c). Getting married as per Zoroastrianism and Parsi customs is one
in the same thing, which is also indicated by the use of the word „also‟ in the question. Hence, no
religious autonomy or conversion law is being violated. Option (b) is incorrect because it is part of
option (c).

79. The correct answer is option (c). The passage clearly states that „any news item should be
published after verifying and clarifying all the facts‟. There is nothing to indicate in the given facts
that the attack was religiously motivated and such problematic reportage can cause deliberate and
unfair targeting of other religions.

80. The correct answer is option (d). The confessions made by the terrorist while in police custody
are not admissible as evidence in law and thus, cannot be used to convict him in a court of law.

81. The correct answer is option (c). The headline takes the statements of solidarity out of their
context and selectively quotes and misquotes the same. Option (d) is not relevant because it is not
directly concerned with the accused terrorist.

82. The correct answer is option (a). The passage clearly mentions that media should have such
basic knowledge of law as readers/viewers consider news item as true without verifying the facts.
Further, general public might not be aware of the law as above mentioned. Hence, the reportage
was problematic because it did not clarify to the general public the legal position on granting bail
wherein, an accused such as Jamun may be kept in custody for a period less than 24 hours.

83. The pen drive per se is not a document but the contents in the pen drive will be considered to be
evidence under section 3 of Evidence Act and hence it will be deemed to be document under section
65B of Evidence Act.

84. CCTV footage is not admissible as evidence. Though it is an electronic record and document
under section 65B of Evidence Act but cannot be evidence as it must be in original form and not
copied or reproduced in any form.

85. Telegram chats over emails are admissible as their source is known, they are electronic records
and fall under documentary evidence of Evidence Act.

86. Call recordings are electronic records and valid documentary evidences. Thus, they are
admissible in the court of law if they are related to relevant facts or facts in issue of the case. The
call recordings stated the presence of all 5 accused at the crime scene and are valid.
87. CDs and DVDs are valid and admissible as documentary evidences in court of law. They are
within the definition of electronic records under section 2 of IT Act and hence court was incorrect in
rejecting them as evidence.

88. As authorised by the magistrate, remand under police custody can be only for the period of 15
days and not beyond that. The period of remand shall be computed from date of detention and not
from date of arrest. Thus, date of detention in the case is 12-10-2019 and period of 15 days ends on
26-10-2019.

89. Period of remand for police custody cannot be extend beyond 15 days time and thus, remand as
contended can be extend for the period of 60 days (inclusive of 15 days from date of custody i.e 15th
July) till 12th September 2020 into Judicial custody only.

90. The Court will not send Arman back to police custody once sent to judicial custody. The
maximum period an accused can be in police custody is 15 days and not beyond that. Court can
send the accused to judicial custody from police custody before the expiry of 15 days time period but
cannot send back to police custody for the remaining time period.

91. The maximum period of remand that can be granted by the court is of 90 days and is inclusive of
police and judicial custody. Thus, after period of 15 days in police custody, if remand is extended to
60 days, then it meant to be of 45 days in judicial remand only.

92. The court is incorrect as the offence of money laundering is different from that of charges of
terrorist attack on Sanjay. Thus, different period of remands can be given to Sanjay for both offences
under police custody.

93. The court is correct in denying bail to Rishu as there are sufficient and strong evidences against
him which proves his involvement behind conspiring bomb blast at the Parliament. Thus, role of
judiciary cannot be questioned here.

94. Police cannot arrest Ms. Vijaya Kala for sedition charges as she was only showing her dissent
towards the functioning of present government and their lethargy in giving equal representation to
women in the Indian Parliament.

95. It is the duty of the state to look after the cases of such human body violations and conduct
investigation in these cases of murder. It is the role of State and it cannot step out of such duties.

96. Police is liable for keeping Shayant into illegal detention for long period of 90 days as there was
no prima facie evidence against him and they can only take remand for maximum period of 15 days
and investigates unless they find strong evidences against the accused/suspect.

97. The idea behind illegal detention is protect the individual‟s right to life and liberty and even
accused person has certain rights which has to be protected by the state. Article 20 of the Indian
Constitution provides support to accused and his rights. Thus, it cannot be violated even if he is
detained or sent to jail.

98. As mentioned in the first few lines of the second paragraph, the choice of an individual to marry a
person of their choice is intrinsic to Article 21, hence making Option C the correct answer. Option A
and B are incorrect, as they are not discussed in the passage. Option D is incorrect as it does not
answer the question asked.

99. Option A and C are unrelated to the paragraph, as Kelly‟s intention and non-disclosure
agreements are not related to the idea in the paragraph. Option C is partially correct as Philip‟s right
to privacy has been violated, but it mentions case law incorrectly. Hence Option D is incorrect, which
is also proved by the first paragraph of the passage.

100. as is evident by the line before it in the last paragraph that talks about same-sex couples having
the same right as those of heterosexual couples when it comes to choosing one‟s partner for
marriage. While Option A, B and D are also discussed in the passage, this line specifically pertains
to and fits with Option C.

101. Option A is although partially correct, it does not correlate with the idea discussed in the
paragraph. Similarly, with Options C and D, the options delve into the facts but fail to discuss the
main idea, which was discrimination on the basis of Elena‟s sexuality, and the subsequent
discrimination.

102. Options A, C and D are all incorrect as they can be included within the ambit of the right to
privacy, namely within informational privacy and privacy involving a person. However, Option B does
not involve the violation of the right as it deals with information related to a language of a lost
civilization, and not that of a person which could fall within these three heads. Hence, Option B is the
correct answer.

103. According to the passage, a tax official is empowered to seize the vehicle only if s/he suspects
that there is a possible tax evasion. In the current case, the official seized the vehicle on a suspicion
that X is a criminal. Hence, the tax official is not empowered to do so.

104. The law clearly states that laptops can‟t be transported with other electronic goods. Zyan is
transporting desktops along with laptops, which is in clear violation of the law. At the same time the
judgment states that if the situations envisaged in the legislation for detention are met, the goods
should be seized or detained without any laxity. Hence, there is no defense to Zyan.

105. In the given case Uma wants to challenge the detention. If Uma wants to challenge the seizure
or detention, she can furnish a security in the form of a bank guarantee covering the prescribed sum
and get his goods/vehicle released provisionally and then make her representation

106. It is explicitly mentioned in the passage that, if aggrieved, the consignee can approach the
relevant authority under the legislation in appeal, but cannot approach the High Court at this stage.
Hence, option B is the correct answer.

107. The arguments of the of the businessmen are not in consonance with the principles laid down
in the passage because the Kerala high court stated that detention and seizure of goods and
vehicles is a reasonable restriction on the exercise of free trade and movement

108. Neither a nor b is incorrect because the GST act states that 2 important things should be
carried by the seller while transporting goods. They are: 1) Invoice 2) e-way bill of the tax paid
LOGICAL REASONING

109. The author is obviously critical of NEET because it has not allowed reservations and in the
same way that privatization of educational institutions has cause(d) Thus, option (c) is the correct
answer.
110. The italicized line forms the conclusion of the opening arguments that the cases against
reservation is an irony. It can also be understood from the observation that the statement starts with
„thus‟. Hence, the answer is option (c).
111. The author mentions that there have been arguments that have said that reservations have
magnified caste lines, he opposes this line by saying that there have been many instances where
reservations is not followed like in NEET, which have omitted so many OBC candidates to the
colleges. Thus, the answer is option (c).
112. The author gives the reference of Shahuji Maharaj to give a historical context to arguments in
favour of reservation. Thus, statement 1 is correct. The author says the government jobs itself are so
scarce that not many have benefitted from the reservation these days. Thus, statement 2 is correct.
The author does mention that it is a wrong notion that reservation compromises merit in the country,
but has not provided arguments against it. Thus the answer is option (a).
113. The author obviously is not asking for equal opportunities for all but “equitable”. If this research
is correct, then also option A will not weaken the author‟s view. Option b will strengthen the author‟s
argument that it is an injustice that a section that is 85% in the country is actually inadequately
represented in key places. Thus, option (D) is the answer.
114. The first line of the paragraph says that dogs are tribal‟s best friend and later in paragraph 2,
the author mentions that government should not decide what tribal plates should serve. Hence it can
be clearly implied that the tribals‟ culture and food habits are different from those living in cities.
115. Tribal rights are protected under schedule 5 and 6 of the constitution. Sentence 2 can be
inferred from the first half of second paragraph
116. Government protects the rights under schedules of constitution and is also mentioned in
paragraph 1 – “…..the elected representatives should not only convince the people for resorting to
ban dog meat but also assure them that, their rights- especially cultural rights will not be encroached
and a commitment that they, the Govt. would be open to relook their decision, if so demanded by the
public”
117. the concluding paragraph of the passage supports the argument. The author clearly mentions
that these self-proclaimed activists can‟t be relied upon to know of sociological challenges and
diversity.
118. The author from the previous argument says that there are many other major issues to be dealt
with, and what the tribals should eat has to be the last one. Moreover they are intelligent enough to
interfere or decide about their affairs
Hence, C
119. The fact that Ursula lost her billionaire status because of huge donations to charity supports
option (a). Option (b) is incorrect, as nowhere is it mentioned that J.K. Ursula didn‟t want to be a
billionaire. Option (c) is incorrect as it isn‟t mentioned that she never believed that she would
become a billionaire. Option (d) is incorrect as there is nothing to suggest that Ursula never wrote for
fame.
Hence (a).
120. According to Nupur, success is measured in the net worth of a person and a role model for her
is someone who built their way from rags to riches. Option (b) states that Anil Kumble is the most
successful bowler in the world because he has scored more runs, which goes against Nupur‟s views
– she would consider the one who has a greater net worth to be more successful. But option (a) is
an example of the success she values, and option (c) is an example of a role model like Ursula.
Hence (d).
121. The word „contemporary‟ refers to „a person living at the same time as another‟. Hence when
Priyanshu says that „Anita Desai can easily surpass the level of writing done by her contemporaries‟,
he means that no one who lived around Anita Desai‟s time (1937- till now) is better than Anita Desai.
Option (a) is incorrect as Priyanshu says Anita Desai was undisputedly better than other authors
around the world. Option (b) is incorrect as there‟s no mention of Anita Desai being the best author
to ever exist. Option (c) is incorrect as „contemporary‟ and „temporary‟ aren‟t interchangeable.
Hence (d).
122. Nupur states that Ursula‟s „biggest donation has been the Earthsea books, which acted as a
safe space for several children who felt shunned‟. This would imply that her books helped children
who were feeling lonely. Option (a) is incorrect as there‟s no mention of donating books.
Option (b) is incorrect, as Nupur considers donating money to be Ursula‟s second biggest
contribution. Option (d) is incorrect as although Nupur does love Ursula‟s books, she talks about her
contribution being specifically about helping suffering children.
Hence (c).
123. Priyanshu says that „It wasn‟t her style of writing, but it was her enthralling and engaging plots
that got readers of various ages hooked from the very first word‟
Hence (c)
124. The conclusion that we are asked to weaken is: “the widespread belief that allergies cannot be
cured is incorrect.” The scoring option would say that the belief is correct. Option A states that the
immunotherapy “is effective only as long as patient stays on the therapy‟ implying that „there is no
(permanent) cure‟ thus weakening the claim. Option B is specific to Pine nut- allergy. Option C
implicitly strengthens the argument as the “body continues to produce antibodies … long after one is
exposed to the allergen.” In effect, there may be a cure. Option D is a general statement about
immunotherapy does not address the argument.
Hence, [A].
125. In the given statement, the lights are not off means „the lights are on‟. Unless the lights are not
off [on] means „if the lights are not on''. So, Nora does not dance if the lights are not on, which
means that she dances only when the lights are on. Hence, [B]
126. Statement 3 is not implied in the passage. Statements 1 and 2 are assumptions underlying the
passage. Elaborate practices of concealment are implied in the secrets “growing larger and scarier”
– hence more sophisticated methods of concealment become necessary. “if we shine light on them”
implies voluntary and public disclosure. Hence, [A].
127. The author points out (in paragraph 2) that the idea that imports make you poor is a fallacy, so
he is unlikely to disapprove of [A]. The passage does not mention taxation at all, so it is difficult to
say whether the author would approve of [B] or not. A closer reading of paragraph 1 shows that the
author does not disapprove of buying at bargain sales, but rather he dislikes the mentality of buyers
who do not think in terms of the benefit to the sellers. Throughout the passage, the author
demonstrates that he disapproves of zero-sum (i.e. win-lose) economic transactions. In paragraph 2,
he claims „some speculative markets in capital and in assets may be a zero-sum game‟. So it is
likely that he would disapprove of [C].
Hence, [C].
128. Option [B] can be inferred from the fact that the author calls the Marxists‟ notions a „fallacy‟. [C]
can be inferred from the last sentence of the first paragraph. [D] is clearly stated in the last sentence
of the second paragraph. However, the author would disagree with [A], as he says in the last
paragraph that people‟s negative view of capitalism long predated the credit crunch of 2008.
Hence, [A].
129. The last two sentences of the last paragraph – including the quoted sentence – describe
people‟s misunderstanding of the free market and capitalism in general. The quoted sentence
literally means that people take it for granted that free exchange is bad; but the author‟s view
regarding it is the opposite, and that is what he implies in this sentence.
Hence, [B].
130. Option (A) talks only about advertisements. Other parts of the movement may still use her
name. Option (B) is not acceptable as she is also the village chief and has responsibilities. Option
(D) too falls under avoiding personal responsibility. Hence, option (C) is correct.
131. Only option „A‟ will create a conflict of interest situation. Chhavi can support her father in a
personal capacity without affecting her role as a village chief. But A will be a clear case of unethical
conduct. Hence, option (A) is correct.
132. If this is true, it will strengthen Dholu''s case the most. It is the duty of the village chief to
maintain law and order. Chhavi''s failure in this regard can support Dholu''s demand for her
resignation. The other options don''t strengthen his case.

133.

134.

135.
136.

QUANTITATIVE TECHNIQUES

137. Males in X in 1992 : 80 × 58=50 thousand


Males in Y in 1992 : 50 × 25=20 thousand
No. of population in village Z in 1995 = 90 thousand
Required ratio = 7090=7 :9
138. It is clearly visible from the graph that population in village X decline continuously and uniformly

139. No of males in Z over the years = 80 × 916+70 ×37+120×35+59×90+49×90


= 45 + 30 + 72 + 50 + 40 = 237 thousand
No/ of females in X over the years = 100 × 920 +80×38+60×38+58×40+35×20
= 45 + 30 + 22.5 + 25 + 12 = 134500
237000134500=1.76 Required Ratio = times
140. No. of males in 1993 = 60 × 58+60×815+120×35=141.5 thosuand
No. of males in 1994 = 40 × 38+100×310+90×59=95 thosuand
Total males = 236.5 thousand
No. of females from Y in 1993=60×715=28No. of females from Y in 1994=100×710=70Total=98
Difference = 236.5 – 98 = 138.5 thousand.
Required% = 138.598×100=141.3% more
141. Average No. of female in X over the years
= 15100×920+80×38+60×38+40×58+20×35=1345005=26,900
Average population of village Z over the years = 1580+70+120+90+90=4500005=90000
Required difference = 90000 – 26900 = 63,100
142. Hint: Total number Of student in school
180 × 12 = 2160
Total students like M.S. Dhoni = 2160 × 25100 =540
Total students like Virat Kohli = 540 × 140100 = 756
Total students like Rohit Sharma = (2160 - 540 - 756) × 75100 = 648
Total students do not like any players = (2160 - 540 - 756 - 648 ) = 216
Total number of students who like only M.S Dhoni & Virat Kohli but not Rohit Sharm = 756 × 25100 =
189
Total students who like only Virat Kohli & Rohit Sharma but not M.S Dhoni = 189 + 21 =210
Total Students who like M.S Dhoni & Rohit Sharma but not Virat Kohli= 648 × 18=81+39 = 120
Total students who like all three players = 120 × 50100 = 60
Total Students who like Only M.S Dhoni = 540 - (189 + 120 + 60) = 171
Total students who like only Virat Kohli = 756 - (189 + 210 + 60 ) = 297
Total students who like only Rohit Sharma = 648 – (210 + 120 + 60) = 258
Total number of students who like only M.S. Dhoni &
Virat Kohli together = 171+297=468
Total students who like only Rohit Sharma = 258
Required percentage = 468-258258×100=811743%
143. Hint: Total number Of student in school
180 × 12 = 2160
Total students like M.S. Dhoni = 2160 × 25100 =540
Total students like Virat Kohli = 540 × 140100 = 756
Total students like Rohit Sharma = (2160 - 540 - 756) × 75100 = 648
Total students do not like any players = (2160 - 540 - 756 - 648 ) = 216
Total number of students who like only M.S Dhoni & Virat Kohli but not Rohit Sharm = 756 × 25100 =
189
Total students who like only Virat Kohli & Rohit Sharma but not M.S Dhoni = 189 + 21 =210
Total Students who like M.S Dhoni & Rohit Sharma but not Virat Kohli= 648 × 18=81+39 = 120
Total students who like all three players = 120 × 50100 = 60
Total Students who like Only M.S Dhoni = 540 - (189 + 120 + 60) = 171
Total students who like only Virat Kohli = 756 - (189 + 210 + 60 ) = 297
Total students who like only Rohit Sharma = 648 – (210 + 120 + 60) = 258
Total number of students who like at least two players = 189 + 210 + 120 + 60 = 579
144. Hint: Total number Of student in school
180 × 12 = 2160
Total students like M.S. Dhoni = 2160 × 25100 =540
Total students like Virat Kohli = 540 × 140100 = 756
Total students like Rohit Sharma = (2160 - 540 - 756) × 75100 = 648
Total students do not like any players = (2160 - 540 - 756 - 648 ) = 216
Total number of students who like only M.S Dhoni & Virat Kohli but not Rohit Sharm = 756 × 25100 =
189
Total students who like only Virat Kohli & Rohit Sharma but not M.S Dhoni = 189 + 21 =210
Total Students who like M.S Dhoni & Rohit Sharma but not Virat Kohli= 648 × 18=81+39 = 120
Total students who like all three players = 120 × 50100 = 60
Total Students who like Only M.S Dhoni = 540 - (189 + 120 + 60) = 171
Total students who like only Virat Kohli = 756 - (189 + 210 + 60 ) = 297
Total students who like only Rohit Sharma = 648 – (210 + 120 + 60) = 258
Required percentage = 216-60216×100
=156216×100
=72 2/9%
145. Hint: Total number Of student in school
180 × 12 = 2160
Total students like M.S. Dhoni = 2160 × 25100 =540
Total students like Virat Kohli = 540 × 140100 = 756
Total students like Rohit Sharma = (2160 - 540 - 756) × 75100 = 648
Total students do not like any players = (2160 - 540 - 756 - 648 ) = 216
Total number of students who like only M.S Dhoni & Virat Kohli but not Rohit Sharm = 756 × 25100 =
189
Total students who like only Virat Kohli & Rohit Sharma but not M.S Dhoni = 189 + 21 =210
Total Students who like M.S Dhoni & Rohit Sharma but not Virat Kohli= 648 × 18=81+39 = 120
Total students who like all three players = 120 × 50100 = 60
Total Students who like Only M.S Dhoni = 540 - (189 + 120 + 60) = 171
Total students who like only Virat Kohli = 756 - (189 + 210 + 60 ) = 297
Total students who like only Rohit Sharma = 648 – (210 + 120 + 60) = 258
Total number of students who like at least one player =2160-216=1944
146. Hint: Total number Of student in school
180 × 12 = 2160
Total students like M.S. Dhoni = 2160 × 25100 =540
Total students like Virat Kohli = 540 × 140100 = 756
Total students like Rohit Sharma = (2160 - 540 - 756) × 75100 = 648
Total students do not like any players = (2160 - 540 - 756 - 648 ) = 216
Total number of students who like only M.S Dhoni & Virat Kohli but not Rohit Sharm = 756 × 25100 =
189
Total students who like only Virat Kohli & Rohit Sharma but not M.S Dhoni = 189 + 21 =210
Total Students who like M.S Dhoni & Rohit Sharma but not Virat Kohli= 648 × 18=81+39 = 120
Total students who like all three players = 120 × 50100 = 60
Total Students who like Only M.S Dhoni = 540 - (189 + 120 + 60) = 171
Total students who like only Virat Kohli = 756 - (189 + 210 + 60 ) = 297
Total students who like only Rohit Sharma = 648 – (210 + 120 + 60) = 258
Required ratio = 189210 =9 :10
147. Given, rate offered by IDBI : rate offered by UCO = 3 : 4
Rate offered by UCO bank = 63×4 = 8%
Principle invested in UCO bank 2 year CI on 8% = 8 + 8 + 8 × 8100 =16.64%
Principle = 29160116.64×100=25000 Rs.
Amount obtained from IDBI = 10000 × 2 × 6100+10000
=1200+10000=11200
Required difference = 25000-11200=13800 Rs.
148. Rate = 10%
According to question
Principle invested in SBI = 26250100+4×10×100=18750 Rs.
Amounts obtained from Yes bank 3 years CI on 10% = 33.15
=20000×133.1100=26620 Rs.
Required % =1875026620×100=70.435% %
149. Principle invested in ICICI =200005×7=28000 Rs
Time = 3-1=2 year
2 year CI on 15% =15+15+15 × 15100=32.25% %
Required interest =28000× 32.25100=9030 Rs.
150. Rete offered by SBI =63×5=10% %
Time =42×1=2 year
Principle invested in SBI = 26250100 + 10 × 4×100=18750 Rs.
Amount obtained from IDBI =10000×100 + 2 × 6100
=10000×112100
=11200 Rs.
Required sum = 11200 + 18750 = 29950 Rs.
Legal Edge 22 Mock

Answers & Details

ENGLISH

1. Option A summarises the passage and the views of the author best. Thus, the correct answer is A.

2. The author in the passage commends Canberra’s move and urges India to follow the same. Thus, the
correct answer is A.

3. The following line indicates the same:

“Given the massive scale of Google and Facebook, monetising online traffic is to their advantage.
Meanwhile, it’s the media houses that painstakingly curate, verify and publish news through a network
of experienced reporters and editors”

The author believes that all the hard work is being done by the media houses and the social media
giants are just monetizing on it. Thus, the correct answer is B.

4. The following lines indicate the same:

“The argument that the internet giants bring additional traffic for news websites is specious.” Thus, the
correct answer is B.

5. In the lines before this one, the author says that all the hard work is being done by the media houses
and the social media giants are just monetizing on it, which he finds unfair. Thus, the correct answer is
D.

6. The author in the passage talks mainly about the freedom of press and how its freedom has been held
to be of paramount importance in a democracy. Thus, the correct answer is D.

7. Nowhere in the passage does the author talk about directive principles of state policy. Thus, the
correct answer is D.

8. The following line indicates the gist of the judgment:

“In the particular facts before it, the Court observed that the excessive nature of the burden owing to
the levy of duties on newsprint was neither adequately proven by the petitioners nor adequately
refuted by the respondents.”

Neither side could prove whether the taxes were excessive or not, thus the correct answer is D.

9. Impinge means to encroach upon. The SC is of the opinion that the government can levy taxes till they
do not encroach upon the freedom of press. Thus, the correct answer is A.
10. By saying that the swans “mate for life”, they provide an example of commitment, dedication, and
partnership. Thus, the correct answer is B.

11. The speaker notices that their hands are “folded” like “a pair of wings settling after flight.” The
simile here, comparing held hands to settled wings, suggests that a relationship is a place of comfort and
relief, like a home, to which a couple can always return. Thus, the correct answer is C.

12. Option C best summarises the passage and the author’s views best. Thus, the correct answer is C.

13. The author is of the view that the charge of sedition is imposed even when it lawfully shouldn’t be,
implying that the police officers do a lax job of reading the said section. Thus, the correct answer is A.

14. Infirmity means weakness, which is the closest in meaning to frailty. Thus, the correct answer is C.

15. The following lines indicate the same:

“The guard rails imposed by the Supreme Court, requiring direct and immediate incitement to violence
to qualify as sedition, have had no effect.” The author says that the guard rails imposed by the supreme
court have had no effect. Thus, the correct answer is B.

16. The author has mentioned all of the above statements in the above passage and all of them describe
his views. Thus, the correct answer is D.

17. The following lines indicate the same:

“The guard rails imposed by the Supreme Court, requiring direct and immediate incitement to violence
to qualify as sedition, have had no effect.” The author says that ‘direct and immediate incitement to
violence’ is a guard rail, i.e., a sort of protection against frivolous sedition cases, the avoidance of which
must invite contempt of court, according to the author. Thus, the correct answer is A.

18. Egregious means outstandingly bad or shocking. Option A is the most opposite the term and thus, is
the right answer.

19. The author is critical, disapproving and angry at the police using sedition charges laxly. Thus, the
correct answer is D.

20. The following line indicates the same:

“Such a glacial pace with no prospect of bail heightens the injustice towards pretrial/ undertrial
prisoners and those ultimately acquitted.” Thus, the correct answer is C.
21. Option A is the essence of the passage and the subject matter of the passage. Thus, the correct
answer is A.

22. The following line indicates the same:

“News content already undertakes compliance with various standalone legislations. Plus news sites
follow print and TV norms, besides the extensive self-regulation done in multiple layers between
journalists and editors every day.”

Thus, the correct answer is A.

23. The author contrasts social media with news sites, saying that the former has more accountability,
while the latter has almost none. Thus, the best suited word for it is “amuck”.

24. The following line indicates the same:

“Government’s aim to impose accountability on social media to tackle a pandemic of fake news and
hatred is sound.” This shows the aim of the government behind the new rules. Thus, the correct answer
is B.

25. State overreach is the State overstepping its boundaries and doing acts which are outside its power,
thus encroaching upon civil liberties. Option C says that the oversight authority has been granted
immense powers which might lead to strong arming by the state. Thus, the correct answer is C

26. The correct answer is option a) Narrative. Narrative writing is used in almost every longer piece of
writing, whether fiction or nonfiction. When an author writes in a narrative style, they are not just trying
to impart information, they are trying to construct and communicate a story, complete with characters,
conflict, and settings which is true to the passage.

27. the correct answer is option a). Evangelists are people who seeks to convert others to the Christian
faith, especially by public preaching.

28. The correct answer is option b). The meaning of the word ‘phalanx’ is ‘a body of troop or police
officers standing or moving in close formation

29. The correct answer is option d) as none of the statements are true. The first statement is not true
because the third paragraph states that, ‘Rosita is from Loreto, and if you listen carefully you can still
catch in her voice an echo of the delightful singsong accent of eastern Peru.’ Which clearly signifies that
the accent in Rosita’s voice was very subtle and not distinct. The second statement is false as well as the
third and last paragraph stated about the controversies and accusations surrounding the university.
30. the correct answer is option a) because only statement 1 is true as has been described in the first
paragraph of the passage. The second statement is not true as it is not mentioned when did the author
actually meet Rosita for the first time as the sentence says, ‘I first became acquainted with the Amazon
jungle halfway through 1958, thanks to my friend Rosita Corpancho’.

GENERAL KNOWLEDGE

31. A victory pillar (Vijay Sthamb) was erected in Koregaon by the British, commemorating the dead
soldiers. In 1928, the Dalit leader B. R. Ambedkar led the first commemoration ceremony here.

The victory pillar (also known as Ransthamb) in Bhima-Koregaon village (Pune district of Maharashtra) is
to celebrate the 202nd anniversary of the Bhima-Koregaon battle of 1818 on January 1, 2020.

The term "Elgar" meant loud invitation or loud declaration.

Source – Dristhi IAS and Wikipedia

The Battle of Koregaon (also called the Battle of Koregaon Bhima, after the river Bhima that flows close
to it) was fought on 1 January 1818 between the British East India Company and the Peshwa faction of
the Maratha Confederacy, at Koregaon Bhima.

Unlawful Activities (Prevention) Act is an Indian law aimed at prevention of unlawful activities
associations in India. Its main objective was to make powers available for dealing with activities directed
against the integrity and sovereignty of India.

32. In 2018, incidents of violence were registered during the celebration of the 200th anniversary of the
Bhima-Koregaon battle.

33. A victory pillar (Vijay Sthamb) was erected in Koregaon by the British, commemorating the dead
soldiers. In 1928, the Dalit leader B. R. Ambedkar led the first commemoration ceremony here.

The victory pillar (also known as Ransthamb) in Bhima-Koregaon village (Pune district of Maharashtra) is
to celebrate the 202nd anniversary of the Bhima-Koregaon battle of 1818 on January 1, 2020.

The term "Elgar" meant loud invitation or loud declaration.

Source – Dristhi IAS and Wikipedia

The Battle of Koregaon (also called the Battle of Koregaon Bhima, after the river Bhima that flows close
to it) was fought on 1 January 1818 between the British East India Company and the Peshwa faction of
the Maratha Confederacy, at Koregaon Bhima.

Unlawful Activities (Prevention) Act is an Indian law aimed at prevention of unlawful activities
associations in India. Its main objective was to make powers available for dealing with activities directed
against the integrity and sovereignty of India.
34. A victory pillar (Vijay Sthamb) was erected in Koregaon by the British, commemorating the dead
soldiers. In 1928, the Dalit leader B. R. Ambedkar led the first commemoration ceremony here.

The victory pillar (also known as Ransthamb) in Bhima-Koregaon village (Pune district of Maharashtra) is
to celebrate the 202nd anniversary of the Bhima-Koregaon battle of 1818 on January 1, 2020.

The term "Elgar" meant loud invitation or loud declaration.

Source – Dristhi IAS and Wikipedia

The Battle of Koregaon (also called the Battle of Koregaon Bhima, after the river Bhima that flows close
to it) was fought on 1 January 1818 between the British East India Company and the Peshwa faction of
the Maratha Confederacy, at Koregaon Bhima.

Unlawful Activities (Prevention) Act is an Indian law aimed at prevention of unlawful activities
associations in India. Its main objective was to make powers available for dealing with activities directed
against the integrity and sovereignty of India.

35. A victory pillar (Vijay Sthamb) was erected in Koregaon by the British, commemorating the dead
soldiers. In 1928, the Dalit leader B. R. Ambedkar led the first commemoration ceremony here.

The victory pillar (also known as Ransthamb) in Bhima-Koregaon village (Pune district of Maharashtra) is
to celebrate the 202nd anniversary of the Bhima-Koregaon battle of 1818 on January 1, 2020.

The term "Elgar" meant loud invitation or loud declaration.

Source – Dristhi IAS and Wikipedia

The Battle of Koregaon (also called the Battle of Koregaon Bhima, after the river Bhima that flows close
to it) was fought on 1 January 1818 between the British East India Company and the Peshwa faction of
the Maratha Confederacy, at Koregaon Bhima.

Unlawful Activities (Prevention) Act is an Indian law aimed at prevention of unlawful activities
associations in India. Its main objective was to make powers available for dealing with activities directed
against the integrity and sovereignty of India.

36. British used Sedition law to convict and sentence freedom fighters. It was first used to prosecute Bal
Gangadhar Tilak in 1897.

Law was borrowed and inserted into the Section 124A of IPC in 1870, by the British.

Sir James Fitzjames Stephen, when introducing this Bill in August 1870 to amend the Penal Code
observed that the provision in question was one which, by some unaccountable mistake, had been
omitted from the Penal Code as ultimately passed.

Ever since the separatist movement gathered force in the 1980s, the territorial ambitions of Khalistan
have at times included Chandigarh, sections of the Indian Punjab, including the whole of North India,
and some parts of the western states of India.Prime Minister of Pakistan Zulfikar Ali Bhutto, according to
Jagjit Singh Chohan, had proposed all out help to create Khalistan during his talks with Chohan, following
the conclusion of the Indo-Pakistani War of 1971.

The Khalistan movement is a Sikh separatist movement seeking to create a homeland for Sikhs by
establishing a sovereign state, called Khālistān (''Land of the Khalsa''), in the Punjab region.The proposed
state would consist of land that currently forms Punjab, India and Punjab, Pakistan.

37. British used Sedition law to convict and sentence freedom fighters. It was first used to prosecute Bal
Gangadhar Tilak in 1897.

Law was borrowed and inserted into the Section 124A of IPC in 1870, by the British.

Sir James Fitzjames Stephen, when introducing this Bill in August 1870 to amend the Penal Code
observed that the provision in question was one which, by some unaccountable mistake, had been
omitted from the Penal Code as ultimately passed.

Ever since the separatist movement gathered force in the 1980s, the territorial ambitions of Khalistan
have at times included Chandigarh, sections of the Indian Punjab, including the whole of North India,
and some parts of the western states of India.Prime Minister of Pakistan Zulfikar Ali Bhutto, according to
Jagjit Singh Chohan, had proposed all out help to create Khalistan during his talks with Chohan, following
the conclusion of the Indo-Pakistani War of 1971.

The Khalistan movement is a Sikh separatist movement seeking to create a homeland for Sikhs by
establishing a sovereign state, called Khālistān (''Land of the Khalsa''), in the Punjab region.The proposed
state would consist of land that currently forms Punjab, India and Punjab, Pakistan.

38. British used Sedition law to convict and sentence freedom fighters. It was first used to prosecute Bal
Gangadhar Tilak in 1897.

Law was borrowed and inserted into the Section 124A of IPC in 1870, by the British.

Sir James Fitzjames Stephen, when introducing this Bill in August 1870 to amend the Penal Code
observed that the provision in question was one which, by some unaccountable mistake, had been
omitted from the Penal Code as ultimately passed.

Ever since the separatist movement gathered force in the 1980s, the territorial ambitions of Khalistan
have at times included Chandigarh, sections of the Indian Punjab, including the whole of North India,
and some parts of the western states of India.Prime Minister of Pakistan Zulfikar Ali Bhutto, according to
Jagjit Singh Chohan, had proposed all out help to create Khalistan during his talks with Chohan, following
the conclusion of the Indo-Pakistani War of 1971.

The Khalistan movement is a Sikh separatist movement seeking to create a homeland for Sikhs by
establishing a sovereign state, called Khālistān (''Land of the Khalsa''), in the Punjab region.The proposed
state would consist of land that currently forms Punjab, India and Punjab, Pakistan.
39. British used Sedition law to convict and sentence freedom fighters. It was first used to prosecute Bal
Gangadhar Tilak in 1897.

Law was borrowed and inserted into the Section 124A of IPC in 1870, by the British.

Sir James Fitzjames Stephen, when introducing this Bill in August 1870 to amend the Penal Code
observed that the provision in question was one which, by some unaccountable mistake, had been
omitted from the Penal Code as ultimately passed.

Ever since the separatist movement gathered force in the 1980s, the territorial ambitions of Khalistan
have at times included Chandigarh, sections of the Indian Punjab, including the whole of North India,
and some parts of the western states of India.Prime Minister of Pakistan Zulfikar Ali Bhutto, according to
Jagjit Singh Chohan, had proposed all out help to create Khalistan during his talks with Chohan, following
the conclusion of the Indo-Pakistani War of 1971.

The Khalistan movement is a Sikh separatist movement seeking to create a homeland for Sikhs by
establishing a sovereign state, called Khālistān (''Land of the Khalsa''), in the Punjab region.The proposed
state would consist of land that currently forms Punjab, India and Punjab, Pakistan.

40. British used Sedition law to convict and sentence freedom fighters. It was first used to prosecute Bal
Gangadhar Tilak in 1897.

Law was borrowed and inserted into the Section 124A of IPC in 1870, by the British.

Sir James Fitzjames Stephen, when introducing this Bill in August 1870 to amend the Penal Code
observed that the provision in question was one which, by some unaccountable mistake, had been
omitted from the Penal Code as ultimately passed.

Ever since the separatist movement gathered force in the 1980s, the territorial ambitions of Khalistan
have at times included Chandigarh, sections of the Indian Punjab, including the whole of North India,
and some parts of the western states of India.Prime Minister of Pakistan Zulfikar Ali Bhutto, according to
Jagjit Singh Chohan, had proposed all out help to create Khalistan during his talks with Chohan, following
the conclusion of the Indo-Pakistani War of 1971.

The Khalistan movement is a Sikh separatist movement seeking to create a homeland for Sikhs by
establishing a sovereign state, called Khālistān (''Land of the Khalsa''), in the Punjab region.The proposed
state would consist of land that currently forms Punjab, India and Punjab, Pakistan.

41. Disguised Unemployment: It is a phenomenon wherein more people are employed than actually
needed.

Mahatma Gandhi National Rural Employment Guarantee Act (MNREGA) was launched in 2005 to
provide social security by guaranteeing a minimum of 100 days paid work per year to all the families
whose adult members opt for unskilled labour-intensive work.
The General Theory of Employment, Interest and Money was written by the English economist John
Maynard Keynes.

We can calculate the unemployment rate by dividing the number of unemployed people by the total
number in the labor force, then multiplying by 100.

National Sample Survey Office (NSSO), an organization under Ministry of Statistics and Programme
Implementation (MoSPI) measures unemployment in India on following approaches:

Usual Status Approach: This approach estimates only those persons as unemployed who had no gainful
work for a major time during the 365 days preceding the date of survey.

Weekly Status Approach: This approach records only those persons as unemployed who did not have
gainful work even for an hour on any day of the week preceding the date of survey.

Daily Status Approach: Under this approach, unemployment status of a person is measured for each day
in a reference week. A person having no gainful work even for 1 hour in a day is described as
unemployed for that day.

42. Disguised Unemployment: It is a phenomenon wherein more people are employed than actually
needed.

Mahatma Gandhi National Rural Employment Guarantee Act (MNREGA) was launched in 2005 to
provide social security by guaranteeing a minimum of 100 days paid work per year to all the families
whose adult members opt for unskilled labour-intensive work.

The General Theory of Employment, Interest and Money was written by the English economist John
Maynard Keynes.

We can calculate the unemployment rate by dividing the number of unemployed people by the total
number in the labor force, then multiplying by 100.

National Sample Survey Office (NSSO), an organization under Ministry of Statistics and Programme
Implementation (MoSPI) measures unemployment in India on following approaches:

Usual Status Approach: This approach estimates only those persons as unemployed who had no gainful
work for a major time during the 365 days preceding the date of survey.

Weekly Status Approach: This approach records only those persons as unemployed who did not have
gainful work even for an hour on any day of the week preceding the date of survey.

Daily Status Approach: Under this approach, unemployment status of a person is measured for each day
in a reference week. A person having no gainful work even for 1 hour in a day is described as
unemployed for that day.
43. Disguised Unemployment: It is a phenomenon wherein more people are employed than actually
needed.

Mahatma Gandhi National Rural Employment Guarantee Act (MNREGA) was launched in 2005 to
provide social security by guaranteeing a minimum of 100 days paid work per year to all the families
whose adult members opt for unskilled labour-intensive work.

The General Theory of Employment, Interest and Money was written by the English economist John
Maynard Keynes.

We can calculate the unemployment rate by dividing the number of unemployed people by the total
number in the labor force, then multiplying by 100.

National Sample Survey Office (NSSO), an organization under Ministry of Statistics and Programme
Implementation (MoSPI) measures unemployment in India on following approaches:

Usual Status Approach: This approach estimates only those persons as unemployed who had no gainful
work for a major time during the 365 days preceding the date of survey.

Weekly Status Approach: This approach records only those persons as unemployed who did not have
gainful work even for an hour on any day of the week preceding the date of survey.

Daily Status Approach: Under this approach, unemployment status of a person is measured for each day
in a reference week. A person having no gainful work even for 1 hour in a day is described as
unemployed for that day.

44. Disguised Unemployment: It is a phenomenon wherein more people are employed than actually
needed.

Mahatma Gandhi National Rural Employment Guarantee Act (MNREGA) was launched in 2005 to
provide social security by guaranteeing a minimum of 100 days paid work per year to all the families
whose adult members opt for unskilled labour-intensive work.

The General Theory of Employment, Interest and Money was written by the English economist John
Maynard Keynes.

We can calculate the unemployment rate by dividing the number of unemployed people by the total
number in the labor force, then multiplying by 100.

National Sample Survey Office (NSSO), an organization under Ministry of Statistics and Programme
Implementation (MoSPI) measures unemployment in India on following approaches:

Usual Status Approach: This approach estimates only those persons as unemployed who had no gainful
work for a major time during the 365 days preceding the date of survey.
Weekly Status Approach: This approach records only those persons as unemployed who did not have
gainful work even for an hour on any day of the week preceding the date of survey.

Daily Status Approach: Under this approach, unemployment status of a person is measured for each day
in a reference week. A person having no gainful work even for 1 hour in a day is described as
unemployed for that day.

45. Disguised Unemployment: It is a phenomenon wherein more people are employed than actually
needed.

Mahatma Gandhi National Rural Employment Guarantee Act (MNREGA) was launched in 2005 to
provide social security by guaranteeing a minimum of 100 days paid work per year to all the families
whose adult members opt for unskilled labour-intensive work.

The General Theory of Employment, Interest and Money was written by the English economist John
Maynard Keynes.

We can calculate the unemployment rate by dividing the number of unemployed people by the total
number in the labor force, then multiplying by 100.

National Sample Survey Office (NSSO), an organization under Ministry of Statistics and Programme
Implementation (MoSPI) measures unemployment in India on following approaches:

Usual Status Approach: This approach estimates only those persons as unemployed who had no gainful
work for a major time during the 365 days preceding the date of survey.

Weekly Status Approach: This approach records only those persons as unemployed who did not have
gainful work even for an hour on any day of the week preceding the date of survey.

Daily Status Approach: Under this approach, unemployment status of a person is measured for each day
in a reference week. A person having no gainful work even for 1 hour in a day is described as
unemployed for that day.

46. As of 2019, FATF has blacklisted North Korea and Iran over terror financing. Twelve countries are in
the grey list, namely: Bahamas, Botswana, Cambodia, Ethiopia, Ghana, Pakistan, Panama, Sri Lanka,
Syria, Trinidad and Tobago, Tunisia and Yemen.

The FATF had issued the 27-point action plan after placing Pakistan on the ‘Grey List’ in June 2018. The
action plan pertains to curbing money laundering and terror financing.

The FATF currently has 39 members including two regional organisations — the European Commission
and Gulf Cooperation Council. India is a member of the FATF.

i. The FATF listing makes it extremely difficult for Pakistan to get financial aid from the International
Monetary Fund (IMF), the World Bank and the European Union.
ii. Turkey proposed that the members should consider Pakistan’s good work and instead of waiting for
completion of the remaining six of the 27 parameters. When the proposal was placed before the
Plenary, no other member seconded the move. It was not supported by even China, Malaysia and Saudi
Arabia.

Dr. Marcus Pleyer of Germany assumed the position of President of the FATF on 1 July 2020. He
succeeded Xiangmin Liu of the People''s Republic of China.

47. As of 2019, FATF has blacklisted North Korea and Iran over terror financing. Twelve countries are in
the grey list, namely: Bahamas, Botswana, Cambodia, Ethiopia, Ghana, Pakistan, Panama, Sri Lanka,
Syria, Trinidad and Tobago, Tunisia and Yemen.

The FATF had issued the 27-point action plan after placing Pakistan on the ‘Grey List’ in June 2018. The
action plan pertains to curbing money laundering and terror financing.

The FATF currently has 39 members including two regional organisations — the European Commission
and Gulf Cooperation Council. India is a member of the FATF.

i. The FATF listing makes it extremely difficult for Pakistan to get financial aid from the International
Monetary Fund (IMF), the World Bank and the European Union.

ii. Turkey proposed that the members should consider Pakistan’s good work and instead of waiting for
completion of the remaining six of the 27 parameters. When the proposal was placed before the
Plenary, no other member seconded the move. It was not supported by even China, Malaysia and Saudi
Arabia.

Dr. Marcus Pleyer of Germany assumed the position of President of the FATF on 1 July 2020. He
succeeded Xiangmin Liu of the People''s Republic of China.

48. As of 2019, FATF has blacklisted North Korea and Iran over terror financing. Twelve countries are in
the grey list, namely: Bahamas, Botswana, Cambodia, Ethiopia, Ghana, Pakistan, Panama, Sri Lanka,
Syria, Trinidad and Tobago, Tunisia and Yemen.

The FATF had issued the 27-point action plan after placing Pakistan on the ‘Grey List’ in June 2018. The
action plan pertains to curbing money laundering and terror financing.

The FATF currently has 39 members including two regional organisations — the European Commission
and Gulf Cooperation Council. India is a member of the FATF.

i. The FATF listing makes it extremely difficult for Pakistan to get financial aid from the International
Monetary Fund (IMF), the World Bank and the European Union.

ii. Turkey proposed that the members should consider Pakistan’s good work and instead of waiting for
completion of the remaining six of the 27 parameters. When the proposal was placed before the
Plenary, no other member seconded the move. It was not supported by even China, Malaysia and Saudi
Arabia.
Dr. Marcus Pleyer of Germany assumed the position of President of the FATF on 1 July 2020. He
succeeded Xiangmin Liu of the People''s Republic of China.

49. As of 2019, FATF has blacklisted North Korea and Iran over terror financing. Twelve countries are in
the grey list, namely: Bahamas, Botswana, Cambodia, Ethiopia, Ghana, Pakistan, Panama, Sri Lanka,
Syria, Trinidad and Tobago, Tunisia and Yemen.

The FATF had issued the 27-point action plan after placing Pakistan on the ‘Grey List’ in June 2018. The
action plan pertains to curbing money laundering and terror financing.

The FATF currently has 39 members including two regional organisations — the European Commission
and Gulf Cooperation Council. India is a member of the FATF.

i. The FATF listing makes it extremely difficult for Pakistan to get financial aid from the International
Monetary Fund (IMF), the World Bank and the European Union.

ii. Turkey proposed that the members should consider Pakistan’s good work and instead of waiting for
completion of the remaining six of the 27 parameters. When the proposal was placed before the
Plenary, no other member seconded the move. It was not supported by even China, Malaysia and Saudi
Arabia.

Dr. Marcus Pleyer of Germany assumed the position of President of the FATF on 1 July 2020. He
succeeded Xiangmin Liu of the People''s Republic of China.

50. Assamese singer, music composer, film director and actor, Bhupen Hazarika. Popularly known as the
‘Bard of the Brahmaputra’, the inimitable Bhupen da enthralled many music lovers across the world and
significantly so in Assam, Bengal, Bangladesh, Bollywood and beyond.

Will provide seamless connectivity to the Eastern parts of India

Will enhance various development activities for the people living around River Brahmaputra and River
Barak.

Ro-Pax services will help in reducing the travel time

Speedier connectivity between banks will reduce the distance to be travelled by road

Ro-Pax operation between Neamati and Majuli will reduce the total distance of 420 Km currently being
travelled by vehicles to only 12 Km

Small-scale industries of the region will derive logistical benefits to grow faster.

The Dhubri-Phulbari bridge will reduce the distance of 205km to 19km, which is the total length of the
bridge.

The Majuli bridge will be located on NH-715K and will connect Neematighat (on Jorhat side) and
Kamalabari (on Majuli side).
L&T Construction, the construction arm of infrastructure major Larsen and Toubro (L&T), has bagged the
contract to build India''s longest river bridge.

Recently, the company has won a ''mega'' contract worth more than ₹7,000 crore from National High-
Speed Rail Corpto construct 87.57 km long stretch of Mumbai-Ahmedabad high-speed rail also known as
the bullet train project.

51. Assamese singer, music composer, film director and actor, Bhupen Hazarika. Popularly known as the
‘Bard of the Brahmaputra’, the inimitable Bhupen da enthralled many music lovers across the world and
significantly so in Assam, Bengal, Bangladesh, Bollywood and beyond.

Will provide seamless connectivity to the Eastern parts of India

Will enhance various development activities for the people living around River Brahmaputra and River
Barak.

Ro-Pax services will help in reducing the travel time

Speedier connectivity between banks will reduce the distance to be travelled by road

Ro-Pax operation between Neamati and Majuli will reduce the total distance of 420 Km currently being
travelled by vehicles to only 12 Km

Small-scale industries of the region will derive logistical benefits to grow faster.

The Dhubri-Phulbari bridge will reduce the distance of 205km to 19km, which is the total length of the
bridge.

The Majuli bridge will be located on NH-715K and will connect Neematighat (on Jorhat side) and
Kamalabari (on Majuli side).

L&T Construction, the construction arm of infrastructure major Larsen and Toubro (L&T), has bagged the
contract to build India''s longest river bridge.

Recently, the company has won a ''mega'' contract worth more than ₹7,000 crore from National High-
Speed Rail Corpto construct 87.57 km long stretch of Mumbai-Ahmedabad high-speed rail also known as
the bullet train project.

52. Assamese singer, music composer, film director and actor, Bhupen Hazarika. Popularly known as the
‘Bard of the Brahmaputra’, the inimitable Bhupen da enthralled many music lovers across the world and
significantly so in Assam, Bengal, Bangladesh, Bollywood and beyond.

Will provide seamless connectivity to the Eastern parts of India

Will enhance various development activities for the people living around River Brahmaputra and River
Barak.
Ro-Pax services will help in reducing the travel time

Speedier connectivity between banks will reduce the distance to be travelled by road

Ro-Pax operation between Neamati and Majuli will reduce the total distance of 420 Km currently being
travelled by vehicles to only 12 Km

Small-scale industries of the region will derive logistical benefits to grow faster.

The Dhubri-Phulbari bridge will reduce the distance of 205km to 19km, which is the total length of the
bridge.

The Majuli bridge will be located on NH-715K and will connect Neematighat (on Jorhat side) and
Kamalabari (on Majuli side).

L&T Construction, the construction arm of infrastructure major Larsen and Toubro (L&T), has bagged the
contract to build India''s longest river bridge.

Recently, the company has won a ''mega'' contract worth more than ₹7,000 crore from National High-
Speed Rail Corpto construct 87.57 km long stretch of Mumbai-Ahmedabad high-speed rail also known as
the bullet train project.

53. Assamese singer, music composer, film director and actor, Bhupen Hazarika. Popularly known as the
‘Bard of the Brahmaputra’, the inimitable Bhupen da enthralled many music lovers across the world and
significantly so in Assam, Bengal, Bangladesh, Bollywood and beyond.

Will provide seamless connectivity to the Eastern parts of India

Will enhance various development activities for the people living around River Brahmaputra and River
Barak.

Ro-Pax services will help in reducing the travel time

Speedier connectivity between banks will reduce the distance to be travelled by road

Ro-Pax operation between Neamati and Majuli will reduce the total distance of 420 Km currently being
travelled by vehicles to only 12 Km

Small-scale industries of the region will derive logistical benefits to grow faster.

The Dhubri-Phulbari bridge will reduce the distance of 205km to 19km, which is the total length of the
bridge.

The Majuli bridge will be located on NH-715K and will connect Neematighat (on Jorhat side) and
Kamalabari (on Majuli side).

L&T Construction, the construction arm of infrastructure major Larsen and Toubro (L&T), has bagged the
contract to build India''s longest river bridge.
Recently, the company has won a ''mega'' contract worth more than ₹7,000 crore from National High-
Speed Rail Corpto construct 87.57 km long stretch of Mumbai-Ahmedabad high-speed rail also known as
the bullet train project.

54. The government refused to support code and expressed displeasure at a model suggested by the
Internet and Mobile Association of India (IAMAI), and Digital Curated Content Complaints Council
(DCCC).

Recently, the government has notified Information Technology (Intermediary Guidelines and Digital
Media Ethics Code) Rules 2021.

These rules have been framed in exercise of powers under section 87 (2) of the Information Technology
(IT) Act, 2000 and in supersession of the earlier Information Technology (Intermediary Guidelines) Rules
2011.

These new rules broadly deal with social media and over-the-top (OTT) platforms.

The Cable Television Network (Regulation) Act, 1995 penalises television channels for any violation of
the programming and advertising.

Content that deliberately and maliciously disrespects the national emblem or national flag.

Any visual or storyline that promotes child pornography.

Any content that “maliciously” intends to outrage religious sentiments.

Content that “deliberately and maliciously” promotes or encourages terrorism.

Any content that has been banned for exhibition or distribution by law or court.

55. The government refused to support code and expressed displeasure at a model suggested by the
Internet and Mobile Association of India (IAMAI), and Digital Curated Content Complaints Council
(DCCC).

Recently, the government has notified Information Technology (Intermediary Guidelines and Digital
Media Ethics Code) Rules 2021.

These rules have been framed in exercise of powers under section 87 (2) of the Information Technology
(IT) Act, 2000 and in supersession of the earlier Information Technology (Intermediary Guidelines) Rules
2011.

These new rules broadly deal with social media and over-the-top (OTT) platforms.

The Cable Television Network (Regulation) Act, 1995 penalises television channels for any violation of
the programming and advertising.
Content that deliberately and maliciously disrespects the national emblem or national flag.

Any visual or storyline that promotes child pornography.

Any content that “maliciously” intends to outrage religious sentiments.

Content that “deliberately and maliciously” promotes or encourages terrorism.

Any content that has been banned for exhibition or distribution by law or court.

56. The government refused to support code and expressed displeasure at a model suggested by the
Internet and Mobile Association of India (IAMAI), and Digital Curated Content Complaints Council
(DCCC).

Recently, the government has notified Information Technology (Intermediary Guidelines and Digital
Media Ethics Code) Rules 2021.

These rules have been framed in exercise of powers under section 87 (2) of the Information Technology
(IT) Act, 2000 and in supersession of the earlier Information Technology (Intermediary Guidelines) Rules
2011.

These new rules broadly deal with social media and over-the-top (OTT) platforms.

The Cable Television Network (Regulation) Act, 1995 penalises television channels for any violation of
the programming and advertising.

Content that deliberately and maliciously disrespects the national emblem or national flag.

Any visual or storyline that promotes child pornography.

Any content that “maliciously” intends to outrage religious sentiments.

Content that “deliberately and maliciously” promotes or encourages terrorism.

Any content that has been banned for exhibition or distribution by law or court.

57. The government refused to support code and expressed displeasure at a model suggested by the
Internet and Mobile Association of India (IAMAI), and Digital Curated Content Complaints Council
(DCCC).

Recently, the government has notified Information Technology (Intermediary Guidelines and Digital
Media Ethics Code) Rules 2021.

These rules have been framed in exercise of powers under section 87 (2) of the Information Technology
(IT) Act, 2000 and in supersession of the earlier Information Technology (Intermediary Guidelines) Rules
2011.
These new rules broadly deal with social media and over-the-top (OTT) platforms.

The Cable Television Network (Regulation) Act, 1995 penalises television channels for any violation of
the programming and advertising.

Content that deliberately and maliciously disrespects the national emblem or national flag.

Any visual or storyline that promotes child pornography.

Any content that “maliciously” intends to outrage religious sentiments.

Content that “deliberately and maliciously” promotes or encourages terrorism.

Any content that has been banned for exhibition or distribution by law or court.

58. A proclamation imposing President’s Rule must be approved by both the Houses of Parliament
within two months from the date of its issue.

The state governor, on behalf of the President, carries on the state administration with the help of the
chief secretary of the state or the advisors appointed by the President.

The Government of Union Territories Act, 1963 enacted by the Parliament in accordance with the
provisions of article 239A.

The Union Territory of Puducherry was placed under President’s Rule, three days after the V
Narayanasamy-led Congress government failed to prove its majority in a floor test in the Assembly.

President’s Rule implies the suspension of a state government and the imposition of direct rule of the
Centre. It is also known as ‘State Emergency’ or ‘Constitutional Emergency’.

59. A proclamation imposing President’s Rule must be approved by both the Houses of Parliament
within two months from the date of its issue.

The state governor, on behalf of the President, carries on the state administration with the help of the
chief secretary of the state or the advisors appointed by the President.

The Government of Union Territories Act, 1963 enacted by the Parliament in accordance with the
provisions of article 239A.

The Union Territory of Puducherry was placed under President’s Rule, three days after the V
Narayanasamy-led Congress government failed to prove its majority in a floor test in the Assembly.

President’s Rule implies the suspension of a state government and the imposition of direct rule of the
Centre. It is also known as ‘State Emergency’ or ‘Constitutional Emergency’.

60. A proclamation imposing President’s Rule must be approved by both the Houses of Parliament
within two months from the date of its issue.
The state governor, on behalf of the President, carries on the state administration with the help of the
chief secretary of the state or the advisors appointed by the President.

The Government of Union Territories Act, 1963 enacted by the Parliament in accordance with the
provisions of article 239A.

The Union Territory of Puducherry was placed under President’s Rule, three days after the V
Narayanasamy-led Congress government failed to prove its majority in a floor test in the Assembly.

President’s Rule implies the suspension of a state government and the imposition of direct rule of the
Centre. It is also known as ‘State Emergency’ or ‘Constitutional Emergency’.

61. A proclamation imposing President’s Rule must be approved by both the Houses of Parliament
within two months from the date of its issue.

The state governor, on behalf of the President, carries on the state administration with the help of the
chief secretary of the state or the advisors appointed by the President.

The Government of Union Territories Act, 1963 enacted by the Parliament in accordance with the
provisions of article 239A.

The Union Territory of Puducherry was placed under President’s Rule, three days after the V
Narayanasamy-led Congress government failed to prove its majority in a floor test in the Assembly.

President’s Rule implies the suspension of a state government and the imposition of direct rule of the
Centre. It is also known as ‘State Emergency’ or ‘Constitutional Emergency’.

62. A proclamation imposing President’s Rule must be approved by both the Houses of Parliament
within two months from the date of its issue.

The state governor, on behalf of the President, carries on the state administration with the help of the
chief secretary of the state or the advisors appointed by the President.

The Government of Union Territories Act, 1963 enacted by the Parliament in accordance with the
provisions of article 239A.

The Union Territory of Puducherry was placed under President’s Rule, three days after the V
Narayanasamy-led Congress government failed to prove its majority in a floor test in the Assembly.

President’s Rule implies the suspension of a state government and the imposition of direct rule of the
Centre. It is also known as ‘State Emergency’ or ‘Constitutional Emergency’.

63. A proclamation imposing President’s Rule must be approved by both the Houses of Parliament
within two months from the date of its issue.
The state governor, on behalf of the President, carries on the state administration with the help of the
chief secretary of the state or the advisors appointed by the President.

The Government of Union Territories Act, 1963 enacted by the Parliament in accordance with the
provisions of article 239A.

The Union Territory of Puducherry was placed under President’s Rule, three days after the V
Narayanasamy-led Congress government failed to prove its majority in a floor test in the Assembly.

President’s Rule implies the suspension of a state government and the imposition of direct rule of the
Centre. It is also known as ‘State Emergency’ or ‘Constitutional Emergency’.

64. Only A is true as the legislation is towards all tech giants including Google. Therefore, A is the correct
answer.

Iceland takes the crown as the country with the highest percentage of news consumed online (93
percent) through digital devices such as mobiles, online, on social media or news apps.

Google, meanwhile, had already been trying to get ahead of the new legislation by announcing
partnerships with media organizations in Australia, including Seven and Rupert Murdoch''s News Corp

The amended legislation - the News Media Bargaining Code - was passed by Australia''s House of
Representatives on Thursday, after earlier going through the Senate.

Facebook said after those revisions were made that the new agreement would allow it to "support the
publishers we choose to." It later revealed a deal with major Australian news company Seven West
Media, with plans to sign more with other publishers.

Facebook said it has struck a deal with Australian government and would once again allow users and
news publishers in the country to share and post links to news articles. Last week, Facebook had banned
all users and news publishers based in Australia from posting links to news on the platform. The ban also
impacted pages of Australian hospitals, charities, government organisations who found their pages had
been wiped clean.

Source – Indian Express

65. Only A is true as the legislation is towards all tech giants including Google. Therefore, A is the correct
answer.

Iceland takes the crown as the country with the highest percentage of news consumed online (93
percent) through digital devices such as mobiles, online, on social media or news apps.

Google, meanwhile, had already been trying to get ahead of the new legislation by announcing
partnerships with media organizations in Australia, including Seven and Rupert Murdoch''s News Corp

The amended legislation - the News Media Bargaining Code - was passed by Australia''s House of
Representatives on Thursday, after earlier going through the Senate.
Facebook said after those revisions were made that the new agreement would allow it to "support the
publishers we choose to." It later revealed a deal with major Australian news company Seven West
Media, with plans to sign more with other publishers.

Facebook said it has struck a deal with Australian government and would once again allow users and
news publishers in the country to share and post links to news articles. Last week, Facebook had banned
all users and news publishers based in Australia from posting links to news on the platform. The ban also
impacted pages of Australian hospitals, charities, government organisations who found their pages had
been wiped clean.

Source – Indian Express

66. Only A is true as the legislation is towards all tech giants including Google. Therefore, A is the correct
answer.

Iceland takes the crown as the country with the highest percentage of news consumed online (93
percent) through digital devices such as mobiles, online, on social media or news apps.

Google, meanwhile, had already been trying to get ahead of the new legislation by announcing
partnerships with media organizations in Australia, including Seven and Rupert Murdoch''s News Corp

The amended legislation - the News Media Bargaining Code - was passed by Australia''s House of
Representatives on Thursday, after earlier going through the Senate.

Facebook said after those revisions were made that the new agreement would allow it to "support the
publishers we choose to." It later revealed a deal with major Australian news company Seven West
Media, with plans to sign more with other publishers.

Facebook said it has struck a deal with Australian government and would once again allow users and
news publishers in the country to share and post links to news articles. Last week, Facebook had banned
all users and news publishers based in Australia from posting links to news on the platform. The ban also
impacted pages of Australian hospitals, charities, government organisations who found their pages had
been wiped clean.

Source – Indian Express

67. Only A is true as the legislation is towards all tech giants including Google. Therefore, A is the correct
answer.

Iceland takes the crown as the country with the highest percentage of news consumed online (93
percent) through digital devices such as mobiles, online, on social media or news apps.

Google, meanwhile, had already been trying to get ahead of the new legislation by announcing
partnerships with media organizations in Australia, including Seven and Rupert Murdoch''s News Corp
The amended legislation - the News Media Bargaining Code - was passed by Australia''s House of
Representatives on Thursday, after earlier going through the Senate.

Facebook said after those revisions were made that the new agreement would allow it to "support the
publishers we choose to." It later revealed a deal with major Australian news company Seven West
Media, with plans to sign more with other publishers.

Facebook said it has struck a deal with Australian government and would once again allow users and
news publishers in the country to share and post links to news articles. Last week, Facebook had banned
all users and news publishers based in Australia from posting links to news on the platform. The ban also
impacted pages of Australian hospitals, charities, government organisations who found their pages had
been wiped clean.

Source – Indian Express

LEGAL REASONING

68. The correct answer is (c.. The Supreme Court in Arjun Pandit Rao v. Kailash Kushanrao held that a
Certificate is not necessary if the ''original document'' itself is produced as primary evidence. In the
present case, since the original chats in the phone of the deceased has been produced before the court,
therefore there is no need for a certificate as per section 65B.

69. The correct answer is (a.. Oral evidence about the contents of the documents is barred by the
Evidence Act by Section 92 of the Indian Evidence Act. In the present case, oral testimony regarding
contents of a ledger is inadmissible under section 92 of the Indian Evidence Act.

70. The correct answer is (d.. The Court in National Lawyers Campaign for Judicial Transparency and
Reforms v. Union of India held that a forwarded message, without its original source, cannot be
regarded as ''document'' under the Evidence Act. Therefore, in the present case, the forwarded message
would not act as a valid evidence for Prajjawal.

71. The correct answer is (b.. the owner of the electronic device shall prove that the concerned device,
on which the original information is first stored, is owned and/or operated by him/ her. In the present
case, Archana is the person who has produced the evidence and she is not the owner of the device
where the document was first stored. Therefore, the evidence cannot be admitted by the court in
absence of a certificate as required by section 65B.

72. The correct answer is (b.. As per Section 64 of the Indian Evidence Act, documents must be proved
by primary evidence except in the circumstances mentioned in Section 65. Screenshots are secondary
evidence but they can be admitted by the court if coupled with the certificate as per section 65B of the
Indian Evidence Act.
73. The correct answer is (b.. The court in State of West Bengal v. Mir Mohammad, ruled that when any
fact is especially within the knowledge of any person, the burden of proving that fact is upon him. In the
present case, since the prosecution has produced supporting evidence, the liability to furnish an
explanation lies on the defendant.

74. The correct answer is (c.. In Phula Singh v. State of H.P., the court held that the accused has a duty to
furnish an explanation regarding any incriminating material that has been produced against him. In the
present case, it is the duty of Guddu and Bablu to explain the incriminating evidence against them.
When they remained silent, the court rightly convicted them relying on the evidence before it.

75. The correct answer is (d.. The court in State of Maharashtra v. Ashok Chotelal Shukla held that even
though the respondent failed to explain incriminating circumstances against him, the other
circumstances do not coherently lead to his guilt. In the present case, although Hari remained silent
instead of explaining the circumstances, he cannot be held guilty in absence of any other evidence to
incriminate him.

76. The correct answer is (a.. Circumstances II, III and IV are not self-incrimination, as a person is not
being forced to be a witness against themselves. However, in circumstances I and II, the accused is being
compelled to confess to guilt against lesser sentence or threat of conviction.

77. The correct answer is (b.. The court in State of West Bengal v. Mir Mohammad, ruled that when any
fact is especially within the knowledge of any person, the burden of proving that fact is upon him. In the
present case, the explanation of the incriminating factors and circumstantial evidence were only within
the knowledge of Simran, so when she fails to explain them, the court can draw adverse inferences. This
along with the evidence present can be a ground for conviction.

78. Mr. X will be liable since he advocated and promoted performing black magic and evil practices
which is punishable under the act.

79. All three of them will be liable including Ms. Z because she propagated and advocated the black
magic practices followed by the Guru. Propagation is also punishable under the act.

80. He can made liable since he was propagating black magic and evil practices by being the brand
ambassador of such device. Propagation is also an offence under the act.

81. all three will be made liable including Sushma since she helped in procuring the lambs knowing the
nature of the yagna. The act done by her is Abetment of black magic which is also punishable under the
act.

82. He will not be liable of kidnapping since he merely took her to the café and had no intention to
kidnap her. Mere recovery of a missing minor from the custody of a stranger would not ipso facto
establish the offence of kidnapping.
83. They will not be charged for kidnapping since there was no direct or indirect incitement done by
Rohan and Lilly.

84. Vivek did not commit kidnapping since Rose is not a minor (she studies in an MBA college) and
because she is a major, her parents cannot restrict her from going on a trip.

85. The custody of the minor was given by the court to her mother, though her father did not entice her,
but he did take her from the legal custody of her mother without her permission.

86. Article 25 is not absolute and is restricted if against public order, morality and health. )

87. Because as per the Supreme Court’ view, clause 1 does not permit conversion rather propagation by
exposition of its tenets)

88. Since no ceremonial conversion has taken place, neither has Article 25 or Supreme Court has
analyzed such a situation. Also, Talpade has the freedom to choose his own religion and conscience.)

89. Refer to ‘the inclusion of such a term in the Indian Constitution would be rendered meaningless. The
mere right to propagate for the enlightenment of others would already be covered under the right to
free speech and expression under Article 19(1)(a. of the Indian Constitution.’)

90. The court can ask Asim to pay the compensation to Pihu under section 357A of the Cr.P.C because
the accused here in the case is identified and not unknown. Thus, court can ask for medical expenditure
of Pihu to be compensated without actually quantifying the amount.

91. The court cannot grant victim compensation in the case because it has no power to quantify the
amount of compensation under sec. 357A of Cr.P.C and cannot prejudge the guilt of the accused by
asking them to pay compensation. It has to be paid from state funds. Court has no power to assess the
injuries and ascertain the amount nor can direct the legal authorities for the same.

92. The court was correct in granting victim compensation to Munii and contention put forth by Mr.
Rajesh is invalid because court has recommendatory power under section 357A to ask DLSA to supervise
such paying of compensation but cannot quantify the amount and has not done it in the case above.

93. The court can grant the conditional bail as to marrying the girl but not as to paying compensation to
the victim as it amounts to prejudging the guilt of the accused person. Thus it is not under the power of
court to grant such condition of compensation in bail.

94. egal Service Authorities are being made to protect the interest of poor people and providing them
with free legal aid services and it is the duty of the state to protect the interests of its citizens and not
the ambit of court to decide or assess the compensation. Thus, it is only recommendatory power under
section 357A.

95. Sneha cannot claim the profits earned from extra margins as there is no consideration from Sneha to
Shaily for that extra profits. As agreed between them, Shaily has given Sneha all part of the profits as
decided but there has to be some consideration for that extra margin.

96. Dheeraj is liable for breach of contract as he has accepted the proposal of Manoj and agreed to give
him the apartment on rent for Rs. 5000 and is estopped from giving it to Mrinal.

97. Rahul is liable for breach of contract as there was a valid contract between him and Mr. Shahjad for
the concert and consideration is also there. But Rahul cannot unilaterally withdraw his consent from the
agreement to not perform and also not to refund the amount on non-performance.

98. Sushil can withdraw his consent unilaterally from the promise given to Priya as there was no valid
agreement between them. It was nowhere mentioned in the facts that Sushil agreed to Priya on her
proposal and it cannot be said to be a valid contract.

99. Arti can do so as none of the party has performed their part as decided in the agreement. The
decision is yet to be reached and then only they will be held to perform their parts to which they cannot
withdraw their consent unilaterally if anyone of the parties had performed their parts.

100. The correct answer is (b.. In the case of Karam Singh v. Hardayal Singh, it was held that there
should be an unlawful assembly with the object of committing violence or an assembly of five or more
persons likely to cause a disturbance of the public peace, in order to use force against them. In this case,
the crowd on the platform was not gathered with the object of committing any violence or to disturb
public peace.

101. The correct answer is (d.. In the case of Karam Singh v. Hardayal Singh, it was held that there
should be an unlawful assembly with the object of committing violence or an assembly of five or more
persons likely to cause a disturbance of the public peace. And such an assembly should be ordered to be
dissolved and, thirdly, the assembly should have refused to disperse despite such orders to disperse, in
order to use force against them.

102. The correct answer is (d.. Under Section 129 CrPC, any executive magistrate or officer in charge of a
police station or, in the absence of such officer in charge, any police officer, not below the rank of a sub-
inspector or any one of them can order the use of force to disperse an unlawful assembly.

103. The correct answer is (c.. Under Section 129 CrPC, any executive magistrate or officer in charge of a
police station or, in the absence of such officer in charge, any police officer, not below the rank of a sub-
inspector can order the use of force to disperse an unlawful assembly. If there is an executive magistrate
around, it is generally considered good practice to obtain her or his orders, but it is not stated anywhere
that it is mandatory.

104. The correct answer is (d.. In the case of Karam Singh v. Hardayal Singh, it was held that there are
three prerequisites to be fulfilled before any force can be used.
Firstly, there should be an unlawful assembly with the object of committing violence or an assembly of
five or more persons likely to cause a disturbance of the public peace. Secondly, such an assembly
should be ordered to be dissolved and, thirdly, the assembly should have refused to disperse despite
such orders to disperse. Here, Richa and her group have already dispersed on the orders of Inspector
Varun, so use of any force against them is illegal.

LOGICAL REASONING

105. The author has mentioned the hawa feature that is followed by people. Here people follow the
trends that are followed by people. Since some people moved out of the social media sites like Orkut,
Google+, MySpace etc. many people have moved out of them which ultimately led to the collapse of
these sites. Hence, option (a) is correct. Option (b) has no context whatsoever to the passage. The same
goes for option (c).

Hence, the answer is option (a).

106. The starting of Mastodon and the liberals moving away from Twitter to join the same has been
discredited by the author himself as anything but network effect. The moving away from WhatsApp to
join Signal is a consequence of Network effect. Since a few people moved to Signal due to changed
privacy policies of WhatsApp, it sort of started an exodus.

Thus, option (a) is the answer.

107. The author doesn’t say that the Hindi heartland of India is more prone to Network effects. He says
that the phenomenon in this region is known as ‘hawa’. Thus, option (a) is not the answer. Option (b) is
not correct as all mass-market social media platforms are made by tech-whiz and nerds. Social media
platforms formed for peddling some certain belief are not. The author says “This is why every new user
is of great value to any platform”. This is actually because every new user increases the value of the
platform for everyone on it, making it more likely for even more people to join.

Option (c) is the answer.

108. Signal has better encryption than WhatsApp, this is not a misconception and also, the author
doesn’t mention this as well. Thus option (a) is not the answer. The author merely mentions that “Some
time ago, liberals rebelling against Twitter wanted to shift to Mastodon, but the idea soon fizzled out”.
Thus option (b) is not a misconception. Option (c) is not a misconception, it’s a fact as stated by the
author. The author does mention that “A recent privacy policy update on WhatsApp doesn’t change
much, because it was already sharing your metadata with Facebook. Thanks to end-to-end encryption,
your content data is not shared. Or so we are supposed to believe.”

Thus, option (d) is the answer.

109. The author makes it clear that it is only in universal platforms that network effects are possible.
Hence, statement 1 is correct. Trump is starting his social media platform and TV channel for his like-
minded people. It is thus not universal, hence the author would believe that this would not be
successful. Hence, statement 2 is correct. The author says “Everyone you know is on Facebook, Twitter,
Instagram, and WhatsApp. Almost everyone.” Thus not everyone is on social media platforms. Hence,
statement 3 is correct.

Thus, option (D) is the answer.

110. Option A is just an example of the case. Option C is not what we are arguing in the passage. Option
D is the consequences of bullying. Option B is the suitable conclusion than can be drawn from the
passage.

111. Option A and D weakens the passage. Option B is out of context. Option C describes the reason and
consequences of bullying in children.

Hence C is the correct option.

112. The term “in fact” is an indicator of the premises and it gives evidence to support the passage.

113. Option A strengthen the passage. Option B is out of context comparison. Option C is an evidence
that supports the passage. Option D weakens the conclusion.

Hence D is the correct option

114. Option A and B are not what the passage is arguing for. Option C is the perfect linkage between
Premises and conclusion.
115. Option A can be inferred as the ice melts into water and then goes into the oceans, which in turn
rises the level of water in the ocean. Option B can also be inferred as melting of Ice cap as big as Rhode
Island which attracted the attention of the world towards the climate change must be a bad news.
Option C can also be inferred as the CO2 heats up the ozone layer.

Therefore, D is the correct answer.

116. The author is assuming that the heat which is trapped from the absorption of the CO2 is aiding in
the rise of temperature worldwide, which leads to the melting of the ice caps in turn increasing the
ocean level.
Therefore, C is the correct answer.

117. Option C is correct as if it is true that Ozone layer does not trap any CO2 (Picayune = having little
value or importance) then it does not absorb heat and in turn cannot be the reason for the rise of
climate change. Option B is irrelevant and option A is opposite of option C and strengthens the author’s
argument. Hence, C

118. Option B is the correct answer as the author in the whole passage is explaining how the excessive
release of CO2 into the atmosphere is leading to the global warming and he is deriving it after
concluding in the first line that excessive release of the CO2 is the reason for the Global Warming.
Hence, B

119. All the option given in the question are correct as all the option are true based on the information
given in the passage.

Hence, D

120. Option C explains the discrepancy very well. Not all the people are crossing the speed limit, but the
few of those who are doing it, are doing it at a very high margin. If only few people are driving at 100
and above, then the average speed increases. However, only few people are doing this, hence the total
number remains low.

121. Argument I is self-explanatory and is a strong argument. II does not show how and why money
matters will have a negative impact on the school going students. Thus, II is a weak argument.

Hence, A

122. If Abhishek has had six flops in a row, movie goers will be cautious of watching another movie
which stars the same actor. Hence C is the correct answer.

123. Option B is out of context. Option C is the result of the economic growth in nation. Option D does
not make sense. Option A is the best described conclusion in the first para from the above passage.

Hence, A

124. Option A strengthen the passage. Option C is comparison among nations. Option B is arguing
against the author’s point.

125. Option B and D is the same point and has nothing to do with the main point of the passage. Option
A is irrelevant. Option C describes the reason of economic development of a country.

Hence, C
126. Option A is opposite the given argument. Option B is strong but irrelevant. Arguer does not talk
about policies of a nation, hence C is wrong as well. Option D is the correct option.

Hence, D

127. The given statement provides the facts that directly support the conclusion.

Therefore, C is the correct answer.

128. The vertical North South streets are N, O, P, Q, R. From the basic information we have two relative
positions are available - one between O, P and N and the other between Q and R.

The horizontal east west streets are: S, T, U: V, W, X, Y. Of these seven streets the relative positioning is
given in 3 distinct part as shown here:

The solution is visible from the figure below. Option (b) is correct.
129. The vertical North South streets are N, O, P, Q, R. From the basic information we have two relative
positions are available - one between O, P and N and the other between Q and R.

The horizontal east west streets are: S, T, U: V, W, X, Y. Of these seven streets the relative positioning is
given in 3 distinct part as shown here:

If O is ½ km east of Q, then R and P would coincide. Option (d) makes two streets coincide.
130. The vertical North South streets are N, O, P, Q, R. From the basic information we have two relative
positions are available - one between O, P and N and the other between Q and R.

The horizontal east west streets are: S, T, U: V, W, X, Y. Of these seven streets the relative positioning is
given in 3 distinct part as shown here:

if R is placed between O and P. So, Q to P would be 1.25 km. Option (d) correct.

131. The vertical North South streets are N, O, P, Q, R. From the basic information we have two relative
positions are available - one between O, P and N and the other between Q and R.

The horizontal east west streets are: S, T, U: V, W, X, Y. Of these seven streets the relative positioning is
given in 3 distinct part as shown here:
If R is between O and P, then Q being 1 km to the west of R, would be more than 1 km to the west of P.
Option (b) is false.

132. The vertical North South streets are N, O, P, Q, R. From the basic information we have two relative
positions are available - one between O, P and N and the other between Q and R.

The horizontal east west streets are: S, T, U: V, W, X, Y. Of these seven streets the relative positioning is
given in 3 distinct part as shown here:

Y is 1.5 km north of X is correct in all cases as the figure between X, W and Y shows. Option (c) is correct.

133. F > B (50)> C > D (30) > E > A

F has highest money.


134. F > B (50)> C > D (30) > E > A

Either A or E have possibly 20 rupee.

QUANTITATIVE TECHNIQUES

135. Total average rainfall in all the years (from June to September ) = 51556=859.166

Average rainfall in August = 15406=256.66

256.66859.166=29.87 ≈30% Reqd % =

136. Reqd. % = 1901540×100=12.33 %

137. In the year 2006 → 300890×100=33.33

In the year 2007 → 250900×100 = 27.77

In the year 2008 → 255880×100 = 28.97

In the year 2009 → 90700×100 = 27.14

In the year 2010 → 265895×100 = 29.60

In the year 2011 → 280890×100 = 31.46

Hence , in the year 2006.

138. (d) is the correct answer.

139. In the year 2007 = 1060 ×100=16.66

2009 = 1068 ×100=14.70

2010 = 1278×100=15.38

Hence in the year 2007.

140. 100%increase for Mortein in Kolkata from 2003 to 2004, i.e. from 10% to 20%

141. In Chennai only Mortein decreased its market share by 10%.

142. Shares of all companies have either increased or decreased from 2003 to 2004 in some city.
143. 1 –only in Delhi.

144. 3 –Finit in Chennai, Hit in Chennai, Mortein in Delhi.

145. The production in 1991 = 1000 kg per acre. So total = 4000 kgs, so share 1800:2200 = 9 : 11

146. Production in 1993 = 6000 kgs and in 1994 = 8000 = kgs, so 2000 kgs increase.

147. Share in 1991 = 1800 and in 1995 = 9000, so ratio = 1800 : 9000 = 1 : 5

148. Production in 1995 = 12000 kgs and Sanjay’s share in 1994 = 4800 kgs, which is 40% of 12000.

149. 400012000 ×100=33.33%

150. 40006000 ×100=66.67%


Legal Edge 23 Mock

Answers & Details

ENGLISH

1. Ribeiro implores the police to be fair and launch an investigation without any bias or
selectiveness.
While all options are true, A, C and D are just the sub-themes of the views of Robeiro. The main
theme of the passage is Robeiro‟s disapproval of the bias that police has shown while investigating
the riots. Thus, A, B and C are not the most suited answers.
Thus, the correct answer is (b)
2. The following lines reflect the same:
“Riots recur in India because of the impunity accorded to one section by the political establishment
of the day.” Thus author here mentions that one section is always favored by the reigning political
party which affords them immunity.
A and B are not reflected in the passage anywhere. Thus, they are incorrect.
Thus, the correct answer is (c)
3. The following line indicates the same:
“Police must clearly distinguish between peaceful anti-CAA protesters and those who actively
instigate violence. The latter could fall on either side of the CAA-NRC political divide, and police
must strictly follow the evidence in this regard rather than taking sides themselves.”
The author says that only peaceful protests can be allowed, not all kinds of protests- as mentioned in
(a) thus, A is incorrect.
Option B is nowhere mentioned in the passage. Thus, B is incorrect.
(c) The author nowhere suggest who should be arrested- Pro CAA or anti CAA protestors. He just
says that those protestors should be arrested who incite violence. Thus, C is incorrect.
Thus, the correct answer is (d)
4. The following line reflects the same:
“This democratic right must be permitted, while those actively instigating violence must be hauled
up.”
The author calls the right to protest a democratic right, thereby indicating that this is granted to us in
furtherance of democracy. While options A, C and D are true, they do not form the basis of right to
protest as per the passage. They themselves are elements of democracy. Thus, the correct answer
is (b)
5. The author‟s political affiliation is not reflected in the passage, which means A, B or C is not the
correct answer. He is just stating facts as have been said by Robeiro. Thus, the correct answer is (d)
His unbiased attitude is also reflected in the following line:
“The latter could fall on either side of the CAA-NRC political divide, and police must strictly follow the
evidence in this regard rather than taking sides themselves.”
Thus, the correct answer is (d)
6. Expedite means to speed up. Thus, the correct answer is (b)

7. The following lines indicate the same:


“Additional Sessions Judge Amitabh Rawat discharging Babu and Imran for the attempted murder
charge, but accepting prima facie evidence to charge them with rioting and similar offences”
While the proceedings are in full swing, they have not been convicted yet. Further, all the options are
mutually exclusive, i.e, since C is true, none of the other can be true. Thus, the correct answer is (c)
8. The following line indicates the same:
“The innocent until proven guilty dictum is rarely subscribed to, evident in jails packed with undertrial
prisoners.” This line shows that innocent until proven is the norm. No other options, i.e. B,C,D are
implied in the passage anywhere. Thus, the correct answer is (a)
9. While A, C, D are true, the main anomaly which led the court to discharge the accused of murder
charges was the lack of evidence. The judge‟s decision was based on that and that alone.
Thus, the correct answer is (c)
10. Allegory: a story, poem, or picture that can be interpreted to reveal a hidden meaning, typically a
moral or political one.
Contrast: an author uses contrast when they describe the difference between two or more entities.
Juxtaposition: the fact of two things being seen or placed close together with contrasting effect.
Irony: the expression of one''s meaning by using language that normally signifies the opposite,
typically for humorous or emphatic effect.
The fact that the alleged victimless crime has now made the two accused victims in a manner is
ironic, i.e. happening in a way contrary to what is expected, and typically causing wry amusement
because of this. Thus, the correct answer is (d)
11. Specific question
Options A and B are reflected in the following line: “BMC‟s demolition of alleged illegal alterations in
Kangana Ranaut‟s building at incredibly short notice indicates political motivation.”
Option C is reflected in the following line:
“…while proceeding to demolish without informing the court, betrays contempt for judiciary and lack
of due process for citizens”
All of these actions combined give the impression that the demolition was politically motivated, due
process was not followed. Thus, the correct answer is (d)
12. Specific question
It is not reflected anywhere that Riya has the backing of a political party. Thus, A is incorrect.
If anything, the reverse of B is true. Riya went through a media trial as per the passage, but
Kangana did not. Thus, B is incorrect.
C is nowhere reflected in the passage, thus, is incorrect.
D is reflected in the following line:
“Rhea‟s plight is in many ways even worse – she‟s a non-political citizen in jail, while Kangana‟s
baiting of Shiv Sena has BJP‟s support, evident from quick deployment of elite Y+ category security.”
Thus, the correct answer is (d)
13. Vocab-in-Context question
Caveat means a warning or proviso of specific stipulations, conditions, or limitations in general
parlance. Thus, the correct answer is (a)
14. Specific question
The following line reflects the same:
“A top NCB official has admitted this wasn‟t a case – one with no drug recoveries from Rhea – the
agency normally dealt with.”
No other option is reflected in the passage. Thus, the correct answer is (b)
15. Explanation: Inference question
The following lines reflect the same:
“Rhea‟s plight is in many ways even worse – she‟s a non-political citizen in jail, while Kangana‟s
baiting of Shiv Sena has BJP‟s support, evident from quick deployment of elite Y+ category security.”
None of the other options are reflected in the passage. Thus, the correct answer is (a)
16. Tone question
While the Author does not explicitly share his views regarding the rejection of PIL, a clear reading of
the passage gives us the idea that he is against sedition laws and thus, implicitly is in favor of the
rejection. He even goes on to implore the SC to junk sedition laws.
Further, option C, D are nowhere reflected in the passage. Thus, the correct answer is (a)
17. Specific question
Option A: the author implores the court to check the constitutionality of the laws, thereby implying
that they are constitutional presently. Thus, Option A is incorrect
Option B: The author says that nowadays even dissenters are being booked under the sedition
charges, even those which are not genuine cases of sedition. Thus, option B is incorrect.
Option C: The following line negates option C:
“The expression of a view which is a dissent from a decision taken by the Central Government itself
cannot be said to be seditious.”
Thus, the correct answer is (d)
18. Specific question
A: while the author implores the court to check the constitutionality of sedition laws, it is at present
not unconstitutional. Thus, A is incorrect
(b) the author says that both the laws leave a wide berth for misinterpretation, i.e. they are
ambiguous.
C: the passage says that section 66A has been squashed, while there is no surety about the
squashing of sedition laws. Thus, section B is incorrect.
Thus, the correct answer is (b)
19. Application question
A: archaic means very old or old-fashioned. The author nowhere says that the law is old fashioned.
Thus, A is incorrect
(b) unconstitutional means a legislation that goes against the basic tenets of the constitution. While
the author implores the courts to declare the law unconstitutional, it is not so today. Thus, B is
incorrect
(c) Draconian means overly severe, wherein the punishments prescribed are disproportionate to the
crime. This is implied nowhere in the passage. Thus, C is incorrect.
(d) ambiguous means unclear. The author says that sedition is a law leaving a wide berth for
misinterpretation, which implies that it is unclear. Thus, the correct answer is (d)
20. Universal question
A: this is an implied in the passage, but is not the underlying theme.
B: this, too, is mentioned in the passage but is not significantly focused upon to be the theme of the
passage.
(c) the author talks about the consequences of sedition law, and throughout the passage implores
the court to rethink about the constitutionality of the sedition law at present.
(d) this is nowhere mentioned in the passage.
Thus, the correct answer is (c)
21. Paternalism is the interference of a state or an individual with another person, against their will,
and defended or motivated by a claim that the person interfered with will be better off or protected
from harm. The act in question is an example of state paternalism and thus, the correct answer is (a)

22. The following lines indicate the same:


“Centre‟s affidavit rejected a fundamental right to same sex marriage, noting its incompatibility with
the “Indian family unit concept of a husband, a wife and children, which necessarily presuppose a
biological man as „husband‟, a biological woman as „wife‟ and the children born out of the union
between the two.” The heterosexual marital union described here is a limited interpretation of family.”
Thus, the correct answer is (d)
23. The following lines indicate the same:
“The legal challenge against Section 377 IPC was preceded by a long struggle to humanise the
plight of homosexual persons and reverse the colonisation of the Indian mind by Victorian morality,
which effaced the much older Indian tradition of acceptance of gender and sexual diversity. Centre‟s
affidavit won‟t be the last chapter of this epic struggle.”
The author by “Victorian morality” here means the influence of British on Indian laws, a product of
which is the regressive laws relating to homosexuality in India. Thus, the correct answer is (b)
24. A clear reading of the passage leads us to the answer. The author compares India to more
“progressive” nations, calls the current laws an influence of Victorian Morality, and calls out the
limited definition of family as decided by the courts. Thus, the correct answer is (d)

25. Only option A hints at the patriarchy prevalent in the country. Thus, the correct answer is (a)

26. Option C best describes the theme of the passage and the author‟s views, thus, C is the correct
answer.

27. Option C cannot be inferred from the passage. Thus, the correct answer is (c)
28. To censure is to express severe disapproval of (someone or something). Thus, the correct
answer is (b)

29. The following line indicates the same:


“While modernity is making men and women engage on technically equal terms on the outside, that
transition needs to happen on the mental plane as well.”
Thus, the Answer is (a).

GENERAL KNOWLEDGE

30. Rationale: The National Council for Transgender Persons (NCTP) is the statutory body of the
Government of India, generally concerned with advising the government on all policy matters
affecting transgender persons. It was established in 2020 under the provisions of the Transgender
Persons (Protection of Rights) Act, 2019. In the Board there will be five members of the transgender
community.
As of October 2020, NCTP is lead by the Minister of Social Justice and Empowerment, Thawar
Chand Gehlot. The council is composed of five representatives of the transgender community, one
each from five different regions: the north, south, east, west and northeast. Additionally, several Joint
secretary-level ex-officio members from various governmental ministries serve on the council as well
as five expert members from nongovernmental organisations.
31. Rationale: The National Council for Transgender Persons (NCTP) is the statutory body of the
Government of India, generally concerned with advising the government on all policy matters
affecting transgender persons. It was established in 2020 under the provisions of the Transgender
Persons (Protection of Rights) Act, 2019. In the Board there will be five members of the transgender
community.
As of October 2020, NCTP is lead by the Minister of Social Justice and Empowerment, Thawar
Chand Gehlot. The council is composed of five representatives of the transgender community, one
each from five different regions: the north, south, east, west and northeast. Additionally, several Joint
secretary-level ex-officio members from various governmental ministries serve on the council as well
as five expert members from nongovernmental organisations.
32. Rationale: The National Council for Transgender Persons (NCTP) is the statutory body of the
Government of India, generally concerned with advising the government on all policy matters
affecting transgender persons. It was established in 2020 under the provisions of the Transgender
Persons (Protection of Rights) Act, 2019. In the Board there will be five members of the transgender
community.
As of October 2020, NCTP is lead by the Minister of Social Justice and Empowerment, Thawar
Chand Gehlot. The council is composed of five representatives of the transgender community, one
each from five different regions: the north, south, east, west and northeast. Additionally, several Joint
secretary-level ex-officio members from various governmental ministries serve on the council as well
as five expert members from nongovernmental organisations.
33. Rationale: The National Council for Transgender Persons (NCTP) is the statutory body of the
Government of India, generally concerned with advising the government on all policy matters
affecting transgender persons. It was established in 2020 under the provisions of the Transgender
Persons (Protection of Rights) Act, 2019. In the Board there will be five members of the transgender
community.
As of October 2020, NCTP is lead by the Minister of Social Justice and Empowerment, Thawar
Chand Gehlot. The council is composed of five representatives of the transgender community, one
each from five different regions: the north, south, east, west and northeast. Additionally, several Joint
secretary-level ex-officio members from various governmental ministries serve on the council as well
as five expert members from nongovernmental organisations.
34. Rationale: The National Council for Transgender Persons (NCTP) is the statutory body of the
Government of India, generally concerned with advising the government on all policy matters
affecting transgender persons. It was established in 2020 under the provisions of the Transgender
Persons (Protection of Rights) Act, 2019. In the Board there will be five members of the transgender
community.
As of October 2020, NCTP is lead by the Minister of Social Justice and Empowerment, Thawar
Chand Gehlot. The council is composed of five representatives of the transgender community, one
each from five different regions: the north, south, east, west and northeast. Additionally, several Joint
secretary-level ex-officio members from various governmental ministries serve on the council as well
as five expert members from nongovernmental organisations.
35. Vajiralongkorn, reigning title Phrabat Somdet Phra Vajira Klao Chao Yu Hua, is the King of
Thailand since 2016. He is the only son of King Bhumibol Adulyadej and Queen Sirikit. In 1972, at
the age of 20, he was made crown prince by his father.
Prayut Chan-o-cha is a Thai politician, retired Royal Thai Army general officer and former leader of
the National Council for Peace and Order, the military junta which governed Thailand between 22
May 2014 and 10 July 2019. As of August 2019 he serves as the prime minister and defence
minister.
Panusaya “Roong” Sithijirawattanakul is one of the leading protestors of the United Front of
Thammasat and Demonstration. The Bad Students Movement is something else. She shot to fame
in the democratic protests‟ realm, when she read out the Ten Demands, where she highlighted the
plight of the Thais and suggested 10 demands that would help the people. This was the first time in
Thailand‟s history, that such a thing was done.
The Crown Property Bureau (CPB) is the quasi-government agency responsible for managing the
property of the Monarchy of Thailand. The bureau is legally defined as a juristic person and not a
government agency. It has no tax obligations. It manages the finances of the Crown and has been
taken by the King under his control. The removal of the CPB from the Crown‟s control is one of the
key demands.
The country held the delayed Parliamentary election in 2019, which was seen as a mere exercise to
transfer power from the junta to an elected government.
36. Vajiralongkorn, reigning title Phrabat Somdet Phra Vajira Klao Chao Yu Hua, is the King of
Thailand since 2016. He is the only son of King Bhumibol Adulyadej and Queen Sirikit. In 1972, at
the age of 20, he was made crown prince by his father.
Prayut Chan-o-cha is a Thai politician, retired Royal Thai Army general officer and former leader of
the National Council for Peace and Order, the military junta which governed Thailand between 22
May 2014 and 10 July 2019. As of August 2019 he serves as the prime minister and defence
minister.
Panusaya “Roong” Sithijirawattanakul is one of the leading protestors of the United Front of
Thammasat and Demonstration. The Bad Students Movement is something else. She shot to fame
in the democratic protests‟ realm, when she read out the Ten Demands, where she highlighted the
plight of the Thais and suggested 10 demands that would help the people. This was the first time in
Thailand‟s history, that such a thing was done.
The Crown Property Bureau (CPB) is the quasi-government agency responsible for managing the
property of the Monarchy of Thailand. The bureau is legally defined as a juristic person and not a
government agency. It has no tax obligations. It manages the finances of the Crown and has been
taken by the King under his control. The removal of the CPB from the Crown‟s control is one of the
key demands.
The country held the delayed Parliamentary election in 2019, which was seen as a mere exercise to
transfer power from the junta to an elected government.
37. Vajiralongkorn, reigning title Phrabat Somdet Phra Vajira Klao Chao Yu Hua, is the King of
Thailand since 2016. He is the only son of King Bhumibol Adulyadej and Queen Sirikit. In 1972, at
the age of 20, he was made crown prince by his father.
Prayut Chan-o-cha is a Thai politician, retired Royal Thai Army general officer and former leader of
the National Council for Peace and Order, the military junta which governed Thailand between 22
May 2014 and 10 July 2019. As of August 2019 he serves as the prime minister and defence
minister.
Panusaya “Roong” Sithijirawattanakul is one of the leading protestors of the United Front of
Thammasat and Demonstration. The Bad Students Movement is something else. She shot to fame
in the democratic protests‟ realm, when she read out the Ten Demands, where she highlighted the
plight of the Thais and suggested 10 demands that would help the people. This was the first time in
Thailand‟s history, that such a thing was done.
The Crown Property Bureau (CPB) is the quasi-government agency responsible for managing the
property of the Monarchy of Thailand. The bureau is legally defined as a juristic person and not a
government agency. It has no tax obligations. It manages the finances of the Crown and has been
taken by the King under his control. The removal of the CPB from the Crown‟s control is one of the
key demands.
The country held the delayed Parliamentary election in 2019, which was seen as a mere exercise to
transfer power from the junta to an elected government.
38. Vajiralongkorn, reigning title Phrabat Somdet Phra Vajira Klao Chao Yu Hua, is the King of
Thailand since 2016. He is the only son of King Bhumibol Adulyadej and Queen Sirikit. In 1972, at
the age of 20, he was made crown prince by his father.
Prayut Chan-o-cha is a Thai politician, retired Royal Thai Army general officer and former leader of
the National Council for Peace and Order, the military junta which governed Thailand between 22
May 2014 and 10 July 2019. As of August 2019 he serves as the prime minister and defence
minister.
Panusaya “Roong” Sithijirawattanakul is one of the leading protestors of the United Front of
Thammasat and Demonstration. The Bad Students Movement is something else. She shot to fame
in the democratic protests‟ realm, when she read out the Ten Demands, where she highlighted the
plight of the Thais and suggested 10 demands that would help the people. This was the first time in
Thailand‟s history, that such a thing was done.
The Crown Property Bureau (CPB) is the quasi-government agency responsible for managing the
property of the Monarchy of Thailand. The bureau is legally defined as a juristic person and not a
government agency. It has no tax obligations. It manages the finances of the Crown and has been
taken by the King under his control. The removal of the CPB from the Crown‟s control is one of the
key demands.
The country held the delayed Parliamentary election in 2019, which was seen as a mere exercise to
transfer power from the junta to an elected government.
39. Vajiralongkorn, reigning title Phrabat Somdet Phra Vajira Klao Chao Yu Hua, is the King of
Thailand since 2016. He is the only son of King Bhumibol Adulyadej and Queen Sirikit. In 1972, at
the age of 20, he was made crown prince by his father.
Prayut Chan-o-cha is a Thai politician, retired Royal Thai Army general officer and former leader of
the National Council for Peace and Order, the military junta which governed Thailand between 22
May 2014 and 10 July 2019. As of August 2019 he serves as the prime minister and defence
minister.
Panusaya “Roong” Sithijirawattanakul is one of the leading protestors of the United Front of
Thammasat and Demonstration. The Bad Students Movement is something else. She shot to fame
in the democratic protests‟ realm, when she read out the Ten Demands, where she highlighted the
plight of the Thais and suggested 10 demands that would help the people. This was the first time in
Thailand‟s history, that such a thing was done.
The Crown Property Bureau (CPB) is the quasi-government agency responsible for managing the
property of the Monarchy of Thailand. The bureau is legally defined as a juristic person and not a
government agency. It has no tax obligations. It manages the finances of the Crown and has been
taken by the King under his control. The removal of the CPB from the Crown‟s control is one of the
key demands.
The country held the delayed Parliamentary election in 2019, which was seen as a mere exercise to
transfer power from the junta to an elected government.
40. The Sri Lankan Government has passed the 20th Amendment. It envisages expansive powers
and greater immunity for the Executive President.
The Amendment rolls back Sri Lanka‟s 19th Amendment, a 2015 legislation that sought to clip
presidential powers, while strengthening Parliament. The new legislation in turn reduces the Prime
Minister‟s role to a ceremonial one. It mainly sought to weaken the power of the presidency which
the 18th amendment had greatly expanded.
The Indo-Sri Lanka Peace Accord was an accord signed in Colombo on 29 July 1987, between
Indian Prime Minister Rajiv Gandhi and Sri Lankan President J. R. Jayewardene.
Recently, the Reserve Bank of India (RBI) has signed an agreement for extending a USD 400 million
currency swap facility to Sri Lanka to boost the foreign reserves and ensure financial stability of the
country, which is badly hit by Covid-19 pandemic.
India and Sri Lanka conduct joint Military (Mitra Shakti) and Naval (SLINEX) exercises.
41. The Sri Lankan Government has passed the 20th Amendment. It envisages expansive powers
and greater immunity for the Executive President.
The Amendment rolls back Sri Lanka‟s 19th Amendment, a 2015 legislation that sought to clip
presidential powers, while strengthening Parliament. The new legislation in turn reduces the Prime
Minister‟s role to a ceremonial one. It mainly sought to weaken the power of the presidency which
the 18th amendment had greatly expanded.
The Indo-Sri Lanka Peace Accord was an accord signed in Colombo on 29 July 1987, between
Indian Prime Minister Rajiv Gandhi and Sri Lankan President J. R. Jayewardene.
Recently, the Reserve Bank of India (RBI) has signed an agreement for extending a USD 400 million
currency swap facility to Sri Lanka to boost the foreign reserves and ensure financial stability of the
country, which is badly hit by Covid-19 pandemic.
India and Sri Lanka conduct joint Military (Mitra Shakti) and Naval (SLINEX) exercises.
42. The Sri Lankan Government has passed the 20th Amendment. It envisages expansive powers
and greater immunity for the Executive President.
The Amendment rolls back Sri Lanka‟s 19th Amendment, a 2015 legislation that sought to clip
presidential powers, while strengthening Parliament. The new legislation in turn reduces the Prime
Minister‟s role to a ceremonial one. It mainly sought to weaken the power of the presidency which
the 18th amendment had greatly expanded.
The Indo-Sri Lanka Peace Accord was an accord signed in Colombo on 29 July 1987, between
Indian Prime Minister Rajiv Gandhi and Sri Lankan President J. R. Jayewardene.
Recently, the Reserve Bank of India (RBI) has signed an agreement for extending a USD 400 million
currency swap facility to Sri Lanka to boost the foreign reserves and ensure financial stability of the
country, which is badly hit by Covid-19 pandemic.
India and Sri Lanka conduct joint Military (Mitra Shakti) and Naval (SLINEX) exercises.
43. The Sri Lankan Government has passed the 20th Amendment. It envisages expansive powers
and greater immunity for the Executive President.
The Amendment rolls back Sri Lanka‟s 19th Amendment, a 2015 legislation that sought to clip
presidential powers, while strengthening Parliament. The new legislation in turn reduces the Prime
Minister‟s role to a ceremonial one. It mainly sought to weaken the power of the presidency which
the 18th amendment had greatly expanded.
The Indo-Sri Lanka Peace Accord was an accord signed in Colombo on 29 July 1987, between
Indian Prime Minister Rajiv Gandhi and Sri Lankan President J. R. Jayewardene.
Recently, the Reserve Bank of India (RBI) has signed an agreement for extending a USD 400 million
currency swap facility to Sri Lanka to boost the foreign reserves and ensure financial stability of the
country, which is badly hit by Covid-19 pandemic.
India and Sri Lanka conduct joint Military (Mitra Shakti) and Naval (SLINEX) exercises.
44. The Sri Lankan Government has passed the 20th Amendment. It envisages expansive powers
and greater immunity for the Executive President.
The Amendment rolls back Sri Lanka‟s 19th Amendment, a 2015 legislation that sought to clip
presidential powers, while strengthening Parliament. The new legislation in turn reduces the Prime
Minister‟s role to a ceremonial one. It mainly sought to weaken the power of the presidency which
the 18th amendment had greatly expanded.
The Indo-Sri Lanka Peace Accord was an accord signed in Colombo on 29 July 1987, between
Indian Prime Minister Rajiv Gandhi and Sri Lankan President J. R. Jayewardene.
Recently, the Reserve Bank of India (RBI) has signed an agreement for extending a USD 400 million
currency swap facility to Sri Lanka to boost the foreign reserves and ensure financial stability of the
country, which is badly hit by Covid-19 pandemic.
India and Sri Lanka conduct joint Military (Mitra Shakti) and Naval (SLINEX) exercises.
45. The tunnel is 9.02 km in length.
Atal tunnel is the world‟s longest highway tunnel above 10,000 ft. This 9.02 km long tunnel connects
Manali to Lahaul & Spiti Valley throughout the year. This all-weather tunnel will reduce the distance
between Manali & Leh by 46 kms. It is also the country‟s first tunnel to have an escape tunnel within
the main tunnel because of the topography.
It connects Solang Valley near Manali to Sissu in Lahaul and Spiti district. Thus, we can say that it
connects Manali to Leh.
Atal Tunnel has been designed for traffic density of 3000 cars per day and 1,500 trucks per day with
max speed of 80 km/hr. It has the state of the art electromechanical system including semi
transverse ventilation system.
The Chang La is a high mountain pass in Ladakh at an elevation of 5,360 m or 17,590 ft in the
Indian Kashmir between India and Pakistan.
46. The tunnel is 9.02 km in length.
Atal tunnel is the world‟s longest highway tunnel above 10,000 ft. This 9.02 km long tunnel connects
Manali to Lahaul & Spiti Valley throughout the year. This all-weather tunnel will reduce the distance
between Manali & Leh by 46 kms. It is also the country‟s first tunnel to have an escape tunnel within
the main tunnel because of the topography.
It connects Solang Valley near Manali to Sissu in Lahaul and Spiti district. Thus, we can say that it
connects Manali to Leh.
Atal Tunnel has been designed for traffic density of 3000 cars per day and 1,500 trucks per day with
max speed of 80 km/hr. It has the state of the art electromechanical system including semi
transverse ventilation system.
The Chang La is a high mountain pass in Ladakh at an elevation of 5,360 m or 17,590 ft in the
Indian Kashmir between India and Pakistan.
47. The tunnel is 9.02 km in length.
Atal tunnel is the world‟s longest highway tunnel above 10,000 ft. This 9.02 km long tunnel connects
Manali to Lahaul & Spiti Valley throughout the year. This all-weather tunnel will reduce the distance
between Manali & Leh by 46 kms. It is also the country‟s first tunnel to have an escape tunnel within
the main tunnel because of the topography.
It connects Solang Valley near Manali to Sissu in Lahaul and Spiti district. Thus, we can say that it
connects Manali to Leh.
Atal Tunnel has been designed for traffic density of 3000 cars per day and 1,500 trucks per day with
max speed of 80 km/hr. It has the state of the art electromechanical system including semi
transverse ventilation system.
The Chang La is a high mountain pass in Ladakh at an elevation of 5,360 m or 17,590 ft in the
Indian Kashmir between India and Pakistan.
48. The tunnel is 9.02 km in length.
Atal tunnel is the world‟s longest highway tunnel above 10,000 ft. This 9.02 km long tunnel connects
Manali to Lahaul & Spiti Valley throughout the year. This all-weather tunnel will reduce the distance
between Manali & Leh by 46 kms. It is also the country‟s first tunnel to have an escape tunnel within
the main tunnel because of the topography.
It connects Solang Valley near Manali to Sissu in Lahaul and Spiti district. Thus, we can say that it
connects Manali to Leh.
Atal Tunnel has been designed for traffic density of 3000 cars per day and 1,500 trucks per day with
max speed of 80 km/hr. It has the state of the art electromechanical system including semi
transverse ventilation system.
The Chang La is a high mountain pass in Ladakh at an elevation of 5,360 m or 17,590 ft in the
Indian Kashmir between India and Pakistan.
49. The tunnel is 9.02 km in length.
Atal tunnel is the world‟s longest highway tunnel above 10,000 ft. This 9.02 km long tunnel connects
Manali to Lahaul & Spiti Valley throughout the year. This all-weather tunnel will reduce the distance
between Manali & Leh by 46 kms. It is also the country‟s first tunnel to have an escape tunnel within
the main tunnel because of the topography.
It connects Solang Valley near Manali to Sissu in Lahaul and Spiti district. Thus, we can say that it
connects Manali to Leh.
Atal Tunnel has been designed for traffic density of 3000 cars per day and 1,500 trucks per day with
max speed of 80 km/hr. It has the state of the art electromechanical system including semi
transverse ventilation system.
The Chang La is a high mountain pass in Ladakh at an elevation of 5,360 m or 17,590 ft in the
Indian Kashmir between India and Pakistan.
50. India has been ranked at 94 among 107 countries in the Global Hunger Index (GHI) 2020.

51. Jointly published by Concern Worldwide and Welthungerhilfe.


It was first produced in 2006. It is published every October. The 2020 edition marks the 15th edition
of the GHI.
52. In Global Hunger Index (GHI) Report-2019, India was ranked at 102nd position out of 117
countries. India‟s rank had slipped from 95th position (in 2010) to 102nd (in 2019). Over a longer-
term duration, the fall in India‟s rank is sharper, i.e, from 83rd out of 113 countries in 2000 to 102nd
out of 117 in 2019.

53. Seventeen nations, including China, Belarus, Ukraine, Turkey, Cuba and Kuwait, shared the top
rank with GHI scores of less than five.

54. In 2015 the global community adopted the 17 Global Goals for Sustainable Development to
improve people''s lives by 2030.
55. India has signed the Basic Exchange and Cooperation Agreement (BECA) with the USA.BECA
will enable the exchange of geospatial data and information between the two countries and will
improve the accuracy of India‟s missiles in precision strikes.
India holds such talks with Japan and Australia, at the foreign secretary and defense secretary
level. India holds ministerial-level talks only with the USA. Apart from India, the United States
holds such ministerial dialogues with Australia and Japan also. This is an invention of the Japanese.
BECA will enable the exchange of geospatial data and information between the two countries and
will improve the accuracy of India‟s missiles in precision strikes.The geospatial maps and charts to
be shared, will be acquired from multiple sources like satellites, unmanned aerial vehicles (UAVs),
reconnaissance aircraft, aerostats among others.As part of measures to enhance military to military
cooperation, Liaison Officers at each other''s establishments could be leveraged to enhance
information sharing.
BECA is an important precursor to
India acquiring armed unmanned aerial vehicles such as the Predator-B from the USA.
Predator-B uses spatial data for accurate strikes on enemy targets.
Both countries released a Joint Statement on shared Indo-USA goals in the Asia-Pacific region and
emphasised that the Code of Conduct in the South China Sea should not prejudice the legitimate
rights and interests of any nation in accordance with international law.They also decided to expand
joint capacity building activities with partner countries in the Indo-Pacific region.
56. India has signed the Basic Exchange and Cooperation Agreement (BECA) with the USA.BECA
will enable the exchange of geospatial data and information between the two countries and will
improve the accuracy of India‟s missiles in precision strikes.
India holds such talks with Japan and Australia, at the foreign secretary and defense secretary
level. India holds ministerial-level talks only with the USA. Apart from India, the United States
holds such ministerial dialogues with Australia and Japan also. This is an invention of the Japanese.
BECA will enable the exchange of geospatial data and information between the two countries and
will improve the accuracy of India‟s missiles in precision strikes.The geospatial maps and charts to
be shared, will be acquired from multiple sources like satellites, unmanned aerial vehicles (UAVs),
reconnaissance aircraft, aerostats among others.As part of measures to enhance military to military
cooperation, Liaison Officers at each other''s establishments could be leveraged to enhance
information sharing.
BECA is an important precursor to
India acquiring armed unmanned aerial vehicles such as the Predator-B from the USA.
Predator-B uses spatial data for accurate strikes on enemy targets.
Both countries released a Joint Statement on shared Indo-USA goals in the Asia-Pacific region and
emphasised that the Code of Conduct in the South China Sea should not prejudice the legitimate
rights and interests of any nation in accordance with international law.They also decided to expand
joint capacity building activities with partner countries in the Indo-Pacific region.
57. India has signed the Basic Exchange and Cooperation Agreement (BECA) with the USA.BECA
will enable the exchange of geospatial data and information between the two countries and will
improve the accuracy of India‟s missiles in precision strikes.
India holds such talks with Japan and Australia, at the foreign secretary and defense secretary
level. India holds ministerial-level talks only with the USA. Apart from India, the United States
holds such ministerial dialogues with Australia and Japan also. This is an invention of the Japanese.
BECA will enable the exchange of geospatial data and information between the two countries and
will improve the accuracy of India‟s missiles in precision strikes.The geospatial maps and charts to
be shared, will be acquired from multiple sources like satellites, unmanned aerial vehicles (UAVs),
reconnaissance aircraft, aerostats among others.As part of measures to enhance military to military
cooperation, Liaison Officers at each other''s establishments could be leveraged to enhance
information sharing.
BECA is an important precursor to
India acquiring armed unmanned aerial vehicles such as the Predator-B from the USA.
Predator-B uses spatial data for accurate strikes on enemy targets.
Both countries released a Joint Statement on shared Indo-USA goals in the Asia-Pacific region and
emphasised that the Code of Conduct in the South China Sea should not prejudice the legitimate
rights and interests of any nation in accordance with international law.They also decided to expand
joint capacity building activities with partner countries in the Indo-Pacific region.
58. India has signed the Basic Exchange and Cooperation Agreement (BECA) with the USA.BECA
will enable the exchange of geospatial data and information between the two countries and will
improve the accuracy of India‟s missiles in precision strikes.
India holds such talks with Japan and Australia, at the foreign secretary and defense secretary
level. India holds ministerial-level talks only with the USA. Apart from India, the United States
holds such ministerial dialogues with Australia and Japan also. This is an invention of the Japanese.
BECA will enable the exchange of geospatial data and information between the two countries and
will improve the accuracy of India‟s missiles in precision strikes.The geospatial maps and charts to
be shared, will be acquired from multiple sources like satellites, unmanned aerial vehicles (UAVs),
reconnaissance aircraft, aerostats among others.As part of measures to enhance military to military
cooperation, Liaison Officers at each other''s establishments could be leveraged to enhance
information sharing.
BECA is an important precursor to
India acquiring armed unmanned aerial vehicles such as the Predator-B from the USA.
Predator-B uses spatial data for accurate strikes on enemy targets.
Both countries released a Joint Statement on shared Indo-USA goals in the Asia-Pacific region and
emphasised that the Code of Conduct in the South China Sea should not prejudice the legitimate
rights and interests of any nation in accordance with international law.They also decided to expand
joint capacity building activities with partner countries in the Indo-Pacific region.
59. India has signed the Basic Exchange and Cooperation Agreement (BECA) with the USA.BECA
will enable the exchange of geospatial data and information between the two countries and will
improve the accuracy of India‟s missiles in precision strikes.
India holds such talks with Japan and Australia, at the foreign secretary and defense secretary
level. India holds ministerial-level talks only with the USA. Apart from India, the United States
holds such ministerial dialogues with Australia and Japan also. This is an invention of the Japanese.
BECA will enable the exchange of geospatial data and information between the two countries and
will improve the accuracy of India‟s missiles in precision strikes.The geospatial maps and charts to
be shared, will be acquired from multiple sources like satellites, unmanned aerial vehicles (UAVs),
reconnaissance aircraft, aerostats among others.As part of measures to enhance military to military
cooperation, Liaison Officers at each other''s establishments could be leveraged to enhance
information sharing.
BECA is an important precursor to
India acquiring armed unmanned aerial vehicles such as the Predator-B from the USA.
Predator-B uses spatial data for accurate strikes on enemy targets.
Both countries released a Joint Statement on shared Indo-USA goals in the Asia-Pacific region and
emphasised that the Code of Conduct in the South China Sea should not prejudice the legitimate
rights and interests of any nation in accordance with international law.They also decided to expand
joint capacity building activities with partner countries in the Indo-Pacific region.
60. The Sudan deal comes weeks after similar moves by the UAE & Bahrain. The two Gulf States
became the first in the Middle East to recognise Israel in 26 years.
The Abraham Accord between Israel, the United Arab Emirates and Bahrain is mediated by the
USA. It is the first Arab-Israeli peace deal in 26 years. There were the only two peace deals between
Israel and the Arab States in more than a quarter of a century. Egypt was the first Arab State to sign
a peace deal with Israel in 1979.Jordan signed a peace pact in 1994.
In South Asia, it will put Pakistan in a bind, whether to follow UAE‟s steps (will be seen as giving up
Islamic cause of Palestine) or not to follow the UAE (since it is already in feud with the Saudis over
not taking up the Kashmir case, Pakistan cannot afford another hostile Islamic Country). Shia-Sunni
rifts in the region may get wide and violent.Saudi Arabia (Sunni) and Iran (representing Shia) have a
long history of enmity. For decades, one of the main sources of instability in West Asia has been the
cold war between Saudi Arabia and Iran.The Sunni-Shiite schism may also provoke violence
between Muslims in such places as Pakistan, Nigeria and Indonesia.
Hamas is a Palestinian Sunni-Islamic fundamentalist, militant and nationalist organization. It has a
social service wing, Dawah, and a military wing, the Izz ad-Din al-Qassam Brigades.
61. The Sudan deal comes weeks after similar moves by the UAE & Bahrain. The two Gulf States
became the first in the Middle East to recognise Israel in 26 years.
The Abraham Accord between Israel, the United Arab Emirates and Bahrain is mediated by the
USA. It is the first Arab-Israeli peace deal in 26 years. There were the only two peace deals between
Israel and the Arab States in more than a quarter of a century. Egypt was the first Arab State to sign
a peace deal with Israel in 1979.Jordan signed a peace pact in 1994.
In South Asia, it will put Pakistan in a bind, whether to follow UAE‟s steps (will be seen as giving up
Islamic cause of Palestine) or not to follow the UAE (since it is already in feud with the Saudis over
not taking up the Kashmir case, Pakistan cannot afford another hostile Islamic Country). Shia-Sunni
rifts in the region may get wide and violent.Saudi Arabia (Sunni) and Iran (representing Shia) have a
long history of enmity. For decades, one of the main sources of instability in West Asia has been the
cold war between Saudi Arabia and Iran.The Sunni-Shiite schism may also provoke violence
between Muslims in such places as Pakistan, Nigeria and Indonesia.
Hamas is a Palestinian Sunni-Islamic fundamentalist, militant and nationalist organization. It has a
social service wing, Dawah, and a military wing, the Izz ad-Din al-Qassam Brigades.
62. The Sudan deal comes weeks after similar moves by the UAE & Bahrain. The two Gulf States
became the first in the Middle East to recognise Israel in 26 years.
The Abraham Accord between Israel, the United Arab Emirates and Bahrain is mediated by the
USA. It is the first Arab-Israeli peace deal in 26 years. There were the only two peace deals between
Israel and the Arab States in more than a quarter of a century. Egypt was the first Arab State to sign
a peace deal with Israel in 1979.Jordan signed a peace pact in 1994.
In South Asia, it will put Pakistan in a bind, whether to follow UAE‟s steps (will be seen as giving up
Islamic cause of Palestine) or not to follow the UAE (since it is already in feud with the Saudis over
not taking up the Kashmir case, Pakistan cannot afford another hostile Islamic Country). Shia-Sunni
rifts in the region may get wide and violent.Saudi Arabia (Sunni) and Iran (representing Shia) have a
long history of enmity. For decades, one of the main sources of instability in West Asia has been the
cold war between Saudi Arabia and Iran.The Sunni-Shiite schism may also provoke violence
between Muslims in such places as Pakistan, Nigeria and Indonesia.
Hamas is a Palestinian Sunni-Islamic fundamentalist, militant and nationalist organization. It has a
social service wing, Dawah, and a military wing, the Izz ad-Din al-Qassam Brigades.
63. The Sudan deal comes weeks after similar moves by the UAE & Bahrain. The two Gulf States
became the first in the Middle East to recognise Israel in 26 years.
The Abraham Accord between Israel, the United Arab Emirates and Bahrain is mediated by the
USA. It is the first Arab-Israeli peace deal in 26 years. There were the only two peace deals between
Israel and the Arab States in more than a quarter of a century. Egypt was the first Arab State to sign
a peace deal with Israel in 1979.Jordan signed a peace pact in 1994.
In South Asia, it will put Pakistan in a bind, whether to follow UAE‟s steps (will be seen as giving up
Islamic cause of Palestine) or not to follow the UAE (since it is already in feud with the Saudis over
not taking up the Kashmir case, Pakistan cannot afford another hostile Islamic Country). Shia-Sunni
rifts in the region may get wide and violent.Saudi Arabia (Sunni) and Iran (representing Shia) have a
long history of enmity. For decades, one of the main sources of instability in West Asia has been the
cold war between Saudi Arabia and Iran.The Sunni-Shiite schism may also provoke violence
between Muslims in such places as Pakistan, Nigeria and Indonesia.
Hamas is a Palestinian Sunni-Islamic fundamentalist, militant and nationalist organization. It has a
social service wing, Dawah, and a military wing, the Izz ad-Din al-Qassam Brigades.
64. Aung San Suu Kyi is a Myanmar‟s State Counsellor and Nobel Peace laureate (awarded in
1990).
The Kaladan project connects Sittwe Port in Myanmar to the India-Myanmar border.The project was
jointly initiated by India and Myanmar to create a multi-modal platform for cargo shipments from the
eastern ports to Myanmar and to the North-eastern parts of the country through Myanmar.
India supports the process of democratic transition in Myanmar.Though India has expressed deep
concern over recent developments in Myanmar, cutting off from the Myanmar military is not a viable
option as India has significant economic and strategic interests in Myanmar and its neighbourhood.
o A one-year state of emergency has been imposed and democratically elected leader Aung San
Suu Kyi has been detained.
o When the newly elected Myanmar lawmakers were to hold the first session of Parliament in 2021,
the military imposed a state of emergency for one year citing massive voting fraud in the
parliamentary elections.
o In the Myanmars‟ Parliament, the military holds 25% of the total seats according to the 2008
military-drafted constitution and several key ministerial positions are also reserved for military
appointees.
Min Aung Hlaing is a Burmese army general who has served as Chairman of the State
Administration Council of Myanmar since 2 February 2021, a position which makes him the
country''s de facto leader. He has also been the commander-in-chief of Defence Services since
March 2011.
65. Aung San Suu Kyi is a Myanmar‟s State Counsellor and Nobel Peace laureate (awarded in
1990).
The Kaladan project connects Sittwe Port in Myanmar to the India-Myanmar border.The project was
jointly initiated by India and Myanmar to create a multi-modal platform for cargo shipments from the
eastern ports to Myanmar and to the North-eastern parts of the country through Myanmar.
India supports the process of democratic transition in Myanmar.Though India has expressed deep
concern over recent developments in Myanmar, cutting off from the Myanmar military is not a viable
option as India has significant economic and strategic interests in Myanmar and its neighbourhood.
o A one-year state of emergency has been imposed and democratically elected leader Aung San
Suu Kyi has been detained.
o When the newly elected Myanmar lawmakers were to hold the first session of Parliament in 2021,
the military imposed a state of emergency for one year citing massive voting fraud in the
parliamentary elections.
o In the Myanmars‟ Parliament, the military holds 25% of the total seats according to the 2008
military-drafted constitution and several key ministerial positions are also reserved for military
appointees.
Min Aung Hlaing is a Burmese army general who has served as Chairman of the State
Administration Council of Myanmar since 2 February 2021, a position which makes him the
country''s de facto leader. He has also been the commander-in-chief of Defence Services since
March 2011.
66. Aung San Suu Kyi is a Myanmar‟s State Counsellor and Nobel Peace laureate (awarded in
1990).
The Kaladan project connects Sittwe Port in Myanmar to the India-Myanmar border.The project was
jointly initiated by India and Myanmar to create a multi-modal platform for cargo shipments from the
eastern ports to Myanmar and to the North-eastern parts of the country through Myanmar.
India supports the process of democratic transition in Myanmar.Though India has expressed deep
concern over recent developments in Myanmar, cutting off from the Myanmar military is not a viable
option as India has significant economic and strategic interests in Myanmar and its neighbourhood.
o A one-year state of emergency has been imposed and democratically elected leader Aung San
Suu Kyi has been detained.
o When the newly elected Myanmar lawmakers were to hold the first session of Parliament in 2021,
the military imposed a state of emergency for one year citing massive voting fraud in the
parliamentary elections.
o In the Myanmars‟ Parliament, the military holds 25% of the total seats according to the 2008
military-drafted constitution and several key ministerial positions are also reserved for military
appointees.
Min Aung Hlaing is a Burmese army general who has served as Chairman of the State
Administration Council of Myanmar since 2 February 2021, a position which makes him the
country''s de facto leader. He has also been the commander-in-chief of Defence Services since
March 2011.
67. Aung San Suu Kyi is a Myanmar‟s State Counsellor and Nobel Peace laureate (awarded in
1990).
The Kaladan project connects Sittwe Port in Myanmar to the India-Myanmar border.The project was
jointly initiated by India and Myanmar to create a multi-modal platform for cargo shipments from the
eastern ports to Myanmar and to the North-eastern parts of the country through Myanmar.
India supports the process of democratic transition in Myanmar.Though India has expressed deep
concern over recent developments in Myanmar, cutting off from the Myanmar military is not a viable
option as India has significant economic and strategic interests in Myanmar and its neighbourhood.
o A one-year state of emergency has been imposed and democratically elected leader Aung San
Suu Kyi has been detained.
o When the newly elected Myanmar lawmakers were to hold the first session of Parliament in 2021,
the military imposed a state of emergency for one year citing massive voting fraud in the
parliamentary elections.
o In the Myanmars‟ Parliament, the military holds 25% of the total seats according to the 2008
military-drafted constitution and several key ministerial positions are also reserved for military
appointees.
Min Aung Hlaing is a Burmese army general who has served as Chairman of the State
Administration Council of Myanmar since 2 February 2021, a position which makes him the
country''s de facto leader. He has also been the commander-in-chief of Defence Services since
March 2011.
68. Aung San Suu Kyi is a Myanmar‟s State Counsellor and Nobel Peace laureate (awarded in
1990).
The Kaladan project connects Sittwe Port in Myanmar to the India-Myanmar border.The project was
jointly initiated by India and Myanmar to create a multi-modal platform for cargo shipments from the
eastern ports to Myanmar and to the North-eastern parts of the country through Myanmar.
India supports the process of democratic transition in Myanmar.Though India has expressed deep
concern over recent developments in Myanmar, cutting off from the Myanmar military is not a viable
option as India has significant economic and strategic interests in Myanmar and its neighbourhood.
o A one-year state of emergency has been imposed and democratically elected leader Aung San
Suu Kyi has been detained.
o When the newly elected Myanmar lawmakers were to hold the first session of Parliament in 2021,
the military imposed a state of emergency for one year citing massive voting fraud in the
parliamentary elections.
o In the Myanmars‟ Parliament, the military holds 25% of the total seats according to the 2008
military-drafted constitution and several key ministerial positions are also reserved for military
appointees.
Min Aung Hlaing is a Burmese army general who has served as Chairman of the State
Administration Council of Myanmar since 2 February 2021, a position which makes him the
country''s de facto leader. He has also been the commander-in-chief of Defence Services since
March 2011.

LEGAL REASONING

69. Mr. Purohit can be held liable for the negligence as he should take reasonable care and due dilig
ence to ask Shushma of her being claustrophobic and diabetic before putting her to tests. This can b
e expected out of Mr. Purohit as Shushma was having breathing problems and it’s reasonable to ask
her for such personal information.

70. Dr. Malti can be held liable for medical negligence as she has to take reasonable care and due di
ligence before reading the reports and prescribing medicines to the child. It is expected out of her be
cause she is professionally competent for the position.

71. Rahul cannot be held liable for medical negligence as he was not a competent professional indivi
dual with requisite knowledge and skills to perform such medical help to injured persons. He is yet to
become a graduate and hence require practical field exxperirence before said to be competent prof
essionally.

72. Abhinav cannot be held liable for medical negligence as facts nowhere mentioned about his prof
essional competency and thus no reasonable care or due diligence can be expected out of him for N
irmala in prescribing homeopathic medicines.

73. Dr. Suman cannot be held liable for medical negligence as she has performed the surgery well a
nd is professionally competent with requisite knowledge and skills. It was mere mistake on her part n
ot to stitch the organ and not gross negligence.

74. (Reason: Akash had the immediate reasonable care towards his patient. Refer to the definition
and essential elements required to constitute tortious liability of negligence. )

75. Option (A) is correct answer. As Akash is negligent because he doesn't take reasonable
care and not show reasonable duty toward the patient, as this is essential element in negligence.
76. (Reason: A was within the speed limit and not driving rashly inspite of the clear road. Also, he
paid attention to driving and to reasonable care to apply the brakes. Thus, option B. A, C and D are
vague)

77. (Reason: The act was such that it can be considered gross negligence. Section 304A of IPC
provisions that any act that causes death by negligence is a crime.)

78. (Reason: The clearly does not deprive the accused of a trial. The first paragraph mentions “The
first half bars a legislature from enacting a law that names and convicts someone of a crime without
a trial.” Options (c) and (d) are not relevant for the question)

79. (Reason: “The second half forbids a legislature from retroactively applying a statute defining a
new crime or enhancing the penalty for an old one”)

80. (Reason: This will invoke the first half of the Bill of Attainder and Ex Post Facto Clause)

81. (Reason: This is with “enhancing the penalty” for an old crimes. Therefore reducing penalty will
not be a matter of concern)

82. The passage states that the state cannot discriminate on the basis of place of birth. The very fact
that the Sarpanch had gotten the wells constructed is enough to show that the state owned the
same. The refusal to let them draw water is in contravention of Article 15.

83. The passage clearly states that private wells do not belong to the state. Thus, in this case, the
fact that Suraj Rathi has a private well gives him enough right to decide who will draw water out of i.

84. The passage clearly states that the right available under Article 15 are only available to citizens
and not to foreigners. Thus, they have no recourse and Option C is correct.

85. This is merely a special provision for the betterment of women under Article 15. The same
cannot be challenged before a court of law.

86. The passage states that equality must be in equal circumstances. Equality in equal
circumstances is the fundamental governing principle here. Thus, Option C is correct.

87. The passage states that there has to be reasonable classification. It does not state on the
number of entities in such classification. Thus, such classification even if done for 1 entity should be
valid as long as the same is reasonable.

88. The absence of dishonest intention is one of the most important elements according to the
passage above. Thus, in the present case, it cannot be stated that Gangotri had dishonest intention
in his mind at the time of taking the book.

89. The passage above states that for theft to occur, the book has to be in some person‟s
possession and that the same has to be taken away from that person‟s possession. The book was
not in any person‟s possession at any time. Thus, Aakriti is not liable.

90. The passage states that a property must be moved from someone‟s possession dishonestly for
theft to occur. In this case, it cannot be stated that the property was in Menon‟s possession when it
was kept in the cabinet. The theft was completed the moment the book was moved from that specific
place.
91. The passage states that the property must be in the possession of the owner. In this case,
however the possession of Menon is when the property is at the place where she had kept it. Thus,
taking the property out of the place is enough to constitute theft.

92. The passage states that courts are always on the lookout for free consent. In this case, even if
Menon knew that Gangotri is a trickster and was cautious with him, there is nothing to prove that
there was consent in this specific instance. Thus, theft has occurred.

93. The passage states that provisions pertaining to theft leading to death must be construed
reasonably and the intent of the thief must be kept in mind. It cannot be reasonably stated that theft
of a book would lead to someone‟s death. Thus, Gangotri is not liable.

94. The passage does not present the full picture and thus we‟ll have to think within it‟s limits. Per
the passage crime may not just be committed by human beings but also by corporation. The scope
of a person does not extend to anything beyond the species of human beings in the passage.

95. The passage states that the definition of person includes companies within itself. It cannot be
said that the different forms of companies can be used to evade liability from crime. Partnership
Firms are equally persons for a crime as they are also companies.

96. The passage states that the intention to kill is important in a crime. In this case, C only had the
intention to kill a deer and not a man. It cannot be thus said that C has committed a crime.

97. The passage above mentions the various standards of intent. Knowledge is one such standard.
The applicability of the same however only depends upon the provision A is being made liable under
and thus D is the correct option.

98. The passage above mentions that omission to act on one‟s duty is sufficient actus reus to
constitute a crime. In this case, even if B had consented to his death, it cannot be said that A was
absolved of his duty of treating B for there is nothing known about euthanasia in the passage. Thus,
A is liable.

99. The passage above states that an injury is necessary for a crime to be caused. In this case,
irrespective of whether Undertaker resurrects or not, there is injury in the nature of 1 life of his being
taken away by Brett Lee. Thus, he has caused a crime.

100. Apple Inc is a well know brand and defendant launching a company with a similar name is
causing deception. Also option (b) is not correct as there is no mention of any information related to
the compensation.

101. Though Forever 21 is a well-known trademark but one can easily distinguish between Forever
21 and Ever 100. So it will not amount to trademark violation.

102. “, that is to say that the said term ISKCON did not exist prior to the Plaintiff''s adoption and use
of the same. Since it is a coined trademark which is associated exclusively with the Plaintiff, it
undoubtedly deserves the highest degree of protection.” Here, the case is completely different. Term
“Magic Masala” is used by Lays, chips brand and many other in general terms therefore one
particular company should not be granted a trademark right against such marks.

103. Referring to the above passage “(a) the Plaintiff''s trademark ISKCON has wide acceptability;
(b) the popularity of the Plaintiff‟s trademark ISKCON extends not only in India but in other countries
as well;” The present case of Dabur qualifies as well known and well acceptable brand across the
country and outside. Whereas the plaintiff is a new brand having recognition at local level so Dabur‟s
mark is not violating Plaintiff‟s trademark.

104. Here Bata Pvt. Ltd. used already existing Bata trademark and tried to make good money out of
someone else‟s brand reputation.

105. Price at which the trademark disputed products are sold is not relevant to decide the trademark
violation cases.

LOGICAL REASONING

106. Options (C) & (D) are incorrect as per the references in the passage. Option (B) is also
incorrect as not all state control may result in the above-mentioned tactic.
Therefore, the correct answer is Option (A).
107. Rationale – Options (A) or (C) cannot be affirmed or denied on the basis of the information
provided above. Option (D) may be correct but is not useful in the discussion posed by the author.
Therefore, the correct option is (C).
108. Rationale – The author emphasizes on the fact that the Thai government has put restrictions on
its citizens that are hampering and unreasonable. In light of this, all Option (A), (C) & (B) do not
undermine the argument posed by the author and the premise still stands.
Therefore, the correct option is Option (D).
109. Rationale – Option (D) is the correct answer is the statement is a supposition or proposed
explanation made on the basis of limited evidence as a starting point for further investigation.

110. Rationale – Only Option (A) covers the theme of the discussion and is suited to the content in
the passage. Option B and D are too general and don‟t cover the key idea of cyber control in
Thailand.
Option C talks about cyberspace, but not about Thailand.
Hence, A
111. (A) Notice that the argument is ambiguous. Exclusivity is said to be a large component of a
potential buyer''s mindset, but not the only one.
(B) This option mentions the actions of galleries without any supporting data.
(C) This choice assumes that exclusivity is the only factor, which it is not.
(D) Correct. This option states that the knowledge of a large number of paintings being sold "may"
hamper the artist''s chances of selling more paintings. This directly follows from the argument stated
above.
112. Rationale – Options (A), (C) & (D) are incorrect as they have no bearing on the premise stated
by the author in the passage. Option (B), on the other hand, nullifies the solution proposed by the
author.
Therefore, the correct answer is Option (B).
113. Rationale – The author states a pressing threat to all small island states however, if the others
are prepared to combat rising sea water, then the concern is not placed well and therefore, the
correct answer is Option (A).

114. Rationale – Only Option (B) cannot be traced to the information mentioned in the passage. Rest
all the other ones can be found in the text of the passage.

115. Rationale – As per the author, the solution proposed is the upcoming JV‟s between big
countries and island states which will lead to heavy investment and development in the same.
Therefore, a great probability lies in the statement mentioned coming true.

116. Rationale – Only Option (D) covers the theme of the discussion and is suited to the content in
the passage.

117. Carrot and stick (Maxim): Which means: A persuasive strategy that utilizes a prize and
discipline framework to support improved execution or conduct. Model: The executives hung the
carrot of a potential raise before strikers, and yet waved the stick of losing their annuity benefits.
Subsequently, alternative (a) is the right answer.
118. The leader held out carrots galore, succulent ones. Better pay rates and administration
conditions for government workers and J&K police officers, a quick whirlwind of occupations as the
public authority moved to fill all opening, new grants and instructive freedoms, new rights as
undeniable residents of India and new openings that Indian ventures, State-claimed and private,
would make. The featured part shows that separated from maternity leave, any remaining
advantages have been referenced in the passage.
Unmistakably, alternative „a‟ is the right answer.
119. The most striking component of Head administrator Narendra Modi's address to the country on
the choice to scrap Articles 370 and 35A was its fleeting focus: WHAT’S TO COME. This served to
both fill the story with expectation and SKIRT THE MUDDLED POLITICAL CONFLICT GOING
BEFORE and going with the choice to eliminate Jammu and Kashmir's extraordinary status. The
featured part approves the data given in articulations I and II. In this manner, proclamations II and III
are right. Notwithstanding, the location never really energize further discussion with respect to the
upsides and downsides of rejecting the articles once the choice had been made. Thus, articulation III
is off base.
Plainly, alternative (d) is the right answer.
120. In the event that a pioneer holds out a carrot, a stick can't yet be essential for the bundle. Be
that as it may, if the PM employed one, it was not in plain view. He alluded to dissidence and dread,
however certainly regarded them as simply knocks along the way to standardization and get back to
statehood. The author accepts that the Head administrator was utilizing a carrot and stick strategy
however was not showing the stick straightforwardly. He just brushed over the dangers that were
approaching by referencing them momentarily. Out of the given choices just choice c can be an
admirable statement which can be gathered from the passage.
Choice c is consequently the right answer.
121. Statement I: There was a lockdown in Jammu and Kashmir. Refer to: The PM did well to report
unwinding, as right on time as next Monday, when Eid would commence, on the lockdown set up in
the state. The featured piece of the passage approves explanation I to be right.
Statement II: Advantages given to the J&K occupants have taken the spotlight from the expense.
Refer to: The definite rundown of advantages and legitimate privileges that individuals of Jammu and
Kashmir presently remain to acquire, represented as they would be by the Constitution and overall
set of laws of India, is probably going to have the ideal effect of moving the focus on the advantages
of the political change, from the costs that have been generally self-evident. The featured piece of
the passage approves explanation II to be right.
Statement III: India is not, at this point under any danger from Pakistani terrorists. Pakistani
psychological oppressors have not been referenced in the passage. Consequently, proclamation III
is invalid.
Thus, choice b is the right answer.

122.

123.

124.

125.
126.

127.

128.
129. All are true
A>D>G
C>E>H
D>B>F
G>C
F>G
So, final order of height is as following
A>D>B>F>G>C>E>H
130. A > D > G
C>E>H
D>B>F
G>C
F>G
So, final order of height is as following
A>D>B>F>G>C>E>H
J must be placed between G and E with C but it is not clear that between C and J, who is taller.
From the above arrangement J is shorter than D.
131. A > D > G
C>E>H
D>B>F
G>C
F>G
So, final order of height is as following
A>D>B>F>G>C>E>H
B will be the third form top when they are arranged in descending order of height.
132. Clearly, vowels, A, E, I, O, U are coded as 1, 2, 3, 4, 5 respectively. Each of the consonants in
the word is moved one step forward to give the corresponding letter of the code. So, the code for
PEARL becomes Q21SM.

133. The sequence is accab/accab/accab. Thus, the pattern „accab‟ is repeated.

134. Logic: 4, (4×2) + 1, (9×3) -1, (26×4) +1, (105×5) -1, (524×6) + 1 = 3145
QUANTITATIVE TECHNIQUES

135. Number of horns produces= 800000/32×3=75000

136. Percentage of tubes = (28-25)/25×100 = 12

137. Ratio of number of horns produced at Nagpur and Pune = 3/7

138. From the graph it is clear that option (a) is our answer.

139. Total students in 1990-91 = 1350


And number of students in science = 400
So, 4001350×100 =29.6%
140. Total student in 1991-92
= 550 + 500 + 250 + 200 = 1500
Commerce student is 250
So, Total student is = 1500250
= 6 times of commerce student
141. percent increase in science student from
1990 – 91 to 1992 – 93 = 600-400400×100 = 50
142. Total no. of mobiles of model A + B + E in 2007
= (30 + 15 + 10) × 35 100 = 55 × 35100 = 19.25 lakh
143. 2007 2008
For „A‟ 30% 40% = 1030×100 = 3313 %
For „B‟ 15% 20% = 515×100 = 3313 %
For „C‟ 20% 15% = 520×100 = 25%
For „D‟ 15% 10% = 515 × 100 = 3313%
For „E‟ 10% 10% =
And for „F‟ 10% 5% = 510×100 = 50%
144. Production of B in 2007 = 15100×35 = 5.25 Lakh.
And Production of B in 2008 = 20100×44
= 8.8 Lakh
So, difference = 355000
145. According to question
Number of „A‟ type mobiles in 2008
= 44 × 30100 Lakh = 1320000
146. required number of unsold mobile (D type)
2007 and 2008 = 35 × 15100×15100+44×10100× 15100
= .7875 lakh + .66 lakh = 1.4475 lakh or 144750
147. Time taken by T1 to fill 20% = 10 minutes Hence, T1 fills 100% = 50 minutes
Time taken by T2 to fill 10% = 15 minutes Hence, T2 fills 100% = 150 minutes
Time taken by Empty tap to empty 75% = 60 min. Hence, empty Tap empties 100% = 80 minutes
Let total capacity of the tank = LCM (50, 150, 80) = 1200 units
Capacity of T1 = 1200/50 = 24 units
Capacity of T2 = 1200/150 = 8 units
Capacity of empty tap = 1200/80 = 15 units
Filled tank in 1 minute = 24 + 8 – 15 = 17 units.
Filled tank in 14 minutes = 14 × 17 = 238 units.
Rest units = 1200 – 238 = 962 units.
Capacity of T1 + T2 = 24 + 8 = 32 units.
Time taken by T1 and T2 = 962/32 = 30.0625 minutes
148. T3 taken time to fill 15% = 45 minutes Hence, T3 fills 100% = 300 minutes
T4 takes time to fill 30% = 30 minutes Hence, T4 fills 100% = 100 minutes
Empty tap empties 100% = 50 minutes
Let total capacity of the tank = LCM (300, 100, 50) = 300 units
Capacity of T3 = 300/300 = 1 units
Capacity of T4 = 300/100 = 3 units
Capacity of empty tap = 300/50 = 6 units
Tank filled in 10 minutes = 10 × 1 + 10 × 3 = 40 units
Work by T3 + T4 + Empty = 1 + 3 – 6 = -2 units
Tank emptied in = 40/2 = 20 minutes
149. Time taken by T1 to fill 20% = 10 minutes Hence, T1 fills 100% = 50 minutes
Time taken by T5 to fill 25 % = 35 minutes Hence, T5 fills 100% = 140 minutes
Let empty tap empties 100% in y minutes
Let total capacity of tank = LCM (50, 140) = 700 units
Capacity of T1 = 700/50 = 14 units
Capacity of T5 = 700/140 = 5 units
Capacity of empty tap = 700/y units
Tank filled in 1 minute = (19 – 700)/y unit
Tank filled in 50 minutes = {50 × (19 – 700)} / y unit
y = 140 minutes
150. Time taken by T1 to fill 20% = 10 minutes Hence, T1 fills 100% = 50 minutes
Time taken by T2 to fill 10% = 15 minutes Hence, T2 fills 100% = 150 minutes
T3 taken time to fill 15% = 45 minutes Hence, T3 fills 100% = 300 minutes
T4 takes time to fill 30% = 30 minutes Hence, T4 fills 100% = 100 minutes
Time taken by T5 to fill 25 % = 35 minutes Hence, T5 fills 100% = 140 minutes
1/50 + 1/150 + 1/300 + 1/100 + 1/140
= (42+14+7+21+ 15)2100 = 99//2100 = 33/700
Time = 700/33 = 21.21 min
Legal Edge 24 Mock

Answers & Details

ENGLISH

1. The following line indicates the same:


“The announcement of dates for Bengal, Tamil Nadu, Kerala, Assam and Puducherry assembly
elections has focused attention on the strong repercussions their outcome will have for national
politics.”
The first line of the passage has set the tone of the passage and the whole passage henceforth is
just the author justifying his view. Thus, the correct answer is A.

2. the following line indicates the same:


“BJP governs Assam, is a strong challenger in Bengal, and its ally AIADMK administers TN.”
A correct reading of the line gives us the answer that BJP governs only Assam. Thus, the correct
answer is D.

3. The following lines indicate the same:


“Any setback for BJP in Bengal and Assam and ally AIADMK in TN could bring the opposition
roaring back into contention in national politics. Consequently, BJP will face greater questioning over
farm policy, high fuel prices and attempts to pursue economic reforms like privatisation.”
Thus, the correct answer is C.

4. The following line indicates the same:


“Much like 2019, PM Modi remains BJP‟s most saleable face in Bengal. But an assembly election
may dim Modi‟s traction, requiring local leaders to up their game.”

5. The following line indicates the same:


“the CAA has forced BJP into a delicate balancing act”. This line implies that it was the BJP
government responsible for this policy.
Option C is untrue, as the author says that the delay in notifying the rules will lead to TMC calling it a
gimmick, but will not necessarily lead to their victory. Thus, the correct answer is A.

6. Option A is the essence of the passage and the subject matter of the passage. Thus, the correct
answer is A.

7. The following line indicates the same:


“News content already undertakes compliance with various standalone legislations. Plus news sites
follow print and TV norms, besides the extensive self-regulation done in multiple layers between
journalists and editors every day.”
Thus, the correct answer is A.

8. The author contrasts social media with news sites, saying that the former has more accountability,
while the latter has almost none. Thus, the best suited word for it is “rogue”.

9. The following line indicates the same:


“Government‟s aim to impose accountability on social media to tackle a pandemic of fake news and
hatred is sound.” This shows the aim of the government behind the new rules. Thus, the correct
answer is B.
10. The correct answer is option c). The entire passage is dominantly based on statistical
information on international migrants before and after the pandemic.

11. The correct answer is option a). The meaning of the word remittance is „a sum of money sent in
payment or as a gift.‟

12. The correct answer is option b). Statement 2 is the correct statement as the last paragraph
mentions that, „Women migrants, especially domestic workers, are particularly at risk of
discrimination, violence, abuse and exploitation.‟ Statement 1 is not true as third paragraph mentions
that, „But since COVID-19, remittances have declined drastically. Those to Eastern Europe and
Central Asia declined over 16% from $57 billion in 2019 to $48 billion in 2020.‟

13. The correct answer is option c). The passage clearly states that, „The Asia-Pacific Migration
Report 2020 shows that voluntary and involuntary causes drive migration between countries in Asia
and the Pacific and in other regions of the world. The primary reason is temporary labour migration.‟

14. The correct option is c). the correct meaning of the word involuntary means „done against
someone‟s will; compulsory.‟

15. The correct answer is option b). Statement 2 is the correct answer as it has been mentioned in
the second paragraph of the passage.

16. The correct answer is option a). The meaning of the word „hegemony‟ is „leadership or
dominance, especially by one state or social group over others‟.

17. The correct answer is option a). The entire passage mainly focuses on a brief account of a
recent webinar organized by the UCLA Center for India and South Asia, where scholars and artists
laid bare the seldom-discussed problem of caste in the performing arts of South India.

18. The correct answer is option a). Plebeian, also spelled Plebian, Latin Plebs, plural Plebes,
member of the general citizenry in ancient Rome as opposed to the privileged patrician class.
Plebeian may have first been shortened to PLEB by English public school pupils in the 18th century,
when it was used as a derogatory term for pupils of lower social classes. In modern usage, PLEB is
often used as an insult to imply that someone lacks intelligence, is unsophisticated or of low social
standing.
19. The correct answer is option d). the passage clearly mentions, „Bhatkhande was not alone in his
conception and more than a hundred years after his not-so-pleasant discovery, this narrative of the
„pure South‟ continues to drive the scholarship and public consumption of its classical arts.
According to Davesh Soneji, Associate Professor, Department of South Asia Studies, University of
Pennsylvania, this notion has resulted in the erasure of a rich tradition of Tamil Muslim and Christian
music and many other art forms practised by non-Brahmin or Hindu artist communities.‟

20. the correct answer is option a). the correct meaning of the word tenet‟ means „a principle or
belief, especially one of the main principles of a religion or philosophy.‟

21. The correct answer is option b). statement 2 is the correct answer as it has been mentioned in
the passage.

22. the correct answer is option d). The meaning of the word „camaraderie‟ means „mutual trust and
friendship among people who spend a lot of time together.‟

23. the correct answer is option c). the paragraph mentions that, „Through rituals of song, drink and
dance, the Nazis‟ actions could be collectivised and normalised – and their larger project of violence
that much easier to pull off. Ultimately, genocide is a societal endeavour; music and song – like
political philosophies – are part of a society‟s cultural artefacts.‟

24. the correct answer is option c). Expository writing, as the title suggests, is predicated on
exposition, or the description and explanation of a particular idea. Topics cover pretty much the
entire gamut of human experience, from inventions to nature, emotions to politics, family to hobbies
and more.

25. The correct answer is option a). This passage is clearly an excerpt of a book, of which the main
topic has been mentioned in the first paragraph, „Under the auspices of the Corbett Centre for
Maritime Policy Studies comes a thoughtful book by Jeremy Stocker, Architects of Continental Sea-
power: Comparing Tirpitz and Gorschkov, shedding light on Grand Admiral Tirpitz and Admiral
Sergei Gorschkov, two important naval figures of the 20th century, who wrestled with the problems
and challenges of creating formidable navies for primarily continental countries.

26. the correct answer is option c). The correct meaning of the word „nascent‟ is „especially of a
process or organization) just coming into existence and beginning to display signs of future
potential.‟

27. the correct answer is option c). Both of the statements are true.

28. the correct answer is option a). the meaning of the word „infantry‟ is „soldiers marching or fighting
on foot; foot soldiers collectively.‟

29. The correct option is b). The passage clearly states that, „Tirpitz‟s ambition set off a naval arms
race with Britain, in which the latter, with its vast financial resources, easily outpaced the German
building capacity. Eventually, Britain, the chief trading partner of pre-war Germany, opted for a
strategy of a distant blockade, which crushed the morale of the German people and forced the yet
undefeated German army to sue for peace.‟
GENERAL KNOWLEDGE

30. Starlink is a SpaceX project to build a


broadband network with a cluster of orbiting spacecraft that could eventually number thousands.
Recently, the world‟s leading private company in space technology SpaceX has launched
the Starlink Network in Lower Earth Orbit (LEO) aimed at providing low-
cost and reliable space-based internet services to the world.
Space Internet with Low Earth Orbit:
The LEO extends up to 2,000 km above the Earth’s surface.
Advantages:
I. Reduced Latency: The presence of satellite at a lower height from the Earth‟s surface, will help
to bring the lag down to 20-30 milliseconds, roughly the time it takes for terrestrial systems to
transfer data.
II. More Viable: The signals from satellites in space can overcome obstacles faced by fibre-optic
cables or wireless networks easily.
Kuiper Systems LLC is a subsidiary of Amazon that was set up in 2019 to deploy a large broadband
satellite internet constellation to provide broadband internet connectivity. The deployment is also
referred to by its project name "Project Kuiper".
31. Starlink is a SpaceX project to build a
broadband network with a cluster of orbiting spacecraft that could eventually number thousands.
Recently, the world‟s leading private company in space technology SpaceX has launched
the Starlink Network in Lower Earth Orbit (LEO) aimed at providing low-
cost and reliable space-based internet services to the world.
Space Internet with Low Earth Orbit:
The LEO extends up to 2,000 km above the Earth’s surface.
Advantages:
I. Reduced Latency: The presence of satellite at a lower height from the Earth‟s surface, will help
to bring the lag down to 20-30 milliseconds, roughly the time it takes for terrestrial systems to
transfer data.
II. More Viable: The signals from satellites in space can overcome obstacles faced by fibre-optic
cables or wireless networks easily.
Kuiper Systems LLC is a subsidiary of Amazon that was set up in 2019 to deploy a large broadband
satellite internet constellation to provide broadband internet connectivity. The deployment is also
referred to by its project name "Project Kuiper".
32. Starlink is a SpaceX project to build a
broadband network with a cluster of orbiting spacecraft that could eventually number thousands.
Recently, the world‟s leading private company in space technology SpaceX has launched
the Starlink Network in Lower Earth Orbit (LEO) aimed at providing low-
cost and reliable space-based internet services to the world.
Space Internet with Low Earth Orbit:
The LEO extends up to 2,000 km above the Earth’s surface.
Advantages:
I. Reduced Latency: The presence of satellite at a lower height from the Earth‟s surface, will help
to bring the lag down to 20-30 milliseconds, roughly the time it takes for terrestrial systems to
transfer data.
II. More Viable: The signals from satellites in space can overcome obstacles faced by fibre-optic
cables or wireless networks easily.
Kuiper Systems LLC is a subsidiary of Amazon that was set up in 2019 to deploy a large broadband
satellite internet constellation to provide broadband internet connectivity. The deployment is also
referred to by its project name "Project Kuiper".
33. Starlink is a SpaceX project to build a
broadband network with a cluster of orbiting spacecraft that could eventually number thousands.
Recently, the world‟s leading private company in space technology SpaceX has launched
the Starlink Network in Lower Earth Orbit (LEO) aimed at providing low-
cost and reliable space-based internet services to the world.
Space Internet with Low Earth Orbit:
The LEO extends up to 2,000 km above the Earth’s surface.
Advantages:
I. Reduced Latency: The presence of satellite at a lower height from the Earth‟s surface, will help
to bring the lag down to 20-30 milliseconds, roughly the time it takes for terrestrial systems to
transfer data.
II. More Viable: The signals from satellites in space can overcome obstacles faced by fibre-optic
cables or wireless networks easily.
Kuiper Systems LLC is a subsidiary of Amazon that was set up in 2019 to deploy a large broadband
satellite internet constellation to provide broadband internet connectivity. The deployment is also
referred to by its project name "Project Kuiper".
34. Starlink is a SpaceX project to build a
broadband network with a cluster of orbiting spacecraft that could eventually number thousands.
Recently, the world‟s leading private company in space technology SpaceX has launched
the Starlink Network in Lower Earth Orbit (LEO) aimed at providing low-
cost and reliable space-based internet services to the world.
Space Internet with Low Earth Orbit:
The LEO extends up to 2,000 km above the Earth’s surface.
Advantages:
I. Reduced Latency: The presence of satellite at a lower height from the Earth‟s surface, will help
to bring the lag down to 20-30 milliseconds, roughly the time it takes for terrestrial systems to
transfer data.
II. More Viable: The signals from satellites in space can overcome obstacles faced by fibre-optic
cables or wireless networks easily.
Kuiper Systems LLC is a subsidiary of Amazon that was set up in 2019 to deploy a large broadband
satellite internet constellation to provide broadband internet connectivity. The deployment is also
referred to by its project name "Project Kuiper".
35. Maitri Setu is a bridge built over the Feni river, which flows between the Indian boundary in
Tripura and Bangladesh. This 1.9 km long bridge
connects Sabroom in South Tripura and Ramgarh in Bangladesh. It was constructed by
the National Highway and Infrastructure Development Corporation at the cost of Rs.133 crore.
This bridge makes Tripura the ‘Gateway of North East’ since provides access to
Bangladesh‟s Chittagong Port.

36. Maitri Setu is a bridge built over the Feni river, which flows between the Indian boundary in
Tripura and Bangladesh. This 1.9 km long bridge
connects Sabroom in South Tripura and Ramgarh in Bangladesh. It was constructed by
the National Highway and Infrastructure Development Corporation at the cost of Rs.133 crore.
This bridge makes Tripura the ‘Gateway of North East’ since provides access to
Bangladesh‟s Chittagong Port.

37. Maitri Setu is a bridge built over the Feni river, which flows between the Indian boundary in
Tripura and Bangladesh. This 1.9 km long bridge
connects Sabroom in South Tripura and Ramgarh in Bangladesh. It was constructed by
the National Highway and Infrastructure Development Corporation at the cost of Rs.133 crore.
This bridge makes Tripura the ‘Gateway of North East’ since provides access to
Bangladesh‟s Chittagong Port.

38. Maitri Setu is a bridge built over the Feni river, which flows between the Indian boundary in
Tripura and Bangladesh. This 1.9 km long bridge
connects Sabroom in South Tripura and Ramgarh in Bangladesh. It was constructed by
the National Highway and Infrastructure Development Corporation at the cost of Rs.133 crore.
This bridge makes Tripura the ‘Gateway of North East’ since provides access to
Bangladesh‟s Chittagong Port.

39. Maitri Setu is a bridge built over the Feni river, which flows between the Indian boundary in
Tripura and Bangladesh. This 1.9 km long bridge
connects Sabroom in South Tripura and Ramgarh in Bangladesh. It was constructed by
the National Highway and Infrastructure Development Corporation at the cost of Rs.133 crore.
This bridge makes Tripura the ‘Gateway of North East’ since provides access to
Bangladesh‟s Chittagong Port.

40. The QUAD Leaders will discuss the regional and global issues which are of shared interest.
They will also exchange their views on several practical areas of the cooperation to maintain a free,
open and inclusive Indo-Pacific region. The virtual summit will also provide the countries an
opportunity to exchange their views on the contemporary challenges like resilient supply chains,
maritime security, climate change and emerging & critical technologies. Leaders will also discuss the
ongoing efforts to combat the Covid-19 pandemic. They would also explore the opportunities to
collaborate so as to ensure a safe, equitable and affordable vaccines in Indo-Pacific region.
QUAD is also known as the Asian NATO. It is an informal strategic forum comprising of India, United
States, Japan, and Australia. The forum is maintained by semi-regular summits, military drills and
information between member countries.
Under the agreement, India will be using its manufacturing capacity in order to make U.S. vaccines.
For that matter, the financing will be coming from U.S. International Development Finance
Corporation and Japan Bank for International Cooperation. On the other hand, Australia will be
financing the training. The country will also provide the last-mile logistical support to distribute the
vaccines in the Southeast Asia, Pacific Islands and countries in across the Indian Ocean.
41. The QUAD Leaders will discuss the regional and global issues which are of shared interest.
They will also exchange their views on several practical areas of the cooperation to maintain a free,
open and inclusive Indo-Pacific region. The virtual summit will also provide the countries an
opportunity to exchange their views on the contemporary challenges like resilient supply chains,
maritime security, climate change and emerging & critical technologies. Leaders will also discuss the
ongoing efforts to combat the Covid-19 pandemic. They would also explore the opportunities to
collaborate so as to ensure a safe, equitable and affordable vaccines in Indo-Pacific region.
QUAD is also known as the Asian NATO. It is an informal strategic forum comprising of India, United
States, Japan, and Australia. The forum is maintained by semi-regular summits, military drills and
information between member countries.
Under the agreement, India will be using its manufacturing capacity in order to make U.S. vaccines.
For that matter, the financing will be coming from U.S. International Development Finance
Corporation and Japan Bank for International Cooperation. On the other hand, Australia will be
financing the training. The country will also provide the last-mile logistical support to distribute the
vaccines in the Southeast Asia, Pacific Islands and countries in across the Indian Ocean.
42. The QUAD Leaders will discuss the regional and global issues which are of shared interest.
They will also exchange their views on several practical areas of the cooperation to maintain a free,
open and inclusive Indo-Pacific region. The virtual summit will also provide the countries an
opportunity to exchange their views on the contemporary challenges like resilient supply chains,
maritime security, climate change and emerging & critical technologies. Leaders will also discuss the
ongoing efforts to combat the Covid-19 pandemic. They would also explore the opportunities to
collaborate so as to ensure a safe, equitable and affordable vaccines in Indo-Pacific region.
QUAD is also known as the Asian NATO. It is an informal strategic forum comprising of India, United
States, Japan, and Australia. The forum is maintained by semi-regular summits, military drills and
information between member countries.
Under the agreement, India will be using its manufacturing capacity in order to make U.S. vaccines.
For that matter, the financing will be coming from U.S. International Development Finance
Corporation and Japan Bank for International Cooperation. On the other hand, Australia will be
financing the training. The country will also provide the last-mile logistical support to distribute the
vaccines in the Southeast Asia, Pacific Islands and countries in across the Indian Ocean.
43. The QUAD Leaders will discuss the regional and global issues which are of shared interest.
They will also exchange their views on several practical areas of the cooperation to maintain a free,
open and inclusive Indo-Pacific region. The virtual summit will also provide the countries an
opportunity to exchange their views on the contemporary challenges like resilient supply chains,
maritime security, climate change and emerging & critical technologies. Leaders will also discuss the
ongoing efforts to combat the Covid-19 pandemic. They would also explore the opportunities to
collaborate so as to ensure a safe, equitable and affordable vaccines in Indo-Pacific region.
QUAD is also known as the Asian NATO. It is an informal strategic forum comprising of India, United
States, Japan, and Australia. The forum is maintained by semi-regular summits, military drills and
information between member countries.
Under the agreement, India will be using its manufacturing capacity in order to make U.S. vaccines.
For that matter, the financing will be coming from U.S. International Development Finance
Corporation and Japan Bank for International Cooperation. On the other hand, Australia will be
financing the training. The country will also provide the last-mile logistical support to distribute the
vaccines in the Southeast Asia, Pacific Islands and countries in across the Indian Ocean.
44. The QUAD Leaders will discuss the regional and global issues which are of shared interest.
They will also exchange their views on several practical areas of the cooperation to maintain a free,
open and inclusive Indo-Pacific region. The virtual summit will also provide the countries an
opportunity to exchange their views on the contemporary challenges like resilient supply chains,
maritime security, climate change and emerging & critical technologies. Leaders will also discuss the
ongoing efforts to combat the Covid-19 pandemic. They would also explore the opportunities to
collaborate so as to ensure a safe, equitable and affordable vaccines in Indo-Pacific region.
QUAD is also known as the Asian NATO. It is an informal strategic forum comprising of India, United
States, Japan, and Australia. The forum is maintained by semi-regular summits, military drills and
information between member countries.
Under the agreement, India will be using its manufacturing capacity in order to make U.S. vaccines.
For that matter, the financing will be coming from U.S. International Development Finance
Corporation and Japan Bank for International Cooperation. On the other hand, Australia will be
financing the training. The country will also provide the last-mile logistical support to distribute the
vaccines in the Southeast Asia, Pacific Islands and countries in across the Indian Ocean.
45. The Democracy Index report 2020 by the covers the 167 countries.
It has classified the economies in categories such as flawed democracies, full democracies, hybrid
regimes, and authoritarian regimes. The report highlights that, 23 countries are full democracies, 52
are flawed democracies, 35 countries are hybrid regimes while 57 countries are authoritarian
regimes out of total 167 countries.
In the Democracy Index, Norway has topped. The 2nd 3rd 4th and the 5th ranks have been secured
by Iceland, Sweden, New Zealand and Canada respectively in the index. India‟s rank has degraded
by two places from the previous year ranking. This year India has been ranked at 53rd place. Sri
Lanka was placed at 68th position. It is also classified as a flawed democracy. Pakistan was ranked
at 105th position. It has been classified as a „hybrid regime‟.
The report highlights that, multiple instances of “crackdowns” on civil liberties and the “democratic
backsliding” by authorities has degraded the India‟s ranking. However, India was ranked higher than
most of its neighbouring nations. In the index, India has been classified as a „flawed democracy‟.
The other flawed democracies include U.S., France, Brazil and Belgium.
46. The Democracy Index report 2020 by the covers the 167 countries.
It has classified the economies in categories such as flawed democracies, full democracies, hybrid
regimes, and authoritarian regimes. The report highlights that, 23 countries are full democracies, 52
are flawed democracies, 35 countries are hybrid regimes while 57 countries are authoritarian
regimes out of total 167 countries.
In the Democracy Index, Norway has topped. The 2nd 3rd 4th and the 5th ranks have been secured
by Iceland, Sweden, New Zealand and Canada respectively in the index. India‟s rank has degraded
by two places from the previous year ranking. This year India has been ranked at 53rd place. Sri
Lanka was placed at 68th position. It is also classified as a flawed democracy. Pakistan was ranked
at 105th position. It has been classified as a „hybrid regime‟.
The report highlights that, multiple instances of “crackdowns” on civil liberties and the “democratic
backsliding” by authorities has degraded the India‟s ranking. However, India was ranked higher than
most of its neighbouring nations. In the index, India has been classified as a „flawed democracy‟.
The other flawed democracies include U.S., France, Brazil and Belgium.
47. The Democracy Index report 2020 by the covers the 167 countries.
It has classified the economies in categories such as flawed democracies, full democracies, hybrid
regimes, and authoritarian regimes. The report highlights that, 23 countries are full democracies, 52
are flawed democracies, 35 countries are hybrid regimes while 57 countries are authoritarian
regimes out of total 167 countries.
In the Democracy Index, Norway has topped. The 2nd 3rd 4th and the 5th ranks have been secured
by Iceland, Sweden, New Zealand and Canada respectively in the index. India‟s rank has degraded
by two places from the previous year ranking. This year India has been ranked at 53rd place. Sri
Lanka was placed at 68th position. It is also classified as a flawed democracy. Pakistan was ranked
at 105th position. It has been classified as a „hybrid regime‟.
The report highlights that, multiple instances of “crackdowns” on civil liberties and the “democratic
backsliding” by authorities has degraded the India‟s ranking. However, India was ranked higher than
most of its neighbouring nations. In the index, India has been classified as a „flawed democracy‟.
The other flawed democracies include U.S., France, Brazil and Belgium.
48. The Democracy Index report 2020 by the covers the 167 countries.
It has classified the economies in categories such as flawed democracies, full democracies, hybrid
regimes, and authoritarian regimes. The report highlights that, 23 countries are full democracies, 52
are flawed democracies, 35 countries are hybrid regimes while 57 countries are authoritarian
regimes out of total 167 countries.
In the Democracy Index, Norway has topped. The 2nd 3rd 4th and the 5th ranks have been secured
by Iceland, Sweden, New Zealand and Canada respectively in the index. India‟s rank has degraded
by two places from the previous year ranking. This year India has been ranked at 53rd place. Sri
Lanka was placed at 68th position. It is also classified as a flawed democracy. Pakistan was ranked
at 105th position. It has been classified as a „hybrid regime‟.
The report highlights that, multiple instances of “crackdowns” on civil liberties and the “democratic
backsliding” by authorities has degraded the India‟s ranking. However, India was ranked higher than
most of its neighbouring nations. In the index, India has been classified as a „flawed democracy‟.
The other flawed democracies include U.S., France, Brazil and Belgium.
49. The Democracy Index report 2020 by the covers the 167 countries.
It has classified the economies in categories such as flawed democracies, full democracies, hybrid
regimes, and authoritarian regimes. The report highlights that, 23 countries are full democracies, 52
are flawed democracies, 35 countries are hybrid regimes while 57 countries are authoritarian
regimes out of total 167 countries.
In the Democracy Index, Norway has topped. The 2nd 3rd 4th and the 5th ranks have been secured
by Iceland, Sweden, New Zealand and Canada respectively in the index. India‟s rank has degraded
by two places from the previous year ranking. This year India has been ranked at 53rd place. Sri
Lanka was placed at 68th position. It is also classified as a flawed democracy. Pakistan was ranked
at 105th position. It has been classified as a „hybrid regime‟.
The report highlights that, multiple instances of “crackdowns” on civil liberties and the “democratic
backsliding” by authorities has degraded the India‟s ranking. However, India was ranked higher than
most of its neighbouring nations. In the index, India has been classified as a „flawed democracy‟.
The other flawed democracies include U.S., France, Brazil and Belgium.
50. Recently, the Government of India, while presenting the Union budget 2021, announced that
Operation Green (OG) will be expanded beyond tomatoes, onions, and potatoes (TOP) to 22
perishable commodities. Operation Green was originally launched in 2018. The idea was to build
value chains of TOP on the lines of “Operation Flood” (AMUL model) for milk in such a way that will
ensure a higher share of consumer‟s rupee goes to farmers and stabilizes their prices. There were
three basic objectives when OG was launched viz. containing the wide price volatility, building
efficient value chains, and reducing the post-harvest losses.
Objectives of Operation Green
Containing Price Volatility: It should contain the wide price volatility in the three largest vegetables of
India (TOP). Tomatoes-onions-potatoes (TOP) are the three basic vegetables that face extreme
price volatility and the government often finds itself on the edge in fulfilling its dual objectives of
ensuring remunerative prices for farmers and affordable prices for consumers.
For price stabilization, NAFED has to intervene in the market wherever prices crashed due to a glut,
to procure some excess arrivals from the surplus regions to store them near major consuming
centers.
Building Efficient Value Chains: It envisages building efficient value chains of these from fresh to
value-added products with a view to give a larger share of the consumers‟ rupee to the farmers.
In order to fulfill this target, there is a provision of providing subsidy to the Farmer Producer
Organisation (FPO).
Reducing the Post-Harvest Losses: It should reduce the post-harvest losses by building modern
warehouses, cold storage wherever needed, and food processing clusters.
Issues in Operation Green Replicating Operation Flood
I. Heterogenous TOP: Each commodity under OG has its own specificity, production and
consumption cycle, unlike the homogeneity of milk as a single commodity.
II. There are so many varieties of TOP vegetables, grown in different climatic conditions and in
different seasons, making marketing intervention (processing and storage) all the more complex.
III. APMC Barrier: Milk does not pass through any APMC, involves no commissions, and farmers
normally get 75-80% of the consumer‟s rupee.
However, TOP are mostly traded in APMC markets, with layers of mandi fees and commissions, and
farmers get less than one-third of the consumer‟s rupee.
51. Recently, the Government of India, while presenting the Union budget 2021, announced that
Operation Green (OG) will be expanded beyond tomatoes, onions, and potatoes (TOP) to 22
perishable commodities. Operation Green was originally launched in 2018. The idea was to build
value chains of TOP on the lines of “Operation Flood” (AMUL model) for milk in such a way that will
ensure a higher share of consumer‟s rupee goes to farmers and stabilizes their prices. There were
three basic objectives when OG was launched viz. containing the wide price volatility, building
efficient value chains, and reducing the post-harvest losses.
Objectives of Operation Green
Containing Price Volatility: It should contain the wide price volatility in the three largest vegetables of
India (TOP). Tomatoes-onions-potatoes (TOP) are the three basic vegetables that face extreme
price volatility and the government often finds itself on the edge in fulfilling its dual objectives of
ensuring remunerative prices for farmers and affordable prices for consumers.
For price stabilization, NAFED has to intervene in the market wherever prices crashed due to a glut,
to procure some excess arrivals from the surplus regions to store them near major consuming
centers.
Building Efficient Value Chains: It envisages building efficient value chains of these from fresh to
value-added products with a view to give a larger share of the consumers‟ rupee to the farmers.
In order to fulfill this target, there is a provision of providing subsidy to the Farmer Producer
Organisation (FPO).
Reducing the Post-Harvest Losses: It should reduce the post-harvest losses by building modern
warehouses, cold storage wherever needed, and food processing clusters.
Issues in Operation Green Replicating Operation Flood
I. Heterogenous TOP: Each commodity under OG has its own specificity, production and
consumption cycle, unlike the homogeneity of milk as a single commodity.
II. There are so many varieties of TOP vegetables, grown in different climatic conditions and in
different seasons, making marketing intervention (processing and storage) all the more complex.
III. APMC Barrier: Milk does not pass through any APMC, involves no commissions, and farmers
normally get 75-80% of the consumer‟s rupee.
However, TOP are mostly traded in APMC markets, with layers of mandi fees and commissions, and
farmers get less than one-third of the consumer‟s rupee.
52. Recently, the Government of India, while presenting the Union budget 2021, announced that
Operation Green (OG) will be expanded beyond tomatoes, onions, and potatoes (TOP) to 22
perishable commodities. Operation Green was originally launched in 2018. The idea was to build
value chains of TOP on the lines of “Operation Flood” (AMUL model) for milk in such a way that will
ensure a higher share of consumer‟s rupee goes to farmers and stabilizes their prices. There were
three basic objectives when OG was launched viz. containing the wide price volatility, building
efficient value chains, and reducing the post-harvest losses.
Objectives of Operation Green
Containing Price Volatility: It should contain the wide price volatility in the three largest vegetables of
India (TOP). Tomatoes-onions-potatoes (TOP) are the three basic vegetables that face extreme
price volatility and the government often finds itself on the edge in fulfilling its dual objectives of
ensuring remunerative prices for farmers and affordable prices for consumers.
For price stabilization, NAFED has to intervene in the market wherever prices crashed due to a glut,
to procure some excess arrivals from the surplus regions to store them near major consuming
centers.
Building Efficient Value Chains: It envisages building efficient value chains of these from fresh to
value-added products with a view to give a larger share of the consumers‟ rupee to the farmers.
In order to fulfill this target, there is a provision of providing subsidy to the Farmer Producer
Organisation (FPO).
Reducing the Post-Harvest Losses: It should reduce the post-harvest losses by building modern
warehouses, cold storage wherever needed, and food processing clusters.
Issues in Operation Green Replicating Operation Flood
I. Heterogenous TOP: Each commodity under OG has its own specificity, production and
consumption cycle, unlike the homogeneity of milk as a single commodity.
II. There are so many varieties of TOP vegetables, grown in different climatic conditions and in
different seasons, making marketing intervention (processing and storage) all the more complex.
III. APMC Barrier: Milk does not pass through any APMC, involves no commissions, and farmers
normally get 75-80% of the consumer‟s rupee.
However, TOP are mostly traded in APMC markets, with layers of mandi fees and commissions, and
farmers get less than one-third of the consumer‟s rupee.
53. Recently, the Government of India, while presenting the Union budget 2021, announced that
Operation Green (OG) will be expanded beyond tomatoes, onions, and potatoes (TOP) to 22
perishable commodities. Operation Green was originally launched in 2018. The idea was to build
value chains of TOP on the lines of “Operation Flood” (AMUL model) for milk in such a way that will
ensure a higher share of consumer‟s rupee goes to farmers and stabilizes their prices. There were
three basic objectives when OG was launched viz. containing the wide price volatility, building
efficient value chains, and reducing the post-harvest losses.
Objectives of Operation Green
Containing Price Volatility: It should contain the wide price volatility in the three largest vegetables of
India (TOP). Tomatoes-onions-potatoes (TOP) are the three basic vegetables that face extreme
price volatility and the government often finds itself on the edge in fulfilling its dual objectives of
ensuring remunerative prices for farmers and affordable prices for consumers.
For price stabilization, NAFED has to intervene in the market wherever prices crashed due to a glut,
to procure some excess arrivals from the surplus regions to store them near major consuming
centers.
Building Efficient Value Chains: It envisages building efficient value chains of these from fresh to
value-added products with a view to give a larger share of the consumers‟ rupee to the farmers.
In order to fulfill this target, there is a provision of providing subsidy to the Farmer Producer
Organisation (FPO).
Reducing the Post-Harvest Losses: It should reduce the post-harvest losses by building modern
warehouses, cold storage wherever needed, and food processing clusters.
Issues in Operation Green Replicating Operation Flood
I. Heterogenous TOP: Each commodity under OG has its own specificity, production and
consumption cycle, unlike the homogeneity of milk as a single commodity.
II. There are so many varieties of TOP vegetables, grown in different climatic conditions and in
different seasons, making marketing intervention (processing and storage) all the more complex.
III. APMC Barrier: Milk does not pass through any APMC, involves no commissions, and farmers
normally get 75-80% of the consumer‟s rupee.
However, TOP are mostly traded in APMC markets, with layers of mandi fees and commissions, and
farmers get less than one-third of the consumer‟s rupee.
54. Recently, the Government of India, while presenting the Union budget 2021, announced that
Operation Green (OG) will be expanded beyond tomatoes, onions, and potatoes (TOP) to 22
perishable commodities. Operation Green was originally launched in 2018. The idea was to build
value chains of TOP on the lines of “Operation Flood” (AMUL model) for milk in such a way that will
ensure a higher share of consumer‟s rupee goes to farmers and stabilizes their prices. There were
three basic objectives when OG was launched viz. containing the wide price volatility, building
efficient value chains, and reducing the post-harvest losses.
Objectives of Operation Green
Containing Price Volatility: It should contain the wide price volatility in the three largest vegetables of
India (TOP). Tomatoes-onions-potatoes (TOP) are the three basic vegetables that face extreme
price volatility and the government often finds itself on the edge in fulfilling its dual objectives of
ensuring remunerative prices for farmers and affordable prices for consumers.
For price stabilization, NAFED has to intervene in the market wherever prices crashed due to a glut,
to procure some excess arrivals from the surplus regions to store them near major consuming
centers.
Building Efficient Value Chains: It envisages building efficient value chains of these from fresh to
value-added products with a view to give a larger share of the consumers‟ rupee to the farmers.
In order to fulfill this target, there is a provision of providing subsidy to the Farmer Producer
Organisation (FPO).
Reducing the Post-Harvest Losses: It should reduce the post-harvest losses by building modern
warehouses, cold storage wherever needed, and food processing clusters.
Issues in Operation Green Replicating Operation Flood
I. Heterogenous TOP: Each commodity under OG has its own specificity, production and
consumption cycle, unlike the homogeneity of milk as a single commodity.
II. There are so many varieties of TOP vegetables, grown in different climatic conditions and in
different seasons, making marketing intervention (processing and storage) all the more complex.
III. APMC Barrier: Milk does not pass through any APMC, involves no commissions, and farmers
normally get 75-80% of the consumer‟s rupee.
However, TOP are mostly traded in APMC markets, with layers of mandi fees and commissions, and
farmers get less than one-third of the consumer‟s rupee.
55. JCPOA was signed in 2015, but former US President Trump has withdrawn from it (in 2018) and
embarked on a policy of „maximum pressure‟ to coerce Iran back to the negotiating table.
The JCPOA was the result of prolonged negotiations from 2013 and 2015 between Iran and P5+1
(China, France, Germany, Russia, the United Kingdom, the United States, and the European Union,
or the EU). When Donald Trump became president, he withdrew from the deal and called it a
“horrible, one-sided deal that should have never, ever been made”.
The U.S. decision was criticized by all other parties to the JCPOA (including the European allies)
because Iran was in compliance with its obligations, as certified by the International Atomic Energy
Agency (IAEA). For the first year after the U.S. withdrawal, Iran‟s response was muted as the E-3
(France, Germany, the U.K.) and the EU promised to find ways to mitigate the U.S. decision.
The E-3‟s promised relief Instrument in Support of Trade Exchanges (INSTEX), created in 2019 to
facilitate limited trade with Iran.
56. JCPOA was signed in 2015, but former US President Trump has withdrawn from it (in 2018) and
embarked on a policy of „maximum pressure‟ to coerce Iran back to the negotiating table.
The JCPOA was the result of prolonged negotiations from 2013 and 2015 between Iran and P5+1
(China, France, Germany, Russia, the United Kingdom, the United States, and the European Union,
or the EU). When Donald Trump became president, he withdrew from the deal and called it a
“horrible, one-sided deal that should have never, ever been made”.
The U.S. decision was criticized by all other parties to the JCPOA (including the European allies)
because Iran was in compliance with its obligations, as certified by the International Atomic Energy
Agency (IAEA). For the first year after the U.S. withdrawal, Iran‟s response was muted as the E-3
(France, Germany, the U.K.) and the EU promised to find ways to mitigate the U.S. decision.
The E-3‟s promised relief Instrument in Support of Trade Exchanges (INSTEX), created in 2019 to
facilitate limited trade with Iran.
57. JCPOA was signed in 2015, but former US President Trump has withdrawn from it (in 2018) and
embarked on a policy of „maximum pressure‟ to coerce Iran back to the negotiating table.
The JCPOA was the result of prolonged negotiations from 2013 and 2015 between Iran and P5+1
(China, France, Germany, Russia, the United Kingdom, the United States, and the European Union,
or the EU). When Donald Trump became president, he withdrew from the deal and called it a
“horrible, one-sided deal that should have never, ever been made”.
The U.S. decision was criticized by all other parties to the JCPOA (including the European allies)
because Iran was in compliance with its obligations, as certified by the International Atomic Energy
Agency (IAEA). For the first year after the U.S. withdrawal, Iran‟s response was muted as the E-3
(France, Germany, the U.K.) and the EU promised to find ways to mitigate the U.S. decision.
The E-3‟s promised relief Instrument in Support of Trade Exchanges (INSTEX), created in 2019 to
facilitate limited trade with Iran.
58. JCPOA was signed in 2015, but former US President Trump has withdrawn from it (in 2018) and
embarked on a policy of „maximum pressure‟ to coerce Iran back to the negotiating table.
The JCPOA was the result of prolonged negotiations from 2013 and 2015 between Iran and P5+1
(China, France, Germany, Russia, the United Kingdom, the United States, and the European Union,
or the EU). When Donald Trump became president, he withdrew from the deal and called it a
“horrible, one-sided deal that should have never, ever been made”.
The U.S. decision was criticized by all other parties to the JCPOA (including the European allies)
because Iran was in compliance with its obligations, as certified by the International Atomic Energy
Agency (IAEA). For the first year after the U.S. withdrawal, Iran‟s response was muted as the E-3
(France, Germany, the U.K.) and the EU promised to find ways to mitigate the U.S. decision.
The E-3‟s promised relief Instrument in Support of Trade Exchanges (INSTEX), created in 2019 to
facilitate limited trade with Iran.
59. Critical Information Infrastructure: According to Section 70(1) of the Information Technology Act,
CII is defined as a “computer resource, the incapacitation or destruction of which, shall have
debilitating impact on national security, economy, public health or safety”.
The International Telecommunication Union (ITU) is a specialized agency within the United Nations
which plays a leading role in the standardization and development of telecommunications and cyber
security issues.
Protection and resilience of critical information infrastructure with the National Critical Information
Infrastructure Protection Centre (NCIIPC) operating as the nodal agency. NCIIPC has been created
under Information Technology Act, 2000 to secure India‟s critical information infrastructure. It is
based in New Delhi.
Cyber Surakshit Bharat Initiative: It was launched in 2018 with an aim to spread awareness about
cybercrime and building capacity for safety measures for Chief Information Security Officers (CISOs)
and frontline IT staff across all government departments. Online cybercrime reporting portal has
been launched to enable complainants to report complaints pertaining to Child Pornography/Child
Sexual Abuse Material, rape/gang rape imageries or sexually explicit content.
60. Critical Information Infrastructure: According to Section 70(1) of the Information Technology Act,
CII is defined as a “computer resource, the incapacitation or destruction of which, shall have
debilitating impact on national security, economy, public health or safety”.
The International Telecommunication Union (ITU) is a specialized agency within the United Nations
which plays a leading role in the standardization and development of telecommunications and cyber
security issues.
Protection and resilience of critical information infrastructure with the National Critical Information
Infrastructure Protection Centre (NCIIPC) operating as the nodal agency. NCIIPC has been created
under Information Technology Act, 2000 to secure India‟s critical information infrastructure. It is
based in New Delhi.
Cyber Surakshit Bharat Initiative: It was launched in 2018 with an aim to spread awareness about
cybercrime and building capacity for safety measures for Chief Information Security Officers (CISOs)
and frontline IT staff across all government departments. Online cybercrime reporting portal has
been launched to enable complainants to report complaints pertaining to Child Pornography/Child
Sexual Abuse Material, rape/gang rape imageries or sexually explicit content.
61. Critical Information Infrastructure: According to Section 70(1) of the Information Technology Act,
CII is defined as a “computer resource, the incapacitation or destruction of which, shall have
debilitating impact on national security, economy, public health or safety”.
The International Telecommunication Union (ITU) is a specialized agency within the United Nations
which plays a leading role in the standardization and development of telecommunications and cyber
security issues.
Protection and resilience of critical information infrastructure with the National Critical Information
Infrastructure Protection Centre (NCIIPC) operating as the nodal agency. NCIIPC has been created
under Information Technology Act, 2000 to secure India‟s critical information infrastructure. It is
based in New Delhi.
Cyber Surakshit Bharat Initiative: It was launched in 2018 with an aim to spread awareness about
cybercrime and building capacity for safety measures for Chief Information Security Officers (CISOs)
and frontline IT staff across all government departments. Online cybercrime reporting portal has
been launched to enable complainants to report complaints pertaining to Child Pornography/Child
Sexual Abuse Material, rape/gang rape imageries or sexually explicit content.

62. Critical Information Infrastructure: According to Section 70(1) of the Information Technology Act,
CII is defined as a “computer resource, the incapacitation or destruction of which, shall have
debilitating impact on national security, economy, public health or safety”.
The International Telecommunication Union (ITU) is a specialized agency within the United Nations
which plays a leading role in the standardization and development of telecommunications and cyber
security issues.
Protection and resilience of critical information infrastructure with the National Critical Information
Infrastructure Protection Centre (NCIIPC) operating as the nodal agency. NCIIPC has been created
under Information Technology Act, 2000 to secure India‟s critical information infrastructure. It is
based in New Delhi.
Cyber Surakshit Bharat Initiative: It was launched in 2018 with an aim to spread awareness about
cybercrime and building capacity for safety measures for Chief Information Security Officers (CISOs)
and frontline IT staff across all government departments. Online cybercrime reporting portal has
been launched to enable complainants to report complaints pertaining to Child Pornography/Child
Sexual Abuse Material, rape/gang rape imageries or sexually explicit content.

63. Recently, the government has notified Information Technology (Intermediary Guidelines and
Digital Media Ethics Code) Rules 2021. These new rules broadly deal with social media and over-
the-top (OTT) platforms.
These rules have been framed in exercise of powers under section 87 (2) of the Information
Technology (IT) Act, 2000 and in supersession of the earlier Information Technology (Intermediary
Guidelines) Rules 2011. In case, due diligence is not followed by the intermediary, safe harbor
provisions will not apply to them.
The safe harbour provisions have been defined under Section 79 of the IT Act, and protect social
media intermediaries by giving them immunity from legal prosecution for any content posted on their
platforms.
The intermediaries shall appoint a Grievance Officer to deal with such complaints and share the
name and contact details of such officers. The Grievance Officer will have to acknowledge the
complaint within 24 hours and resolve it within 15 days from its receipt.
64. Recently, the government has notified Information Technology (Intermediary
Guidelines and Digital Media Ethics Code) Rules 2021. These new rules broadly deal with social
media and over-the-top (OTT) platforms.
These rules have been framed in exercise of powers under section 87 (2) of the Information
Technology (IT) Act, 2000 and in supersession of the earlier Information Technology (Intermediary
Guidelines) Rules 2011. In case, due diligence is not followed by the intermediary, safe harbor
provisions will not apply to them.
The safe harbour provisions have been defined under Section 79 of the IT Act, and protect social
media intermediaries by giving them immunity from legal prosecution for any content posted on their
platforms.
The intermediaries shall appoint a Grievance Officer to deal with such complaints and share the
name and contact details of such officers. The Grievance Officer will have to acknowledge the
complaint within 24 hours and resolve it within 15 days from its receipt.
65. Recently, the government has notified Information Technology (Intermediary Guidelines and
Digital Media Ethics Code) Rules 2021. These new rules broadly deal with social media and over-
the-top (OTT) platforms.
These rules have been framed in exercise of powers under section 87 (2) of the Information
Technology (IT) Act, 2000 and in supersession of the earlier Information Technology (Intermediary
Guidelines) Rules 2011. In case, due diligence is not followed by the intermediary, safe harbor
provisions will not apply to them.
The safe harbour provisions have been defined under Section 79 of the IT Act, and protect social
media intermediaries by giving them immunity from legal prosecution for any content posted on their
platforms.
The intermediaries shall appoint a Grievance Officer to deal with such complaints and share the
name and contact details of such officers. The Grievance Officer will have to acknowledge the
complaint within 24 hours and resolve it within 15 days from its receipt.
66. Recently, the government has notified Information Technology (Intermediary Guidelines and
Digital Media Ethics Code) Rules 2021. These new rules broadly deal with social media and over-
the-top (OTT) platforms.
These rules have been framed in exercise of powers under section 87 (2) of the Information
Technology (IT) Act, 2000 and in supersession of the earlier Information Technology (Intermediary
Guidelines) Rules 2011. In case, due diligence is not followed by the intermediary, safe harbor
provisions will not apply to them.
The safe harbour provisions have been defined under Section 79 of the IT Act, and protect social
media intermediaries by giving them immunity from legal prosecution for any content posted on their
platforms.
The intermediaries shall appoint a Grievance Officer to deal with such complaints and share the
name and contact details of such officers. The Grievance Officer will have to acknowledge the
complaint within 24 hours and resolve it within 15 days from its receipt.

LEGAL REASONING

67. Since it is already mentioned in the passage that mere possession doesn‟t mean that the
possessor will have any rights, title and interest in respect of the said premises. Hence option d. is
correct.

68. Since the petition has been already evicted with a notice from the court, he therefore is no longer
entitled to electricity. None of options therefore gave a proper reason which is why option d. is
correct

69. He won‟t lose the case since he has not been evicted by law which in turn makes him entitled to
electricity. Therefore, the reasoning given in all the option are incorrect which is why option d. is
correct.

70. Although A has the possession of B‟s property since long, but that also means that the rights and
title of the same still lies with B. Therefore, B is liable for the same. Hence option b. is correct.

71. Since she has already been served with an eviction notice by law, she is therefore no longer
entitled to electricity at present. Therefore, she will lose. Hence, option b. is correct.

72. The central idea of the passage is that many people will lose their citizenship due to the CAA
NRC. While Hindus may be able to come back, muslims will not be able to come back. This idea is
summarized in Option C.

73. The question pertains to the intent of the CAA law. The passage states that the intent of the law
is to bring non-muslims back to the country as citizens. Please note that only CAA is being asked
about and not NRC.

74. The primary objective of the CAA Law is that any person who is a non-muslim shall get his
citizenship back through the law. Thus, Y will get his citizenship back for he is a hindu and X will not
for he is a muslim.

75. The passage states that the concept of who is an Indian has itself come into question after the
government's response to criticism

76. The passage clearly mentions that any person who does not agree with the version of the
government is labeled as an anti-national and is stated to be less Indian as they are raising a voice
against the law.

77. The hospital is the indemnifier for the wheelchair company as it promises to account for all the
loss caused by the act of the hospital itself.
78. The indemnity clause states protection from any liability if any other person also gets injured on
her property, of which walkway is a part, not just her home. Hence if the neighbor were to sue her, it
would fall in the protection of the insurer.

79. Luke would pay, as he signed an explicit agreement to be the indemnifier in case of any such
loss caused through the vehicle while it was in his possession.

80. No, as Robert protected himself by getting them to sign an indemnity clause, hence saving
himself form such liability and making the third person responsible.

81. There are generally two parties in indemnity contracts. The person who promises to indemnify
for a loss is the Indemnifier. On the other hand, the person whose losses the indemnifier promises to
make good is the Indemnified.

82. The nature of circumstances may also create indemnity obligations impliedly. For example, A
does an act at the request of B. If B suffers some losses and A offers to compensate him, they
impliedly create an indemnity contract.

83. In case of grave and sudden provocation, the act committed should be a sudden reaction to the
situation, wheareas in this situation the act was committed after a long gap of time.

84. In case of private defence the force used should be adequate and necessary without any
intention of doing more harm than is necessary for the purpose of such defence.

85. Culpable homicide is not murder if the offender, being a public servant or aiding a public servant
acting for the advancement of public justice, exceeds the powers given to him by law, and causes
death by doing an act which he, in good faith, believes to be lawful and necessary for the due
discharge of his duty as such public servant and without ill-will towards the person whose death is
caused.

86. The exception of sudden fight applies to cases wherein death is caused in a sudden fight without
premeditation in the heat of passion in a sudden quarrel. In this case the act was well planned.

87. Free and voluntary consent is required to take the exception of consent but in this case, there
was no voluntary and free consent as he was instigated by his guardian.

88. Exception of private defence can be taken if the offender causes death of a person against
whom he was exercising the right of defence without premeditation and without any intention of
doing more harm than is necessary for the purpose of such defence and in this case the harm
inflicted was necessary as otherwise “A” would have been killed.

89. The passage states that the CPC does not apply to the NCDRC and thus they are not bound by
the rule of circumstances being “beyond his control” as stipulated under the CPC. Thus Narain may
not be granted an adjournment.

90. Narain has successfully convinced the judge. Since the judge is now bound by the CPC, the rule
of circumstances being beyond Narain‟s control will bind the judge and an adjournment will have to
be granted.

91. Narain may be granted an adjournment for he is genuinely unwell and thus the circumstances
are beyond his control. The 3 adjournments rule may not apply for he has only sought for 2
adjournments.
92. Devgan is most likely to get an adjournment. B cannot be the answer for more than 3
adjournments cannot be granted. Devgan has not asked for 3 adjournments yet and thus will get it.

93. The passage above mentions rules of the Civil Procedure Code. It also states that these rules
are only applicable to a civil court and not to any other court. In this case, the court is a high court
and thus the rules do not apply to them.

94. The passage states that for an adjournment to be given, the circumstances must be beyond a
person‟s control. Such is not the case here. However, It cannot be said that a matter being close to
her heart is a circumstance beyond Laxmi‟s control.

95. Suraj is liable for Sexual Assault of Sheetal as from the circumstances and acts of Suraj it can be
construed that he had enough sexual intention to get into physical contact with Sheetal and had
touched upon her private parts.

96. Sexual intention is simply the state of mind of the accused person and has to be inferred from
the circumstances only and cannot be relied solely upon direct evidences. But it cannot amount to
sexual assault as one of the essential of it is act or conduct along with such sexual intention.

97. The act of Mehul amounts to sexual assault with Tamana as there was sexual intention and
physical act both present which are the essential of the offence of sexual assault. It is immaterial
whether there was skin to skin contact or not. Sexual intercourse with protection still amounts to
physical contact.

98. Maulana is not liable for sexual assault as there had to be sexual intention for the same and it is
absent in this case. Moreover touch upon private parts under FGM was part of religious practices set
up by the society and followed by whole community. Thus, it cannot be construed to be sexual
assault.

99. POCSO Act was implemented to provide protection to minors against Sexual Assault and Sexual
Harassment and is gender neutral i.e for both male and female child.

100. The correct answer is (c). In the previous year 2019-20, Dheer has been in India for 183 days
at least which makes him a resident of India. Since, a person “resident” in India is taxed on his or her
worldwide income, therefore income earned by Dheer both at TCS in India and Google in the USA
would be taxable in India.

101. The correct answer is (a). II is wrong as the article suggests that if a person involuntarily stays
due to compulsion the period of stay shall be excluded from the time to determine residential status.
IV is wrong as the Supreme Court did not allow the petition and made an order for exclusion of the
time period but ordered the petitioner to make representation before CBDT.

102. The correct answer is (d). Azaan was compelled to stay in India due to the war and did not
volunteer to do so. In the light of CIT v Suresh Nanda, he shall not be treated as an Indian resident
for the concerned previous year.

103. The correct answer is (b). A resident of India is liable to pay income tax on income earned
worldwide. Mitchell stayed in India for 193 days and is thus a resident. All his income shall be
taxable in India. He voluntarily decided to stay in India until the court gives the judgment. Therefore,
Mitchell is liable to pay income tax on all the income earned by him across the world.

104. The correct answer is (a). In the present case, Anas is a resident of India for the previous year
2021-22. He has been in India in the previous year for more than 60 days and has been in India for
more than 365 days in the last 4 years. Therefore, he is a resident of India in the previous year and
is liable to pay income tax on all the income earned worldwide.

LOGICAL REASONING

105. Rationale – Option (A) is incorrect and goes against the point stated by the author. Option (D)
cannot be traced to the information provided above. Option (C) may be true but cannot be
reasonable inferred from the passage. Therefore, the correct answer is Option (B).

106. Rationale – Option (B) is incorrect as giving the vaccine to „all‟ is something the author doesn‟t
discuss. The article simply brings forth the plight of certain individuals. Option (C) is incorrect as the
author sets off to find a co-relation between internet access and vaccination drive but does not
emphasize on providing internet access but giving vaccines reservations. Option (D) is incorrect as
the statement is too general and does not cater to the author‟s argument.
Hence, A
107. Rationale – Option (A) is incorrect as the problem posed by the author relates to access to
healthcare and not the behavior of professionals once such access is given. Options (B) & (C) are
incorrect as they have no impact on the discussion posed by the author.

108. Rationale – Premises are the facts or evidence that support or lead to the conclusion.
Therefore, the above statement forms premise of the passage which leads to the conclusion. Hence,
option (A) is the correct answer.

109. Rationale – Options (C) cannot be reasonably inferred from the passage. Options (A) & (B) are
true and can be traced to various sentences in the passage and therefore, the correct Option is (D).

110. We will get from the first two statements T>H>C>F. Thus Statement 3 is false if the first two
statements are true.

111. Rationale – Option (A) is incorrect as the author‟s point on virtual classrooms does not get
affected even if the schools are planning to reopen next year. Option (B) & (D) support the claim of
the author. Therefore, the correct answer is Option (C).

112. Rationale – Though this may be connotation in problems relating to virtual classrooms, it is not
a point discussed by the author. Therefore, the correct answer is Option (C).

113. Rationale – Option (A) goes against the point made by the author in the last paragraph. Options
(C) & (B) cannot be found in the passage.
Hence, D
114. Rationale – The author brings to light the issue of distance learning and how a lot of screen
time is affecting the attention span of students. This is further debated upon from the side of
teachers as well who are facing issues in coming up with new and innovative learning tools.
Therefore, the correct answer is Option (B).
115. Rationale – Refer to “it is possible to ENHANCE NON-VISUAL STIMULI to allow students TO
TAKE A BREAK from their screens for a brief moment.”
The author presents the example of giving a break. Therefore, only Option (C) fits the bill.
116. From I: We get the following table

From the above table R is the person who teaches Geography. Obviously, T teaches Biology.
From II: We get the following chart:

From the chart given above, we can conclude that

But, still we do not know who among R and T teaches Biology.


117. Rationale – The author discusses the main cause of concern to be the displacement of human
writers by AI. However, the proposed solution is proper schooling of children to make them write in a
manner than machines cannot copy. This points towards Option (C) to be the correct answer as
without it, the argument would fail.
Hence, C
118. Rationale – Option (B) is immaterial to the discussion posed by the author. Options (C) & (D)
are relevant but only observations that do not affect the author‟s argument.
Therefore, the correct answer is Option (A).
119. Rationale – Option (B) is incorrect as the author doesn‟t propose to break „all‟ rules. Option (C)
is incorrect as it cannot be based out of the information in the passage. Therefore, the correct
answer is (A).

120. Rationale – The author suggest in the last paragraph that certain human factors cannot be
replaced or replicated by computers. This makes Option (D) the correct option.
121. Rationale – Options (A), (B) & (D) cannot be reasonably inferred from the passage. They relate
to the theme but not the topic of discussion. Therefore, the correct Option is (C).

122.

123. Refer to, "Insurance regulator IRDAI has permitted appropriation of all micro-insurance items
through point-of-sales (PoS), with a point to further increment insurance infiltration in the country.
Unmistakably the solitary target of the IRDAI to permit the conveyance of the micro insurance
strategies through the POS channel is the point of the authority to build the insurance inclusion in the
country and additionally micro insurance approaches are intended for in reverse individuals just,
thus, it is extraordinary to make them accessible through this channel of appropriation. Just
alternative (a) clarifies the right explanation of the choice taken by the IRDAI with respect to the
circulation of the micro insurance approaches. This makes alternative (a) the right choice among the
given choices.

124. Refer to, "The Insurance Regulatory and Development Authority of India (IRDAI) has made a
unique class of insurance strategies called micro-insurance arrangements to advance insurance
inclusion among financially weak areas of society. A micro-insurance strategy is a general or
disaster protection strategy with a whole guaranteed of Rs 50,000 or less." Explanation II is right
since it is referenced that micro insurance strategies are intended to be given to individuals from the
regressive segment of the general public. Explanation III is additionally right since it is likewise
obvious that micro insurance strategies are intended to surrender inclusion of to Rs 50000 or less. „I‟
is not right since it isn''t referenced anyplace if banks and insurance organizations are simply
authorized to sell the micro insurance arrangements in the country however the Point of Sales
people are additionally qualified and that is the fundamental theme of the passage. Thus, the two
Statements II and III are right. This settles on choice A the right choice among the given alternatives.

125. Proclamation III is right since it is given in the passage that the POS station of dissemination
will be utilized to give strategies which cross the reasonable furthest reaches of inclusion pertinent to
such channel by the IRDAI. Refer to, "On the way of managing instances of wellbeing/individual
mishap (Dad) approaches where entirety safeguarded crosses the breaking point indicated under
the POS rules, IRDAI said the supporting substance is permitted to perceive such arrangements as
being sourced by the POS and pay the expenses to the POS." Proclamation II is right since the facts
confirm that the insurance organizations made portrayals to get rid of the arrangement of expansion
of the prefix POS to all the insurance items sold through this channel of dissemination. Refer to,
"Guarantors made portrayals to change the standards that made it mandatory for each arrangement
sold through the ''Point of Sales People''s to be independently distinguished and pre-fixed by the
name ''POS''." Explanation I isn''t right since there is no reference in the passage that IRDAI can just
utilize the POS channel of dissemination to sell the insurance strategies and not the insurance
organizations. This is silly in light of the fact that IRDAI is the regulator and it doesn''t sell any
arrangement itself. In this way, articulations II and III are right as indicated by the data given in the
passage. This makes alternative (a) the right choice among the given choices.

126. Refer to, "The IRDAI (Assurance of Policyholder''s Advantage) Guidelines, 2017, under the
issue to be expressed throughout everyday life, general and health care coverage strategy, makes it
mandatory to give the subtleties of the individual associated with the sales cycle. "By temperance of
this prerequisite, the need to have the prefix ''PoS'' gets excess as the insurance strategy itself will
convey the subtleties of the individual selling such an approach," It is obvious from the above lines
that IRDAI has made it mandatory presently to add the insights about the point of sales people
selling the insurance arrangements in the country and therefore, presently there is no compelling
reason to add the prefix for the sake of the item. Just alternative (d) clarifies the genuine explanation
of the new rules gave by IRDAI with respect to the POS channel of selling the insurance
approaches. This makes choice (d) the right choice among the given choices.

127. Refer to, "Correspondingly, IRDAI said it has been seen that points of interest like higher
insurance infiltration, lower costs, expanded choice to clients, which would otherwise accumulate to
the policyholder by making micro-insurance items accessible through POS channel are being lost.
"Therefore, the Authority subsequent to checking on the position, thus permits all Micro Insurance
results of Life, General and Health care coverage to be conveyed through the POS additionally," the
round said." Presently, proclamation (a) is right since and Explanation (c) is likewise right. This
makes alternative (d) the right choice among the given choices.

128. Age 17 18 19 20 21 22
Person D/B F B/D C A E
Clearly, F is 18 years old.
129. Age 17 18 19 20 21 22
Person D/B F B/D C A E
There are 2 possible arrangements in terms of increasing age.
130. Rationale – Options (C) & (D) do not find proper references in the passage. Option (A) has not
been clearly mentioned by the author and may be true but is not the main theme of discussion of the
author. Therefore, the correct answer is Option (B).

131. Rationale – The author is critical of the film wonder Woman as being an anti-feminist tool which
may promote certain oppressive elements. However, the statements mentioned in Options (A) & (B)
suggest that it actually has made a very big impact in the field of gender empowerment and that
people view it as what it stands for and not literally. C is extraneous.
Hence, D
132. Rationale – Option (B), (C) & (D) have not been mentioned in the passage nor can they be
inferred from the information stated above. Therefore, the correct option is Option (A).

133. Rationale – The author claims that Gal Godot‟s portrayal of Wonder Woman sends out the
wrong message to women around the world as some of the expectations are wrong and oppressive.
Therefore, Option (B) is the correct answer.

134. Rationale – Only Option (B) covers the main theme discussed by the author and pertains to the
text supplied above.

135. Line X is perpendicular to line Y and line V. Hence Y and V are parallel.
Line U is perpendicular to lines V and W. Hence V and W are parallel.
Hence, Lines Y, V and W are parallel.

QUANTITATIVE TECHNIQUES

136.

From the graph and it can be easily seen that „Diwans‟ has the lowest average cost price.

137.

From the graph it can be observed that „Carpets‟ has the highest average selling price.
138.

The article which earn the highest profit is Carpets.


139.

Jars have the lowest average profit.


140.

Two articles Diwans and Carpets have profit more than 50%.

141. Number of families having daily income which are at least ₹7000 = 350 + 300 + 600 + 350 =
1600
142. Number of families having daily income less than ₹4000 = 350 + 300 + 600 = 1250
143. Number of families whose daily income not less than ₹2000 but less than ₹6000= 300 + 600 +
250 + 400 = 1550
144. The total number of families surveyed = 350 + 300 + 600 + 250 + 400 + 500 + 350 + 300 + 600
+ 350 = 4000
Number of families having daily income which are at least Rs. 6000 = 500 + 350 +300+ 600+350 =
2100
Required % = 2100/4000x100% = 52.50%
145. Required avg. = (500 x 400 + 6000 x 500 + 7000 x 350 + 8000 x 300)/400 + 500 + 350 + 300
= (2000000 + 3000000 + 2450000 + 2400000)/1550
= 9850000/1550=6354.8 = Rs. 6355 approx
146.
147.

148.

149.

150.
Legal Edge 25 Mock

Answers & Details

ENGLISH

1. The correct answer is C, as a thorough reading of the passage leads us to the answer. The author
goes to the lengths of talking about the various ―wards‖ taken in by the Queen to further the talk
about the prevalent racism in the institution. This discussion becomes overly relevant in the light of
Meghan‘s interview, where she claims that she has been subjected to racism by the Royal family.
Thus, the bigger picture points to option C
Option A is not the right answer, as it is just mentioned in passing by the author just to convey the
changes taking place in Great Britain in mid-19th century and is used by the author just to lay the
ground for the main theme of the passage, i.e., option C
Option B, while a true fact mentioned in the passage, is not the right answer as it has been
mentioned just in the passing, like option A, and cannot be assumed to be the main theme of the
passage.
Option D is correct and the second most suited answer as per the passage. However, as option C is
more suited, thus D is not the right answer.
2. Option D draws the most appropriate similarity between the two and hits the theme of the passage
which is the prevalent racism in the Riyal Family. Thus, the correct answer is D
Option A, B and C are correct, however, do not point to the similarity on basis of race, which is the
theme of the passage. Thus, they are not the right answers.
3. Meghan became a target for the hate and racism as being a person of color, she married Prince
Harry.
Option A is true, but not the complete reason for the backlash. Only being a person of color is not
the reason. Option B is a furtherance of option A and is the more suited answer.
Option C and D cannot be inferred from the passage and are thus not the right answers.
4. The following line reflects the same: ―And while Atwal says that Victoria did take a personal
interest in all of these young children from across the Empire and took them under her wing, ―they
essentially were put up as poster children in many respects.‖
―It tries to assimilate these people, because ultimately, it doesn‘t care about those people to the
same degree as it does about the crown,‖ says Atwal. ―And if the interests of the crown are being
messed with, then it doesn‘t really matter what collateral damage happens to the lives of those
people that are being assimilated. They are expendable.‖
A combined reading of the above two lines from the passage gives us the idea that while the queen
did care about the wards that she adopted, primacy was always given to the Crown and the wards
were cared for and controlled in a manner fitting and benefitting the crown. Thus, the correct answer
is D
A, B and C are not the answers as they cannot be inferred from the passage. Further, all the options
are mutually exclusive, thus as D is the most suited answer, none of the other options can be
correct.
5. The author has written the passage with the aim to show how deeply embedded racism is in the
Royal family and how the situation hasn‘t changed much ever today. Thus, the correct answer is B
A, C are definitely mentioned in the passage and form an integral part of the author‘s views.
However, they cannot be said to be the main aim of the passage as option B summarizes it best.
Option D cannot be inferred from the passage and thus is not the right answer.
6. The following line reflects the same: ――Victoria doesn‘t allow Gouramma to see her father again,
and Gouramma eventually loses the ability to speak Hindi, her mother tongue. It‘s really cruel,‖ says
Atwal. ―The lens through which Gouramma is seen is through this colonial mindset.‖ This line shows
that Gouramma was made to cut ties with her own culture and tradition, so much so that she forgot
her own native tongue. Thus, the correct answer is B
7. The following line reflects the same:
―The most important architects of contemporary legal positivism are the Austrian jurist Hans Kelsen
and the two dominating figures in the analytic philosophy of law, H.L.A Hart and Joseph Raz, among
whom there are clear lines of influence, but also important contrasts.‖ Thus, the correct answer is D
Option A is incorrect as can be seen from the following lines: ―Legal positivism has a long history
and a broad influence. It has antecedents in ancient political philosophy and is discussed in
mediaeval legal and political thought. The modern doctrine, however, owes little to these forbears‖.
The author here states that while legal positivism finds its roots in mediaeval legal and political
thought, the present day legal positivism is starkly different from it.
Option B and C are incorrect as the author in the second paragraph says that while its most
important roots lie in the political philosophies of Hobbes and Hume, Bentham and Austin, their
theory of Legal positivism had lost influence by the mid-twentieth century.
8. A clear reading of the passage leads us to the answer. The following lines from the passage
reflect that legal positivism is a theory of what law is.
―For much of the next century an amalgam of their views, according to which law is the command of
a sovereign backed by force, dominated English philosophical reflection about law.‖
―Although they disagree on many other points, these writers all acknowledge that law is essentially a
matter of social fact.‖
―Lawyers often use ―positivist‖ abusively, to condemn a formalistic doctrine according to which law is
always clear and, however pointless or wrong, is to be rigorously applied by officials and obeyed by
subjects.‖ Thus, the correct answer is C
Option A is incorrect, as that is not legal positivism, it is the definition of law propounded by Austin.
Option B and D are incorrect as they cannot be inferred from the passage.
9. A clear reading of the passage takes us to the right answer. The author starts by tracing the
history of what legal positivism has been throughout ages and what it stands for today. Thus, the
correct answer is A
Option B is incorrect as the author mentions the importance of legal positivism only in the passing, to
show how legal positivism has evolved over the years. Option B only finds home in the fourth
paragraph and cannot be said to be the main aim of the author.
Option C is incorrect, as while the author does trace the history of legal positivism, it is done so in
order to define what legal positivism is, i.e., in furtherance of option (a)
Options D is incorrect as they cannot be inferred from the passage.
10. Option D is incorrect as the author nowhere makes a comparison between natural law and legal
positivism. The author only mentions that both of them are complimentary in the following line:
―Hence, many traditional ―natural law‖ moral doctrines—including the belief in a universal, objective
morality grounded in human nature—do not contradict legal positivism.‖ Thus, the correct answer is
D
Option A finds mention in the first and the second paragraph of the passage as they talk in depth
about the evolution of legal positivism. Thus, A is not the answer.
Option B finds mention in the second and third paragraphs of the passage which talks about the
main architects of legal positivism like Bentham, Hugh, Raz, Hart etc. in the second and the third
paragraphs. Thus, B is not the answer.
Option C is incorrect as it finds mention in the following lines:
―Legal positivism‘s importance, however, is not confined to the philosophy of law. It can be seen
throughout social theory, particularly in the works of Marx, Weber, and Durkheim, and also among
many lawyers, including the American ―legal realists‖ and most contemporary feminist scholars.‖
Thus, option C is not the answer.
11. The following lines reflect the same: ―Their discomfort is sometimes the product of confusion.
Lawyers often use ―positivist‖ abusively, to condemn a formalistic doctrine according to which law is
always clear and, however pointless or wrong, is to be rigorously applied by officials and obeyed by
subjects. It is doubtful that anyone ever held this view, but it is in any case false and has nothing to
do with legal positivism.‖ The author here states that legal positivism is misunderstood as being a
rigid theory, when in actuality it is anything but. This prevalent notion is just a result of misinformation
about the theory. Thus, the correct answer is A
Options B, C, D are all a result of the misinformation about and the wrong perception of legal
positivism, i.e., a subsequent result of option A, thus are all incorrect options.
12. None of the above options describe legal positivism as it stands today. Thus, the correct answer
is d
Option A and C are incorrect as they are negated in the following lines: ―For much of the next
century an amalgam of their views, according to which law is the command of a sovereign backed by
force, dominated English philosophical reflection about law. By the mid-twentieth century, however,
this account had lost its influence among working legal philosophers.‖ The author here says that
while the definition which was an amalgamation of the theories of all the authors, it soon lost
relevance thereafter. Thus, A and C are incorrect.
Option B is incorrect as nowhere can it be inferred from that legal positivism implies this notion.
13. The correct option is (c). The entire passage revolves around the main topic of explaining the
importance of paying salaries to women for housework in order to provide them with respect,
recognition, dignity and empowerment.
14. The correct answer is option (b). Expository writing, as the title suggests, is predicated on
exposition, or the description and explanation of a particular idea. Topics cover pretty much the
entire gamut of human experience, from inventions to nature, emotions to politics, family to hobbies
and more.
15. The correct answer is option (a). Statement 1 is the incorrect one as the first paragraph clearly
states that ‗The ―Statement of the International Feminist Collective‖ issued in 1972 in Italy, rejected a
separation between unwaged work in the home and waged work in the factory, pronouncing
housework as a critical terrain in the class struggle against capitalism.‘
16. The correct answer is option (c). The meaning of the word ‗imperialism‘ is ‗a policy of extending a
country''s power and influence through colonization, use of military force, or other means.‘
17. The correct answer is option (a). The correct meaning of the word ‗communism‘ is ‗an economic
and political system in which a country''s trade and industry are controlled by private owners for
profit, rather than by the state.‘
18. vocab-in-context question
Belligerent is defined as a state, nation or military personnel at war or ready to fight, similar to
China‘s as per the passage. Thus, the only correct answer is (a)
B, C and D are not suitable options and are thus incorrect.
19. Specific question
The following line indicates the same:
―Retaining what each side has and making it the basis for settling the international border is the best
solution to the India-China confrontation.‖ The author says that as per the proposal, both the
countries will be ―retaining‖ what they already possess. Thus, the correct answer is (b)
A, C and D cannot be inferred from the passage, thus are incorrect.
20. function question
Pipe dream means an idea or plan that is impossible or very unlikely to happen. Thus, the correct
answer is (b)
All the options are mutually exclusive and thus, options A, C and D are incorrect.
21. Specific question
The following line indicates the same:
―Narendra Modi alone has the capacity to sell an LAC deal in India, as no one will question his
nationalist credentials.‖A nationalist is a person who strongly identifies with their own nation and
vigorously supports its interests, especially to the exclusion or detriment of the interests of other
nations. Thus, the correct answer is (c)
Options A and B cannot be inferred from the passage, thus are incorrect.
22. Apostle means a vigorous and pioneering advocate or supporter of a particular policy, idea, or
cause. Thus, the correct answer is (c)
As C is the most suitable option, the others cannot be correct.
23. vocab-in-context question
Irony is a state of affairs or an event that seems deliberately contrary to what one expects and is
often wryly amusing as a result. The irony here is that Gandhi propounded peace, which is the
opposite of the draconian laws prevalent today. The first paragraph showcases this irony. Thus, the
correct answer is (b)
As the answer can only be one of the above, B is the most suited answer and A, C D are incorrect.
24. Tone question
As per the passage, Gandhi was against sedition laws. Further, UAPA and NSA is being used to
crush dissenters, thereby crushing there right to peaceful protest. More so, CAA is only being
mentioned in the passage, without much detail being given about it. Thus, the correct answer is (b)
As all the options are mutually exclusive, A, C and D are incorrect.
25. vocab-in-context question
To turn in one‘s grave means to (hypothetically) show enormous anger, disfavor, or regret for
something that happens after one has died. Thus, the best suited idiom here is A and B, C, D are
incorrect.
26. universal question
The first para indicates the same and sets the tone for the passage. Thus, the correct answer is (b)
While A, C and D are slightly touched upon by the author, it is done so to high light the main theme,
which is (b)
27. Universal question
The theme of the passage is the new SMA, which has regressive clauses intruding upon Individual‘s
right to marry. Thus, the correct answer is (c)
A and D are not mentioned anywhere in the passage and thus cannot be the theme of the passage.
B is slightly touched upon by the author to more properly focus upon option (c) thus, B is not the
answer.
But SC ruling privacy as a fundamental right renders its ―objection‖ provisions [X] and [Y].
28. Vocab-in-context question
Archaic means very old fashioned and thus, is the best replacement for X.
The author says that the notice clause goes against the Supreme‘s court‘s ruling, which makes it
illegal. Thus, it is the best replacement for Y.
29. specific question
The following lines indicate the same:
―For decades, the Act has been upheld as a template for a uniform civil code. Men and women
opting for a secular marriage or escaping the clutches of arch-conservative families and religious
personal laws find refuge under the Act.‖
Options A and C are not mentioned anywhere in the passage and are not the answers.
Option D might sound right, but is actually incorrect as UCC not only aims to sanctify interfaith
marriages, but same faith marriages as well.
Thus, the correct answer is (b)
30. specific question
Option A is not directly mentioned in the passage, but can be inferred. The author says that the
notice provision has too many downfalls and gives excess powers to the marriage officer, who is a
statutory authority.
Option B is reflected in the following lines: ―Some provisions of the Special Marriage Act, 1954, that
violate the fundamental right to privacy could come under the judicial scanner, which is most
welcome‖
As per the passage, C is not mentioned anywhere. Thus, the correct answer is (c)
GENERAL KNOWLEDGE

31. GDP had shrunk in the first two quarters by 24.4% and 7.3% as per revised data, amid the
COVID-19 pandemic and lockdowns, marking a technical recession. The NSO has also revised its
advance national income estimates for FY21 to project an 8% decline in GDP, compared with the
4% growth seen in FY20. The NSO had earlier estimated a 7.7% shrinkage for FY21.
V-shaped recovery is witnessed when an economy suffers a sharp economic decline followed by a
strong recovery. "2020 turned out to be a year in which everything changed. The year 2021 has
commenced with countries across the world in a massive vaccination drive," the RBI report
mentioned.
A technical recession is when a country faces a continuous decline for two consecutive quarters in
the GDP.
The Table shows India‘s absolute Gross Domestic Product (GDP) is likely to struggle to even come
back to the 2019-20 level by 2023-24. India is likely to end up with an ―elongated U-shape‖ recovery
due to the weakness of the economy going into the Covid crisis as well as the inadequate fiscal
stimulus measure taken by the government.
The economists are unanimous that in the current financial year 2020-21, India‘s economy will
contract. According to the World Bank‘s South Asia Economic Focus report, India‘s growth is likely to
remain at 1.5-2.8% in 2020-21 which is the slowest since 1991 economic reforms. Many economists
are also of the opinion that after hitting the bottom this year, the Indian economy will start its
recovery in the next financial year (2021-22).
India is a member of the International Monetary Fund (IMF) since its inception. The Finance Minister
is an ex-officio member of the IMF Board of Governors, and the Governor of the Reserve Bank of
India is India''s Alternate Governor.
32. GDP had shrunk in the first two quarters by 24.4% and 7.3% as per revised data, amid the
COVID-19 pandemic and lockdowns, marking a technical recession. The NSO has also revised its
advance national income estimates for FY21 to project an 8% decline in GDP, compared with the
4% growth seen in FY20. The NSO had earlier estimated a 7.7% shrinkage for FY21.
V-shaped recovery is witnessed when an economy suffers a sharp economic decline followed by a
strong recovery. "2020 turned out to be a year in which everything changed. The year 2021 has
commenced with countries across the world in a massive vaccination drive," the RBI report
mentioned.
A technical recession is when a country faces a continuous decline for two consecutive quarters in
the GDP.
The Table shows India‘s absolute Gross Domestic Product (GDP) is likely to struggle to even come
back to the 2019-20 level by 2023-24. India is likely to end up with an ―elongated U-shape‖ recovery
due to the weakness of the economy going into the Covid crisis as well as the inadequate fiscal
stimulus measure taken by the government.
The economists are unanimous that in the current financial year 2020-21, India‘s economy will
contract. According to the World Bank‘s South Asia Economic Focus report, India‘s growth is likely to
remain at 1.5-2.8% in 2020-21 which is the slowest since 1991 economic reforms. Many economists
are also of the opinion that after hitting the bottom this year, the Indian economy will start its
recovery in the next financial year (2021-22).
India is a member of the International Monetary Fund (IMF) since its inception. The Finance Minister
is an ex-officio member of the IMF Board of Governors, and the Governor of the Reserve Bank of
India is India''s Alternate Governor.
33. GDP had shrunk in the first two quarters by 24.4% and 7.3% as per revised data, amid the
COVID-19 pandemic and lockdowns, marking a technical recession. The NSO has also revised its
advance national income estimates for FY21 to project an 8% decline in GDP, compared with the
4% growth seen in FY20. The NSO had earlier estimated a 7.7% shrinkage for FY21.
V-shaped recovery is witnessed when an economy suffers a sharp economic decline followed by a
strong recovery. "2020 turned out to be a year in which everything changed. The year 2021 has
commenced with countries across the world in a massive vaccination drive," the RBI report
mentioned.
A technical recession is when a country faces a continuous decline for two consecutive quarters in
the GDP.
The Table shows India‘s absolute Gross Domestic Product (GDP) is likely to struggle to even come
back to the 2019-20 level by 2023-24. India is likely to end up with an ―elongated U-shape‖ recovery
due to the weakness of the economy going into the Covid crisis as well as the inadequate fiscal
stimulus measure taken by the government.
The economists are unanimous that in the current financial year 2020-21, India‘s economy will
contract. According to the World Bank‘s South Asia Economic Focus report, India‘s growth is likely to
remain at 1.5-2.8% in 2020-21 which is the slowest since 1991 economic reforms. Many economists
are also of the opinion that after hitting the bottom this year, the Indian economy will start its
recovery in the next financial year (2021-22).
India is a member of the International Monetary Fund (IMF) since its inception. The Finance Minister
is an ex-officio member of the IMF Board of Governors, and the Governor of the Reserve Bank of
India is India''s Alternate Governor.
34. GDP had shrunk in the first two quarters by 24.4% and 7.3% as per revised data, amid the
COVID-19 pandemic and lockdowns, marking a technical recession. The NSO has also revised its
advance national income estimates for FY21 to project an 8% decline in GDP, compared with the
4% growth seen in FY20. The NSO had earlier estimated a 7.7% shrinkage for FY21.
V-shaped recovery is witnessed when an economy suffers a sharp economic decline followed by a
strong recovery. "2020 turned out to be a year in which everything changed. The year 2021 has
commenced with countries across the world in a massive vaccination drive," the RBI report
mentioned.
A technical recession is when a country faces a continuous decline for two consecutive quarters in
the GDP.
The Table shows India‘s absolute Gross Domestic Product (GDP) is likely to struggle to even come
back to the 2019-20 level by 2023-24. India is likely to end up with an ―elongated U-shape‖ recovery
due to the weakness of the economy going into the Covid crisis as well as the inadequate fiscal
stimulus measure taken by the government.
The economists are unanimous that in the current financial year 2020-21, India‘s economy will
contract. According to the World Bank‘s South Asia Economic Focus report, India‘s growth is likely to
remain at 1.5-2.8% in 2020-21 which is the slowest since 1991 economic reforms. Many economists
are also of the opinion that after hitting the bottom this year, the Indian economy will start its
recovery in the next financial year (2021-22).
India is a member of the International Monetary Fund (IMF) since its inception. The Finance Minister
is an ex-officio member of the IMF Board of Governors, and the Governor of the Reserve Bank of
India is India''s Alternate Governor.
35. GDP had shrunk in the first two quarters by 24.4% and 7.3% as per revised data, amid the
COVID-19 pandemic and lockdowns, marking a technical recession. The NSO has also revised its
advance national income estimates for FY21 to project an 8% decline in GDP, compared with the
4% growth seen in FY20. The NSO had earlier estimated a 7.7% shrinkage for FY21.
V-shaped recovery is witnessed when an economy suffers a sharp economic decline followed by a
strong recovery. "2020 turned out to be a year in which everything changed. The year 2021 has
commenced with countries across the world in a massive vaccination drive," the RBI report
mentioned.
A technical recession is when a country faces a continuous decline for two consecutive quarters in
the GDP.
The Table shows India‘s absolute Gross Domestic Product (GDP) is likely to struggle to even come
back to the 2019-20 level by 2023-24. India is likely to end up with an ―elongated U-shape‖ recovery
due to the weakness of the economy going into the Covid crisis as well as the inadequate fiscal
stimulus measure taken by the government.
The economists are unanimous that in the current financial year 2020-21, India‘s economy will
contract. According to the World Bank‘s South Asia Economic Focus report, India‘s growth is likely to
remain at 1.5-2.8% in 2020-21 which is the slowest since 1991 economic reforms. Many economists
are also of the opinion that after hitting the bottom this year, the Indian economy will start its
recovery in the next financial year (2021-22).
India is a member of the International Monetary Fund (IMF) since its inception. The Finance Minister
is an ex-officio member of the IMF Board of Governors, and the Governor of the Reserve Bank of
India is India''s Alternate Governor.
36. GDP had shrunk in the first two quarters by 24.4% and 7.3% as per revised data, amid the
COVID-19 pandemic and lockdowns, marking a technical recession. The NSO has also revised its
advance national income estimates for FY21 to project an 8% decline in GDP, compared with the
4% growth seen in FY20. The NSO had earlier estimated a 7.7% shrinkage for FY21.
V-shaped recovery is witnessed when an economy suffers a sharp economic decline followed by a
strong recovery. "2020 turned out to be a year in which everything changed. The year 2021 has
commenced with countries across the world in a massive vaccination drive," the RBI report
mentioned.
A technical recession is when a country faces a continuous decline for two consecutive quarters in
the GDP.
The Table shows India‘s absolute Gross Domestic Product (GDP) is likely to struggle to even come
back to the 2019-20 level by 2023-24. India is likely to end up with an ―elongated U-shape‖ recovery
due to the weakness of the economy going into the Covid crisis as well as the inadequate fiscal
stimulus measure taken by the government.
The economists are unanimous that in the current financial year 2020-21, India‘s economy will
contract. According to the World Bank‘s South Asia Economic Focus report, India‘s growth is likely to
remain at 1.5-2.8% in 2020-21 which is the slowest since 1991 economic reforms. Many economists
are also of the opinion that after hitting the bottom this year, the Indian economy will start its
recovery in the next financial year (2021-22).
India is a member of the International Monetary Fund (IMF) since its inception. The Finance Minister
is an ex-officio member of the IMF Board of Governors, and the Governor of the Reserve Bank of
India is India''s Alternate Governor.
37. Additional Due Diligence to Be Followed by Significant Social Media Intermediary:
Appoint a Chief Compliance Officer who shall be responsible for ensuring compliance with the Act
and Rules. Such a person should be a resident in India.
Appoint a Nodal Contact Person for 24x7 coordination with law enforcement agencies. Such a
person shall be a resident in India.
Appoint a Resident Grievance Officer who shall perform the functions mentioned under Grievance
Redressal Mechanism. Such a person shall be a resident in India.
Publish a monthly compliance report mentioning the details of complaints received and action taken
on the complaints as well as details of contents removed proactively by the significant social media
intermediary.
Significant social media intermediaries providing services primarily in the nature of messaging shall
enable identification of the first originator of the information that is required only for the purposes of
prevention, detection, investigation, prosecution or punishment of an offence related to sovereignty
and integrity of India, the security of the State, friendly relations with foreign States, or public order or
of incitement to an offence relating to the above or in relation with rape, sexually explicit material or
child sexual abuse material punishable with imprisonment for a term of not less than five years.
Intermediary shall not be required to disclose the contents of any message or any other information
to the first originator.
Social media companies with more than 50 lakh registered users will be considered ''significant
social media intermediaries'', as per the new norms under the Intermediary Guidelines and Digital
Media Ethics Code of 2021, the Central government has clarified.
Notified under section 87 of Information Technology Act, these Rules empower the Ministry of
Information and Broadcasting to implement Part-III of the Rules.
Self-regulation by the Publisher: Publisher shall appoint a Grievance Redressal Officer based in
India who shall be responsible for the redressal of grievances received by it. The officer shall take
decision on every grievance received by it within 15 days. Self-Regulatory Body: There may be one
or more self-regulatory bodies of publishers. Such a body shall be headed by a retired judge of the
Supreme Court, a High Court or independent eminent person and have not more than six members.
Such a body will have to register with the Ministry of Information and Broadcasting. This body will
oversee the adherence by the publisher to the Code of Ethics and address grievances that have not
been resolved by the publisher within 15 days.
In case, due diligence is not followed by the intermediary, safe harbour provisions will not apply to
them. The safe harbour provisions have been defined under Section 79 of the IT Act and protect
social media intermediaries by giving them immunity from legal prosecution for any content posted
on their platforms.
Oversight Mechanism: Ministry of Information and Broadcasting shall formulate an oversight
mechanism. It shall publish a charter for self-regulating bodies, including Codes of Practices. It shall
establish an Inter-Departmental Committee for hearing grievances. The IIIrd point hasn‘t been
mentioned. The publishers have been asked to set up a self-regulatory structure.
38. Additional Due Diligence to Be Followed by Significant Social Media Intermediary:
Appoint a Chief Compliance Officer who shall be responsible for ensuring compliance with the Act
and Rules. Such a person should be a resident in India.
Appoint a Nodal Contact Person for 24x7 coordination with law enforcement agencies. Such a
person shall be a resident in India.
Appoint a Resident Grievance Officer who shall perform the functions mentioned under Grievance
Redressal Mechanism. Such a person shall be a resident in India.
Publish a monthly compliance report mentioning the details of complaints received and action taken
on the complaints as well as details of contents removed proactively by the significant social media
intermediary.
Significant social media intermediaries providing services primarily in the nature of messaging shall
enable identification of the first originator of the information that is required only for the purposes of
prevention, detection, investigation, prosecution or punishment of an offence related to sovereignty
and integrity of India, the security of the State, friendly relations with foreign States, or public order or
of incitement to an offence relating to the above or in relation with rape, sexually explicit material or
child sexual abuse material punishable with imprisonment for a term of not less than five years.
Intermediary shall not be required to disclose the contents of any message or any other information
to the first originator.
Social media companies with more than 50 lakh registered users will be considered ''significant
social media intermediaries'', as per the new norms under the Intermediary Guidelines and Digital
Media Ethics Code of 2021, the Central government has clarified.
Notified under section 87 of Information Technology Act, these Rules empower the Ministry of
Information and Broadcasting to implement Part-III of the Rules.
Self-regulation by the Publisher: Publisher shall appoint a Grievance Redressal Officer based in
India who shall be responsible for the redressal of grievances received by it. The officer shall take
decision on every grievance received by it within 15 days. Self-Regulatory Body: There may be one
or more self-regulatory bodies of publishers. Such a body shall be headed by a retired judge of the
Supreme Court, a High Court or independent eminent person and have not more than six members.
Such a body will have to register with the Ministry of Information and Broadcasting. This body will
oversee the adherence by the publisher to the Code of Ethics and address grievances that have not
been resolved by the publisher within 15 days.
In case, due diligence is not followed by the intermediary, safe harbour provisions will not apply to
them. The safe harbour provisions have been defined under Section 79 of the IT Act and protect
social media intermediaries by giving them immunity from legal prosecution for any content posted
on their platforms.
Oversight Mechanism: Ministry of Information and Broadcasting shall formulate an oversight
mechanism. It shall publish a charter for self-regulating bodies, including Codes of Practices. It shall
establish an Inter-Departmental Committee for hearing grievances. The IIIrd point hasn‘t been
mentioned. The publishers have been asked to set up a self-regulatory structure.
39. Additional Due Diligence to Be Followed by Significant Social Media Intermediary:
Appoint a Chief Compliance Officer who shall be responsible for ensuring compliance with the Act
and Rules. Such a person should be a resident in India.
Appoint a Nodal Contact Person for 24x7 coordination with law enforcement agencies. Such a
person shall be a resident in India.
Appoint a Resident Grievance Officer who shall perform the functions mentioned under Grievance
Redressal Mechanism. Such a person shall be a resident in India.
Publish a monthly compliance report mentioning the details of complaints received and action taken
on the complaints as well as details of contents removed proactively by the significant social media
intermediary.
Significant social media intermediaries providing services primarily in the nature of messaging shall
enable identification of the first originator of the information that is required only for the purposes of
prevention, detection, investigation, prosecution or punishment of an offence related to sovereignty
and integrity of India, the security of the State, friendly relations with foreign States, or public order or
of incitement to an offence relating to the above or in relation with rape, sexually explicit material or
child sexual abuse material punishable with imprisonment for a term of not less than five years.
Intermediary shall not be required to disclose the contents of any message or any other information
to the first originator.
Social media companies with more than 50 lakh registered users will be considered ''significant
social media intermediaries'', as per the new norms under the Intermediary Guidelines and Digital
Media Ethics Code of 2021, the Central government has clarified.
Notified under section 87 of Information Technology Act, these Rules empower the Ministry of
Information and Broadcasting to implement Part-III of the Rules.
Self-regulation by the Publisher: Publisher shall appoint a Grievance Redressal Officer based in
India who shall be responsible for the redressal of grievances received by it. The officer shall take
decision on every grievance received by it within 15 days. Self-Regulatory Body: There may be one
or more self-regulatory bodies of publishers. Such a body shall be headed by a retired judge of the
Supreme Court, a High Court or independent eminent person and have not more than six members.
Such a body will have to register with the Ministry of Information and Broadcasting. This body will
oversee the adherence by the publisher to the Code of Ethics and address grievances that have not
been resolved by the publisher within 15 days.
In case, due diligence is not followed by the intermediary, safe harbour provisions will not apply to
them. The safe harbour provisions have been defined under Section 79 of the IT Act and protect
social media intermediaries by giving them immunity from legal prosecution for any content posted
on their platforms.
Oversight Mechanism: Ministry of Information and Broadcasting shall formulate an oversight
mechanism. It shall publish a charter for self-regulating bodies, including Codes of Practices. It shall
establish an Inter-Departmental Committee for hearing grievances. The IIIrd point hasn‘t been
mentioned. The publishers have been asked to set up a self-regulatory structure.
40. Additional Due Diligence to Be Followed by Significant Social Media Intermediary:
Appoint a Chief Compliance Officer who shall be responsible for ensuring compliance with the Act
and Rules. Such a person should be a resident in India.
Appoint a Nodal Contact Person for 24x7 coordination with law enforcement agencies. Such a
person shall be a resident in India.
Appoint a Resident Grievance Officer who shall perform the functions mentioned under Grievance
Redressal Mechanism. Such a person shall be a resident in India.
Publish a monthly compliance report mentioning the details of complaints received and action taken
on the complaints as well as details of contents removed proactively by the significant social media
intermediary.
Significant social media intermediaries providing services primarily in the nature of messaging shall
enable identification of the first originator of the information that is required only for the purposes of
prevention, detection, investigation, prosecution or punishment of an offence related to sovereignty
and integrity of India, the security of the State, friendly relations with foreign States, or public order or
of incitement to an offence relating to the above or in relation with rape, sexually explicit material or
child sexual abuse material punishable with imprisonment for a term of not less than five years.
Intermediary shall not be required to disclose the contents of any message or any other information
to the first originator.
Social media companies with more than 50 lakh registered users will be considered ''significant
social media intermediaries'', as per the new norms under the Intermediary Guidelines and Digital
Media Ethics Code of 2021, the Central government has clarified.
Notified under section 87 of Information Technology Act, these Rules empower the Ministry of
Information and Broadcasting to implement Part-III of the Rules.
Self-regulation by the Publisher: Publisher shall appoint a Grievance Redressal Officer based in
India who shall be responsible for the redressal of grievances received by it. The officer shall take
decision on every grievance received by it within 15 days. Self-Regulatory Body: There may be one
or more self-regulatory bodies of publishers. Such a body shall be headed by a retired judge of the
Supreme Court, a High Court or independent eminent person and have not more than six members.
Such a body will have to register with the Ministry of Information and Broadcasting. This body will
oversee the adherence by the publisher to the Code of Ethics and address grievances that have not
been resolved by the publisher within 15 days.
In case, due diligence is not followed by the intermediary, safe harbour provisions will not apply to
them. The safe harbour provisions have been defined under Section 79 of the IT Act and protect
social media intermediaries by giving them immunity from legal prosecution for any content posted
on their platforms.
Oversight Mechanism: Ministry of Information and Broadcasting shall formulate an oversight
mechanism. It shall publish a charter for self-regulating bodies, including Codes of Practices. It shall
establish an Inter-Departmental Committee for hearing grievances. The IIIrd point hasn‘t been
mentioned. The publishers have been asked to set up a self-regulatory structure.
41. Additional Due Diligence to Be Followed by Significant Social Media Intermediary:
Appoint a Chief Compliance Officer who shall be responsible for ensuring compliance with the Act
and Rules. Such a person should be a resident in India.
Appoint a Nodal Contact Person for 24x7 coordination with law enforcement agencies. Such a
person shall be a resident in India.
Appoint a Resident Grievance Officer who shall perform the functions mentioned under Grievance
Redressal Mechanism. Such a person shall be a resident in India.
Publish a monthly compliance report mentioning the details of complaints received and action taken
on the complaints as well as details of contents removed proactively by the significant social media
intermediary.
Significant social media intermediaries providing services primarily in the nature of messaging shall
enable identification of the first originator of the information that is required only for the purposes of
prevention, detection, investigation, prosecution or punishment of an offence related to sovereignty
and integrity of India, the security of the State, friendly relations with foreign States, or public order or
of incitement to an offence relating to the above or in relation with rape, sexually explicit material or
child sexual abuse material punishable with imprisonment for a term of not less than five years.
Intermediary shall not be required to disclose the contents of any message or any other information
to the first originator.
Social media companies with more than 50 lakh registered users will be considered ''significant
social media intermediaries'', as per the new norms under the Intermediary Guidelines and Digital
Media Ethics Code of 2021, the Central government has clarified.
Notified under section 87 of Information Technology Act, these Rules empower the Ministry of
Information and Broadcasting to implement Part-III of the Rules.
Self-regulation by the Publisher: Publisher shall appoint a Grievance Redressal Officer based in
India who shall be responsible for the redressal of grievances received by it. The officer shall take
decision on every grievance received by it within 15 days. Self-Regulatory Body: There may be one
or more self-regulatory bodies of publishers. Such a body shall be headed by a retired judge of the
Supreme Court, a High Court or independent eminent person and have not more than six members.
Such a body will have to register with the Ministry of Information and Broadcasting. This body will
oversee the adherence by the publisher to the Code of Ethics and address grievances that have not
been resolved by the publisher within 15 days.
In case, due diligence is not followed by the intermediary, safe harbour provisions will not apply to
them. The safe harbour provisions have been defined under Section 79 of the IT Act and protect
social media intermediaries by giving them immunity from legal prosecution for any content posted
on their platforms.
Oversight Mechanism: Ministry of Information and Broadcasting shall formulate an oversight
mechanism. It shall publish a charter for self-regulating bodies, including Codes of Practices. It shall
establish an Inter-Departmental Committee for hearing grievances. The IIIrd point hasn‘t been
mentioned. The publishers have been asked to set up a self-regulatory structure.
42. Additional Due Diligence to Be Followed by Significant Social Media Intermediary:
Appoint a Chief Compliance Officer who shall be responsible for ensuring compliance with the Act
and Rules. Such a person should be a resident in India.
Appoint a Nodal Contact Person for 24x7 coordination with law enforcement agencies. Such a
person shall be a resident in India.
Appoint a Resident Grievance Officer who shall perform the functions mentioned under Grievance
Redressal Mechanism. Such a person shall be a resident in India.
Publish a monthly compliance report mentioning the details of complaints received and action taken
on the complaints as well as details of contents removed proactively by the significant social media
intermediary.
Significant social media intermediaries providing services primarily in the nature of messaging shall
enable identification of the first originator of the information that is required only for the purposes of
prevention, detection, investigation, prosecution or punishment of an offence related to sovereignty
and integrity of India, the security of the State, friendly relations with foreign States, or public order or
of incitement to an offence relating to the above or in relation with rape, sexually explicit material or
child sexual abuse material punishable with imprisonment for a term of not less than five years.
Intermediary shall not be required to disclose the contents of any message or any other information
to the first originator.
Social media companies with more than 50 lakh registered users will be considered ''significant
social media intermediaries'', as per the new norms under the Intermediary Guidelines and Digital
Media Ethics Code of 2021, the Central government has clarified.
Notified under section 87 of Information Technology Act, these Rules empower the Ministry of
Information and Broadcasting to implement Part-III of the Rules.
Self-regulation by the Publisher: Publisher shall appoint a Grievance Redressal Officer based in
India who shall be responsible for the redressal of grievances received by it. The officer shall take
decision on every grievance received by it within 15 days. Self-Regulatory Body: There may be one
or more self-regulatory bodies of publishers. Such a body shall be headed by a retired judge of the
Supreme Court, a High Court or independent eminent person and have not more than six members.
Such a body will have to register with the Ministry of Information and Broadcasting. This body will
oversee the adherence by the publisher to the Code of Ethics and address grievances that have not
been resolved by the publisher within 15 days.
In case, due diligence is not followed by the intermediary, safe harbour provisions will not apply to
them. The safe harbour provisions have been defined under Section 79 of the IT Act and protect
social media intermediaries by giving them immunity from legal prosecution for any content posted
on their platforms.
Oversight Mechanism: Ministry of Information and Broadcasting shall formulate an oversight
mechanism. It shall publish a charter for self-regulating bodies, including Codes of Practices. It shall
establish an Inter-Departmental Committee for hearing grievances. The IIIrd point hasn‘t been
mentioned. The publishers have been asked to set up a self-regulatory structure.
43. Iran with the P5+1 group of world powers - the USA, UK, France, China, Russia, and Germany
agreed on a long-term deal on its nuclear programme. P5 means the five permanent members of the
UNSC, namely the USA, UK, France, China and Russia. +1 means Germany.
The U.S. decision was criticized by all other parties to the JCPOA (including the European allies)
because Iran was in compliance with its obligations, as certified by the International Atomic Energy
Agency (IAEA). For the first year after the U.S. withdrawal, Iran‘s response was muted as the E-3
(France, Germany, the U.K.) and the EU promised to find ways to mitigate the U.S. decision.
It requires Iran to undertake eight- to 15-year-long curbs in nuclear activities with potential for
weaponization in exchange for international sanctions relief.
The EU, United Nations, and United States all committed to lifting their nuclear-related sanctions on
Iran. However, many other U.S. sanctions on Iran, some dating back to the 1979 hostage crisis,
remained in effect. They cover matters such as Iran‘s ballistic missile program, support for terrorist
groups, and human rights abuses. Though the United States committed to lifting its sanctions on oil
exports, it kept restrictions on financial transactions, which have deterred international trade with
Iran.
Iran accused the United States of reneging on its commitments, and faulted Europe for submitting to
U.S. unilateralism. In a bid to keep the agreement alive, France, Germany, and the United Kingdom
launched a barter system, known as INSTEX, to facilitate transactions with Iran outside of the U.S.
banking system. However, the system is only meant for food and medicine, which are already
exempt from U.S. sanctions.
The current President of Iran is Hassan Rouhani, assumed office on 3 August 2013, after the 2013
Iranian presidential election. He succeeded Mahmoud Ahmadinejad, who served 8 years in office
from 2005 to 2013.
44. Iran with the P5+1 group of world powers - the USA, UK, France, China, Russia, and Germany
agreed on a long-term deal on its nuclear programme. P5 means the five permanent members of the
UNSC, namely the USA, UK, France, China and Russia. +1 means Germany.
The U.S. decision was criticized by all other parties to the JCPOA (including the European allies)
because Iran was in compliance with its obligations, as certified by the International Atomic Energy
Agency (IAEA). For the first year after the U.S. withdrawal, Iran‘s response was muted as the E-3
(France, Germany, the U.K.) and the EU promised to find ways to mitigate the U.S. decision.
It requires Iran to undertake eight- to 15-year-long curbs in nuclear activities with potential for
weaponization in exchange for international sanctions relief.
The EU, United Nations, and United States all committed to lifting their nuclear-related sanctions on
Iran. However, many other U.S. sanctions on Iran, some dating back to the 1979 hostage crisis,
remained in effect. They cover matters such as Iran‘s ballistic missile program, support for terrorist
groups, and human rights abuses. Though the United States committed to lifting its sanctions on oil
exports, it kept restrictions on financial transactions, which have deterred international trade with
Iran.
Iran accused the United States of reneging on its commitments, and faulted Europe for submitting to
U.S. unilateralism. In a bid to keep the agreement alive, France, Germany, and the United Kingdom
launched a barter system, known as INSTEX, to facilitate transactions with Iran outside of the U.S.
banking system. However, the system is only meant for food and medicine, which are already
exempt from U.S. sanctions.
The current President of Iran is Hassan Rouhani, assumed office on 3 August 2013, after the 2013
Iranian presidential election. He succeeded Mahmoud Ahmadinejad, who served 8 years in office
from 2005 to 2013.
45. Iran with the P5+1 group of world powers - the USA, UK, France, China, Russia, and Germany
agreed on a long-term deal on its nuclear programme. P5 means the five permanent members of the
UNSC, namely the USA, UK, France, China and Russia. +1 means Germany.
The U.S. decision was criticized by all other parties to the JCPOA (including the European allies)
because Iran was in compliance with its obligations, as certified by the International Atomic Energy
Agency (IAEA). For the first year after the U.S. withdrawal, Iran‘s response was muted as the E-3
(France, Germany, the U.K.) and the EU promised to find ways to mitigate the U.S. decision.
It requires Iran to undertake eight- to 15-year-long curbs in nuclear activities with potential for
weaponization in exchange for international sanctions relief.
The EU, United Nations, and United States all committed to lifting their nuclear-related sanctions on
Iran. However, many other U.S. sanctions on Iran, some dating back to the 1979 hostage crisis,
remained in effect. They cover matters such as Iran‘s ballistic missile program, support for terrorist
groups, and human rights abuses. Though the United States committed to lifting its sanctions on oil
exports, it kept restrictions on financial transactions, which have deterred international trade with
Iran.
Iran accused the United States of reneging on its commitments, and faulted Europe for submitting to
U.S. unilateralism. In a bid to keep the agreement alive, France, Germany, and the United Kingdom
launched a barter system, known as INSTEX, to facilitate transactions with Iran outside of the U.S.
banking system. However, the system is only meant for food and medicine, which are already
exempt from U.S. sanctions.
The current President of Iran is Hassan Rouhani, assumed office on 3 August 2013, after the 2013
Iranian presidential election. He succeeded Mahmoud Ahmadinejad, who served 8 years in office
from 2005 to 2013.
46. Iran with the P5+1 group of world powers - the USA, UK, France, China, Russia, and Germany
agreed on a long-term deal on its nuclear programme. P5 means the five permanent members of the
UNSC, namely the USA, UK, France, China and Russia. +1 means Germany.
The U.S. decision was criticized by all other parties to the JCPOA (including the European allies)
because Iran was in compliance with its obligations, as certified by the International Atomic Energy
Agency (IAEA). For the first year after the U.S. withdrawal, Iran‘s response was muted as the E-3
(France, Germany, the U.K.) and the EU promised to find ways to mitigate the U.S. decision.
It requires Iran to undertake eight- to 15-year-long curbs in nuclear activities with potential for
weaponization in exchange for international sanctions relief.
The EU, United Nations, and United States all committed to lifting their nuclear-related sanctions on
Iran. However, many other U.S. sanctions on Iran, some dating back to the 1979 hostage crisis,
remained in effect. They cover matters such as Iran‘s ballistic missile program, support for terrorist
groups, and human rights abuses. Though the United States committed to lifting its sanctions on oil
exports, it kept restrictions on financial transactions, which have deterred international trade with
Iran.
Iran accused the United States of reneging on its commitments, and faulted Europe for submitting to
U.S. unilateralism. In a bid to keep the agreement alive, France, Germany, and the United Kingdom
launched a barter system, known as INSTEX, to facilitate transactions with Iran outside of the U.S.
banking system. However, the system is only meant for food and medicine, which are already
exempt from U.S. sanctions.
The current President of Iran is Hassan Rouhani, assumed office on 3 August 2013, after the 2013
Iranian presidential election. He succeeded Mahmoud Ahmadinejad, who served 8 years in office
from 2005 to 2013.
47. Iran with the P5+1 group of world powers - the USA, UK, France, China, Russia, and Germany
agreed on a long-term deal on its nuclear programme. P5 means the five permanent members of the
UNSC, namely the USA, UK, France, China and Russia. +1 means Germany.
The U.S. decision was criticized by all other parties to the JCPOA (including the European allies)
because Iran was in compliance with its obligations, as certified by the International Atomic Energy
Agency (IAEA). For the first year after the U.S. withdrawal, Iran‘s response was muted as the E-3
(France, Germany, the U.K.) and the EU promised to find ways to mitigate the U.S. decision.
It requires Iran to undertake eight- to 15-year-long curbs in nuclear activities with potential for
weaponization in exchange for international sanctions relief.
The EU, United Nations, and United States all committed to lifting their nuclear-related sanctions on
Iran. However, many other U.S. sanctions on Iran, some dating back to the 1979 hostage crisis,
remained in effect. They cover matters such as Iran‘s ballistic missile program, support for terrorist
groups, and human rights abuses. Though the United States committed to lifting its sanctions on oil
exports, it kept restrictions on financial transactions, which have deterred international trade with
Iran.
Iran accused the United States of reneging on its commitments, and faulted Europe for submitting to
U.S. unilateralism. In a bid to keep the agreement alive, France, Germany, and the United Kingdom
launched a barter system, known as INSTEX, to facilitate transactions with Iran outside of the U.S.
banking system. However, the system is only meant for food and medicine, which are already
exempt from U.S. sanctions.
The current President of Iran is Hassan Rouhani, assumed office on 3 August 2013, after the 2013
Iranian presidential election. He succeeded Mahmoud Ahmadinejad, who served 8 years in office
from 2005 to 2013.
48. Iran with the P5+1 group of world powers - the USA, UK, France, China, Russia, and Germany
agreed on a long-term deal on its nuclear programme. P5 means the five permanent members of the
UNSC, namely the USA, UK, France, China and Russia. +1 means Germany.
The U.S. decision was criticized by all other parties to the JCPOA (including the European allies)
because Iran was in compliance with its obligations, as certified by the International Atomic Energy
Agency (IAEA). For the first year after the U.S. withdrawal, Iran‘s response was muted as the E-3
(France, Germany, the U.K.) and the EU promised to find ways to mitigate the U.S. decision.
It requires Iran to undertake eight- to 15-year-long curbs in nuclear activities with potential for
weaponization in exchange for international sanctions relief.
The EU, United Nations, and United States all committed to lifting their nuclear-related sanctions on
Iran. However, many other U.S. sanctions on Iran, some dating back to the 1979 hostage crisis,
remained in effect. They cover matters such as Iran‘s ballistic missile program, support for terrorist
groups, and human rights abuses. Though the United States committed to lifting its sanctions on oil
exports, it kept restrictions on financial transactions, which have deterred international trade with
Iran.
Iran accused the United States of reneging on its commitments, and faulted Europe for submitting to
U.S. unilateralism. In a bid to keep the agreement alive, France, Germany, and the United Kingdom
launched a barter system, known as INSTEX, to facilitate transactions with Iran outside of the U.S.
banking system. However, the system is only meant for food and medicine, which are already
exempt from U.S. sanctions.
The current President of Iran is Hassan Rouhani, assumed office on 3 August 2013, after the 2013
Iranian presidential election. He succeeded Mahmoud Ahmadinejad, who served 8 years in office
from 2005 to 2013.
49. The President was satisfied that a situation had arisen in which the administration of the Union
Territory of Puducherry could not carry on in accordance with the provisions of the Government of
Union Territories Act, 1963. The Government of Union Territories Act, 1963 enacted by the
Parliament in accordance with the provisions of article 239A.
Article 239 to 242 under Part VIII of the Indian Constitution deals with the administration of Union
Territories. Every union territory is administered by the President acting through an administrator
appointed by him.
The Union Territories of Puducherry (in 1963), Delhi (in 1992) and Jammu and Kashmir (in 2019) are
provided with a legislative assembly and a council of ministers headed by a chief minister. But, the
establishment of such institutions in the union territories does not diminish the supreme control of the
President and Parliament over them.
President‘s Rule implies the suspension of a state government and the imposition of direct rule of
the Centre. It is also known as ‗State Emergency‘ or ‗Constitutional Emergency‘.
The President‘s Rule is imposed through the invocation of Article 356 of the Constitution by the
President on the advice of the Union Council of Ministers.
Under Article 356, President‘s Rule is imposed if the President, upon receipt of the report from the
Governor of the State or otherwise, is satisfied that a situation has arisen in which the government of
the State cannot be carried on in accordance with the provisions of the Constitution.
50. The President was satisfied that a situation had arisen in which the administration of the Union
Territory of Puducherry could not carry on in accordance with the provisions of the Government of
Union Territories Act, 1963. The Government of Union Territories Act, 1963 enacted by the
Parliament in accordance with the provisions of article 239A.
Article 239 to 242 under Part VIII of the Indian Constitution deals with the administration of Union
Territories. Every union territory is administered by the President acting through an administrator
appointed by him.
The Union Territories of Puducherry (in 1963), Delhi (in 1992) and Jammu and Kashmir (in 2019) are
provided with a legislative assembly and a council of ministers headed by a chief minister. But, the
establishment of such institutions in the union territories does not diminish the supreme control of the
President and Parliament over them.
President‘s Rule implies the suspension of a state government and the imposition of direct rule of
the Centre. It is also known as ‗State Emergency‘ or ‗Constitutional Emergency‘.
The President‘s Rule is imposed through the invocation of Article 356 of the Constitution by the
President on the advice of the Union Council of Ministers.
Under Article 356, President‘s Rule is imposed if the President, upon receipt of the report from the
Governor of the State or otherwise, is satisfied that a situation has arisen in which the government of
the State cannot be carried on in accordance with the provisions of the Constitution.
51. The President was satisfied that a situation had arisen in which the administration of the Union
Territory of Puducherry could not carry on in accordance with the provisions of the Government of
Union Territories Act, 1963. The Government of Union Territories Act, 1963 enacted by the
Parliament in accordance with the provisions of article 239A.
Article 239 to 242 under Part VIII of the Indian Constitution deals with the administration of Union
Territories. Every union territory is administered by the President acting through an administrator
appointed by him.
The Union Territories of Puducherry (in 1963), Delhi (in 1992) and Jammu and Kashmir (in 2019) are
provided with a legislative assembly and a council of ministers headed by a chief minister. But, the
establishment of such institutions in the union territories does not diminish the supreme control of the
President and Parliament over them.
President‘s Rule implies the suspension of a state government and the imposition of direct rule of
the Centre. It is also known as ‗State Emergency‘ or ‗Constitutional Emergency‘.
The President‘s Rule is imposed through the invocation of Article 356 of the Constitution by the
President on the advice of the Union Council of Ministers.
Under Article 356, President‘s Rule is imposed if the President, upon receipt of the report from the
Governor of the State or otherwise, is satisfied that a situation has arisen in which the government of
the State cannot be carried on in accordance with the provisions of the Constitution.
52. The President was satisfied that a situation had arisen in which the administration of the Union
Territory of Puducherry could not carry on in accordance with the provisions of the Government of
Union Territories Act, 1963. The Government of Union Territories Act, 1963 enacted by the
Parliament in accordance with the provisions of article 239A.
Article 239 to 242 under Part VIII of the Indian Constitution deals with the administration of Union
Territories. Every union territory is administered by the President acting through an administrator
appointed by him.
The Union Territories of Puducherry (in 1963), Delhi (in 1992) and Jammu and Kashmir (in 2019) are
provided with a legislative assembly and a council of ministers headed by a chief minister. But, the
establishment of such institutions in the union territories does not diminish the supreme control of the
President and Parliament over them.
President‘s Rule implies the suspension of a state government and the imposition of direct rule of
the Centre. It is also known as ‗State Emergency‘ or ‗Constitutional Emergency‘.
The President‘s Rule is imposed through the invocation of Article 356 of the Constitution by the
President on the advice of the Union Council of Ministers.
Under Article 356, President‘s Rule is imposed if the President, upon receipt of the report from the
Governor of the State or otherwise, is satisfied that a situation has arisen in which the government of
the State cannot be carried on in accordance with the provisions of the Constitution.
53. The President was satisfied that a situation had arisen in which the administration of the Union
Territory of Puducherry could not carry on in accordance with the provisions of the Government of
Union Territories Act, 1963. The Government of Union Territories Act, 1963 enacted by the
Parliament in accordance with the provisions of article 239A.
Article 239 to 242 under Part VIII of the Indian Constitution deals with the administration of Union
Territories. Every union territory is administered by the President acting through an administrator
appointed by him.
The Union Territories of Puducherry (in 1963), Delhi (in 1992) and Jammu and Kashmir (in 2019) are
provided with a legislative assembly and a council of ministers headed by a chief minister. But, the
establishment of such institutions in the union territories does not diminish the supreme control of the
President and Parliament over them.
President‘s Rule implies the suspension of a state government and the imposition of direct rule of
the Centre. It is also known as ‗State Emergency‘ or ‗Constitutional Emergency‘.
The President‘s Rule is imposed through the invocation of Article 356 of the Constitution by the
President on the advice of the Union Council of Ministers.
Under Article 356, President‘s Rule is imposed if the President, upon receipt of the report from the
Governor of the State or otherwise, is satisfied that a situation has arisen in which the government of
the State cannot be carried on in accordance with the provisions of the Constitution.
54. The Line of Control was created in 1972 in the aftermath of the third India-Pakistan war. The
original intention was to create a working border while efforts continued to secure a permanent one.
Ever since, the Line of Control has seen periodic clashes between the two warring parties.
The two DGsMO of India and Pakistan had come to a ceasefire agreement in the year 2003.
Dialogue broke down soon thereafter because of the Pathankot airbase attack of 2nd January, 2016,
which was followed by the attack at the garrison in Uri and the Indian response with a surgical strike
along the border. Bilateral ties continued to nosedive because of the Pulwama terror attack of 14th
February, 2019, and the Balakot operation by India.
The 2003 Ceasefire agreement was successful only till 2006. In these 3 years not a single bullet was
fired. However, after 2006 ceasefire violations has become the norm.
The Chief of the Army Staff is the head of the military staff of the Indian Army and the chairman of
the Indian Army Board. The current COAS is General Manoj Mukund Naravane, who took office on
31 December 2019.
Ceasefire violations along the India-Pakistan border increased by almost 48 per cent in 2020 as
compared to 2019, even as terrorist attacks in Kashmir were less than half during this period, the
government told Lok Sabha. In reply to a question on ceasefire violations by Pakistan and terror
incidents in the Valley over the past three years, MoS Home G Kishan Reddy told Lok Sabha in a
written reply that the year 2020 witnessed 5,133 ceasefire violations as compared to 3,479 in 2019
and just 2,140 in 2018. As many as 22, 18 and 30 civilians were killed in these incidents,
respectively, while 24, 19 and 29 security personnel were killed, respectively. The 2003 Ceasefire
Agreement though loosely worded is not a written document. Thus option (b) is wrong. There has
been a general softening of stance by both India and Pakistan with Bajwa saying Pakistan should
extend the hand of peace on all sides, Imran Khan not bringing the ―Kashmir‖ issue in the recent
SAARC meet, for which the invitation was sent by India and the latter allowing Khan‘s flight to Sri
Lanka over India‘s air-zone.
Chinar Corps Commander Lt Gen. B.S. Raju has been appointed as the new Director General of
Military Operations (DGMO) and Lt Gen. D.P. Pandey will take over from him. Sources in the
defence and security establishment said that the appointment order has come and Raju will take
over as the new DGMO in April after having completed his tenure as the Corps Commander.He will
succeed Lt Gen. Paramjit Singh Sangha, who took over as the Deputy Chief (Strategy) in the Army
Headquarters last week.
55. The Line of Control was created in 1972 in the aftermath of the third India-Pakistan war. The
original intention was to create a working border while efforts continued to secure a permanent one.
Ever since, the Line of Control has seen periodic clashes between the two warring parties.
The two DGsMO of India and Pakistan had come to a ceasefire agreement in the year 2003.
Dialogue broke down soon thereafter because of the Pathankot airbase attack of 2nd January, 2016,
which was followed by the attack at the garrison in Uri and the Indian response with a surgical strike
along the border. Bilateral ties continued to nosedive because of the Pulwama terror attack of 14th
February, 2019, and the Balakot operation by India.
The 2003 Ceasefire agreement was successful only till 2006. In these 3 years not a single bullet was
fired. However, after 2006 ceasefire violations has become the norm.
The Chief of the Army Staff is the head of the military staff of the Indian Army and the chairman of
the Indian Army Board. The current COAS is General Manoj Mukund Naravane, who took office on
31 December 2019.
Ceasefire violations along the India-Pakistan border increased by almost 48 per cent in 2020 as
compared to 2019, even as terrorist attacks in Kashmir were less than half during this period, the
government told Lok Sabha. In reply to a question on ceasefire violations by Pakistan and terror
incidents in the Valley over the past three years, MoS Home G Kishan Reddy told Lok Sabha in a
written reply that the year 2020 witnessed 5,133 ceasefire violations as compared to 3,479 in 2019
and just 2,140 in 2018. As many as 22, 18 and 30 civilians were killed in these incidents,
respectively, while 24, 19 and 29 security personnel were killed, respectively. The 2003 Ceasefire
Agreement though loosely worded is not a written document. Thus option (b) is wrong. There has
been a general softening of stance by both India and Pakistan with Bajwa saying Pakistan should
extend the hand of peace on all sides, Imran Khan not bringing the ―Kashmir‖ issue in the recent
SAARC meet, for which the invitation was sent by India and the latter allowing Khan‘s flight to Sri
Lanka over India‘s air-zone.
Chinar Corps Commander Lt Gen. B.S. Raju has been appointed as the new Director General of
Military Operations (DGMO) and Lt Gen. D.P. Pandey will take over from him. Sources in the
defence and security establishment said that the appointment order has come and Raju will take
over as the new DGMO in April after having completed his tenure as the Corps Commander.He will
succeed Lt Gen. Paramjit Singh Sangha, who took over as the Deputy Chief (Strategy) in the Army
Headquarters last week.
56. The Line of Control was created in 1972 in the aftermath of the third India-Pakistan war. The
original intention was to create a working border while efforts continued to secure a permanent one.
Ever since, the Line of Control has seen periodic clashes between the two warring parties.
The two DGsMO of India and Pakistan had come to a ceasefire agreement in the year 2003.
Dialogue broke down soon thereafter because of the Pathankot airbase attack of 2nd January, 2016,
which was followed by the attack at the garrison in Uri and the Indian response with a surgical strike
along the border. Bilateral ties continued to nosedive because of the Pulwama terror attack of 14th
February, 2019, and the Balakot operation by India.
The 2003 Ceasefire agreement was successful only till 2006. In these 3 years not a single bullet was
fired. However, after 2006 ceasefire violations has become the norm.
The Chief of the Army Staff is the head of the military staff of the Indian Army and the chairman of
the Indian Army Board. The current COAS is General Manoj Mukund Naravane, who took office on
31 December 2019.
Ceasefire violations along the India-Pakistan border increased by almost 48 per cent in 2020 as
compared to 2019, even as terrorist attacks in Kashmir were less than half during this period, the
government told Lok Sabha. In reply to a question on ceasefire violations by Pakistan and terror
incidents in the Valley over the past three years, MoS Home G Kishan Reddy told Lok Sabha in a
written reply that the year 2020 witnessed 5,133 ceasefire violations as compared to 3,479 in 2019
and just 2,140 in 2018. As many as 22, 18 and 30 civilians were killed in these incidents,
respectively, while 24, 19 and 29 security personnel were killed, respectively. The 2003 Ceasefire
Agreement though loosely worded is not a written document. Thus option (b) is wrong. There has
been a general softening of stance by both India and Pakistan with Bajwa saying Pakistan should
extend the hand of peace on all sides, Imran Khan not bringing the ―Kashmir‖ issue in the recent
SAARC meet, for which the invitation was sent by India and the latter allowing Khan‘s flight to Sri
Lanka over India‘s air-zone.
Chinar Corps Commander Lt Gen. B.S. Raju has been appointed as the new Director General of
Military Operations (DGMO) and Lt Gen. D.P. Pandey will take over from him. Sources in the
defence and security establishment said that the appointment order has come and Raju will take
over as the new DGMO in April after having completed his tenure as the Corps Commander.He will
succeed Lt Gen. Paramjit Singh Sangha, who took over as the Deputy Chief (Strategy) in the Army
Headquarters last week.
57. The Line of Control was created in 1972 in the aftermath of the third India-Pakistan war. The
original intention was to create a working border while efforts continued to secure a permanent one.
Ever since, the Line of Control has seen periodic clashes between the two warring parties.
The two DGsMO of India and Pakistan had come to a ceasefire agreement in the year 2003.
Dialogue broke down soon thereafter because of the Pathankot airbase attack of 2nd January, 2016,
which was followed by the attack at the garrison in Uri and the Indian response with a surgical strike
along the border. Bilateral ties continued to nosedive because of the Pulwama terror attack of 14th
February, 2019, and the Balakot operation by India.
The 2003 Ceasefire agreement was successful only till 2006. In these 3 years not a single bullet was
fired. However, after 2006 ceasefire violations has become the norm.
The Chief of the Army Staff is the head of the military staff of the Indian Army and the chairman of
the Indian Army Board. The current COAS is General Manoj Mukund Naravane, who took office on
31 December 2019.
Ceasefire violations along the India-Pakistan border increased by almost 48 per cent in 2020 as
compared to 2019, even as terrorist attacks in Kashmir were less than half during this period, the
government told Lok Sabha. In reply to a question on ceasefire violations by Pakistan and terror
incidents in the Valley over the past three years, MoS Home G Kishan Reddy told Lok Sabha in a
written reply that the year 2020 witnessed 5,133 ceasefire violations as compared to 3,479 in 2019
and just 2,140 in 2018. As many as 22, 18 and 30 civilians were killed in these incidents,
respectively, while 24, 19 and 29 security personnel were killed, respectively. The 2003 Ceasefire
Agreement though loosely worded is not a written document. Thus option (b) is wrong. There has
been a general softening of stance by both India and Pakistan with Bajwa saying Pakistan should
extend the hand of peace on all sides, Imran Khan not bringing the ―Kashmir‖ issue in the recent
SAARC meet, for which the invitation was sent by India and the latter allowing Khan‘s flight to Sri
Lanka over India‘s air-zone.
Chinar Corps Commander Lt Gen. B.S. Raju has been appointed as the new Director General of
Military Operations (DGMO) and Lt Gen. D.P. Pandey will take over from him. Sources in the
defence and security establishment said that the appointment order has come and Raju will take
over as the new DGMO in April after having completed his tenure as the Corps Commander.He will
succeed Lt Gen. Paramjit Singh Sangha, who took over as the Deputy Chief (Strategy) in the Army
Headquarters last week.
58. The Line of Control was created in 1972 in the aftermath of the third India-Pakistan war. The
original intention was to create a working border while efforts continued to secure a permanent one.
Ever since, the Line of Control has seen periodic clashes between the two warring parties.
The two DGsMO of India and Pakistan had come to a ceasefire agreement in the year 2003.
Dialogue broke down soon thereafter because of the Pathankot airbase attack of 2nd January, 2016,
which was followed by the attack at the garrison in Uri and the Indian response with a surgical strike
along the border. Bilateral ties continued to nosedive because of the Pulwama terror attack of 14th
February, 2019, and the Balakot operation by India.
The 2003 Ceasefire agreement was successful only till 2006. In these 3 years not a single bullet was
fired. However, after 2006 ceasefire violations has become the norm.
The Chief of the Army Staff is the head of the military staff of the Indian Army and the chairman of
the Indian Army Board. The current COAS is General Manoj Mukund Naravane, who took office on
31 December 2019.
Ceasefire violations along the India-Pakistan border increased by almost 48 per cent in 2020 as
compared to 2019, even as terrorist attacks in Kashmir were less than half during this period, the
government told Lok Sabha. In reply to a question on ceasefire violations by Pakistan and terror
incidents in the Valley over the past three years, MoS Home G Kishan Reddy told Lok Sabha in a
written reply that the year 2020 witnessed 5,133 ceasefire violations as compared to 3,479 in 2019
and just 2,140 in 2018. As many as 22, 18 and 30 civilians were killed in these incidents,
respectively, while 24, 19 and 29 security personnel were killed, respectively. The 2003 Ceasefire
Agreement though loosely worded is not a written document. Thus option (b) is wrong. There has
been a general softening of stance by both India and Pakistan with Bajwa saying Pakistan should
extend the hand of peace on all sides, Imran Khan not bringing the ―Kashmir‖ issue in the recent
SAARC meet, for which the invitation was sent by India and the latter allowing Khan‘s flight to Sri
Lanka over India‘s air-zone.
Chinar Corps Commander Lt Gen. B.S. Raju has been appointed as the new Director General of
Military Operations (DGMO) and Lt Gen. D.P. Pandey will take over from him. Sources in the
defence and security establishment said that the appointment order has come and Raju will take
over as the new DGMO in April after having completed his tenure as the Corps Commander.He will
succeed Lt Gen. Paramjit Singh Sangha, who took over as the Deputy Chief (Strategy) in the Army
Headquarters last week.
59. The Line of Control was created in 1972 in the aftermath of the third India-Pakistan war. The
original intention was to create a working border while efforts continued to secure a permanent one.
Ever since, the Line of Control has seen periodic clashes between the two warring parties.
The two DGsMO of India and Pakistan had come to a ceasefire agreement in the year 2003.
Dialogue broke down soon thereafter because of the Pathankot airbase attack of 2nd January, 2016,
which was followed by the attack at the garrison in Uri and the Indian response with a surgical strike
along the border. Bilateral ties continued to nosedive because of the Pulwama terror attack of 14th
February, 2019, and the Balakot operation by India.
The 2003 Ceasefire agreement was successful only till 2006. In these 3 years not a single bullet was
fired. However, after 2006 ceasefire violations has become the norm.
The Chief of the Army Staff is the head of the military staff of the Indian Army and the chairman of
the Indian Army Board. The current COAS is General Manoj Mukund Naravane, who took office on
31 December 2019.
Ceasefire violations along the India-Pakistan border increased by almost 48 per cent in 2020 as
compared to 2019, even as terrorist attacks in Kashmir were less than half during this period, the
government told Lok Sabha. In reply to a question on ceasefire violations by Pakistan and terror
incidents in the Valley over the past three years, MoS Home G Kishan Reddy told Lok Sabha in a
written reply that the year 2020 witnessed 5,133 ceasefire violations as compared to 3,479 in 2019
and just 2,140 in 2018. As many as 22, 18 and 30 civilians were killed in these incidents,
respectively, while 24, 19 and 29 security personnel were killed, respectively. The 2003 Ceasefire
Agreement though loosely worded is not a written document. Thus option (b) is wrong. There has
been a general softening of stance by both India and Pakistan with Bajwa saying Pakistan should
extend the hand of peace on all sides, Imran Khan not bringing the ―Kashmir‖ issue in the recent
SAARC meet, for which the invitation was sent by India and the latter allowing Khan‘s flight to Sri
Lanka over India‘s air-zone.
Chinar Corps Commander Lt Gen. B.S. Raju has been appointed as the new Director General of
Military Operations (DGMO) and Lt Gen. D.P. Pandey will take over from him. Sources in the
defence and security establishment said that the appointment order has come and Raju will take
over as the new DGMO in April after having completed his tenure as the Corps Commander.He will
succeed Lt Gen. Paramjit Singh Sangha, who took over as the Deputy Chief (Strategy) in the Army
Headquarters last week.
60. The Line of Control was created in 1972 in the aftermath of the third India-Pakistan war. The
original intention was to create a working border while efforts continued to secure a permanent one.
Ever since, the Line of Control has seen periodic clashes between the two warring parties.
The two DGsMO of India and Pakistan had come to a ceasefire agreement in the year 2003.
Dialogue broke down soon thereafter because of the Pathankot airbase attack of 2nd January, 2016,
which was followed by the attack at the garrison in Uri and the Indian response with a surgical strike
along the border. Bilateral ties continued to nosedive because of the Pulwama terror attack of 14th
February, 2019, and the Balakot operation by India.
The 2003 Ceasefire agreement was successful only till 2006. In these 3 years not a single bullet was
fired. However, after 2006 ceasefire violations has become the norm.
The Chief of the Army Staff is the head of the military staff of the Indian Army and the chairman of
the Indian Army Board. The current COAS is General Manoj Mukund Naravane, who took office on
31 December 2019.
Ceasefire violations along the India-Pakistan border increased by almost 48 per cent in 2020 as
compared to 2019, even as terrorist attacks in Kashmir were less than half during this period, the
government told Lok Sabha. In reply to a question on ceasefire violations by Pakistan and terror
incidents in the Valley over the past three years, MoS Home G Kishan Reddy told Lok Sabha in a
written reply that the year 2020 witnessed 5,133 ceasefire violations as compared to 3,479 in 2019
and just 2,140 in 2018. As many as 22, 18 and 30 civilians were killed in these incidents,
respectively, while 24, 19 and 29 security personnel were killed, respectively. The 2003 Ceasefire
Agreement though loosely worded is not a written document. Thus option (b) is wrong. There has
been a general softening of stance by both India and Pakistan with Bajwa saying Pakistan should
extend the hand of peace on all sides, Imran Khan not bringing the ―Kashmir‖ issue in the recent
SAARC meet, for which the invitation was sent by India and the latter allowing Khan‘s flight to Sri
Lanka over India‘s air-zone.
Chinar Corps Commander Lt Gen. B.S. Raju has been appointed as the new Director General of
Military Operations (DGMO) and Lt Gen. D.P. Pandey will take over from him. Sources in the
defence and security establishment said that the appointment order has come and Raju will take
over as the new DGMO in April after having completed his tenure as the Corps Commander.He will
succeed Lt Gen. Paramjit Singh Sangha, who took over as the Deputy Chief (Strategy) in the Army
Headquarters last week.
61. Yemen‘s civil war began in 2014 when Houthi insurgents—Shiite rebels with links to Iran and a
history of rising up against the Sunni government—took control of Yemen‘s capital and largest city,
Sana‘a, demanding lower fuel prices and a new government.
In June 2018, the coalition launched a major offensive to capture from the Houthis the Red Sea city
of Hudaydah, whose port is the principal lifeline for almost two thirds of Yemen''s population. Later
an agreement for ceasefire was signed which required the parties to redeploy their forces from
Hudaydah, establish a prisoner exchange mechanism, and to address the situation in Taiz.
Ali Abdullah Saleh was a Yemeni politician who served as the first President of Yemen, from Yemeni
unification on 22 May 1990 to his resignation on 25 February 2012, following the Yemeni Revolution.
The Houthis since they are the Zaidi Shia sect minority community of Yemen are believed to be
receiving aid from Iran.
War‘s significance for the world: What happens in Yemen can greatly exacerbate regional tensions.
It is also seen as part of a regional power struggle between Shia-ruled Iran and Sunni-ruled Saudi
Arabia. Gulf Arab states - backers of President Hadi - have accused Iran of bolstering the Houthis
financially and militarily, though Iran has denied this. It also worries the West because of the threat
of terrorist attacks - such as from al-Qaeda or IS affiliates - emanating from the country as it
becomes more unstable. Yemen is also strategically important because it sits on Bab-el-Mandeb
strait.
62. Yemen‘s civil war began in 2014 when Houthi insurgents—Shiite rebels with links to Iran and a
history of rising up against the Sunni government—took control of Yemen‘s capital and largest city,
Sana‘a, demanding lower fuel prices and a new government.
In June 2018, the coalition launched a major offensive to capture from the Houthis the Red Sea city
of Hudaydah, whose port is the principal lifeline for almost two thirds of Yemen''s population. Later
an agreement for ceasefire was signed which required the parties to redeploy their forces from
Hudaydah, establish a prisoner exchange mechanism, and to address the situation in Taiz.
Ali Abdullah Saleh was a Yemeni politician who served as the first President of Yemen, from Yemeni
unification on 22 May 1990 to his resignation on 25 February 2012, following the Yemeni Revolution.
The Houthis since they are the Zaidi Shia sect minority community of Yemen are believed to be
receiving aid from Iran.
War‘s significance for the world: What happens in Yemen can greatly exacerbate regional tensions.
It is also seen as part of a regional power struggle between Shia-ruled Iran and Sunni-ruled Saudi
Arabia. Gulf Arab states - backers of President Hadi - have accused Iran of bolstering the Houthis
financially and militarily, though Iran has denied this. It also worries the West because of the threat
of terrorist attacks - such as from al-Qaeda or IS affiliates - emanating from the country as it
becomes more unstable. Yemen is also strategically important because it sits on Bab-el-Mandeb
strait.
63. Yemen‘s civil war began in 2014 when Houthi insurgents—Shiite rebels with links to Iran and a
history of rising up against the Sunni government—took control of Yemen‘s capital and largest city,
Sana‘a, demanding lower fuel prices and a new government.
In June 2018, the coalition launched a major offensive to capture from the Houthis the Red Sea city
of Hudaydah, whose port is the principal lifeline for almost two thirds of Yemen''s population. Later
an agreement for ceasefire was signed which required the parties to redeploy their forces from
Hudaydah, establish a prisoner exchange mechanism, and to address the situation in Taiz.
Ali Abdullah Saleh was a Yemeni politician who served as the first President of Yemen, from Yemeni
unification on 22 May 1990 to his resignation on 25 February 2012, following the Yemeni Revolution.
The Houthis since they are the Zaidi Shia sect minority community of Yemen are believed to be
receiving aid from Iran.
War‘s significance for the world: What happens in Yemen can greatly exacerbate regional tensions.
It is also seen as part of a regional power struggle between Shia-ruled Iran and Sunni-ruled Saudi
Arabia. Gulf Arab states - backers of President Hadi - have accused Iran of bolstering the Houthis
financially and militarily, though Iran has denied this. It also worries the West because of the threat
of terrorist attacks - such as from al-Qaeda or IS affiliates - emanating from the country as it
becomes more unstable. Yemen is also strategically important because it sits on Bab-el-Mandeb
strait.
64. Yemen‘s civil war began in 2014 when Houthi insurgents—Shiite rebels with links to Iran and a
history of rising up against the Sunni government—took control of Yemen‘s capital and largest city,
Sana‘a, demanding lower fuel prices and a new government.
In June 2018, the coalition launched a major offensive to capture from the Houthis the Red Sea city
of Hudaydah, whose port is the principal lifeline for almost two thirds of Yemen''s population. Later
an agreement for ceasefire was signed which required the parties to redeploy their forces from
Hudaydah, establish a prisoner exchange mechanism, and to address the situation in Taiz.
Ali Abdullah Saleh was a Yemeni politician who served as the first President of Yemen, from Yemeni
unification on 22 May 1990 to his resignation on 25 February 2012, following the Yemeni Revolution.
The Houthis since they are the Zaidi Shia sect minority community of Yemen are believed to be
receiving aid from Iran.
War‘s significance for the world: What happens in Yemen can greatly exacerbate regional tensions.
It is also seen as part of a regional power struggle between Shia-ruled Iran and Sunni-ruled Saudi
Arabia. Gulf Arab states - backers of President Hadi - have accused Iran of bolstering the Houthis
financially and militarily, though Iran has denied this. It also worries the West because of the threat
of terrorist attacks - such as from al-Qaeda or IS affiliates - emanating from the country as it
becomes more unstable. Yemen is also strategically important because it sits on Bab-el-Mandeb
strait.
65. Yemen‘s civil war began in 2014 when Houthi insurgents—Shiite rebels with links to Iran and a
history of rising up against the Sunni government—took control of Yemen‘s capital and largest city,
Sana‘a, demanding lower fuel prices and a new government.
In June 2018, the coalition launched a major offensive to capture from the Houthis the Red Sea city
of Hudaydah, whose port is the principal lifeline for almost two thirds of Yemen''s population. Later
an agreement for ceasefire was signed which required the parties to redeploy their forces from
Hudaydah, establish a prisoner exchange mechanism, and to address the situation in Taiz.
Ali Abdullah Saleh was a Yemeni politician who served as the first President of Yemen, from Yemeni
unification on 22 May 1990 to his resignation on 25 February 2012, following the Yemeni Revolution.
The Houthis since they are the Zaidi Shia sect minority community of Yemen are believed to be
receiving aid from Iran.
War‘s significance for the world: What happens in Yemen can greatly exacerbate regional tensions.
It is also seen as part of a regional power struggle between Shia-ruled Iran and Sunni-ruled Saudi
Arabia. Gulf Arab states - backers of President Hadi - have accused Iran of bolstering the Houthis
financially and militarily, though Iran has denied this. It also worries the West because of the threat
of terrorist attacks - such as from al-Qaeda or IS affiliates - emanating from the country as it
becomes more unstable. Yemen is also strategically important because it sits on Bab-el-Mandeb
strait.
66. The issue is going on in the Delhi High Court.
Such declaration of law is binding on all courts throughout India under Article 141 of the Constitution.
While referring to Article 16 of Universal Declaration of Human Rights and the Puttaswamy case, the
SC held that the right to marry a person of one‘s choice is integral to Article 21 of the Constitution.
LGBTQ Community Entitled to all Constitutional Rights (Navjet Singh Johar and others v. Union of
India 2018): The SC held that members of the LGBTQ community ―are entitled, as all other citizens,
to the full range of constitutional rights including the liberties protected by the Constitution‖ and are
entitled to equal citizenship and ―equal protection of law‖.
67. The issue is going on in the Delhi High Court.
Such declaration of law is binding on all courts throughout India under Article 141 of the Constitution.
While referring to Article 16 of Universal Declaration of Human Rights and the Puttaswamy case, the
SC held that the right to marry a person of one‘s choice is integral to Article 21 of the Constitution.
LGBTQ Community Entitled to all Constitutional Rights (Navjet Singh Johar and others v. Union of
India 2018): The SC held that members of the LGBTQ community ―are entitled, as all other citizens,
to the full range of constitutional rights including the liberties protected by the Constitution‖ and are
entitled to equal citizenship and ―equal protection of law‖.
68. The issue is going on in the Delhi High Court.
Such declaration of law is binding on all courts throughout India under Article 141 of the Constitution.
While referring to Article 16 of Universal Declaration of Human Rights and the Puttaswamy case, the
SC held that the right to marry a person of one‘s choice is integral to Article 21 of the Constitution.
LGBTQ Community Entitled to all Constitutional Rights (Navjet Singh Johar and others v. Union of
India 2018): The SC held that members of the LGBTQ community ―are entitled, as all other citizens,
to the full range of constitutional rights including the liberties protected by the Constitution‖ and are
entitled to equal citizenship and ―equal protection of law‖.

LEGAL REASONING

69. the legal information in the passages mentions res ipsa loquitor. The point been that the fan fell
on the ground due to poor fixability is enough to hold the school authorities liable.
70. ―The thing causing the damage must be under the control of the defendant or his servants‖ As all
the three essential are met and Bimla is working for Sureeli, therefore Sureeli will be held liable.
71. (c) The fact that a wall collapses when someone leans against it, proves that there has been
some negligence in its construction. Also, falling of the wall is sufficient enough evidence to make
Manoj liable
72. As per the doctrine, ―res ipsa loquitur i.e. things speak for themselves.‖ In the instant case, the
presence of abdominal pack in her abdomen is sufficient proof therefore, she doesn‘t need to prove
her case in the court of law.
73. It is clear from the factual situation that even though Kumaon Cups Ltd had quoted the lowest bid
yet it was allotted to a company run by local politician belonging to Hindu community. This is a
violation of equality on the basis of religion.
74. According to the information provided in the passage, the Constitution provides for equality
before law. ―Equality before the law means that equality among equals, the law should be equal for
all. And should be equally administered, that like should treated alike.‖ In the given factual situation,
the tax slabs are different for different income groups. This is not discriminatory as people who earn
more than Rs. 5lakhs per annum cannot be treated equally with people who do not earn that amount
of income.
75. As per the information provided in the passage, the State shall strive to provide equal pay for
equal work to anyone, male or female, who are placed equally in all respects. In the instant case,
Jasmine is a female employee, but she works in the same position as her male colleagues, doing
equal work for equal number of hours. Hence, she is entitled to the same salary as her male
colleagues.
76. Anandi was not treated equally as she was not getting the same remuneration as Bismil. There
was a discrimination on the basis of sex.
77. As per the information given in the passage, the State shall not deny to any person equality
before the law or the equal protection of the laws within the territory of India. Prohibition of
discrimination on grounds of religion, race, caste, sex or place of birth. Since the institution is
governed by government funds, it does not matter that it is a minority institution or not. Hence option
c.
78. According to the information given in the passage, the State shall not deny to any person
equality before the law or the equal protection of the laws within the territory of India Prohibition of
discrimination on grounds of religion, race, caste, sex or place of birth. In this case there is no
violation of right to equality as there is difference in the timing of the work of men and women made
due to security reasons of the females. Hence option a.
79. The correct answer is option (d). The incidence of income tax under the Income-tax Act, 1961 is
based on the residence of the taxpayer and the sources of his or her income. Since Mr Worldwide
spent 184 days in the United States, he does not qualify as an Indian resident as per the passage.
Hence, only his Indian income will be taxed. Therefore, his income from New York, New Jersey and
Miami will not be taxable.
80. The correct answer is option (b). Suresh is incorrect in his assessment because he is liable to
pay income tax for the Financial Year of 2019 as he qualifies as a resident of India for that year.
Option (a) is incorrect because Suresh spent 182+ days in 2019 and the question is silent on his
stay during the four years preceding 2019. Therefore, he is a resident due to the applicability of
Section 6(a) only and not 6(b).
81. The correct answer is option (c). The passage clearly mentions that the lockdown period from
March 22, 2020 to March 31, 2020 would be excluded in determining the residential status of an
individual under Section 6 for FY 2019-20. Additionally, as the lockdown had continued into FY
2020-21, a further extension to exclude the period of stay of individuals stranded in India up to the
date of normalisation of international flight operations, for determining the residential status during
FY 2020-21. Hence, Rakesh is not a resident of India for this period and is not liable to pay income
tax in India.
82. The correct answer is option (c). As per the passage, the period from March 22, 2020 to March
31, 2020 would be excluded in determining the residential status of an individual. Additionally, the
extended lockdown was to be excluded "up to the date of normalisation of international flight
operations" for determining the residential status of an individual. Since flight operations to Dubai
were normalised on 15 May 2020, Mohini''s stay prior to the lockdown in March and after 15th of
May will be counted in calculating the period of stay for determining residential status. Since this
exceeds 182 days, Mohini''s income will be taxable during this period.
83. The correct answer is option (c). Article 191 (1) says a person can be disqualified as MLA ―if he
holds any office of profit under the Government of India or the Government of any State specified in
the First Schedule, other than an office declared by the Legislature of the State by law not to
disqualify its holder‖. The Wakandan law in the question does not explicitly provide for the exception
to this article. Option (b) is incorrect because it is indeed an ―office of profit‖ as Anju and Panju are
deriving a profit, i.e. the stipend under the government, i.e. the government-run think tank.
84. The correct answer is option (d). Anju and Panju were holding an office since December 2018,
which is before the Wakandan law, which provided for an exemption to the ―office of profit‖ condition
was passed and subsequently repealed. Option (b) is incorrect because it provides the reason as
repeal of the law in question. However, Anju and Panju can be disqualified because they had been
working in that position since before the law even came into existence. There is no retrospective
application of law either.
85. The correct answer is option (b). Article 191(1) of the Constitution forbids holding an office profit
under the government but there is no such bar on an individual‘s private business or private practice.
Therefore, Jonty cannot be disqualified because he is not in violation of the Constitution.
86. The correct answer is option (c). Public prosecutor for the state is an office of profit under the
government. Hence, this move could threaten Jonty with disqualification as an MLA. Option (d) is
incorrect because there have been no such specific rules for a public prosecutor which have been
stated in the passage.
87. There is no valid contract in the present case as no acceptance was communicated to Anandi. In
absence of any contract, there arise no case of breach of contract. Refer ―whether and when a valid
contract was formed recourse should be taken to the principles of ''offer'', ''acceptance'' and
''revocation of acceptance'' as mentioned in Chapter 1 of the Contract Act. Parties by agreement
cannot deviate from the said principles, and yet contend that valid contract was formed.‖
88. (b) Refer ―Without consensus ad-idem no valid contract can be formed.‖ The contract is not valid
as both parties did not agree on same the same thing in the same sense. Monica thought it is the
dog Bentley and Ross thought of the car Bentley.
89. (C) ―whether and when a valid contract was formed recourse should be taken to the principles of
''offer'', ''acceptance'' and ''revocation of acceptance'' as mentioned in Chapter 1 of the Contract Act.
Parties by agreement cannot deviate from the said principles, and yet contend that valid contract
was formed.‖ As there was no proper communication of revocation, it will not be considered as a
valid revocation.
90. ―Parties by agreement cannot deviate from the said principles, and yet contend that valid
contract was formed. The offer, acceptance, revocation etc should be well communicated to all the
concerned parties.‖ The communication of the decision had to be initiated by the council exclusively
and since they decided to revoke the appointment before such a communication was made, Sharma
will not succeed.
91. The children cannot sue Rahul in the court of law for the breach of contract, as condition for the
valid offer were not met.
92. Abhishek is not entitled to the reward, because he acted in ignorance of the offer. Unless he has
the knowledge of the offer (communicated) he cannot give acceptance to it so as to make a binding
contract.
93. The passage above states that robbery is an offence of aggravated theft. In this case, the
property had dropped in the yard of Thakur, but it moved out of possession of Thakur without his
consent. Thus, it can be said that robbery has been committed.
94. The passage states that violence must be committed during the process of theft for robbery to
happen. In this case, the theft happened the moment the property was taken out of the possession
of Thakur. It thus cannot be said that Gabbar is liable for robbery. Hitting and theft were two different
acts.
95. The passage states that for extortion to become robbery, there has to be presence during the
time such extortion is being committed. In this case, presence is being sought to be established
through video call. However the same cannot be said to be sufficient presence for purposes of fair
implementation of the law. Thus no robbery is established.
96. The passage states that property has to be obtained in some form. Thus, extortion is complete
the moment some property is obtained. In this case, bitcoins are property that was obtained by
Gabbar. Thus, the extortion was complete on 24/02/2010.
97. The passage states that for extortion to occur, property has to be delivered in some form. Thus,
extortion is complete the moment some property is obtained. In this case, even if the bitcoins were
returned and cash was transferred, the first instance of transfer was on 24/02/2010.
98. The passage states that stealing and hurt have to occur together. In this case, X is not liable for
robbery. The act of theft and causing hurt are two separate acts which are not connected to each
other.
99. (d) Evelyn is the owner of the property and was responsible to ensure that there is a sufficient
notice of warning. The fact that the couple had wrongful intentions to enter the farmhouse is
important in this case and therefore it will amount to contributory negligence.
100. A - The facts show that Dhram knew about Veer not possessing a license and having no
expertise in driving and yet agreeing to be a party to the act, makes it a case of volenti non fit injuria.
This will not qualify as contributory negligence as there is no act done by Dhram which contributed to
his injuries.
101. C - It is clear that there has been contributory negligence on Tiwari''s part by allowing the dog
on her premises. Tiwari further cannot complain regarding the loss of roses as her daughter
contributed to the same.
102. The passage states that if a person causes injury to another (the victim), he will be held liable.
However, if that injury is caused by the victim‘s own carelessness, the person will no longer be
liable. Here, the victim himself contributed to his injuries.
103. The passage states that there must be a prior agreement to commit an offence. Thus, the only
responsibility in this case lies with A.
104. The passage states that there must be a prior agreement and consent to commit conspiracy. In
this case, there is no prior agreement and there is no conspiracy therefore.
105. The passage states that there must be prior agreement and consent to commit conspiracy. In
this case, in the lack of evidence to show prior agreement, it cannot be said that there is criminal
conspiracy.
106. There is no criminal conspiracy as the prior agreement was only of robbing the house and not
murder of the owner‘s son. Therefore, Paras and Rohit are not responsible for the murder committed
by Qasim as they had intention to do so.
LOGICAL REASONING

107. Options (a) & (b) are mentioned in the passage but can be seen to be incorrect. Option (c)
cannot be reasonable inferred from the passage. Therefore, the correct answer is Option (d).
108. Option (a) has been stated in the passage as an argument against the movement and
therefore, it is not an assumption. Option (b) or (c) are unfounded in the passage. Option (d) is
imperative to the discussion as without involvement of people, the movement would fail and the
passage would become redundant.
109. Only Option (c) can be reasonably inferred from the passage. The other options may have been
touched upon but are incorrect in some way or the other.
110. Options (a) & (b) are incorrect as the number of projects or the number of people involved is
immaterial to the discussion. Option (d) has no relevance to the discussion and the time period is too
long for it to be a good alternate.
111. Options (c) cannot be reasonably inferred from the passage. Options (a) & (b) are true and can
be traced to various sentences in the passage and therefore, the correct Option is (d).
112. Options (a) & (b) are irrelevant to the premise discussed by the author. Option (d) actually
strengthens the stance taken by the author. Therefore, the correct Option is (c).
113. Only Option (a) is incorrect as the author specifies how the affluence of a country implies a
need for different planning. This is particularly so as the poor people in rich countries often get
ignored.
114. Only option (c) follows from the arguments posed by the author and is in line with the author‘s
thoughts.
115. Both the statements are correct and can be inferred from the information provided in the
passage. Therefore, the correct answer is option (c).
116. Premises are the facts or evidence that support or lead to the conclusion. Therefore, the above
statement forms premise of the passage which leads to the conclusion. Hence, option (a) is the
correct answer.
117. Options (a), (b) & (c) find proper references in the passage. They also weaken the stance taken
by the author as they bring out the ineffectiveness of the program.
Therefore, the correct answer is Option (d).
118. As the author has repeatedly pointed out, microfinancing is infamous for failing in certain
countries and that is mostly associated to ignorance of certain features unique to that country.
Therefore, a uniform approach is very damaging and may even be responsible for its failure.
Hence, (d)
119. The author is working on the premise that microfinancing schemes are good but they are not
foolproof and sometimes they fail. All the given Options highlight the problems that may be
encountered while enforcing the said schemes.
120. As the author suggests, it has been observed and proven that the microfinancing is not a 100%
effective when it comes to alleviating poverty. Therefore, the correct answer is Option (b).
121. Options (c) & (d) are of no relevance to the discussion. Option (a) actually weakens the stance
taken by the author. Therefore, the correct answer is Option (b) which proves that the microfinancing
scheme may just work as the only thing needed by people is the money
122. The tone of the paragraph is optimistic throughout. Hence choice (b) that talks of auguring well
fits well. All the other options are pessimistic, which is not in keeping with the paragraph.
Hence, choice (b)
123. any option that questions the definition of ―badly managed‖ will weaken the argument. (a) and
(c) will, to some extent, strengthen the argument. (d) will not directly weaken as the private sector
might be badly managed too. (b) will challenge it by giving another reason for the price hike and not
blaming bad management as the sole reason for this.
Hence, (b)

124. STTIBUDIONRI

125. R
Given wordUNPRECEDENTED
After RearrangementDETNEDECERPNU
Third letter from the left after rearrangement is T, so seventh letter to the right of T is R.
126. Wife of my father-in-law is my mother-in-law, and the only daughter of my mother-in-law is my
wife. Hence, the lady is the wife of Vikram.
127. Option (c) is incorrect as it is unfounded in the passage. Options (a) & (b) are both correct and
have been discussed by the author. Therefore, the correct Option is (d).
128. Option (c) & (d) are incorrect as per the premise established by the author in the passage.
Option (b) relates to the topic of discussion but it‘s not the point the author is trying to make.
Therefore, the correct answer is Option (a).
129. None of the statements negatively affect the propositions stated by the author or weaken the
stance taken above.
130. Premises are the facts or evidence that support or lead to the conclusion. Therefore, the above
statement forms premise of the passage which leads to the conclusion. Hence, option (a) is the
correct answer.
131. Option (c) covers the theme of the discussion and captures the authors view on the topic as
well about the participation of local community in saving forests and how it is not the most ideal
method. Therefore, the correct answer is option (c).
132.

H is husband of A. C is husband of D. E is husband of B.


(4) Option (4) is true.
133.

H is husband of A. C is husband of D. E is husband of B.


(c) E is husband of B.
134.

H is husband of A. C is husband of D. E is husband of B.


(c) Two persons
135.

H is husband of A. C is husband of D. E is husband of B.


(a) D is wife of C.
136.

H is husband of A. C is husband of D. E is husband of B.


(d) Second to the right.

QUANTITATIVE TECHNIQUES

137.

138.
139.

140.

141. maximum number of articles manufactured from both companies together is (30 + 35) = 65
lakhs in both years 2005 and 2007.
142.

143.

144.

145. Required period = 1980 - 81


146. Required period = 1982 – 83
147.

148.

149.

150.
Legal Edge 26 Mock

Answers & Details

ENGLISH

1. The above-given passage has been written in the form of a memoir of the narrator. A memoir is
any nonfiction narrative writing based on the author''s memories. The assertions made in the work
are thus understood to be factual.
It can‘t be a commentary as a commentary is a line-by-line or even word-by-word explication usually
attached to an edition of a text in the same or an accompanying volume. The description in no way
fits the passage.
We can say for certain that it is not reporting. The past of the narrator is being recounted here.
Reporting means narrating the present.
Thus, without doubt, the answer is option (a)
2. The narrator says that he has to use his intelligence to dodge the police officials when he carried
out the work for his mother because they were against the business. This helps to come to the
inference that his mother‘s work was illegal. Thus, option (a) can be correct.
It was illegal but it didn‘t have any political opposition as has been mentioned by the author. Thus,
we can say for sure that though it was forbidden by law it was not that sensitive an issue when it
comes to politics. Thus, option (b) is incorrect.
Thus, the answer is option (a).
3. Question Type – Universal
Level - Difficult
Option (a) can be inferred from the passage because the narrator says that his father‘s business
was less popular than his mother‘s. However, many people who were dog-owners didn‘t approve of
this, yet, it was a very effective medicine.
Option (b) can be easily inferred as the narrator says that his father had as his partners many
doctors of the town who would prescribe dog-oil in the name of Ol. can.
Option (c) can be inferred because the narrator says ―Looking back upon those days, I cannot but
regret, at times, that by indirectly bringing my beloved parents to their death I was the author of
misfortunes profoundly affecting my future.‖
Option (d) is difficult to be inferred as the doctors never prescribed dog-oil, but Ol. can. This means
dog-oil was prescribed in a hidden way. Thus, maybe nobody or at max maybe just a few knew that
Ol. can contained dog-oil. However, the doctors of the town fully knew that it has dog-oil.
Thus, option (d) is the answer.
4. The narrator had a twisted mind. As a child, he liked playing with them but also procured the dogs
for his father‘s dog-oil business. That is why he was forbidden by many in the town to play with fatty
dogs. We can understand his feeling as the narrator mentions in the passage that this pained his
young sensibilities. Thus, we can say for sure that he liked playing with them.
Thus, option (b) is the answer.
5. Option (a) can be inferred from the passage as the narrator justifies his actions of throwing babies
into the river by saying that that is the reason why nature has put the river in the first place. This is a
twisted thought, but, that‘s how the narrator‘s character has been carved. Plus, he has been doing
that since his birth – assisting his mother in disposing of ―unwelcome babies‖. Option (a) can be the
answer.
Option (b) can be inferred from the narrator‘s thought "it cannot greatly matter if I put it into this
cauldron. My father will never know the bones from those of a puppy, and the few deaths which may
result from administering another kind of oil for the incomparable ol. can. are not important in a
population which increases so rapidly."
Now consider the fact that he was comprehending this thought when he saw dog-oil being formed
from the dogs that he procured while holding the body of a baby that his mother has killed. He had
no problems in throwing the body of the baby in the furnace and if a few people die of due to some
adverse effects due to this, he justified that it wouldn‘t matter in a world whose population is
increasing.
Thus, the answer is option (c).
6. Indolent means averse to activity, effort, or movement or habitually lazy.
Languid means having or showing a disinclination for physical exertion or effort.
Ameliorate means make (something bad or unsatisfactory) better.
Doggedly means in a manner that shows tenacity and grim persistence.
Unwieldy means difficult to move because of its size, shape, or weight.
Thus, the answer is option (a)
7. Option (a) can be taken away from the passage because the narrator‘s mother‘s business was
illegal and still she used to carry out the same and had her son assist her. This means sections of
the society also had a part to play in this.
Option (b) can be taken away as the author says ―In my boyhood, I was trained to habits of industry;
I assisted my father in procuring dogs for his vats and my mother in carrying away the debris of her
work in the studio.‖
Option (c) can be taken away as the author says that his mother‘s studio was in the shadow of the
church.
Option (d) is not correct as the narrator says that his parents were honest, yet, we see his mother
pursuing an illegal business; his father sometimes catching dogs of dog-owners for his business.
And what‘s worse is they have dragged their child into this business since his childhood.
Thus, option (d) is the answer.
8. Before discussing the options, one thing needs to be made clear. While Kamal Hassan has
mentioned the word homemakers, which can include women and men, Kangana has looked at it
narrowly through the gender point-of-view and limited the activity of homemaking to only belonging
to the women. Tharoor and Hassan have mostly talked about the work of homemaking, which has
been unaccounted for in the nation‘s monetary quantification.
Option (a) is wrong and is a highly sexist thought, that what Kangana thinks is a reflection of what all
women will think.
Option (b) is wrong, as maybe there can be a semblance of somewhat of a philosophy in Ranaut‘s
tweet, Tharoor‘s response can‘t be ignored by saying that it is narrow.
Option (c) is wrong neither has Ranaut taken a family-member point of view nor has Tharoor taken
up any political standpoint.
Option (d) is correct as it corresponds to the above-given explanation of the difference between the
approach of Tharoor and Ranaut. Thus, option (d) is the answer.
9. That the issue has become a very popular one and that it has cut across political lines can be
understood from the fact that after Hassan‘s statement, both DMK and AIADMK have announced the
same in their manifestos. Thus, option (a) is correct.
In impulse, we might think that option (b) can be correct because the author says that no party
including Hassan‘s have made an announcement of the source of funds for this. However, it might
be the case that the parties might not have made it public as of yet. Thus, we can‘t be sure if the
author would agree on this.
Thus, the answer is option (a).
10. The question asks about the origin of gender disparities. Thus, we have to go back before the
time of the Constitution, also, the author mentions that our Constitution has given equal rights to
men and women. So, option (b) is incorrect.
We can‘t say that patriarchal society is the origin of gender disparity. It is a cause though. Patriarchal
society has been caused due to practice followed and documents found in our ancient times. The
author points out the example of Manusmriti to highlight the same. Thus, the patriarchal society is a
result of the ancient times which is the reason for gender disparities.
Thus only option (c) is correct and hence will be our answer.
Question Type – Universal
Level - Difficult
11. Option (a) can be inferred from the passage, because of the statistic shown in the passage.
While the value of women‘s unpaid household work amounts to nearly 40% of India‘s current GDP,
globally, it is 13% of the economy. Thus, this work must be monetarily rewarding abroad.
Option (b) can‘t be inferred as it will be wrong to say that Kangana doesn‘t support female
empowerment and independence of women. She has looked at the issue in a different perspective.
Option (c) is easily inferrable as the statistics provided in the passage is enough.
Option (d) is inferrable because of the statistics that women spend more time on household chores
than on studying in India.
Thus, the answer is option (b).
12. The correct answer is option c). The passage states that, ‗The highest absolute figures for food
waste were unsurprisingly recorded in the two countries with populations of more than a billion
people. China wastes an estimated 91.6 million tonnes of food per year while India discards 68.8
million tonnes. The US is a little further down the ranking with 19.4 million tonnes of food waste while
in Europe, France and Germany produce between five and six million tonnes annually.‘
13. the correct answer is option b). the passage clearly states that, ‗According to the United Nations
Environment Programme‘s 2021 food waste index, the world produces 931 million tonne of food
waste every year, of which 569 tonne comes from households. The remaining amount is attributed to
the food service (244 million tonne) and retail sectors (118 million tonne).‘
14. The correct answer is option c). The passage revolves around providing statistical information
which is a main characteristic of Expository writing style.
15. the correct answer is option a). Per capita is a Latin term that translates to "by head." Per capita
means the average per person and is often used in place of "per person" in statistical observances.
The phrase is used with economic data or reporting but is also applied to almost any other
occurrence of population description.
16. the correct answer is option a). the proper meaning of the word ‗attributed‘ is ‗regard something
as being caused by.‘
17. The correct answer is option a).
Statement 1 is the correct answer as the passage mentions in the last paragraph that, ‗ .. The party
is in disarray. It is bereft of cadre and leaders on the ground…‘
Statement 2 is not the correct statement as the passage mentions in the last paragraph that,
‗..Ironically, this comes when the Congress has three family members active in politics for the first
time in recent years..‘ Hence the correct answer is option a).
18. The correct answer is option b). The last paragraph mentions that, ‗Rahul Gandhi was recently
seen trying to cultivate a man of the masses image by diving into the sea and joining students in
push-up contests. Priyanka Gandhi hit the ground running in Assam with photo-ops of her picking
tea, trying to make a dent on her party‘s behalf. But it could be a case of too little, too late. The party
is in disarray. It is bereft of cadre and leaders on the ground.‘ The author meant that Congress could
have made a strong impact on people‘s mind if they had put more efforts a lot earlier at the
grassroots level.
Option (a) is not the right answer as there were no ideological differences among the G23 members
of Congress. The passage mentions that, ‗To compound its woes, seven Congress leaders,
including two former CMs — Bhupinder Singh Hooda and Ghulam Nabi Azad — and former
ministers like Raj Babbar, Manish Tewari and Anand Sharma, held a rally in Jammu last week,
declaring the party was weakening and they had come together to strengthen it. These leaders are
part of a group of 23 Congressmen, the so-called G23..‘
Option (c) is not the right answer as the author didn‘t mention it along the lines of how the Narendra
Modi-Amit Shah duo has been affecting common people in recent times.
Option (d) is not the right answer as the idea of portraying Sonia Gandhi‘s role in the organizational
elections and more active and available leadership has nothing to do with the above said sentence
that the author used.
19. The passage is based on the current scenario of the Indian National Congress as a national level
political party. The author discusses the party‘s organizational issues and other irregularities which
have failed them to reach out and relate to the common people at the ground level.
All of the above options are definitions of the word dent, used in different contexts. The author used
the word ‗dent‘ in the sentence: ‗Priyanka Gandhi hit the ground running in Assam with photo-ops of
her picking tea, trying to make a dent on her party‘s behalf .‘ Thus, options (a) and (b) and (c) are
incorrect. Option (d) defines ‗dent‘ correctly in context with the sentence and also fits the context of
the passage. Thus, option (d) is the answer.
20. Wit is a literary device used to make the readers laugh. It is, in fact, a clever expression of
thought; whether harmless or aggressive, with or without any disparaging intent toward something or
someone. So, option a) is correct.
Humour is a literary tool that makes audiences laugh, or that intends to induce amusement or
laughter. Its purpose is to break the monotony, boredom, and tedium, and make the audience‘s
nerves relax. So option b) is incorrect.
Anaphora is a rhetorical device that features a repetition of a word or phrase at the beginning of
successive sentences, phrases, or clauses. So, option (c) is incorrect.
Fallacy is an erroneous argument dependent upon an unsound or illogical contention. There are
many fallacy examples that we can find in everyday conversations. So, option (d) is incorrect.
There can be confusion between options (a) and (b), but it is option (a) as humour has many devices
and none of them would fit in the italicised line. The line gives us the information about
Congress party‘s present role as an opposition party but in a fun way. Thus, it is wit. Hence, the
answer will be option (a).
21. The correct line is -
― The Congress party, or whatever remains of it, still believes their meal ticket lies with the Gandhis‖
22. The correct answer is option a) as the entire passage is just a narration of a tragic event
surrounding death and decay for which the author feels responsible as the last line suggests.
23. An aphorism is a concise, terse, laconic, or memorable expression of a general truth or principle.
They are often handed down by tradition from generation to generation. Thus, option (a) is not the
answer.
An allegory is a literary device, an allegory is a narrative in which a character, place, or event is used
to deliver a broader message about real-world issues and occurrences. It can be a story, poem, or
picture that can be interpreted to reveal a hidden meaning, typically a moral or political one. The
passage is an apt allegory. Option (b) is the answer.
A parable is a succinct, didactic story, in prose or verse, that illustrates one or more instructive
lessons or principles. It differs from a fable in that fables employ animals, plants, inanimate objects,
or forces of nature as characters, whereas parables have human characters. Thus, option (c) can‘t
be the answer.
In English literature, an elegy is a poem of serious reflection, usually a lament for the dead. Thus,
option (d) can‘t be the answer.
24. Option (a) is not correct as metaphor is a figure of speech that makes a comparison between two
unlike things. As a literary device, metaphor creates implicit comparisons without the express use of
―like‖ or ―as.‖ Whereas we see an abundance of usage of ―like‖ or ―as‖.
Option (b) is correct as simile is a figure of speech that directly compares two things. Similes differ
from metaphors by highlighting the similarities between two things using words such as "like" or "as",
while metaphors create an implicit comparison. Thus, option (b) is the answer.
Option (c) is not correct as ellipsis is a literary device that is used in narratives to omit some parts of
a sentence or event, which gives the reader a chance to fill the gaps while acting or reading it out. It
is usually written between the sentences as a series of three dots, like this: ―…‖
Option (d) is not correct synesthesia refers to a technique adopted by writers to present ideas,
characters, or places in such a manner that they appeal to more than one sense, like hearing, sight,
smell, and touch at a given time.
25. The correct line is -
―As I surveyed the colours of the canvas before me, a few patches of yellow stood out amongst the
different shades of green.‖
26. The author first thought that he was thankful that poison that saves crops exist as the passage
mentions: ‗Soon healthy green would turn sickly brown and we could lose up to 50 per cent of the
expected yield.
Thank God for poisons. The next day the entire farm was treated to a granular insecticide called
Hevvy Vate 10G .But, then he realised and felt guilty that he was responsible for the death and
devastation among the wildlife around the fields caused by it .So, statement 1 is wrong. Thus,
options (a) or (c) can be the answers. Option (d) can‘t be so.
Thus, option (c) is the answer.
27. Section 230 as is mentioned in the passage shields the companies from the liability for the user
generated content if it is related to hate speech or violence. Therefore, b is the correct answer.
28. The Answer could be clearly inferred from this para from the passage:
―Transparency can help hold the companies accountable as to what accuracy and effectiveness
they‘re achieving,‖ Zuckerberg told Congress on Thursday. Facebook wouldn‘t have to change much
if such a system was the industry norm, he added. ―As a model, Facebook has been doing
something to this effect for every quarter.‖
29. The author points out that the AI takes only the denominator of the equation and does not report
the total number of hate crimes and therefore, it is not as transparent as it sound. The Correct
answer is C.
30. Social Media companies were criticized for playing a part in spreading misinformation that fuelled
the Capitol riots in January. The correct answer is c.
31. Option d cannot be inferred from the passage as there is nothing which suggests in the passage
that Social Media proved useful at the time when lockdown was placed. The correct answer is d.

GENERAL KNOWLEDGE

32. The 46th UNHRC Session was held recently from 22 February to 24 March 2021. The UN Human
Rights Council on Tuesday adopted a strong resolution against Sri Lanka‘s rights record, in a
setback to Colombo, which described the move as ―unwarranted and unjustified‖ interference in its
internal affairs.
The resolution titled Promotion of Reconciliation Accountability and Human Rights in Sri Lanka‘ was
adopted by the 47-member Council after 22 members voted in favour of it at the ongoing UNHRC
session here. 14 countries which abstained from voting. Eleven countries voted against the
resolution.
India, which abstained from voting, said its approach to the question of human rights in Sri Lanka is
guided by two fundamental considerations. One is India‘s support to the Tamils of Sri Lanka for
equality, justice, dignity and peace. The other is in ensuring the unity, stability and territorial integrity
of Sri Lanka. India has been against the formation of the new Constitution of Sri Lanka. Thus, option
(c) is not a consideration that has guided India‘s stand at the UNHRC.
The Thirteenth Amendment to the Constitution of Sri Lanka (13A) is amendment to the Constitution
of Sri Lanka which created Provincial Councils in Sri Lanka. This amendment also made Sinhala and
Tamil as the official languages of the country and English as the "link language". The
20th Amendment has been in news recently with the present Sri Lankan Government trying to bring
in the same amendment that would bring back unitary features in the Sri Lankan governance. India
has always asked Sri Lanka to implement the 13th Amendment.
The draft resolution was in response to a scathing report released by the OHCHR on January 27.
The UN report had warned that lack of accountability of Sri Lanka‘s previous violations had not only
increased the risk of replicating those crimes. It also highlighted ―worrying trends over the past year,
such as deepening impunity, increasing militarisation of governmental functions, ethno-nationalist
rhetoric, and intimidation of civil society‖. The report has not named the Tamils per se. Thus, 1 and 2
are correct.
33. The 46th UNHRC Session was held recently from 22 February to 24 March 2021. The UN Human
Rights Council on Tuesday adopted a strong resolution against Sri Lanka‘s rights record, in a
setback to Colombo, which described the move as ―unwarranted and unjustified‖ interference in its
internal affairs.
The resolution titled Promotion of Reconciliation Accountability and Human Rights in Sri Lanka‘ was
adopted by the 47-member Council after 22 members voted in favour of it at the ongoing UNHRC
session here. 14 countries which abstained from voting. Eleven countries voted against the
resolution.
India, which abstained from voting, said its approach to the question of human rights in Sri Lanka is
guided by two fundamental considerations. One is India‘s support to the Tamils of Sri Lanka for
equality, justice, dignity and peace. The other is in ensuring the unity, stability and territorial integrity
of Sri Lanka. India has been against the formation of the new Constitution of Sri Lanka. Thus, option
(c) is not a consideration that has guided India‘s stand at the UNHRC.
The Thirteenth Amendment to the Constitution of Sri Lanka (13A) is amendment to the Constitution
of Sri Lanka which created Provincial Councils in Sri Lanka. This amendment also made Sinhala and
Tamil as the official languages of the country and English as the "link language". The
20th Amendment has been in news recently with the present Sri Lankan Government trying to bring
in the same amendment that would bring back unitary features in the Sri Lankan governance. India
has always asked Sri Lanka to implement the 13th Amendment.
The draft resolution was in response to a scathing report released by the OHCHR on January 27.
The UN report had warned that lack of accountability of Sri Lanka‘s previous violations had not only
increased the risk of replicating those crimes. It also highlighted ―worrying trends over the past year,
such as deepening impunity, increasing militarisation of governmental functions, ethno-nationalist
rhetoric, and intimidation of civil society‖. The report has not named the Tamils per se. Thus, 1 and 2
are correct.
34. The 46th UNHRC Session was held recently from 22 February to 24 March 2021. The UN Human
Rights Council on Tuesday adopted a strong resolution against Sri Lanka‘s rights record, in a
setback to Colombo, which described the move as ―unwarranted and unjustified‖ interference in its
internal affairs.
The resolution titled Promotion of Reconciliation Accountability and Human Rights in Sri Lanka‘ was
adopted by the 47-member Council after 22 members voted in favour of it at the ongoing UNHRC
session here. 14 countries which abstained from voting. Eleven countries voted against the
resolution.
India, which abstained from voting, said its approach to the question of human rights in Sri Lanka is
guided by two fundamental considerations. One is India‘s support to the Tamils of Sri Lanka for
equality, justice, dignity and peace. The other is in ensuring the unity, stability and territorial integrity
of Sri Lanka. India has been against the formation of the new Constitution of Sri Lanka. Thus, option
(c) is not a consideration that has guided India‘s stand at the UNHRC.
The Thirteenth Amendment to the Constitution of Sri Lanka (13A) is amendment to the Constitution
of Sri Lanka which created Provincial Councils in Sri Lanka. This amendment also made Sinhala and
Tamil as the official languages of the country and English as the "link language". The
20th Amendment has been in news recently with the present Sri Lankan Government trying to bring
in the same amendment that would bring back unitary features in the Sri Lankan governance. India
has always asked Sri Lanka to implement the 13th Amendment.
The draft resolution was in response to a scathing report released by the OHCHR on January 27.
The UN report had warned that lack of accountability of Sri Lanka‘s previous violations had not only
increased the risk of replicating those crimes. It also highlighted ―worrying trends over the past year,
such as deepening impunity, increasing militarisation of governmental functions, ethno-nationalist
rhetoric, and intimidation of civil society‖. The report has not named the Tamils per se. Thus, 1 and 2
are correct.
35. The 46th UNHRC Session was held recently from 22 February to 24 March 2021. The UN Human
Rights Council on Tuesday adopted a strong resolution against Sri Lanka‘s rights record, in a
setback to Colombo, which described the move as ―unwarranted and unjustified‖ interference in its
internal affairs.
The resolution titled Promotion of Reconciliation Accountability and Human Rights in Sri Lanka‘ was
adopted by the 47-member Council after 22 members voted in favour of it at the ongoing UNHRC
session here. 14 countries which abstained from voting. Eleven countries voted against the
resolution.
India, which abstained from voting, said its approach to the question of human rights in Sri Lanka is
guided by two fundamental considerations. One is India‘s support to the Tamils of Sri Lanka for
equality, justice, dignity and peace. The other is in ensuring the unity, stability and territorial integrity
of Sri Lanka. India has been against the formation of the new Constitution of Sri Lanka. Thus, option
(c) is not a consideration that has guided India‘s stand at the UNHRC.
The Thirteenth Amendment to the Constitution of Sri Lanka (13A) is amendment to the Constitution
of Sri Lanka which created Provincial Councils in Sri Lanka. This amendment also made Sinhala and
Tamil as the official languages of the country and English as the "link language". The
20th Amendment has been in news recently with the present Sri Lankan Government trying to bring
in the same amendment that would bring back unitary features in the Sri Lankan governance. India
has always asked Sri Lanka to implement the 13th Amendment.
The draft resolution was in response to a scathing report released by the OHCHR on January 27.
The UN report had warned that lack of accountability of Sri Lanka‘s previous violations had not only
increased the risk of replicating those crimes. It also highlighted ―worrying trends over the past year,
such as deepening impunity, increasing militarisation of governmental functions, ethno-nationalist
rhetoric, and intimidation of civil society‖. The report has not named the Tamils per se. Thus, 1 and 2
are correct.
36. The 46th UNHRC Session was held recently from 22 February to 24 March 2021. The UN Human
Rights Council on Tuesday adopted a strong resolution against Sri Lanka‘s rights record, in a
setback to Colombo, which described the move as ―unwarranted and unjustified‖ interference in its
internal affairs.
The resolution titled Promotion of Reconciliation Accountability and Human Rights in Sri Lanka‘ was
adopted by the 47-member Council after 22 members voted in favour of it at the ongoing UNHRC
session here. 14 countries which abstained from voting. Eleven countries voted against the
resolution.
India, which abstained from voting, said its approach to the question of human rights in Sri Lanka is
guided by two fundamental considerations. One is India‘s support to the Tamils of Sri Lanka for
equality, justice, dignity and peace. The other is in ensuring the unity, stability and territorial integrity
of Sri Lanka. India has been against the formation of the new Constitution of Sri Lanka. Thus, option
(c) is not a consideration that has guided India‘s stand at the UNHRC.
The Thirteenth Amendment to the Constitution of Sri Lanka (13A) is amendment to the Constitution
of Sri Lanka which created Provincial Councils in Sri Lanka. This amendment also made Sinhala and
Tamil as the official languages of the country and English as the "link language". The
20th Amendment has been in news recently with the present Sri Lankan Government trying to bring
in the same amendment that would bring back unitary features in the Sri Lankan governance. India
has always asked Sri Lanka to implement the 13th Amendment.
The draft resolution was in response to a scathing report released by the OHCHR on January 27.
The UN report had warned that lack of accountability of Sri Lanka‘s previous violations had not only
increased the risk of replicating those crimes. It also highlighted ―worrying trends over the past year,
such as deepening impunity, increasing militarisation of governmental functions, ethno-nationalist
rhetoric, and intimidation of civil society‖. The report has not named the Tamils per se. Thus, 1 and 2
are correct.
37. Synthetic-aperture radar (SAR) is a form of radar that is used to create two-dimensional images
or three-dimensional reconstructions of objects, such as landscapes. SAR uses the motion of the
radar antenna over a target region to provide finer spatial resolution than conventional beam-
scanning radars.
NISAR will scan the globe every 12 days over the course of its three-year mission of imaging the
Earth‘s land, ice sheets and sea ice to give an unprecedented view of the planet.
The partnership agreement was signed between NASA and ISRO in September 2014, according to
which NASA will provide one of the radars for the satellite, a high-rate communication subsystem for
science data, GPS receivers and a payload data subsystem. ISRO, on the other hand, will provide
the spacecraft bus, the second type of radar (called the S-band radar), the launch vehicle and
associated launch services. Significantly, NISAR will be equipped with the largest reflector antenna
ever launched by NASA and its primary goals include tracking subtle changes in the Earth‘s surface,
spotting warning signs of imminent volcanic eruptions, helping to monitor groundwater supplies and
tracking the rate at which ice sheets are melting.
The GSLV Mk-II will be used to launch NISAR. The satellite will be launched in 2022 from the Satish
Dhawan Space Center in Sriharikota, India, into a near-polar orbit. Because of the precision of SAR,
the radar can penetrate clouds and darkness, which means that it can collect data day and night in
any weather.
The primary goals of NISAR are –
• Tracking subtle changes in the Earth‘s surface,
• Spotting warning signs of imminent volcanic eruptions,
• Helping to monitor groundwater supplies, and
• Tracking the rate at which ice sheets are melting. It is a satellite that will help study the earth‘s
surface and not the weather or climate.
38. Synthetic-aperture radar (SAR) is a form of radar that is used to create two-dimensional images
or three-dimensional reconstructions of objects, such as landscapes. SAR uses the motion of the
radar antenna over a target region to provide finer spatial resolution than conventional beam-
scanning radars.
NISAR will scan the globe every 12 days over the course of its three-year mission of imaging the
Earth‘s land, ice sheets and sea ice to give an unprecedented view of the planet.
The partnership agreement was signed between NASA and ISRO in September 2014, according to
which NASA will provide one of the radars for the satellite, a high-rate communication subsystem for
science data, GPS receivers and a payload data subsystem. ISRO, on the other hand, will provide
the spacecraft bus, the second type of radar (called the S-band radar), the launch vehicle and
associated launch services. Significantly, NISAR will be equipped with the largest reflector antenna
ever launched by NASA and its primary goals include tracking subtle changes in the Earth‘s surface,
spotting warning signs of imminent volcanic eruptions, helping to monitor groundwater supplies and
tracking the rate at which ice sheets are melting.
The GSLV Mk-II will be used to launch NISAR. The satellite will be launched in 2022 from the Satish
Dhawan Space Center in Sriharikota, India, into a near-polar orbit. Because of the precision of SAR,
the radar can penetrate clouds and darkness, which means that it can collect data day and night in
any weather.
The primary goals of NISAR are –
• Tracking subtle changes in the Earth‘s surface,
• Spotting warning signs of imminent volcanic eruptions,
• Helping to monitor groundwater supplies, and
• Tracking the rate at which ice sheets are melting. It is a satellite that will help study the earth‘s
surface and not the weather or climate.
39. Synthetic-aperture radar (SAR) is a form of radar that is used to create two-dimensional images
or three-dimensional reconstructions of objects, such as landscapes. SAR uses the motion of the
radar antenna over a target region to provide finer spatial resolution than conventional beam-
scanning radars.
NISAR will scan the globe every 12 days over the course of its three-year mission of imaging the
Earth‘s land, ice sheets and sea ice to give an unprecedented view of the planet.
The partnership agreement was signed between NASA and ISRO in September 2014, according to
which NASA will provide one of the radars for the satellite, a high-rate communication subsystem for
science data, GPS receivers and a payload data subsystem. ISRO, on the other hand, will provide
the spacecraft bus, the second type of radar (called the S-band radar), the launch vehicle and
associated launch services. Significantly, NISAR will be equipped with the largest reflector antenna
ever launched by NASA and its primary goals include tracking subtle changes in the Earth‘s surface,
spotting warning signs of imminent volcanic eruptions, helping to monitor groundwater supplies and
tracking the rate at which ice sheets are melting.
The GSLV Mk-II will be used to launch NISAR. The satellite will be launched in 2022 from the Satish
Dhawan Space Center in Sriharikota, India, into a near-polar orbit. Because of the precision of SAR,
the radar can penetrate clouds and darkness, which means that it can collect data day and night in
any weather.
The primary goals of NISAR are –
• Tracking subtle changes in the Earth‘s surface,
• Spotting warning signs of imminent volcanic eruptions,
• Helping to monitor groundwater supplies, and
• Tracking the rate at which ice sheets are melting. It is a satellite that will help study the earth‘s
surface and not the weather or climate.
40. Synthetic-aperture radar (SAR) is a form of radar that is used to create two-dimensional images
or three-dimensional reconstructions of objects, such as landscapes. SAR uses the motion of the
radar antenna over a target region to provide finer spatial resolution than conventional beam-
scanning radars.
NISAR will scan the globe every 12 days over the course of its three-year mission of imaging the
Earth‘s land, ice sheets and sea ice to give an unprecedented view of the planet.
The partnership agreement was signed between NASA and ISRO in September 2014, according to
which NASA will provide one of the radars for the satellite, a high-rate communication subsystem for
science data, GPS receivers and a payload data subsystem. ISRO, on the other hand, will provide
the spacecraft bus, the second type of radar (called the S-band radar), the launch vehicle and
associated launch services. Significantly, NISAR will be equipped with the largest reflector antenna
ever launched by NASA and its primary goals include tracking subtle changes in the Earth‘s surface,
spotting warning signs of imminent volcanic eruptions, helping to monitor groundwater supplies and
tracking the rate at which ice sheets are melting.
The GSLV Mk-II will be used to launch NISAR. The satellite will be launched in 2022 from the Satish
Dhawan Space Center in Sriharikota, India, into a near-polar orbit. Because of the precision of SAR,
the radar can penetrate clouds and darkness, which means that it can collect data day and night in
any weather.
The primary goals of NISAR are –
• Tracking subtle changes in the Earth‘s surface,
• Spotting warning signs of imminent volcanic eruptions,
• Helping to monitor groundwater supplies, and
• Tracking the rate at which ice sheets are melting. It is a satellite that will help study the earth‘s
surface and not the weather or climate.
41. Synthetic-aperture radar (SAR) is a form of radar that is used to create two-dimensional images
or three-dimensional reconstructions of objects, such as landscapes. SAR uses the motion of the
radar antenna over a target region to provide finer spatial resolution than conventional beam-
scanning radars.
NISAR will scan the globe every 12 days over the course of its three-year mission of imaging the
Earth‘s land, ice sheets and sea ice to give an unprecedented view of the planet.
The partnership agreement was signed between NASA and ISRO in September 2014, according to
which NASA will provide one of the radars for the satellite, a high-rate communication subsystem for
science data, GPS receivers and a payload data subsystem. ISRO, on the other hand, will provide
the spacecraft bus, the second type of radar (called the S-band radar), the launch vehicle and
associated launch services. Significantly, NISAR will be equipped with the largest reflector antenna
ever launched by NASA and its primary goals include tracking subtle changes in the Earth‘s surface,
spotting warning signs of imminent volcanic eruptions, helping to monitor groundwater supplies and
tracking the rate at which ice sheets are melting.
The GSLV Mk-II will be used to launch NISAR. The satellite will be launched in 2022 from the Satish
Dhawan Space Center in Sriharikota, India, into a near-polar orbit. Because of the precision of SAR,
the radar can penetrate clouds and darkness, which means that it can collect data day and night in
any weather.
The primary goals of NISAR are –
• Tracking subtle changes in the Earth‘s surface,
• Spotting warning signs of imminent volcanic eruptions,
• Helping to monitor groundwater supplies, and
• Tracking the rate at which ice sheets are melting. It is a satellite that will help study the earth‘s
surface and not the weather or climate.
42. According to the recently released report of the National Statistic Office, the Indian economy has
recovered itself out of the phase of technical recession and has registered a meagre but a rare in the
present circumstances growth of 0.4% in the third quarter of the present financial year.
It was the recently released Economic Survey that had said that Indian economy will contract 7.7 per
cent in current fiscal and growth to rebound to 11 per cent next fiscal.
In simpler words, a technical recession is two quarters in a row of economic contraction. India had
registered contraction in the first and second quarters of the financial year 2020-21.
In the world at the moment the business sentiments are gloomy and there is no positive cycle
globally. Thus, option (a) is incorrect. Biden‘s win has not boosted any trade-ties with US, thus
option (b) is not correct. The vaccine-drive of India is now the world‘s largest vaccine-drive. As per
the RBI‘s reports it is the vaccine-drive that has surged the markets. Sensex has also recovered
after the first day of the vaccine drive. Thus, option (c) is the answer.
Only India, China (6.5%) and Turkey (6.7%) have registered positive growth in these times.
A W-shaped recovery is a dangerous creature. In this, growth falls and rises, but falls again before
recovering, thus forming a W-like chart. The double-dip depicted by a W-shaped recovery can be
due to the second wave of the pandemic.

Prime Minister Narendra Modi''s government had rolled out plans to fund a huge vaccination drive,
while outlining a slew of tax incentives to boost manufacturing. However, some analysts warn that a
recent rise in crude oil prices and a surge of Covid-19 cases in parts of the country may pose risks to
the nascent recovery. Moreover, some sectors, such as retail, airlines, hotels and hospitality, are still
reeling from the pandemic blow.
The government also revised annual GDP estimates for the fiscal year 2020-21, predicting an 8 per
cent contraction, deeper than an earlier estimate of -7.7 per cent. With further relaxation on activities
throughout the country major business and economic indicators such as GST collections,
manufacturing PMI, forex reserves, railway freight, merchandise exports and passenger vehicle
sales have shown positive sequential in growth in the past couple of months. The Q3 GDP numbers
showed the success of the government‘s initial policy of ―lives over livelihood‖. ―The sharp V- shaped
recovery has been driven by rebounds in both Private Final Consumption Expenditure (PFCE) and
Gross Fixed Capital Formation (GFCF) as a combination of astute handling of the lockdown and a
calibrated fiscal stimulus. India‘s eight core sectors recorded a meagre 0.1% rise in output in
January 2021, propped up by a 5.1% rise in electricity, 2.7% growth in fertilizers and 2.6% growth in
steel production, even as the other five sectors contracted.
43. According to the recently released report of the National Statistic Office, the Indian economy has
recovered itself out of the phase of technical recession and has registered a meagre but a rare in the
present circumstances growth of 0.4% in the third quarter of the present financial year.
It was the recently released Economic Survey that had said that Indian economy will contract 7.7 per
cent in current fiscal and growth to rebound to 11 per cent next fiscal.
In simpler words, a technical recession is two quarters in a row of economic contraction. India had
registered contraction in the first and second quarters of the financial year 2020-21.
In the world at the moment the business sentiments are gloomy and there is no positive cycle
globally. Thus, option (a) is incorrect. Biden‘s win has not boosted any trade-ties with US, thus
option (b) is not correct. The vaccine-drive of India is now the world‘s largest vaccine-drive. As per
the RBI‘s reports it is the vaccine-drive that has surged the markets. Sensex has also recovered
after the first day of the vaccine drive. Thus, option (c) is the answer.
Only India, China (6.5%) and Turkey (6.7%) have registered positive growth in these times.
A W-shaped recovery is a dangerous creature. In this, growth falls and rises, but falls again before
recovering, thus forming a W-like chart. The double-dip depicted by a W-shaped recovery can be
due to the second wave of the pandemic.

Prime Minister Narendra Modi''s government had rolled out plans to fund a huge vaccination drive,
while outlining a slew of tax incentives to boost manufacturing. However, some analysts warn that a
recent rise in crude oil prices and a surge of Covid-19 cases in parts of the country may pose risks to
the nascent recovery. Moreover, some sectors, such as retail, airlines, hotels and hospitality, are still
reeling from the pandemic blow.
The government also revised annual GDP estimates for the fiscal year 2020-21, predicting an 8 per
cent contraction, deeper than an earlier estimate of -7.7 per cent. With further relaxation on activities
throughout the country major business and economic indicators such as GST collections,
manufacturing PMI, forex reserves, railway freight, merchandise exports and passenger vehicle
sales have shown positive sequential in growth in the past couple of months. The Q3 GDP numbers
showed the success of the government‘s initial policy of ―lives over livelihood‖. ―The sharp V- shaped
recovery has been driven by rebounds in both Private Final Consumption Expenditure (PFCE) and
Gross Fixed Capital Formation (GFCF) as a combination of astute handling of the lockdown and a
calibrated fiscal stimulus. India‘s eight core sectors recorded a meagre 0.1% rise in output in
January 2021, propped up by a 5.1% rise in electricity, 2.7% growth in fertilizers and 2.6% growth in
steel production, even as the other five sectors contracted.
44. According to the recently released report of the National Statistic Office, the Indian economy has
recovered itself out of the phase of technical recession and has registered a meagre but a rare in the
present circumstances growth of 0.4% in the third quarter of the present financial year.
It was the recently released Economic Survey that had said that Indian economy will contract 7.7 per
cent in current fiscal and growth to rebound to 11 per cent next fiscal.
In simpler words, a technical recession is two quarters in a row of economic contraction. India had
registered contraction in the first and second quarters of the financial year 2020-21.
In the world at the moment the business sentiments are gloomy and there is no positive cycle
globally. Thus, option (a) is incorrect. Biden‘s win has not boosted any trade-ties with US, thus
option (b) is not correct. The vaccine-drive of India is now the world‘s largest vaccine-drive. As per
the RBI‘s reports it is the vaccine-drive that has surged the markets. Sensex has also recovered
after the first day of the vaccine drive. Thus, option (c) is the answer.
Only India, China (6.5%) and Turkey (6.7%) have registered positive growth in these times.
A W-shaped recovery is a dangerous creature. In this, growth falls and rises, but falls again before
recovering, thus forming a W-like chart. The double-dip depicted by a W-shaped recovery can be
due to the second wave of the pandemic.

Prime Minister Narendra Modi''s government had rolled out plans to fund a huge vaccination drive,
while outlining a slew of tax incentives to boost manufacturing. However, some analysts warn that a
recent rise in crude oil prices and a surge of Covid-19 cases in parts of the country may pose risks to
the nascent recovery. Moreover, some sectors, such as retail, airlines, hotels and hospitality, are still
reeling from the pandemic blow.
The government also revised annual GDP estimates for the fiscal year 2020-21, predicting an 8 per
cent contraction, deeper than an earlier estimate of -7.7 per cent. With further relaxation on activities
throughout the country major business and economic indicators such as GST collections,
manufacturing PMI, forex reserves, railway freight, merchandise exports and passenger vehicle
sales have shown positive sequential in growth in the past couple of months. The Q3 GDP numbers
showed the success of the government‘s initial policy of ―lives over livelihood‖. ―The sharp V- shaped
recovery has been driven by rebounds in both Private Final Consumption Expenditure (PFCE) and
Gross Fixed Capital Formation (GFCF) as a combination of astute handling of the lockdown and a
calibrated fiscal stimulus. India‘s eight core sectors recorded a meagre 0.1% rise in output in
January 2021, propped up by a 5.1% rise in electricity, 2.7% growth in fertilizers and 2.6% growth in
steel production, even as the other five sectors contracted.
45. According to the recently released report of the National Statistic Office, the Indian economy has
recovered itself out of the phase of technical recession and has registered a meagre but a rare in the
present circumstances growth of 0.4% in the third quarter of the present financial year.
It was the recently released Economic Survey that had said that Indian economy will contract 7.7 per
cent in current fiscal and growth to rebound to 11 per cent next fiscal.
In simpler words, a technical recession is two quarters in a row of economic contraction. India had
registered contraction in the first and second quarters of the financial year 2020-21.
In the world at the moment the business sentiments are gloomy and there is no positive cycle
globally. Thus, option (a) is incorrect. Biden‘s win has not boosted any trade-ties with US, thus
option (b) is not correct. The vaccine-drive of India is now the world‘s largest vaccine-drive. As per
the RBI‘s reports it is the vaccine-drive that has surged the markets. Sensex has also recovered
after the first day of the vaccine drive. Thus, option (c) is the answer.
Only India, China (6.5%) and Turkey (6.7%) have registered positive growth in these times.
A W-shaped recovery is a dangerous creature. In this, growth falls and rises, but falls again before
recovering, thus forming a W-like chart. The double-dip depicted by a W-shaped recovery can be
due to the second wave of the pandemic.

Prime Minister Narendra Modi''s government had rolled out plans to fund a huge vaccination drive,
while outlining a slew of tax incentives to boost manufacturing. However, some analysts warn that a
recent rise in crude oil prices and a surge of Covid-19 cases in parts of the country may pose risks to
the nascent recovery. Moreover, some sectors, such as retail, airlines, hotels and hospitality, are still
reeling from the pandemic blow.
The government also revised annual GDP estimates for the fiscal year 2020-21, predicting an 8 per
cent contraction, deeper than an earlier estimate of -7.7 per cent. With further relaxation on activities
throughout the country major business and economic indicators such as GST collections,
manufacturing PMI, forex reserves, railway freight, merchandise exports and passenger vehicle
sales have shown positive sequential in growth in the past couple of months. The Q3 GDP numbers
showed the success of the government‘s initial policy of ―lives over livelihood‖. ―The sharp V- shaped
recovery has been driven by rebounds in both Private Final Consumption Expenditure (PFCE) and
Gross Fixed Capital Formation (GFCF) as a combination of astute handling of the lockdown and a
calibrated fiscal stimulus. India‘s eight core sectors recorded a meagre 0.1% rise in output in
January 2021, propped up by a 5.1% rise in electricity, 2.7% growth in fertilizers and 2.6% growth in
steel production, even as the other five sectors contracted.
46. According to the recently released report of the National Statistic Office, the Indian economy has
recovered itself out of the phase of technical recession and has registered a meagre but a rare in the
present circumstances growth of 0.4% in the third quarter of the present financial year.
It was the recently released Economic Survey that had said that Indian economy will contract 7.7 per
cent in current fiscal and growth to rebound to 11 per cent next fiscal.
In simpler words, a technical recession is two quarters in a row of economic contraction. India had
registered contraction in the first and second quarters of the financial year 2020-21.
In the world at the moment the business sentiments are gloomy and there is no positive cycle
globally. Thus, option (a) is incorrect. Biden‘s win has not boosted any trade-ties with US, thus
option (b) is not correct. The vaccine-drive of India is now the world‘s largest vaccine-drive. As per
the RBI‘s reports it is the vaccine-drive that has surged the markets. Sensex has also recovered
after the first day of the vaccine drive. Thus, option (c) is the answer.
Only India, China (6.5%) and Turkey (6.7%) have registered positive growth in these times.
A W-shaped recovery is a dangerous creature. In this, growth falls and rises, but falls again before
recovering, thus forming a W-like chart. The double-dip depicted by a W-shaped recovery can be
due to the second wave of the pandemic.

Prime Minister Narendra Modi''s government had rolled out plans to fund a huge vaccination drive,
while outlining a slew of tax incentives to boost manufacturing. However, some analysts warn that a
recent rise in crude oil prices and a surge of Covid-19 cases in parts of the country may pose risks to
the nascent recovery. Moreover, some sectors, such as retail, airlines, hotels and hospitality, are still
reeling from the pandemic blow.
The government also revised annual GDP estimates for the fiscal year 2020-21, predicting an 8 per
cent contraction, deeper than an earlier estimate of -7.7 per cent. With further relaxation on activities
throughout the country major business and economic indicators such as GST collections,
manufacturing PMI, forex reserves, railway freight, merchandise exports and passenger vehicle
sales have shown positive sequential in growth in the past couple of months. The Q3 GDP numbers
showed the success of the government‘s initial policy of ―lives over livelihood‖. ―The sharp V- shaped
recovery has been driven by rebounds in both Private Final Consumption Expenditure (PFCE) and
Gross Fixed Capital Formation (GFCF) as a combination of astute handling of the lockdown and a
calibrated fiscal stimulus. India‘s eight core sectors recorded a meagre 0.1% rise in output in
January 2021, propped up by a 5.1% rise in electricity, 2.7% growth in fertilizers and 2.6% growth in
steel production, even as the other five sectors contracted.
47. According to the recently released report of the National Statistic Office, the Indian economy has
recovered itself out of the phase of technical recession and has registered a meagre but a rare in the
present circumstances growth of 0.4% in the third quarter of the present financial year.
It was the recently released Economic Survey that had said that Indian economy will contract 7.7 per
cent in current fiscal and growth to rebound to 11 per cent next fiscal.
In simpler words, a technical recession is two quarters in a row of economic contraction. India had
registered contraction in the first and second quarters of the financial year 2020-21.
In the world at the moment the business sentiments are gloomy and there is no positive cycle
globally. Thus, option (a) is incorrect. Biden‘s win has not boosted any trade-ties with US, thus
option (b) is not correct. The vaccine-drive of India is now the world‘s largest vaccine-drive. As per
the RBI‘s reports it is the vaccine-drive that has surged the markets. Sensex has also recovered
after the first day of the vaccine drive. Thus, option (c) is the answer.
Only India, China (6.5%) and Turkey (6.7%) have registered positive growth in these times.
A W-shaped recovery is a dangerous creature. In this, growth falls and rises, but falls again before
recovering, thus forming a W-like chart. The double-dip depicted by a W-shaped recovery can be
due to the second wave of the pandemic.

Prime Minister Narendra Modi''s government had rolled out plans to fund a huge vaccination drive,
while outlining a slew of tax incentives to boost manufacturing. However, some analysts warn that a
recent rise in crude oil prices and a surge of Covid-19 cases in parts of the country may pose risks to
the nascent recovery. Moreover, some sectors, such as retail, airlines, hotels and hospitality, are still
reeling from the pandemic blow.
The government also revised annual GDP estimates for the fiscal year 2020-21, predicting an 8 per
cent contraction, deeper than an earlier estimate of -7.7 per cent. With further relaxation on activities
throughout the country major business and economic indicators such as GST collections,
manufacturing PMI, forex reserves, railway freight, merchandise exports and passenger vehicle
sales have shown positive sequential in growth in the past couple of months. The Q3 GDP numbers
showed the success of the government‘s initial policy of ―lives over livelihood‖. ―The sharp V- shaped
recovery has been driven by rebounds in both Private Final Consumption Expenditure (PFCE) and
Gross Fixed Capital Formation (GFCF) as a combination of astute handling of the lockdown and a
calibrated fiscal stimulus. India‘s eight core sectors recorded a meagre 0.1% rise in output in
January 2021, propped up by a 5.1% rise in electricity, 2.7% growth in fertilizers and 2.6% growth in
steel production, even as the other five sectors contracted.
48. According to the recently released report of the National Statistic Office, the Indian economy has
recovered itself out of the phase of technical recession and has registered a meagre but a rare in the
present circumstances growth of 0.4% in the third quarter of the present financial year.
It was the recently released Economic Survey that had said that Indian economy will contract 7.7 per
cent in current fiscal and growth to rebound to 11 per cent next fiscal.
In simpler words, a technical recession is two quarters in a row of economic contraction. India had
registered contraction in the first and second quarters of the financial year 2020-21.
In the world at the moment the business sentiments are gloomy and there is no positive cycle
globally. Thus, option (a) is incorrect. Biden‘s win has not boosted any trade-ties with US, thus
option (b) is not correct. The vaccine-drive of India is now the world‘s largest vaccine-drive. As per
the RBI‘s reports it is the vaccine-drive that has surged the markets. Sensex has also recovered
after the first day of the vaccine drive. Thus, option (c) is the answer.
Only India, China (6.5%) and Turkey (6.7%) have registered positive growth in these times.
A W-shaped recovery is a dangerous creature. In this, growth falls and rises, but falls again before
recovering, thus forming a W-like chart. The double-dip depicted by a W-shaped recovery can be
due to the second wave of the pandemic.

Prime Minister Narendra Modi''s government had rolled out plans to fund a huge vaccination drive,
while outlining a slew of tax incentives to boost manufacturing. However, some analysts warn that a
recent rise in crude oil prices and a surge of Covid-19 cases in parts of the country may pose risks to
the nascent recovery. Moreover, some sectors, such as retail, airlines, hotels and hospitality, are still
reeling from the pandemic blow.
The government also revised annual GDP estimates for the fiscal year 2020-21, predicting an 8 per
cent contraction, deeper than an earlier estimate of -7.7 per cent. With further relaxation on activities
throughout the country major business and economic indicators such as GST collections,
manufacturing PMI, forex reserves, railway freight, merchandise exports and passenger vehicle
sales have shown positive sequential in growth in the past couple of months. The Q3 GDP numbers
showed the success of the government‘s initial policy of ―lives over livelihood‖. ―The sharp V- shaped
recovery has been driven by rebounds in both Private Final Consumption Expenditure (PFCE) and
Gross Fixed Capital Formation (GFCF) as a combination of astute handling of the lockdown and a
calibrated fiscal stimulus. India‘s eight core sectors recorded a meagre 0.1% rise in output in
January 2021, propped up by a 5.1% rise in electricity, 2.7% growth in fertilizers and 2.6% growth in
steel production, even as the other five sectors contracted.
49. According to the recently released report of the National Statistic Office, the Indian economy has
recovered itself out of the phase of technical recession and has registered a meagre but a rare in the
present circumstances growth of 0.4% in the third quarter of the present financial year.
It was the recently released Economic Survey that had said that Indian economy will contract 7.7 per
cent in current fiscal and growth to rebound to 11 per cent next fiscal.
In simpler words, a technical recession is two quarters in a row of economic contraction. India had
registered contraction in the first and second quarters of the financial year 2020-21.
In the world at the moment the business sentiments are gloomy and there is no positive cycle
globally. Thus, option (a) is incorrect. Biden‘s win has not boosted any trade-ties with US, thus
option (b) is not correct. The vaccine-drive of India is now the world‘s largest vaccine-drive. As per
the RBI‘s reports it is the vaccine-drive that has surged the markets. Sensex has also recovered
after the first day of the vaccine drive. Thus, option (c) is the answer.
Only India, China (6.5%) and Turkey (6.7%) have registered positive growth in these times.
A W-shaped recovery is a dangerous creature. In this, growth falls and rises, but falls again before
recovering, thus forming a W-like chart. The double-dip depicted by a W-shaped recovery can be
due to the second wave of the pandemic.

Prime Minister Narendra Modi''s government had rolled out plans to fund a huge vaccination drive,
while outlining a slew of tax incentives to boost manufacturing. However, some analysts warn that a
recent rise in crude oil prices and a surge of Covid-19 cases in parts of the country may pose risks to
the nascent recovery. Moreover, some sectors, such as retail, airlines, hotels and hospitality, are still
reeling from the pandemic blow.
The government also revised annual GDP estimates for the fiscal year 2020-21, predicting an 8 per
cent contraction, deeper than an earlier estimate of -7.7 per cent. With further relaxation on activities
throughout the country major business and economic indicators such as GST collections,
manufacturing PMI, forex reserves, railway freight, merchandise exports and passenger vehicle
sales have shown positive sequential in growth in the past couple of months. The Q3 GDP numbers
showed the success of the government‘s initial policy of ―lives over livelihood‖. ―The sharp V- shaped
recovery has been driven by rebounds in both Private Final Consumption Expenditure (PFCE) and
Gross Fixed Capital Formation (GFCF) as a combination of astute handling of the lockdown and a
calibrated fiscal stimulus. India‘s eight core sectors recorded a meagre 0.1% rise in output in
January 2021, propped up by a 5.1% rise in electricity, 2.7% growth in fertilizers and 2.6% growth in
steel production, even as the other five sectors contracted.
50. A video on the new height Mount Everest as being 8,848.86 meters. The world''s highest
mountain is now officially a little higher. After years of debate, China and Nepal finally agreed on a
precise elevation for Mount Everest. The new agreed height of 8,848.86 meters (29,031.69 feet) was
announced at a virtual ceremony.
A reason to re-measure the height of Everest was the devastating earthquake that struck Nepal in
2015. We know that quakes are caused by the shifting of tectonic plates – and researchers have
good reason to believe that they have shifted significantly enough to cause a change in the height of
Everest in 2015.
The range''s total length is some 1,400 miles (2,300 km), and it has an average elevation of more
than 20,000 feet (6,100 metres). The Great Himalayas contain many of the world''s tallest peaks,
including (from west to east) Nanga Parbat, Annapurna, Mount Everest, and Kanchenjunga. There
are 9/10 world‘s highest peaks are located in the Himalayas.
The height of the Himalayas was determined by the Survey of India in 1954, using instruments like
theodolites and chains, with GPS still decades away. The elevation of 8,848 m came to be accepted
in all references worldwide — except by China. Mount Everest rises from the border between Nepal
and China.
New Zealand, which shares a bond with Nepal over the mountain, provided technical assistance. Sir
Edmund Hillary, the first climber on the peak along with Nepal‘s Tenzing Norgay in May 1953,
worked as the mountain‘s undeclared brand ambassador to the world. In May 2019, the New
Zealand government provided Nepal‘s Survey Department (Napi Bibhag) with a Global Navigation
Satellite, and trained technicians. Christopher Pearson, a scientist from University of Otago, travelled
to Nepal on a special assignment.
The passage of the Transgender Persons (Protection of Rights) Bill, 2019, has caused great
disappointment to the transgender community in India that had urged the Rajya Sabha to refer the
Bill to a select committee.
51. A video on the new height Mount Everest as being 8,848.86 meters. The world''s highest
mountain is now officially a little higher. After years of debate, China and Nepal finally agreed on a
precise elevation for Mount Everest. The new agreed height of 8,848.86 meters (29,031.69 feet) was
announced at a virtual ceremony.
A reason to re-measure the height of Everest was the devastating earthquake that struck Nepal in
2015. We know that quakes are caused by the shifting of tectonic plates – and researchers have
good reason to believe that they have shifted significantly enough to cause a change in the height of
Everest in 2015.
The range''s total length is some 1,400 miles (2,300 km), and it has an average elevation of more
than 20,000 feet (6,100 metres). The Great Himalayas contain many of the world''s tallest peaks,
including (from west to east) Nanga Parbat, Annapurna, Mount Everest, and Kanchenjunga. There
are 9/10 world‘s highest peaks are located in the Himalayas.
The height of the Himalayas was determined by the Survey of India in 1954, using instruments like
theodolites and chains, with GPS still decades away. The elevation of 8,848 m came to be accepted
in all references worldwide — except by China. Mount Everest rises from the border between Nepal
and China.
New Zealand, which shares a bond with Nepal over the mountain, provided technical assistance. Sir
Edmund Hillary, the first climber on the peak along with Nepal‘s Tenzing Norgay in May 1953,
worked as the mountain‘s undeclared brand ambassador to the world. In May 2019, the New
Zealand government provided Nepal‘s Survey Department (Napi Bibhag) with a Global Navigation
Satellite, and trained technicians. Christopher Pearson, a scientist from University of Otago, travelled
to Nepal on a special assignment.
The passage of the Transgender Persons (Protection of Rights) Bill, 2019, has caused great
disappointment to the transgender community in India that had urged the Rajya Sabha to refer the
Bill to a select committee.
52. A video on the new height Mount Everest as being 8,848.86 meters. The world''s highest
mountain is now officially a little higher. After years of debate, China and Nepal finally agreed on a
precise elevation for Mount Everest. The new agreed height of 8,848.86 meters (29,031.69 feet) was
announced at a virtual ceremony.
A reason to re-measure the height of Everest was the devastating earthquake that struck Nepal in
2015. We know that quakes are caused by the shifting of tectonic plates – and researchers have
good reason to believe that they have shifted significantly enough to cause a change in the height of
Everest in 2015.
The range''s total length is some 1,400 miles (2,300 km), and it has an average elevation of more
than 20,000 feet (6,100 metres). The Great Himalayas contain many of the world''s tallest peaks,
including (from west to east) Nanga Parbat, Annapurna, Mount Everest, and Kanchenjunga. There
are 9/10 world‘s highest peaks are located in the Himalayas.
The height of the Himalayas was determined by the Survey of India in 1954, using instruments like
theodolites and chains, with GPS still decades away. The elevation of 8,848 m came to be accepted
in all references worldwide — except by China. Mount Everest rises from the border between Nepal
and China.
New Zealand, which shares a bond with Nepal over the mountain, provided technical assistance. Sir
Edmund Hillary, the first climber on the peak along with Nepal‘s Tenzing Norgay in May 1953,
worked as the mountain‘s undeclared brand ambassador to the world. In May 2019, the New
Zealand government provided Nepal‘s Survey Department (Napi Bibhag) with a Global Navigation
Satellite, and trained technicians. Christopher Pearson, a scientist from University of Otago, travelled
to Nepal on a special assignment.
The passage of the Transgender Persons (Protection of Rights) Bill, 2019, has caused great
disappointment to the transgender community in India that had urged the Rajya Sabha to refer the
Bill to a select committee.
53. A video on the new height Mount Everest as being 8,848.86 meters. The world''s highest
mountain is now officially a little higher. After years of debate, China and Nepal finally agreed on a
precise elevation for Mount Everest. The new agreed height of 8,848.86 meters (29,031.69 feet) was
announced at a virtual ceremony.
A reason to re-measure the height of Everest was the devastating earthquake that struck Nepal in
2015. We know that quakes are caused by the shifting of tectonic plates – and researchers have
good reason to believe that they have shifted significantly enough to cause a change in the height of
Everest in 2015.
The range''s total length is some 1,400 miles (2,300 km), and it has an average elevation of more
than 20,000 feet (6,100 metres). The Great Himalayas contain many of the world''s tallest peaks,
including (from west to east) Nanga Parbat, Annapurna, Mount Everest, and Kanchenjunga. There
are 9/10 world‘s highest peaks are located in the Himalayas.
The height of the Himalayas was determined by the Survey of India in 1954, using instruments like
theodolites and chains, with GPS still decades away. The elevation of 8,848 m came to be accepted
in all references worldwide — except by China. Mount Everest rises from the border between Nepal
and China.
New Zealand, which shares a bond with Nepal over the mountain, provided technical assistance. Sir
Edmund Hillary, the first climber on the peak along with Nepal‘s Tenzing Norgay in May 1953,
worked as the mountain‘s undeclared brand ambassador to the world. In May 2019, the New
Zealand government provided Nepal‘s Survey Department (Napi Bibhag) with a Global Navigation
Satellite, and trained technicians. Christopher Pearson, a scientist from University of Otago, travelled
to Nepal on a special assignment.
The passage of the Transgender Persons (Protection of Rights) Bill, 2019, has caused great
disappointment to the transgender community in India that had urged the Rajya Sabha to refer the
Bill to a select committee.
54. A video on the new height Mount Everest as being 8,848.86 meters. The world''s highest
mountain is now officially a little higher. After years of debate, China and Nepal finally agreed on a
precise elevation for Mount Everest. The new agreed height of 8,848.86 meters (29,031.69 feet) was
announced at a virtual ceremony.
A reason to re-measure the height of Everest was the devastating earthquake that struck Nepal in
2015. We know that quakes are caused by the shifting of tectonic plates – and researchers have
good reason to believe that they have shifted significantly enough to cause a change in the height of
Everest in 2015.
The range''s total length is some 1,400 miles (2,300 km), and it has an average elevation of more
than 20,000 feet (6,100 metres). The Great Himalayas contain many of the world''s tallest peaks,
including (from west to east) Nanga Parbat, Annapurna, Mount Everest, and Kanchenjunga. There
are 9/10 world‘s highest peaks are located in the Himalayas.
The height of the Himalayas was determined by the Survey of India in 1954, using instruments like
theodolites and chains, with GPS still decades away. The elevation of 8,848 m came to be accepted
in all references worldwide — except by China. Mount Everest rises from the border between Nepal
and China.
New Zealand, which shares a bond with Nepal over the mountain, provided technical assistance. Sir
Edmund Hillary, the first climber on the peak along with Nepal‘s Tenzing Norgay in May 1953,
worked as the mountain‘s undeclared brand ambassador to the world. In May 2019, the New
Zealand government provided Nepal‘s Survey Department (Napi Bibhag) with a Global Navigation
Satellite, and trained technicians. Christopher Pearson, a scientist from University of Otago, travelled
to Nepal on a special assignment.
The passage of the Transgender Persons (Protection of Rights) Bill, 2019, has caused great
disappointment to the transgender community in India that had urged the Rajya Sabha to refer the
Bill to a select committee.
55. The Bill defines a transgender person as one whose gender does not match the gender
assigned at birth. It includes transmen and trans-women, persons with intersex variations, gender-
queers, and persons with socio-cultural identities, such as kinnar and hijra.
Certificate of identity: A transgender person may make an application to the District Magistrate for a
certificate of identity, indicating the gender as ‗transgender‘.
The Bill does not have any provision for self-determination of gender. The transgender community
has questioned the certificate of identity.
It fails to address the lack of an effective mechanism to enforce the legal prohibition against
discrimination on the ground of gender identity.
It does not make provision for affirmative action in employment or education despite the Supreme
Court‘s mandate in the National Legal Services Authority NALSA v. Union of India (UOI) case
(2014).
The Bill sets out lighter sentences for several criminal offences, such as ―sexual abuse" and
―physical abuse", when they are committed against transgender people.
National Legal Services Authority v. Union of India is a landmark decision by the Supreme Court of
India, which declared transgender people the ''third gender'', affirmed that the fundamental rights
granted under the Constitution of India will be equally applicable to them, and gave them the right to
self-identification of their gender as male, female or third gender.
This judgement has been distinguished as a major step towards gender equality in India. Moreover,
the court also held that because transgenders were treated as socially and economically backward
classes, they will be granted reservations in admissions to educational institutions and jobs.
Ministry of Social Justice and Empowerment has constituted the National Council for Transgender
Persons, under the Transgender Persons (Protection of Rights) Act, 2019.
56. The Bill defines a transgender person as one whose gender does not match the gender
assigned at birth. It includes transmen and trans-women, persons with intersex variations, gender-
queers, and persons with socio-cultural identities, such as kinnar and hijra.
Certificate of identity: A transgender person may make an application to the District Magistrate for a
certificate of identity, indicating the gender as ‗transgender‘.
The Bill does not have any provision for self-determination of gender. The transgender community
has questioned the certificate of identity.
It fails to address the lack of an effective mechanism to enforce the legal prohibition against
discrimination on the ground of gender identity.
It does not make provision for affirmative action in employment or education despite the Supreme
Court‘s mandate in the National Legal Services Authority NALSA v. Union of India (UOI) case
(2014).
The Bill sets out lighter sentences for several criminal offences, such as ―sexual abuse" and
―physical abuse", when they are committed against transgender people.
National Legal Services Authority v. Union of India is a landmark decision by the Supreme Court of
India, which declared transgender people the ''third gender'', affirmed that the fundamental rights
granted under the Constitution of India will be equally applicable to them, and gave them the right to
self-identification of their gender as male, female or third gender.
This judgement has been distinguished as a major step towards gender equality in India. Moreover,
the court also held that because transgenders were treated as socially and economically backward
classes, they will be granted reservations in admissions to educational institutions and jobs.
Ministry of Social Justice and Empowerment has constituted the National Council for Transgender
Persons, under the Transgender Persons (Protection of Rights) Act, 2019.
57. The Bill defines a transgender person as one whose gender does not match the gender
assigned at birth. It includes transmen and trans-women, persons with intersex variations, gender-
queers, and persons with socio-cultural identities, such as kinnar and hijra.
Certificate of identity: A transgender person may make an application to the District Magistrate for a
certificate of identity, indicating the gender as ‗transgender‘.
The Bill does not have any provision for self-determination of gender. The transgender community
has questioned the certificate of identity.
It fails to address the lack of an effective mechanism to enforce the legal prohibition against
discrimination on the ground of gender identity.
It does not make provision for affirmative action in employment or education despite the Supreme
Court‘s mandate in the National Legal Services Authority NALSA v. Union of India (UOI) case
(2014).
The Bill sets out lighter sentences for several criminal offences, such as ―sexual abuse" and
―physical abuse", when they are committed against transgender people.
National Legal Services Authority v. Union of India is a landmark decision by the Supreme Court of
India, which declared transgender people the ''third gender'', affirmed that the fundamental rights
granted under the Constitution of India will be equally applicable to them, and gave them the right to
self-identification of their gender as male, female or third gender.
This judgement has been distinguished as a major step towards gender equality in India. Moreover,
the court also held that because transgenders were treated as socially and economically backward
classes, they will be granted reservations in admissions to educational institutions and jobs.
Ministry of Social Justice and Empowerment has constituted the National Council for Transgender
Persons, under the Transgender Persons (Protection of Rights) Act, 2019.
58. The Bill defines a transgender person as one whose gender does not match the gender
assigned at birth. It includes transmen and trans-women, persons with intersex variations, gender-
queers, and persons with socio-cultural identities, such as kinnar and hijra.
Certificate of identity: A transgender person may make an application to the District Magistrate for a
certificate of identity, indicating the gender as ‗transgender‘.
The Bill does not have any provision for self-determination of gender. The transgender community
has questioned the certificate of identity.
It fails to address the lack of an effective mechanism to enforce the legal prohibition against
discrimination on the ground of gender identity.
It does not make provision for affirmative action in employment or education despite the Supreme
Court‘s mandate in the National Legal Services Authority NALSA v. Union of India (UOI) case
(2014).
The Bill sets out lighter sentences for several criminal offences, such as ―sexual abuse" and
―physical abuse", when they are committed against transgender people.
National Legal Services Authority v. Union of India is a landmark decision by the Supreme Court of
India, which declared transgender people the ''third gender'', affirmed that the fundamental rights
granted under the Constitution of India will be equally applicable to them, and gave them the right to
self-identification of their gender as male, female or third gender.
This judgement has been distinguished as a major step towards gender equality in India. Moreover,
the court also held that because transgenders were treated as socially and economically backward
classes, they will be granted reservations in admissions to educational institutions and jobs.
Ministry of Social Justice and Empowerment has constituted the National Council for Transgender
Persons, under the Transgender Persons (Protection of Rights) Act, 2019.
59. Scientists have detected the most ancient type of molecule in our universe in space for the first
time ever.
Helium hydride ion (HeH+) was the first molecule that formed when, almost 14 billion years ago, the
falling temperatures allowed recombination of the light elements (hydrogen, helium, deuterium and
traces of lithium) produced in the Big Bang.
It is the first type of molecule (first molecular bond) that formed in the universe after the Big Bang.
SOFIA stands for the Stratospheric Observatory for Infrared Astronomy. It is a Boeing 747SP aircraft
that carries a 100-inch telescope to observe the universe while flying between 38,000 and 45,000
feet – the layer of Earth''s atmosphere called the stratosphere.
SOFIA is a modified Boeing 747SP jetliner that flies at altitudes up to 45,000 feet. It is a joint project
of NASA and the German Aerospace Center.
Water molecules, discovered in Clavius Crater in the Moon‘s southern hemisphere. And it is the first
time water has been detected on the sunlit side, showing it is not restricted to the shadowy regions.
This was confirmed by NASA‘s Stratospheric Observatory for Infrared Astronomy (SOFIA).
60. Scientists have detected the most ancient type of molecule in our universe in space for the first
time ever.
Helium hydride ion (HeH+) was the first molecule that formed when, almost 14 billion years ago, the
falling temperatures allowed recombination of the light elements (hydrogen, helium, deuterium and
traces of lithium) produced in the Big Bang.
It is the first type of molecule (first molecular bond) that formed in the universe after the Big Bang.
SOFIA stands for the Stratospheric Observatory for Infrared Astronomy. It is a Boeing 747SP aircraft
that carries a 100-inch telescope to observe the universe while flying between 38,000 and 45,000
feet – the layer of Earth''s atmosphere called the stratosphere.
SOFIA is a modified Boeing 747SP jetliner that flies at altitudes up to 45,000 feet. It is a joint project
of NASA and the German Aerospace Center.
Water molecules, discovered in Clavius Crater in the Moon‘s southern hemisphere. And it is the first
time water has been detected on the sunlit side, showing it is not restricted to the shadowy regions.
This was confirmed by NASA‘s Stratospheric Observatory for Infrared Astronomy (SOFIA).
61. Scientists have detected the most ancient type of molecule in our universe in space for the first
time ever.
Helium hydride ion (HeH+) was the first molecule that formed when, almost 14 billion years ago, the
falling temperatures allowed recombination of the light elements (hydrogen, helium, deuterium and
traces of lithium) produced in the Big Bang.
It is the first type of molecule (first molecular bond) that formed in the universe after the Big Bang.
SOFIA stands for the Stratospheric Observatory for Infrared Astronomy. It is a Boeing 747SP aircraft
that carries a 100-inch telescope to observe the universe while flying between 38,000 and 45,000
feet – the layer of Earth''s atmosphere called the stratosphere.
SOFIA is a modified Boeing 747SP jetliner that flies at altitudes up to 45,000 feet. It is a joint project
of NASA and the German Aerospace Center.
Water molecules, discovered in Clavius Crater in the Moon‘s southern hemisphere. And it is the first
time water has been detected on the sunlit side, showing it is not restricted to the shadowy regions.
This was confirmed by NASA‘s Stratospheric Observatory for Infrared Astronomy (SOFIA).
62. Scientists have detected the most ancient type of molecule in our universe in space for the first
time ever.
Helium hydride ion (HeH+) was the first molecule that formed when, almost 14 billion years ago, the
falling temperatures allowed recombination of the light elements (hydrogen, helium, deuterium and
traces of lithium) produced in the Big Bang.
It is the first type of molecule (first molecular bond) that formed in the universe after the Big Bang.
SOFIA stands for the Stratospheric Observatory for Infrared Astronomy. It is a Boeing 747SP aircraft
that carries a 100-inch telescope to observe the universe while flying between 38,000 and 45,000
feet – the layer of Earth''s atmosphere called the stratosphere.
SOFIA is a modified Boeing 747SP jetliner that flies at altitudes up to 45,000 feet. It is a joint project
of NASA and the German Aerospace Center.
Water molecules, discovered in Clavius Crater in the Moon‘s southern hemisphere. And it is the first
time water has been detected on the sunlit side, showing it is not restricted to the shadowy regions.
This was confirmed by NASA‘s Stratospheric Observatory for Infrared Astronomy (SOFIA).
63. India has been ranked at 94 among 107 countries in the Global Hunger Index (GHI) 2020. In
2019 it was the 102nd position. Comparison with Other Countries: India features behind Nepal (73),
Pakistan (88), Bangladesh (75), Indonesia (70) among others. Out of the total 107 countries, only 13
countries fare worse than India including countries like Rwanda (97), Nigeria (98), Afghanistan (99),
Liberia (102), Mozambique (103), Chad (107) among others.
One, the agriculture output from small and marginal holdings are either stagnant or declining due to
reasons such as reduced soil fertility, fragmented lands or fluctuating market price of farm produce.
Almost 50 million households in India are dependent on these small and marginal holdings.
Second, relative income of one section of people has been on the decline. This has adverse effects
on their capacity to buy adequate food, especially when food prices have been on the rise. Third, the
kind of work a section of people have been doing are less remunerative or there is less opportunity
to get remunerative works. Fourth, the public distribution system (PDS) of the state is not functioning
well or is not accessible to everyone.
Concern Worldwide is Ireland''s largest aid and humanitarian agency. Since its foundation 50 years
ago it has worked in 50 countries. In 2019, Concern helped 28.6 million of the world''s poorest and
most vulnerable people to alleviate poverty and hunger. We responded to 82 different emergencies
in 24 countries.
South Asia and Africa South of Sahara have the worst hunger conditions among global regions, the
GHI found.
64. India has been ranked at 94 among 107 countries in the Global Hunger Index (GHI) 2020. In
2019 it was the 102nd position. Comparison with Other Countries: India features behind Nepal (73),
Pakistan (88), Bangladesh (75), Indonesia (70) among others. Out of the total 107 countries, only 13
countries fare worse than India including countries like Rwanda (97), Nigeria (98), Afghanistan (99),
Liberia (102), Mozambique (103), Chad (107) among others.
One, the agriculture output from small and marginal holdings are either stagnant or declining due to
reasons such as reduced soil fertility, fragmented lands or fluctuating market price of farm produce.
Almost 50 million households in India are dependent on these small and marginal holdings.
Second, relative income of one section of people has been on the decline. This has adverse effects
on their capacity to buy adequate food, especially when food prices have been on the rise. Third, the
kind of work a section of people have been doing are less remunerative or there is less opportunity
to get remunerative works. Fourth, the public distribution system (PDS) of the state is not functioning
well or is not accessible to everyone.
Concern Worldwide is Ireland''s largest aid and humanitarian agency. Since its foundation 50 years
ago it has worked in 50 countries. In 2019, Concern helped 28.6 million of the world''s poorest and
most vulnerable people to alleviate poverty and hunger. We responded to 82 different emergencies
in 24 countries.
South Asia and Africa South of Sahara have the worst hunger conditions among global regions, the
GHI found.
65. India has been ranked at 94 among 107 countries in the Global Hunger Index (GHI) 2020. In
2019 it was the 102nd position. Comparison with Other Countries: India features behind Nepal (73),
Pakistan (88), Bangladesh (75), Indonesia (70) among others. Out of the total 107 countries, only 13
countries fare worse than India including countries like Rwanda (97), Nigeria (98), Afghanistan (99),
Liberia (102), Mozambique (103), Chad (107) among others.
One, the agriculture output from small and marginal holdings are either stagnant or declining due to
reasons such as reduced soil fertility, fragmented lands or fluctuating market price of farm produce.
Almost 50 million households in India are dependent on these small and marginal holdings.
Second, relative income of one section of people has been on the decline. This has adverse effects
on their capacity to buy adequate food, especially when food prices have been on the rise. Third, the
kind of work a section of people have been doing are less remunerative or there is less opportunity
to get remunerative works. Fourth, the public distribution system (PDS) of the state is not functioning
well or is not accessible to everyone.
Concern Worldwide is Ireland''s largest aid and humanitarian agency. Since its foundation 50 years
ago it has worked in 50 countries. In 2019, Concern helped 28.6 million of the world''s poorest and
most vulnerable people to alleviate poverty and hunger. We responded to 82 different emergencies
in 24 countries.
South Asia and Africa South of Sahara have the worst hunger conditions among global regions, the
GHI found.
66. India has been ranked at 94 among 107 countries in the Global Hunger Index (GHI) 2020. In
2019 it was the 102nd position. Comparison with Other Countries: India features behind Nepal (73),
Pakistan (88), Bangladesh (75), Indonesia (70) among others. Out of the total 107 countries, only 13
countries fare worse than India including countries like Rwanda (97), Nigeria (98), Afghanistan (99),
Liberia (102), Mozambique (103), Chad (107) among others.
One, the agriculture output from small and marginal holdings are either stagnant or declining due to
reasons such as reduced soil fertility, fragmented lands or fluctuating market price of farm produce.
Almost 50 million households in India are dependent on these small and marginal holdings.
Second, relative income of one section of people has been on the decline. This has adverse effects
on their capacity to buy adequate food, especially when food prices have been on the rise. Third, the
kind of work a section of people have been doing are less remunerative or there is less opportunity
to get remunerative works. Fourth, the public distribution system (PDS) of the state is not functioning
well or is not accessible to everyone.
Concern Worldwide is Ireland''s largest aid and humanitarian agency. Since its foundation 50 years
ago it has worked in 50 countries. In 2019, Concern helped 28.6 million of the world''s poorest and
most vulnerable people to alleviate poverty and hunger. We responded to 82 different emergencies
in 24 countries.
South Asia and Africa South of Sahara have the worst hunger conditions among global regions, the
GHI found.

LEGAL REASONING

67. The passage states that a woman may take resort to domestic violence laws. In this case, the
woman does not identify as a woman anymore, however for the purposes of the law, it is sufficient
that she was born a woman.
68. The passage states that yelling and refusal of eatables is cruelty. However, a reasonable
construction of the same must be made and yelling once is not sufficient to get the benefit of
domestic violence laws.
69. The passage states that for cruelty to occur, there has to be an illegal demand of property. In this
case, the demand of property must be illegal. The demand for smuggling diamonds into India is an
illegal demand of property.
70. The passage states that liability extends to relatives. In this case however, the liability does not
extend to asking a person to do her legal duty as an RTO Officer. Thus, there is no cruelty.
71. The passage states that beating children is cruelty only when it is done with the intent of
harassing the woman. This however is not the case here as the woman is merely complaining. Thus,
the acts cannot constitute cruelty.
72. The passage states that beating children to get an illegal demand met from the woman
constitutes cruelty. However, getting her to sign a contract is not an illegal demand and thus the
entire act cannot constitute cruelty.
73. The correct answer is option (b). By being forced to stamp his thumb print on a confession, he is
being compelled to incriminate himself and his right against self-incrimination is being violated.
Article 20 does provide protection against self –incrimination.
74. The correct answer is option (d). Jaggu‘s right of protection against ex post facto laws is being
violated as he was convicted under ABC law which had been repealed before his arrest. Therefore,
his constitutional right is being violated because he was convicted under a retrospective application
of the law.
75. The correct answer is option (c). Being in possession of military grade firearms and conspiring to
hurt the national integrity of the country are two different offences. Double Jeopardy is not applicable
because Sanju is not being convicted of the same offence twice. Hence, there is no constitutional
violation in this case.
76. The correct answer is option (d). The right to life and personal liberty as guaranteed by the
Constitution establishes that nobody‘s life and liberty can be taken away and nobody can be
deprived of their right to life and personal liberty unless there is a procedure that has been
established by law. The refusal to bail in this case was under TADA and thus, falls in the exception
of ―procedure established by law‖.
77. The correct answer is option (b). The fact that Sea Queen had drowned before signing the
contract is a matter of fact essential to the contract as it could not be performed without it. The Act
provides that ―Where both the parties to an agreement are under a mistake as to a matter of fact
essential to the agreement, the agreement is void‖. Therefore, Chunnu is correct and the contract is
void.
78. The correct answer is option (d). A wanted to buy Chetak for his speed and not his sound mind.
Hence, the matter of fact essential to the contract remains the same i.e. Chetak and his fast pace.
Therefore, although there is a mistake of fact regarding the soundness of Chetak‘s mind, the same
does not make the contract void because this fact is not essential to the agreement between A and
B.
79. The correct answer is option (b). There was no mistake of fact as Hari went into the shop
wanting to buy a painting and paid for the painting only, even after being aware that there was a
necklace with the same name. The Mona Lisa painting is essential subject matter in the contract and
there is no mistake pertaining to it. Hence, the contract is not void and the shopkeeper‘s position is
correct.
80. The correct answer is option (d). The passage clearly mentions that the ―value of the thing which
forms the subject-matter of the agreement is not to be deemed a mistake as to a matter of fact‖.
Therefore, the fact that Chulbul was sold the sunglasses at a cost lower than the market average
does not constitute a mistake of fact and this cannot be used as an excue by Mr. Pay to render the
contract void.
81. The correct answer is option (b). Theft is when someone steals a movable property out of your
possession with the intention of stealing it from you. Therefore, severing the trees off the earth
turned them movable property the moment they were cut and Mogambo had the intention of stealing
them from X‘s possession by taking them away from the farm.
82. The correct answer is option (c). Robbery is an aggravated form of theft, so if someone attempts
or causes any hurt, wrongful restraint or death in order to commit an act of theft, it is known as
robbery. Showing a knife, threatening to kill a dog and then forcefully snatching a chain implies that
Z committed an aggravated form of theft.
83. The correct answer is option (c). Theft is when someone steals a movable property out of one‘s
possession with the intention of stealing it. Dahl moved the book from Roald‘s possession but he
didn‘t have a criminal intention of stealing it, else he would not have asked Roald‘s mother for
permission before borrowing the book.
84. The correct answer is option (a). Dacoity is a crime when 5 or more people commit or attempt to
or aid someone to commit an act of robbery together. Robbery is occurs if someone attempts or
causes any hurt, wrongful restraint or death in order to someone steals a movable property out of
one‘s possession with the intention of stealing it, it is known as robbery. Here, Jagga Daku and his
friends attempted to hurt Shinchan to steal his movable property by using forces and, they were
more than 5 in number. Hence, Shinchan has been a victim a dacoity.
85. The correct answer is option (c). The passage clearly mentions that differing from the views of
the government does not amount to sedition. Sharing of a toolkit does not violate the definition of
Sedition. Option (d) is incorrect because it specifies the medium as "words" and option (c) is wider in
its ambit to address this question.
86. The correct answer is option (b). The passage clearly mentions that it is ―not seditious to have
views different from the government". Therefore, Nemo''s documentary is not seditious because
portraying the government in a different light or bad light does not mean that there is an attempt to
excite hatred towards it.
87. The correct answer is option (b). The entire documentary is seditious as the disclaimer and the
ending, both incite disaffection towards the government. Both of them are beyond just "mere views
different from the government" and are seditious in nature because they make the documentary
inciting in nature.
88. The correct answer is option (a). Organising a pride parade does not incite contempt and does
not constitute a violation of Section 124A. Options (b) and (c) are incorrect because they are
opinions on the arrest instead of justifying why the arrest is wrong.
89. The correct answer is option (b). Balachandra''s poem was published in the newspaper and
therefore it constitutes "written words" for the purposes of this question. The descriptive nature and
particular emphasis on the contentious activities of the government, followed by a call of action
amounts to exciting disaffection towards the government.
90. The court can claim irregularity on the basis that there was no transit remand order authorized by
the judicial magistrate. Police custody cannot be taken beyond 15 days but its not clear whether the
5 days period was due to transit or not. Also, the accused has to be presented before the magistrate
within 24 hours if investigation is not completed within 24 hours.
91. The contention put forth by defence was correct as Manish cannot be under police custody for
more than 15 days period and it is not mentioned in the facts where Manish was taken to police
remand or not. Thus, it amounts to irregularity in the set procedure.
92. Since the crime was committed in district of Bhopal, Indore police cannot take further the
investigation though Pontu was found in Indore District. The trial court that is the Indore District
Court has no jurisdiction over the case and magistrate has to transfer the case in court of its
jurisdiction.
93. The contention put forth by the investigating officer is correct as the time period of 24 hours is
exclusive of time taken to reach out to the judicial magistrate. Thus 24 hours time to present accuse
before the magistrate is after arrest and has to take into consideration the time taken for travelling to
place of magistrate.
94. In all other cases transit remand can be ordered except in case where the accused has to be
presented before the judicial magistrate. This is because accused has to be presented within 24
hours of arrest to the nearby magistrate and there is no need to take the accused to Bikaner district
instead of Barmer.
95. Sunil can claim compensation only from Manohar as there exist customer and service provider
relationship between them and its duty of Manohar to check the repairs work done by Chintu.
96. Sheetal can claim compensation from shop owner as there exist such relation of buyer-seller
with her and the less quantity of such beauty product amount to defect for which compensation can
be claimed.
97. She cannot claim refund from Payzone as she cannot file a complaint against them in forum and
she was negligent on her part to read the refund policy mentioned there.
98. Rakesh cannot claim compensation from railway authorities as he was negligent in reading
terms and conditions of authorities receipt for keeping luggage in cloak room and it was not fault of
railway authorities by any means.
99. In a fiduciary relationship between a buyer and seller, the transaction has to be in good faith and
to protect the interest of the customer. It must not be such where the buyer gets the profit out of such
transaction and also, buyer is being paid for his services in form of remunerations.
100. Court cannot grant anticipatory bail to Manoj as he was already taken into custody by the
investigating officer and no charge sheet has been filed yet. Anticipatory bail is taken prior to arrest
or where there are reasons to believe that the person can be taken into custody for trial of the case.
101. Anticipatory Bail can be claimed by Vijay Malia as there from the leaked information there are
reasonable chances to believe that Vijay Malia is one of the parties in the alleged case of money
laundering and scandals.
102. NCLT cannot grant anticipatory bail to Montu as Bail can be granted only in criminal cases by
the courts and NCLT is a tribunal, not a court. Thus, Power to grant bail, both regular and
anticipatory, is with the court and no such bail can be claimed from NCLT.
103. Pritam can be granted regular bail anytime during the proceedings and post filing of charge
sheet/supplementary charge sheet. Also, the anticipatory bail does not come to an end post filing of
charge sheet/supplementary charge sheet.
104. Both II and III conditions are valid considerations which the court may look into for not granting
anticipatory bail. In I condition when there are no chances of tampering of evidences then
anticipatory bail can be granted easily and where there are no malicious prosecution in a case, then
also anticipatory bail can be granted (Condition IV).
LOGICAL REASONING

105. Option (A) & (B) are incorrect in light of the arguments made by the author in the passage.
Option (D) cannot be affirmed to be correct or not as a plan of action. Therefore, the correct answer
is Option (C), which can be inferred from second paragraph.
106. All the given statements further the point made by the author and help establish his proposition.
Therefore, the correct answer is Option (D).
107. Option (A) is incorrect as the author has never commented about the general population of
India. It may very well be true and therefore, regulating demand may not be the only solution
available.
Options (C) & (D) have no bearing on the argument presented by the author and the latter
strengthens the argument.
Option (B) is correct as it implies that nobody was at fault regarding the water crisis which got solved
by the rain itself.
108. The author paints the picture that the water crisis is kind of a lost cause as its effects are so
damning that we don‘t have many options left.
Hence, D
109. Only Option (C) covers the theme of the passage and points towards the discussion that takes
place in the passage.
110. Only Option (B) negates the argument posed by the author about the negative effect chemical
exposure has on babies. As this happens with everyone, it makes the effect redundant.
111. Option (C) is the only proposition that has not been stated by the author and cannot be inferred
from the information provided above.
112. Option (A) is incorrect as the passage does not refer to the stage at which external factors may
start to affect babies. Option (C) & (D) cannot be derived from the text of the passage. Therefore, the
correct answer is Option (B).
113. The author establishes how eating right i.e., fruits and vegetables may actually counterbalance
the pro-allergenic response. Therefore, the statement mentioned above is definitely false.
114. Option (B) is the only statement that the author would agree with and that supports the author‘s
conclusion. Therefore, the correct answer is Option (B).
115. Weaken Medium
Kushler claims that Swype has laid the foundation for the next level of pace and comfort. Choice D
states that another similar technology has already been introduced in the market, a year ago. Hence,
it effectively weakens the argument.
Choice A talks of how the Swype technology increases the average speed of even the fastest text
massagers. This strengthens the argument instead.
Choice B states a method used by the company to make money.
Choice C states how Swype affects performance of other applications loaded on the phone.
Kushler‘s claim is restricted to Swype improving typing pace and comfort and therefore choice D
does not weaken the argument.
Thus, choice D is the correct answer.
116. Neither of the assumptions is implicit. We cannot assume that the players will be tired after 90
minutes. Similarly the statement is not concerned with what happens after 90 minutes.
Hence, D
117. The last four letters of the word are written in the reverse order, followed by the first three
letters in the same order. In the group of letters so obtained, each letter except the middle letter is
moved one step backward while the middle letter is moved one step forward to get the code. Thus,
we have:

Correct option is (b).


118. If the movie has won an award, it can be considered good. Hence, argument 1 is strong.
Just because Taapsee is a good actor does not make each of her movie good. Hence, argument 2 is
weak.
Hence, A
119. The passage in its concluding lines says ―In addition, when researchers looked at receivers in
the game, they found that those in a sad mood were also more concerned with FAIRNESS, and
rejected unfair offers more than did those in the happy condition…‖ Option (a) follows coherently.
Hence (a).
120. The passage states ―However, in situations where a more cautious, less assertive and more
attentive communication style may be called for, a sad mood may help.‖ Hence (d).
121. The options all seem to support sadness as an emotion. Option (b) with depression as
prolonged sadness affecting interactions weakens the premise of the passage. Hence (b).
122. The article suggests that sadness has obvious benefits and has positive effects on generosity,
justice, fairness and trust. This supports option (c). Option (a) is out of context. Option (b) focuses on
happiness. Option (d) dos the same thing. Hence (c).
123. The statements A and B are in line with the passage that present the obvious benefit of
happiness but also asks us to see the good that sadness can bring out in people. Hence (c).
124. The passage goes to quote research after research to show the benefits of sadness. The
passage also goes on to say that happy people are perceived to be better people. All of this is based
on the assumption that sadness is undervalued. Option (b) negates what the passage states. Option
(c) asks us to replace emotions and not embrace all our emotions. Option (d) is obvious but the
passage talks about sadness. Hence (a).
125. The passage does not look at sadness with pity or as a negative emotion. It sees it as being
good for you and as part of your gamut of emotions. Option (a), (c) and (d) have a very negative or
burdensome approach with respect to sorrow. Option (b) is apt. Hence (b).
126. In the above conversation, the Secretary of Residential Society, while saying that In addition to
the monetary cost, burglaries also take a big emotional toll, assumes that Owners may need to
recover from a financial setback at the same time they‘re mourning the loss of things with
sentimental value and the feeling of safety they had before the break-in. Therefore, option (A) is the
correct answer. Option (B) and (C) are irrelevant.
127. The Secretary of Residential Society recommends leaving the lights of an empty house on as
way to prevent crime. The crime expert points out that leaving the lights on like this will invite crime.
Option (C) is the correct answer because giving potential criminal information about who is on
vacation would be an unintended and undesirable effect of the homeowner representative‘s
recommendation.
128. The argument of the CID guy in the above conversation is that more burglaries are happening
during day time, than at night. Option (B), (C) and (D) will not weaken this argument because they
are not related to the above argument. Option (A) will weaken the above argument if it is true than it
is saying that more burglaries are happening during nights than at day time.
129. Option (A) is not the conclusion of the CID DSP because nowhere in his argument he talked
about renters and homeowners. Option (B) is not the correct answer because it forms only a part of
the idea of the CID guy. Option (C) is irrelevant. Therefore, none of the option is the conclusion of
the CID DSP. Hence, option (D) is the correct answer.
130. CID DSP in his argument states that the CID‘s most current crime data shows that burglaries
fell down to 12% between 2017 and 2018. Therefore, on the basis of this argument, the above
statement is definitely true.
131. The given direction has asked us to mark the cases of possibility. Secondly, Restatements are
taken as correct conclusions. Therefore all should follow.
132. According to the given information, there are two possible arrangements.
A and F are not neighbours.

133. According to the given information, there are two possible arrangements.
D is the second person from E. Similarly, C is the second person from D.

134. According to the given information, there are two possible arrangements.
B,C and F are sitting in a sequence.

135. According to the given information, there are two possible arrangements.
The other neighbour of A is D.
136. According to the given information, there are two possible arrangements.
F is placed opposite to E.

QUANTITATIVE TECHNIQUES

137. Required marks


= 98 + 882+84×32+88×54+64×32+682
= 98 + 44 + 126 + 110 + 96 + 34 = 508
138. Average marks= 1268+80+90+56+72+666
= 124326
= 2166
= 36
139. From the table, we can see that only Ram scored the highest marks in more then one subject.
140. Sanjay‘s overall percentage
= 81+40+126+100+117+45625 ×100 ×
= 509625×100 ×
= 81.44%
141. Ram has greatest marks then Aditya in all subjects except Hindi. Rest are not in picture.
142. Profit in 1983 = 50 – 30 = 20 lakh
And profit in 1984 = 40 – 30 = 10 lakh
So, difference = 20 - 10 = 10 lakhs
143. average income = 30 + 50 + 40 + 60 + 605
= 2405
= 48 lakhs
So, here are three years (1983, 1985 and 1986)
Which have more income than Average income.
144. Ratio of = Average incomeAverage Profit=2405855
Profit = 48 : 17
145. Percentage increase in profit in 1986 over 1982 is
= 25-1010×100 −×= 250 -100 =150%
146. Total income from 1982 to 1986 = 240
Total exp. From 1982 to 1986 = 155
difference = 240 – 150 = 85 lakh.
147. Area of rice and barley together = 318×75+36
= 3× 6= 18 Hundred Acres.
148. Wheat + Rice + Maze
72° 72° 45° = 189°
(which is more than 50%)
149. Ratio between land used for Rice and Barley
= 7236 = 2 : 1
150. 10% of rice = 72°
then after distribution of it between wheat and barley, the new angle of wheat
= 72° + 23 ×7.2°=76.8° ×°=°
Legal Edge 27 Mock

Answers & Details

ENGLISH

1. The correct answer is option a) as Statement 1 is wrong according to the passage. The last
paragraph of the passage mentions that it was not the author‘s personal opinion but the opinion of
‗the worshippers of complete freedom‘ who think that religion is a form of mental slavery.
Statement 2 is true according to the passage as the third paragraph states that, ‗The most important
thing that was reiterated repeatedly and upon which Shriman Bhai Amar Singh ji Jhabaal laid special
emphasis was that the question of religion should not be touched at all. This was very good advice.‘
2. The correct answer is option a). The correct sentence should be,
‗They also feel that telling a child that God is omnipotent and man nothing but a mere statue
of clay, is to make the child weak forever.’

3. The correct answer is option c). In more than half of the of the passage , the author cites some
examples and gives us an idea of the cons of mixing politics and religion which concludes through
this sentence: ‗So much so that mantras and aayats began to be read from the Congress dais as
well. Those days anyone lagging behind in religion was not considered to be good. As a result,
narrow-mindedness was on the rise. The ill-effects of this are not hidden from anyone.‘ the
remaining passage then revolves around portraying the idea of how religion serves no purpose other
than alienating people from each other and lowering one‘s self-esteem.

4. The correct answer is option c) as there are two kinds of grammatical errors present in the
sentence. As option a) suggests, there is an error in subject-verb agreement in the sentence as the
singular verb ‗has‘ does not agree with the plural compound subject ‗the nationalist people and
lovers of freedom‘. As option b) suggests, there is also an error in the usage of article in the
sentence as the correct article for the word ‗obstacle‘ should be ‗an‘. The correct sentence should
be, ‗Now the nationalist people and lovers of freedom have grasped the truth on religion and think of
it as an obstacle in their path.‘

5. The correct option is c). Adjectives are usually placed before the nouns they describe. When
reading a sentence, find the noun first. The noun is the person, place or thing that is the subject of
the sentence. Then, check to see if there is a descriptive word right before the noun. If there is,
then it may be an adjective. Here in the above sentence, the subject of the sentence is
the‗question‘. Which is described by the word ‗concomitant‘. Concomitant means ‗naturally
accompanying or associated.‘

6. The author says ―Those days anyone lagging behind in religion was not considered to be good.
As a result, narrow-mindedness was on the rise. The ill-effects of this are not hidden from anyone.‖
Thus, the author takes the society as a whole. He doesn‘t point out to the fact that if a person
becomes religious he/she becomes narrow-minded or the vice versa. Hence, option (a) and (b) are
wrong. Option (c) is just the reverse of what the author says. Hence, option (d) is the answer.

7. A thorough reading of the passage leads us to the answer. The passage overall talks about the
decrease and the subsequent increase in CO2 emissions, drawing a contrast between the level of
emissions at the beginning of the lockdown and after the lockdowns were lifted in various countries.
Thus, the correct answer is B.
Option A is incorrect as it is incomplete and only talks about half of the process, which is the decline
in carbon emission level. It fails to mention the subsequent increase in the carbon emission level,
which is as important (if not more so) as the decrease.
Option C and D are correct answers, as they are being discussed in the passage. However, they
form smaller parts of the wider narrative of the passage, which is the decrease and the subsequent
increase of carbon emissions not just in specific countries, but globally. Further, the passage also
includes other reasons for the fluctuating emissions like vehicles and aviation, which are not covered
by options C and D. Thus, while they are right answers, they do not summarize the whole process
being asked in the question.
8. A thorough reading of the passage leads us to the answer. Option B summarizes the passage in
one line and thus tells us the theme of the passage.
Option A and C are incorrect as they are mere suggestions mentioned in the passage and are the
need of the hour. In no way are these the theme of the passage. Thus options A and C are incorrect.
Option D is incorrect as it cannot be inferred anywhere from the passage.
9. While all of the above are valid reasons for the decrease in CO2 emissions, options B, C and D
are subsequent results of A, which is the main reason. The question asks for the ―main‖ reason for
the decrease in emission, which is the global pandemic. Options B, C and D are a result of option A
itself. Thus, the correct answer is A.

10. The following line: ―One thing the pandemic didn‘t affect was the ongoing increase in the share of
energy coming from renewable sources. In 2020, renewables accounted for 29% of all energy
generated, compared to 27% in 2019—the largest year-over-year increase on record.‖ This line
indicates that there has been a substantial increase in the use of renewable, which is good for the
environment and has sustained even till today. Thus, the correct answer is C.
Option A is incorrect because as of today, the decreased levels of carbon emissions have not
sustained. The emissions are back to their previous levels. Thus, while option A was the right
answer had the question asked about the earlier half of 2020, as per 2021 it is incorrect.
Option B is incorrect as it cannot be inferred from the passage that seeing the carbon emission
levels, countries have realized the dangers posed by climate change.
Option D is incorrect as it is negated in the following lines: ―But the upward drift of fossil fuel use at
the end of the year nonetheless discourages experts.‖
11. ―The following line leads us to the answer: ―One thing the pandemic didn‘t affect was the ongoing
increase in the share of energy coming from renewable sources. In 2020, renewables accounted for
29% of all energy generated, compared to 27% in 2019—the largest year-over-year increase on
record. But the upward drift of fossil fuel use at the end of the year nonetheless discourages
experts.‖ The author here says that there has thankfully been an increase in the usage of energy
coming for renewable resources but has been followed by the upward drift of fossil fuel usage,
thereby drawing a distinction between the two and implying that fossil fuel does not fall in the
category of renewable resources. Thus, the correct answer is A.
Option B is incorrect as a comprehensive reading of the passage gives us the idea that while the
beginning of the lockdown brought about significant decrease in the carbon emission levels, the
same has not sustained and has currently gone back to the previous increased levels. Thus, option
B is incorrect.
Option C is incorrect as it cannot be deduced from the passage anywhere.
Option D is incorrect as can be seen from the following lines: ―In March 2020, the IEA urged
governments to put clean energy at the heart of their economic stimulus plans to ensure a
sustainable recovery,‖ said Fatih Birol. ―But our numbers show we‘re returning to carbon-intensive
business-as-usual.‖
The author here says that while it was urged in March 2020 that governments must shift to clean
energy sources, it has been disheartening to see the increasing number of carbon based resources.
Thus, D is incorrect.
The following line reflects the same: ―It‘s hard to find any upside in a global pandemic. But
throughout 2020, there was some good news buried in the bad concerning that other great infirmity:
the sickly state of climate‖. The passage begins by talking about the prevailing global pandemic and
then proceeds to talk about the other great infirmity, which is the sad state of the climate. Thus, the
correct answer is D.
12. Option A is incorrect as the passage nowhere suggests that there is a lack of availability of clean
energy sources. Further, it urges governments to switch to cleaner sources of energy, thereby
indicating their sufficiency.
Option B is incorrect as the pollution by aviation industry is a part of carbon emission. Further,
climate change as a whole has not been mentioned in the passage, only that the climate has been
affected by carbon emissions.
Option C is incorrect as the carbon footprint left by big economies forms a part of the bigger
problem, i.e., the overall carbon emissions and its effect on the climate. Further, climate change as a
whole has not been mentioned in the passage, only that the climate has been affected by carbon
emissions.
Option D is incorrect as while the first half of it is correct, the latter part is incorrect. As per the
passage, the usage of renewable sources of energy has increased in 2020 and is at an all time high,
thereby negating the latter part.
13. The correct answer is option b). The entire passage revolves around Pratap Bhanu Mehta‘s life
and work. Options b), c) and d) have little to no mention in the passage.

14. The correct answer is option c) as both the statements are false. Statement 1 is false as the
second paragraph clearly mentions that, ‗Needless to say readers of the Indian Express, and its
editors, also saw this, and Pratap‘s column in the Indian Express, which he has sustained till this
day, was instantly born.‘
Statement 2 is false as the first paragraph states that, ‗I have known Pratap since he was a post-
doctoral fellow at Harvard in the mid-1990s.‘
15. The correct answer is option c). A common noun is the general, non-specific term for a person,
place, thing, or idea. Usually, common nouns are not capitalized unless they begin a sentence. Here
in the sentence, the common noun is ‗country‘. Option a) is used in the line as a pronoun. Option b)
is used in the passage as an adjective. Option d) has been used in the passage as a verb.

16. The correct answer is option b). We can infer that Pratap Bhanu Mehta was the 2 nd vice
chancellor of Ashoka University from the following line of the passage: ‗When the founders and
trustees of Ashok university chose him to succeed its first vice-chancellor, Rudrangshu Mukherjee,
they could not have made a better choice.‘

17. The correct answer is option d). the correct statement should be, ‗Among the several private
universities that were set up in the glory days of the United Progressive Alliance (UPA) government,
the distinctive feature of Ashoka University was its decision not to open the gates of admission wide
to ensure its financial viability but to enroll only students who meet admission standards comparable
to those of the best universities in the world.‘
18. the correct answer is option a). As according to the passage Ferdinand Magellan led the first
expedition to sail around the world in the age of great marine and terrestrial exploration which is the
16th century. This sentence has been followed by another sentence which says ‗as a young
Portuguese noble, he served the king of Portugal‘. Hence we can infer that Ferdinand had served
the king of Portugal in the 16th century.

19. the correct option is c). According to the first paragraph of the passage, Magellan offered to
serve the future Emperor Charles V of Spain. Option a) is wrong as when he found the passage, he
didn‘t name it after himself. Option b) is wrong as no one was sure if the assigned land includes east
indies or not so Magellan offered to prove that it did. Option d) is wrong as the date mentioned in the
option is wrong.

20. the correct option is b) as the word ‗ones‘ should have been replaced by the word ‗once‘ which is
a preposition. A preposition is a function word that typically combines with a noun phrase to form a
phrase that usually expresses a modification or predication.

21. the correct answer is option a) as Magellan died in the 16th century by getting himself involved in
an insular conflict in the Philippines and was killed in a tribal battle.

22. the correct answer is option b) as the narrative style of writing includes the proforma the author
had followed while forming the passage. Narrative style of writing includes novels, creative fiction,
biographies, mythology etc.

23. the correct answer is option d) as according to the passage, it is mentioned that Magellan set sail
from Spain with five ships from which one of the ships sank while searching for a water route and
another ship deserted while in the passage and returned to Spain.

24. The correct answer is option a). In writing, figurative language—using words to convey a
different meaning outside the literal one—helps writers express themselves in more creative ways.
One popular type of figurative language is personification: assigning human attributes to a non-
human entity or inanimate object in an effort to express a point or idea in a more colorful, imaginative
way. The bells are inanimate objects which were assigned human attributes.

25. The correct answer is option d). Maria Franzisca‘s fiancé is Predu Tasca, who was a swineherd,
had accordingly killed his finest little pig, painted it with its blood, filled it with bundles of asphodel,
and sent it as a gift to his fiancée.

26. the correct answer is a) as Statement 1 is the correct statement. We can infer it from the
following sentences of the passage: ‗In the presence of her father, Maria Franzisca would hardly
have dared to sit down close to her dear Predu.‘ ‗And his fiancée returned the basket with a cake of
honey and almonds, giving a scudo of silver [5 lire] to the woman who brought it.‘
Statement 2 is wrong as the passage mentions, ‗Shepherds brought to their masters the first milk of
their calves, and the lady of the house returned the container to the shepherds, filled with vegetables
or other things, having first carefully emptied it in order not to bring down ruin on the cattle.‘
27. The correct answer is option d). The correct sentence should be: Thus, it was Christmas Eve—
a gray day, dimmed but mild since an east wind was blowing, carrying the enervating warmth
of distant deserts and a humid scent of the sea.

28. the correct answer is option c). The passage states that the almond trees had bloomed „pr
ematurely‟ during Christmas eve. As we know, spring comes after winter. Hence it can be infe
rred that the almond trees in the valley usually bloom in spring, otherwise the author wouldn‟
t have used the adverb „prematurely‟ to describe the bloom.
29. the correct answer is option b). Adjectives are words that describe nouns (or pronouns). In the
above sentence, the noun ‗Levantine wind‘ is described as ‗strange‘ by the adjective.

GENERAL KNOWLEDGE

30. The Naga peace process appears to have again hit a roadblock after decades of negotiations.
The non-flexibility of the National Socialist Council of Nagaland (NSCN-IM) on the ―Naga national
flag‖ and ―Naga Yezhabo (constitution) among many more are said to be the primary reasons. But
the issue is more complex than the twin conditions, as it affects Nagaland‘s neighbours in northeast
India.

31. A contentious demand of the NSCN-IM is the integration of the Naga-inhabited areas of Assam,
Arunachal and Manipur with Nagaland. Tirap, Changlang and Longding in Arunachal are parts of the
NSCN-IM‘s envisaged unified Naga homeland called ‗Greater Nagalim‘, to which the neighbouring
states are opposing.

32. The Naga Hills became part of British India in 1881. The effort to bring scattered Naga tribes
together resulted in the formation of the Naga Club in 1918.The Naga club rejected the Simon
Commission in 1929 and asked them ―to leave us alone to determine for ourselves as in ancient
times‖. The club metamorphosed into the Naga National Council (NNC) in 1946. In 1975, when the
government signed the Shillong Accord, under which this section of NNC and NFG agreed to give up
arms. Thus, option (d) is the answer.

33. The Armed Forces (Special Powers) Act, 1958 came into force in the context of increasing
violence in the North-eastern States decades ago, which the State governments found difficult to
control. AFSPA gives armed forces the power to maintain public order in ―disturbed areas‖. They
have the authority to prohibit a gathering of five or more persons in an area, can use force or even
open fire after giving due warning if they feel a person is in contravention of the law. Thus, 1 and 3
are wrong.

34. A significant issue is how the weapons in the NSCN-IM camps are going to be managed. As a
‗ceasefire‘ group, its cadres are supposed to retain their weapons inside the designated camps for
self-defence only, but more often than not, many influential cadres are seen moving with weapons in
civilian localities, leading to many problems. It would be an uphill task for the Centre to ensure that
all weapons are surrendered at the time of the final accord. In the early phase, the Naga insurgents
were provided with what has come to be known as ‗safe haven‘ in Myanmar. India‘s adversaries
(China and Pakistan) also provided them with vital external support at one point in time. The porous
border and rugged terrain make it different for the Security Forces as they cross borders where they
are sheltered and fed. The ‗taxes‘ levied by insurgent groups are intricately intertwined in almost all
developmental activities in Nagaland and one of the major aims of the NSCN-IM has been to acquire
formal recognition of this informal practice through negotiations. Thus, option (d) is the answer.

35. [x] means Naga National Political Groups. It is a collection of 7 political groups.
36. The Indian government has been in a stand-off with the Mizoram state government for the past
few weeks. The reason: their differing stance on how to deal with the ongoing refugee crisis from
Myanmar.

37. Most of them are the Chin people, i.e. they are from the Chin state. They belong to the same
tribe as the Mizo people.

38. Around 160 of them are from police and army. They aren‘t Chin, they are real Burmese. They
don‘t speak the Mizo language but we have to take care of them on humanitarian grounds. India and
Myanmar share 1,643km border and people on either side have familial ties due to ethnic affiliations.
Manipur, Arunachal Pradesh and Nagaland are the Indian states that share borders with Myanmar
but the influx after the coup has been limited to Mizoram that share a 510km border with Myanmar.

39. India-Myanmar border passes through the difficult terrains of mountains and forests. The India –
Myanmar region also hosts large number of tribes who have a relations and interests across the
border. In order to facilitate free movement of the tribal people along the border so that their deep
seated relations among the tribal‘s does not gets affected the mechanism of the free movement
regime was introduced. Free Movement Regime (FMR) which allows the tribes living along the
border to travel 16-km across the boundary without visa restrictions. But this free movement is often
misused by the insurgent groups who use these open borders to carry out attacks and cross over
the border for escaping the authorities. Hence for regulating this menace without affecting the deep
seated tribal relations the government of India had proposed the Border management pact.As per
the border management pact, the four states of Arunachal Pradesh, Nagaland, Manipur and
Mizoram which share the unfenced border with Myanmar were asked to distribute border pass to all
the residents living within 16 km from the border. Thus providing for a structured and regulated
mechanism to facilitate the tribal movements and countering insurgents.

40. The Parliament of India has approved the merger of two of the TV channels namely, Lok Sabha
TV and Rajya Sabha TV into an integrated channel ―Sansad TV‖. This new platform will continue to
live telecast the house proceedings, pack news and the current affairs programme in Hindi as well
as in English languages.

41. On 7 November 2019, a committee was constituted and tasked with chalking out the modalities
of the merger, and preparing guidelines for pooling of resources and technology. The panel was
headed by former Prasar Bharati Chairman A. Surya Prakash.

42. Retired IAS officer of 1986 batch Ravi Capoor has been appointed Sansad TV CEO for one
year. He will be responsible for working out the final details of the merger.

43. RSTV was launched in 2011. Apart from telecasting live the proceedings in Rajya Sabha, it also
brings analyses of parliamentary affairs and provides a platform for knowledge-based programmes.

44. Somnath Chatterjee was appointed as Lok Sabha Speaker from 4 June 2004 to 31 May 2009.
He was associated with the Communist Party of India (Marxist).

45. The Supreme Court in the case of Indra Sawhney v. Union of India had set
the upper limit for reservation in jobs and education at 50%, except in “extraordinary circumst
ances”.

46. The Justice Gaikwad Commission report, whose recommendations supposedly formed the
legislature‘s basis for implementing the Act, is also under scrutiny for allegedly using unreliable and
unscientific data. It concluded that the Marathas are a socially and educationally backward class
whose extraordinary position justifies creating reservations beyond the 50% ceiling limit.
47. The 102nd Constitutional Amendment Act introduced Articles 338B and 342A in the Constitution.
Article 338B deals with the NCBC.

48. In 2018, the Maharashtra government enacted a law, it provides 16 per cent reservation to the
Maratha community in jobs and admissions. After recommendations of the State Backward Classes
Commission, the Bombay High Court upheld the constitutional validity of the Act.

49. The 102nd Constitutional Amendment Act


introduced Articles 338B and 342A in the Constitution.
50. INSTC is a corridor to increase trade between India and Russia. This trade route is 7200 Km
long and the transport of freight is through a multi-mode network of Road, Ship, and Railways. This
route connects India and Russia through Iran and Azerbaijan.

51. The current INSTC project was initiated by Russia, India and Iran in September 2000 in St.
Petersburg. The agreement was signed on 16th May 2002.

52. INSTC that extends from India to Russia, and to expand INSTC membership by including
Afghanistan and Uzbekistan, at a ―Chabahar Day‖ event organised as part of Maritime India Summit.

53. Pitching for Chabahar in the INSTC, Indian proposed that establishing the Eastern Corridor
through Afghanistan would maximise its potential.

54. 1967-68: First organised demand for a Bodo state came under the banner of the political party
Plains Tribals Council of Assam.

55. In 2003, the second Bodo Accord was signed by the extremist group Bodo Liberation Tiger
Force (BLTF), the Centre and the state. This led to the Bodoland Territorial Council (BTC). BTC is
an autonomous body under the Sixth Schedule of the Constitution. The area under the jurisdiction of
BTC was called the Bodo Territorial Autonomous District (BTAD).

56. Bodos are the single largest community among the notified Scheduled Tribes in Assam. Part of
the larger umbrella of Bodo-Kachari, the Bodos constitute about 5-6% of Assam‘s population.

57. The Central government signed a tripartite agreement with the state government and different
Bodo groups, including four factions of the National Democratic Front of Bodoland (NDFB), for a
―permanent‖ solution to the Bodo issue. Some features:
It provides for ―alteration of area of BTAD‖ and ―provisions for Bodos outside BTAD‖
The BTAD was renamed Bodoland Territorial Region (BTR).
It provides for more legislative, executive, administrative and financial powers to BTC.
Provision for rehabilitation of surrendered militants of NDFB and bringing a special development
package of Rs. 1,500 crore for the region.

58. According to historians, they belong to the Tibeto-Burman family of the Mongoloid race. Their
population is concentrated mostly on the North bank of the Brahmaputra river, in western and parts
of Central Assam, the northern part of West Bengal and in some parts of Bhutan and Nepal,
Arunachal Pradesh and Nagaland.

59. the central government introduced the Government of National Capital Territory of Delhi
(Amendment) Bill, 2021 in the Lok Sabha to amend the Government of National Capital Territory of
Delhi Act, 1991.
60. ―Government" to mean "Lieutenant Governor (LG)": The expression ''Government'' referred to in
any law to be made by the Legislative Assembly shall mean the Lieutenant Governor (LG).
Widening of Discretionary Powers of LG: The Bill gives discretionary powers to the LG even in
matters where the Legislative Assembly of Delhi is empowered to make laws.
Necessarily Granted an Opportunity to LG: It seeks to ensure that the LG is ―necessarily granted an
opportunity‖ to give her/his opinion before any decision taken by the Council of Ministers (or the
Delhi Cabinet) is implemented.
Related to Administrative Decisions: The amendment also says that ―Legislative Assembly shall not
make any rule to enable itself to consider the matters of day-to-day administration of the Capital or
conduct inquiries in relation to the administrative decisions‖.
61. Section 44 of the 1991 Act says that all executive actions of the LG, whether taken on the advice
of his Ministers or otherwise shall be expressed to be taken in the name of the LG.

62. Delhi‘s current status as a Union Territory with a Legislative Assembly is an outcome of the 69th
Amendment Act through which Articles 239AA and 239BB were introduced in the Constitution.

63. In its 2018 verdict, the five-judge Bench had held that the LG‘s concurrence is not required on
issues other than police, public order and land.

64. Recently, a glacial break in the Tapovan-Reni area of Chamoli District of Uttarakhand led to
massive Flash Flood in Dhauli Ganga and Alaknanda Rivers, damaging houses and the nearby
Rishiganga power project.

65. It occurred in river Rishi Ganga due to the falling of a portion of Nanda Devi glacier in the river
which exponentially increased the volume of water. Rishiganga meets Dhauli Ganga near Raini. So
Dhauli Ganga also got flooded.

66. Major Power Projects Affected:


Rishi Ganga Power Project: It is a privately owned 130MW project.
Tapovan Vishnugad Hydropower Project on the Dhauliganga: It was a 520 MW run-of-river
hydroelectric project being constructed on Dhauliganga River.
67. According to the topography and the location of the glacier, it can be categorized as Mountain
Glacier (Alpine Glaciers) or Continental Glacier (Ice Sheets). The Continental Glacier moves
outward in all directions whereas the Mountain Glacier moves from a higher to a lower altitude.

68. Dhauliganga
Origination:
It originates from Vasudhara Tal, perhaps the largest glacial lake in Uttarakhand.
About:
Dhauliganga is one of the important tributaries of Alaknanda, the other being the Nandakini, Pindar,
Mandakini and Bhagirathi.
Dhauliganga is joined by the Rishiganga river at Raini.
It merges with the Alaknanda at Vishnuprayag.
There it loses its identity and the Alaknanda flows southwest through Chamoli, Maithana,
Nandaprayag, Karnaprayag until it meets the Mandakini river, coming from the north at
Rudraprayag.
After subsuming Mandakini, the Alaknanda carries on past Srinagar, before joining the Ganga at
Devprayag.
Alaknanda then disappears and the mighty Ganga carries on its journey, first flowing south then
west through important pilgrimage centres such as Rishikesh and finally descending into the Indo-
Gangetic plains at Haridwar.

LEGAL REASONING

69. Sol. The correct answer is (d). As per section 311 court has the power to examine any person at
any stage of the trial in the interest of justice. In the present case, defence has made an application t
o present an eye witness hearing whom would further the interest of justice.

70. Sol. The correct answer is (c). Section 311 of the CrPC ensures there may not be failure of justic
e on account of mistake of either party in bringing the valuable evidence on record or leaving ambigu
ity in the statements of the witnesses examined from either side. If the defence has made an applicat
ion to produce a crucial witness to support their side, the application shall be allowed in the interest o
f justice.

71. Sol. The correct answer is (b). Under section 311 a court can allow cross-
examination of a witness again for the cause of justice. In the present case, since the witness has pr
esented two different versions of testimony on trials on the same matters, the defendant shall be allo
wed to cross-examine the witness again.

72. Sol. The correct answer is (b). As per the judgment of the Supreme Court in Swapan Kumar Cha
tterjee Vs. CBI, the power not be exercised if the application has been filed as an abuse of the proce
ss of law. In the present case, the court has previously given Tarun to produce witnesses when the d
ate for judgment was fixed and defence was able to prove nothing material. Tarun has again made a
n application for re-
examining witnesses, this proves that he is just attempting to abuse the process of law.

73. Sol. The correct answer is (c). Option III is wrong as delay in the procedure of law is no ground f
or the court to not allow summoning or re-
examining a witness. Option IV is wrong as section 311 confers upon courts a discretionary power a
nd not cast upon them a duty to consider evidence or witnesses at any stage.

74. Sol. The correct answer is (d). Referring to the guidelines of the Nilesh Navalakha case, the Bo
mbay High Court restrained news channels from reporting on illicit relation of the deceased with anot
her person. Thus, in the present case, applying the same analogy, media trial interferes with the ad
ministration of justice and shall be restrained.

75. media houses should restrain themselves and ―avoid or regulate‘‘ their coverage, debates and
discussions during ongoing criminal investigations of cases. Media should give information that is
informative and in the public interest. It doesn't mean that information should be defamatory.
76. Sol. The correct answer is (b). In the present case, the news channel was merely reporting on th
e death of Kunal and neither did it share any illicit content in their reporting nor did they attempt to st
eer the public opinion regarding a case towards a different direction.

77. Sol. The correct answer is (a). The Court in Nilesh Navalakha case had laid down various guideli
nes for reporting on "sensitive criminal matters, including death by suicide, and had held that the me
dia should avoid putting photographs of victims, depicting the deceased as a weak character, or try t
o reconstruct the incident while the investigation is underway. In the present case, publishing the vid
eo of the deceased girl on the news channel is violative of the above guidelines.

78. Sol. The correct answer is (b). Bombay High Court had in the instant case that media trial during
criminal investigation interferes with the administration of justice and hence amounts to contempt of
court as defined under the Contempt of Courts Act, 1971. In the present case, Dastak is interfering w
ith the administration of justice by media trial of the case, and thus the appeal shall not be allowed.

79. Sol. To seek whether the permission of RBI is required prior to the sale deed being concluded, w
e have to look at citizenship status of Jatin which were not mentioned in the facts. Thus the sale dee
d was valid or not cannot be commented upon due to insufficiency of facts. Also, resident and citizen
ship status are not same.

80. Sol. The gift deed between Mehul and Shagun was signed in year 2019 and the judgment of the
court with mandatory permission of RBI in cases of sale or gift deed by non-
citizens is passed with prospective effect and thus, the deed was valid. The date of execution is imm
aterial.

81. Sol. The contention is not correct as there is no use of any negative words or there is no such pr
ohibition in the section which states that non-
compliance with such provision will result into any consequence or prejudice to the plaintiff. Thus, its
merely directory in nature.

82. Sol. wagering agreements are void under Indian Contract Act but are not prohibited. It is to say t
hat any sum of money under wager cannot be recovered via machinery of courts as law does not rec
ognize such agreements to be valid and are unlawful. But there is no such prohibition from entering i
nto wagering agreements.

83. I. has a prohibition clause „no court shall....‟ and


both III. and IV. has „Shall’ which means they are mandatory in nature and
II. has „May’ which means it is directory in nature.
84. Sol. Since the complaint has been filed post amendment act 2019, the pecuniary jurisdiction for t
he said amount lies with the District Forum and not the State Commission. So, complain has to be fil
ed with the District Forum (upto One Crore Rs.).

85. Sol. The case has been filled in 2015 and as per the old provisions the pecuniary jurisdiction for t
he stated amount is with the National Commission and it would not be changed even after the amen
ded Act of 2019 as it does not have retrospective effect.

86. Sol. Since the pecuniary jurisdiction has been amended in the new act, the jurisdiction over the
matter will be only of State Commission and not National Commission. Since the previous case was
filed in 2010, pecuniary jurisdiction above one crore was of National Commission, but after amendm
ent, new case has been filed against the subsidiary which will be under jurisdiction of State Commiss
ion (upto 10 Crore Rs.).
87. Sol. The question specifically asked for “Only” pecuniary jurisdiction example which is correctly
mentioned under condition III and IV. In other conditions, I and II is example of subject-
matter jurisdiction, III and IV are of pecuniary jurisdiction only.

88. Sol. Assertion is true but Reasoning is False as parliament had not intended the amended act to
have retrospective effect as it would amount to much financial hardships, uncertainty etc. to consum
ers to file consumer litigations.

89. (c) option (a) has absurd reasoning, option (b) It doesn‘t matter society accepted LGBTQ or not.
Law decriminalizes Section 377 therefore this will be the wrong answer. Correct answer option (c)
because-Supreme court ruling granted same-sex couples the freedom to lead dignified private life
according to passage therefore this is the right answer. Option (d) Question talks about the
companionship of same-sex couples, not about their marriage therefore this will be not an
appropriate answer.
90. (b). option (a) Passage doesn‘t talk about moral principles‘. Option (b) This is the right answer
because the passage talks about not giving same-sex couples the right to marry according to the
decision of the court. Option (c) Absurd reasoning. Option (d) Answer with incomplete reasoning.

91. (b). option (a) Section 377 is decriminalized but with exceptions refers to lines 5 and 6 of
passage. Option (b) This is the right option because companionship was not free and falls under the
exceptions mention in lines 5 and 6 of the passage. Option (c) Reasoning is correct
but yes is written instead of ―No‖ option

92. (d). option (a) and (b) Section 377 was decriminalized on 7 Sep 2018 therefore act done before
7 Sep 2018 will be criminalized. Option (d) They were in a same-sex relationship before 7 Sep 2018
therefore will not be given rights because Section 377 was constitutional in 2017.

93. Sol. (b) the last line of the passage talks about the legislative scheme of marriage and dressing
like a woman not inculcate biological features of the woman into a man. Therefore option b is the
right answer.

94. Sol. (c) As per the information given in the above passage, every agreement in restraint of the
marriage of any person is void. In the instant case the agreement executed by them was in restraint
of marriage, so it will not be considered as valid and therefore the couple can go ahead and marry
each other.

95. Sol. (b) The contract did not restrict Anshul to marry another girl, therefore it was not in restraint
of marriage.

96. Sol. (c) In the third deal Chinamma is not talking about a particular person but an entire
community. To restrict his choice to the Hindu community is not really a restraint considering the
Hindu community is so large. It is just a reduction in the choices he has.

97. Sol. ―Section 5 (iii) of the Hindu Marriage Act, 1955 prohibits marriage of a girl below 18 years
and boy below 21 years of age.‖ Option A could have been the answer if the question was about the
validity if the contract.

98. Sol. (c) “the Live-In-Relationship agreement set up by petitioners being void agreement cannot
be accepted.‖ The agreement is question too involves live-in relationship therefore it will be
considered as invalid.

99. (c) it‘s a contract to marry each other thus valid. There is nothing making it invalid.
100. a.It is a vague and unreasonable answer because it is blaming the consumer for a violation of
its rights.
b.Reasoning behind the answer is not appropriate because the term service provider is missing in
the answer and the question talks about online fraud, not about a product defect.
c.This is the reasonable answer as Murphy is a minor who can file complain with help of a par
ent or legal guardian as mentioned in the passage and online transactions, service providers
come under the act according to passage.
d.This is absurd reasoning.
101. a.saanvi is eligible to file a complaint but the answer is not followed by reasonable legal
reasoning.
b.It is a vague and unsatisfied answer without proper legal reasoning because in the passage
nowhere talks about misleading advertisements.
c.This answer will look satisfied in the first reading and prior legal knowledge will make you tilt
towards this answer, therefore, students are requested to read the passage carefully because the
passage does not mention misleading advertisement it talks only about definitions of advertisement.
102. Definition of the consumer does not include a person who obtains such goods for resale or any
commercial purpose‖ Therefore all except option c are nullified because Beena Devi was not a
consumer she bought sarees for commercial purpose.

103. (a) The consumer has the right to be heard but the passage doesn‘t talk about consumer rights
to be heard you have to answer in light of the passage.
b.This is the right answer because in the passage nowhere is given the consumer right to be heard
so we will infer nonavailability of rights to the consumer.
c.Absurd answer
104. Sol. a. After inferencing this will be appropriate legal reasoning because this option gives an
explanation keeping in light general and common reasons behind the amendment.
b.It is not necessary for a democratic country to always change law according to with time for proper
functioning, therefore this, not a wrong answer which lacks reasonable reasoning.

LOGICAL REASONING

105. Options (A) & (D) are unfounded in the passage. Option (C) is a general statement that cannot
be sourced from the passage and goes against the author‘s view that populism has been present
and gradually affecting ideologies of people.
Option B can be deuced from Para 3.
Hence, B
106. Refer to ―research shows that populism has been advancing slowly yet steadily in Europe since
the 1960s.‖ and to ―To effectively contain populism, Europe must accurately diagnose how and why
it emerged in the first place. The sources for such concerns are not likely to disappear, so populism
is more of a long-term challenge than a temporary crisis‖
Therefore, the correct answer is option (D).
Options (A) & (B) cannot be ascertained to be true of false on the information provided above and
cannot be reasonably inferred. Option (C) is against the view presented by the author about the
change in system that can be seen today.
107. The author never delves into the reason behind the advent of the populist movement. Either of
the two statements could be true but neither can be confirmed from the text of the passage.
Hence, D
108. Only Option (A) impacts the author‘s argument about the growing nature of the populist
movement. If the statement is correct and the political discourse does not change easily then the
movement does not pose such a grave threat as the author paints it to be.
Hence, A
109. All the statements mentioned above can be derived from the passage and therefore, the correct
answer is Option (D).

110. Only Option (B) weakens the author‘s proposition that Brazil is not doing well in the world. The
author paints a picture of how the country is growing through major crisis but it is all negated if the
above statement is deemed to be true.
Hence, B
111. Option (A) is correct as the author never compares the scandal to any other that may have
happened in the country. Thus, ‗LARGEST scam‘ cannot be concluded.
Hence, A
112. Option (A) cannot be reasonably inferred as an outcome of the scenario. Option (B) ‗taking
down the government‘ is incorrect as per the passage. Option (C) MAY be true but cannot be
verified with CERTAINTY.
Therefore, the correct answer is Option (D)
113. Premises are the facts or evidence that support or lead to the conclusion. Therefore, the above
statement forms premise of the passage which leads to the conclusion.
The conclusion is that Political situation in Brazil is in chaos.
Hence, option (A) is the correct answer.
114. Option A and D are too general.
Option C is extreme – heading to the bottom – cannot be reasonably inferred.
Only Option (B) manages to cover the theme of the discussion and refer to the main point of
contention mentioned by the author.
Hence, B
115. The number of accidents should be compared to the number of automobiles in the said states
to get an accurate picture of how safe (or unsafe) it is to drive there. For instance, if UP has thrice
the number of automobiles, than with these number of accidents, it is safer to drive in UP. However,
if the number of automobiles are equal in both states or less in UP, then certainly MP is safer.
Hence, total number of automobiles is an important factor in deciding this safety issue.
Therefore, C
116. d); As Honey has equaled number of persons who are taller than her & who are shorter than
her, Honey is third in the row.
Sujana & Ramya are neither the tallest nor the shortest Sujana isn‘t taller than Ramya From the
above it can be concluded that Ramya & Sujana are the 2nd & the 4th respectively in the queue.
∴ Lalita & Sudha arc the first & the last persons in the queue in any order Hence, the final
arrangement is as follows:
Lalita/Sudha, Ramya, Honey, Sujana, Sudha/Lalita
From the given options, choice (d) is definitely false
117. None of the statements above can be deemed true from the information laid down by the author
in the above passage. Option A is close, but it talks about ‗only‘
Therefore, the correct answer is Option (D).
118. Options (A) & (D) do not hold much relevance to the arguments presented by the author.
Option (C) has been discussed by the author but does not weaken the stance. Option (B) definitely
affects the Chinese intention behind the BRI and implications the project will have once it is finished.
Hence, B
119. The author asserts the importance of BRI for China. Refer to ―China may be the new driver for
globalization and it is seeking to achieve this through the Belt and Road Initiative (BRI)‖
Option (C) provides vital information that has a direct impact on the assertions made by the author
and therefore, is the correct answer.
120. Option (B) may be incorrect as the author never discussed the outcome of the BRI project.
Option (C) & (D) are unfounded in the text of the passage. Therefore, the correct answer is Option
(A) which is made clear from the last paragraph.
Hence, A
121. The author begins with the premise that China will be the driving force for the new wave of
globalization. Further information paints the view that China will be a very important factor in
international trade once the BRI is complete. Therefore, the statement is probably true but it cannot
be said with certainty that China will dominate the entire field. There‘s a chance, albeit small, that it
may not be successful.
Hence, A
122. Pallavi is 21st from right and Reena is 10th to the left of Pallavi. So, Reena is 31st from right.
Malini is 4th to the right of Reena. So, Malini is 27th from the right. Also, Malini is 17th from the left.
Therefore number of girls in the row = ( 26 + 1 + 16 ) = 43.

123. Option (A) cannot be verified from the text. Option (B) is incorrect as per the text. Option (D) is
incorrect because ―there are great EXPECTATIONS for India‖. They are yet to be materialized.
Refer to ―few know the experience of countries such as Canada.‖ Therefore, the correct answer is
Option (C).
124. Only Option (C) can be reasonably inferred from the information provided in the above
passage. Refer to ―there are great expectations for India‖
Hence, C
125. Option (C) counters the view presented by the author about the growth of the demand as well
as the supply of wine in the world and especially the Asian market.
Options A, B and D are somewhat strengthening the argument.
Hence, C
126. The author begins his investigation with the aforementioned thought that has been supplied by
the OIV. Then he takes us to the conclusion that China is leading the new world order in the wine
world. Therefore, the given statement is not the conclusion per se, rather it takes us to the
conclusion.
Hence, C.
127. Only Option (B) manages to cover the theme of the discussion and refer to the main point of
contention mentioned by the author.

128. Let‘s first make a BD according to these statements.

Now, see the BD


Conclusion 1 clearly doesn‘t follow.

But, what about Conclusion 2?


Let‘s make a MD
See the MD, Conclusion 2 follows in it.
And if there is a possibility that All drawers are chairs, then how could No drawer is a chair follow?
So, Conclusion 3 will not follow!
1? Doesn‘t follows
2? Follows
3? Doesn‘t follows
129. Options (A) & (D) are unfounded in the passage. Option (C) may be correct but cannot be
ascertained from the text of the passage.
Option B can be inferred from the last two sentences of the passage.
Therefore, the correct answer is option (B).
130. Refer to the last two sentences of the first paragraph. Only Option (C) follows as a reasonable
outcome from the discussion provided by the author. The rest of the options cannot be either
sourced from the passage or are incorrect.
Hence, C
131. Option (A) not negate the point made by the author. The government may have taken notice
now of a problem that persisted since a long time.
Option B highlights selfish motives and weakens the argument that cannabis is good.
Option (C) has been taken into regard by the author while writing the last paragraph. It‘s MISUSE is
wrong, not the ‗use‘ Therefore, the correct answer is Option (b).
132. Using the information given in this Q, the author states that albeit cannabis may have certain
drawbacks, it can be used to cure crack addiction. Therefore, the given statement forms the premise
of the argument and hence, A is correct.

133. Options (A) and (B) are incorrect as the author mentions both views which counter each other.
Option (D) is incorrect as the author clearly is of the view that cannabis has various safe potential
uses that need to be determined.
Hence, C

QUANTITATIVE TECHNIQUES

134. Population of Village A = 22 Lakh


Percentage of Literate = 75%
Number of Literate population = 22 x 75 / 100 = 16.5 Lakhs
135. Total literate Population from all the cities together = Total population x perentage of literate
City A = 22 x 0.75% = 16.5 Lakh
City B = 35 x 0.80% = 28 Lakh
City C = 25 x 0.60% = 15 Lakh
City D = 12 x 0.90% = 10.8 Lakh
City E = 10 x 0.90% = 9 Lakh
Total Literate population = 16.5 + 28 + 15 + 10.8 + 9
= 79.3 Lakh
136. It is clearly visible from the graph that city C has lowest literate percentage 60%
This means highest Illiterate percentage 40% among all cities.
137. Average number of literate population = Total number of population / no of cities = 79.3 / 5 =
15.86 Lakh
138. Total Literate population in 2008 = 22 x 0.75 + 35 x 0.80 + 25 x 0.60 + 12 x 0.90 + 10 x 0.90 =
79.3 Lakh
Total Illiterate population = Total population – Total Literate population = 104 – 79.3 = 24.7 Lakh
139. Let C takes x hours to fill the cistern,
According to the question:
4/24 + 4/x + 5/x + 5/30 = 1
=> 1/6 + 9/x + 1/6 = 1
=> 2/6 + 9/x = 1
=> 1/3 + 9/x = 1
=> 9/x = 1 – (1/3)
=> 9/x = 2/3
=> x = 27/2 = 13 ½ hours
C alone can fill the cistern in 13 ½ hours
140. Let D should be closed after x hours,
x/30 + x/45 + 20/45 = 1
=> (3x + 2x)/90 = 1 – (4/9)
=> 5x/90 = 5/9
=> x = 10 hours
D should be closed after 10 hours
141. C takes half of the time taken by F to fill the cistern = > C : F = 1 : 2
F takes half of the time taken by B to fill the cistern = > F : B = 1 : 2
The ratio of C, F and B taken to fill the cistern = 1 : 2 : 4 (x, 2x, 4x)
According to the question,
1/x + 1/2x + 1/4x = 1/48
= > (4 + 2 + 1)/4x = 1/48
= > 7/4x = 1/48
= > 4x = 48*7
= > x = 84 hours
Time taken by F to fill the cistern = 2*84 = 168 hours
142. Part of the tank filled in 2 hours = 1/24 + 1/30 = 3/40
= > Part of the tank filled in 26 hours = (3/40)*13 = 39/40
Remaining part = 1 – 39/40 = 1/40
Time taken by A to fill the remaining part,
= > (1/40)*24 = 3/5
Total time = 26 + 3/5 = 26 (3/5) hours
143. Let the time taken by F to fill the cistern be x,
According to the question,
(1/24) + (1/30) + (1/x) = (1/8)
(1/x) = (1/8) – [(1/24) + 1/30)]
(1/x) = (1/8) – (3/40)
(1/x) = 2/40
1/x = 1/20
F alone to fill the cistern is 20 hours
144. Total sell in March at P is 330.
At R in March 10% more than this.
So at R in March total sell is 363.
In April total sell at R is 783-363=420
Total at Q is 800. So at Q in March 320.
In April at P total sell is 320*125/100 =400
Total sell in March is 1324, at T, sell in March is 1324-330-320-363 = 311
Total sell in April is = 1324*125/100 = 1655
So at T in April total sell is 1655-400-480-420 = 355

Total sell of P in both month is 330+400 =730


Total sell of Q in both month is =320+480 =800
So required percent is (730/800)*100 =91.25%
145. Total sell in March at P is 330.
At R in March 10% more than this.
So at R in March total sell is 363.
In April total sell at R is 783-363=420
Total at Q is 800. So at Q in March 320.
In April at P total sell is 320*125/100 =400
Total sell in March is 1324, at T, sell in March is 1324-330-320-363 = 311
Total sell in April is = 1324*125/100 = 1655
So at T in April total sell is 1655-400-480-420 = 355

Total sell in April is =400 +480+420+355 =1655


So the average is 1655/4 =413.75
146. Total sell in March at P is 330.
At R in March 10% more than this.
So at R in March total sell is 363.
In April total sell at R is 783-363=420
Total at Q is 800. So at Q in March 320.
In April at P total sell is 320*125/100 =400
Total sell in March is 1324, at T, sell in March is 1324-330-320-363 = 311
Total sell in April is = 1324*125/100 = 1655
So at T in April total sell is 1655-400-480-420 = 355

Total sell in R is 363+420=783


Total sell in T is = 311+355= 666
So required ratio is = 783:666 =87:74
147. Total sell in March at P is 330.
At R in March 10% more than this.
So at R in March total sell is 363.
In April total sell at R is 783-363=420
Total at Q is 800. So at Q in March 320.
In April at P total sell is 320*125/100 =400
Total sell in March is 1324, at T, sell in March is 1324-330-320-363 = 311
Total sell in April is = 1324*125/100 = 1655
So at T in April total sell is 1655-400-480-420 = 355

Sell at P in june is =400*120/100 =480


Sell at Q in june is =480*125/100 =600
Total sell in June is 1080
148. Total sell in March at P is 330.
At R in March 10% more than this.
So at R in March total sell is 363.
In April total sell at R is 783-363=420
Total at Q is 800. So at Q in March 320.
In April at P total sell is 320*125/100 =400
Total sell in March is 1324, at T, sell in March is 1324-330-320-363 = 311
Total sell in April is = 1324*125/100 = 1655
So at T in April total sell is 1655-400-480-420 = 355

Total amount sell at P in march is 330*5 =1650


Total amount sell at R inmarch is 363 *4= 1452.
So total amount is 3102.
149. average number of deaths per day during the given period = Total deaths/Total days = 970/44 =
22.04.

150. 850 x 0.22 x 0.1 = 19.


Legal Edge 28 Mock

Answers & Details

ENGLISH

1. Option (a) cannot be concluded as in the very first line she says “if I had lately left a good home
and kind parents” implying that she didn‟t. Option (b) can also not be concluded, as it is expressly
mentioned that the circumstances don‟t disturb her peace and rather fills her up with excitement.
Option (c) is a reasonable choice as the narrator says, it was a time when she would have missed
her good home if she had one. Also, she seems to be quite unfamiliar to Burns as well.

2. (1) that wind would then have saddened my heart; (2) this obscure chaos would have disturbed
my peace! as it was, I derived from both a strange excitement”

3. Option (a) is not the most suitable choice as Burns is only calling for patience in the specific
situation and is not advocating it as the solution to all vices. Option (c) is not a suitable choice as
Burns only mentions how a consequence might be far reaching enough to affect all her relations and
not that each is responsible for all. Option (d) is also something Burns won‟t agree to as she is
expressly willing to endure disgrace than to commit an action she will regret. Option (b) is the correct
choice as she clearly chooses to suffer alone than to affect her relations as well.

4. Option (a) is wrong as Mr. Brocklehurst is mentioned as the one in power to expel students from
school and nothing else. Option (c) Lowood is the name of the place they are at. Option (d) Felix is
an unexpanded character. Option (b) is a correct choice as Miss Scatcherd is mentioned to punish
Burns cruelly, hence fitting the term “chastiser”

5. Option (a) when considered literally seems absurd. With two other options at our expense there is
no need to make assumptions about the figurative meaning of the statement. Option (c) cannot be
the right choice, as the narrator emphasizes repeatedly that she doesn‟t understand Helen even
though Helen could be right. Option (b) conveniently states this very thing, that Narrator does not
understand the way Helen Burns thinks.

6. While Option (a) may be a valid answer, it has not been discussed in the passage and hence is
not the correct choice. Option (c) is not the correct choice as summer is not mentioned in the line.
Moreover, the passage points out to García Márquez‟s ability to suggest the passing of time without
using seasons as a marker. Option (b) is the correct choice as the presentation of time as something
endless and something that can be transcended is unusual for temperate climates.

7. Option (a) is the correct choice as the author expands this idea by talking about the landscape in
the tropics and how Gabriel García Márquez‟s writing reflects his tropical origins. Option (b) is never
discussed in the passage with respect to there being rules to writing and whether or not he followed
them. Option (c) is something the author doesn‟t agree with, clearly, as she explains how her side of
the world suggests permanence and eternal nature of things rather than transience. Option (d) is
also never suggested by the author.

8. (1) is not discussed in the passage. Hence all options containing (1) are eliminated.
9. Option (a) is the correct choice as the author mentions the word “discover” gives the impression of
knowledge of previous generations rippling through time. Option (b) and (c) have not been
mentioned in the passage.

10. Surreal means very strange; with images mixed together in a strange way like in a dream.
Surreal is synonymous to dreamlike.

11. Narrator, Narrator‟s grandmother, A short and wide man, two long and thin men

12. (1) cannot be ascertained as she wore four skirts in a cyclic way, not seven. (4) would also be an
assumption and not something certain. Option (b), hence is correct as she wore four skirts at the
same time with a system of rotation, it seems.

13. The man disappears out of sight multiple times. However, he jumps from the brickwork‟s
chimney and not into it.

14. The extract depicts a heated chase after a man running from local police. It goes on for the
entirety of the extract, however, the tone is very relaxed. This is because grandmother is viewing it
only as some kind of amusing performance, treating the time they disappear as intermission to
complete what she was doing. She barely heaves a sigh while hiding the man. Even after it is said
she scarcely felt any difference. Moreover, unlike, Option (b), the reader is seeing the scene through
the narrator‟s eyes, for whom a dramatic incident seems calm because of his grandmother.

15. Spit has got following meaning :


1a : to eject (something) from the mouth : expectorate. b(1) : to express (unpleasant or malicious
feelings) by or as if by spitting. (2) : to utter with a spitting sound or scornful expression spat out his
words.
It's going with the phrase given and hence C is the answer.
16. Gloom means partial or total darkness. Option A ''sadness'' means lowness of spirits. Option B
''shadow'' means a dark or shadowy place.
Option D ''melancholic'' means dejection or an atmosphere of despondency a gloom fell over the
household.
On the other hand, antonyms for gloom includes brighten, cheer (up), lighten, perk (up).
17. While Options (a), (c) and (d) were things that were discussed in the passage, each of them only
sprouted from the discussion of Amitav Ghosh‟s new work „Jungle Nama‟

18. Option (a) is not the answer as the metre is mentioned to be in use only for the last few hundred
years. Moreover, this was a separate piece of information stated in the passage. Option (b) is false
as it was the use of verse which made reading a community affair. Option (c) is clearly mentioned in
the passage as the ability of a verse to preserve the message of the writing even when it was taken
to other regions and local variaions were made of the same.

19. (2) is not a point of difference between modern and premodern literature as discussed in the
passage. Within the passage, evidently, the author himself is writing stories based on legends.
20. Option (a) is wrong as the engagement with nature, exploration of human greed is mentioned a
couple of times throughout the passage. Option (c) is wrong, as we know from the reading of the
passage, he is the author of “the Great Derangement” and “Jungle Nama”. Option (d) is wrong as
the passage discusses how Ghosh thinks the ability of verses to be chanted and sung aloud
contributes to the work‟s posterity. Moreover, he wrote his work in verse with two-footed meter.
21. Sophisticated has got two meanings:-
1. having or showing a lot of experience of the world and social situations; knowing about fashion,
culture, etc.
2. (used about machines, systems, etc.) advanced and complicated.
Second meaning is befitting to the phrase given and thus it''s a correct answer choice.

22. ENCAPSULATE means to express the essential features of (something) succinctly or to sum up
or summarise. Therefore, D is the correct option.

23. The correct option is b). The correct sentence should be, „I was born in a small city (medium by
Spanish standards) called Las Palmas de Gran Canaria, in the Canary Islands, a group of islands
that belong to Spain but are geographically located very close to the coast of northwestern Africa.‟

24. The correct option is (c) . The passage took an emotional turn from the third paragraph as the
author starts to narrate a part of his emotional journey to understand, choose and embrace his
identity while growing up in a multi-cultural atmosphere.
Options a) and b) are not correct as the author has never generalised or used a common concept to
describe the life of every Sindhi kid growing up with their parents in a foreign land. The following
sentence proves the point: „Among people of my generation or circumstances, however, it may not
necessarily be the same; the weight of their or their parents‟ origin in their identities can be weaker
or stronger. Some people feel closer to the land where they have been born and raised, others feel
closer to their parent‟s places of origin. And some, like me, are somewhere in the middle, in between
two poles, where the shades of grey are infinite yet equally valid.‟

25. the correct answer is option b) as the second statement is described in the second paragraph of
the passage. Statement 1 is not correct as of the last paragraph mentions, „Today, being in the grey
areas is a good feeling but it wasn‟t so easy when I was a teenager and into my early twenties to
figure out where I actually belonged‟.

26. The correct answer is option d) as the correct sentence would


be: Among people of my generation or circumstances, however, it may not necessarily be the
same; the weight of their or their parents’ origin in their identities can be weaker or stronger.

27. The correct answer is option b). Extended metaphors build evocative images into a piece of
writing and make prose more emotionally resonant. The author here tried to portray his emotional
state about his identity and sense of belongingness. This extended metaphor comes into place right
after the paragraph: „My identity has a lot to do with my origins; with what and who I feel I am; with
where I come from; with to where I feel a sense of belonging. Perhaps one of the reasons for this is
because I was born and raised in a country and culture-of-the-land totally different to that of my
parents‟.‟

28. Diaspora means a group of people who spread from one original country to other countries, or
the act of spreading in this way.
GENERAL KNOWLEDGE

29. The New START Treaty is a treaty between the United States of America and the Russian
Federation on measures for the further reduction and limitation of strategic offensive arms.

30. It will lapse in February 2021 unless extended for a five-year period.

31. This step by Russia is a welcome move after suspension of the Intermediate-Range Nuclear
Force Treaty (INF Treaty) in 2019 and withdrawal of USA and Russia from Open Skies Treaty
recently.
An extension of the New START Treaty would mark a rare bright spot in the fraught USA-Russian
relationship. This opportunity could be used by both the countries for conducting comprehensive
bilateral negotiations on future control over nuclear missile weapons.

32. The term „strategic offensive arms‟ applies to nuclear warheads deployed by Strategic Nuclear
Delivery Vehicles („SNDVs‟).
SNDVs are Inter-Continental Ballistic Missiles („ICBMs‟) with a range exceeding 5,500 kilometres,
strategic bombers, warships (including strategic submarines) and cruise missiles, including air and
sea-launched cruise missiles.

33. Both houses of Russian Parliament (Kremlin) approved the extension of the New START treaty
for five years. It was done after a recent telephonic conversation between the newly elected USA
President and the Russian President.
At the World Economic Forum‟s virtual meeting, the President of Russia hailed the decision to
extend the treaty as “a step in the right direction,” but also warned about the rising global rivalries
and threats of new conflicts.The pact‟s extension doesn‟t require congressional approval in the USA,
but Russian lawmakers must ratify the move and its President has to sign the relevant Bill into law.

34. The Treaty on Prohibition of Nuclear Weapons was adopted on July 7, 2017. The treaty entered
into force on January 22, 2021. The treaty is the first legally binding international agreement that aim
to eliminate nuclear weapons completely.

35. However, India showed its support to the commencement of negotiations on a “comprehensive
Nuclear Weapons Convention in the Conference on Disarmament”. It is the only multilateral
disarmament negotiating forum that works on the basis of consensus.

36. The United Nations General Assembly (UNGA) had approved the “Treaty on the Prohibition of
Nuclear Weapons” in the year 2017.However, the nine countries that possess the nuclear weapons
did not support it.The treaty was not supported by the NATO alliance as well.Despite that, the treaty
came into force on January 22, 2021.

37. The recognized “Nuclear weapon States” are United States, Russia, United Kingdom, China and
France. The states that have declared possessing the nuclear weapons include India, Pakistan and
North Korea while, Israel is the state that indicated for possessing the nuclear weapons.

38. FMCT is a proposed international agreement that prohibits the production of two main
components of nuclear weapons: highly-enriched Uranium and Plutonium.The consultations under
the treaty laid down the most appropriate arrangement to negotiate a treaty banning the production
of fissile material for nuclear weapons or other nuclear explosive devices.
39. President Donald Trump has held off sanctioning Turkey for the purchase under the 2017
Countering America‟s Adversaries Through Sanctions Act, or CAATSA, the defense bill would order
that five or more sanctions under CAATSA be imposed within 30 days.

40. The USA administration has recently imposed sanctions on Turkey for its purchase of the S-400
missile system from Russia.The issue of sanctions under Section 231 of the Countering America‟s
Adversaries Through Sanctions Act (CAATSA) for purchase of Russian arms is of particular interest
to India, which is also in the process of buying the S-400 from Russia.

41. Sanctions were imposed on Turkey‟s main defence procurement agency, the Presidency of
Defense Industries (SSB).
These sanctions comprise a ban on granting specific U.S. export licenses and authorisations for any
goods or technology.
Also, a ban on loans or credits by U.S. financial institutions totaling more than 10 million USD in any
12-month period.
A ban on U.S. Export-Import Bank assistance for exports and mandated U.S. opposition to loans by
international financial organisations to SSB.

42. The CAATSA contains 12 types of sanctions. Of these, 10 will have very little, or no, impact on
India‟s current relations with either Russia or the US. There are only two sanctions that may impact
either India-Russia relations or India-US relations.

43. Enacted in 2017, it is a US federal law that imposed sanctions on Iran, North Korea and
Russia.Includes sanctions against countries that engage in significant transactions with Russia‟s
defence and intelligence sectors.

44. Israel has adopted a law defining the country as the nation-state of the Jewish people. The
legislation becomes part of the country‟s basic laws, which serve as a de facto constitution.

45. Lays down that Israel, with the „whole and united‟ Jerusalem as its „capital‟ is the historic
homeland of the Jewish people and they have an exclusive right to national self-determination in it.
Defines the promotion, encouragement, establishment and consolidation of Jewish communities as
being in the national interest.
Stripping Arabic, the first language of 1.8 million citizens, of its national language status and only
granting it a „special status‟ while making Hebrew the country‟s national language.

46. Mideast War,which took place in 1967 is also known as the six-day war or Third Arab-Israeli war.
Israel captured the West Bank, east Jerusalem and Gaza Strip in the war. The Palestinians seek
these territories for a future independent state. In the decades since, Israel has built settlements in
the West Bank and east Jerusalem that now house nearly 700,000 Israelis.

47. The Basic Law: Israel as the Nation-State of the Jewish People is informally known as the Nation
State Law. It is one of the fourteen constitutional laws or basic laws of Israel. The law permits Israel
to identify itself as a nation-state of Jewish people.

48. The Israeli Knesset voted on 19 July 2018 by a margin of 62 to 55 to approve the Jewish Nation-
State Basic Law, constitutionally enshrining Jewish supremacy and the identity of the State of Israel
as the nation-state of the Jewish people.

49. Nepal and China have announced the revised height of Mount Everest as 8,848.86 metres. The
new height is 86 cm more than the previous measurement.

50. Everest is also known as Sagarmatha in Nepal and Mount Qomolangma in China.
51. The first effort was carried out in 1847 by a team led by Andrew Waugh, Surveyor General of
India.The survey was based on trigonometric calculations and is known as the Great Trigonometric
Survey of India.

52. It was first scaled in 1953 by the Indian-Nepalese Tenzing Norgay and New Zealander Edmund
Hillary.

53. New Zealand government provided Global Navigation satellite and trained technicians to
measure the mountain.

54. The Government of National Capital Territory of Delhi Act, 1991 gave primacy to the centrally
appointed Lieutenant Governor of Delhi and make the elected Government of Delhi subsidiary.

55. The Delhi government will now have to seek the opinion of the lieutenant governor before any
executive action.

56. Delhi is not a full state. It became a Union Territory with an elected legislative assembly by virtue
of the 69th Amendment Act through which Articles 239AA and 239BB were introduced in the
constitution in 1991

57. Delhi, Puducherry and Jammu and Kashmir have an elected legislative assembly and an
executive council of ministers with partially state like functions.

58. India and Bangladesh have decided to celebrate December 6, the day on which India formally
recognized Bangladesh, as 'Maitri Diwas', a joint statement issued after talks between Prime Minister
Narendra Modi and his Bangladeshi counterpart Sheikh Hasina

59. Indian conferred the father of the nation of Bangladesh with Gandhi Peace Prize for the year
2020.

60. India and Bangladesh have decided to celebrate December 6, the day on which India formally
recgnized Bangladesh, as 'Maitri Diwas', a joint statement issued after talks between Prime Minister
Narendra Modi and his Bangladeshi counterpart Sheikh Hasina

61. India and Bangladesh share 54 common rivers. A bilateral Joint Rivers Commission (JRC) has
been working since June 1972 to maintain liaison between the two countries to maximize benefits
from common river systems.

62. Bangladesh will assume chairmanship of the Indian Ocean Rim Association (IORA) in 2021 and
requested the support of India for working towards greater maritime safety and security.

63. The longest border is shared by Bangladesh which is 4096.70 Km (2,545-mile).


LEGAL REASONING

64. The correct answer is option (a). Pyare Lal can take the defence of necessity because he was
trying to save the bananas from getting rotten. Therefore, the act of selling them to the nearest seller
he could find was necessary and reasonable.

65. The correct answer is option (c). The harm to be prevented or avoided was of not of such a
nature and so imminent as to justify or excuse the risk of doing the act with the knowledge that it was
likely to cause harm. The harm caused to the needy was not so imminent so as to knowingly
distribute half of the consignment of winter clothes which would cause harm to the seller in Srinagar.
The act is not reasonable, even if it was done in good faith or with criminal intention.

66. The correct answer is option (b). The harm to human lives, i.e. death due to sinking of ship was
greater than the harm caused as a result of unloading the COVID-19 vaccine into the water. The
threat to human lives was imminent and the captain threw boxes of the vaccine into water in good
faith to save the passengers. Therefore, he can take the defence of necessity.

67. The correct answer is option (d). The defence of necessity requires the defendant to prove on his
part that the unlawful act committed by him was to avoid a significant risk of harm and no other
alternative lawful means were available. Both of these elements were present as Lakhan drew water
from that particular well as it was the closest and the most reasonable thing to do would be to draw
water from it to save his hut from suffering any more damage. Fire in someone‟s house is a much
greater harm than trespass. Therefore, Lakhan can plead the defence of necessity.

68. The correct answer is option (c). The communication of an acceptance is complete, as against
the proposer, when it is put in a course of transmission to him so as to be out of the power of the
acceptor. Here X is the acceptor and Y is the proposer since he made the offer of buying the cow for
Rs7000. Therefore, acceptance against Y is complete when it was put in the course of transmission
by X i.e. On 5th January.

69. The correct answer is option (b). Telephonic communications become complete when the
acceptance is received by the offeror. In this case, Sonu could not receive the communication of
acceptance due to the noise created by the metro. The communication of acceptance in postal rule
is different from instantaneous modes such as telephone, where the course of transmission is
irrelevant and the acceptance is complete only upon being received. Hence, acceptance of offer is
not complete.

70. The correct answer is option (a). The communication of a revocation is complete, as against the
person who makes it, when it is put into a course of transmission to the person to whom it is made,
so as to be out of the power of the person who makes it. Since Makkhi Chand also made a
communication of revocation, it is complete against him when he wrote the letter to Jalebi Bai i.e. on
30th May 2020.

71. The correct answer is option (b). In electronic modes such as zoom, communication of
acceptance is completed when it is received by the offeror. Here, the acceptance was received by
the supplier as Sanjana‟s words were audible to him so the mere presence of loud noises in the
background does not mean that the acceptance is incomplete.

72. (a) Facts of question prove that statement was not made carelessly therefore this is not the right
option to be chosen. (b) This is an appropriate answer because naming a particular religion in a
political rally shows political motive to outrage the religious feelings of the particular class. (c)
Question is asking about act come under Section 295A or not it is not talking about the restriction on
freedom of speech and expression. (d) Term reasonable restriction is missing therefore this will not
be chosen option.

73. (a) Janvi was giving opinion within a framework of law without outraging any religious feeling of
class this is clear by reading the facts of the question. (b) No derogatory comments were made,
author given her personal opinion considering law.(c) Question not able to prove any derogatory
comments on religion therefore this will be the not correct option.

74. (a) Person has the right to freedom and expression with restrictions due to term without this
option is nullified. (b) Conduct of Ghosh was only of giving personal opinion according to his
understanding of Gita and he commented unwittingly without any malicious intention. (c) It attracts
section 295A but Ghosh''s statement was not derogatory.

75. (a) Section 295 is written instead of Section 295A. Therefore option becomes incorrect. (b) Term
not is missing.(c) Court said mention statement in Sri Dulal v state of Tripura

76. (a) Professor is analysing history topic without any malicious intention. (d) Both B and C are
given in the passage.

77. (a) 1st passage (b) Information will be removed only when it is not in conformity with the user
agreement. Refer to the last line of the 1st passage. (c) The Term NOT is inappropriate.

78. (c) Both A and B are mention in IT RULES OF 2021. (d) On the complaint of person term is
missing.

79. (a) 5th line of 3rd paragraph.

80. Mention in the guidelines of the passage.

81. (b) Passage says that termination of account will take place when information is not in conformity
with the user agreement or privacy policy but in this question it was with the conformity of
user…….policy. Therefore option B is the correct answer.

82. It is mention in the passage that the mere silence of a person is not the consent of the person.
Option (b) polygraph test cannot be obtained without the written consent of the accused therefore
option b is incorrect. Option(c) rights of investigation officer are not discussed in the passage
regarding taking polygraph test instead investigating officer has no right to disclose the private data
is discussed.

83. option (a) only oral consent is not enough for administrating polygraph test, written consent is
mandatory. Option (b) Written permission of the accused is required. Option (c) 1st line of 1st
paragraph. Option (d) yes is written instead of No.

84. if a person was to refuse the administration of polygraph test, no such polygraph the test could
be administered and even if administered, the result of the said the test would be void and cannot be
considered by a Court of Law, Option B, and C are incorrect because the question is asking validity
of polygraph test results not about conditions for administrating polygraph tests.

85. option (a) no permission is granted from the court to the investigation officer for disclosing
information option (b) Incorrect because no relation is there between duty and disclosing information.
Option c is the correct answer because written permission from the court is needed.
86. All the option (a), (b) and (c) are mentioned in the passage are true hence the correct answer is
option (d)

87. The correct answer is (a). After the expiry of 90 days an accused has a default right to get bail. In
the present case, if Kalpana has filed for bail before the filing of the chargesheet her application shall
be allowed and discretion of the magistrate would then be immaterial.

88. The correct answer is (b). In the aforementioned case, the court held that the stipulated period
having expired and the charge sheet not having been led qua those offences (albeit a
supplementary charge sheet), we are of the view the appellant would be entitled to default bail in the
aforesaid facts and circumstances. In the present case, Police filed the chargesheet and then sought
time to file a supplementary chargesheet which had more consolidated data on investigation to
prevent the accused being released on bail. Therefore, the writ petition shall be allowed in the light
of aforementioned judgment.

89. The correct answer is (b). Both A and R are true, but R just defines the stipulated time period as
given in A. The correct explanation to A is that after expiry of the stipulated period an accused has a
fundamental right to be released on bail under article 21.

90. The correct answer is (c). An accused has a default right to bail after the expiry of stipulated
period if they are prepared to furnish the bail. In the present case, Porus‟s lawyer did not apply for
the bail even after the expiry of 60 days and thus Police filed the chargesheet on the basis of which
a magistrate can extend the period of custody.

91. The correct answer is (d). In the present case, the Police does not seek an extension on custody
of the accused and time period to file supplementary chargesheet. The extension of custody was
allowed on filing of the chargesheet and Police later on submitted a supplementary chargesheet with
more charges.

92. The correct answer is (b). Any Court may, at any stage of any trial, re-examine any person
already examined if his evidence appears to it to be essential to the just decision of the case.
Further, an application filed under Section 311 could not be rejected on the sole ground that the
case had been pending for an inordinate amount of time.

93. The correct answer is (b). In Swapan Kumar Chatterjee v. Central Bureau of Investigation, the
bench observed that the first part of section 311 gives purely discretionary authority to the criminal
court and enables it at any stage of trial to examine any person in attendance if their evidence
appears to it to be essential to the just decision of the case.

94. The correct answer is (d). In Swapan Kumar Chatterjee v. Central Bureau of Investigation, the
bench observed that section 311 gives purely discretionary authority to the criminal court and
enables it at any stage of inquiry, trial or other proceedings under the Code to re-examine any
person already examined if his evidence appears to be essential to the just decision of the case.

95. The correct answer is (a). Section 311 deals with the power to summon material witness, or
examine a person present. Any Court may, at any stage of any inquiry, trial or other proceeding
under this Code, summon any person as a witness, or examine any person in attendance, though
not summoned as a witness, if his evidence appears to it to be essential to the just decision of the
case.

96. The correct answer is (a). Section 311 deals with the power to summon material witness, or
examine person present. Any Court may, at any stage of any inquiry, trial or other proceeding under
this Code, summon any person as a witness, or examine any person in attendance.
Application under sec 311 could not be rejected on the ground that case is pending for an inordinate
amount of time.

97. (c) “Defamation is the act of communicating false statements about a person that injure the
reputation of that person when observed through the eyes of ordinary man.” The statement by
Krishna did not lower the reputation of Gopi in the eyes of the right-thinking people and therefore he
will not be liable for defamation.

98. (c) Refer “Any false and unprivileged statement published or spoken deliberately, intentionally,
knowingly with the intention to damage someone''s reputation is defamation”. Though there was a
communication of a defamatory statement to the third party, it was done by Dr. Grey himself. Thus,
Denny will not be held liable for defamation.

99. (b) The statement, even if injurious, was not "about a person" - it was about a loose body of
persons and Christian Gale cannot claim that he was targeted by that statement and his reputation
was damaged.

100. (b) “Any false and unprivileged statement published or spoken deliberately, intentionally,
knowingly with the intention to damage someone''s reputation is defamation”. Aditya''s statement
may not refer to a Specific individual but it makes reference to all members of the communist party.

101. (c) “Defamation is the act of communicating false statements about a person that injure the
reputation of that person when observed through the eyes of ordinary man.” The facts mention that
people returned to work without being bothered by Verma‟s remark on Malhotra shows that there
was no harm done to the reputation. Therefore, not a case of defamation.

102. (c) Written defamation printed, or typed material or images is called as libel and spoken
defamation is called slander. Therefore (a) and (b) are libel whereas (c) is slander.
LOGICAL REASONING
103. option A is not true because all good cooks are not big eaters. Option C is not correct because
fast food cooks use rich ingredients as long as the recipes are easy to follow and take little time.
Option D is not correct because all good cooks are not concerned with nutritional value of food. But
despite all this, all good cooks enjoy preparing and serving foods as provided in passage.
Therefore, option B is the correct answer.
104. Option A is the correct answer because it is the most likely assumption that author may have
made is to see their consumers satisfied after eating the food, which is made by good cooks, gives
utmost happiness to the good cooks. Option (B), (C) and (D) are not correct assumption because
they are not leading to the conclusion.
Hence, A
105. Above passage provides that some good cooks can be characterized as fast food cooks. They
may use rich ingredients as long as the recipes are easy to follow and take little time.
Therefore the correct answer is option (B).
106. Option (A), (B) and (D) may be true or may not be true, but option (C) is definitely not true
because it is provided in passage that all good cooks are not big eaters.
Therefore, the correct answer is option (C).
107. The passage is an example of „either …. or‟ fallacy. It suggests that only two options exist,
whereas in reality there might be more. This fallacy is correctly explained in option C.
Option B is incorrect as it speaks about all economic problems, whereas the passage is specifically
addressing only economic growth and job opportunities. Hence C.
108. Murari says that we should not take sides then only we will be logical and not be a blind
devotee. Thus option (b) is the answer.

109. Murari‟s main idea in the passage is that we should not take sides. The reason for his argument
is that taking definite sides makes a person illogical and a blind bhakt. Thus this forms the premise
of his main conclusion that we should not take sides. So option (a) is correct.

110. Murari‟s arguments is centred around moderation and picking a side one issue at a time. In
option (a) Atul is a die-hard fan, yet he has reservations against Kohli‟s behavior. In option (b) Oliver,
though a proud German, apologises for his country‟s Nazism accepting the fault of his country. In
option (d) Buddha preaches his path of middle way. All these cases are supporting Murari. In option
(c), we see Fahim supporting his son even when his son is wrong. This is Ashish‟s argument. So
option (c) is the answer.
111. Options (a), (b) and (d) form the premise to the arguments of Ashish. The conclusion is option
(c) that we should have a firm stand and support it always.
112. Ashish says that we should have some sort of an opinion in our life and know whom to support
and then support them whole-heartedly even if we are called a blind devotee. Then and only then we
will have a cause in our life. So, option (b) is the answer.
113. This statement properly identifies the point that, for ordering from another city mill to be less
expensive, the shipping and handling fees must be less than the difference in the flour costs of the
two suppliers. Say, for example, that a batch of flour costs Rs.100 from Bhopal Flour Mill. The
passage tells us that the same batch would cost Rs.90 from Indore Flour Mill, yet when purchasing
from Indore, shipping and handling fees would apply. We are told that Indore‟s total cost is cheaper
than Bhopal‟s, so mathematically that is: Rs90 + Shipping & Handling < Rs100, which means that
Shipping & Handling < Rs10 = 10% of the cost of flour from Bhopal‟s.
114.

(a) T is to the right of R. The correct option is (a)


115.

M and V are sitting opposite each other. The correct option is (a)
116.
(c) In the first option, T is sitting opposite to U who is sitting immediately next to P. In the second
option R is sitting opposite to P who is sitting next to V. The same pattern is followed in all the other
options except the option (c). The correct option is (c)
117.
(b) P is sitting left of V. Thus, the correct option is (b).
118.
(d) P is sitting opposite of R. Thus, the correct option is (d).
119. None of the options can be inferred from the information given to us in the passage. The
passage nowhere mentions levels of health and education of the parents or the opportunities,
choices and services that are available to the people decide the number of children that people
have.
Hence, D
120. The author seems to be against the proposals mentioned in the first paragraph. He has also
sighted the NPP. Therefore, the given statement supports the author‟s view.
Hence, B
121. Option 1 rebuts the theory of “population explosion”, and hence is the correct option choices.
Option 2 is incorrect as it talks about just a few states which will move toward a stable population as
fertility rates falls.

122. The passage mentions no link between a healthy population and growth of a nation, and we
cannot assume it to be true unless it is mentioned in the passage.

123. The author‟s argument does not involve pitting Zheng He against Columbus in a „best explorer‟
contest, so neither option (a) nor option (b) is correct. On the other hand, the scholars whose views
he cites at the beginning of the passage do seem to want to compare Zheng He and European
explorers: „Many scholars argue that the voyages of Admiral Zheng He of the Chinese Ming dynasty
heralded and eclipsed the European voyages of discovery.‟ In this case, the author‟s claim about the
massive size of Zheng He‟s fleet („drove of dragons‟) as opposed to Columbus‟ paltry three ships
(„trio of mosquitoes‟) does support these other scholars‟ view that Zheng He‟s voyages eclipsed
European voyages (or at least, Columbus‟).
Hence, (d).
124. The passage describes Zheng He‟s voyages only in the Indian Ocean, not the Pacific Ocean,
so option (a) is not fully inferable. There is no mention whatsoever which ocean/s Columbus
voyaged across, so option (b) is not inferable at all.
Hence (d)
125. Options (a) and (b), both of which wrongly interpret this passage as a competition between
Zheng He and Columbus, can be ruled out at once. Option (c) is factually incorrect, as the passage
makes it clear that Zheng He‟s voyages (1405–1433) predated Columbus‟ (1492). Only option (d)
correctly expresses the main point of the passage, which emphasizes that despite the much greater
scale of Chinese voyages of discovery, the Chinese attitude towards exploration was very different
than that of the Europeans.
Hence (d).
126. The same statement cannot simultaneously strengthen and weaken the conclusion, so option
(d) can be dismissed as illogical. The author‟s conclusion is that the Chinese did not value
exploration the way that the Europeans did, and that the expeditions of Zheng He were not deeply
rooted in Chinese politics and culture. So if he were to go on to claim that Zheng He‟s voyages are
still remembered reverentially by the Chinese and that Zheng is considered a national hero by them,
it would cast doubt on his conclusion and weaken it.
Hence (b).
127. Refer to the second paragraph of the passage: „When the ruling faction in Beijing changed
during the 1430s, the new overlords abruptly terminated the operation. The great fleet was
dismantled, crucial technical and geographical knowledge was lost, and no explorer of such stature
and means ever set out again from a Chinese port.‟ This change in policy was clearly not just a
personal vendetta against Zheng He himself, but rather an abrupt reversal of national priorities. Thus
option (a) can be ruled out. Option (c) is not a suitable explanation either, since if it were true, the
problem could have been much more easily solved by putting another admiral, who was more willing
to conquer and colonize, in charge of the fleet. Given that the new rulers dismantled the entire fleet,
option (d) does not make sense. Only option (b) serves as a suitable explanation, as it provides a
reason for not only why the on-going expeditions were cancelled, but also why there were no such
expeditions in the future.
Hence (b).
128. Ponds appear less deep than they are because of the phenomenon of refraction. Hence, A

129. Weaken the argument Q. Only option C attacks the view of the lawmakers. If consumers are
highly educated, they can certainly be judicious when purchasing such items. Hence, C

130. The conclusion of the study is that playing sounds to new-born babies has the potential of on-
line early sleep-learning. This is based on the stronger DAN which gave the indication that the new-
borns have „learned to recognize‟ the sounds. You have to weaken this argument. Option [A] does
not weaken the argument that new-borns can learn in their sleep and by implication strengthens the
argument that they can learn in their sleep. Option [B] is about school-going children. Hence it
doesn‟t affect the main argument about new-borns. Though [C] says the ability disappears later, that
does not weaken the argument that the new-borns learn in their sleep. On the other hand, [D] breaks
the link between recognition and stronger DAN. If the new-borns who heard a new sound in their
sleep recorded a stronger DAN, there may not be any relation between a stronger DAN and
recognition of a sound. Hence the link between stronger DAN and learning gets broken and
weakens the conclusion. Hence, [D].

131. (c) A and E are the pair facing opposite to the centre. Correct option is (c)
132. (d) B sits second to the left of A. Correct option is (d)
133. (b) G is sitting to the immediate left of E. Correct option is (b)
134. (d) F is third to the left of B. Correct option is (d).
135.
QUANTITATIVE TECHNIQUES

136. Total earnings =1080×100=12.5L


Required % =20100×10L12.5×100=2×8=16%
137. Required ratio =(20% + 10%) : (12% + 5%) = 30 : 17

138. Increase in House rent = 20100×10100×10=0.2L


Percentage decrease in expenditure on Clothing= =0.220100×10×100=10%
Expenditure of Clothing and Food together = (20% + 20%) of 10 L
Expenditure of Other & Education = (28% +12%) of 10 L
Required percentage = =40% of 10L240% of 10L2×100=100%
139. Required average expenditure =90% of 10L5=90 ×10100×5=1.8L

140. Baby boys from district A & B = 30100×450+64100×500


= 135 + 320 =455
Baby girls from district E and F = 52100×650+68100×525
= 338 + 357 =695
Required Difference = 695 - 455 = 240
141. Average no. of children from E, C & D
=650+470+3503
=14703=490
No. of baby boys from D, E and F
= 350 ×36100+650×48100+525×32100
= 606
Required % =606-490606 ×100= 19.14%
142. No. of Baby boys from D & E
= 350 × .36 + 650 ×.48
= 126 + 312 = 438
No. of Baby girls from D, E & F
= 350 × .64 + 650 × .52 + 525 × 68 = 919
Ratio = 438/919
143. No. of Baby boys from A = 450 ×.30 = 135
No. of Baby girls from C = 470 ×.50 = 235
Required % = 235-135135 ×100=74.07%
144. No. of Baby girls from C & A = 470 × .50 + 450 × .70
=235+315=550
Required ratio =500+650550=1150550=2311
145. Number of employees in company
A = 512 × 4800 = 2000
Number of employees in company B
=712×4800=2800
Male employees in company A=70100×2000=1400
Female employees in company A = 2000 - 1400 = 600
Male employees working in ''Ops'' in company A
=60100×1400=840
Male employees working in Admin in company
A = 18(1400-840) = 70
Male employees working in others in company A
= 78(1400-840) =490
Female employees working in Admin in company A
=24100×600=144
Female employees working in ''Ops'' in company
=58600-144=285
Female employees working in Others in company
A = 600-144-285 = 171
Male employees in company B
=80100×2800=2240
Female employees in company B = 2800 - 2240 = 560
Male employees working in ''Ops'' in company B = 65100 × 2240 = 1456
Male employee working in ''other1 in company B
=1210×490=588
Male employees working in ''Admin'' in company B
= 2240-1456-588 = 196
Female employees working in ''Ops'' in company B
=1456×25100=364
Female employees working in ''Admin'' in company B
=14560-364=49
Female employees working in Others in company B
=34560-364=147
For company A, Total = 2000

Required % =4901400×100=35%
146. Number of employees in company
A = 512 × 4800 = 2000
Number of employees in company B
=712×4800=2800
Male employees in company A=70100×2000=1400
Female employees in company A = 2000 - 1400 = 600
Male employees working in ''Ops'' in company A
=60100×1400=840
Male employees working in Admin in company
A = 18(1400-840) = 70
Male employees working in others in company A
= 78(1400-840) =490
Female employees working in Admin in company A
=24100×600=144
Female employees working in ''Ops'' in company
=58600-144=285
Female employees working in Others in company
A = 600-144-285 = 171
Male employees in company B
=80100×2800=2240
Female employees in company B = 2800 - 2240 = 560
Male employees working in ''Ops'' in company B = 65100 × 2240 = 1456
Male employee working in ''other1 in company B
=1210×490=588
Male employees working in ''Admin'' in company B
= 2240-1456-588 = 196
Female employees working in ''Ops'' in company B
=1456×25100=364
Female employees working in ''Admin'' in company B
=14560-364=49
Female employees working in Others in company B
=34560-364=147
For company A, Total = 2000
Required % =49560×100=8.75%
147. Number of employees in company
A = 512 × 4800 = 2000
Number of employees in company B
=712×4800=2800
Male employees in company A=70100×2000=1400
Female employees in company A = 2000 - 1400 = 600
Male employees working in ''Ops'' in company A
=60100×1400=840
Male employees working in Admin in company
A = 18(1400-840) = 70
Male employees working in others in company A
= 78(1400-840) =490
Female employees working in Admin in company A
=24100×600=144
Female employees working in ''Ops'' in company
=58600-144=285
Female employees working in Others in company
A = 600-144-285 = 171
Male employees in company B
=80100×2800=2240
Female employees in company B = 2800 - 2240 = 560
Male employees working in ''Ops'' in company B = 65100 × 2240 = 1456
Male employee working in ''other1 in company B
=1210×490=588
Male employees working in ''Admin'' in company B
= 2240-1456-588 = 196
Female employees working in ''Ops'' in company B
=1456×25100=364
Female employees working in ''Admin'' in company B
=14560-364=49
Female employees working in Others in company B
=34560-364=147
For company A, Total = 2000

Required no. of female =285+364=649


148. Number of employees in company
A = 512 × 4800 = 2000
Number of employees in company B
=712×4800=2800
Male employees in company A=70100×2000=1400
Female employees in company A = 2000 - 1400 = 600
Male employees working in ''Ops'' in company A
=60100×1400=840
Male employees working in Admin in company
A = 18(1400-840) = 70
Male employees working in others in company A
= 78(1400-840) =490
Female employees working in Admin in company A
=24100×600=144
Female employees working in ''Ops'' in company
=58600-144=285
Female employees working in Others in company
A = 600-144-285 = 171
Male employees in company B
=80100×2800=2240
Female employees in company B = 2800 - 2240 = 560
Male employees working in ''Ops'' in company B = 65100 × 2240 = 1456
Male employee working in ''other1 in company B
=1210×490=588
Male employees working in ''Admin'' in company B
= 2240-1456-588 = 196
Female employees working in ''Ops'' in company B
=1456×25100=364
Female employees working in ''Admin'' in company B
=14560-364=49
Female employees working in Others in company B
=34560-364=147
For company A, Total = 2000

Required difference =171+1472-70+1962


=159-133=26
149. Number of employees in company
A = 512 × 4800 = 2000
Number of employees in company B
=712×4800=2800
Male employees in company A=70100×2000=1400
Female employees in company A = 2000 - 1400 = 600
Male employees working in ''Ops'' in company A
=60100×1400=840
Male employees working in Admin in company
A = 18(1400-840) = 70
Male employees working in others in company A
= 78(1400-840) =490
Female employees working in Admin in company A
=24100×600=144
Female employees working in ''Ops'' in company
=58600-144=285
Female employees working in Others in company
A = 600-144-285 = 171
Male employees in company B
=80100×2800=2240
Female employees in company B = 2800 - 2240 = 560
Male employees working in ''Ops'' in company B = 65100 × 2240 = 1456
Male employee working in ''other1 in company B
=1210×490=588
Male employees working in ''Admin'' in company B
= 2240-1456-588 = 196
Female employees working in ''Ops'' in company B
=1456×25100=364
Female employees working in ''Admin'' in company B
=14560-364=49
Female employees working in Others in company B
=34560-364=147
For company A, Total = 2000

Required ratio =196+49:588+147=245 :735=1 :3


150. Number of employees in company
A = 512 × 4800 = 2000
Number of employees in company B
=712×4800=2800
Male employees in company A=70100×2000=1400
Female employees in company A = 2000 - 1400 = 600
Male employees working in ''Ops'' in company A
=60100×1400=840
Male employees working in Admin in company
A = 18(1400-840) = 70
Male employees working in others in company A
= 78(1400-840) =490
Female employees working in Admin in company A
=24100×600=144
Female employees working in ''Ops'' in company
=58600-144=285
Female employees working in Others in company
A = 600-144-285 = 171
Male employees in company B
=80100×2800=2240
Female employees in company B = 2800 - 2240 = 560
Male employees working in ''Ops'' in company B = 65100 × 2240 = 1456
Male employee working in ''other1 in company B
=1210×490=588
Male employees working in ''Admin'' in company B
= 2240-1456-588 = 196
Female employees working in ''Ops'' in company B
=1456×25100=364
Female employees working in ''Admin'' in company B
=14560-364=49
Female employees working in Others in company B
=34560-364=147
For company A, Total = 2000
Required no. of male = 840 + 1456 = 2296
Legal Edge 29 Mock

Answers & Details

ENGLISH

1. Option (a) falls under pinkwashing, as changing the colour of uniforms is an act of tokenism. The
company profits from the discounts offered during Women‘s History Month. Option (b) is wrong as
the entrepreneur presents herself as someone who cares for rights and equality for all and is instead
pushing a hateful agenda. Option (d) is wrong as asking for donations to help the queer community
within their organization is not an act that actually helps the community, especially one that is
targeting its customers. Option (c) is the correct option as it is not an act of pink washing and the
company made structural changes to make the space queer friendly.
2. All three reasons are mentioned in the passage. Therefore, D is the correct option.
3. Options (a) and (c) are not the correct choices as the author very expressly stated how tokenism
through the gay best friend character never provided visibility to the community and only led to a lot
of harmful stereotypes instead.
4. Option (a) is something the author might agree with but it is not what she is implying through the
quoted statement and hence is not the correct choice. Option (c) is not something the author would
agree with and hence wouldn‘t imply the same. Option (b) is the correct choice as the author states
the quoted sentence alongside other statements talking about how money-making is the motive
behind pink washing.
5. Option (b) is the correct option as allegiance means loyalty or commitment to a superior or to a
group or cause.
6. Option (b) is the correct answer as Complacency means self-satisfaction especially when
accompanied by unawareness of actual dangers or deficiencies When it comes to
safety, complacency can be dangerous
7. The passage explains that the colour pink and blue weren‘t gendered initially and were worn by
both boys and girls. In fact pink was treated as more masculine as it‘s closer to red which
represented passion and aggression. It is mentioned that it was in 1890s that the manufacturers
began their colour-coding and that Henry Huntington purchased those paintings in 1921. The status
of the colour pink now is such that men generally cannot be wearing pink without making a
statement or justifying their choice.
8. The gendering of the color pink was not solely accomplished by manufacturers but was a result of
a lot of factors. The passage mentions the manufacturers weren‘t even unanimous in their colour
coding. Hence, 1 is False. 2 is also false as the passae mentions red being considered as a
masculine colour and black being associated with death. 3 is true as pink is increasingly in use in
men‘s clothing according to the passage.
9. The passage follows a simple pattern of informing the reader of facts and opinions as stated by
experts from the field. Hence, the intention and tone is to inform. The author is not being incendiary
as the passage states facts and opinions in a plain manner and not an emotionally stirring one. The
tone is also not analytical as there are no comparisons made, no conclusions drawn. The tone
cannot be said to be ―self-aware‖ because the subject is not personal to the author.
10. It has been used in the context of men wearing pink as a resistance against prevalent gender
norms.
11. Option (b) is incorrect as it hasn‘t been discussed in the passage. Ralph Lauren and similar
brands are credited with popularizing pink among men, not otherwise. Option (c) is a blanket
statement and doesn‘t hold true for all men. Option (a) is the most appropriate choice as the
passage mentions this association as something recent. Despite palpable change in the attitude
towards the colour, social change of course takes time.
12. The correct option is (a), since in the context of passage, Ardent means something which is
glows or burns.
13. Option (b) is the correct choice as the passage mentions "Would wonders never cease. And they
didn‘t". Option (a) is not the correct choice as the author is marvelling at new ice cream flavours and
not sticking to the ones from his childhood. Option (c) is not discussed in the passage.
14. The author was not comparing the vaccine''s development to the development of ice cream
flavours, hence Option (a) is not wrong. Option (b) is not the appropriate choice as the health
benefits of haldi were mentioned to support turmeric''s recent popularity. Option (c) is the most
appropriate choice in light of the passage, as the author consistently speaks about unique new
flavours of ice cream.
15. Option (b) is the most appropriate choice as the author describes the increasing popularity of
turmeric in the culinary spheres and then goes on to say it was to be expected an ice cream flavour
would be made out of it. Option (c) cannot be inferred as the trend may or may not fade away after
an ice cream flavour is made out of it. Option (a) is not the most appropriate choice as the author
talks about it in context of recent trends. Moreover, ice cream is central to this statement rather than
turmeric.
16. Sarcasm: Sarcasm is the use of words usually used to either mock or annoy someone, or for
humorous purposes. Sarcasm may employ ambivalence, although it is not necessarily ironic. Most
noticeable in spoken word, sarcasm is mainly distinguished by the inflection with which it is spoken
and is largely context-dependent.
Assonance: Assonance is repetition of vowel sounds at the beginning, middle, or end of words in
close proximity to each other.
Oxymoron: a figure of speech in which apparently contradictory terms appear in conjunction
Alliteration: Alliteration is used to begin the start of several words in a line of text with the same
consonant sound.
17. Assortment means a miscellaneous collection of things or people. Thus, it is a proper meaning of
the expression mentioned in the question.
18. The passage explains clearly explains that NFTs are unique tokens attached to a digital work of
art that proves the work‘s authenticity. It gives ownership to the buyer, who gains the right to trade
the piece. The tokens are generated on Blockchain which makes it easily traceable and identifiable.
19. The passage explains that the major benefit of an NFT is that artists can sell their art as a
product rather than a service, which means that they can quote higher and better prices. They also
allow an artist to earn royalty on his or her art every time a work is sold.
20. The passage attempts to analyze the NFTs and their current impact on the art world as well
informing the reader of criticisms that it has faced. The passage seeks to give a descriptive view of
NFTs. The author is not being incendiary as the passage states facts and opinions in a plain manner
and not an emotionally stirring one. The tone is not merely informative as there is further analysis of
the subject rather than mere facts. The tone cannot be said to be ―sardonic‖ because it is not cynical
or mocking the subject matter.
21. The passage uses the word exclusivity to describe the unique and distinct nature of an NFT.
22. Option A and B haven‘t been discussed in the passage in detail. However, the passage does
draw its conclusions regarding why crypto-mining, the process of creating cryptocurrency that
involves a network of machines that release CO2, is controversial and a subject of environmental
debate. Therefore option C is the most accurate.
23. The passage mentions that Tom Cruise has joined Scientology but does not convey in any
manner that Scientology looks to recruit celebrities. Therefore, D is the correct answer.
24. Option B is true as the author explicitly mentions that the Character named Xenu can be widely
read online. Therefore, b is the correct answer.
25. If it is true that Senior Scientologists have not paid any money the Church of scientology then the
basis on which the author argument in the passage depends collapses and Scientology cannot be
considered a cult.
26. Option is a directly mentioned in the passage that he has donated money for the cause, which
does mean that he has donated money to the church of Scientology.
27. Espionage is the act of obtaining secret or confidential information or divulging of the same
without the permission of the holder of the information. It is also known as spying. Therefore, C is the
correct answer.
28. He was banned before the admission of guilty and not afterwards and therefore, a.is the correct
answer.
29. unceremoniously means with a lack of courtesy; roughly or abruptly. Therefore, d is the correct
answer:
30. The last passage is trying to take message from Armstrong‘s live for all the other athletes. The
message being that Athletes have a duty to act in responsible manner. Therefore, b is the correct
answer.
31. The correct option is c, which can clearly be made out from the below given line from the
passage:
“He was accused by USADA of running „the most sophisticated, professionalized and successful dop
ing program that professional cycling has ever seen‟, not only taking performance enhancing drug
s himself, but also putting pressure on teammates to do so.”
32. The word Rebuff means reject (someone or something) in an abrupt or ungracious manner.
Therefore, C is the correct answer.

GENERAL KNOWLEDGE

33. The government of India has announced the merger of the Lok Sabha TV and the Rajya Sabha
TV. The channels from now on will be known as Sansad TV.
34. The LSTV was the brainchild of Former Lok Sabha Speaker Somnath Chatterjee. He brought the
idea of a 24×7 Parliamentary Channel with the objective to familiarise citizens with the functioning of
Parliament.
35. The RSTV was launched in 2011. It will telecast the live proceedings of the Rajya Sabha. It also
brings analyses of parliamentary affairs and provides a platform for knowledge-based programmes.
The budget of the RSTV is bigger than that of the LSTV. The Union Budget allocates funds for the
running of channels. The RSTV also employs more people than the LSTV.
36. In 2019, a committee was set up under former Prasar Bharati Chairman Surya Prakash to look
into the merger of two channels. The merger is being done to avoid duplicity of efforts, resources.
Further, establishing Sansad TV will also reduce the financial burden.
37. The committee has suggested that a merged Sansad TV should have two separate channels—
Sansad 1 and Sansad 2.
One channel will telecast the live proceedings of the Lok Sabha. On the other hand, the other
channel will telecast the live proceedings of the Rajya Sabha.
When Parliament is not in session, the first channel can air Hindi content and the second channel
can air English content. This will result in better branding and increased viewership of Sansad TV.
38. The Suez Canal is pivotal in connecting Europe and Asia, as it negates the need to navigate
around the Cape of Good Hope in Africa and thus cutting distances by up to 7,000 km.
39. Located in Egypt, the artificial sea-level waterway was built between 1859 and 1869 linking the
Mediterranean Sea and the Red Sea.
40. The Suez along, with the Panama Canal (that links the Pacific and Atlantic oceans), are the
two most critical canals in the global maritime domain along with the Volga-Don and the Grand
Canal (China).
41. the Suez Canal, one of the world‘s most critical transit routes, got blocked due to a large
container vessel, the MV Ever Given. This has caused enormous disruption to global shipping.
42. Egyptian authorities have asked the owners of Ever Given to pay $1 billion in compensation or
else it wouldn‘t release the ship.
43. The bill is to replace the Religious Freedom Act, 1968. Once enforced the Religious Freedom Bill
will be the most stringent in the country.
44. If the victim is a woman, minor or belongs to Scheduled Caste or Scheduled Tribe community,
then the accused will be punishable with imprisonment ranging from two to ten years and fine of Rs
50,000.
45. According to the bill, those willing to convert will need to apply to the district administration 60
days in advance.
46. The bill provides for a prison term of up to 10 years and a fine of 1 lakh for conversion through
―marriage or by any other fraudulent means‖.
47. The Madhya Pradesh Freedom of Religion Ordinance, 2020 was promulgated on January 7,
2021.
48. India expressed its intent to include Chabahar port in the 13-nation International North South
Transport Corridor (INSTC) that extends from India to Russia, and to expand INSTC membership by
including Afghanistan and Uzbekistan, at a ―Chabahar Day‖ event organised as part of Maritime
India summit.
49. It is the only Iranian port with direct access to the Indian ocean and consists of two separate
ports named Shahid Beheshti and Shahid Kalantari.
50. Afghanistan, Iran and India signed a tripartite agreement on developing Chabahar port and
setting up a trilateral transport and transit corridor in 2016.
51. This corridor connects India Ocean and the Persian Gulf to the Caspian Sea via the Islamic
Republic of Iran and then is connected to St. Petersburg and North Europe via the Russian
Federation.
52. The INSTC was expanded to include eleven new members, namely: the Republic of Azerbaijan,
Republic of Armenia, Republic of Kazakhstan, Kyrgyz Republic, Republic of Tajikistan, Republic of
Turkey, Republic of Ukraine, Republic of Belarus, Oman, Syria, Bulgaria (Observer).
53. A disturbed area is declared by notification under Section 3 of AFSPA. It is invoked in places
where the use of armed forces is necessary to aid the civil power.
54. The Governor of the state or the central government can declare the whole or part of the state or
union territory as disturbed area.
55. NSCN-IM is a Naga Insurgent Group.
56. Once declared ‗disturbed‘, the region is maintained as disturbed for a period of three months
straight, according to The Disturbed Areas (Special Courts) Act, 1976.
57. The NSCN-IM is demanding for separate flag and constitution. This demand is not acceptable by
the central government. This leads to the continuation of insurgency in the region.
58. The number of Hong Kong‘s Legislative Council (HKLC) members will be increased to 90, with
the additional members also nominated, thereby reducing the share of elected representatives.
Currently, only half of the 70 members of HKLC are directly elected and the rest are nominated.
59. The change will give Beijing-appointed politicians a greater say in running the Hong Kong
Special Administrative Region (HKSAR), marking the biggest change since the handover in 1997.
An increased number of pro-Beijing officials would weaken the power of the opposition to influence
the city''s leadership.
60. Hong Kong is the fourth largest export market for India.
61. Sino-British Joint Declaration- According to the treaty, China would reassume control of Hong
Kong, which was occupied by Britain after the Opium War in 1840, from July 1, 1997. Opium Wars:
The Opium Wars were two wars fought between the Chinese Qing dynasty and European powers.
Both the wars were a result of the Qing Dynasty‘s attempts to curb the opium trade.
62. It stated that China''s basic policies regarding Hong Kong "will remain unchanged for 50 years"
and ensured a high degree of autonomy to Hong Kong. These policies are stipulated in the Basic
Law, the city‘s mini-constitution.
63. a five-judge bench at the Supreme Court has decided to examine whether the 1992 verdict by a
nine-judge bench capping quota at 50% (Indra Sawhney case) needed to be revisited in view of
subsequent constitutional amendments and changed social dynamics.
64. The court‘s observation came in a case where a reservation for Marathas in Maharashtra caused
a breach in the 50% ceiling. The Maharashtra government decided to grant 16% reservation of total
seats to the Marathas. However, it was reduced by the Bombay High Court to 12% in govt jobs and
13% in educational institutions. When the decision was challenged before the Supreme Court, it
stayed the former‘s order.
65. Article 335: It says that the claims of SCs and STs shall be taken into consideration constituently
with the maintenance of efficacy of the administration.
66. 102nd, 103rd and 104th Amendments: In the last couple of decades, there have been several
amendments to the constitution like the 102nd amendment, 104th
amendment. 10% reservation for EWS. was made by the 103rd amendment to the Constitution.
67. Indra Sawhney & Others vs Union of India, 1992: The judgement on Indra Sawhney case was
passed on 16 November, 1992. It was a nine-judge verdict which decisively laid down several
landmark propositions such as 50% threshold in reservations.
68. U.S., Australia, Canada, New Zealand and the UK, are the countries that make up the alliance.
69. The Five Eyes (FVEY) is the biggest intelligence alliance of the world.
70. Japan will be joining the alliance to restrict China‘s actions against Uyghurs.
71. The countries are signatories to the UKUSA Agreement for their cooperation in signal
intelligence.

LEGAL REASONING

72. As per the information given in the passage, ―whoever, having a husband or wife living, marries
in any case in which such marriage is void by reason of its taking place during the life of such
husband or wife, shall be punished‖.
73. As per legal information given in the passage, bigamy is not an offence under Muslim law. In the
instant case Salim Ali, who is Muslim by religion, can marry more than once during the lifetime of his
first spouse. Bigamy is not an offence under Muslim law.
74. (b) The passage clearly states that bigamy is a punishable offence and the explanation mentions
that undertaking a second marriage during the lifetime of the first wife is bigamy, no matter the
reason. Since Maya was alive at the time of Atimabh''s second marriage, his marriage to Meekha is
void.
75. (c) It‘s a clear information that marriages are governed as per personal laws and only Muslim
men are allowed to marry more than once, therefore Sourabh would be held liable. Getting married
in Ohio has no bearing in the present case.
76. (a) It is apparent from the facts that Rakhi married Mika while Daniel was alive and hence as per
the information given in the passage Rakhi is guilty of bigamy.
77. The correct answer is option (c). The contract was signed in 2016 when the courts had wide
discretionary powers in choosing whether or not to direct/enforce specific performance of a contract.
Therefore, in this case the court is not "bound" to direct B to enforce the contract as it may exercise
its discretion to order otherwise.
78. The correct answer is option (b). The contract was signed in 2016, prior to the 2018 amendment
to the Specific Relief Act. Hence, under Section 10 "the specific performance of any contract may, in
the discretion of the court, be enforced" since the legislative intention at the time of the enactment of
the Act was to give full judicial discretion to the courts to direct/enforce or not to direct/enforce the
specific performance of a contract. Therefore, the court is not "bound to give an order in A''s favour"
only and may order otherwise.
79. The correct answer is option (a). The passage clearly provides that "the real significance and
impact of the amendment stems from the fact that it has made the specific performance of contracts
the (general) rule rather than an optional remedy". Option (b) is incorrect because it provides the
change in legislative attitude instead of the reason for why Chota can "undoubtedly seek specific
performance" of the contract. Option (c) is incorrect because it says that the amended Act "doesn''t"
significantly curtail the power of the court in this regard. Mentioning the provisions in the question,
especially if they do not concern the contract, is irrelevant in this regard because they don''t affect
the discretion of the court.
80. The correct answer is option (a). The passage clearly mentions that "the specific performance of
a contract shall be enforced by the court subject to the "applicability" of the provisions contained in
Section 11(2), Section 14 and Section 16 of the Act." Hence, if the provisions are applicable, then
the contract is enforceable and Nobita can seek specific performance.
81. The correct answer is option (b). There was an ambiguity in the contract because it does not
define what "premium" is. The contract was silent on whether the term "premium" is inclusive or
exclusive of pulp in the juice. As a result, contra proferentem will be applied against the seller who
drafted the contract i.e. Harry and the ambiguity in the contract will be interpreted against him.
Therefore, Mitsi has a better position to get a remedy in her favour.
82. The passage mentions that damages must also be paid for the mental agony that ensues from
such breach of contract. Thus, there should be a 1,00,000 rupee payment to Somashekhar.
83. The contract was for only construction of a strong wall. It could not have been foreseen that the
child would dig a hole and escape and injure. The damage occurred was too remote and it cannot be
said that breach was causation of such damagebecause he ensured due diligence..
84. The contract was to construct a wall that prevented Hari‘s child from going out. Yet the same
was no done. Krishan will be liable for the breach for the breach has directly caused the injury.
85. Even though Manjappa had committed the breach by failing to build a strong wall, the damage
also occurred partly due to Rakesh‘s failure to water the wall continuously for 10 days which would
have resulted in a stronger bond. Manjappa can only be partly liable.
86. The passage states that in the event there is a clause in the contract stipulating the amount of
damages to be paid then such amount shall be the final amount.
87. The correct answer is (b). The intention to cause disorder or incite people to violence is the sine
qua non of the offence under Section 153 A IPC. Sushmita had no intention to cause disorder or
violence and connecting homosexuality with the ancient culture of the country is not a villifying
remark per se. Therefore, she shall not be held guilty of the offence.
88. The correct answer is (d). One cannot rely on strongly worded and isolated passages for proving
the charge nor indeed can one take a sentence here and a sentence there and connect them by a
meticulous process of inferential reasoning. The intention of Hema was to highlight that many
religious practices led to animal adversity and no intention to insult or spread hatred against any
particular religious group.
89. The correct answer is (a). The third test was for Courts to focus their analysis on the effect of the
expression at issue, namely, whether it is likely to expose the targeted person or group to hatred by
others. In the present case, strong words by Raman exposed the entire Pudali community to hatred
across the state. Therefore, as per the third test Raman shall be convicted of the offence.
90. The correct answer is (c). One cannot rely on strongly worded and isolated passages for proving
the charge nor indeed can one take a sentence here and a sentence there and connect them by a
meticulous process of inferential reasoning. In the present case, Rujhita actually highlighted the
judgments which improved women‘s social condition which was the objective of the entire book, her
intention cannot be deduced from separate fragmentations picked up from different parts of the
book.
91. The correct answer is (b). In both the stated cases, a certain group has been vilified by a strong
manifestation of emotions.
92. refugee is a person targeted based on their identities, in this case phizo was targeted by the
government because of his criminal activities, he cannot qualify as a refugee
93. the constitution allows right to life to the all within the territory of India. The deportation would be
wrong based on the principles of the constitution as well as international law.
94. anyone giving shelter to a foreigner has to report him or her to the police within 24 hours under
Foreigners Act 1946 and this was not done by Sajid, hence he violated the law and is liable.
95. their detention is not illegal in nature, procedure of law was followed.
96. Abel is accused of war crimes and should be tried according to the procedure established by
law. Article 21 cannot be applied as it is also not absolute in nature. His status as a refugee cannot
be a defence for his crimes.
97. (b) Refer to the last passage of Passage. (c) Incomplete answer. (d) J and A comes under the
ambit of the act.
98. The passage talks about who will come under the ambit of act or not and wits and wisdom
doesn‘t come under the ambit of the act. Therefore, this will be the only correct answer.
99. (d) According to the passage registered society comes under the ambit of act and society
mentioned in the question is not registered.
100. (b) Appropriate course of action will be option 2 and option 1 doesn‘t mention the proper
procedure to be followed according to passage. Therefore option 2 will be the correct answer.(c)
Exaggerating
101. (c) Passage talks about subject to change in Act through amendment therefore due to
amendment Concise Pvt Limited is not liable for the violation of the law.
102. option (a) talks about abnormality to women but passage 2nd talks about reduce maternal
mortality and morbidity therefore this will be incorrect answer. Option (b) nowhere in passage
medical negligence is mentioned as a reason for amendment. Option C is the correct reason for
amendment in MPT ACT 1971 because 2nd passage mentions all these points.
103. Not untrue means true. option (a) can be inferred from the 6th line of the last passage that
talks about the privacy of women. Option (b) is the wrong answer because MTP Bill 2020 talks about
the opinion of 1 Dr in 20 weeks pregnancy and the opinion of two Dr will be required for termination
of pregnancy of 20 to 24 weeks. Option(c) is correct as 20 weeks is tantamount to 5 months.
104. Exception is there in a passage that says except to a person authorized in any law which is in
force therefore all other options became incorrect and option A is only correct.
105. This is the confusing question in which facts are given in accordance of MTP Act 1971 b
ut question ask legally applicability of MTP bill 2020.
Option (a) will be incorrect because according to MTP bill 2020 opinion of Two Doctors is mandatory
for termination of pregnancy up to 24 weeks. Option (d) is correct because two doctorsopinion is
mention. Option(c) is incorrect because the question asks about MTP bill 2020 as the date mention
is of the year 2021. And option gives an explanation for MTP Act 1971.
106. According to the opinion of the Doctor in question, the termination of pregnancy will not be
allowed because conditions are not fulfilled for termination of pregnancy as per the MTP BILL 2020
in passage.
LOGICAL REASONING

107. (a) Refer to, "With well more than 80% of its oil request being met through imports, India
unmistakably has a great deal in question as oil costs have ascended by 70% in rupee terms over
the most recent one year." It is obvious from the above lines that India is worried since it imports
most of its oil shares from different nations. That is the reason it is important to the country that the
costs in the global market are going up or down. Among the given choices, just alternative an is
there that consummately clarifies the explanation of the worry appeared by India in regards to the
costs of the oil in the worldwide market. Different choices are either outside any connection to the
subject at hand or they are not right as per the data given in the passage. This settles on alternative
‗A‘ as the right choice among the given options.
108. (d) Refer to, "Another drawn out answer for the oil issue will be to progressively take advantage
of home-grown wellsprings of energy supply while at the same time urging purchasers to change to
other green options. This will require a more grounded strategy system and execution. It is obvious
from the above lines that it is needed with respect to the public authority that home-grown sources
ought to be tapped to deliver more energy in the country. There are additionally different choices
given, for example, making individuals utilize green wellsprings of energy or eliminating the focal and
the state charges to cut down the costs of oil. This settles on choice d the right choice among the
given options
109. (a) Refer to, "Talking at a similar occasion, Saudi Bedouin Energy Pastor Khalid (a) Al-Falih
declined to straightforwardly resolve to bring down oil costs, picking rather to say that the cost of oil
might have been a lot higher however for the endeavours taken by his nation to help supply." It is
obvious from the above lines that Saudi Arabia will do nothing more to cut down the costs of oil in
the global market while it is additionally said that on account of Saudi Arabia just, the costs are not
higher further in the worldwide market. Among the given choices, choice b isn''t right since it isn''t
there in the passage though choices c and d can likewise be precluded because of the very actuality
that these are additionally insignificant with regards to the passage. Just choice a is there that
clarifies appropriately the assessment of the Oil Clergyman of Saudi Arabia. This settles on choice a
the right choice among the given alternatives.
110. (c) Refer to, "for the time being, the public authority could hope to broadening its global provider
base to oversee stuns better. Yet, a particular choice conveys international dangers, for example, on
account of Iran." It is obvious from the above lines that the public authority ought to broaden its
provider base from the worldwide local area so it doesn''t need to stress over the stock of oil to India
whenever. Among the given choices, others are outside the current discussion and not referenced in
the passage likewise with the exception of alternative (c) Alternative c effectively clarifies that India
ought to remember more nations for the global provider base and that would mean no lack of
choices before the country.
111. (d) Refer to, "India''s monetary fortunes keep on being attached to the pointedly fluctuating cost
of oil. At a social affair of unmistakable oil clergymen in New Delhi on Monday, Executive Narendra
Modi asked oil-creating nations to diminish the expense of energy to help the worldwide economy in
its way towards recuperation. Mr. Modi likewise required a survey of instalment terms, requesting the
halfway utilization of the rupee rather than the U.S. dollar to pay for oil, to facilitate the weight on oil-
bringing in nations in the wake of the fortifying of the dollar." Presently, the facts confirm that Indian
Head administrator has mentioned to ensure that the expense of energy descends on the planet to
make it simpler for worldwide financial recuperation. He has additionally asked to modify the
instalment terms with the goal that a portion of the instalment could be made in Indian money as it
were. This settles on choice ‗D‘ as the right choice among the given choices.
112. (a) ''Since 5G move outs are likely even before 4G is broadly conveyed, the received guide for
5G ought to guarantee that the current and not so distant future interests in 4G can be utilized. One
proposed approach has been that since administrators are probably going to convey 5G in groups
that they are right now utilizing for 4G, strategy ought to encourage this by making range utilization a
rationalist help. The job of wired framework, particularly fibre optic links, is basic and correlative to
development of remote. This is on the grounds that enormous limits are needed in the backhaul to
take into account applications empowered through 5G. This would involve quickening the Bharat Net
program for sending fibre optic links to gram panchayats and expanding the contribution of the
private area to abuse complementarities and efficiencies of the private area'' Explanation III is bogus
as the section discusses arrangement of wired foundation. Articulations I and II are right as seen
from the featured parts above. Thus, alternative A is right.
113. (c) Refer to: ''This juggernaut of truly developing ages of innovation and approaching 5G
require a steady arrangement and administrative climate. Without it, the area''s wellbeing and India''s
monetary intensity would be significantly hindered.'' Proclamation III is true.''5G likewise guarantees
super solid, exceptionally quick velocities and high data transfer capacity portable availability and
supports huge interconnected gadgets spread across wide regions. 5G gives top velocities of
multiple times, contrasted and 4G, like that needed in computer generated experience applications.''
Proclamations I and II are valid. None of the assertions are erroneous. Thus, choice c is right.
114. (c) Refer to: ''… They have undeniably less range in contrast with worldwide administrators….
A large number of them are additionally burdened by obligation. At any point quicker adjusts of new
innovation presentation when earlier innovation ventures have not been recovered add further
intricacy. This juggernaut of truly developing ages of innovation and impending 5G require a strong
arrangement and administrative climate. Without it, the area''s wellbeing and India''s monetary
seriousness would be extraordinarily impaired..'' according to the featured parts, explanations III and
I are mistaken. The amount of range is less while nothing about quality has been referenced.
Essentially, there is no notice of absence of obligation accessibility. Explanation II is right. Hence,
choice C is right.
115. (d) Contention III is frail as India should zero in on its own abilities and needs and not
indiscriminately adjust to what different countries appear to do. Contention I is solid. Paragraph 1
shows that ''telecom administrators in India are amidst turning out 4G organizations'' and
subsequently need additional time. Contention II is frail. There is no notice of other countries''
advancement timetable in 5G innovation comparative with India and thus we can''t contrast our
availability with dispatch such items with theirs.
Subsequently, choice d is right
116. (d) Refer to: ''5G gives top velocities of multiple times, contrasted and 4G, like that needed in
augmented simulation applications. It is additionally intended to give idleness at multiple times lower
than 4G. Low dormancy is needed in applications managing basic crisis medical services,
independent vehicles or debacle the board‘.
Subsequently, choice D is right.

117.
118.

119. (b) Option (B) is wrong because the author never means to imply that human immune system
will evolve to resist new diseases AT THE SAME PACE AS pathogens evolve to overcome our
defences. This is, in fact, opposite of what the author believes. The author says that we will evolve to
resist only some diseases and for the rest we will have to depend on medicines. If we evolve at the
same pace then the medicines are not even required!
―But as long as doctors keep having a way to render moot those diseases that used to kill us, natural
immunity isn''t essential.‖ Therefore, Option (A) can be inferred.
―We have all these ways of intervening when people get sick, when otherwise they would have died
and we would see some natural selection for people with more robust immune systems.‖ Had there
been no intervention to stop diseases, people with more robust immune systems would be the ones
to survive. Therefore, the human immune system, on average, would have been more robust than
what it is with medicines and healthcare. Option (C) can also be, therefore, inferred.
―For eons, this war has weeded out the weak, and in a less combative environment, mechanisms to
sustain human survival would have grown lax.‖ Had it not been an intense and long battle (against
pathogens), the immune system would have grown lax, thereby more susceptible to failure. Option
(D) can also be, therefore, inferred.
Hence, B
120. (a) The answers to the questions in options (B), (C) and (D) are not there in the passage. The
answer to option (A) is not only present in the passage but is also the main point of the author‗s
argument.
Hence, A
121. (b) Option (A) strengthens the author‗s argument as the author states that people would either
die or evolve from diseases based on how robust their immune systems are.
Option (C) has no effect on the author‗s claim about the human immune system without medical
intervention. The option talks about an effect of heightened immunity. So, it is irrelevant. ―We have
all these ways of intervening when people get sick, when otherwise they would have died and we
would see some natural selection for people with more robust immune systems.‖ Had there been no
intervention to stop diseases, people with more robust immune systems would be the ones to
survive. Therefore, it can be inferred that there is loss of evolutionary robustness with the
intervention of medicines. The author also makes sweeping statements such as humans are kind of
artificial animals and natural immunity is no longer required in this regard. Option (B) attacks these
claims of the author. Therefore, it is the correct answer.
122. (b) The claim is surely the conclusion of the author‗s arguments. Option (C) is wrong because it
is not the premise. It is the conclusion for which support (premises) are provided in the paragraph.
The remaining options identify it as conclusion but cite different premises so we will examine them
one by one: Option (A) is wrong as the author not only describes microorganisms as being too
numerous but also potent enough to evolve (which is a very important factor in the author‗s
argument) in response to the medicines. Option (B) is correct. ―Most diseases are caused by very
potent and numerous microorganisms whose response to medicines is that they quickly evolve
immunities to those medicines while maintaining their power to infect and even kill humans.‖ Hence,
the conclusion that the chances of a disease–free world are bleak. Option (D): ‗unduly‘ reliance is
incorrect.
Hence, B
123. (a) The questions asks about the disease that was taken by the doctor‗s intervention. Option
(A) is clearly an example of such a disease.
Option (B) talks about decline of TB by introducing a process to stop its transmission from the animal
to human. It didn‗t require the doctor‗s intervention.
Therefore, only Option (A) is correct.
124. (b) A is furthering the statement, it is not reflecting the underlying assumption behind coming up
with persuading the voter. Option II is therefore the correct answer as it captures why political parties
come up with people pleasing schemes.
Hence, B is the right answer.
125. (c) Option C forms a connection between regularly giving complimentary snacks and higher
billing. In the experimental stage, a few customers were ―surprised‖ with complimentary snacks. The
assumption is that customer will order more food even if the complimentary snack is a regular
feature and not a surprise.
Hence, C
126. (b) Assertion is true. Birds need gravity to swallow. This is the reason that birds lift their beaks
when eating or drinking. However, they do not need gravity to lift their beaks, rather, they have to
work against the gravity to do so.
Hence, B
127. (b) Aishwarya was a famous Bollywood celebrity even before her marriage. Hence, Reason is
wrong.
Therefore, correct choice is B
128. (b) A particular sport being hugely popular in a few countries is no reason for its inclusion in
Olympics which invites participation from almost all the countries. Thus, I is weak. If Cricket is played
seriously by only a handful of countries, it‘ll not generate interest as generated by other, more
conventional games of the Olympics. Thus, II holds strong.
Hence, B
129. (a) In the above conversation Yogesh argues in favor of the UPA government. Refer to ―the
economy‘s been going steadily downhill since the UPA‘s golden era. The UPA-2 left behind a
quarterly GDP growth of 8 per cent and the NDA has successfully levelled it to 5 per cent.‖
Therefore, option (A) is the correct answer.
130. (b) In the above conversation Sanju argues in favor of the NDA government. Refer to ―The
UPA-2‘s economic policies are the root of all our current troubles‖
Therefore, option (B) is the correct answer.
131. (a) In the above conversation, Sanju argues that the policies of previous government are
responsible for the slowdown in economy. Option (B) and (C) are further weakening the argument of
Sanju. Option (A) is the correct answer, because it is further strengthening the argument of Sanju.
Hence, option (A) is the correct answer.
132. (c) Rationale- In the second paragraph of the above conversation, Sanju states that ―The UPA-
2‘s economic policies are the root of all our current troubles‖ and all his other arguments are either
premise to his conclusion or providing evidence to his conclusion. Therefore, option (C) is the correct
answer.
133. Q > P(70) >S > T > U > R
(a) Is the correct answer.
134. Q > P(70) >S > T > U > R
(d) Is the correct answer.

QUANTITATIVE TECHNIQUES

135.

136.

137.

138.
139.

140. (c) Amount of tank filled by tap P in 60 min = 30% of 400 = 120 liter
Amount of tank filled by tap S in 60 min = 20% of 400 = 80 liter
Amount of tank filled by tap P and S = 120 + 80 = 200 liter
Time taken to fill 400 liter = 400/200 = 2 hours = 120 min
141. (c) Amount of water fill by tap Q in 60 min = 25% of 400 = 100 liter
Flow rate of tap Q = 100/60 = 5/3 liter/m
Amount of tank filled in 21 min = (5/3) × 21 = 35 liter
For the flow rate of tap R,
Amount of water fill by tap R in 60 min = 25% of 400 = 100 liter
Flow rate of tap R = 100/60 = 5/3 liter/m
Amount of water filled in 18 min = (5/3) × 18 = 30 liter
For the flow rate of tap P,
Amount of water filled by tap P = 30% of 400 = 120 liter
Flow rate of tap P = 120/60 = 2 liter/m
Amount empty by tap P in 10 min = 10 × 2 = 20 liter
Final volume in tank = 35 + 30 - 20 = 45 liter
142. (b) For the flow rate of Q and R,
Amount of water empty by tap Q = Amount of water empty by tap R = 25% of 400 = 100 liter
Flow rate of Q = flow rate of R = 100/60 = 5/3 liter/m
Combine flow rate = 5/3 + 5/3 = 10/3 liter/m
For the time taken to empty the tank,
Time taken to empty 400 liter = 400 × 3/10 = 120 min. = 2 hour
Time when tank empty = 10:00 + 2:00 = 12:00 AM
143. (c) Water in tank = (3/5) × 400 = 240 liter
Water in tank after closing tap S = 240 - 30 = 210 liter
Amount need to fill = 400 - 210 = 190 liter
For the flow rate of Q,
Amount of tank filled in 60 min = 25% of 400 = 100 liter
Flow rate of tap Q = 100/60 = 5/3 liter/m
Time taken to fill the tank = 190 × (3/5) = 114 min = 1 hour 54 min
144. (d) P fills 120 liter in 60 minutes
R fills 100 liter in 60 minutes
Q empty 100 liter in 60 minutes
S empty 80 liter in 60 minutes
Total 40 liter is filled using all four pipes in 60 minutes
Time to fill 200 liters = 200/40 = 5 hours
145.

P1 = 56% of 200 = 112


P2 = 63% of 200 = 126
P3 = 56.5% of 200 = 113
P1 + P2 = 11% of 200 = 22
P1 + P3 = 8% of 200 = 16
Pl + P2 + P3 = 22% of 200 = 44
x + 22 + 16 + 44 = 112
or, x = 112 - 82 = 30
22 + 44 + y + k = 126
or, y + k = 60 ... (1)
z + k + 16 + 44 = 113
or, z + k = 53 ... (2)
30 + 22 + y + 16 + 44 + k + z = 200
or, y + z + k = 88 ... (3)
From eqn (1), (2) and (3),
k = 25, y = 35, z = 28
(c)Passed in paper P2 and P3 but failed in P1 = 25

146.

P1 = 56% of 200 = 112


P2 = 63% of 200 = 126
P3 = 56.5% of 200 = 113
P1 + P2 = 11% of 200 = 22
P1 + P3 = 8% of 200 = 16
Pl + P2 + P3 = 22% of 200 = 44
x + 22 + 16 + 44 = 112
or, x = 112 - 82 = 30
22 + 44 + y + k = 126
or, y + k = 60 ... (1)
z + k + 16 + 44 = 113
or, z + k = 53 ... (2)
30 + 22 + y + 16 + 44 + k + z = 200
or, y + z + k = 88 ... (3)
From eqn (1), (2) and (3),
k = 25, y = 35, z = 28
(c)Only P2 = 35 and only P3 = 28
Ratio = 3528= 54
147.

P1 = 56% of 200 = 112


P2 = 63% of 200 = 126
P3 = 56.5% of 200 = 113
P1 + P2 = 11% of 200 = 22
P1 + P3 = 8% of 200 = 16
Pl + P2 + P3 = 22% of 200 = 44
x + 22 + 16 + 44 = 112
or, x = 112 - 82 = 30
22 + 44 + y + k = 126
or, y + k = 60 ... (1)
z + k + 16 + 44 = 113
or, z + k = 53 ... (2)
30 + 22 + y + 16 + 44 + k + z = 200
or, y + z + k = 88 ... (3)
From eqn (1), (2) and (3),
k = 25, y = 35, z = 28
(a)Only P1 = 30 Only P1 + P3 = 16
Required percentage = 3016 × 100 = 187.5 %
148.

P1 = 56% of 200 = 112


P2 = 63% of 200 = 126
P3 = 56.5% of 200 = 113
P1 + P2 = 11% of 200 = 22
P1 + P3 = 8% of 200 = 16
Pl + P2 + P3 = 22% of 200 = 44
x + 22 + 16 + 44 = 112
or, x = 112 - 82 = 30
22 + 44 + y + k = 126
or, y + k = 60 ... (1)
z + k + 16 + 44 = 113
or, z + k = 53 ... (2)
30 + 22 + y + 16 + 44 + k + z = 200
or, y + z + k = 88 ... (3)
From eqn (1), (2) and (3),
k = 25, y = 35, z = 28
(d)At most one paper = 30 + 35 + 28 = 93
Required percentage = 93200 × 100 = 46.5 %
149.

P1 = 56% of 200 = 112


P2 = 63% of 200 = 126
P3 = 56.5% of 200 = 113
P1 + P2 = 11% of 200 = 22
P1 + P3 = 8% of 200 = 16
Pl + P2 + P3 = 22% of 200 = 44
x + 22 + 16 + 44 = 112
or, x = 112 - 82 = 30
22 + 44 + y + k = 126
or, y + k = 60 ... (1)
z + k + 16 + 44 = 113
or, z + k = 53 ... (2)
30 + 22 + y + 16 + 44 + k + z = 200
or, y + z + k = 88 ... (3)
From eqn (1), (2) and (3),
k = 25, y = 35, z = 28
(c)P3 = 16 + 44 + 25 + 28 = 113
Only P3 = 28
Difference = 113 - 28 = 85
150.

P1 = 56% of 200 = 112


P2 = 63% of 200 = 126
P3 = 56.5% of 200 = 113
P1 + P2 = 11% of 200 = 22
P1 + P3 = 8% of 200 = 16
Pl + P2 + P3 = 22% of 200 = 44
x + 22 + 16 + 44 = 112
or, x = 112 - 82 = 30
22 + 44 + y + k = 126
or, y + k = 60 ... (1)
z + k + 16 + 44 = 113
or, z + k = 53 ... (2)
30 + 22 + y + 16 + 44 + k + z = 200
or, y + z + k = 88 ... (3)
From eqn (1), (2) and (3),
k = 25, y = 35, z = 28
(d)Passed in exactly two papers = 16 + 22 + 25 = 63
Required percentage = 63200 × 100 = 31.5 %
Legal Edge 30 Mock

Answers & Details

ENGLISH

1. vintage and luxurious

The author here has clearly explained the hotel in a manner that the look and feel of the hotel. It must
be the owners’ attempt at giving their hotel a veneer of luxury, a touch of the dignity of age clearly
shows the author’s intentions. Therefore, the correct answer is b.

2. The author in the passage clearly says that he usually sleeps every time he has to wait but that
particular day he was unable to sleep even though he was trying to sleep. This is the tyranny author is
talking about here. Hence, the correct option is d and not other.

3. quickly take hold of something

phrase ‘ready to pounce’ means to jump or move quickly in order to catch or take hold of something.
Therefore, the closest and most appropriate meaning is quickly taking hold of something and hence
option b, c and d are incorrect.

4. nostalgia

correct answer is c because nostalgia is a feeling missing/remembering something in the past. Grief
means feeling of loss, Deja vu means feeling something that one has never experienced and anxious is
worrying about future. Hence, the closest term is nostalgia.

5. the author here has compared directory to the Bible trying to imply that people in today’s world are
forgetting about their values and holy books and same is the condition of the directory being kept in the
hotel room. Hence, the correct option is c. Therefore options a, b and d are incorrect.

6. conserving forests.

The author clearly talks about the importance of forests and why the same should be protected and
conserved. Option A and C can be considered the sub-topics for the passage but the main idea is the
need to protect the forests.

7. Measures should be taken immediately.

Author by the phrase implies that considering the current situation, measures should be taken without a
delay i.e. immediately and hence option A and B are incorrect. The phrase does not necessarily mean
same day, nor does it mean that measure can be taken anytime in future, it talks about the urgency of
situation.
8. Both A and R are true and R is not the correct explanation of A.

The correct reason for the above assertion is the fact that trees provide us with oxygen and food which
are our lifelines to survive.

9. both a and b

The passage clearly says that the need of the hour is to work to save existing forests and help in bringing
larger area under forests. And hence the other options are not the correct answers.

10. As per the passage under DDP, forests were sought to be promoted not only to restore fragile
ecological balance but also to provide people with the means of livelihood. The other statements are
correct and hence the correct answer is b.

11. Downpour

The downpour had started while we were changing, and we found ourselves gathering on the veranda –
which was sheltered by the pavilion roof – while we waited for it to stop. Options b and c were not used
for rain.

12. It is the most appropriate answer because she really was focussing on Lucy’s posture, her way of
looking at rain, etc. Options a and b are incorrect because the same cannot be inferred from the
passage.

13. to quickly look at someone else

The passage says that - I was watching her as carefully as ever in those days, and even as I was laughing
at Laura, I was stealing glances at Miss Lucy’s back. Clearly she was trying to look at something other
than Laura.

14. The unusual thing is the way she was looking at the raindrops so carefully and closely. The other two
options are parts of the way in which was looking at rain.

15. moving without any purpose

The phrase has been used in order to make the reader understand that as soon as the rain started
everyone went veranda and were moving here there with no particular motive. Hence, the other two
options are incorrect.

16. Mass media and communication is the reason for cultural exchange.

It clearly means that media and communication is the reason behind the growth rate and the cultural
mix ups and the same has been brought to India by the Britishers and hence the other options are
incorrect.

17. A is true but R is false


The reason given above is incorrect as the first press appeared in Bombay was on the initiative of an
Indian named Bhimji Parekh. Therefore, the correct option is c and not others.

18. unaffected

Unaffected is the most appropriate answer, as in the above sentence the author is trying to convey that
India was not influenced by modernity till the time colonization happened in India and therefore,
unaffected is the most appropriate term.

19. momentum

The word closest to the above is momentum as gaining momentum means to start/develop something
quickly. Hence, the other words are not thee correct answers.

20. process of modernization.

There was, however, one important feature of Indian modernization during the British period. The
growth of this process was selective and segmental. It was not integrated with the microstructure of
Indian society, such as family, caste, village community and hence the Indian people were selective in
modernization. Therefore the other options are incorrect.

21. Option c is the correct answer as while telling the reader about Ambedkar, the author states that for
Ambedkar, democracy was “a form and a method of government whereby revolutionary changes in the
economic and social life of the people are brought about without bloodshed”.

22. Ambedkar believed that the democracy is more than a form of Government. It is primarily a mode of
associated living. Which means that the lives of people are shared, it is also reflected in the passage
where author says, Ambedkar put equality and fraternity at the heart of democracy.

23. The passage directly provided the answer for this question in the following lines:

The “associated living” that democracy presupposes simply did not exist in India. Caste system has
divided the Indian society into many parallel, self-contained communities that did not allow for the
conversation and negotiation necessary for a healthy democracy. Thus Ambedkar’s critique of the caste
system was not merely that it was unjust and oppressive for the “depressed classes”, but also that it
fractured national unity and made democracy impossible.

24. In the context of the passage, fraternity means feeling of closeness or a sense of belonging with the
country. Therefore, c is the correct answer.

25. All the above are the correct answer. It can be derived from the last paragraph of the passage.
Ambedkar mentioned that there should not be glaring inequality in society, there should be moral order
and respect for the opposition. Therefore, d is the correct answer.

26. Antithetical means directly opposed or contrasted; mutually incompatible. B is the correct answer.
27. Based on this information, we can conclude that Hooker became so confident that Lee would retreat
and that the Union would win, he believed he could not lose. As a result of this, Hooker’s soldiers were
caught “completely unprepared for battle” when Lee’s army attacked, so Hooker lost, despite having
more men and an advantageous position. This tells us that Hooker was too confident. As a result of his
belief that he could not lose, Hooker failed to prepare his soldiers for battle, and so they lost. This
means that Hooker lost the Battle of Chancellorsville mostly because of his overconfidence. Therefore
(C) is correct.

28. Propensity is a noun which indicates and inclination or natural tendency to behave a certain way.

29. Buoy as a verb means to raise the spirits of; hearten; bolster or support.

30. In paragraph 4, the author writes that Hooker “had discerned...that Lee had only 61,000 men to
Hooker’s own 134,000” and that Hooker was “buoyed by his superior numbers.” Since Hooker had over
twice as many men as Lee, this tells us that Hooker’s advantages going into battle included numbers.
This supports option (I). In paragraph 4, we learn that “Hooker covertly moved 70,000 of his men fifteen
miles up and across the river, and then ordered them to sneak back down to position themselves behind
Lee’s army. In effect, Hooker had cut off the Confederate soldiers in front and behind. They were
trapped.” The author also tells us that once Hooker had moved his men into this position, he was
“satisfied with his advantage.” This means that Hooker’s advantages going into battle included position.
This supports option (II). No Support is shown for option III are therefore, c is the correct answer.

31. To understand why the author most likely includes this quote, we need to look at the context of the
quote. In the beginning of paragraph 3, the author notes that Hooker says his plans are perfect in spite
of the fact that his troops are “famished, exhausted, and demoralized.” This means that Hooker is
confident that his plans will be successful even though his army is very weak. In the final sentence of
paragraph 3, the author asks, “Why, aside from a propensity for narcissism, was Hooker so confident?”
This tells us that the quote is intended to show how confident Hooker was in his belief that he would
win. This means the author most likely includes this quote in order to demonstrate Hooker’s belief in his
own infallibility. Therefore (a) is correct

32. Demeanour means the way a person behaves toward other people. Impropriety is opposite as it
means improper behaviour.
GENERAL KNOWLEDGE

33. The New Draft Resolution/The Zero Draft:

It incorporates some of the elements of the UNHRC report, including those on strengthening the HRC’s
capacity on preserving evidence, devising strategies for future accountability processes, and supporting
judicial proceedings in member states with jurisdiction.

As per the UNHRC report, the government of Sri Lanka had created parallel military task forces and
commissions that encroach on civilian functions, and reversed important institutional checks and
balances, threatening democratic gains, the independence of the judiciary and other key institutions.

It also talks about encouraging the Sri Lankan government to implement the requirements of the
previous 30/1 resolution (from which it pulled out).

UNHRC was reconstituted from its predecessor organisation, the UN Commission on Human Rights to
help overcome the “credibility deficit” of the previous organisation.

Headquartered in Geneva, Switzerland.

Composition:

>The UNHRC has 47 members serving at any time with elections held to fill up seats every year, based
on allocations to regions across the world to ensure geographical representation.

>Each elected member serves for a term of three years.

>Countries are disallowed from occupying a seat for more than two consecutive terms.

34. The New Draft Resolution/The Zero Draft:

It incorporates some of the elements of the UNHRC report, including those on strengthening the HRC’s
capacity on preserving evidence, devising strategies for future accountability processes, and supporting
judicial proceedings in member states with jurisdiction.

As per the UNHRC report, the government of Sri Lanka had created parallel military task forces and
commissions that encroach on civilian functions, and reversed important institutional checks and
balances, threatening democratic gains, the independence of the judiciary and other key institutions.

It also talks about encouraging the Sri Lankan government to implement the requirements of the
previous 30/1 resolution (from which it pulled out).

UNHRC was reconstituted from its predecessor organisation, the UN Commission on Human Rights to
help overcome the “credibility deficit” of the previous organisation.

Headquartered in Geneva, Switzerland.


Composition:

>The UNHRC has 47 members serving at any time with elections held to fill up seats every year, based
on allocations to regions across the world to ensure geographical representation.

>Each elected member serves for a term of three years.

>Countries are disallowed from occupying a seat for more than two consecutive terms.

35. The New Draft Resolution/The Zero Draft:

It incorporates some of the elements of the UNHRC report, including those on strengthening the HRC’s
capacity on preserving evidence, devising strategies for future accountability processes, and supporting
judicial proceedings in member states with jurisdiction.

As per the UNHRC report, the government of Sri Lanka had created parallel military task forces and
commissions that encroach on civilian functions, and reversed important institutional checks and
balances, threatening democratic gains, the independence of the judiciary and other key institutions.

It also talks about encouraging the Sri Lankan government to implement the requirements of the
previous 30/1 resolution (from which it pulled out).

UNHRC was reconstituted from its predecessor organisation, the UN Commission on Human Rights to
help overcome the “credibility deficit” of the previous organisation.

Headquartered in Geneva, Switzerland.

Composition:

>The UNHRC has 47 members serving at any time with elections held to fill up seats every year, based
on allocations to regions across the world to ensure geographical representation.

>Each elected member serves for a term of three years.

>Countries are disallowed from occupying a seat for more than two consecutive terms.

36. The New Draft Resolution/The Zero Draft:

It incorporates some of the elements of the UNHRC report, including those on strengthening the HRC’s
capacity on preserving evidence, devising strategies for future accountability processes, and supporting
judicial proceedings in member states with jurisdiction.

As per the UNHRC report, the government of Sri Lanka had created parallel military task forces and
commissions that encroach on civilian functions, and reversed important institutional checks and
balances, threatening democratic gains, the independence of the judiciary and other key institutions.
It also talks about encouraging the Sri Lankan government to implement the requirements of the
previous 30/1 resolution (from which it pulled out).

UNHRC was reconstituted from its predecessor organisation, the UN Commission on Human Rights to
help overcome the “credibility deficit” of the previous organisation.

Headquartered in Geneva, Switzerland.

Composition:

>The UNHRC has 47 members serving at any time with elections held to fill up seats every year, based
on allocations to regions across the world to ensure geographical representation.

>Each elected member serves for a term of three years.

>Countries are disallowed from occupying a seat for more than two consecutive terms.

37. The New Draft Resolution/The Zero Draft:

It incorporates some of the elements of the UNHRC report, including those on strengthening the HRC’s
capacity on preserving evidence, devising strategies for future accountability processes, and supporting
judicial proceedings in member states with jurisdiction.

As per the UNHRC report, the government of Sri Lanka had created parallel military task forces and
commissions that encroach on civilian functions, and reversed important institutional checks and
balances, threatening democratic gains, the independence of the judiciary and other key institutions.

It also talks about encouraging the Sri Lankan government to implement the requirements of the
previous 30/1 resolution (from which it pulled out).

UNHRC was reconstituted from its predecessor organisation, the UN Commission on Human Rights to
help overcome the “credibility deficit” of the previous organisation.

Headquartered in Geneva, Switzerland.

Composition:

>The UNHRC has 47 members serving at any time with elections held to fill up seats every year, based
on allocations to regions across the world to ensure geographical representation.

>Each elected member serves for a term of three years.

>Countries are disallowed from occupying a seat for more than two consecutive terms.

38. NBFID may raise money in the form of loans or otherwise both in Indian rupees and foreign
currencies, or secure money by the issue and sale of various financial instruments including bonds and
debentures. NBFID may borrow money from: (i) central government, (ii) Reserve Bank of India (RBI), (iii)
scheduled commercial banks, (iii) mutual funds, and (iv) multilateral institutions such as World Bank and
Asian Development Bank.

NBFID will be governed by a Board of Directors. The Chairperson will be appointed by the central
government in consultation with RBI.

Development Financial Institution

> DFIs are set up for providing long-term finance for such segments of the economy where the risks
involved are beyond the acceptable limits of commercial banks and other ordinary financial institutions.

> Unlike banks, DFIs do not accept deposits from people.

> They source funds from the market, government, as well as multilateral institutions, and are often
supported through government guarantees.

39. NBFID may raise money in the form of loans or otherwise both in Indian rupees and foreign
currencies, or secure money by the issue and sale of various financial instruments including bonds and
debentures. NBFID may borrow money from: (i) central government, (ii) Reserve Bank of India (RBI), (iii)
scheduled commercial banks, (iii) mutual funds, and (iv) multilateral institutions such as World Bank and
Asian Development Bank.

NBFID will be governed by a Board of Directors. The Chairperson will be appointed by the central
government in consultation with RBI.

Development Financial Institution

> DFIs are set up for providing long-term finance for such segments of the economy where the risks
involved are beyond the acceptable limits of commercial banks and other ordinary financial institutions.

> Unlike banks, DFIs do not accept deposits from people.

> They source funds from the market, government, as well as multilateral institutions, and are often
supported through government guarantees.

40. NBFID may raise money in the form of loans or otherwise both in Indian rupees and foreign
currencies, or secure money by the issue and sale of various financial instruments including bonds and
debentures. NBFID may borrow money from: (i) central government, (ii) Reserve Bank of India (RBI), (iii)
scheduled commercial banks, (iii) mutual funds, and (iv) multilateral institutions such as World Bank and
Asian Development Bank.

NBFID will be governed by a Board of Directors. The Chairperson will be appointed by the central
government in consultation with RBI.

Development Financial Institution


> DFIs are set up for providing long-term finance for such segments of the economy where the risks
involved are beyond the acceptable limits of commercial banks and other ordinary financial institutions.

> Unlike banks, DFIs do not accept deposits from people.

> They source funds from the market, government, as well as multilateral institutions, and are often
supported through government guarantees.

41. NBFID may raise money in the form of loans or otherwise both in Indian rupees and foreign
currencies, or secure money by the issue and sale of various financial instruments including bonds and
debentures. NBFID may borrow money from: (i) central government, (ii) Reserve Bank of India (RBI), (iii)
scheduled commercial banks, (iii) mutual funds, and (iv) multilateral institutions such as World Bank and
Asian Development Bank.

NBFID will be governed by a Board of Directors. The Chairperson will be appointed by the central
government in consultation with RBI.

Development Financial Institution

> DFIs are set up for providing long-term finance for such segments of the economy where the risks
involved are beyond the acceptable limits of commercial banks and other ordinary financial institutions.

> Unlike banks, DFIs do not accept deposits from people.

> They source funds from the market, government, as well as multilateral institutions, and are often
supported through government guarantees.

42. NBFID may raise money in the form of loans or otherwise both in Indian rupees and foreign
currencies, or secure money by the issue and sale of various financial instruments including bonds and
debentures. NBFID may borrow money from: (i) central government, (ii) Reserve Bank of India (RBI), (iii)
scheduled commercial banks, (iii) mutual funds, and (iv) multilateral institutions such as World Bank and
Asian Development Bank.

NBFID will be governed by a Board of Directors. The Chairperson will be appointed by the central
government in consultation with RBI.

Development Financial Institution

> DFIs are set up for providing long-term finance for such segments of the economy where the risks
involved are beyond the acceptable limits of commercial banks and other ordinary financial institutions.

> Unlike banks, DFIs do not accept deposits from people.

> They source funds from the market, government, as well as multilateral institutions, and are often
supported through government guarantees.
43. NASA and ISRO are collaborating on developing an SUV-sized satellite called NISAR, which will detect
movements of the planet’s surface as small as 0.4 inches over areas about half the size of a tennis court.

The Name ‘NISAR’: The name NISAR is short for NASA-ISRO-SAR

Role of NASA:

National Aeronautics and Space Administration (NASA - space agency of the USA) will provide one of the
radars for the satellite, a high-rate communication subsystem for science data, GPS receivers and a
payload data subsystem.

NISAR will be equipped with the largest reflector antenna ever launched by NASA.

Role of ISRO:

Indian Space and Research Organisation (ISRO) will provide the spacecraft bus, the second type of radar
(called the S-band radar), the launch vehicle and associated launch services.

Primary Goals:

Tracking subtle changes in the Earth’s surface,

Spotting warning signs of imminent volcanic eruptions,

Helping to monitor groundwater supplies, and

Tracking the rate at which ice sheets are melting.

44. NASA and ISRO are collaborating on developing an SUV-sized satellite called NISAR, which will detect
movements of the planet’s surface as small as 0.4 inches over areas about half the size of a tennis court.

The Name ‘NISAR’: The name NISAR is short for NASA-ISRO-SAR

Role of NASA:

National Aeronautics and Space Administration (NASA - space agency of the USA) will provide one of the
radars for the satellite, a high-rate communication subsystem for science data, GPS receivers and a
payload data subsystem.

NISAR will be equipped with the largest reflector antenna ever launched by NASA.

Role of ISRO:

Indian Space and Research Organisation (ISRO) will provide the spacecraft bus, the second type of radar
(called the S-band radar), the launch vehicle and associated launch services.

Primary Goals:

Tracking subtle changes in the Earth’s surface,


Spotting warning signs of imminent volcanic eruptions,

Helping to monitor groundwater supplies, and

Tracking the rate at which ice sheets are melting.

45. NASA and ISRO are collaborating on developing an SUV-sized satellite called NISAR, which will detect
movements of the planet’s surface as small as 0.4 inches over areas about half the size of a tennis court.

The Name ‘NISAR’: The name NISAR is short for NASA-ISRO-SAR

Role of NASA:

National Aeronautics and Space Administration (NASA - space agency of the USA) will provide one of the
radars for the satellite, a high-rate communication subsystem for science data, GPS receivers and a
payload data subsystem.

NISAR will be equipped with the largest reflector antenna ever launched by NASA.

Role of ISRO:

Indian Space and Research Organisation (ISRO) will provide the spacecraft bus, the second type of radar
(called the S-band radar), the launch vehicle and associated launch services.

Primary Goals:

Tracking subtle changes in the Earth’s surface,

Spotting warning signs of imminent volcanic eruptions,

Helping to monitor groundwater supplies, and

Tracking the rate at which ice sheets are melting.

46. NASA and ISRO are collaborating on developing an SUV-sized satellite called NISAR, which will detect
movements of the planet’s surface as small as 0.4 inches over areas about half the size of a tennis court.

The Name ‘NISAR’: The name NISAR is short for NASA-ISRO-SAR

Role of NASA:

National Aeronautics and Space Administration (NASA - space agency of the USA) will provide one of the
radars for the satellite, a high-rate communication subsystem for science data, GPS receivers and a
payload data subsystem.

NISAR will be equipped with the largest reflector antenna ever launched by NASA.

Role of ISRO:
Indian Space and Research Organisation (ISRO) will provide the spacecraft bus, the second type of radar
(called the S-band radar), the launch vehicle and associated launch services.

Primary Goals:

Tracking subtle changes in the Earth’s surface,

Spotting warning signs of imminent volcanic eruptions,

Helping to monitor groundwater supplies, and

Tracking the rate at which ice sheets are melting.

47. NASA and ISRO are collaborating on developing an SUV-sized satellite called NISAR, which will detect
movements of the planet’s surface as small as 0.4 inches over areas about half the size of a tennis court.

The Name ‘NISAR’: The name NISAR is short for NASA-ISRO-SAR

Role of NASA:

National Aeronautics and Space Administration (NASA - space agency of the USA) will provide one of the
radars for the satellite, a high-rate communication subsystem for science data, GPS receivers and a
payload data subsystem.

NISAR will be equipped with the largest reflector antenna ever launched by NASA.

Role of ISRO:

Indian Space and Research Organisation (ISRO) will provide the spacecraft bus, the second type of radar
(called the S-band radar), the launch vehicle and associated launch services.

Primary Goals:

Tracking subtle changes in the Earth’s surface,

Spotting warning signs of imminent volcanic eruptions,

Helping to monitor groundwater supplies, and

Tracking the rate at which ice sheets are melting.

48. A disturbed area is declared by notification under Section 3 of AFSPA. It is invoked in places where
the use of armed forces is necessary to aid the civil power.

49. The Governor of the state or the central government can declare the whole or part of the state or
union territory as disturbed area.

50. NSCN-IM is a Naga Insurgent Group.


51. Once declared ‘disturbed’, the region is maintained as disturbed for a period of three months
straight, according to The Disturbed Areas (Special Courts) Act, 1976.

52. The NSCN-IM is demanding for separate flag and constitution. This demand is not acceptable by the
central government. This leads to the continuation of insurgency in the region.

53. In the Budget 2021-22, Jal Jeevan Mission (Urban) has been announced under the Housing and
Urban Affairs Ministry to provide universal coverage of water supply to all households through
functional taps in all statutory towns in accordance with Sustainable Development Goal- 6.

>It complements the Jal Jeevan Mission (Rural) which envisages supply of 55 litres of water per person
per day to every rural household through Functional Household Tap Connections (FHTC) by 2024.

>Recycling used water to meet at least 20% of total city water demand and 40% for industrial water
demand at State level.

Globally, India is ranked 120th among 122 countries in WaterAid''s water quality index.

Outcome based Funding:

Funding from the Government for projects will be in three tranches of 20:40:40.

Third instalment onwards will be released based on outcomes achieved and credible exclusion will be
exercised while funding.

54. In the Budget 2021-22, Jal Jeevan Mission (Urban) has been announced under the Housing and
Urban Affairs Ministry to provide universal coverage of water supply to all households through
functional taps in all statutory towns in accordance with Sustainable Development Goal- 6.

>It complements the Jal Jeevan Mission (Rural) which envisages supply of 55 litres of water per person
per day to every rural household through Functional Household Tap Connections (FHTC) by 2024.

>Recycling used water to meet at least 20% of total city water demand and 40% for industrial water
demand at State level.

Globally, India is ranked 120th among 122 countries in WaterAid''s water quality index.

Outcome based Funding:

Funding from the Government for projects will be in three tranches of 20:40:40.
Third instalment onwards will be released based on outcomes achieved and credible exclusion will be
exercised while funding.

55. In the Budget 2021-22, Jal Jeevan Mission (Urban) has been announced under the Housing and
Urban Affairs Ministry to provide universal coverage of water supply to all households through
functional taps in all statutory towns in accordance with Sustainable Development Goal- 6.

>It complements the Jal Jeevan Mission (Rural) which envisages supply of 55 litres of water per person
per day to every rural household through Functional Household Tap Connections (FHTC) by 2024.

>Recycling used water to meet at least 20% of total city water demand and 40% for industrial water
demand at State level.

Globally, India is ranked 120th among 122 countries in WaterAid''s water quality index.

Outcome based Funding:

Funding from the Government for projects will be in three tranches of 20:40:40.

Third instalment onwards will be released based on outcomes achieved and credible exclusion will be
exercised while funding.

56. In the Budget 2021-22, Jal Jeevan Mission (Urban) has been announced under the Housing and
Urban Affairs Ministry to provide universal coverage of water supply to all households through
functional taps in all statutory towns in accordance with Sustainable Development Goal- 6.

>It complements the Jal Jeevan Mission (Rural) which envisages supply of 55 litres of water per person
per day to every rural household through Functional Household Tap Connections (FHTC) by 2024.

>Recycling used water to meet at least 20% of total city water demand and 40% for industrial water
demand at State level.

Globally, India is ranked 120th among 122 countries in WaterAid''s water quality index.

Outcome based Funding:

Funding from the Government for projects will be in three tranches of 20:40:40.

Third instalment onwards will be released based on outcomes achieved and credible exclusion will be
exercised while funding.
57. In the Budget 2021-22, Jal Jeevan Mission (Urban) has been announced under the Housing and
Urban Affairs Ministry to provide universal coverage of water supply to all households through
functional taps in all statutory towns in accordance with Sustainable Development Goal- 6.

>It complements the Jal Jeevan Mission (Rural) which envisages supply of 55 litres of water per person
per day to every rural household through Functional Household Tap Connections (FHTC) by 2024.

>Recycling used water to meet at least 20% of total city water demand and 40% for industrial water
demand at State level.

Globally, India is ranked 120th among 122 countries in WaterAid''s water quality index.

Outcome based Funding:

Funding from the Government for projects will be in three tranches of 20:40:40.

Third instalment onwards will be released based on outcomes achieved and credible exclusion will be
exercised while funding.

58. Bangladesh and India have a long standing and time-tested Protocol on Transit and Trade through
inland waterways. It would serve as a new trade corridor between the two countries, helping the
Northeast states grow. It will enhance people-to-people contact.

Maitri Setu is a bridge built over the Feni river, which flows between the Indian boundary in Tripura and
Bangladesh. This 1.9 km long bridge connects Sabroom in South Tripura and Ramgarh in Bangladesh. It
was constructed by the National Highway and Infrastructure Development Corporation at the cost of
Rs.133 crore. This bridge makes Tripura the ‘Gateway of North East’ since provides access to
Bangladesh’s Chittagong Port.

Now Agartala (capital of Tripura) will become the nearest city to an international sea port in India. Along
with the Maitri bridge, Modi also laid the foundation stone of an Integrated Check Post (ICP) at
Sabroom. The ICP aims to ensure seamless movement of goods and passengers between India and
Bangladesh.

59. Bangladesh and India have a long standing and time-tested Protocol on Transit and Trade through
inland waterways. It would serve as a new trade corridor between the two countries, helping the
Northeast states grow. It will enhance people-to-people contact.

Maitri Setu is a bridge built over the Feni river, which flows between the Indian boundary in Tripura and
Bangladesh. This 1.9 km long bridge connects Sabroom in South Tripura and Ramgarh in Bangladesh. It
was constructed by the National Highway and Infrastructure Development Corporation at the cost of
Rs.133 crore. This bridge makes Tripura the ‘Gateway of North East’ since provides access to
Bangladesh’s Chittagong Port.

Now Agartala (capital of Tripura) will become the nearest city to an international sea port in India. Along
with the Maitri bridge, Modi also laid the foundation stone of an Integrated Check Post (ICP) at
Sabroom. The ICP aims to ensure seamless movement of goods and passengers between India and
Bangladesh.

60. Bangladesh and India have a long standing and time-tested Protocol on Transit and Trade through
inland waterways. It would serve as a new trade corridor between the two countries, helping the
Northeast states grow. It will enhance people-to-people contact.

Maitri Setu is a bridge built over the Feni river, which flows between the Indian boundary in Tripura and
Bangladesh. This 1.9 km long bridge connects Sabroom in South Tripura and Ramgarh in Bangladesh. It
was constructed by the National Highway and Infrastructure Development Corporation at the cost of
Rs.133 crore. This bridge makes Tripura the ‘Gateway of North East’ since provides access to
Bangladesh’s Chittagong Port.

Now Agartala (capital of Tripura) will become the nearest city to an international sea port in India. Along
with the Maitri bridge, Modi also laid the foundation stone of an Integrated Check Post (ICP) at
Sabroom. The ICP aims to ensure seamless movement of goods and passengers between India and
Bangladesh.

61. Bangladesh and India have a long standing and time-tested Protocol on Transit and Trade through
inland waterways. It would serve as a new trade corridor between the two countries, helping the
Northeast states grow. It will enhance people-to-people contact.
Maitri Setu is a bridge built over the Feni river, which flows between the Indian boundary in Tripura and
Bangladesh. This 1.9 km long bridge connects Sabroom in South Tripura and Ramgarh in Bangladesh. It
was constructed by the National Highway and Infrastructure Development Corporation at the cost of
Rs.133 crore. This bridge makes Tripura the ‘Gateway of North East’ since provides access to
Bangladesh’s Chittagong Port.

Now Agartala (capital of Tripura) will become the nearest city to an international sea port in India. Along
with the Maitri bridge, Modi also laid the foundation stone of an Integrated Check Post (ICP) at
Sabroom. The ICP aims to ensure seamless movement of goods and passengers between India and
Bangladesh.

62. Bangladesh and India have a long standing and time-tested Protocol on Transit and Trade through
inland waterways. It would serve as a new trade corridor between the two countries, helping the
Northeast states grow. It will enhance people-to-people contact.
Maitri Setu is a bridge built over the Feni river, which flows between the Indian boundary in Tripura and
Bangladesh. This 1.9 km long bridge connects Sabroom in South Tripura and Ramgarh in Bangladesh. It
was constructed by the National Highway and Infrastructure Development Corporation at the cost of
Rs.133 crore. This bridge makes Tripura the ‘Gateway of North East’ since provides access to
Bangladesh’s Chittagong Port.

Now Agartala (capital of Tripura) will become the nearest city to an international sea port in India. Along
with the Maitri bridge, Modi also laid the foundation stone of an Integrated Check Post (ICP) at
Sabroom. The ICP aims to ensure seamless movement of goods and passengers between India and
Bangladesh.

63. September 11 is a highly symbolic date as it will be 20 years since Al-Qaeda attacked the U.S. with
hijacked airliners, triggering military intervention in Afghanistan.

64. The International Security Assistance Force was a NATO-led military mission in Afghanistan,
established by the United Nations Security Council in December 2001.

65. President Joe Biden announced that U.S. will begin its final withdrawal from Afghanistan on May 1,
2021.

66. Taliban was ousted from power by a U.S. invasion in late 2001 and has since waged an insurgency.

67. A key reason for a coordinated withdrawal is the fact that NATO relies on U.S. airlift capabilities and
shipping to move valuable equipment in and out of landlocked Afghanistan.

68. Piyush Goyal is the current Trade Minister from Bharatiya Janata Party.

69. NSAC was constituted by DPIIT which stands for Department of Promotion of Industry and Internal
Trade.

70. The Government of India has nominated 28 non-official members on the National Startup Advisory
Council.

71. The National Startup Advisory Council will be chaired by the Minster for Commerce & Industry.
LEGAL REASONING

72. (C) It was not against the will of Y because she was not disapproving the act of boss means she was
willing to do a particular activity. Therefore, answer will be C. Option (b) Facts of question not shown
any kind of sexual favours from X.

73. (B) option (a) is incorrect because it is blaming all women for misusing the law therefore this is
incorrect inference. Option(c) and (d) are out of the context of the passage. Option (b) is the right
inference as per the line because it can be inferred that some people misuse the law for ulterior motives
against men.

74. (C) C is changing cloth openly in a public place where there is the expectation of being observed and
Z has no the motive to see her intentionally will not attract section 354C. Therefore, Option C is the right
answer.

75. D all of the above options can be inferred from the last line of paragraph 1st and Paragraph 2nd of
the passage.

76. (d) It is not mentioned in the passage about what punishment should be granted to women for false
allegations.

77. (B) Option (A) is incorrect because the court guidelines says that “The act perpetrated on the
survivor constitutes an offense in law and is not a minor transgression that can be remedied by way of
an apology, rendering community service, tying a rakhi or presenting a gift to the survivor,” the court
said. Therefore, after inference from this line option (B) is correct answer.

78. option (A) can be directly infer from the passage and Option (B) is incorrect because “The bench also
said courts must desist from expressing any stereotype opinion about women, in words were spoken
during proceedings, or in the course of a judicial order.” And in question no words were spoken during
judicial proceeding .Option (C) and (D) have absurd reasoning.

79. (D) Option (A) is incorrect because consumption of alcohol is one factor for violence against women
but not only the factor. Both Option (B) AND (C) can be inferred from the lines.

80. (B) Option A) is incorrect because NOT is not mentioned therefore it changes the lines. Option C) is
incorrect as instead of judicial proceeding the bail order is written.
81. (B) Option B is only correct option because court order is insuring safety and protection for women
and not giving any stereotypical view about weakness of women. No exact phrase can be inferred from
the court order which proves stereotypical view.

82. She will not win case against Mahadev Bus Services as the context is not talking about the agent-
principle liability. Here only Prabhu and Kapil Mechanic will be liable for negligence as they should have
applied reasonable care and due diligence while taking back the Bus after repairing and also during
carrying out the repairing works respectively. Thus, both are liable.

83. Priya is not liable for negligence as she has taken reasonable and due diligence while constructing
the pool by putting filters to prevent overflowing. The staff was negligent in their work as they have left
pool unattended for filing. Thus, she cannot be held liable for negligent act of the staff. Hence, Jatin fails
in the suit.

Note- We cannot take the principle-agent relationship and principle of vicarious liability as the context
of the passage restricts to negligence.

84. Mr. Sharma can validly claim compensation from Manohar as he was negligent in his acts. He should
have taken reasonable care and due diligence in checking the expiry dates of the cheese cubes before
using them for his dishes.

85. Both Mehul and Jetali were liable for negligence as Mehul should have taken reasonable care and
due diligence while driving the car and for other pedestrians on the road. Jetali being the owner the car
is liable as Mehul was acting under her authority and for her purpose. Thus, applying the principle
mentioned, both are liable.

86. Assertion and Reason are correct and reason is an correct explanation because the standard test of
reasonable man has to be applied while determining the negligent act as reasonable person is one who
is careful enough in his conduct and differentiates between merits and demerits perfectly.

87. Authors stand with regards to the test and principle of reasonable man seems to be an orthodox test
which has not passed the test of time and has to be changed with respect to negligence.

88. NLUJ is not a state or instrumentality of state within the ambit of article 12 of the constitution. It’s a
private body and serving the function of public importance, but not of itself an public function. Thus, it is
not liable for violation of any fundamental right.

89. Life Insurance Corporation Ltd. is instrumentality of state as it discharges public function of giving
life insurance cover and hence is liable for damages to Mr. Bhide. It is not a private body. Also, the
function of providing life insurance cover is one of public function and not importance.
90. BCCI is not an instrumentality of state or a state within the ambit of article 12 as it only performs the
function of public importance of representing Indian at global level in Game of Cricket but is not a public
function per se. Thus, it is not liable for infringing Fundamental Rights of its players.

91. Balaji Electricity Suppliers are not state or instrumentality of state although they perform similar
functions of RSEB. But performing similar functions as of government do not make such bodies’ state or
instrumentalities of state. They are private body performing function of public importance. It is not a
public function per se.

92. Writ of Mandamus is filed against the government, cooperation or public authority to execute duty
that they are legally bound to complete. But it cannot be filed against the Chief Justice of SC or other
distinct HCs, President or Governor of the state, MPs, MLAs etc.

93. The forecast by the Priest was undue influence falling under section 171C amounting to divine
displeasure and spiritual censure. Thus, it held be a violative practice.

94. Though there was divine displeasure and spiritual censure by way of fraud from Prakash, but it does
not amount to undue influence as reason for rejection of his application for candidature.

95. Meenakshi withdrawn her candidature only due to the reason of undue influence of her husband
Ratan and it does not fall within the category of section 171C i.e. divine displeasure or spiritual censure.

96. Anant has taken back / withdrawn his candidature due to threat and compulsion of Munna but it
does not become ground for undue influence and even not under section 171C.

97. Right to vote and candidature right is part of civil right and important feature to safeguard Indian
democracy. Also, civil rights are being performed by positive government action in form of legislation
rather than civil liberties, which are freedoms, secured by restraining government actions.

98. The correct answer is option (a). It is a violation of Article 23 because the children are being
economically exploited and being subjected to forced labour. Just because they are being given two
meals a day and a roof to sleep under does not discount the fact that X is forcing them to clean houses
on contractual basis for money, which is not even being paid to the children. X is exploiting the poor
condition of the children.

99. The correct answer is option (b). Being forced into prostitution as well as cleaning of sewers by
children is in contravention of Article 39. Y and his mother were forced into these activities due to
economic necessities. However, it is not human trafficking as there is no buying and selling yet, it can be
termed as forced labour as Y’s family was ‘forced’ to do this for economic reasons.

100. The correct answer is option (a). This is a clear case of human trafficking as the girls were bought
and sold by the drug dealer like inanimate objects. It is not beggar or forced labour from the given facts
as the situation after being sold has not been given. However, it is a case of trafficking as the girls were
being used to further an immoral business and traded like a commodity.

101. The correct answer is option (c). Article 29 provides for equal pay for equal work. Here, Rama and
Ramesh are not doing the same work: Rama is working as an assistant to the regional manager while
Ramesh is working as assistant regional manager. There is nothing arbitrary in their salaries owing to
their different positions. Hence, Rama cannot claim to be paid the same salary as Ramesh.

102. (b) Only Arya is guilty as she had knowledge (mens rea) and she induced Bhakti to fire (actus rea).

103. (c) Punnu''s crime was not the refusal to move the car but that of having driven on to the foot of
the officer and decided not to cease the act, he had established a continual act of battery (a criminal
act). The act was a continual act of battery, as he did not remove the car off his foot wantonly even after
being repeatedly asked by the Police officer to do so. Moreover, there was not any lawful justification
for Punnu’s act and according to the information above; the offence of battery can be constituted if the
act of applying force is done intentionally without lawful justification.

104. (b) As per the information given in the passage, “As in all crimes, mens rea has to be established”.
In the instant case, Pooja had picked up the gun with a bona fide intention of shooting the lion and
saving Amit. However, if it hit Amit, Pooja cannot be said to be liable.

105. Since Osama brainwashed Ballu Badshah to help out with the attacks, he did abet the attacks by
instigation.

106. Since Ballu Badshah intentionally engaged with Osama and Chota Chetan to carry out the attacks,
he is guilty of abetment.

107. Chota Chetan cannot be held liable since it was not his intention to aid and abet the terrorist
attacks in the country.

LOGICAL REASONING

108. Option (B) is incorrect as there could be multiple reasons to prevent the spread of COVID-19.
Options (A) and (D) cannot be deemed to be absolutely true from the passage. Therefore, the correct
answer is Option (C) which can be inferred from the last paragraph.
109. Option (C) cannot be completely accepted as a true proposition. Option (A) is immaterial to the
discussion presented by the author as situation is still not great and can easily become worse with time.

Option B is correct as can be inferred from the second half of first para and first sentence of second
par(a)

Therefore, the correct option is (B).

110. Option (A) is incorrect as the paragraph does not signify such reasoning or it’s opposite to be true.
Option (B) can be sourced from the information given in the second paragraph.

111. Options (A) & (D) support the arguments posed by the author. Option (C) is an observation that
does not affect what the author has stated or the reasoning behind it. However, option (B) undermines
the author’s argument that elections have made the matter worse by diverting attention of the public as
well as the government. If these states do not have any COVID related cases then the authorities and
the people are not behaving irresponsibly by not taking about this issue.

112. Premises are the facts or evidence that support or lead to the conclusion. Therefore, the above
statement forms premise of the passage which leads to the conclusion. Hence, option (A) is the correct
answer.

113. Options (A) & (B) are incorrect as per the passage. Options (D) is unfounded in the information
provided above and cannot be held to be true with certainty. Repeated offenders are ones who commit
the same offence twice. But the passage does not give information relating to the offences committed
by the prisoners that are being released now. Therefore, such an inference cannot be derived and the
correct answer is Option (C).

114. Only Option (A) follows from the second half of first paragraph of the passage given above. Option
(B) may be a solution to the problem but its effectiveness or morality has not been discussed by the
author. Therefore, the correct Option is (A).

115. Option (A) does not affect the topic of discussion introduced by the author.

Options C and D talk of ‘foreign’ and not ‘native’ jurisdiction.

Option (B), however, debunks the author’s theory about the effect of lockdowns and COVID-19 on the
economy that is causing such hardship to these prisoners.

116. None of the options mentioned above can be sourced, inferred or concluded from the information
mentioned above. Therefore, the correct Option is (D).
117. Option (D) forms the conclusion that reformation does not stop at putting criminals in prisons as
such persons, who are being released, need some support systems to make sure they don’t fall back into
the same cycle that brought them to this juncture in the first place.

118. Choice B can be eliminated as the main contention made in the text is not to criticize colonial
writers but talk about how culture is not fixed and how it is open to interpretations. The criticism is only
done in the beginning of the passage to elucidate the context in which books by insiders such as Achebe
were written to bust the myths with regard to his culture (African Community).

Choice C dooes not include the part about forms of writing and giving autonomy to the text and hence
can''t be the answer.

Choice D is a tad extreme as although the passage criticizes the colonial writers for their representation
of the African community, it doesn''t go so far as saying that they do not have a "right" to write about
other cultures.

Hence, A

119. Assumption 1 is correct because if there were only one form of writing, the author wouldn''t have
used the phrase "new forms of writing".

Assumption 2 is correct because if the author were not in a position of authority, the displacement of
this authority would not be talked about in th last line of the passage.

120. Choice C can be understood in the beginning of the passage where the author argues that Achebe
meant to challenge the writings of the colonial writers such as Cary and Conrad who see African
communities as "primitive" and "without history"

121. Choice D is correct as these four authors agree with each other in the passage while discussing
about culture and modes of representation.

122. The passage says that Clifford, who talks about culture not being a unified corpus of symbols and
meanings that can be definitively interpreted, borrows it from Bakhtin. So, it can be inferrred that
culture is not definitive in nature and is in a state of flux depending on interpretion.

123. In the passage new traffic violation rules introduced, which led to the increase in the violation and
then subsequently there is an increase in number of fines collecte(d) None of the above statement
resembles to the pattern of reasoning of the above passage. Therefore, the correct answer is option (D).

124. Option (B) and (C) is not the correct answer because they both are the incomplete conclusion of
the above passage. Option (D) is not relevant to the above passage. Option (A) is the correct answer
because it is completely concluding the remarks which are made by the author.
125. option (A) is not correct because there may also be other sources of income. Option (B) is not
correct because this statement is extra fact. Option (C) may be correct may not be correct. Option (D) is
the correct answer because that statement is certain.

126. Option (B) and (C) are further weakening the argument in favour of automated traffic enforcement.
Option (A) is the correct answer because it is providing the benefits of having automated traffic
enforcement. Hence, option (A) is the correct answer.

127. Argument Evaluation: Situation: The roof of a recently constructed building collapsed under heavy
snowfall. The only way the building did not meet safety standards was that some nails for the roof
supports were smaller than prescribed by the building codes.

Reasoning: What would make it less likely that the building''s collapse resulted from a single, apparently
minor departure from safety standards? The building met safety standards except for the size of the
nails. So, if the collapse exemplifies how a departure from safety standards can have severe
consequences, as the conclusion claims, then the size of the nails had to be responsible for the collapse.
Thus, evidence that a factor other than the size of the nails could fully account for the collapse would
weaken the argument.

a) - This suggests that the snow would not have been heavy enough to collapse the roof if the
construction had completely met the safety standards, so it strengthens, rather than weakens, the
argument.

b) - Correct. This suggests that the snow could have collapsed the roof even if the nails had met the
safety standards, thus casting doubt on the assumption that the nails'' inadequacy was responsible for
the collapse.

c) - The claim that the safety requirements for this building were weaker than some others tends slightly
to strengthen, rather than weaken, the hypothesis that the bad consequences resulted partly from a
failure to comply. Even if safety-code provisions for an equipment-storage building differ from those for
an office building, they may still be adequate to ensure the roof''s stability.

d) - The question of who was responsible for ensuring compliance with the safety codes is irrelevant to
whether a failure to comply was responsible for the roof''s collapse.

128. The statement mentions that the southern grid collapsed ''yet again''. This means that it had
collapsed earlier also. So, 1 is implicit. Also, the statement talks of a particular fault in the system but
does not condemn the grid system. So, 2 is not implicit.

Hence, A
129. Method of Reasoning ; LOD - Difficult

a) Frida makes no reference to the price.

b) Correct. Picasso says a copy could have all the qualities as the original. Frida says a copy would have a
different history and thus would not have all the same qualities. She does undermine the claims of all
qualities being the same.

c) Frida has not questioned the copy being visually indistinguishable.

d) Though Frida says that Picasso’s understanding of the history of art is inadequate, it does not refute
his argument since this statement does not address the argument but the person making it.

Thus, choice B is the correct answer.

130. Explanation: Since the Government has allowed private airline companies in India to operate to
overseas, so the national air carrier has increased its flights to overseas destinations.

131. LOD: V. Difficult

In the above statement, there is no direct co-relation depicted between magistrate Kavindra and
reasonable and fair NBA panel, except that Kavindra is one of the seven members of this panel. Panel
were reasonable and fair in its decision, we don''t know whether any one member contributed more
than any other.

In option D, Kareena has sold real estate for Budhwara Realty. We don''t know whether she is the only
one or there are others as well who sell property for Budhwara realty. So a depreciation in number of
houses sold by Budhwara realty cannot be directly linked to Kareena''s performance last year.

132. Prabhas may be more intelligent and hence topped the exam. Hard work alone is not sufficient to
become a topper. Therefore, options A, B and C are eliminated and D is the correct answer.

133.
134.

135.

QUANTITATIVE TECHNIQUES

136.

137.

138.
139.

140.

141.
142.

143.
144.

145.

146.

147.

148.
149.

150.

You might also like